Sunteți pe pagina 1din 686

[

y
y +
x
y
z

yz z + b = 0

z =
xz
E

y
+
=
+
+
b
0
3
z
,33
y +
= 1
( +
x
z
y
3
z
r
r r
x
z =
10 5

+ b = 0
t (n ) =n

3
)
z
,
E
3
=
( +
323x1 a r4ax 2
4ax 2 z
2,31
x
n ) = 0 2 x1 a
10 5
(

x
1

t
=

n
xy
0
x +
8
xz y =
4ax 2 2ax1 x +
z )=
+
=
+
b
0

10 5

4
ax
2
ax
2
2
1x
y =

G
x
y
z
x
,
y =
3

2 2

1
4,
+
8 (2 x1a )(2ax1 ) 16
2 2
x(22ax a=0 )(2ax )2,16
10 5
xy
x xy
1 xyzy = 1 1 2 x1 a5x120a 5=40ax 2 348
y =
y
10 6
=0
2
2
+
+
+G2by = 0 2

4,13
(4ax 2+) +

=
(
)
2
x
a

1
x
y

x =
x
y (2 x1xaz ) =xy(4=ax02 )x41 ax 2 2ax1
48

dx1 t
0 5
x
1

=
y
2
2
2
x
a
4
ax

E x 10 6

1
2
(2x11 a==2 xd1)(
xy y
a 41ax2 ) 16 x 2 a = 0

1yz +
2 x1 a = 4ax
xz
z
=xb4ax

2 + 2
t 2ax
+
+
X1 x1

+
=

G
x
a
0

=
+
t
1
2
z
=
y
y = 1
y2 2 x1za 4ax 2 0 ( =02x )a2 +
x1
x =l2n1
x
x
y
y + 4ax2 x
2
x
a

4
ax

=
1
(
)
2
1
2
x
2
x
a
4
ax

2
1
2
= t
=
z =
0
E
dt
E

dx2
Xx
t
y x(4ax 2 2ax1 3,3
yz

xz
33
0
1 1=2 x1 a 4ax 2 x1 =

ln 1

=
d
+
+
z = 1
t
r
1
x =+

=
y
X
x22 x1 a = 4ax 2
10 52 2 t (n ) = n X
y +
1xexp t y

t
2

=
(
)(
)

16 x a
0

1 2 x a z ) =2ax1
E z
0 2t + 3
E lyn2=x22 x1 a + 4axz2 = 3
2 2xx1 =a X 2 4ax
dt
1
x
=

,33
3
( = ln
= 1
X3
2
1182)2 1 exp t
3
+2x1 a )2 = (,43ax
x (
=0
z
X2
2t=+ 13

r
x + 2t +
10 5

1
x
5

0
l
4
ax
2
ax
n
t(

y
2
1
l
n
)
G xy =
z
E

==ln4
=
z
3
2 x1 a r4axr2
2
2,5 X

,3 +2 ( 1= )(
r r

+ 1b =)0 16 x 22 a 2 = 0 +xb
,31
1
8 3 2xln1a = 22xax
x
1
10 25x21a = 24t4ax
=
0
+
=
y
x
8
+

a
4
ax
1

1
2
0 6 23 2
10 5

2
y
G xy = x1
=
G xy =
(2 x1 a ) = (4ax 2x )
0 dx
4,3
2,5
t

2 = X
2
1
48
r
x
1
r
2
=
y

1
t
+
=
5
0+ b = 0
1 = 23x1 a 14ax 2
10 6
x

dt

X1 x1

2 x a = 4ax
G xy = x1
y =
2 x1 a 4a
0
ln x1 1 x + 2 xy +2 xz= 2+x1 ab+=4ax
0 x2

0xz dx W.
xy
V.
Chaves
t

= t
x x 0x= 2+1 Eduardo
+ 1b = 0
+
y
4ax 2 2ax1
z
y
dx2
X1
t
x

2 x1 a
zX x =x dt
x
xy

x
=
dt
=
G
1

1
x (
xy r 1 y X1exyzp t
y =
X 2 x2
0
2 )(2a2
l

0
r
n
x1 a4ax
2

+ by = xy
x + +b = 0 +
0 + y + xyz2 +
bx =0 xy 2x1xza =4tax
0 2t +13
2
l

d
2
=

X
yt+
y
x z
x3 = X x
+ b1 =20x a
+

x =x1
y
x z x2=
1
(2x1a=)02 =
x
z4ax 2 x2(ax
3
dty
x

X 2 = ln 2

E x
1
t
x
+3
X2
E

ln 3z + b =2x0xz1 a yz 4ax

z2 2 xy0 2t +3y
xz +x yz
+
(2x1 a=)
z
y = 1

bx)(
4
+= 0b z += 0 + (yz2 x+1 aln
+
+
y + y = 1
22
a1
x3 = X
y y +z
=2x01ax
x
r1 rxx

y
=

z =
= 2X y 2ax
y
x
E y
X ln
2 1 z 3x
3E
, x yxy xz xz+==b = 0 42 ax
(z2 x1 a 2)2= 2(4xaxa 22)t 2+
(
2

3
2 x=1 a 1 4ax
t
+
+
=
+
+
b
3
0
3

1
2
,
1
E
x
33x 3(2 x a 3)(2ax ) 16 x2 ayz2 = 0
3 y ( zr n
z
x =+0 z = 1x
1
( )

xz

z
1
1
2
5 x +t

r
0

n
n
+
+
+
=
b

1
( )
4Eax 2 2ax1
0

1
0 5
z )=
=
z

2xyz y yzyz =) = (2 x a y + )2 t= (4ax


)x2n y z2 x1 a z = 4
ax
E

2
4a
2 x1 a

1
2
z =
Eby2=,y30
+
+

xy
= (12x x=1 a 1 )(2axx 1 + )xy 16= x 221a 2,3=108 +x +
3

,33 = 2 x a 4ax 4ax


x=z x1 xy 18 (xz
x 10y5
ax
2

3
y
2
1
z
1 r
2
1
+
+ 10x ++5 b = 0
)2 = (4ax )2 = G x
G(2 xx1yaE= 2

= n
4,3 y = 2 yz y E=x 4 y 2xz1 a z =)x=4ax 2 10=52 x a t+(n4)ax
x5
2

,
xz
4
z

2
xy = 1
1
2 (2 x1 a )(2a
2
10 5y xy+ =1 = 8 1 +,5 1 + + b,z3z4=80
y = 1
,31
2x1ax 04ax62y 0 xy5 =z 1 xy

y 10
yz

+
6
x
8
G x2yEx1=a x1 = 4ax 2 Gz =
1 5
+
+ by = 0
(2 x1 a )2 =
+
0 y dx t x =12 =xy32,=x10a +x14ax 2
0

x
y
x
=
3
z
G x
xy = 1
1(
4,3
z = 1
=xxy+d= 1E 33 1dx1 = t r (n ) y = 2,5

5
x
x

t
x
0
1
X

y =dtt =xz n

2 x1 a = 4
G xyE = 1 1 Gz )

1
yz

x
z
5
0 + b = 0 8 10 6
4ax

x111 a21
+
+
xy = ln 2Xx

0y = 1 =
0 z x2
2

x
x
a
4
ax
+
y

0
l
0
G
n
=
1
2

1
2

x
x
0

= 0 1=y 0 =z
,31 Xx2 = t
yx

dx
xy = 1
t
+
x
8
z
4
ax
2
ax

dx t t
= 1
2
E y 1d1x02xy524=ax
(+ = t12dt3,323ax3x11=XX1xeXx1p1 =t2r 2dt= x1 = X + br = 0r
G xy =X 2 x2 =z = y d1t =
4,
X 2x x2(2x1 aG )(
1
2,5

x 1 ex p t
2 a(n
=)(12ax10)1516
3
(2 x a xy2ax
) x16
0 2t +
x )2 a02 =
0
xy = 1
x3 = X 10 5 E 3z 48 x1ln x2 z)0 =21t+ 23 0 1x2 2 ln x0t2 2 = n ln 1 =

r0=XX
3
rx1 a=ln2), = (24ax 2 )dx 2 t
(620
X 1 t
x = 1

G xy = x1 x3y = 1
3t1+)8 = (4ax22 ) X = ln

b
=
23 (2 x1 a 2

+
x
3

0 dx
x
2
=
d

t
2

+
=
t

t
1
1X 20 lxn5231 = 2 x1 a 4ax 2 3 ln
X1 E

e
tx

2= 1
1
x
y
p
=+2 x1 a 4ax 2 3
1
=0
x =
= G
=
0 21t
x1=a2
x xy y = xz1 x =
dt y
3
x
2,5 2
x41,a34 4=axx24ax
X1 x1
x

l
n
G xy =

=
+
2
2+ =
x+ = X + 2bEx = 0x y +
X 22 2 = +
24 ax
8 3 = X 3 2 22 x=1 a2 x1 4aax
ln1 0x15
0 x2 xy = 01

10 6
Xt 2+1x ln x 2=ty +1 2 E
z 2 y t+

=
3

3 ln
y = 31 1(
dx2 Gt xy = x1 X1 xy t

xy E
0 dxx + t + xz + bx == 0

x1 a 4ax 2
zy = x1

+
x
yz
=
d

t
1
x
1x
1
X1 exp t
+
+ +E b y y
=0 x

r
r
X 2 yx2=

=
x
y
z
(2
x = 1
x
x1 a + 4ax 2
y +
dt
+ b = 0 x y = 1y Ez x=z z 1
0 2t +
X
x

x3 = X G xy = 30 1 1xy ln0x2y

z
l
n

Exy = y1E x + = 3x
E x

x2
= ln yz + b 1 = =0

+
+
X

dx
t

z xz= +1 yzG+ x(z+ zb


y ,
x = 1
1
2x t y2 x z 2t +X31y
z =0
y =

+
x
+
y
l
=
y
n

x y xy = z1 2,
=
dt
3 x1 r= Xr e tz
5 z ) = x +
E x
X 2 x2
Exy = z 1
E
y+b = 01 xp
=

yz

2
3

(x2 = X 2 ,33
z

xz 0+ t + 3 +
xz 20= 1
+ lnbz =
x Gxx =+xy1 = 2 0 5 2,3
3
y = 1
x
1

G
2

=
y + x3 = X
y
x
,5
r
+
x
y

1
y
z
3 t + 1 10 xy5 = 1t=(n0) E
3x
t 10 5

=dy
n
X 2 E= ln 2t +
z =
x
)
E y
z
=
x
x
G
1
=
y = xy1 =
x
z 3
3,33
(
ln 3 2,3
r= 1r
2,G
x1= 4d,3

y
=

5
1
x
t4yt8+
3
z b = 0
1 5
xy = 118 G
xy = 1
r
x +
xy =y =10 X15 01
x +

n
x
(
)

01

t = n
xy
1x0y =5 21 0 E x2
z )=
= d
E z
=bx20x==y X
y
G xy = x + x + +xyy +=xz+xzb+xG
x1
=
2
x
t

2
X
0
d
t

G
(
x
1
t

1
0
+
4
x

2
x
y
z

=
y
,
,
2
d
1
3
,
t
1
5x
z3 = x1
y
18
0 x2 = xdt0+
xyz=341
= 1
x +
xy = 1
8

0 5
=X x
10 5

1 6

y =
x22t +t z
G xy = xyx1+xy y + y yz+yzGb yx=y 00= X1 x1E 2z dt2 d0
4
G
xy xy = xz2,

0
0
= l3nd
+
+
+
=
b
0
,

0
x
1

348
5+ b x = 0
x =
y
x dx1 yt x z
xy x2= 1
3 = X
1 +
1
X 2 x2x +

x
=
x
1
=
y
1

x
y
x
3
0 5
z
dx2 t
X
10 6 E
z
y
0 y2t +
X x = dt
x G
G =xy = xdt =
G xy = x1
E
1
yz

1
x

x
xz
z

=
y
1

23,5 X 3
y 0 yz dx t
0 x2 +xz +0 +yz + + z bl+nz=bx10=xy 0=X 2 1x2
y =
y +
z = t
5
2

y = 1
t
+ 3 10
+
+1 =b y = 0
x
y
z
0
+
y


z X 1
dx2x t y
ln
x
z
x3G=
x y
E
dt
=
Xx1 z x1

y
z
X 3y = x1 =x1
E y(

3,3=33 =
x
y =
d
t
0

0
X

0
l
n z 1= 1
dx1 expt t
X x32,3
(+
33

x
1

)]

ij =

X 0

0
2X
0
X 0

2 3
l
2X

00
l2 3
X 2

X 2 X
0

2
l2

2
2
0
l

2
l

ij =
3
X

2X
l 2
X
0

l 2 3
X
2X

2X

2 3
0

2 3

0
0

ij =
X 2

3
X 2
l2
0
3

X 0

X l
2X
0
2X

2 3
l

l 2 3
0

X 2

X
2
l2
3

l 2
X

2X

0
ij =
l 2 3

X 0

2X

0
2 3
l

X 2

X
2
2
l
3

2
X l
2X
l 2 3

)]

[ [

[ [

( (

)] )]

)]

=
0
ij

() ( )
)]

)]

) ( )[
[(

[(

[
(
)
[ ( )[
[ (
[
[ ( )]
( )]
[
]
(
[)]

(
)]

=
0

ij

0
ij =

0 0
0
ij =
X
2X
0 0
Xl 2 0 3

X 22 X3

0
l
32

X
2
l 2
X 2 20 3
3
X l X 2
l2
3
2lX32
2
X
l
2X

l 2 3

ij

X 0

2X

3
2
0
l ij =
0

X
2

0
3 0

X
2
X 0
ij 2=
X3
l
2X
00 0l 2 2 X

3
X
2
l
X 2 l 2 3
2 XX
0
0
X 2

l 22 X3332
3

X
l

2
X
2
2

2
3
l

0
3

X 2

2

l 2
3
X l
X
2
l2
2X
23 X

2
l 2 3
X ll 2 3
2X

l 2 3

2
2

=
0

X 2
3

2
X l
2X
l 2 3

l
X 2
3
l2

] ) ( )

)]

)]

=
0

) ( )

([ )

(( )
)] [
)] [
(
] [
]
[
(
)]
(
)]
)] [
]
(
(
)]
)]
[
]
(
[
)]
)

[
]
(
( )
( )
( )
)]
( )

X 0
2X
2 3
l

X 2

X
2
l2
3

2
X l
2X
l 2 3

CONTINUUM
MECHANICS
(
[
)]

ij

ij

BY MEANS OF

=
0

X 0
2X
2 3
l
0
X 2

X
2
l2
3

2
X l
2X
l 2 3

X l
2X

)]
SOLVING PROBLEMS

X
2

)]

)]

2
l
X 2
3
l2

)]

II

MECNICA DEL MEDIO CONTINUO

Nomenclature

III

Solving Problems by means of


Continuum Mechanics
EDUARDO WALTER VIEIRA CHAVES

IV

MECNICA DEL MEDIO CONTINUO

Presentacin

s
anic
mec
h

dif
fu
sio

Flu

x
Flu

ids

n-

Soil

Co
nv
ec
tio

er
transf
t
a
e
H

Presentation

s
lic
u
a
dr
y
H

Structure

s
am
e
B

Plate

ds
i
l
So
IBVP and Numerical Treatment

Rigid Body Motion

Constitutive equations

Fundamental equations of C.M.


Stress
Continuum kinematics

Tensors

VI

MECNICA DEL MEDIO CONTINUO

Abbreviations

VII

Contents

Contents

VIII

MECNICA DEL MEDIO CONTINUO

Abbreviations

Abbreviations
IBVP
BVP
FEM
BEM
FDM

Initial Boundary Value Problem


Boundary Value Problem
Finite Element Method
Boundary Element Method
Finite Difference Method

Latin
i.e.
et al.
e.g.
etc.
Q.E.D.
v., vs.
viz.

id est
et alii
exempli gratia
et cetera
Quod Erat Demonstrandum
versus
vidilicet

that is
and the others
for example
and so on
which had to be demonstrated
versus
namely

Operators and Symbols

Operators and Symbols


=

+
2

Tr ()
() T
() 1
() T
() sym
() skew
() sph
() dev

[[]]

det()

D
&
Dt
cof ()
Adj()
Tr ()
:
2

grad()
div ()

I , II , III
r

1
I
I sym I

Macaulay bracket
Euclidian norm of
trace of ()
transpose of ()
inverse of ()
inverse of the transpose of ()
symmetric part of ()
antisymmetric (skew-symmetric) part of ()
spherical part of ()
deviatoric part of ()
module of
jump of
scalar product
determinant of ()
material time derivative of ()
cofactor of ;
adjugate of ()
trace of ()
double scalar product (or double contraction or double dot product)
Scalar differential operator
tensorial product
gradient of
divergence of
vector product (or cross product)
first, second and third principal invariants of the tensor
vector
unity vector
Second-order unit tensor
fourth-order unit tensor
symmetric fourth-order unit tensor

SI-Units

SI-Units
length
mass
time
temperature

velocity
acceleration
energy
force
pressure, stress
frequency
thermal
conductivity
mass density

Prefix
pico
nano
micro
mili
centi
deci

m - meter
kg - kilogram
s - second
K - kelvin

m
s
m
s2
J = Nm - Joules
N - Newton
N
Pa 2 - Pascal
m
1
Hz Hertz
s
W
mK
kg
m3

Symbol

10 n

10 12

10 6

m
c
d

10 9
10 3
10 2
10

electric current
amount of substance
luminous intensity

A - ampere
mol - mole
cd - candela

energy, work, heat

J = Nm - Joules

power

J
W watt
s

permeability

m2

dynamic viscosity

Pa s
kg
m2s
J
m2s
J
m3

mass flux
energy flux
energy density

Prefix
kilo
Mega
Giga
Tera

Symbol

10 n

10 3

M
G

10 6

1012

10 9

XII

SOLVING PROBLEMS BY MEANS OF CONTINUUM MECHANICS

Physical Constants
Newtonian constant of gravitation: G = 6.67384 10 11

m3

kg s 2
m
m
Speed of light in vacuum: c = 299 792 458 300 000 000
s
s

Nomenclature

Nomenclature
r r
r r
A( X , t ) a ( X , t )

r r
a ( x, t )

B0
B

B
r r
b( x , t )

Acceleration (reference configuration)


Transformation matrix
Acceleration (current configuration)
t=0

Continuum medium in the current configuration at


time t
Boundary of B
Body force (per unit mass)

Entropy created inside

Local entropy per unit mass per unit time

Ce

Elasticity tensor
Elasticity matrix (Voigt notation)
Inelasticity tensor
Cauchy deformation tensor
Calor especfico a volumen constante
Calor especfico a presin constante
Cohesion

cc

Solute concentration

Right deformation Cauchy-Green tensor

DV

Dilation

D
r
dA
r
da
dV

Rate-of-Deformation tensor
Area element vector in the reference configuration
Area element vector in the current configuration
Volume element

[C ]

in

C
c
Cv

Cp

m
s2

Continuum medium in the reference configuration at

Left deformation Cauchy-Green tensor, Finger


deformation tensor
Piola deformation tensor

m
s2

N
m3

J
sK
J
kg s K
Pa
Pa
Pa

Pa
mol
m3
m
m
m2
m2
m3

XIV

SOLVING PROBLEMS BY MEANS OF CONTINUUM MECHANICS

Green-Lagrange strain tensor, or Lagrangian finite


strain tensor, or Green-St_Venant strain tensor

Almansi strain tensor, or Eulerian finite strain tensor

E
e i
i , j, k

Youngs modulus, or elastic modulus


Cartesian basis in symbolic notation
Cartesian basis

Deformation gradient (pseudo-tensor)

G
H

Shear modulus
Biot strain tensor

Total entropy

r
HO

Angular momentum

Jacobian determinant
r

J ( X , t)
r

Material displacement gradient tensor

j ( x, t )

Spatial displacement gradient tensor

r
J

Diffusion tensor

Thermal conductivity tensor

Kinetic energy

r
L

Linear momentum

Spatial velocity gradient

m
M

Mass
Mandel stress tensor
Outward unit normal to the boundary (current
configuration)
Outward unit normal to the boundary (reference
configuration)

r
p

Body force (per unit volume)

P
p

First Piola-Kirchhoff stress tensor


Thermodynamic pressure

r r
q( x , t )

Cauchy heat flux (non-convective vector)

Q
Q
r
r ( x, t )

Orthogonal tensor
Thermal work
Radiant heat constant, or heat source (per unit mass)

m
m
m
m
Pa

m
m
Pa
J
K
kgm 2
= Js
s
m3
m3
m
m
m
m
mol
m2s
W
J
=
mK smK
J
kg m
s
m
sm
kg
Pa

N
m3
Pa
Pa
J
m2s
J

J
kg s

XV

NOMENCLATURE

R
S

Orthogonal tensor of polar decomposition


Second Piola-Kirchhoff stress tensor

Entropy flux

T
r r
t (n) ( x , t , n )
r (N )
t0
r
T ( x, t )
t
t0 t = 0

Biot stress tensor


Traction vector (current configuration)
Temperature
Time
Initial time

K
s
s
J
=W
s
J
kg

Specific internal energy

Displacement vector (Eulerian)


m
Displacement vector (Lagrangian)
m
Right stretch tensor, or Lagrangian stretch tensor, or
material stretch tensor
Left stretch tensor, or Eulerian stretch tensor, or
spatial stretch tensor

r r
r r
V ( X , t) v ( X , t)

Velocity (reference configuration)

r r
v ( x, t )

Velocity (current configuration)

Spin tensor, rate-of-rotation tensor, or vorticity


tensor

w int
r
X
r
x

Stress power

Coefficient of thermal expansion

ij
1 , 2 , 3

Kronecker delta
Principal strains (infinitesimal strain)

Unit Extension

ijk

Permutation symbol, or Levi-Civita tensor


components
Linear dilatation (volume ratio) (small deformation
regime)

Pa
Pa

Rate of change of the internal energy

r
V ( x, t )

J
kg s m 2
Pa

Traction pseudo-vector (reference configuration)

U&

r r
u( x , t )
r r
u( X , t )
r
U( X , t )

Pa

Vector position (material coordinate)


Vector position (spatial coordinate)

Infinitesimal strain tensor

Specific entropy

Bulk modulus

m
s
m
s
m rad
=
ms
s
J
=W
s
m
m
1
K

m
m

m
m
m
m
J
kg K
Pa

XVI

SOLVING PROBLEMS BY MEANS OF CONTINUUM MECHANICS

Thermal diffusivity

Stretch

Lam constants
Poissons ratio

Solute mass density

Fluid mass density


r

0 (X )
r

( x, t )
1

Mass density (reference configuration)


Mass density (current configuration)

Specific volume

r
N
r
S
m
1 , 2 , 3
r
oct
r
oct
max

Cauchy stress tensor, or true stress tensor


Normal traction vector
Tangential traction vector
Mean stress
Principal stresses
Normal octahedral vector
Tangential octahedral vector
Maximum shear stress
Kirchhoff stress tensor
Angle of internal friction

Helmholtz free energy, specific (per unit mass)

Helmholtz free energy (per unit volume)

( ) = e

Strain energy density

m2
s
m
m
Pa

kg
m3
kg
m3
kg
m3
kg
m3
m3
kg
Pa
Pa
Pa
Pa
Pa
Pa
Pa
Pa

J
kg

J
m3
J
m3

Solving Problems by Menas of Continuum Mechanics

Useful Formulas
Some Trigonometric Identities
sin( ) = sin( ) cos( ) cos( ) sin( )
cos( ) = cos( ) sin( ) m sin( ) sin( )
1
cos( ) cos( ) = [cos( + ) + cos( )]
2
1
sin( ) sin( ) = [cos( ) cos( + )]
2
1
sin( ) cos( ) = [sin( + ) + sin( )]
2
1
cos 2 ( ) = [1 + cos(2 )]
2
1
sin 2 ( ) = [1 cos(2 )]
2
+
cos( ) + cos( ) = 2 cos
cos

2 2
+
cos( ) cos( ) = 2 sin
sin

2 2
m
sin( ) sin( ) = 2 sin
cos

2 2

cos 2 ( ) + sin 2 ( ) = 1
sin( )
cos( )
1
sec( ) =
cos( )
1
cos( )
=
cot( ) =
tan( ) sin( )
tan( ) =

sec2 ( ) + tan 2 ( ) = 1

sin( x)
=1
x 0
x

lim

1 cos( x)
=0
x 0
x

lim

List of trigonometric identity

SOLVING PROBLEMS BY MEANS OF CONTINUUM MECHANICS

XVIII

http://en.wikipedia.org/wiki/Trigonometric_identity

Some Series Expansions


f
1 2 f
1 3 f
2
( x a) +
(
x

a
)
+
( x a ) 3 + L (Taylors series)
x
2! x 2
3! x 3
n(n 1) 2
x +L ;
( x < 1) (binomial series)
(1 + x) n = 1 + nx +
2!
1
1
exp x = 1 + x + x 2 + x 3 + L
2!
3!
1 3 1 5
Ln(1 + x) = x x + x L
3!
5!
1
1
cos( x) = 1 x 2 + x 4 L
2!
4!
1 3 1 5
sin( x) = x x + x L
3!
5!
1 2 1 4
cosh( x) = 1 + x + x + L
2!
4!
1
1
sinh( x) = x + x 3 + x 5 + L
3!
5!
1
1

tan( x) = x + x 3 + x 5 + L x <
3
15
2

f ( x) = f (a ) +

Some Derivatives
d
d x
d
d
1
1
(exp x ) = exp x
;
(a ) = Ln(a) a x
;
[Ln( x)] =
;
[log a ( x)] =
dx
dx
dx
x
dx
xLn(a )
d
1 f ( x)
[Ln( f ( x))] =
dx
f ( x) x
where e exp stands for exponential and Ln for natural logarithm, where it fulfils:
Ln(exp x ) = x

and

d
[sin( x)] = cos( x) ;
dx
1
d
[arcsin( x)] =
dx
1 x2

exp Ln( x ) = x

d
d
[cos( x)] = sin( x) ;
[tan( x)] = sec 2 ( x)
dx
dx
d
d
1
1
;
[arccos(x)] =
;
[arctan(x)] =
dx
dx
1 + x2
1 x2

List of derivatives
http://en.wikipedia.org/wiki/List_of_derivatives

Some Integrals

XIX

USEFUL FORMULAS

exp dx = exp
x

x dx = Ln( x)

f ( x)
exp f ( x ) dx = exp f ( x )
x

Ln( x)dx = xLn( x) x + C

where e exp stands for exponential and Ln for natural logarithm.

u
= sin 1 + C
a
a u
du
1
u
= tan 1 + C
2
2
a
a +u
a

du

du
2

u a

u
1
sec 1 + C
a
a

List of integrals
http://en.wikipedia.org/wiki/List_of_integrals

Some Function Solutions


Quadratic function
ax 2 + bx + c = 0

solution

x=

b b 2 4ac
2a

( a 0)

Ruffinis rule
http://en.wikipedia.org/wiki/Ruffini%27s_rule

Expressions related to the circle:


Equation of the circle: ( x1 a ) 2 + ( x2 b) 2 r 2
Area enclosed by a circumference: A = r 2
Length of circumference: C = 2r
x2
r

x1

The relationship ds = rd holds, where ds is the infinitesimal arc length.

XX

SOLVING PROBLEMS BY MEANS OF CONTINUUM MECHANICS

Expressions related to the ellipse:


x2
r
x

f2

f1

x1

Equation of the ellipse: x = r =


Eccentricity: e =

a 2 b2
a2

p
1 + e cos
0 < e < 1 , where a 2 =

Area enclosed by an ellipse: A = ab .

p2
holds.
(1 e 2 ) 2

The indicial notation was introduced by Einstein (1916, sec.


5), who later jested to a friend, "I have made a great discovery
in mathematics; I have suppressed the summation sign every
time that the summation must be made over an index which
occurs twice..." (Kollros 1956; Pais 1982, p. 216).
Ref. (Wolfram MathWorld (Einstein Summation))

1 Tensors

1.1 Solved Problems


1.1.1

Vectors, Indicial Notation

Problem 1.1
r

Let a and b be arbitrary vectors. Prove that the following relationship is true:

(ar br ) (ar br ) = (ar ar )(br br ) (ar br )

Solution:

(ar br ) (ar br )

2
r r 2
r r
= a b = a b sin
r 2 r 2
r 2 r 2
r 2 r
= a b sin 2 = a b 1 cos 2 = a b
2
r 2 r 2
r r
r 2 r 2 r r 2
= a b a b cos = a b a b
r r r r
r r 2
= (a a) b b a b

( ) ( )

r r

( )

r r

r
a

r 2
b cos 2

where we have taking into account that a a = a and b b = b .

Problem 1.2
r

Show that: if c = a + b , the module of c can be expressed by means of the following


relationship:
r
c =

r
a

r
r r
+ 2 a b cos + b
r

where is the angle formed by the vectors a and b .


Solution:
Starting from the module definition of a vector it fulfills that:

SOLVING PROBLEMS BY MEANS OF CONTINUUM MECHANICS

r r
a+b

)(

r r r r
= a+b a+b
r r r r r r r r
= aa + ab + b a + b b
r r r r
r r r 2 r r r 2
Taking into account that a a = a , b b = b and a b = b a (commutative), we can
2

conclude that:
r r
a+b

r r r r r r r r
= aa + ab + b a + b b
r
= a

r r r
+ 2a b + b

r
= a

r
r r
+ 2 a b cos + b

r
b

with which we demonstrate a + b =


r r
ab =

r
a

r
r r
2 a b cos + b

r
a

r
r r
+ 2 a b cos + b

r r
a+b

. Then, it is easy to show that

.
r

r r

NOTE: Starting from the equation a + b = a + 2a b + b


value a + b

r
a

we can conclude that the

is maximum when = 0 holds, then

r
= a

r r r
+ 2a b + b

r
= a

r
r r
+2a b + b

r
r
= a + b

= 0
2

r
b

r
a

r
r r
r
a+b = a + b

Then, for any value of 0 < 180 the outcome a + b will be less than a + b . Then,
r
r r
r
a + b a + b holds:

r
b

r
r
r r
r
c = a+b a + b

r r r
c = a+b
r
b

r
a
r

In a similar fashion we can show that a c + b and b a + c which is known as the


triangle inequality, where it holds that:
a<c+b

b<a+c

c<a+b

University of Castilla-La Mancha


Ciudad Real - Spain

Draft

By: Eduardo W. V. Chaves (2014)

1 TENSORS

Problem 1.3
1
2

Given the following functions () = E and () = E 2 , demonstrate whether these


functions show a linear transformation or not.
Solution:
( 1 + 2 ) = E [1 + 2 ] = E 1 + E 2 = (1 ) + ( 2 ) (linear transformation)

()
( 1 + 2 ) = ( 1 ) + ( 2 )
( 2 )
( 1 )

1 + 2

1
2
(1 + 2 ) = (1 ) + ( 2 ) has not been satisfied:

The function () = E 2 does not show a linear transformation because the condition

(1 + 2 ) = E[1 + 2 ]2 = E [12 + 21 2 + 22 ] = E12 + E 22 + E 21 2


1
2

1
1
2
2
= (1 ) + ( 2 ) + E 1 2 ( 1 ) + ( 2 )

1
2

1
2

()
(1 + 2 )

(1 ) + ( 2 )
( 2 )
(1 )
1

1 + 2

Problem 1.4
Consider the points: A(1,3,1) , B (2,1,1) , C (0,1,3) and D(1,2,4 ) , defined in the Cartesian
coordinate system.

University of Castilla-La Mancha


Ciudad Real - Spain

Draft

By: Eduardo W. V. Chaves (2014)

SOLVING PROBLEMS BY MEANS OF CONTINUUM MECHANICS

1) Find the parallelogram area defined by AB and AC ; 2) Find the volume of the

parallelepiped defined by AB , AC and AD ; 3) Find the projection vector of AB onto BC .


Solution:

1) Firstly we calculate the vectors AB and AC :

) (
)
r
b = AC = OC OA = (0i + 1j + 3k ) (1i + 3j + 1k ) = 1i 2j + 2k

r
a = AB = OB OA = 2i 1j + 1k 1i + 3j + 1k = 1i 4j + 0k

Next, we evaluate the vector product as follows:


i
r r
ab= 1

j k
4 0 = ( 8)i 2j + ( 6)k

1 2

Then, the parallelogram area can be obtained by using the following definition:
r r
A = a b = (8) 2 + (2) 2 + ( 6) 2 = 104

2) Next, we can evaluate the vector AD as:

) (

r
c = AD = OD OA = 1i + 2j + 4k 1i + 3j + 1k = 0i 1j + 3k

we can obtain the volume of the parallelepiped as follows:

) (0i 1j + 3k ) ( 8i 2j 6k )

r r r
r r r
V (a, b, c ) = c a b =

= 0 + 2 18 = 16

3) The BC vector can be calculated as:

) (

BC = OC OB = 0i + 1j + 3k 2i 1j + 1k = 2i + 2j + 2k

Hence, it is possible to evaluate the projection vector of AB onto BC as follows:

proj BC AB =

BC AB

BC
BC
4
1
42
3

BC

BC

( 2i + 2j + 2k ) (1i 4j + 0k ) ( 2i + 2j + 2k )
( 2i + 2j + 2k ) ( 2i + 2j + 2k )

( 2 8 + 0 ) ( 2i + 2j + 2k ) = 5 i 5 j 5 k
(4 + 4 + 4 )
3
3
3

Problem 1.5
Rewrite the following equations using indicial notation:
1) a1 x1 x 3 + a 2 x 2 x 3 + a 3 x3 x 3
Solution:

a i xi x 3

(i = 1,2,3)

2) x1 x1 + x2 x2
Solution:

xi x i

(i = 1,2)

University of Castilla-La Mancha


Ciudad Real - Spain

Draft

By: Eduardo W. V. Chaves (2014)

1 TENSORS

a11 x + a12 y + a13 z = b x

3) a 21 x + a 22 y + a 23 z = b y

a 31 x + a 32 y + a 33 z = b z

Solution:
a11 x1 + a12 x 2 + a13 x 3 = b1

a 21 x1 + a 22 x 2 + a 23 x 3 = b2
a x + a x + a x = b
32 2
33 3
3
31 1

a1 j x j = b1

a 2 j x j = b2

a 3 j x j = b3
dummy
index j

free

index
i

a ij x j = bi

As we can appreciate in this problem, the use of the indicial notation means that the equation
becomes very concise. In many cases, if algebraic operation do not use indicial or tensorial
notation they become almost impossible to deal with due to the large number of terms
involved.
Problem 1.6
Show that:
a) 3 p v p = v3 ; b) 3i A ji = A j 3 ; c) ij ijk ; d) i 2 j 3 Aij .
Solution:
The Kronecker delta components are:
11
ij = 21
31

12 13 1 0 0
22 23 = 0 1 0
32 33 0 0 1

(1.1)

a) The expression ( 3 p v p ) has no free index, then the result is a scalar:

3 p v p = 31v1 + 32 v 2 + 33 v 3 = v3

(1.2)

b) The expression 3i A ji has one free index ( j ), then the result is a vector:

3i A ji = 31 A j1 + 32 A j 2 + 33 A j 3 = A j 3

(1.3)

c) The expression ij ijk has one free index ( k ), then the result is a vector:

ij ijk = 1 j 1 jk

123

1111k

+ 2 j 2 jk + 3 j 3 jk
1
424
3
123
+ 21 21k
+

+ 31 31k
+

12 12 k + 22 22 k

+ 32 32 k

+
+

13 13k

+ 23 23k

(1.4)

+
+ 33 33k

thus ij ijk = 0 k is the null vector. Note that ij ijk = iik = 11k + 22 k + 33k = 0 k .
d)

i 2 j 3 Aij = A23

University of Castilla-La Mancha


Ciudad Real - Spain

Draft

(1.5)

By: Eduardo W. V. Chaves (2014)

SOLVING PROBLEMS BY MEANS OF CONTINUUM MECHANICS

Problem 1.7
Expand the equation: Aij x i x j

(i, j = 1,2,3)

Solution: The indices i, j are dummy indices, and indicate index summation and there is no
free index in the expression Aij x i x j , therefore the result is a scalar. So, we expand first the
dummy index i and later the index j to obtain:

expanding j

expanding i
A1 j x1 x j + A2 j x 2 x j + A3 j x 3 x j
Aij x i x j
1
424
3 1
424
3 1
424
3
A11 x1 x1 A21 x 2 x1 A31 x 3 x1
+
+
+

A12 x1 x 2

A22 x 2 x 2

A32 x 3 x 2

A13 x1 x 3

A23 x 2 x 3

A33 x 3 x 3

Rearranging the terms we obtain:


Aij x i x j = A11 x1 x1 + A12 x1 x 2 + A13 x1 x3 + A21 x 2 x1 + A22 x 2 x 2 +
A23 x 2 x3 + A31 x3 x1 + A32 x 3 x 2 + A33 x 3 x3

Problem 1.8
Obtain the numerical value of:
1) ii jj
Solution:

ii jj = ( 11 + 22 + 33 )( 11 + 22 + 33 ) = 3 3 = 9

2) 1 1
Solution:

1 1 = 1 1 = 11 = 1

NOTE: Note that the following algebraic operation is incorrect 1 1 = 3 11 = 1 ,


since what must be replaced is the repeated index, not the number
Problem 1.9
a) Prove the following is true ijk pjk = 2 ip , ijk ijk = 6 and ijk a j a k = 0 i . b) Obtain the
numerical value of ijk 2 j 3k 1i .
Solution: a) Using the equation ijk pqk = ip jq iq jp , and by substituting q for j , we
obtain:
ijk pjk = ip jj ij jp = ip 3 ip = 2 ip

Based on the above result, it is straight forward to check that:


ijk ijk = 2 ii = 6
c) Note that ijk = ikj , i.e. it is antisymmetric in jk and also note that a j a k is a symmetric
second-order tensor. So, as we know, the double scalar product between a symmetric and an
antisymmetric second-order tensors is zero, thus:
r

ijk a j ak = ijk (a a ) jk = 0i = (a a )i = 0i
b) ijk 2 j 3k 1i = 123 = 1

University of Castilla-La Mancha


Ciudad Real - Spain

Draft

By: Eduardo W. V. Chaves (2014)

1 TENSORS

Problem 1.10
Get the value of the following expressions:
a) ijk i1 j 2 3k
b) ijk pqk = ip jq iq jp for the following cases:
b.1) i = 1, j = q = 2, p = 3
b.2) i = q = 1, j = p = 2
c) ( ijk A jp c p A kq c q + i1 )( ist A sa c a A tb c b + i1 )
where ijk is the permutation symbol and ij is the Kronecker delta.
Solution:
a) ijk i1 j 2 3k = 123 = 1 ; b.1)
b.2)

12 k 32 k = 121 321 + 122 322 + 123 323 =


= 0 (1) + 0 0 + 0 0 = 0

12 k 21k = 121 211 + 122 212 + 123 213


= 0 0 + 0 0 + 1 (1) = 1

c) Note that the result of A jp c p = b j is a vector, and also note that the following is true
r

ijk A jp c p A kq c q = [( A c) ( A c )]i = (b b) i = 0 i , with which we can obtain:


r

( ijk A jp c p A kq c q + i1 )( ist A sa c a A tb c b + i1 ) = i1 i1 = 11 = 1

Reminder: Permutation symbol


ijk = jki = kij

ijk = ikj = kji = jik

ijk = 1

ijk = 1

k =3
i =1

k =2

i=2

j =1
i =1

k =1

i=2

j =1

i =1
i=2
i=3

0
0
0

j=2

0
0
-1

i=3
j =3

j=2

0
1

0
1

0
0
0

i=3
j =3

-1

j =1

0
-1
0

j=2

1
0
0

j =3

0
0
0
ij 3

0
0
ij 2

0
ij1

University of Castilla-La Mancha


Ciudad Real - Spain

Draft

By: Eduardo W. V. Chaves (2014)

SOLVING PROBLEMS BY MEANS OF CONTINUUM MECHANICS

NOTE: The second-order tensor ijk wk can easily be obtained as follows:


ijk wk = ij1 w1 + ij 2 w2 + ij 3 w3
0 0 0
0 0 1
0 1 0 0

= w1 0 0 1 + w2 0 0 0 + w3 1 0 0 = w3
0 1 0
1 0 0
0 0 0 w2

w3
0
w1

w2
w1
0

Problem 1.11
r

Write in indicial notation: a) the modulus of the vector a ; b) cos , where is the angle
r
r
between the vectors a and b .
Solution:
r
a

r r
= a a = a i e i a j e j = a i a j ij = a i a i = a j a j
r
thus, it is also true that b = b i b i .
2

r r

r
a = ai ai

r r

By definition a b = a b cos where:


r
a b = a i e i b j e j = a i b j ij = a i b i = a j b j

Taking into account that the index cannot appear more than twice in a term of the expression,
we can express cos as follows:
r r
a jb j
ab
cos = r r =
ai ai b k b k
a b

Problem 1.12
Show the Schwarz inequality:
r r
r r
ab a b

Schwarz inequality

(1.6)

Solution:
Let us consider a scalar , then the following is true:
r2 r
r r
r r r
r r
r r
r
r r
a b = (a b) (a b) = a a 2 a b b a + b b 0
1424
3
0

r2
r2
r r
= a 2 2a b + b 0

r r
r 2
r 2 2
r r
ab
f ( ) = a 2a b + b 0 , if we now obtain the value when = r 2 we obtain that:
a

University of Castilla-La Mancha


Ciudad Real - Spain

Draft

By: Eduardo W. V. Chaves (2014)

1 TENSORS

2
r r
r r

r r
r
r r ( a b) r 2
2
(a b)
(a b )

f = r 2 = a r 2 2(a b) r 2 + b 0

a
a
a

r
r
r r
r r
r r
r 2
r 2 ( a b) 2
r r (a b) r 2 (a b) 2
( a b) 2
= a
r 4 2 ( a b) r 2 + b = r 2 2 r 2 + b 0
a
a
a
a
r
r
r 2
(a b ) 2
= r 2 + b 0
a

r r 2
r 2 (a
b)
b r 2
a

r 2 r 2
r r
a b ( a b) 2

r r
r r
a b ab

Q.E.D.
Alternative solution
r r

r r

r r

Taking in account that 0 cos 1 we obtain a b = a b cos a b , thus we conclude


r r

r r

that a b a b .

Problem 1.13

(r r ) (r r )
r r
r r
Solution: The vector product (a b ) can be expressed as (a b ) = a e b e =
r r
r r
Likewise, it is possible to express (c d) as (c d) = c d e , thus:
(ar br ) (cr dr ) = a b e ) ( c d e ) = a b c d e e

Rewrite the expression a b c d without using the vector product symbol.

nlm

ijk

nlm

ijk

nlm

ijk a j b k e i

= ijk nlm a j b k c l d m in = ijk ilm a j b k c l d m

Taking into account that ijk ilm = jki lmi


jki lmi = jl km jm kl = jki ilm , we obtain:

and

by

applying

the

equation

ijk ilm a j b k c l d m = ( jl km jm kl ) a j b k c l d m = a l b m c l d m a m b l c l d m
r r

(r r )
(ar br ) (cr dr ) = (ar cr ) (br dr ) (ar dr )(br cr )

Since a l c l = (a c ) and b m d m = b d holds true, we can conclude that:


r

Therefore, it is also valid when a = c and b = d , thus:

(ar br ) (ar br ) = ar br

( ) ( )( )

r r r r
r r r r
r
= (a a) b b a b b a = a

r
b

( )

r r
ab

which is the same equation obtained in Problem 1.1.


r

r r

NOTE: We can start from the above equation to show a b = a b sin , i.e.:

r r 2 r 2 r 2 r r
r 2 r 2
r r
a b = a b (a b) 2 = a b a b cos

University of Castilla-La Mancha


Ciudad Real - Spain

Draft

) = ar
2

r 2
r 2 r 2
b 1 cos 2 = a b sin 2

By: Eduardo W. V. Chaves (2014)

SOLVING PROBLEMS BY MEANS OF CONTINUUM MECHANICS

10

r r
r r
a b = a b sin
r

r r

r r

Note that 0 sin 1 , with that we can prove that a b = a b sin a b , thus
r r
r r
ab a b

Problem 1.14
Show that:
a) ijk a i a j b k = 0 ;
r r

b) ijk (a k b 3 i1 j 2 + a j b 2 i1 k 3 + a i b1 j 2 k 3 ) = a b ;
c) Aij A ji is an invariant.
Solution:
a) ijk a i a j b k = ij1a i a j b1 + ij 2 a i a j b 2 + ij 3 a i a j b 3 . The term ij1a i a j b1 can be evaluated as
follows:
ij1a i a j b1 = 1 j1 a1 a j b1 + 2 j1a 2 a j b1 + 3 j1a 3 a j b1
= 111 a1 a1b1 + 211 a 2 a1b1 + 311 a 3 a1b1 +
+ 121 a1 a 2 b1 + 221 a 2 a 2 b1 + 321 a 3 a 2 b1 +
+ 131 a1 a 3 b1 + 231 a 2 a 3b1 + 331 a 3 a 3 b1
= 321 a 3 a 2 b1 + 231 a 2 a 3b1 = a 3 a 2 b1 + a 2 a 3b1
=0

In the same way we obtain ij 2 a i a j b 2 = ij 3 a i a j b 3 = 0 .


b)
ijk a k b 3 i1 j 2 + ijk a j b 2 i1 k 3 + ijk a i b1 j 2 k 3 =

r r

12 k a k b 3 + 1 j 3 a j b 2 + i 23 a i b1 = a 3b 3 + a 2 b 2 + a1b1 = a i b i = a b

Problem 1.15

(r r ) (r r ) r [r

] r [r

Prove that a b c d = c d (a b) d c (a b)

Solution: Expressing the correct equality term in indicial notation we obtain:

] [

r r r
rr r r
cr d (a
b) d c (a b) = c p d i ijk a j b k d p c i ijk a j b k

[ (

ijk a j b k c p d i ijk a j b k c i d p

)]

)]

ijk a j b k (c p d i c i d p )

Using the Kronecker delta the above equation becomes:


ijk a j b k pm c m d n ni im c m d n np

[ (

ijk a j b k

)c

m dn

pm

ni im np )

and by applying the equation pm ni im np = pil mnl , the above equation can be rewritten
as follows:
( ijk a j b k ) c m d n ( pil mnl )

University of Castilla-La Mancha


Ciudad Real - Spain

Draft

pil ( ijk a j b k ) ( mnl c m d n )

By: Eduardo W. V. Chaves (2014)

1 TENSORS

11

(r r )

(r r )

Since ijk a j b k and mnl c m d n represent the components of a b and c d , respectively,


we can conclude that:

[(r r ) (r r )]

pil [( ijk a j b k ) ( mnl c m d n )] = a b c d

Problem 1.16
r

Let a , b , c be linearly independent vectors, and v be a vector, demonstrate that:


r
r
r r
r
v = a + b + c 0

v i = a i + b i + c i 0 i

components

where the scalars , , are given by:

ijk v i b j c k
pqr a p b q c r

; =

ijk a i v j c k
pqr a p b q c r

; =

ijk a i b j v k
pqr a p b q c r

b) Given three linearly independent vectors, show that: when interchanging two rows or two
r r r
columns the sign of the determinant a (b c ) changes.
r

Solution: a) The scalar product made up of v and ( b c ) becomes:


r r r
r r r
r r r
r r r
v (b c ) = a (b c ) + b (b c ) + c (b c )
14243
14243
=0

=0

r r r
v (b c )
= r r r
a (b c )

which is the same as:

v1
b1

v2
b2

v3
b3

c1

c2

c3

a1
b1

a2
b2

a3
b3

c1

c2

c3

v1
v2

b1
b2

c1
c2

v3

b3

c3

a1
a2

b1
b2

c1
c2

a3

b3

c3

ijk v i b j c k
pqr a p b q c r

One can obtain the parameters and in a similar fashion, i.e.:


r r r
r r r
r r r
r r r
v (a c ) = a (a c ) + b ( a c ) + c ( a c )
14243
14243
=0
=0
r r r
r r r
jik a j v i c k
v (a c ) ijk v i a j c k
a (v c)
= r r r
= r r r =
=
b (a c ) pqr b p a q c r qpr a q b p c r a (b c )
r r r
r r r
r r r
r r r
v (a b ) = a ( a b ) + b ( a b ) + c (a b )
14243
14243
=0
=0
r r r
r r r
jki a j b k v i a (b v )
v (a b) ijk v i a j b k
= r r r =
=
= r r r
c (a b) pqr c p a q b r qrp a q b r c p a (b c )
r
NOTE 1: We can restructure the v -components as follows:

v 1 a1

v i = v 2 = a 2
v a
3 3

b1
b2
b3

c 1 a1

c 2 = a 2
c 3 a 3

b1
b2
b3

c 1 z1

c 2 z 2 = B ij z j
c 3 z 3

where we have denoted by z1 = , z 2 = , z 3 = , in which:

University of Castilla-La Mancha


Ciudad Real - Spain

Draft

By: Eduardo W. V. Chaves (2014)

SOLVING PROBLEMS BY MEANS OF CONTINUUM MECHANICS

12

v1
v2

b1
b2

c1
c2

vb c
v
=z1 = ijk i j k = 3
a1
pqr a p b q c r

b3

c3

a2

b1
b2

c1
c2

a3

b3

c3

a1
a2

b1
b2

v1
v2

ab v
a b3
=z 3 = ijk i j k = 3
a1 b 1
pqr a p b q c r

v3

a2

b2

c1
c2

a3

b3

c3

a1
a2

v1
v2

c1
c2

av c
a
; =z 2 = ijk i j k = 3
=
a1
B
pqr a p b q c r

v3

c3

a2

b1
b2

c1
c2

a3

b3

c3

B (1)

B (2)
B

B (3)
B

where B (i ) is the determinant of the resulting matrix by replacing the column (i) of the
r

matrix B by the v -components. With that we can state that:


Given v i = B ij z j

zi =

B (i )
B

Cramers rule

NOTE 2: Although we have demonstrated for 3 3 matrix, this procedure is also valid for
matrices of n-dimensions, which is known, in the literature, as Cramers Rule.
NOTE 3: The solution ( z i ) is possible if B 0 .
NOTE 4: If v i = 0 i we have B ij z j = 0 i and B (i ) = 0 i , with that according to Cramers rule
we have:
z i B = B (i ) = 0 i

Note that the non-trivial solution z i 0 i is only possible if and only if B = 0 , (see Problem
1.51).

University of Castilla-La Mancha


Ciudad Real - Spain

Draft

By: Eduardo W. V. Chaves (2014)

1 TENSORS

13

r r r

b) The determinant defined by a (b c ) = [a, b, c ] in indicial notation becomes ijk a i b j c k .


Note that the following holds:
ijk = jki = kij

ijk = ikj = kji = jik


i

Thus:
j

a1
ijk a i b j c k = b 1

a2
b2

a3
b3

c1

c2

c3

a1

a2

a3

= c1
b1

c2
b2

c 3 = ikj a i b j c k
b3

b1

b2

b3

c1
a1

c2
a2

c 3 = jki a i b j c k
a3

r r r

r r r

r r r

ijk aib j c k = [a, b, c ] = ikj aib j c k = [a, c , b] = jki aib j c k = [b, c , a]

Problem 1.17
a) Show that

( ) (

r r
r
r r r r r r r r r r r
a (b c ) = (a c ) b a b c = b c c b a
r r
r
r r
r r r
a (b a) = [(a a)1 a a] b
r r
r r
b) Obtain the explicit component of the tensor [(a a)1 a a] .

Solution:

(r r )

(r r )

a) Taking into account that (d) i = b c i = ijk b j c k and that a d q = qjk b j c k , we obtain:

[ar (br cr )]

= rsi a s ( ijk b j c k )
= rsi ijk a s b j c k = rsi jki a s b j c k

= rj sk rk sj a s b j c k
= rj sk a s b j c k rk sj a s b j c k
= ak br c k a jb j c r
r r
r r
= b r (a c ) c r a b
r r r rr r
= b(a c ) c a b r

( )
( )]

So, proving that:

( ) (

= asb r c s asb s c r
= (b r c s b s c r )a s
r r r r r
= b c c b a r

[(

) ]

r r r
r r r r r r r r r r r
a b c = (a c ) b a b c = b c c b a

Note that it is also true that:

University of Castilla-La Mancha


Ciudad Real - Spain

Draft

By: Eduardo W. V. Chaves (2014)

SOLVING PROBLEMS BY MEANS OF CONTINUUM MECHANICS

14

) (

r r r r r
r r r
r r
r r r r r r r
r
a b c = b c c b a = [(a c )1 c a] b = b a (a b)1 c
r

In the particular case when a = c we obtain:

[ar (br ar )] = (a a )b
r

(a j b j )a r = (a j a j )b p rp (a j b p

jp ) a r

= (a j a j ) rp (a j jp )a r b p = (a j a j ) rp a p a r b p
r r
r r r
= [(a a)1 a a] b r

b) Taking into account the above equation we can obtain:


r r
r r
[(a a)1 a a] ij = (a k a k )
(a 22

ij

+ a 32 )

= a1 a 2
a1 a 3

ai a j =

(a12

+ a 22

1 0 0 a 1 a 1
1 0 a1 a 2
0 0 1 a1 a 3

a 32 ) 0

a1 a 2
a2a2
a1 a 3

a1 a 3
a1 a 3
a 3 a 3

a1 a 3

a1 a 3
(a12 + a 32 )
a1 a 3
(a12 + a 22 )
a1 a 2

Problem 1.18
Show the Jacobi identity:

r r r r r r r r r r
a b c + b (c a) + c a b = 0
r

(r r )

r r r

) ( )

(r r ) r

Solution: By means of Problem 1.17 in which a b c = (a c ) b a b c was proven, we


can obtain that:

( ) ( )

r r r
r r r r r r
b (c a) = b a c b c a

r r r
r r r r r r
c a b = c b a (c a) b
r r
r r
r r
r r
Then, by considering that the dot product is commutative, i.e. (a c ) = (c a) , a b = b a ,
r r
r r
b c = c b , we can conclude that:
r r r r r r
(a c )b a b c
r r r+ r r r r
r r r r
r r r r r
a b c + b (c a) + c a b = b a c b c a = 0
r r r+ r r r
c b a (c a) b
;

( ) ( )

1.1.2

( ) ( )

( )
) ( ) ( )
( )

Algebraic Operations with Higher Order Tensors

Problem 1.19
Define the order of the tensors represented by their Cartesian components: v i , ijk , Fijj , ij ,
C ijkl , ij . Determine the number of components in tensor C .
Solution: The order of the tensor is given by the number of free indices, so it follows that:
r r

First-order tensor (vector): v , F ; Second-order tensor: , ; Third-order tensor: ;


Fourth-order tensor: C

University of Castilla-La Mancha


Ciudad Real - Spain

Draft

By: Eduardo W. V. Chaves (2014)

1 TENSORS

15

The number of tensor components is given by the maximum index range value, i.e.
i, j , k , l = 1,2,3 , to the power of the number of free indices which is equal to 4 in the case of
C ijkl . Thus, the number of independent components in C is given by:
3 4 = (i = 3) ( j = 3) (k = 3) (l = 3) = 81

The fourth-order tensor C ijkl has 81 components.


Problem 1.20

(r r ) r (r r ) r

(r r ) (r r ) (r r ) r

Show that a) a b c = b c a ; b) a b c d = b c a d
Solution:

a)

r r r
r r
r r r
r
a b c = (ai e i b j e j ) c k e k = ai e ib j c k jk = (b k c k )ai e i = (b c )a (b c ) a
r r r r
b) The expression a b c d , which is a second-order tensor, can be expressed in indicial

)(

notation as follows:

[(ar br ) (cr dr )] = (ar br ) (cr dr ) r = (a b )(c d ) = a b c d


ij

ik

kj

i k

r r
= (b k c k )(ai d j ) = (b c )(a d)ij
123

i k k

= b k c k ai d j

scalar

Problem 1.21
Expand and simplify the expression A ij xi x j when a) A ij = A ji ; b) A ij = A ji .
Solution:
By expanding A ij xi x j we obtain:
A ij xi x j = A 1 j x1 x j + A 2 j x 2 x j + A 3 j x3 x j =
= A 11 x1 x1 + A 21 x 2 x 1 + A 31 x3 x 1 +

(1.7)

A 12 x1 x 2 + A 22 x 2 x 2 + A 32 x 3 x 2 +
A 13 x1 x 3 + A 23 x 2 x 3 + A 33 x3 x 3

a) If A ij = A ji (symmetry) we have
A ij xi x j = A11x12 + 2A12 x1x 2 +2 A13 x1x 3 + A 22 x22 + 2A 23 x2 x3 + A 33 x32

(1.8)

b) If A ij = A ji (antisymmetry) we have
A ij xi x j = 0

(1.9)

as expected, since:

r
r
r r
A ij xi x j = x A x = A : ( x x )
(1.10)
r r
That is, if A is an antisymmetric and ( x x ) is a symmetric tensor, the double scalar product

between them is always equal to zero.


Problem 1.22
University of Castilla-La Mancha
Ciudad Real - Spain

Draft

By: Eduardo W. V. Chaves (2014)

SOLVING PROBLEMS BY MEANS OF CONTINUUM MECHANICS

16

Let and T be second-order tensors, whose Cartesian components are:


5 2 4
ij = 1 2 1
4 3 6

3 1 2
Tij = 4 2 1
1 3 8

(1.11)

Obtain T : .
Solution:
T : = Tij ij

(1.12)

Tij ij = T1 j 1 j + T2 j 2 j + T3 j 3 j
123
123
123
T1111
+

T21 21
+

T31 31
+

T12 12

T22 22

T32 32

+
T23 23

+
T33 33

+
T13 13

(1.13)

thus,
Tij ij = 5 3 + 2 1 + 4 2 + (1) 4 + 2 2 + 1 1 + 4 1 + 3 3 + 6 8 = 87

(1.14)

Problem 1.23
Given the B tensor components:
3 2 4
B ij = 1 5 3
5 7 9

(1.15)

Obtain:
a) C ij = B ik B kj ; b) D ij = B ik B jk ; c) E ij = B ki B kj ; d) C ii , D ii , E ii
Solution:
C = B B

3 2 4 3 2 4 31 44 54
C ij = B ik B kj = 1 5 3 1 5 3 = 23 48 46
5 7 9 5 7 9 67 108 122

(1.16)

D = B BT

3 2 4 3 2 4
29 25 65

= 1 5 3 1 5 3 = 25 35 67
5 7 9 5 7 9
65 67 155

D ij = B ik B jk

3 2 4 3 2 4 35 46 60
E ij = B ki B kj = 1 5 3 1 5 3 = 46 78 86
5 7 9 5 7 9 60 86 106

(1.17)

E = BT B

(1.18)

Then:

University of Castilla-La Mancha


Ciudad Real - Spain

Draft

By: Eduardo W. V. Chaves (2014)

1 TENSORS

17

C ii = C 11 + C 22 + C 33 = 31 + 48 + 122 = 201
D ii = D11 + D 22 + D 33 = 29 + 35 + 155 = 219

(1.19)

E ii = E11 + E 22 + E 33 = 35 + 78 + 106 = 219

NOTE: We can verify that the following is true: Tr (B B T ) = Tr (B T B) = B : B .


Problem 1.24
Given the B second-order tensor components:
1 0 2
B ij = 0 1 2
3 0 3

Obtain: a) B kk

b) B ij B ij

c) B jk B kj

Solution:
a) B kk = B 11 + B 22 + B 33 = 1 + 1 + 3 = 5
b) B ij B ij = B 1 j B 1 j

123

+ B 2 jB 2 j + B 3 jB 3 j
123
123

B 11B 11
+

B 21B 21
+

+ B 31B 31
+

B 12B 12

+ B 22B 22

+ B 32B 32

+
B 13B 13

+
+ B 23B 23

+
+ B 33B 33

which the result is:


B ij B ij = 1 1 + 0 0 + 2 2 + 0 0 + 1 1 + 2 2 + 3 3 + 0 0 + 3 3 = 28

c) B jk B kj = B 1k B k1 + B 2k B k 2 + B 3k B k 3
123

123

B 11B 11
+

B 12B 21
+
B 13B 31

+ B 22B 22
+
+ B 23B 32

B 21B 12
+

123

B 31B 13
+

+ B 32B 23
+
+ B 33B 33

B jk B kj = B 11B 11 + B 22B 22 + B 33B 33 + 2B 21B 12 + 2B 31B 13 + 2B 32B 23


= 1 1 + 1 1 + 3 3 + 2(0 0) + 2(3 2 ) + 2(0 2 ) = 23

Problem 1.25
The D tensor is given by the algebraic operation D = A : B . Obtain the D components for
the following cases:
a)

when

2 3 2
Aij = 4 1 1
1 1 5

University of Castilla-La Mancha


Ciudad Real - Spain

2 3 1
Bij = 1 2 1
1 2 5

Draft

By: Eduardo W. V. Chaves (2014)

SOLVING PROBLEMS BY MEANS OF CONTINUUM MECHANICS

18

b)

when

7 13 14
Aik B kj = 11 18 11
16 27 31

Aik B jk

13 9 17
= 15 9 13
18 12 32

Solution:
a) A : B = 2 2 + 3 3 + 2 1 + 4 1 + 1 2 + 1 1 + 1 1 + 1 2 + 5 5 = 50
b) Taking into account that Tr ( A B T ) = Tr ( AT B) = A : B and Aik B jk = A B T , we can
conclude that A : B = Tr ( A B T ) = 13 + 9 + 32 = 54 .
Problem 1.26
Let us consider the following second-order tensor T = Tr ( E )1 + ( F : E ) E which in indicial
notation is Tij = E kk ij + ( Fkp E kp ) E ij . If the components of E and F are given by:
2 1 4
E ij = 1 5 0
2 0 1

4 3 1
Fij = 2 0 3
2 0 0

a) Obtain the T tensor components. b) Are T and E coaxial tensors? Prove it.
Solution:
Next, we obtain the following scalars:
Tr ( E ) = 2 + 5 + 1 = 8
F : E = 2 4 + 1 3 + 4 1 + 1 2 + 5 0 + 0 3 + 2 2 + 0 0 + 1 0 = 21

Then
1 0 0
2 1 4 50 21 84

Tij = 80 1 0 + 211 5 0 = 21 113 0


0 0 1
2 0 1 42 0 29

Two tensors are coaxial when they have the same eigenvectors or when the relationship
T E = E T holds:
50 21 84 2 1 4 289
Tik E kj = 21 113 0 1 5 0 = 155
42 0 29 2 0 1 142
2 1 4 50 21 84 289
E ik Tkj = 1 5 0 21 113 0 = 155
2 0 1 42 0 29 142

155 284
586 84
42 197
155 284
586 84
42 197

with that we conclude that they are coaxial.


Problem 1.27
Obtain the result of the following algebraic operations: I : I , I : I , I : I , I : I , I : I , I : I ,
I : I , I : I , I sym : I sym , I sym : I , I : I sym , where

I = 11 = I ijkl e i e j e k e l

where

I ijkl = ik jl

(1.20)

I = 11 = I ijkl e i e j e k e l

where

I ijkl = il jk

(1.21)

University of Castilla-La Mancha


Ciudad Real - Spain

Draft

By: Eduardo W. V. Chaves (2014)

1 TENSORS

I = 1 1 = I ijkl e i e j e k e l

19

I ijkl = ij kl

where

(1.22)

Solution:
(I : I ) ijkl = I ijpq I pqkl = ip jq pk ql = ik jl = I ijkl
( I : I ) ijkl = I ijpq I pqkl = iq jp pl qk = ik jl = I ijkl

( I : I ) ijkl = I ijpq I pqkl = ij pq pq kl = qq ij kl = 3I ijkl


( I : I ) ijkl = I ijpq I pqkl = iq jp pk ql = il jk = I ijkl
(I : I ) ijkl = I ijpq I pqkl = ip jq pl qk = il jk = I ijkl
(I : I ) ijkl = I ijpq I pqkl = ip jq pq kl = iq jq kl = ij kl = I ijkl
( I : I ) ijkl = I ijpq I pqkl = iq jp pq kl = iq jq kl = ij kl = I ijkl

We summarize the above in tensorial notation as follows:


I : I = (11) : (11) = 11 = I
I : I = (11) : (11) = 11 = I

I : I = (1 1) : (1 1) = 3(1 1) = 3I
I : I = (11) : (11) = 11 = I
I : I = (11) : (11) = 11 = I
I : I = (11) : (1 1) = 1 1 = I
I : I = (11) : (1 1) = 1 1 = I

Taking into account the definition I sym =


I sym : I sym

(
[(
[
(

)(
) (

)
) (
]

1
11 + 11 : 11 + 11
4
1
= 11 : 11 + 11 : 11 + 11 : 11 + (11 : 11)
4
1
= 11 + 11 + 11 + 11
4
1
= 11 + 11
2
= I sym
=

(1 1) : I sym = I : I sym

(
(

) ( )
) ( )

1
1
1
I + I : I = I : I + I : I = I + I = I =1 1
2
2
2
1
1
1
= I : I + I = I :I + I : I = I + I = I =1 1
2
2
2

I sym : (1 1) = I sym : I =

1.1.3

1
1
I + I = 11 + 11 , we conclude that:
2
2

Tensor Transpose

University of Castilla-La Mancha


Ciudad Real - Spain

Draft

By: Eduardo W. V. Chaves (2014)

SOLVING PROBLEMS BY MEANS OF CONTINUUM MECHANICS

20

Problem 1.28
Let A , B and C be arbitrary second-order tensors. Show that:

A : (B C ) = B T A : C = A C T : B

Solution: Expressing the term A : (B C ) in indicial notation we obtain:

A : (B C ) = A ij e i e j : B lk e l e k C pq e p e q
= A ij B lk C pq e i e j : kp e l e q
= A ij B lk C pq kp il

jq

= A ij B ik C kj

Note that, when we are dealing with indicial notation the position of the terms does not
matter, i.e.:
A ij B ik C kj = B ik A ij C kj = A ij C kj B ik

We can now observe that the algebraic operation B ik A ij is equivalent to the components of
the second-order tensor (B T A ) kj , thus,

B ik A ij C kj = (B T A ) kj C kj = B T A : C .

Likewise, we can state that A ij C kj B ik = (A C T ): B .


Problem 1.29
r

Let u , v be vectors and A be a second-order tensor. Show that the following relationship
holds:
r
r r
r
u AT v = v A u

Solution:

1.1.4

r
r
u AT v
u i e i A jl e l e j v k e k

r
r
= v A u
= v k e k A jl e j e l u i e i

u i A jl il v k jk
u l A jl v j

= v k kj A jl u i il
= v j A jl u l

Symmetry and Antisymmetry

Problem 1.30
Show that : W = 0 is always true when is a symmetric second-order tensor and W is an
antisymmetric second-order tensor.
Solution:
: W = ij (e i e j ) : Wlk (e l e k ) = ij Wlk il

jk

= ij Wij (scalar)

Thus,

University of Castilla-La Mancha


Ciudad Real - Spain

Draft

By: Eduardo W. V. Chaves (2014)

1 TENSORS

21

ij Wij = 1 j W1 j + 2 j W2 j + 3 j W3 j
123
1
424
3
1
424
3
31W31
21W21
11W11
+
+
+
32 W32
22 W22
12 W12
+
+
+
33W33
23W23
13W13

Taking into account the characteristics of a symmetric and an antisymmetric tensor, i.e.
12 = 21 , 31 = 13 , 32 = 23 , and W11 = W22 = W33 = 0 , W21 = W12 , W31 = W13 ,
W32 = W23 , the equation above becomes:
:W =0

Q.E.D.
Problem 1.31

Show that a) M Q M = M Q sym M ; b) A : B = A sym : B sym + A skew : B skew where M is a


vector, and Q , A , B are arbitrary second-order tensors; c) Show that the relationship
ijk T jk = 0 i holds, where T is symmetric, i.e. Tij = T ji .
Solution:
r

a) M Q M = M (Q sym + Q skew ) M = M Q sym M + M Q skew M


r

(r

Since the relation M Q skew M = Q skew : 1


M M = 0 holds, it follows that:
424
3
symmetric tensor

r
r r
r
M Q M = M Q sym M
r
r
NOTE: We can make the geometric interpretation of M Q skew M = 0 as follows. Note that
r r r
r
r r r r
the algebraic operation Q skew M = q (M) is a vector, thus M Q skew M = M q (M) = 0 , which
r
r r
implies that M and q (M) are orthogonal vectors. With that we conclude that: the projection of
r
r r
an antisymmetric second-order tensor according to the direction ( M ) is a vector ( q (M) ) which
r
is orthogonal to M , see figure below:

r
Q M

r r r
q (M ) M = 0
r
r r
q (M) = Q skew M

r
M

r
M

b)
University of Castilla-La Mancha
Ciudad Real - Spain

Draft

By: Eduardo W. V. Chaves (2014)

SOLVING PROBLEMS BY MEANS OF CONTINUUM MECHANICS

22

A :B

= ( A sym + A skew ) : (B sym + B skew )


skew
= A sym : B sym + 1
A sym
: B43
+1
A skew
B sym + A skew : B skew
42
42: 43
=0

=0

= A sym : B sym + A skew : B skew

Then, it is also valid that:


A : B sym = A sym : B sym

A : B skew = A skew : B skew

c)

Q.E.D.

ijk T jk = ij1 T j1 + ij 2 T j 2 + ij 3 T j 3 = 0 i
= i11 T11 + i 21 T21 + i 31 T31 + i12 T12 + i 22 T22 + i 32 T32 + i13 T13 + i 23 T23 + i 33 T33
= i 21 T21 + i 31 T31 + i12 T12 + i 32 T32 + i13 T13 + i 23 T23 = 0 i

Then, the vector components are:


i =1

1 jk T jk = 132 T32 + 123 T23 = T32 + T23 = 0 T32 = T23

i=2

2 jk T jk = 231 T31 + 213 T13 = T31 T13 = 0 T31 = T13

i=3

3 jk T jk = 321 T21 + 312 T12 = T21 + T12 = 0 T21 = T12

with that we demonstrate that if ijk T jk = 0 i holds, this implies that T is symmetric, i.e.
T = TT .

Problem 1.32
Given a second-order tensor A in which the components of the symmetric part is known in
the Cartesian system:
A ijsym

4 2 0
= 2 1 0
0 0 3

A N
, where the unit vector components are N = [1 0 0] .
Obtain N
i

Solution:
A N
=N
A sym N
with that we obtain:
In Problem 1.31 it was shown that N
4 2 0 1
sym
sym

N A N = N A N = N i A ij N j = [1 0 0] 2 1 0 0 = 4
0 0 3 0

Problem 1.33
Let W be an antisymmetric tensor. a) Show that W W is a symmetric second-order tensor.
b) Show also that (W T W W) : 1 = 0 .
Solution:
a) If we show that (W W ) skew = 0 holds, then we prove that W W is symmetric.
(W W ) skew =

University of Castilla-La Mancha


Ciudad Real - Spain

] [

1
1
1
( W W) (W W )T = (W W) WT WT = [(W W) W W ] = 0
2
2
2

Draft

By: Eduardo W. V. Chaves (2014)

1 TENSORS

23

where we have applied the antisymmetric tensor property W = W T .


Alternative solutions a) Taking into account the definition of antisymmetric tensor W = W T ,
we obtain:
W W = W T W = W T W T = (W W ) T

We can also check the symmetry by means of the tensor components:


W12
W13 0
W12
W13
0

( W W ) ij = W12
0
W23 W12
0
W23
W13 W23
0 W13 W23
0
W122 W132
W12 W23
W13 W23

2
2
= W13 W23
W12 W23
W12 W13
2
W12 W23
W12 W13
W132 W23

b) (W T W W ) : 1 = (W pi W pk Wkj ) ij = W pi (W pk Wki ) = W : (W W) = 0 , since the double


scalar product between a symmetric tensor (W W ) and an antisymmetric tensor ( W ) is zero.
Problem 1.34
1
2

Let B be a second-order tensor such that B pq = pqs a s with a i = ijk B jk . Prove that B is an
antisymmetric tensor.
Solution:
1
1
1
B pq = pqs a s = pqs sjk B jk = pqs sjk B jk = pqs jks B jk
2
2
2

Taking into account the relationship pqs jks = pj qk pk qj we obtain:


1
1
1
1
B pq = pqs jksB jk = ( pj qk pk qj )B jk = ( pj qk B jk pk qjB jk ) = (B pq B qp ) = B skew
pq
2
2
2
2

Alternative solution:
Taking into account that B qp = qps a s and pqs = qps , we can conclude that:
B pq = pqs a s = qps a s = B qp

B = B T (antisymmetric)

Problem 1.35
Show that the tensor A skew A sym + A sym A skew is an antisymmetric tensor.
Solution: Denoting by B = A skew A sym + A sym A skew , and by taking into account that
A skew = (A skew ) T , A sym = (A sym ) T , we can conclude that:
B = A skew A sym + A sym A skew = A skew A sym A sym ( A skew )T = A skew A sym ( A skew A sym )T

= 2( A skew A sym ) skew

Problem 1.36

Let
T be an arbitrary second-order tensor, and n be a vector. Check if the relationship
r
r
n T = T n is valid.

University of Castilla-La Mancha


Ciudad Real - Spain

Draft

By: Eduardo W. V. Chaves (2014)

SOLVING PROBLEMS BY MEANS OF CONTINUUM MECHANICS

24

Solution:

r
n T = n i e i Tkl (e k e l )
= n i Tkl ik e l

r
T n = Tlk (e l e k ) n i e i
= n i Tlk ki e l

and

= n k Tkl e l

= n k Tlk e l

= (n1 T1l + n 2 T2 l + n 3 T3l )e l

= (n1 Tl1 + n 2 Tl 2 + n 3 Tl 3 )e l

With the above we can prove that n k Tkl n k Tlk , then:


r
r
n T T n

If T is a symmetric tensor, it follows that the relationship n T sym = T sym n holds.


Problem 1.37
r

Obtain the axial vector w associated with the antisymmetric tensor ( x a ) skew .
r

Solution: Let z be an arbitrary vector, it then holds that:


r r
r r r
( x a ) skew z = w z
r
r r
where w is the axial vector associated with ( x a ) skew . Using the definition of an

antisymmetric tensor:

r r
r r
1 r r
1 r r r r
( x a ) skew = ( x a ) ( x a ) T = [ x a a x ]
2
2
r r skew r r r
and by replacing it with ( x a ) z = w z , we obtain:
1 r r r r r r r
[x a a x ] z = w z [xr ar ar xr ] zr = 2wr zr
2
r r r r r r
r r
By using the equation [x a a x ] z = z ( x a ) , (see Problem 1.17), the above

equation becomes:

[xr ar ar xr ] zr = zr ( xr ar ) = (ar xr ) zr = 2wr zr

with the above we can conclude that:


r 1 r r
r r
w = (a x ) is the axial vector associated with ( x a ) skew
2

Problem 1.38
Let us consider two symmetric tensors W (1) and W ( 2) , and their axial vectors represented
r
r
respectively by w (1) and w ( 2) . Show that:
r
r
r
r
W (1) W ( 2 ) = ( w ( 2 ) w (1) ) ( w (1) w ( 2) )1
r
r
Tr W (1) W ( 2 ) = 2( w (1) w ( 2 ) )

Solution: Given antisymmetric tensor properties, we can obtain that:


r r
r
W (1) a = w (1) a
r
r r
T
a W (1) = a w (1)
r
r r
a W (1) = a w (1)
r
r r
a W (1) = a w (1)

University of Castilla-La Mancha


Ciudad Real - Spain

and

Draft

r r
r
W ( 2) a = w ( 2 ) a

By: Eduardo W. V. Chaves (2014)

1 TENSORS

25

Then, by applying the dot product (a W (1) ) (W ( 2) a) we obtain:


r

(ar W ) (W
(1)

( 2)

a) = (a w (1) ) ( w ( 2) a)
r

We will continue the development in indicial notation:


(a i Wij(1) )(W jk(1) a k ) = ( ijk a j wk(1) )( ipq w (p2 ) a q )

a i (Wij(1) W (jk1) )a k = a j ( ijk ipq wk(1) w (p2 ) )a q = a j ( jp kq jq kp ) wk(1) w (p2) a q

[
[w

= a j jp
=aj

jq

(1) ( 2 )
kq w k w p

(1) ( 2 )
q wj

(1) ( 2 )
jq wk wk

(1) ( 2 )
kp wk w p

]a

]a

In tensorial notation the above equation becomes:

] ar

r
r r r
r
r
r
a W (1) W ( 2 ) a = a ( w ( 2 ) w (1) ) ( w (1) w ( 2 ) )1

With that we demonstrate that W (1) W ( 2) = ( w ( 2) w (1) ) ( w (1) w ( 2) )1 .


b)

r
r
r
r
r
r
r
r
Tr W (1) W ( 2 ) = Tr ( w ( 2 ) w (1) ) ( w (1) w ( 2) )1 = Tr ( w ( 2 ) w (1) ) Tr ( w (1) w ( 2 ) )1
r
r
r
r
r
r
= ( w ( 2 ) w (1) ) ( w (1) w ( 2) ) 1
Tr2
[3
1] = 2( w (1) w ( 2 ) )
=3

Alternative solution
In this alternative solution we use the tensor components in which it fulfills:
Wij(1)

= W12(1)
W (1)
12

Wij( 2 )

= W12( 2 )
W ( 2 )
12

W12(1)
0
W12(1)
W12( 2 )
0
W12( 2 )

W13(1) 0
(1)
(1)
W23
= w3
0 w2(1)

w3(1)
0
w1(1)

W13( 2 ) 0
(2)
(2)
W23
= w3
0 w2( 2)

w2(1)

w1(1)
0

w3( 2 )
0
w1( 2 )

w2( 2 )

w1( 2 )
0

With that we obtain:

[W

(1)

W ( 2) ]ij

Wik(1) Wkj( 2 )

= Wik(1) Wkj( 2 )

= w3(1)
w (1)
2

w3(1) w3( 2 ) w2(1) w2( 2 )

w1(1) w2( 2)
=

w3( 2 ) w1(1)

w3(1)
0
w1(1)

w2(1) 0

w1(1) w3( 2 )
0 w2( 2)

w3( 2 )
0
w1( 2 )

w2(1) w1( 2 )

w3( 2 ) w3(1) w1(1) w1( 2 )


w3( 2 ) w2(1)

w2( 2 )

w1( 2 )
0

w3(1) w1( 2 )

w2( 2) w3(1)

(1) ( 2 )
( 2 ) (1)
w2 w2 w1 w1

In the term (11) we sum and subtract the term w1( 2) w1(1) , in the term (22) we sum and subtract
the term w2( 2) w2(1) and in the term (33) we add and subtract the term w3( 2) w3(1) , so,

University of Castilla-La Mancha


Ciudad Real - Spain

Draft

By: Eduardo W. V. Chaves (2014)

SOLVING PROBLEMS BY MEANS OF CONTINUUM MECHANICS

26

Wik(1) Wkj( 2 )

w1( 2) w1(1)

= w2( 2) w1(1)
w ( 2) w (1)
3 1

w1( 2 ) w3(1)

w2( 2 ) w3(1) +
w3( 2 ) w3(1)

w1( 2) w2(1)
w2( 2) w2(1)
w3( 2) w2(1)

w1(1) w1( 2 ) w2(1) w2( 2) w3(1) w3( 2)

0
+

w1(1) w1( 2 )

w2(1) w2( 2 )

w3(1) w3( 2 )

w1(1) w1( 2 )

w2(1) w2( 2 )

(1) ( 2 )
w3 w3

which is the same as:


Wik(1) Wkj( 2 ) = wi( 2) w (j1) ( w1(1) w1( 2) + w2(1) w2( 2) + w3(1) w3( 2) ) ij = wi( 2) w (j1) ( wk(1) wk( 2) ) ij

We leave to the reader to obtain the trace.


NOTE: The alternative solution by means of components was made only as a check. The
reader must give priority to the solution via indicial or tensorial notation, since the solution via
components is not always so simple to obtain.

1.1.5

Cofactor. Adjugate. Inverse. Particular Tensors.


Determinant

Problem 1.39

r r

Show that Tr (a b) = a b .
Solution:

r r
r r
Tr (a b) = Tr (ai e i ) (b j e j ) = aib j Tr e i e j = aib j (e i e j ) = aib j ij = aibi = a b

Problem 1.40
1
2

Given Tij = E kk ij + 2 Eij , W = Tij E ij , and P = Tij Tij , show that:


W = E : E +

[Tr( E )]2

P = 4 2 E : E + (3 + 4 )[Tr ( E )]

and

Solution 1: (Indicial notation)

1
1
1
1
W = Tij E ij = E kk ij + 2 E ij E ij = E kk ij E ij + 2 E ij E ij = E kk E ii + 2 E ij E ij
2
2
2
2

since E kk = E ii = Tr (E ) and Eij Eij = E : E , we can conclude that W = E : E +

)(

P = TijTij = Ekk ij + 2 Eij Eqq ij + 2 Eij

[Tr( E )]2 .

= Ekk ijEqq ij + Ekk ij 2 Eij + 2 EijEqq ij + 2 Eij 2 Eij


= 2 Ekk ii Eqq + 2 Ekk Eii + 2 Eii Eqq + 4 2 Eij Eij = 3 2 Ekk Eqq + 4 Ekk Eii + 4 2 Eij Eij
= (3 + 4 )Ekk Eqq + 4 2 Eij Eij

With that we demonstrate that P = 4 2 E : E + (3 + 4 )[Tr ( E )]2 .


Solution 2: (Tensorial notation)
University of Castilla-La Mancha
Ciudad Real - Spain

Draft

By: Eduardo W. V. Chaves (2014)

1 TENSORS

27

In tensorial notation we obtain:


1
T = Tr ( E )1 + 2 E , W = T : E , and P = T : T
2

Then
1
1
1
1
W = T : E = (Tr ( E )1 + 2 E ) : E = (Tr ( E )1 : E + 2 E : E ) = (Tr ( E ) Tr ( E ) + 2 E : E )
2
2
2
2
=

[Tr( E )]2 + E : E
P = T : T = (Tr ( E )1 + 2 E ) : (Tr ( E )1 + 2 E )
= [Tr ( E )] 1
: 1 + 2 Tr ( E ) 1
: E + 2 Tr ( E ) {
E : 1 + (2 ) 2 E : E
{
{
2

=3

= Tr ( E )

= Tr ( E )

= 3 [Tr ( E )] + 4 [Tr ( E )] + 4 E : E = (3 + 4 )[Tr ( E )] + 4 2 E : E


2

Problem 1.41
Let ij be the second-order tensor components which are a function of ij , ij = ij ( ij ) , and
is given by:
ij = kk ij + 2 ij

Tensorial

= Tr ( )1 + 2

where and are scalars. Starting with the above equation, obtain an expression for ij in
function of ij , i.e. ij = ij ( ij ) . Express the result in indicial and tensorial notation.
Solution:
Indicial notation

Tensorial notation

ij = kk ij + 2 ij

= Tr ( )1 + 2

2 ij = ij kk ij

2 = Tr ( )1

ij =

1
ij
kk ij
2
2

Tr ( )1

2
2

Next, we need to obtain the following trace kk , to do this we obtain the trace of ij :
Indicial notation
ij = kk ij + 2 ij

Tensorial notation

(i = j )

ii = kk ii + 2 ii = kk 3 + 2 kk

kk = (3 + 2 ) kk
kk =

1
kk
(3 + 2 )

: 1 = Tr ( )1 : 1 + 2 : 1
Tr ( ) = Tr ( )3 + 2 Tr ( )

Tr ( ) =

1
Tr ( )
(3 + 2 )

Then
Indicial notation

University of Castilla-La Mancha


Ciudad Real - Spain

Tensorial notation

Draft

By: Eduardo W. V. Chaves (2014)

SOLVING PROBLEMS BY MEANS OF CONTINUUM MECHANICS

28

ij =
=

ij
kk ij
2
2

Tr ( )1

2
2

1
1

ij
kk ij
2
2 (3 + 2 )

Tr ( )1

2
2 (3 + 2 )

Problem 1.42
Let T be a second-order tensor. Show that:

(T ) = (T )
m T

T m

and

( )

Tr T T

( )

= Tr T m .

Solution:

(T )

( )
For the second demonstration we can use the trace property Tr (T ) = Tr (T ) , thus:
Tr (T ) = Tr (T ) = Tr (T )
m T

= (T T L T ) = T T T T L T T = T T
T

T m

m T

Problem 1.43
Show that T : 1 = Tr (T ) , where T is an arbitrary second-order tensor.
Solution:
T : 1 = Tij e i e j : kl e k e l = Tij kl ik

jl

= Tij ij = Tii = T jj = Tr ( T )

Problem 1.44
Show that if and D are second-order tensors, the following relationship is valid:
D = Tr ( D )

Solution: We start with the following definition:


D = ij D ji
= kj D jl ik il = kj D jl lk
= kj D jl lk
123
( D )

kl

= ( D) kl lk = ( D) kk = ( D) ll
= Tr ( D)

An alternative demonstration would be:


D = ijD ji = ijD jk ik = ( D ) : 1 = Tr ( D )

Problem 1.45
Show that:
det (S ) S =

1
rjk tpq S rt S jp S kq
6

(1.23)

Solution:

University of Castilla-La Mancha


Ciudad Real - Spain

Draft

By: Eduardo W. V. Chaves (2014)

1 TENSORS

29

det (S ) = ijk S 1i S 2 j S 3k

(1.24)

pqr det(S ) = ijk S pi S qj S rk

(1.25)

pqr pqr det (S ) = pqr ijk S pi S qj S rk

(1.26)

1
424
3
6

det (S ) =

1
pqr ijk S pi S qj S rk
6

(1.27)

Problem 1.46
Show that A tpq = rjk A rt A jp A kq .
Solution:
We start with the following definition:
A = rjk A r1 A j 2 A k 3

A tpq = rjk tpq A r1A j 2 A k 3

(1.28)

and also taking into account that the term rjk tpq can be replaced by:
rjk tpq

rt rp rq
= jt jp jq = rt jp kq + rp jq kt + rq jt kp rq jp kt jq kp rt kq jt rp
kt kp kq
(1.29)

Then, by substituting (1.29) into (1.28) we can obtain:


A tpq = A t1A p 2 A q 3 + A p1A q 2 A t 3 + A q1A t 2 A p 3 A q1A p 2 A t 3 A t1A q 2 A p 3 A p1A t 2 A q 3

= A t1 1 jk A pj A qk + A t 2 2 jk A pj A qk + A t 3 3 jk A pj A qk = rjk A rt A jp A kq = rjk A tr A pj A qk

NOTE: Let us consider that C = A B ( C ij = A ik B kj ), then we can obtain


C = A B = rjk C r1C j 2 C k 3 = rjk [A B ]r1 [A B ] j 2 [A B ]k 3 = rjk ( A rt B t1 )( A jpB p 2 )( A kqB q 3 )
= rjk A rt A jp A kqB t1B p 2B q 3 = A tpq B t1B p 2B q 3 = A B

So, we have shown that A B = A B .


Alternative solution:
Considering that tpq

1t 1 p 1q 1t 2t 3t
= 2t 2 p 2 q = 1 p 2 p 3 p
3 t 3 p 3 q 1q 2 q 3 q

and that A B = A B we can

obtain:
A tpq

1t 2t 3t A11 A12 A13 1t 2t 3t A11 A12 A13

= rjk tpq A r1A j 2 A k 3 = 1 p 2 p 3 p A 21 A 22 A 23 = 1 p 2 p 3 p A 21 A 22 A 23


1q 2 q 3q A 31 A 32 A 33 1q 2 q 3q A 31 A 32 A 33

Note that 1t A11 + 2t A 21 + 3t A 31 = st A s1 = A t1 , with that we can obtain:

University of Castilla-La Mancha


Ciudad Real - Spain

Draft

By: Eduardo W. V. Chaves (2014)

SOLVING PROBLEMS BY MEANS OF CONTINUUM MECHANICS

30

A tpq = rjk tpq A r1A j 2 A k 3

A t1

= A p1
A q1

At2
A p2
A q2

At3

A p 3 = rjk A tr A pj A qk
A q 3

Problem 1.47
1
6

Show that A = rjk tpq A rt A jp A kq .


Solution:
Starting with the definition A tpq = rjk A rt A jp A kq , and by multiplying both sides of the
equation by tpq , we obtain:
A tpq tpq = rjk tpq A rt A jp A kq

(1.30)

Note that tpq tpq = tt pp tp tp = tt pp tt = 6 . Then, the relationship (1.30) becomes:


A =

1
rjk tpq A rt A jp A kq
6

Problem 1.48
Show the following property:

r r r
r r r
r r r
r r r
(B a) (b c) (B b) (a c) + (B c ) (a b) = Tr (B) a (b c)

(1.31)

Solution:
Expressing in Voigt notation the left side of the above equation we obtain:
r

ijk (B a) i b j c k ijk (B b) i a j c k + ijk (B c ) i a j b k =


= ijk [(B i1a1 + B i 2 a 2 + B i 3 a 3 )b j c k (B i1b1 + B i 2 b 2 + B i 3b 3 )a j c k +
+ (B i1 c 1 + B i 2 c 2 + B i 3 c 3 )a j b k ]
= ijk [(B i1a1b j c k + B i 2 a 2 b j c k + B i 3 a 3b j c k ) (B i1b1 a j c k + B i 2 b 2 a j c k + B i 3b 3 a j c k ) +
+ (B i1 c 1 a j b k + B i 2 c 2 a j b k + B i 3 c 3 a j b k )]
= ijk [B i1 (a1b j c k b1a j c k + c 1a j b k ) + B i 2 (a 2 b j c k b 2 a j c k + c 2 a j b k ) +
+ B i 3 (a 3b j c k b 3 a j c k + c 3 a j b k )]
= (1 jk B 11 + 2 jk B 21 + 3 jk B 31 )(a1b j c k b1a j c k + c 1 a j b k ) +
+ (1 jk B 12 + 2 jk B 22 + 3 jk B 32 )(a 2 b j c k b 2 a j c k + c 2 a j b k ) +

(1.32)

+ (1 jk B 13 + 2 jk B 23 + 3 jk B 33 )(a 3b j c k b 3 a j c k + c 3 a j b k )

Note that:
a1 a 2
1 jk (a1b j c k b1a j c k + c 1a j b k ) = b1 b 2
c1

c2

a3
b 3 = ijk a i b j c k
c3

2 jk (a1b j c k b1a j c k + c 1a j b k ) = 3 jk (a1b j c k b1a j c k + c 1a j b k ) = 0

University of Castilla-La Mancha


Ciudad Real - Spain

Draft

By: Eduardo W. V. Chaves (2014)

1 TENSORS

31

whereby the equation in (1.32) becomes:

r r r
B 11 ijk a i b j c k + B 22 ijk a i b j c k + B 33 ijk a i b j c k = (B 11 + B 22 + B 33 ) ijk a i b j c k = Tr (B) a (b c )

Q.E.D.
Note also that:

r r r
r r r
r r r
r r r
(B T a) (b c ) (B T b) (a c ) + (B T c ) (a b) = Tr (B) a (b c )

since Tr (B T ) = Tr (B) I B . It is also valid the following:

r
r r r r
r r r
r
r r r
(B a) (b c ) + a ((B b) c) + a (b (B c )) = Tr (B) a (b c )
r r
r r r
r
r r
r
r r r
[(B a), b, c ] + [a, (B b), c ] + [a, b, (B c )] = I B [a, b, c ]

(1.33)

Problem 1.49
Show the following property:

r
r
r
r r r
( A a) ( A b) ( A c ) = det ( A ) a (b c )
r

(1.34)

where A is a non-singular second order tensor, and a , b and c are linearly independent
vectors.
Solution:
A non-singular tensor det( A ) A 0
r r r
r r r
a , b , c linearly independent vectors a b c 0 .

(r r )

We express the scalar triple product in indicial notation, i.e. a b c = ijk a i b j c k , and by
multiply both sides of this equation by the determinant of A we obtain:

r r r
a b c A = ijk a i b j c k A

It was proven in Problem 1.46 that A ijk = pqr A pi A qj A rk , thus:

r r r
a b c A = ijk aib j c k A = pqr A pi A qj A rk aib j c k = pqr ( A pi ai )( A qjb j )( A rk c k )
r
r
r
= ( A a) ( A b) ( A c )

Problem 1.50
r

Let a , b be arbitrary vectors and , be scalars. Show that:

r r
r r
det 1 + a b = 3 + 2 a b

(1.35)

Solution: The determinant of A is given by A = ijk A i1 A j 2 A k 3 . If we denote by


A ij ij + a i b j , thus, A i1 = i1 + a i b 1 , A j 2 = j 2 + a j b 2 , A k 3 = k 3 + a k b 3 , then
the equation in (1.35) can be rewritten as:

r r
det 1 + a b = ijk ( i1 + a i b 1 )

University of Castilla-La Mancha


Ciudad Real - Spain

Draft

j2

+ a j b 2 (

k3

+ ak b3 )

(1.36)

By: Eduardo W. V. Chaves (2014)

SOLVING PROBLEMS BY MEANS OF CONTINUUM MECHANICS

32

By developing the equation (1.36), we obtain:

r r
det 1 + a b = ijk 3 i1 j 2 k 3 + 2 a k b 3 i1 j 2 + 2 a j b 2 i1 k 3 + 2 a i b 1 j 2 k 3 +
+ 2 a j b 2 a k b 3 i1 + 2 a i a k b 1b 3 j 2 + 2 a i a j b 1b 2 k 3 + 3 a i a j a k b 1b 2 b 3

Note that: 3 ijk i1 j 2 k 3 = 3 123 = 3 ,

2 ( ijk a k b 3 i1 j 2 + ijk a j b 2 i1 k 3 + ijk a i b1 j 2 k 3 ) =

r r

2 (12 k a k b 3 + 1 j 3 a j b 2 + i 23 a i b1 ) = 2 (a 3b 3 + a 2 b 2 + a1b1 ) = 2 (a k b k ) = 2 (a b)
ijk a i a k b1b 3 j 2 = i 2 k a i a k b1b 3 = a1a 3b1b 3 a 3 a1b1b 3 = 0
ijk a i a j b1b 2 k 3 = ij 3 a i a j b1b 2 = 123 a1a 2 b1b 2 213 a 2 a1b1b 2 = 0
ijk a i a j a k b1b 2 b 3 = 0

Notice that, there was no need to expand the terms ijk a i a k b1b 3 j 2 , ijk a i a j b1b 2 k 3 , and
ijk a i a j a k b1b 2 b 3
to
realize
that
these
terms
equal
zero,
since
r

ijk a i a k b1b 3 j 2 = (a a) j b1b 3 j 2 = 0 , similarly for other terms.

Taking into account the above considerations we can prove that:

r r
r r
det 1 + a b = 3 + 2 a b

For the particular case when = 1 the above equation becomes:

r r
r r
det 1 + a b = 1 + a b
r r
Then, it is simple to prove that det a b = 0 , since
r r
r r r
det a b = 3 ijk a i a j a k b1b 2 b 3 = 3b1b 2 b 3 [a (a a)] = 0

NOTE: We can extrapolate the equation in (1.35) in such a way that:

det I sym + A B = 3 + 2 A : B

(1.37)

where I sym is the symmetric fourth-order unit tensor, A and B are second-order tensors.
Note that det ( I sym ) = (1) 3 + (1) 2 (0)(0 : 0 ) = 1 and det (1 1) = (0) 3 + (0) 2 (1)(1 : 1) = 0 .
Problem 1.51
r

Let A be an arbitrary second-order tensor. Show that there is a nonzero vector n 0 so that
r r
A n = 0 if and only if det ( A ) = 0 , Chadwick (1976).
r

Solution: Firstly, we show that, if det ( A ) A = 0 n 0 . Secondly, we show that, if


r r
n 0 det ( A ) A = 0 .

r r r

We assume that det ( A ) A = 0 , and we choose an arbitrary basis {f , g, h} (linearly


independent), then:

University of Castilla-La Mancha


Ciudad Real - Spain

Draft

By: Eduardo W. V. Chaves (2014)

1 TENSORS

r r r
r
r
r
f g h A = ( A f ) ( A g) ( A h)

33

(see Problem 1.49)

Due to the fact that det ( A ) A = 0 , the implication is that:

r
r
r
( A f ) ( A g) ( A h) = 0
r
r
r
Thus, we can conclude that the vectors ( A f ) , ( A g) , ( A h) , are linearly dependent. This

implies that there are nonzero scalars , , so that:


r

(r

( A f ) + ( A g) + ( A h) = 0 A f + g + h = 0 A n = 0
r

r r r

where n = f + g + h 0 since {f , g, h} is linearly independent.


r

Now we choose two vectors k , m , which are linearly independent to n . Once more, we
apply definition:
r r r
r
r
r
k (m n) A = ( A k ) [( A m) ( A n)]
r r r
r r r
r r
Considering that A n = 0 , and k (m n) 0 owing to the fact that k , m , n are linearly

independent, we can conclude that:

r r r
k (m n) A = 0
14243

A =0

Problem 1.52
Let F be an arbitrary second-order tensor. Show that the resulting tensors C = F T F and
b = F F T are symmetric tensors and semi-positive definite tensors. Also check in what condition are
C and b positive definite tensors.
Solution: Symmetry:
C T = (F T F )T = F T (F T )T = F T F = C
b T = (F F T ) T = (F T )T F T = F F T = b

Thus, we have shown that C = F T F and b = F F T are symmetric tensors.


To prove
start with
definite if

that the tensors C = F T F and b = F F T are semi-positive definite tensors, we


the definition of a semi-positive definite tensor, i.e., a tensor A is semi-positive
r
x A x 0 holds, for all x 0 . Thus:

x ( F T F ) x = F x F x
= ( F x ) ( F x )
= F x

x ( F F T ) x = x F F T x
= ( F T x ) ( F T x )
2
= F T x 0

Or in indicial notation:
x i C ij x j

= x i ( Fki Fkj ) x j
= ( Fki x i )( Fkj x j )
= Fki x i

x i bij x j

= x i ( Fik F jk ) x j
= ( Fik x i )( F jk x j )
= Fik x i

Thus, we proved that C = F T F and b = F F T are semi-positive definite tensors. Note that
x C x = F x

University of Castilla-La Mancha


Ciudad Real - Spain

equals zero, when x 0 , if F x = 0 . Furthermore, by definition


Draft

By: Eduardo W. V. Chaves (2014)

SOLVING PROBLEMS BY MEANS OF CONTINUUM MECHANICS

34

r
r
F x = 0 with x 0 if and only if det ( F ) = 0 , (see Problem 1.51). Then, the tensors
C = F T F and b = F F T are positive definite if and only if det ( F ) 0 .

Problem 1.53
r

Let dX (1) , dX ( 2) , dX (3) , dx (1) , dx ( 2) , dx (3) be vectors, and they are related to each other as
r
r
r
r
r
r
follows dx (1) = F dX (1) , dx ( 2) = F dX ( 2) , dx (3) = F dX (3) , where F is a non-singular
r
r
r
second-order tensor and F 1 . a.1) Considering dV = dx (1) (dx ( 2) dx (3) ) 0 and
r
r
r
dV0 = dX (1) (dX ( 2 ) dX (3) ) 0 , obtain a relationship between the scalars dV and dV0 in
r
r
r
r
terms of F . a.2) Obtain the relationship between c = dX ( 2) dX (3) 0 and
r
r
r
r
c * = dx ( 2 ) dx ( 3 ) 0 .
Solution
a.1) Taking into account the problem statement it fulfills that:

r
r
r
r
r
r
dV = dx (1) (dx ( 2 ) dx (3) ) = ( F dX (1) ) ( F dX ( 2 ) ) ( F dX (3) )
r

(r r )

In Problem 1.49 it was proven that a b c A = ( A a) ( A b) ( A c) , so

r
r r r
r
r
a b c A = ( A a) ( A b) ( A c )
r
r
r
r
r
r
dX (1) dX ( 2 ) dX (3) F = ( F dX (1) ) ( F dX ( 2 ) ) ( F dX (3) )

With that we conclude that:

r
r
r
r
r
r
r
r
r
dV = dx (1) (dx ( 2 ) dx (3) ) = ( F dX (1) ) ( F dX ( 2 ) ) ( F dX (3) ) = F dX (1) (dX ( 2 ) dX (3) )

thus
dV = F dV0

a.2) Taking into account the previous equations we get:

r
r
r
r
r
r
dV = F dV0
dx (1) (dx ( 2) dx (3) ) = F dX (1) dX ( 2) dX (3)
r
r
r
r
r
r
dx (1) ( dx ( 2 ) dx (3) ) = F ( F 1 dx (1) ) dX ( 2) dX (3)
r
r
r
r
r
r
dx (1) ( dx ( 2 ) dx (3) ) = dx (1) F F T dX ( 2 ) dX (3)
r
r
r
r
r
r
(dx ( 2 ) dx (3) ) = F F T dX ( 2 ) dX (3)
c * = F F T c

[
[

]
])

NOTE 1: Note that c * F c . We can rewrite the above equation as follows

r
r
r
r
dx ( 2 ) dx (3) = F F T dX ( 2 ) dX (3)

r
r
r
r
( F dX ( 2 ) ) ( F dX ( 3 ) ) = F F T dX ( 2 ) dX ( 3 )

The tensor F F T is known as the cofactor of F , i.e. cof ( F ) = F F T with this we define
the inverse of a tensor:
cof ( F ) = F F T
F 1 =

[F F ]

T T

= [cof ( F )]

F F 1 = [cof ( F )]

1
[cof (F )]T = 1 [adj( F )]
F
F

University of Castilla-La Mancha


Ciudad Real - Spain

Draft

By: Eduardo W. V. Chaves (2014)

1 TENSORS

r
dX (1)

35

r
r
dx (1) = F dX (1)

F
r
dX ( 3 )

r
r
r
c = dX ( 2) dX (3)

r
r
c* F c
r
r
c * = [cof ( F )] c

r
r
r
c * = dx ( 2 ) dx ( 3 )
r
r
dx (3) = F dX (3)

dV = F dV0

r
dX ( 2 )

r
r
r
dV0 = dX (1) (dX ( 2 ) dX (3) ) 0

r
r
dx ( 2 ) = F dX ( 2 )

r
dV = dx (1)

(dx ( 2) dx (3) ) 0
r

NOTE 2: Let us suppose now that F = A B , and let us consider three vectors a (b c ) 0 ,
r r
r
r r
r
and a * = B a , b * = B b , c * = B c , thus by apply the previous definitions we can state:

r
r r r
r
r
F a (b c ) = ( F a ) ( F b ) ( F c )

r
r
r
= ( A B a) ( A B b) ( A B c )
r
r
r
= ( A a* ) ( A b*) ( A c * )
r
r r r
r
r
= A a * (b * c * ) = A ( B a ) ( B b ) ( B c )
r r r
= A B a (b c )

With that we can conclude that: if F = A B then F = A B = A B .


Problem 1.54
Let A and B be orthogonal tensors, show that the tensor C = A B is also an orthogonal
tensor.
Solution: By definition, a tensor is orthogonal if C 1 = C T holds:
C 1 = ( A B) 1 = B 1 A 1 = B T A T = ( A B) T = C T

Q.E.D.
Problem 1.55
Show that adj( A B) = adj(B) adj( A ) and cof( A B) = [cof( A )] [cof(B)] .
Solution:
Based on the definition of the inverse of a tensor we can say that:
B 1 A 1 =

[adj(B)] [adj(A )]
B

A B B 1 A 1 = [adj(B)] [adj( A )] = [cof(B)]

A B (A B ) = [adj(B)] [adj( A )] =
1

AB

[cof( A)]T

( [cof(A)] [cof(B)] )

(1.38)

[adj(A B)] = [adj(B)] [adj(A)] = ([cof(A)] [cof(B)])T


A B

adj( A B) = [adj(B)] [adj( A )] = ([cof( A )] [cof(B)])

University of Castilla-La Mancha


Ciudad Real - Spain

Draft

By: Eduardo W. V. Chaves (2014)

SOLVING PROBLEMS BY MEANS OF CONTINUUM MECHANICS

36

where we have used the property A B = A B . Also taking into account the definition of
adjugate and cofactor we can conclude that:
adj( A B) = ([cof( A B)]) = ([cof( A )] [cof(B)])
T

[cof(A B)] = [cof(A)] [cof(B)]

(1.39)

Problem 1.56
Show that:
r
r
r r
( A a) ( A b) = [cof( A )] (a b)

(1.40)

Solution:
Starting from the equation A tpq = rjk A rt A jp A kq (see Problem 1.46), and by multiply both
sides by a t b p , we obtain:
A tpq a t b p = rjk A rt A jp A kq a t b p = rjk ( A rt a t )( A jp b p ) A kq

Multiplying both sides by A qs1 we obtain:


A tpq a t b p A qs1 = rjk ( A rt a t )( A jp b p ) A kq A qs1 = rjk ( A rt a t )( A jp b p ) ks = rjs ( A rt a t )( A jp b p )
1
Note that A qs
=

[cof ( A )] sq

holds, whereby the above equation becomes:

1
A tpq a t b p A qs
= A tpq a t b p

[cof ( A )] sq
A

= [cof ( A )] sq tpq a t b p = rjs ( A rt a t )( A jp b p )

r
r r
r
[cof( A )] (a b) = ( A a) ( A b)

Problem 1.57
Show that:

r
r
r
r
r r
r
r
r
r r r
a ( A b) ( A c ) + ( A a) b ( A c ) + ( A a) ( A b) c = Tr ([cof ( A )]) a (b c )

(1.41)
Solution:

In Problem 1.56 it was demonstrated that [cof( A )] (a b) = ( A a) ( A b) , thus the


following relationships hold:

r r
r
r
r
r
a [cof( A )] (b c ) = a ( A b) ( A c )
r
r
r r
r
r
r r
r
b [cof( A )] (a c ) = b [( A a) ( A c ) ] = ( A a) b ( A c )
r
r
r
r r
r
r
r
r
c [cof( A )] (a b) = c ( A a) ( A b) = ( A a) ( A b) c

Summing the three above equations we obtain:

r r
r
r
r r
r
r r
a [cof( A )] (b c ) b [cof( A )] (a c ) + c [cof( A )] (a b) =
r
r
r
r
r r
r
r
r
= a ( A b) ( A c ) + ( A a) b ( A c ) + ( A a) ( A b) c

University of Castilla-La Mancha


Ciudad Real - Spain

Draft

By: Eduardo W. V. Chaves (2014)

1 TENSORS

37

According to Problem 1.48 the following is true:

([cof( A)] ar ) (b cr ) ([cof( A)] b) (ar cr ) + ([cof( A)] cr ) (ar b) = Tr ([cof( A)])[rcr (ar b)]
r

where II A = Tr [cof( A )] is the second principal invariant of A , thus:

r r
= II A [c (a b)]

r
r
r
r
r r
r
r
r
r r r
a ( A b) ( A c ) + ( A a) b ( A c ) + ( A a) ( A b) c = II A a (b c )

NOTE 1: We can summarize that:

] [

] [
] (see Problem 1.48)
r
r
r
r
r r
r
r
r
r r r
a [( A b) ( A c )] + ( A a) [b ( A c )] + ( A a) [( A b) c ] = II [a (b c )]
r
r
r
r r r
( A a) [( A b) ( A c )] = III [a (b c )] (see Problem 1.49)
r
r r r r
r
r r
r
r r r
( A a) (b c) + a ( A b) c ) + a b ( A c ) = I A a (b c )

(1.42)
(1.43)

(1.44)

r r r

where I A = Tr (A ) , II A = Tr ([cof( A )]) , III A = det (A ) . Using the notation a (b c ) [a, b, c ] ,


the above equations can also be written as follows:
r r
r r r
r
r r
r
r r r
[( A a), b, c ] + [a, ( A b), c] + [a, b, ( A c )] = I A [a, b, c ]
r
r r
r
r
r r
r
r
r r r
[a, ( A b), ( A c )] + [( A a), b, ( A c )] + [( A a), ( A b), c ] = II A [a, b, c]
r
r
r
r r r
[( A a), ( A b), ( A c )] = III A [a, b, c ]
r

r r r

NOTE 2: If we consider three linearly independent vectors [a (b c )] [a, b, c ] 0 , and


three vectors such as:
r
r
r
r
f = 1a + 2 b + 3 c
r
r
r
r
g = 1 a + 2 b + 3 c
r
r
r
r
h = 1 a + 2 b + 3 c

r
f
r 1
g = 1
r
h 1

2 3 ar

2 3 b
r
2 3 c

(1.45)

And according to Cramers rule, (see Problem 1.16), the following relationships are true:
r r r
r r r
r r r
r r r
[a, f , c ]
[a, b, f ]
f (b c ) [ f , b, c ]
;
;
1 = r r r r r r
2 = r r r
3 = r r r
a (b c ) [a, b, c ]
[a, b, c ]
[a, b, c ]
r r r
r r r
r r r
[g, b, c ]
[a, g, c ]
[a, b, g]
;
;
1 = r r r
2 = r r r
3 = r r r
[a, b, c ]
[a, b, c ]
[a, b, c ]
r r r
r r r
r r r
[h, b, c ]
[a, h, c ]
[a, b, h]
;
;
1 = r r r
2 = r r r
3 = r r r
[a, b, c ]
[a, b, c ]
[a, b, c ]
r r r
r r r
By performing the triple scalar product [ f (g h)] [f , g, h] , we can obtain:

University of Castilla-La Mancha


Ciudad Real - Spain

Draft

By: Eduardo W. V. Chaves (2014)

SOLVING PROBLEMS BY MEANS OF CONTINUUM MECHANICS

38

r r r
[ f (g h)] =

1
1

2 3
r r r
2 3 [a, b, c ]
2 3

r r r
r r r
r r r
[ f , b, c ] [a, f , c ] [a, b, f ]
r r r
r r r r r r
r r r
r r r
1
, b, c ] [a, g, c ] [a, b, g] [a, b, c ] = P [a, b, c ]
r r r [g
r r r
r r r
[a, b, c ] r r r
[h, b, c ] [a, h, c ] [a, b, h]

where
r r r
r r r
r r r
[ f , b, c ] [a, f , c ] [a, b, f ]
1 2 3
r r r
r r r
r r r
1
P = 1 2 3 = r r r [g, b, c ] [a, g, c ] [a, b, g]
r r r
r r r
r r r
1 2 3 [a, b, c ] [h, b, c ] [a, h, c ] [a, b, h]

r
r r
r r
r
For the case when f = A a , g = A b , h = A c , the principal invariants of P are:

(1.46)

r r
r
r r r
r
r r
1
I P = Tr ( P ) = r r r [ A a, b, c] + [a, A b, c] + [a, b, A c ] = I A
[a, b, c]
r r
r
r
r r
r r r
r r
r
r r r
r
r r
[a
1
, A b, c ] [a, b, A b] [ A a, b, c] [a, b, A a] [ A a, b, c ] [a, A a, c ]

r r r
r r
II P = r r r
r
r r
r r
r +
r r r
r r
r +
r
([a, b, c ]) 2 [a, A c, c] [a, b, A c] [ A c, b, c ] [a, b, A c] [ A b, b, c] [a, A b, c]
= II A
III P = III A = det (A )

NOTE 3: Let us consider the Cartesian system where


r
a = a1e 1 + a 2 e 2 + a 3 e 3
r

b = b1e 1 + b 2 e 2 + b 3 e 3
r
c = c 1e 1 + c 2 e 2 + c 3 e 3
r
r
r
Also let us consider that f = e 1 , g = e 2 , h = e 3 ,

r
a a
r 1
b = b1
cr c
1

a2
b2
c2

a 3 e 1

b 3 e 2
c 3 e 3

so, taking into account the above equation

and the equation in (1.45) we can conclude that:


r
f
r 1
g = 1
r
h 1

2 3 ar

2 3 b
r
2 3 c

1 0 0 e 1 1
0 1 0 e =

2 1
0 0 1 e 3 1

2 3 a1 a 2 a 3 e 1

2 3 b1 b 2 b 3 e 2
2 3 c 1 c 2 c 3 e 3

thus
1

1
1

2 3 a 1 a 2 a 3 1 0 0
2 3 b1 b 2 b 3 = 0 1 0
2 3 c 1 c 2 c 3 0 0 1

1
1

2 3 a1 a 2 a 3
2 3 = b1 b 2 b 3
2 3 c 1 c 2 c 3

With that we can obtain the inverse of a tensor. Let us consider the tensor A where the
components are:
A11
A ij = A 21
A 31

University of Castilla-La Mancha


Ciudad Real - Spain

A12
A 22
A 32

A13 a1 a 2
A 23 = b1 b 2
A 33 c1 c 2

Draft

a3
b 3
c 3

r r r
A = [a, b, c ]

By: Eduardo W. V. Chaves (2014)

1 TENSORS

39

Then, the inverse P = A 1 , (see equation (1.46)), becomes:


A 1

r r r
r r r
r r r
[ f , b, c ] [a, f , c ] [a, b, f ]
r r r
r r r
r r r
1
= r r r [g, b, c ] [a, g, c ] [a, b, g]
r r r
r r r
[a, b, c ] r r r
[h, b, c ] [a, h, c ] [a, b, h]

b1

c1

0
1
b1
=
A
c1

0
b1

c
1

a1

a2

a3

b2

b3

c2

c3

c1

c2

c3

a1

a2

a3

b2
c2

b3
c3

0
c1

1
c2

0
c3

0
b2

1
b3

a1
0

a2
0

a3
1

c2

c3

c1

c2

c3

Taking into account that A 1 =

[cof( A )]ij

b
2
c2

b
= 1
c
1
b1

c1

b3
c3

a2

a3

c2

c3

a1

a3

c3

c1

c3

c2

a2

b3

c3

a2

a3

c2

c3

b3

a1

a3

c3

c1

c3

b2
c2

a1
c1

a2
c2

a3

b2 b3

a1 a 3

b1 b 3

a1 a 2

b1 b 2

a2

1
[cof( A )]T = 1 [adj( A )] , we can conclude that:
A
A

b3
b2

a 3
b1 b 2 b 3
b2

1 0
0

c2

a1 a 2 a 3

1 b1

b1 b 2 b 3 =

A c1

0 1
0
b1

a1 a 2 a 3
c 1

b1 b 2 b 3

0 0
1

a1

a1

a2

c1

c2

b
a3
2

b2 b3
c2

a
a1 a3
= 2

c
b1 b 3
2

a2
a1 a 2

b1 b 2
b 2

a2

b3
c3

b1 b 3
c1

c3

a3

a1

a3

c3

c1

c3

a3
b3

a1 a3
b1 b 3

b1 b 2

c1 c 2

a1 a 2

c1 c 2

a1 a 2

b1 b 2

Note that the coefficient of the above matrix, [cof(A )]ij , can be obtained by solving the
determinant of the resulting matrix by removing the i th row and the j th column, which result
we multiply by (1) i + j , for example:

[cof(A)]12 = (1)

1+ 2

a1 a 2
b1 b 2
c1

c2

a3
b b3
b3 = 1
c1 c 3
c3

Problem 1.58
Given the scalars I C , II C , III C in terms of the scalars I E , II E , III E :
I C = 2I E + 3

II C = 4 I E + 4 II E + 3

III C = 2 I E + 4 II E + 8 III E + 1

(1.47)

Obtain the reverse form of the above equations, i.e. obtain I E , II E , III E in terms of I C , II C ,
III C .
Solution:
University of Castilla-La Mancha
Ciudad Real - Spain

Draft

By: Eduardo W. V. Chaves (2014)

SOLVING PROBLEMS BY MEANS OF CONTINUUM MECHANICS

40

The equations in (1.47) can be restructured as follows:


I C 2 0 0 I E 3

II C = 4 4 0 II E + 3
III 2 4 8 III 1
E
C

2 0 0 I E I C 3
4 4 0 II = II 3

E C



2 4 8 III E III C 1

2 0 0 2 0 0 I E 2 0 0 I C 3

4 4 0 4 4 0 II E = 4 4 0 II C 3
2 4 8 2 4 8 III E 2 4 8 III C 1
1

I E 2 0 0 I C 3



II E = 4 4 0 II C 3
III 2 4 8 III 1
C

where

2 0 0
= 4 4 0
2 4 8

[cof( A )]T

1 0
=

64 4

8
0
8
0
0

4 0
2 8
2 0
2 8

2 0
4 0

4 4

1
2 4
2

1
2 0

=
2 4
2

1
2 0
8

4 4

1
8

0
1
4
1
8

with that the scalars I E , II E , III E can be obtained as follows:


1
1

I E 2 0 0 I C 3 2
1


II E = 4 4 0 II C 3 =
III 2 4 8 III 1 2
C
E
1
8

1.1.6

0
1
4
1
8

2 ( I C 3)
IC 3

1
0 II C 3 = ( 2 I C + II C + 3)


I
I
I
1

1 C
1

8 ( I C II C + III C 1)
8

Additive Decomposition of Tensors

Problem 1.59
Find a fourth-order tensor P so that P : A = A dev , where A is a second-order tensor.
Solution: Taking into account the additive decomposition into spherical and deviatoric parts,
we obtain:
A = A sph + A dev =

Tr ( A )
1 + A dev
3

A dev = A

Tr ( A )
1
3

By definition the fourth-order tensors are:


I = 11 = ik jl e i e j e k e l = I ijkl e i e j e k e l

(1.48)

I = 11 = il jk e i e j e k e l = I ijkl e i e j e k e l

(1.49)

I = 1 1 = ij kl e i e j e k e l = I ijkl e i e j e k e l

(1.50)

where it holds that:


University of Castilla-La Mancha
Ciudad Real - Spain

Draft

By: Eduardo W. V. Chaves (2014)

1 TENSORS

)(

I : A = ik jl e i e j e k e l : A pq e p e q
= ik jl A pq kp lq e i e j

= ik jl A kl e i e j
= A ij e i e j

)(

I : A = ij kl e i e j e k e l : A pq e p e q
= ij kl A pq kp lq e i e j
= ij kl A kl e i e j

= A kk ij e i e j
= Tr ( A )1

)
(1.51)

=A

41

)
(1.52)

Referring to the definition of fourth-order unit tensors seen in (1.51), and (1.52), where the
relations I : A = Tr ( A )1 and I : A = A hold, we can now state:
A dev = A

Tr ( A )
1
1
1

1 = I : A I : A = I I : A = I 1 1 : A
3
3
3
3

Therefore, we can conclude that:


1
P = I 1 1
3

The tensor P is known as a fourth-order projection tensor, Holzapfel(2000).

1.1.7

Transformation Law of Tensor Components. Invariants.

Problem 1.60
Under the base transformation e i = a ij e j and by considering that the second-order tensor
components in this new base are given by:
Tij = a ik a jl Tkl

Show that:
a) Tii = Tkk = Tr (T ) ; b) Tij T ji = Tkl Tlk ; c) det ( T ) = det ( T )
Solution:
=j
a) Tij = a ik a jl Tkl i
Tii = a ik a il Tkl = kl Tkl = Tkk = Tll

b) Tij T ji = (a ik a jl Tkl )(a jp a iq T pq ) = a ik a iq a jl a jp Tkl T pq = kq lp Tkl T pq = Tqp T pq = Tkl Tlk


123 123
= kq

= lp

with that we show that Tr ( T 2 ) = Tr ( T T ) = Tij T ji


c) det ( Tij ) = det(a ik a jl Tkl ) = det (a ik )det (a jl )det ( Tkl ) = det( Tkl )
1
424
31
424
3
=1

=1

we have just shown that Tkk = Tr ( T ) , Tkl Tlk = Tr ( T T ) and det ( T ) are invariants.

University of Castilla-La Mancha


Ciudad Real - Spain

Draft

By: Eduardo W. V. Chaves (2014)

SOLVING PROBLEMS BY MEANS OF CONTINUUM MECHANICS

42

Problem 1.61
Let T be a symmetric second-order tensor and I T , II T , III T be scalars, where:
I T = Tr ( T ) = Tii

II T =

1
2
I T Tr ( T 2 )
2

III T = det ( T )

Show that I T , II T , III T are invariant with a change of basis.


Solution:
a) Taking into account the transformation law for the second-order tensor components
Tij = a ik a jl Tkl or in matrix form T = A T A T . Then, Tii is:
Tii = a ik a il Tkl = kl Tkl = Tkk = I T

Hence we have proved that I T is independent of the adopted system.


b) To prove that II T is an invariant, one only needs to show that Tr ( T 2 ) is one also, since
2
I T is already an invariant.
Tr ( T 2 ) = Tr ( T T ) = T : T = Tij Tij

= ( a ik a jl Tkl )( a ip a jq T pq )
= a ik a ip a jl a jq Tkl T pq
123 123
kp

lq

= T pl T pl
= T : T = Tr ( T T ) = Tr ( T 2 )

c) Matrix form: det ( T ) = det (T ) = det (A T A T ) = det


(A )det ( T )det (A T ) = det ( T )
1
424
3
1424
3
=1

=1

Problem 1.62
Show that the following relations are invariants:
C12 + C 22 + C 32

C13 + C 23 + C 33

C14 + C 24 + C 34

where C1 , C 2 , C 3 are the eigenvalues of the second-order tensor C .


Solution: Any combination of invariants is also an invariant, so, on this basis, we can try to
express the above expressions in terms of their principal invariants.

I C2 = (C1 + C 2 + C3 ) = C12 + C 22 + C 32 + 2 C1 C 2 + C1 C3 + C 2 C3 C12 + C 22 + C 32 = I C2 2 II C


1444
424444
3
2

II C

So, we have proved that C12 + C 22 + C 32 is an invariant. Similarly, we can obtain the other
relationships, so, we summarize:
C1 + C 2 + C3 = I C
C12 + C 22 + C32 = I C2 2 II C
C13 + C 23 + C33 = I C3 3 II C I C + 3 III C
C14 + C 24 + C34 = I C4 4 II C I C2 + 4 III C I C + 2 II C2
C15 + C 25 + C35 = I C5 5 II C I C3 + 5 III C I C2 + 5 II C2 I C 5 III C II C

Note also that

C1n+1 + C 2n+1 + C3n +1 = C1n + C 2n + C3n I C C1 C 2n 1 + C3n1 C 2 C1n1 + C3n1 C3 C1n 1 + C 2n1

University of Castilla-La Mancha


Ciudad Real - Spain

Draft

By: Eduardo W. V. Chaves (2014)

1 TENSORS

43

Problem 1.63
Show that, if a symmetric second-order tensor T has three different real eigenvalues
( 1 2 3 ) , the principal space of T is formed by an orthonormal basis.
Solution:
Consider a symmetric second-order tensor T . By the definition of eigenvalues, given in
T n ( a ) = a n ( a ) , if 1 , 2 , 3 are the eigenvalues of T , then it follows that:
T n (1) = 1n (1)

T n ( 2 ) = 2n ( 2 )

T n (3) = 3n (3)

(1.53)

Applying the dot product between n ( 2) and T n (1) = 1n (1) , and the dot product between n (1)
and T n ( 2) = 2 n ( 2) we obtain:
n ( 2 ) T n (1) = 1n ( 2 ) n (1)

n (1) T n ( 2 ) = 2n (1) n ( 2)

(1.54)

Since T is symmetric, it holds that n ( 2) T n (1) = n (1) T n ( 2) , so:


1n ( 2) n (1) = 2 n (1) n ( 2 ) = 2 n ( 2 ) n (1)
(1 2 ) n

(1)

(2)

(1.55)

=0

(1.56)

To satisfy the equation (1.56), with 1 2 0 , the following must be true:


n (1) n ( 2 ) = 0

(1.57)

Similarly, it is possible to show that n (1) n (3) = 0 and n ( 2) n (3) = 0 and then we can conclude
that the eigenvectors are mutually orthogonal, and constitute an orthogonal basis, where the
transformation matrix between systems is:
n (1) n 1(1)


A = n ( 2) = n 1( 2)
n (3) n (3)

n (21)
n (22 )
n (23)

n 3(1)

n 3( 2 )
n (33)

(1.58)

NOTE: If the tensor is not symmetric the principal space not necessarily is orthogonal.
Problem 1.64
Obtain the components of T , given by the transformation:
T = A T AT

where the components of T and A are shown, respectively, as Tij and a ij . Afterwards, given
that a ij are the components of the transformation matrix, represent graphically the
components of the tensors T and T on both systems.
Solution: The expression T = A T A T in symbolic notation is given by:
(e a e b ) = ars (e r e s ) T pq (e p e q ) akl (e l e k ) = ars T pq akl sp ql (e r e k )
Tab
= arp T pq akq (e r e k )

To obtain the components of T one only need make the double scalar product with the basis
(e i e j ) , the result of which is:

University of Castilla-La Mancha


Ciudad Real - Spain

Draft

By: Eduardo W. V. Chaves (2014)

SOLVING PROBLEMS BY MEANS OF CONTINUUM MECHANICS

44

(e a e b ) : (e i e j ) = arp T pq akq (e r e k ) : (e i e j )
Tab
ai bj = arp T pq akq ri kj
Tab

Tij = aip T pq a jq

The above equation is shown in matrix notation as:


T = A T A T inverse
T = A 1 T A T

Since A is an orthogonal matrix, it holds that A T = A 1 . Thus, T = A T T A . The


graphical representation of the tensor components in both systems can be seen in Figure 1.1.

x3

T = A T AT

T33

T23

x3

T13

T33

T31
T13

T23

T32

T22

T12

x2

T21

T22

T31

T11

T11

T32

T21

T12

x1

x2

x1

T = AT T A

Figure 1.1: Transformation law of the second-order tensor components.


Problem 1.65
Let T be a second-order tensor whose components in the Cartesian system (x1 , x 2 , x3 ) are
given by:

(T )ij

3 1 0
= Tij = T = 1 3 0
0
0 1

Given that the transformation matrix between two systems, (x1 , x 2 , x3 ) - (x1 , x 2 , x 3 ) , is:

0
2
A=
2

2
University of Castilla-La Mancha
Ciudad Real - Spain

Draft

0
2
2
2
2

By: Eduardo W. V. Chaves (2014)

1 TENSORS

45

Obtain the tensor components Tij in the new coordinate system (x1 , x 2 , x 3 ) .
Solution: The transformation law for second-order tensor components is Tij = aik a jl Tkl .
To enable the previous calculation to be carried out in matrix form we use:

[ ]

Tij = [a i k ] [Tk l ] a l j

Thus
T = A T AT
0

2
T =
2

1
0

3 1 0
0 1 3 0 0

0 1
0
1
0

0
2
2
2
2

2
2
2
2
0

2
2
2

On carrying out the operation of the previous matrices we now have:


1 0 0
T = 0 2 0
0 0 4

Problem 1.66
Find the transformation matrix between the systems: x, y , z and x, y, z . These systems are
represented in Figure 1.2.
z = z
z = z

y
y = y

x
x

Figure 1.2: Rotation.


Solution: Note that: if we have an initial space and successive transformations until the final
space, the transformation law from the initial space to the final space is formed by the product
of the transformations in the opposite direction. That is, we place in the final space and we
follow opposite direction of the arrows until the initial space, (see figure below).

University of Castilla-La Mancha


Ciudad Real - Spain

Draft

By: Eduardo W. V. Chaves (2014)

SOLVING PROBLEMS BY MEANS OF CONTINUUM MECHANICS

46

B
r
a

r
a

B 1
r
a

A 1

initial
space

C 1
CBA

C
r
a

current
space

A 1B 1 C 1 = (CBA ) 1 iforthogonal
s A T B T C T = (CBA ) T

The coordinate system x, y, z can be obtained by different combinations of rotations as


follows:

Rotation along the z -axis


z = z

from x, y , z to x, y, z
cos sin 0
A = sin cos 0
0
0
1

University of Castilla-La Mancha


Ciudad Real - Spain

with 0 360

Draft

By: Eduardo W. V. Chaves (2014)

1 TENSORS

Rotation along the y -axis

from x, y, z to x, y, z

z = z

cos
B = 0
sin

y = y

47

0 sin
1
0
0 cos

with 0 180
y

z = z

Rotation along the z -axis


z = z
z = z

from x, y, z to x, y, z

y
y = y

cos
C = sin
0

sin
cos
0

0
0
1

with 0 360

x
x

The transformation matrix from ( x, y , z ) to ( x, y, z ) is given by


After multiplying the matrices, we obtain:

D = CBA

(sin cos cos + cos sin ) sin cos


(cos cos cos sin sin )

D = ( cos cos sin sin cos ) ( sin cos sin + cos cos ) sin sin

cos sin
sin sin
cos

The angles , , are known as Euler angles and were introduced by Leonhard Euler to
describe the orientation of a rigid body motion.

University of Castilla-La Mancha


Ciudad Real - Spain

Draft

By: Eduardo W. V. Chaves (2014)

SOLVING PROBLEMS BY MEANS OF CONTINUUM MECHANICS

48

Problem 1.67
If a ij represent the components of the base transformation matrix, show that the following
equations are fulfilled:
2
2
2
a11
+ a12
+ a13
=1
2
2
2
a 21 + a 22 + a 23 = 1
2
2
2
a 31 + a 32 + a 33 = 1

a11 a 21 + a12 a 22 + a13 a 23 = 0

a 21 a31 + a 22 a 32 + a 23 a 33 = 0
a a + a a + a a = 0
12 32
13 33
11 31

or

2
2
2
a11
+ a 21
+ a 31
=1
2
2
2
a12 + a 22 + a 32 = 1
2
2
2
a13 + a 23 + a 33 = 1

a11 a12 + a 21 a 22 + a 31 a 32 = 0

a12 a13 + a 22 a 23 + a 32 a 33 = 0
a a + a a + a a = 0
21 23
31 33
11 13

Solution:
We start from the principle that the basis transformation matrix is an orthogonal matrix, i.e.
a ik a jk = a ki a kj = ij . Then:

a ik a jk = a i1 a j1 + a i 2 a j 2 + a i 3 a j 3 = ij

(i = 1, j = 1)

(i = 2, j = 2)

(i = 3, j = 3)

(i = 1, j = 2)

(i = 2, j = 3)
(i = 1, j = 3)

2
2
2
a11
+ a12
+ a13
=1
2
2
2
a 21
+ a 22
+ a 23
=1
2
2
2
a31
+ a 32
+ a 33
=1

a11 a 21 + a12 a 22 + a13 a 23 = 0


a 21 a 31 + a 22 a 32 + a 23 a 33 = 0
a11 a 31 + a12 a 32 + a13 a 33 = 0

Alternative solution:
T

AA = 1

a11
a
21
a 31

a12
a 22
a 32

a13 a11
a 23 a12
a 33 a13

a 21
a 22
a 23

a 31 1 0 0
a 32 = 0 1 0
a 33 0 0 1

Performing the matrix multiplication we obtain:


2
2
2

+ a12
+ a13
a11

a11 a 21 + a12 a 22 + a13 a 23


a a + a a + a a
12 32
13 33
11 31

1.1.8

a11 a 21 + a12 a 22 + a13 a 23


2
2
2
+ a 22
+ a 23
a 21
a 21 a 31 + a 22 a 32 + a 23 a 33

a11 a 31 + a12 a 32 + a13 a 33 1 0 0

a 21 a 31 + a 22 a 32 + a 23 a 33 = 0 1 0
2
2
2
0 0 1
+ a 32
+ a 33
a 31

Eigenvalues, Eigenvectors, Orthogonal Transformation

Problem 1.68
Let Q be a proper orthogonal tensor, and E be an arbitrary second-order tensor. Show that
the eigenvalues of E do not change with the following orthogonal transformation:
E* = Q E QT

Solution:

We can prove this as follows:

University of Castilla-La Mancha


Ciudad Real - Spain

Draft

By: Eduardo W. V. Chaves (2014)

1 TENSORS

(
)
= det (Q E Q 1)
= det (Q E Q Q 1 Q )
= det [Q (E 1 ) Q ]
(3) det (E 1) det
(Q3)
= det
12Q
1
424

0 = det E * 1

T
T

= det (E 1 )

49

(
= det (Q
= det (Q

0 = det E *ij ij

ik E kp Q jp

ij

ik E kp Q jp

Q ik Q jp kp

[ (

) ]
= det (Q )det (E ) det (Q )
= det (E )
= det Q ik E kp kp Q jp
ik

kp

kp

kp

jp

kp

Thus, we have proved that E and E * have the same eigenvalues.


Problem 1.69
Let A be a second-order tensor and Q be an orthogonal tensor. If the orthogonal
transformation law to A is given by A * = Q A Q T , show that A 2 = Q A 2 Q T .
*

Solution:
A 2 = A* A*

( A 2 ) ij = ( A * A * ) ij = A *ik A *kj

= ( Q A Q T ) (Q A Q T )
= QA Q QA Q
123
T

= (Q ip A pr Q kr )(Q ks A st Q jt )

= Q ip A pr Q kr Q ks A st Q jt
123

=1

= rs

= Q A A QT

= Q ip A pr rs A st Q jt = Q ip A ps A st Q jt

= Q A 2 QT

= Q ip ( A A ) pt Q jt
= (Q A 2 Q T ) ij

Problem 1.70
Given the tensor components:
5 3 3
Tij = 2 6 3
2 2 4

a) Obtain the principal invariants of T , i.e. obtain I T , II T and III T ;


b) Obtain the characteristic polynomial associated with T ;
c) If 1 , 2 and 3 are the eigenvalues of T and 1 = 10 . Obtain 2 and 3 > 2 .
Solution:
a) The principal invariants of T are:
I T = Tr ( T ) = 5 + 6 + 4 = 15

II T =

6 3
2 4

5 3
2 4

5 3
2 6

= 56

III T = det ( T ) = 60

b) The characteristic polynomial can be obtained by solving the determinant:


University of Castilla-La Mancha
Ciudad Real - Spain

Draft

By: Eduardo W. V. Chaves (2014)

SOLVING PROBLEMS BY MEANS OF CONTINUUM MECHANICS

50

5
3
2
6
2

3
3

=0

3 2 I T + II T III T = 0

thus:
3 152 + 56 60 = 0

c) In the principal space the following is true:


1 = 10 0
2
Tij = 0
0
0

3 > 2
0
0

where the principal invariants are


I T = Tr ( T ) = 1 + 2 + 3 = 15
III T = det ( T ) = 1 2 3 = 60

2 + 3 = 5
23 = 6

By combining these two equations we obtain:


(1)

3 = 3
2
(5 3 ) 3 = 6 3 5 3 + 6 = 0 ( 2 )
2 + 3 = 5
3 = 2

23 = 6

We discard the solution (32) = 2 , thus 3 = 3 . Then:


10 0 0
Tij = 0 2 0
0 0 3
I T = Tr ( T ) = 10 + 2 + 3 = 15

In this space we can check that II T = 2 3 + 10 3 + 10 2 = 56


III = det ( T ) = 10 2 3 = 60
T

Problem 1.71
Find the principal values and directions of the second-order tensor T , where the Cartesian
components of T are:

(T )ij

3 1 0
= Tij = T = 1 3 0
0
0 1

Solution: We need to find nontrivial solutions for (Tij ij ) n j = 0 i , which are constrained by
n j n j = 1 (unit vector). As we have seen, the nontrivial solution requires that:
Tij ij = 0

Explicitly, the above equation is:

University of Castilla-La Mancha


Ciudad Real - Spain

Draft

By: Eduardo W. V. Chaves (2014)

1 TENSORS

T11
T21

T12
T22

T13
T23

T31

T32

T33

51

3 1
= 1 3
0

0
0

=0

Developing the above determinant, we can obtain the cubic equation:

(1 ) (3 ) 2 1 = 0
3 7 2 + 14 8 = 0

We could have obtained the characteristic equation directly in terms of invariants:


I T = Tr ( Tij ) = Tii = T11 + T22 + T33 = 7
II T =

T
1
Tii T jj Tij Tij = 22
T32
2

T23
T33

T11

T13

T31

T33

T11

T12

T21

T22

= 14

III T = Tij = ijk Ti1 T j 2 Tk 3 = 8

Then, the characteristic equation becomes:


3 2 I T + II T III T = 0

3 72 + 14 8 = 0

On solving the cubic equation we obtain three real roots, namely:


1 = 1;

2 = 2;

3 = 4

We can also verify that:


I T = 1 + 2 + 3 = 1 + 2 + 4 = 7

II T = 1 2 + 2 3 + 3 1 = 1 2 + 2 4 + 4 1 = 14
III T = 1 2 3 = 8

Thus, we can see that the invariants are the same as those evaluated previously.
Principal directions:
Each eigenvalue, i , is associated with a corresponding eigenvector, n (i ) . We can use the
equation ( Tij ij ) n j = 0 i to obtain the principal directions.
1 = 1
3 1
1

1
3 1

0 n1 0
0 n 1 3 1 1

0 n 2 = 1 3 1 0 n 2 = 0
0 1 1 n 3 0
1 1 n 3 0

These become the following system of equations:


2n1 n 2 = 0

n1 = n 2 = 0
n1 + 2n 2 = 0
0n = 0
3
n i n i = n12 + n 22 + n 32 = 1

Then we can conclude that: 1 = 1

n i(1) = [0 0 1] .

NOTE 1: This solution could have been directly determined by the specific features of the
T matrix. As the terms T13 = T23 = T31 = T32 = 0 imply that T33 = 1 is already a principal
value, then, consequently, the original direction is a principal direction.

University of Castilla-La Mancha


Ciudad Real - Spain

Draft

By: Eduardo W. V. Chaves (2014)

SOLVING PROBLEMS BY MEANS OF CONTINUUM MECHANICS

52

2 = 2

3 2
1

1
3 2
0

0 n1 0
n 1 3 2 1
n = 1 3 2
0 n 2 = 0
2
0
1 2 n 3 0
1 2 n 3 0
0
0

n1 n 2 = 0 n1 = n 2

n1 + n 2 = 0
n = 0
3

The first two equations are linearly dependent, after which we need an additional equation:
n i n i = n12 + n 22 + n 32 = 1 2n12 = 1 n1 =

1
2

Thus:
2 = 2

1
n i( 2 ) =
2

1
2

3 = 4
3 3
1

1
3 3
0

0 n 1 3 4 1
0 n1 0

0 n 2 = 1 3 4
0 n 2 = 0
1 3 n 3 0
0
1 4 n 3 0

n1 n 2 = 0

n1 = n 2
n1 n 2 = 0
3n = 0
3

n i n i = n12 + n 22 + n 32 = 1 2n 22 = 1 n 2 =

1
2

Then:
3 = 4

n i(3) = m

1
2

1
2

Afterwards, we summarize the eigenvalues and eigenvectors of T :


1 = 1

n i(1)

= [0 0 1]

2 = 2

n i( 2)

1
2

1
2

3 = 4

n i(3)

= m

1
2

1
2

NOTE 2: The tensor components of this problem are the same as those used in Problem
1.65. Additionally, we can verify that the eigenvectors make up the transformation matrix, A ,
between the original system, (x1 , x 2 , x3 ) , and the principal space, (x1 , x 2 , x 3 ) , (see Problem
1.65).

University of Castilla-La Mancha


Ciudad Real - Spain

Draft

By: Eduardo W. V. Chaves (2014)

1 TENSORS

53

Problem 1.72
Let Q be a proper orthogonal tensor a) show that Q has one real eigenvalue and equals to 1 .
b) Also show that Q can be represented by means of the angle as follows:
Q = p p + cos (q q + r r ) sin (q r r q )

where p , q , r , are unit vectors which form an orthonormal basis, where p is the direction
associated with the eigenvalue = 1 , i.e. p is an eigenvector of Q . c) Obtain the principal
r
invariants of Q in function of the angle . d) Given a vector position x , obtain the new
r
vector originated by the orthogonal transformation Q x in the space formed by p , q .
Solution:
a) Taking into account the definition of the orthogonal tensor we can conclude that:
QT Q = 1

QT Q QT = 1 QT

QT (Q 1) = (QT 1)

QT (Q 1) = (Q 1)T

Then we obtain the determinant of the two previous tensors:

det QT (Q 1) = det (Q 1)T = (1)3 det (Q 1)T

[ ]

[
]
[
]
det
1
42Q
4
3 det (Q 1) = det (Q 1) = det (Q 1)
T

det[(Q 1)] = det[(Q 1)]

= detQ =1

where

[ ]

det A T

we

have used

the

following

determinant

properties:

det[A ] = 3 det[A ] ,

= det[A ] , det[A B ] = det[A ]det[B ] . The unique scalar which satisfies the expression

above is zero, then:


det[(Q 1)] = 0

Taking into account the definition of eigenvalue, det[(Q 1)] = 0 , we conclude that when
= 1 it fulfills that det[(Q 1)] = 0 , then = 1 is eigenvalue of Q . Hence, there is a direction
(eigenvector) satisfying that Q e 1* = e 1* = e 1* .
b) We consider that the vectors p e 1* , q e *2 , r e *3 form an orthonormal basis.

e 3
q e *2

e 1* p

Q e 1* = e 1*

e 2
e 1

r e *3

The symbolic representation of the tensor in the basis e 1* , e *2 , e *3 is given by:

University of Castilla-La Mancha


Ciudad Real - Spain

Draft

By: Eduardo W. V. Chaves (2014)

SOLVING PROBLEMS BY MEANS OF CONTINUUM MECHANICS

54

Q = Q *ij e *i e *j
* *
* *
* *
= Q11
e1 e 1* + Q12
e1 e *2 + Q13
e1 e *3 +

(1.59)

+ Q *21e *2 e 1* + Q *22 e *2 e *2 + Q *23 e *2 e *3 +


+ Q *31e *3 e 1* + Q *32 e *3 e *2 + Q *33 e *3 e *3

Taking into account that e 1* is eigenvector of Q associated with the eigenvalue = 1 , it holds
that Q e 1* = e 1* = e 1* . In addition, making the projection of Q , given by (1.59), according to
direction e 1* , we obtain:
Q e 1* = e 1*

* *
* *
* *
Q e 1* = [ Q11
e1 e 1* + Q12
e1 e *2 + Q13
e1 e *3 +

+ Q *21e *2 e 1* + Q *22 e *2 e *2 + Q *23 e *2 e *3 +


+ Q *31e *3 e 1* + Q *32 e *3 e *2 + Q *33 e *3 e *3

] e 1*

* *
= Q11
e1 + Q *21e *2 + Q *31e *3
*
with that we conclude that Q11
= 1 , Q *21 = 0 , Q *31 = 0 .

Remember that two coaxial tensors have the same principal directions (eigenvectors). A tensor
and its inverse are coaxial tensors, then if Q 1 = Q T , this implies that Q T and Q are coaxial
tensors, and e 1* is also principal direction of Q T , then it fulfills that:
Q T e 1* = e 1*

* *
Q T e 1* = [ Q11
e1 e 1* + Q *21e 1* e *2 + Q *31e 1* e *3 +
* *
+ Q12
e 2 e 1* + Q *22 e *2 e *2 + Q *32 e *2 e *3 +
* *
+ Q13
e 3 e 1* + Q *23 e *3 e *2 + Q *33 e *3 e *3

] e 1*

* *
* *
* *
= Q11
e1 + Q12
e 2 + Q13
e3
*
*
*
with that we conclude that Q11
= 1 , Q12
= 0 , Q13
= 0 . Then, the equation (1.59) becomes:

Q = e 1* e 1* + Q *22 e *2 e *2 + Q *23 e *2 e *3 + Q *32 e *3 e *2 + Q *33 e *3 e *3

(1.60)

In matrix form, the components of Q , in the basis e *i , are given by:


x 2*

Q *ij

1 0
= 0 Q *22
0 Q *32

Q *22
*
Q11
=1

0
Q *23
Q *33

Q *32
Q *23

x1*

Q *33

x3*

University of Castilla-La Mancha


Ciudad Real - Spain

Draft

By: Eduardo W. V. Chaves (2014)

1 TENSORS

55

Once more we use the orthogonality condition Q T Q = Q Q T = 1 , or in terms of the


components in the space e *i :
Q *ki Q *kj
1
0
0

= ij

[(Q )
[Q Q

* 2
22
*
*
22 23

1 0
0 Q *
22

0 Q *23

0 1 0
Q *32 0 Q *22
Q *33 0 Q *32
0

] [Q Q
] [(Q )

+ (Q *32 ) 2
+ Q *32 Q *33

*
*
22 23
* 2
33

0 1 0 0
Q *23 = 0 1 0
Q *33 0 0 1
1 0 0
= 0 1 0

0 0 1

(1.61)

]
]

+ Q *32 Q *33
+ (Q *23 ) 2

The determinant of a proper orthogonal tensor is det (Q) = +1 , thus


1 0
0 Q *
22

0 Q *32

0
Q *23 = 1
Q *33

Q *22 Q *33 Q *23 Q *32 = 1

(1.62)

Taking into account (1.61) and (1.62) we obtain the following set of equations:
(Q *22 ) 2 + (Q *32 ) 2 = 1
* *
*
*
Q 22 Q 23 + Q 32 Q 33 = 0
* 2
* 2
(Q 33 ) + (Q 23 ) = 1
* *
*
*
Q 22 Q 33 Q 23 Q 32 = 1

cos 2 + sin 2 = 1

cos ( sin ) + sin cos = 0


2
2
cos + sin = 1
cos cos ( sin )(sin ) = 1

whereupon we have demonstrated the existence of an angle that meets the above conditions:
Q *ij

1 0
= 0 Q *22
0 Q *23

0 1
0
0

*
Q 32 = 0 cos sin
Q *33 0 sin cos

(1.63)

Returning to the equation (1.60), and taking into account (1.63), we conclude that:
Q = e 1* e 1* + (cos ) e *2 e *2 + ( sin )e *2 e *3 + (sin )e *3 e *2 + (cos ) e *3 e *3

= e 1* e 1* + cos e *2 e *2 + e *3 e *3 sin e *2 e *3 e *3 e *2

Considering that p e 1* , q e *2 , r e *3 , we show that:


Q = p p + cos (q q + r r ) sin (q r r q )

It is interesting to note that the additive decomposition of Q in an antisymmetric and a


symmetric part, in the space e *i , is:
0
0
1

0
= 0 cos
0
0
cos
1444
4244443

sym
Q *ij

[p p +cos (q q +r r )]ij

0
0
0

0
= 0
sin
0 sin
0
14444
4244444
3

skew
Q *ij

[sin (q r r q ) ]ij

skew

Note that the format of Q *ij


has the same format as the antisymmetric tensor ( W ) in the
space defined by the axial vector:

University of Castilla-La Mancha


Ciudad Real - Spain

Draft

By: Eduardo W. V. Chaves (2014)

SOLVING PROBLEMS BY MEANS OF CONTINUUM MECHANICS

56

Wij*

0 0
= 0 0
0

0

0

where is the magnitude of the axial vector.


c) By means of (1.63) it is easy to show that I Q = II Q = 1 + 2 cos , III Q = 1 .
r

d) We represent the vector x by means its components and the basis p , q , r , as follows:
r
x = pp + qq + rr .
r

Then, it fulfills that: x p = ( pp + qq + rr ) p = p

r
x q = q

r
x r = r

Thus, (see Figure 1.3), it holds that:

r
~ = Q xr = p p + cos (q q + r r ) sin (q r r q )
x
= pp + (q cos r sin )q + (r cos + q sin )r

] [pp + qq + rr ]

e 1* p
q e *2

r
x

r
r
~
x =Q x

r e *3

Figure 1.3

University of Castilla-La Mancha


Ciudad Real - Spain

Draft

By: Eduardo W. V. Chaves (2014)

1 TENSORS

Problem 1.73

57

Let us consider the tensorial transformations p = U p and p = R p , where R is an


orthogonal tensor and U is a second-order tensor with U U 1 = 1 , i.e. U 1 . Obtain the
r
r
transformation law between p and p .
Solution:
The proposed problem can be represented by the following figure:
U

r
p

r
p

r
p

?
Taking into account that R 1 = R T (orthogonal tensor), we can guarantee that the inverse of
r
r
R exists, and considering that p = R p we obtain:
r
r
r
r
r
r r
p = R p R 1 p = R 1 R p R 1 p = 1 p = p
r
r
r
r
Substituting p = R 1 p into p = U p , we obtain:
r
r
r
r
p = U p
p = U p
r
r
r
r
R 1 p = U p
R 1 p = U p
r
r
r
r
R R 1 p = R U p
U 1 R 1 p = U 1 U p
r
r
r
r r
1 p = R U p
(R U) 1 p = 1 p = p
r
r
r
r
p = (R U) p
p = (R U) 1 p

(1.64)

Or in indicial notation:
p i = U ij p j

p i = U ij p j

R ij1p j = U ij p j

R ij1p j = U ij p j

R ki R ij1p j = R ki U ij p j

U ki1R ij1p j = U ki1U ij p j

kj p j = R ki U ij p j

(R ki U ij ) 1 p j = kj p j = p k

p k = (R ki U ij )p j

p k = (R ki U ij ) 1 p j

University of Castilla-La Mancha


Ciudad Real - Spain

Draft

(1.65)

By: Eduardo W. V. Chaves (2014)

SOLVING PROBLEMS BY MEANS OF CONTINUUM MECHANICS

58

r
p

U 1

R 1 = R T

r
p

r
p

(R U)

(R U) 1 = U 1 R T

1.1.9

Spectral Representation of Tensors

Problem 1.74
Let w be an antisymmetric second-order tensor and V be a positive definite symmetric
tensor whose spectral representation is given by:
V=

n ( a ) n ( a )

a =1

Show that the antisymmetric tensor

w can be represented by:


3

w = w ab n (a ) n (b)
a ,b =1
a b

Demonstrate also that:


3

w V V w = w ab ( b a ) n ( a ) n (b)
a ,b =1
a b

Solution:
It is true that

a =1

a =1

a ,b =1

w 1 = w n ( a) n ( a) = w n ( a) n ( a ) = (wr n (a ) ) n ( a) = wb (n (b) n ( a ) ) n (a )
3

a =1

r
r
where we have applied the antisymmetric tensor property w n = w n , where w is the axial

vector associated with

w . Expanding the above equation, we obtain:

w = wb (n (b) n (1) ) n (1) + wb (n (b) n ( 2) ) n ( 2) + wb (n (b) n (3) ) n (3) =

(
+ w (n
+ w (n

= w1 n (1) n (1) n (1) + w2 n ( 2) n (1) n (1) + w3 n (3) n (1) n (1) +


1

)
) n

(
(n

)
) n

(
(n

)
) n

(1)

n ( 2 ) n ( 2 ) + w2 n ( 2 ) n ( 2) n ( 2) + w3 n (3) n ( 2) n ( 2 ) +

(1)

( 3)

( 3)

+ w2

( 2)

( 3)

( 3)

+ w3

( 3)

( 3)

( 3)

On simplifying the above expression we obtain:

University of Castilla-La Mancha


Ciudad Real - Spain

Draft

By: Eduardo W. V. Chaves (2014)

1 TENSORS

59

w = w2 (n (3) ) n (1) + w3 (n ( 2) ) n (1) + + w1 (n (3) ) n ( 2) w3 (n (1) ) n ( 2) +

( )

( )

w1 n ( 2 ) n (3) + w2 n (1) n (3)

Taking into account that w1 = w 23 = w 32 , w2 = w13 = w 31 , w3 = w12 = w 21 , the above


equation becomes:

w = w31 n (3) n (1) + w21 n ( 2) n (1) + +w32 n (3) n (2) + w12 n (1) n ( 2) +
+ w23 n ( 2) n (3) + w13 n (1) n (3)
which is the same as:
3

w = w ab n (a ) n (b)
a ,b =1
a b

The terms

wV

and V w can be expressed as follows:

3
3

(
)
(
)
a
b
w V = w ab n n b n (b) n (b)
b =1
a ,b =1

ab

w
b

ab

n ( a ) n (b ) n (b ) n (b ) =

a ,b =1
a b

w
b

ab

n ( a ) n (b )

a ,b =1
ab

and

3
3
a
a
a
b
(
)
(
)
(
)
(
)
V w = a n n
w ab n n = a w ab n ( a) n (b)
a ,b =1
a =1
a ,b =1
ab
ab

Then,

3
3
3

(
)
(
)
(
)
(
)
a
b
a
b
w V V w = bwab n n awab n n = wab (b a ) n (a ) n (b)
a ,b =1
a ,b =1
a ,b =1
ab
a b
a b

Similarly, it is possible to show that:


3

w V 2 V 2 w = w ab (2b 2a ) n ( a ) n (b)
a ,b =1
a b

Problem 1.75
Let C be a positive definite tensor, whose Cartesian components are given by:
2 0 1
C ij = 0 4 0
1 0 2

Obtain the following tensors: a) C 2 ; b) U = C . c) Check if the tensors C and U are coaxial.
Solution:

University of Castilla-La Mancha


Ciudad Real - Spain

Draft

By: Eduardo W. V. Chaves (2014)

SOLVING PROBLEMS BY MEANS OF CONTINUUM MECHANICS

60

Note that the tensors C 2 and U = C are coaxial with the tensor C . By means of the
spectral representation of C :
C=

N
a

(a)

(a )
N

a =1

( a ) are the eigenvectors of C , we can obtain:


where a are the eigenvalues of C , and N
C2 =

(a) N
(a)
2a N

U= C =

a =1

(a) N
(a )
aN

a =1

Calculation of the eigenvalues and eigenvectors of the tensor C .


Due to the structure of the C tensor components we already know one eigenvalue 2 = 4

which is associated with the principal direction N i( 2) = [0 1 0] . To calculate the remaining


eigenvalues is sufficient to solve the following characteristic determinant:
2
1
=0
1
2

( 2 ) 2 = 12

1 = 2 1 = 1
( 2 ) = 1
3 = 2 + 1 = 3

Associated with the eigenvalue 1 = 1 we have the following eigenvector:


1 N 1(1) 0

=
2 1 N 3(1) 0

2 1
1

1 1 N 1(1) 0
1 1 (1) = 0

N 3

N 1(1) = N 3(1)

with the restriction N i(1) N i(1) = 1 , thus


(1)N (1) + N
(1)N (1) + N
(1)N (1) = 1
N
1
1
2
2
3
3
(1) + N (1)N (1) = 1
N 1(1)N
1
1
1

(1) = 1
N
1
2

1
N 3(1) = N 1(1) = m
2

Associated with the eigenvalue 3 = 3 we have the following eigenvector:


2 3
1

N 1(3) 0

=
2 3 N (33) 0
1

(3) 0
1 1 N
1
1 1 (3) = 0

N 3

N 1(3) = N 3(3)

with the restriction N i(3) N i(3) = 1 , thus


(3)N
(3) + N (3)N (3) + N (3)N
( 3) = 1
N
1
1
2
2
3
3
( 3) = 1
N 1(3)N 1(3) + N 1(3)N
1

( 3) = 1
N
1
2

( 3) = 1
N 3(3) = N
1
2

Summarizing we have:
1 = 1
2 = 4
3 = 3

(1) =
N
i

N ( 2 ) = [0
i
( 3) =
N
i

2
2

1 0]

1
1
0

2
2

0 m

Matrix
Transforma
tion

A=

1
2
0
1
2

0
1
0

2
0
1
2

Then it holds that:


C = A C AT C = AT C A

University of Castilla-La Mancha


Ciudad Real - Spain

Draft

By: Eduardo W. V. Chaves (2014)

1 TENSORS

61

In the principal space we have:

1 0 0
C ij = 0 4 0
0 0 3

1
2
C ij = 0

U = C
ij
ij

0
16 0
0 9
1 0

4
= 0
0
0

0 1 0

0 = 0 2
3 0 0

0
0
3

Note that the tensor C is a positive definite tensor, so, its eigenvalues are positive. In the
original space we have the following components:

2
C ij =

1
2
0
1
2

0
1
0

2
0
1
2

1 0 0
0 16 0

0 0 9

1
2
0
1
2

0
1
0

2 5 0 4
0 = 0 16 0
1
4 0 5
2

(1.66)

Note that this result could have been obtained easily by means of the operation C 2 = C C ,
which in components becomes:
C ij2

2 0 1 2 0 1 5 0 4
= C ik C kj = 0 4 0 0 4 0 = 0 16 0
1 0 2 1 0 2 4 0 5

Similarly to (1.66), we obtain the components of U in the original Cartesian system:

U ij =

1
2
0
1
2

1
0

2
1
0
1
0
2

1 0
0 2

0 0

0
0

1
2
0
1
2

0
1
0

1 3 +1
0

2 2
2
0 = 0
1 3 1
0
2 2

3 1

2
0
3 + 1
2

c) The tensors C and U are coaxial, because the eigenvalues of U were obtained in the
principal space of C . We can also verify that C and U are coaxial by means of C U = U C ,
i.e.:
3 +1
0
2 0 1
2

2
C ik U kj = 0 4 0 0

3
1

1 0 2
0
2

3 +1
0

2
2
U ik C kj = 0
3 1 0
2

3 1
2 0 1 3.098 0 2.098
2
0 0 4 0 = 0
8
0
3 + 1 1 0 2 2.098 0 3.098

University of Castilla-La Mancha


Ciudad Real - Spain

3 1
3.098 0 2.098
2
0 = 0
8
0
3 + 1 2.098 0 3.098

Draft

By: Eduardo W. V. Chaves (2014)

SOLVING PROBLEMS BY MEANS OF CONTINUUM MECHANICS

62

Problem 1.76
Let C be a symmetric second-order tensor and R a proper orthogonal tensor. The
components of these tensors, in the Cartesian system, are given by:
2 0 1
C ij = 0 4 0
1 0 2

R ij =

0
2
2
2
2

0
2
2
2
2

a) Obtain the following tensors: a.1) C 8 ; a2) U = C .


b) Obtain the principal invariants of C .
c) Taking into account that the tensors b and C are related to each other by the following
proper orthogonal transformation C = R T b R , obtain the third principal invariant of b .
Solution:
0
3280
3281

65536
0
a) Similarly to Problem 1.75. Answer: C = 0
3280
0
3281
8

b)
I C = Tr (C ij ) = C ii = C11 + C 22 + C 33 = 8
II C =

4 0 2 1 2 0
1
C ii C jj C ij C ij =
+
+
= 19 ; III C = C = ijk C i1C j 2 C k 3 = 12
0 2 1 2 0 4
2

c) Taking into account the determinant property, the third principal invariant of b can be
expressed as follows:
C det (C ) = det (R T b R ) = det (R T )det (b)det (R ) = det (b) = III b = 12
1
424
3
123
= +1

= +1

Problem 1.77
Let S be a symmetric second-order tensor with det (S ) 0 . Considering that S has two
equal eigenvalues, i.e. S 2 = S 3 and S1 S 2 , show that S can be represented by:
S = S 1n (1) n (1) + S 2 (1 n (1) n (1) )

where n (1) is the eigenvector of S associated with the eigenvalue S1 , 1 is the second-order
unit tensor.
Solution: We start from the spectral representation of S :
S=

S n
a

(a)

n ( a ) = S 1n (1) n (1) + S 2 n ( 2) n ( 2) + S 3n (3) n (3)

a =1

= S 1n

(1)

(1)

+ S 2 (n

University of Castilla-La Mancha


Ciudad Real - Spain

( 2)

( 2)

+ n

Draft

( 3)

(1.67)

n )
( 3)

By: Eduardo W. V. Chaves (2014)

1 TENSORS

63

Remember that 1 is a spherical tensor, whereby any direction is a principal direction. Based
on this principle, we adopt the principal space of S to make the spectral representation of 1 ,
i.e.:
1=

(a)

n ( a ) = n (1) n (1) + n ( 2 ) n ( 2 ) + n (3) n (3)

(1.68)

a =1

n ( 2 ) n ( 2 ) + n (3) n (3) = 1 n (1) n (1)

By substituting the above equation into (1.67) we obtain:


S = S1n (1) n (1) + S 2 (n ( 2 ) n ( 2) + n (3) n (3) ) = S1n (1) n (1) + S 2 (1 n (1) n (1) )

1.1.10 Cayley-Hamilton Theorem


Problem 1.78
Let T be an arbitrary second-order tensor, show the Cayley-Hamilton theorem, which states
that any tensor satisfies its own characteristic equation.
Solution:
We start from the characteristic equation of the tensor: 3 2 I T + II T III T = 0 , which
fulfils for each eigenvalue 1 , 2 , 3 , then:
31 21 I T + 1 II T III T = 0
32 22 I T + 2 II T III T = 0
33 23 I T + 3 II T III T = 0

Restructuring the above equations in matrix form we obtain:


31

0
0

0
32
0

0 21

00
33 0

0
22
0

1
0

0 I T + 0
0
23

0
2
0

0
1 0 0
0 0 0

0 II T 0 1 0 III T = 0 0 0
0 0 1
0 0 0
3

(1.69)

Tij 3 Tij 2 I T + Tij II T III T ij = 0 ij

Note that in the principal space of T the following relationships are true:
1

Tij = 0
0

0
2
0

0
3

Tij 2 = Tik Tkj = 0


0

0
2
0

T pj = 0
Tij 3 = Tik Tkp
0

0 1

0 0
3 0
0
2
0

University of Castilla-La Mancha


Ciudad Real - Spain

0
2
0

0 1

0 0
3 0

0 21

0 =0
3 0
0
2
0

0 1

0 0
3 0

Draft

0
22
0
0
2
0

0
23
31

=0
3 0
0
0

0
32
0

0
33

By: Eduardo W. V. Chaves (2014)

SOLVING PROBLEMS BY MEANS OF CONTINUUM MECHANICS

64

The component transformation law between spaces for a second-order tensor is


Tij = Fik Tkp F pj1 , where Fij is the transformation matrix from the original space ( Tij ) to the
principal space ( Tij ). Note also that the relationships Tij 2 = Fik Tkp2 F pj1 and Tij 3 = Fik Tkp3 F pj1
hold, (see Problem 1.69). With that we can conclude that the equation in (1.69) can be
rewritten as follows:
Tij 3 Tij 2 I T + Tij II T III T ij = 0 ij
3
Fik Tkp
F pj1 Fik Tkp2 F pj1 I T + Fik Tkp F pj1 II T III T Fik kp F pj1 = 0 ij

Fik Tkp3 Tkp2 I T + Tkp II T III T kp F pj1 = 0 ij

3
Fsi1 Fik Tkp
Tkp2 I T + Tkp II T III T kp F pj1 F jt = Fsi1 0 ij F jt = 0 st

sk Tkp3 Tkp2 I T + Tkp II T III T kp pt = Fsi1 0 ij F jt = 0 st


Tst3 Tst2 I T + Tst II T III T st = 0 st
T 3 T 2 I T + T II T III T 1 = 0

Alternative solution:
In Problem 1.57 (NOTE 1) we have summarized that:
r r
r r r
r
r r
r
r r r
[( A a), b, c] + [a, ( A b), c ] + [a, b, ( A c )] = I A [a, b, c]
r
r r
r
r
r r
r
r
r r r
[a, ( A b), ( A c )] + [( A a), b, ( A c )] + [( A a), ( A b), c ] = II A [a, b, c]
r
r
r
r r r
[( A a), ( A b), ( A c )] = III A [a, b, c ]
r r r

where [a, b, c ] a (b c ) 0 holds with a 0 , b 0 , c 0 . Now if we consider that the


r

vector a is given by a = A f we can get:


r r
r r r
r
r r
r
r r r
[( A a), b, c] + [a, ( A b), c ] + [a, b, ( A c)] = I A [a, b, c ]
r r r
r
r r
r r
r r r
r
[( A A f ), b, c ] + [( A f ), ( A b), c ] + [( A f ), b, ( A c )] = I A [( A f ), b, c ]
r r r
r
r r
r r
r r r
r
[( A 2 f ), b, c ] + [( A f ), ( A b), c] + [( A f ), b, ( A c)] = I A [( A f ), b, c ]
r r r
r r r
r
r r
r r
r
[( A 2 f ), b, c ] I A [( A f ), b, c ] = [( A f ), ( A b), c ] [( A f ), b, ( A c)]

(1.70)

According to the definition of II A it is also true that:


r
r
r r
r
r r
r r r
r
r
[f , ( A b), ( A c )] + [( A f ), b, ( A c )] + [( A f ), ( A b), c ] = II A [f , b, c ]
r
r
r r r
r r
r
r r
r
r
[f , ( A b), ( A c )] II A [ f , b, c ] = [( A f ), b, ( A c )] [( A f ), ( A b), c]

Taking into account the above equation into the equation (1.70) we can obtain:

r r r
r r r
r
r r
r r
r
[( A 2 f ), b, c ] I A [( A f ), b, c ] = [( A f ), ( A b), c] [( A f ), b, ( A c )]
r r r
r r r
r
r
r r r
r
[( A 2 f ), b, c ] I A [( A f ), b, c] = [f , ( A b), ( A c )] II A [ f , b, c ]
r r r
r r r
r r r
r
r
r
[( A 2 f ), b, c ] I A [( A f ), b, c ] + II A [f , b, c ] [f , ( A b), ( A c )] = 0
r r r
r r r
r r r r
r
r
( A 2 f ) (b c ) I A ( A f ) (b c) + II A f (b c ) f ( A b) ( A c ) = 0
r
r r
r
In Problem 1.56 we have shown that ( A b) ( A c) = [cof( A )] (b c ) holds, then the

above equation becomes

University of Castilla-La Mancha


Ciudad Real - Spain

Draft

By: Eduardo W. V. Chaves (2014)

1 TENSORS

65

r r r
r r r
r r r r
r
r
( A 2 f ) (b c ) I A ( A f ) (b c) + II A f (b c ) f ( A b) ( A c ) = 0
r r r
r r r
r r r r
r r
( A 2 f ) (b c ) I A ( A f ) (b c) + II A f (b c) f [cof( A )] (b c) = 0
r
r
r r
r r
( A 2 f ) I A ( A f ) + II A f f [cof( A )] (b c ) = 0
r
r r r r
r r
r
Note that the vectors ( A 2 f ) , ( A f ) , f 0 , (f [cof(A )]) are not orthogonal to (b c ) 0 ,

so, we can conclude that

r
r
r r
r
( A 2 f ) I A ( A f ) + II A f f [cof( A )] = 0
r
r
r
r r
T
A 2 f I A A f + II A 1 f [cof( A )] f = 0
r r
T
A 2 I A A + II A 1 [cof( A )] f = 0
r
T
A 2 I A A + II A 1 [cof( A )] = 0

Using the definition A A 1 = [cof(A )]T , the above equation becomes


r
T
A 2 I A A + II A 1 [cof( A )] = 0
r
A 2 I A A + II A 1 A A 1 = 0

r
A 2 A I A A A + II A 1 A A A 1 A = 0 A
r
A 3 I A A 2 + II A A A 1 = 0

Problem 1.79
Based on the Cayley-Hamilton theorem, find the inverse of a tensor T in terms of tensor
power.
Solution:

The Cayley-Hamilton theorem states that:


T 3 T 2 I T + T II T III T 1 = 0

Carrying out the dot product between the previous equation and the tensor T 1 , we obtain:
T 3 T 1 T 2 T 1I T + T T 1 II T III T 1 T 1 = 0 T 1
T 2 TI T + 1 II T III T T 1 = 0

T 1 =

1
T 2 I T T + II T 1
III T

Problem 1.80
Check the Cayley-Hamilton theorem by using a second-order tensor whose Cartesian
components are given by:
5 0 0
T = 0 2 0
0 0 1

Solution:
The Cayley-Hamilton theorem states that:
T 3 T 2 I T + T II T III T 1 = 0

where I T = 5 + 2 + 1 = 8 , II T = 10 + 2 + 5 = 17 , III T = 10 , and

University of Castilla-La Mancha


Ciudad Real - Spain

Draft

By: Eduardo W. V. Chaves (2014)

SOLVING PROBLEMS BY MEANS OF CONTINUUM MECHANICS

66

5 3

=0
0

0 125 0 0

0 = 0 8 0
1 0 0 1

0
23
0

; T

5 2

=0
0

0
22
0

0 25 0 0

0 = 0 4 0
1 0 0 1

By applying the Cayley-Hamilton theorem, we can verify that the following is true:
125 0 0
25 0 0
5 0 0
1 0 0 0 0 0
0 8 0 8 0 4 0 + 17 0 2 0 10 0 1 0 = 0 0 0

0 0 1
0 0 1
0 0 1
0 0 1 0 0 0

Problem 1.81
Given the matrix P which is represented by its components Pij (i, j = 1,2,3,4) . a) Obtain the
inverse of P , b) the invariants, y c) the characteristic equation. Consider that:
1
2
P=
4

2 3 1
2 1 2
1 5 3

1 2 4

and

1
0
1=
0

0 0 0
1 0 0
0 1 0

0 0 1

Solution:
By applying the Cayley-Hamilton theorem we obtain:
P 4 + P 3 I1 + P 2 I 2 + P I 3 + I 41 = 0

(
P (P (P

P P 3 + P 2 I1 + P I 2 + 1 I 3 + I 41 = 0
2

+ P I1 + 1 I 2 + 1 I 3 + I 41 = 0

P P P P + 1 I 1 + 1 I 2 + 1 I 3 + I 41 = 0
1442443


C1

P P C1 + 1 I 2 + 1 I 3 + I 4 1 = 0
1
4
4
2
4
4
3

C2

P C2 + 1 I 3 + I 4 1 = 0

C3 + I 4 1 = 0

where we denoted by:

C0 = P

(
= P (C
= P (C

)
+1 I )
+1 I )

C1 = P C0 + 1 I 1

C2

C3

1
2

We can obtain the trace of C3 + I 41 = 0 as follows:


Tr (C3 + I 41) = Tr (0 )

Tr (C3 ) + Tr ( I 41) = Tr (C3 ) + I 4 Tr (1) = Tr (C3 ) + 4 I 4 = 0

I4 =

Tr (C3 )
4

Similarly, we can define that:


University of Castilla-La Mancha
Ciudad Real - Spain

Draft

By: Eduardo W. V. Chaves (2014)

1 TENSORS

Tr (C2 )
3

I3 =

I2 =

67

Tr (C1 )
2

I1 =

Tr (C0 )
1

With that we obtain:


I1 =

Tr (C0 )
= (1 + 2 + 5 + 4) = 12
1

thus we evaluate the matrix C1 = P C0 + 1 I 1 :


1
2
C1 =
4

2 3 1 1

2 1 2 2
1 5 3 4

1 2 4 3

I2 =

2 3 1
1
0

2 1 2
12
0
1 5 3

1 2 4
0

Tr (C1 ) (8 13 16 21) (42)


=
=
= 21
2
2
2

In turn we get C2 = P C1 + 1 I 2
1
2
C2 =
4

I3 =

2 3 1 8
6
14 14
1

2 1 2 8 13
5
7

+
21
0
1 5 3 13
6
16 3


1 2 4 11
2
4
21
0

0 0 0 37 22
15 17

2 5 5
1 0 0 7
=
0 1 0 10 12 14
2


14 11
0 0 1 9
5

Tr (C2 ) ( 37 2 14 + 5) ( 48)
=
=
= 16
3
3
3

In turn we get C3 = P C2 + 1 I 3

2 3 1 37 22
15 17
1

2 1 2 7
2 5 5
+ 16
0
1 5 3 10 12 14
2


1 2 4 9
5
14 11
0

1
2
C3 =
4

I4 =

14 14
0 0 0 8
6

7
1 0 0 8 13
5
=
16 3
0 1 0 13
6


21
0 0 1 11
2
4

0 0 0 32 0 0 0

1 0 0 0 32 0 0
=
0 1 0 0 0 32 0

0 0 1 0 0 0 32

Tr (C3 ) 4(32)
=
= 32 = det (P )
4
4

Then, the characteristic equation becomes:


P 4 + P 3 I1 + P 2 I 2 + P I 3 + I 41 = 0

P 4 12P 3 + 21P 2 + 16P 321 = 0

The characteristic equation coefficients could have been obtained by evaluates the
determinant:
det ( P 1) P 1

c) The inverse can be obtained by starting from:


P (C2 + 1 I 3 ) + I 4 1 = 0

P 1 P C2 + 1 I 3 + I 4 P 11 = 0
P 1 =

1
C2 + 1 I 3
I4

thus:

University of Castilla-La Mancha


Ciudad Real - Spain

P 1 =

1
adj[P ]
det (P )

Draft

C2 + 1 I 3 + I 4 P 1 = 0

adj[P ] = C2 + 1 I 3

By: Eduardo W. V. Chaves (2014)

SOLVING PROBLEMS BY MEANS OF CONTINUUM MECHANICS

68

P 1

37 22
15 17
1

0
2 5 5
1 7

=
+
16

0
2
(32) 10 12 14

9
14 11 5
0

0 0 0
15 17
21 22

5 5
1 0 0 1 7
14
=
0 1 0 32 10 12 2
2

14 11 21
0 0 1
9

NOTE 1: This procedure just performed, in the literature, is called Faddeev-Leverrier method.
Note that the inverse can also be obtained by using the same procedure as the one used in the
equation (1.46), i.e.:

1
32

2
4
3

0
2
4
3

0
2
4
3

0
2
4
3

0 0 0
2 1 2
1 5 3
1 2 4
1 0 0
2 1 2
1 5 3
1 2 4
0 1 0
2 1 2
1 5 3
1 2 4
0 0 1
2 1 2
1 5 3
1 2 4

1
4
3

1
0
4
3

1
0
4
3

1
0
4
3

2 3 1
0 0 0
1 5 3
1 2 4
2 3 1
1 0 0
1 5 3
1 2 4
2 3 1
0 1 0
1 5 3
1 2 4
2 3 1
0 0 1
1 5 3
1 2 4

University of Castilla-La Mancha


Ciudad Real - Spain

2
1
3

1
2
0
3

1
2
0
3

1
2
0
3

2 3 1
2 1 2
0 0 0
1 2 4
2 3 1
2 1 2
1 0 0
1 2 4
2 3 1
2 1 2
0 1 0
1 2 4
2 3 1
2 1 2
0 0 1
1 2 4

Draft

2
4
1

1
2
4
0

1
2
4
0

1
2
4
0

2 3 1
2 1 2
1 5 3
0 0 0
2 3 1
2 1 2
1 5 3
1 0 0
2 3 1
2 1 2
1 5 3
0 1 0
2 3 1
2 1 2
1 5 3
0 0 1

= 1
32

21
7
10
9

22 15 17

14

12

14

11 21

By: Eduardo W. V. Chaves (2014)

1 TENSORS

69

NOTE 2: We can also obtain the characteristic coefficients by means of the following
procedure. Considering P 4 P 3 I 1 + P 2 I 2 P I 3 + I 4 1 = 0
The last coefficient is I 4 = det(P ) = 32 .

The coefficient I 3 is obtained by the sum of the determinants of the resulting matrices by
eliminating 1 row and 1 column associated with the main diagonal, i.e.
1
2
I3 =
4

2 3 1 1 2
2 1 2 2 2
+
1 5 3 4 1

1 2 4 3 1

3 1 1 2
1 2 2 2
+
5 3 4 1

2 4 3 1

3 1 1 2
1 2 2 2
+
5 3 4 1

2 4 3 1

3 1
1 2
5 3

2 4

2 1 2 1 3 1 1 2 1 1 2 3
= 1 5 3 + 4 5 3 + 2 2 2 + 2 2 1 = 16
1 2 4 3 2 4 3 1 4 4 2 5

The coefficient I 2 is obtained by the sum of the determinants of the resulting matrices by
eliminating 2 rows and 2 columns associated with the main diagonal, i.e.
1
2
I2 =
4

2 3 1
2 1 2
+
1 5 3

1 2 4

1
2

2 3 1
2 1 2
+
1 5 3

1 2 4

1
2
+
4

2 3 1 1
2 1 2 2
+
1 5 3 4

1 2 4 3

2 3 1
2 1 2
+
1 5 3

1 2 4

1
2
+
4

2 3 1
2 1 2
1 5 3

1 2 4

1
2

2 3 1
2 1 2
+
1 5 3

1 2 4

5 3 2 2 2 1 1 1 1 3 1 2
=
= 21
+
+
+
+
+
2 4 1 4 1 5 3 4 4 5 2 2

The coefficient I 1 is obtained by the sum of the determinants of the resulting matrices by
eliminating 3 rows and 3 columns associated with the main diagonal, i.e.
1
2
I1 =
4

2 3 1 1 2
2 1 2 2 2
+
1 5 3 4 1

1 2 4 3 1

3 1 1
1 2 2
+
5 3 4

2 4 3

= [4] + [1] + [2] + 5 = 12 = Tr (P )

University of Castilla-La Mancha


Ciudad Real - Spain

2 3 1 1 2
2 1 2 2 2
+
1 5 3 4 1

1 2 4 3 1

Draft

3 1
1 2
5 3

2 4

By: Eduardo W. V. Chaves (2014)

SOLVING PROBLEMS BY MEANS OF CONTINUUM MECHANICS

70

Problem 1.82
Let A be a second-order tensor, show that:
a) II A =

1
( I A ) 2 Tr ( A 2 )
2

b) det ( A ) =

1
[Tr (A )]3 + 2 Tr( A 3 ) 3Tr(A ) Tr(A 2 )
6

Solution:

a) It was shown in Problem 1.79 that III A A 1 = (A 2 AI A + 1 II A ) , then, by applying the


double scalar product with the second-order unit tensor we obtain:

III A A 1 : 1 = A 2 AI A + 1 II A : 1 = A 2 : 1 A : 1 I A + 1 : 1 II A
1

III A Tr ( A ) = Tr ( A ) Tr ( A ) I A + Tr (1) II A = Tr ( A 2 ) ( I A ) 2 + 3 II A

Taking into account the inverse of a tensor A 1 =

[cof ( A )]T
III A Tr ( A 1 ) = Tr ( III A A 1 ) = Tr III A

III A

[cof ( A )]T , we can conclude that:


III A

= Tr [cof ( A )]T = Tr ([cof ( A ) ]) = II A

With that, we can obtain:


III A Tr ( A 1 ) = II A = Tr ( A 2 ) ( I A ) 2 + 3 II A
II A =

II A 3 II A = Tr ( A 2 ) ( I A ) 2

1
( I A ) 2 Tr ( A 2 )
2

b) We start from the Cayley-Hamilton theorem, which states that any tensor satisfies its own
characteristic equation, i.e.:
A 3 A 2 I A + AII A III A 1 = 0

where I A = [Tr (A )] , II A =

1
[Tr( A )]2 Tr( A 2 ) , III
2

(1.71)

= det (A ) are the principal invariants of

A . Applying the double scalar product between the second-order unit tensor ( 1 ) and the
equation in (1.71) we obtain:

A 3 : 1 A 2 : 1 I A + A : 1II A III A 1 : 1 = 0 : 1
Tr ( A 3 ) Tr ( A 2 ) I A + Tr ( A ) II A III

[Tr(1)] = [Tr(0)]

1
[Tr(A )]2 Tr(A 2 ) III A 3 = 0
2
1
1
3
Tr ( A 3 ) Tr ( A 2 ) Tr ( A ) + [Tr ( A )] Tr ( A ) Tr ( A 2 ) III A 3 = 0
2
2
1
3
2 Tr ( A 3 ) 3 Tr ( A 2 ) Tr ( A ) + [Tr ( A )] III A 3 = 0
2
Tr ( A 3 ) Tr ( A 2 ) Tr ( A ) + Tr ( A )

with which we obtain:


III

= det ( A ) =

1
[Tr( A )]3 + 2 Tr(A 3 ) 3Tr( A 2 ) Tr(A )
6

= det ( A ) =

1
A ii A jj A kk + 2 A ij A jk A ki 3A ij A ji A kk
6

or in indicial notation:
III

University of Castilla-La Mancha


Ciudad Real - Spain

Draft

By: Eduardo W. V. Chaves (2014)

1 TENSORS

71

NOTE: It is interesting to note that the principal invariants of A are formed by the three
fundamental invariants of a second-order tensor, namely Tr (A ) , Tr ( A 2 ) , Tr ( A 3 ) , i.e.:
I A = Tr ( A )

III

} {

1
1
[Tr ( A )]2 Tr ( A 2 )
[ Tr ( A )] 2 Tr ( A 2 ) =
2
2
1
[Tr ( A )]3 + 2 Tr ( A 3 ) 3 Tr ( A 2 ) Tr ( A )
= det ( A ) =
6

II A =

Problem 1.83
Show that II T = III T Tr ( T 1 ) , where II T =
invariant of T , and III

1
[Tr( T )]2 Tr(T 2 )
2

is the second principal

is the third principal invariant (the determinant of T ).

Solution:
It was shown in Problem 1.79 that T 1 =

1
T 2 TI T + 1 II T , then, by applying the double
III T

scalar product with the second-order unit tensor we obtain:


T 1 : 1 =
Tr ( T 1 ) =

1
1
T 2 TI T + 1 II T : 1 =
T 2 : 1 T : 1 I T + 1 : 1 II T
III T
III T

1
Tr ( T 2 ) Tr ( T ) I T + Tr (1) II T
III T

III T Tr ( T 1 ) = Tr ( T 2 ) I T2 + 3 II T
14243

)
III T Tr ( T 1 ) = II T

= 2 II T

Problem 1.84
Show that:
r r
1
(1 + c b) 1 = 1

(c b )
r r
( + c b)

where c and b are vectors, 1 is the second-order unit tensor, and and are scalars.
Solution:
r

Let us consider that T = (1 + c b) , and the inverse of a tensor obtained in Problem


1.79:
T 1 =

1
T 2 TI T + 1 II T
III T

(1.72)

Next, we obtain T 2 :

r r
r r
T 2 = T T = (1 + c b) (1 + c b)
r r
r r
r r r r
= 2 1 1 + 1 (c b) + (c b) 1 + 2 (c b) (c b)
r r r r
r r r r
where it fulfills that (c b) (c b) = (c b)(c b) , (see Problem 1.20). Then, the above

expression can be rewritten as follows:


University of Castilla-La Mancha
Ciudad Real - Spain

Draft

By: Eduardo W. V. Chaves (2014)

SOLVING PROBLEMS BY MEANS OF CONTINUUM MECHANICS

72

r r
r r r r
T 2 = 2 1 + 2 (c b) + 2 (c b)(c b)

and its trace is given by:

r r
r r r r
r r
r r
r r
Tr ( T 2 ) = Tr 2 1 + 2 (c b) + 2 (c b)(c b) = 2 Tr (1) + 2Tr (c b) + 2 (c b) Tr (c b)
r r
r r r r
r r
r r
= 3 2 + 2 (c b) + 2 (c b)(c b) = 3 2 + 2 (c b) + 2 (c b) 2

Next, we calculate the principal invariants of T

r r
r r
r r
I T = Tr (1 + c b) = Tr (1) + Tr (c b) = 3 + (c b)
r r 2
r r
r r
( I T ) 2 = 3 + (c b) = 9 2 + 6 (c b) + 2 (c b) 2

} {

r r
r r
r r
r r
1
1
2
I T Tr ( T 2 ) = 9 2 + 6 (c b) + 2 (c b) 2 3 2 + 2 (c b) + 2 (c b) 2
2
2
r r
2
= 3 + 2 (c b)

II T =

]}

r r
r r
III T = det (1 + c b) = 3 + 2 c b (see Problem 1.50)

Then, the equation in (1.72) becomes:


III T T 1 = T 2 I T T + II T 1
r r
r r r r
= 2 1 + 2 (c b ) + 2 (c b)(c b )
r r
r r
r r
3 + (c b ) ( 1 + c b) + 3 2 + 2 (c b) 1
r r
r r r r
r r
r r
= 2 1 + 2 (c b ) + 2 (c b)(c b ) 3 2 1 3 (c b ) (c b)1
r r r r
r r
2 (c b)(c b ) + 3 2 1 + 2 (c b)1
r r
r r
r r
r r
= 1 2 + (c b)1 (c b) = ( 2 + c b )1 (c b)
r r
r r
1
T
= ( 3 + 2 c b)1 (c b) = [adj( T )] = [cof ( T )]

(1.73)

r r

Taking into account that T = (1 + c b) , III T = 3 + 2 c b , the above equation


becomes:
T

r r
r r
(c b) 1
(c b)
1 III T
=
1
= 1
r r
III T
III T

( 3 + 2 c b )

(1.74)

or:
r r
1
(1 + c b) 1 = 1

r r

r
b)
(
c
r
( + c b)

Tensorial notation

(1.75)

(c b )
( + c k b k ) i j

Indicial notation

(1.76)

[{c}{b}T ]
( + {c}T {b})

Matrix notation

(1.77)

( ij + c i b j ) 1 =

[[1] + [{c}{b} ] ]
T

[ 1]

ij

NOTE 1: The above equation is also valid for matrices of n-dimensions.


In the particular case when = 1 , = 1 , we obtain:

University of Castilla-La Mancha


Ciudad Real - Spain

Draft

By: Eduardo W. V. Chaves (2014)

1 TENSORS

73

r r
r r 1
(c b)
(1 + c b) = 1
r r
1+ c b

(1.78)

NOTE 2: The equation in (1.75) or in (1.73) can be rewritten as follows:


r r
1
T 1 = ( 1 + c b) 1 = 1
=

r r

r r (c b )
( + c b)

r r
r r
1
r ( 2 + c b)1 (c b) =
[adj( T )]
r
3
2
det ( T )
( + c b)
1

with that we conclude that:

r r
r r
r r
adj(1 + c b) = ( 2 + c b)1 (c b)

NOTE 3: We can extend the equation in (1.75) such that:


( I sym + A B ) 1 =

I sym

( A B)
( + A : B )

where we now have that I sym is the symmetric fourth-order unit tensor, A and B are secondorder tensors, and and are scalars. With that it is easy to show that ( I sym ) 1 = I sym .
Problem 1.85
r

Taking into account that ( 1 + c b) 1 =


r r
1
( A + a b) 1 = A 1

r r

r r (c b) , show that:
( + c b)

r
r
( A 1 a) (b A 1 )
r
r
( + b A 1 a)

(1.79)

where a and b are vectors, A is a second-order tensor, with det ( A ) 0 ( A 1 ), and and
are scalars.
Solution:
r

Note that the expression (A + a b) can be rewritten as follows:


r r
r r
r r
r r
(A + a b) = (A 1 + 1 a b) = (A 1 + ( A A 1 ) a b) = A (1 + A 1 a b)

Using the inverse property such as ( A B ) 1 = B 1 A 1 , we can obtain:

r r
r r 1
r r
(A + a b) 1 = A (1 + A 1 a b) = (1 + A 1 a b) 1 A 1
r
Note that the result of the algebraic operation A 1 a is a vector in which we denote
r
r
c = A 1 a , with that we rewrite the above equation as follows:

University of Castilla-La Mancha


Ciudad Real - Spain

Draft

By: Eduardo W. V. Chaves (2014)

SOLVING PROBLEMS BY MEANS OF CONTINUUM MECHANICS

74

r r
r r
r r
(A + a b) 1 = ( 1 + A 1 a b) 1 A 1 = ( 1 + c b) 1 A 1
1
r r 1 1
r r

1
1
= 1
r r (c b) A = 1 A
r r (c b) A

( + c b)
( + c b)

r
r
r

1
1 1
1
1 r
1
= A 1
r r c bA = A
r r ( A a) (b A )

( + c b)
( + c b)
r
1

1 r
r
= A 1
(

(
A 1 )
A
a
b
r

( + b A 1 a)

The above equation in indicial notation becomes:


(A ij + a i b j ) 1 =

A ij1

( + b p A pq1 a q )

( A ik1 a k )(b s A sj1 )


r

The reader should be aware here with the algebraic operation ( A 1 a) b

r r
A 1 ( a b )
14243

Invalid Expression

the latter has no consistency, since we can not have a scalar product (contraction) with the
r r
scalar (a b) . We can check this fact by means of indicial notation
r r
r
then,
the
possible
c b = c i b i = ( A 1 a) i b i = A ik1 a k b i ,
r
1 r
1
1
1
1
1
( A a) b = b i A ik a k = a k A ik b i = A ik b i a k = A ik a k b i = A ik a k b i .
1
4
3 1
424
3
1
42r4
3
1
424
3
1
424
3
r 24
r
r
r
r
r r
r r
bA 1a

aA T b

A 1:(ba)

A T :( ab )

expressions

are

A 1:( ab )T

For the particular case when = 1 , = 1 , we fall back on the Sherman-Morrison formula:
r
r
r r 1
( A 1 a) (b A 1 )
1
r
( A + a b) = A
r
1 + b A 1 a

Sherman-Morrison formula

(1.80)

(tensorial notation)

The above equation in matrix notation becomes

[[ A] + [{a}{b} ] ]
T

= [ A]

}{

[ A] 1 {a} {b}T [ A] 1

1 + {b}T [ A] 1 {a}

}
T

Sherman-Morrison formula

(1.81)

(matrix notation)
r

NOTE 1: Note that if (A + a b) = A (1 + A 1 a b) , the determinant is defined


as follows:

r r
r r
r r
det (A + a b) = det A (1 + A 1 a b) = det [A ]det (1 + A 1 a b)
r
r
= det [A ]( 3 + 2 b A 1 a)

with that, the equation in (1.79) can be rewritten as follows:

{ A (

r r
1
(A + a b) 1 =

r
r
r
r
+ b A 1 a) A 1 A ( A 1 a) (b A 1 )

with = det (A + a b) = A ( 3 + 2 b A 1 a) .
with that we conclude that:

(1.82)

r
r
r r
r
r
adj(A + a b ) = A ( 2 + b A 1 a) A 1 A ( A 1 a) (b A 1 )

University of Castilla-La Mancha


Ciudad Real - Spain

Draft

]}

]}

By: Eduardo W. V. Chaves (2014)

1 TENSORS

75

NOTE 2: We can extend the equation in (1.79) such that:


( D + A B ) 1 =

D 1

( + B : D 1 : A )

[(D

: A ) (B : D 1 )

(1.83)

where we now have that D is a fourth-order tensor, A and B are second-order tensors, and
and are scalars.
Note that:
(D + A B ) 1 =

{ D (

+ B : D 1 : A )D 1 D (D 1 : A ) (B : D 1 )

]}

with = det (D + A B ) = D ( 3 + 2 B : D 1 : A ) .

(1.84)

where we can conclude that:


det (D + A B ) = det (D )( 3 + 2 B : D 1 : A )

adj(D + A B ) = D ( 2 + B : D 1 : A )D 1 D (D 1 : A ) (B : D 1 )

Problem 1.86

(1.85)

]}

(1.86)

a) Let C = (1 + a b + c d) be a second-order tensor. Show that:


r

r r

[r

r r

r r r

r r r r

1 + a b + c d = 3 + 2 (c d) + 2 (a b) + (a b)(c d) (a d)(b c )

(1.87)
r

where 1 + a b + c d = det (1 + a b + c d) represents the determinant of


r

the tensor C . b) For the particular case when = 1 , d = a , c = b , show that:


r r
r r
r r
r r
det (1 + a b + b a) = 1 + ( + )(a b) a b

Solution:
r

(1.88)
r

We define an auxiliary tensor D = 1 + a b and in turn we have C = (D + c d) .


According to Problem 1.85, (see equation (1.79)), it holds that:
r
r r
r
det (D + c d) = D (1 + d D 1 c ) , where:
r r
r r
det (D) D = det (1 + a b ) = 3 + 2 (a b) and
r r
1
(D ) 1 = (1 + a b) 1 = 1

r r

r r (a b)
( + a b )

With that, we can say that:

University of Castilla-La Mancha


Ciudad Real - Spain

Draft

By: Eduardo W. V. Chaves (2014)

SOLVING PROBLEMS BY MEANS OF CONTINUUM MECHANICS

76

r
r r
r
det (D + c d) = D (1 + d D 1 c )

r 1
r r
r r r

= 3 + 2 (a b ) (1 + d 1
b) c
(
a
r
r

( + a b )

r r
= 3 + 2 (a b ) 1 +

r r r r
1 r
r

d 1 c
r d (a b) c
r

( + a b )

r r
r r
1 r r

dc

c )
(
d
a
)
(
b
r
r

( + a b )


r r
r r r r
r r

= 3 + 2 (a b ) 1 + (c d)
r (a d)(b c )
r
2 ( + a b )

r r
r r
r r
r r
r r r r
Note that (d a) (b c ) = (1
a d) (b c ) = (a d)(b c ) .
23 123
r r
= 3 + 2 (a b ) 1 +

scalar

scalar

[
[

r r
r r
r r
r r r r
r r

r (a d)(b c )
det (D + c d) = 3 + 2 (a b) 1 + (c d) 3 + 2 (a b)
r
2

( + a b)

r
r
r
r
r
r
r
r

= 3 + 2 (a b) 1 + (c d) (a d)(b c )

r
r
r
r
r r r r
r r r r
= 3 + 2 (c d) + 2 (a b) + (a b)(c d) (a d)(b c )

Then:

r r
r r
r r
r r
r r r r
r r r r
det (1 + a b + c d) = 3 + 2 (c d) + 2 (a b ) + (a b )( c d) (a d)(b c )

thereby demonstrating the equation (1.87).


r

For the particular case when d = a , c = b , we have:

[
[

r r
r r
r r
r r
r r r r
r r r r
det (1 + a b + c d) = 3 + 2 (c d) + 2 (a b) (a d)(b c ) (a b)(c d)
r r
r r
r r
r r
r r r r
r r r r
det (1 + a b + b a) = 3 + 2 (b a) + 2 (a b) (a a)(b b) (a b)(b a)
r r
r r r r
r r r r
= 3 + 2 ( + )(a b) (a a)(b b) (a b)(a b)
r r 2 r 2 r 2 r r 2
In Problem 1.1 we have shown that a b = a b a b holds, thus:

]
]

( )

r r
r r
r r
r r
det (1 + a b + b a) = 3 + 2 ( + )(a b) a b

For the particular case when = 1 we get:


r r
r r
r r
r r
det (1 + a b + b a) = 1 + ( + )(a b) a b

Problem 1.87
r

a) Obtain the inverse of the tensor C = (1 + a b + c d) .


r r
p p
pq

b.1) Given the second-order tensor D = B + r r

r
r
(B q) (B q)
where B = B T and
r
r
q B q

B 1 , show that:

University of Castilla-La Mancha


Ciudad Real - Spain

Draft

By: Eduardo W. V. Chaves (2014)

1 TENSORS

=B

77

r r r
r
( p q + p B 1 p ) r r
[q q ] r 2 r qr ( B 1 pr )
+
r r 2
( p q)
( p q)

sym

(1.89)

b.2) If B is a positive definite tensor, obtain the conditions under which D is a non-singular
tensor.
Solution:

Denoting by A = (1 + a b) we obtain C = ( A + c d) , and by taking into account


r r
1
( A + a b) 1 = A 1

r
r
r
( A 1 a) (b A 1 )
r
( + b A 1 a)

(1.90)

which was obtained in Problem 1.85, (see equation (1.79), thus


r r
( A + c d) 1 = A 1

r
r

r
( A 1 c ) (d A 1 )
r
(1 + d A 1 c )

(1.91)

It was shown in Problem 1.84 that:


r r
1
(1 + c b) 1 = 1

With that we obtain:


A

r r
1
= (1 + a b) 1 = 1

(c b )
r r
( + c b)

(1.92)

(a b)
r r
( + a b )

Furthermore, we have
r r
r r
r r
r 1
(a b) r 1 r
(a b) r 1 r
(b c ) r

A c =
1
r r c = 1c
r r c = c
r r a

( + a b)
( + a b )
( + a b )

r r
r r
r r
r
r 1
r
r
1 r

(
)
(

)
1
(
a) r
d

a
b
a
b
1
r = d 1 d
r = d
d A = d 1
r
r
r r b

b)
a
+
a
b
a
b

With that we conclude that

] [

r r r
r
r r
r r
r r
r
r r r
(1 + a b + c d) 1 = (1) 1 + ( 2 ) (a b) + ( 3) (1) c + ( 2 ) (b c )a (1) d + ( 2 ) (a d)b

(1.93)
where

(1) =
( 2) =
( 3) =

r r

( + a b )

r
r
(1 + d A 1 c )

r
r 1 r r
d A 1 c = (d c )

r r r r

r r (d a )(b c )
( + a b )

NOTE: The equation in (1.93) is also valid for matrices of n-dimensions.


University of Castilla-La Mancha
Ciudad Real - Spain

Draft

By: Eduardo W. V. Chaves (2014)

SOLVING PROBLEMS BY MEANS OF CONTINUUM MECHANICS

78

b.1) We can rewrite the tensor D as follows:


r r
r
r
r r
r
r
p p
p p
(B q) (B q)
(B q) (B q)
1
1
B
B
B
B
B
(
)
(
)

D = B 1 + 1 r r 1
1
=
+

r
r
r
r
r r
pq
q B q
pq
q Bq
r r
r
r
r
r
r
r

p p
( B 1 p) p ( B 1 B q ) ( B q )
1
1 ( B q ) ( B q )

= B 1 + B r r B
r
r
r r
r
r

= B 1 +
pq
q B q
pq
q Bq

r r
r
1 r

( B p) p q ( B q )
= B 1 +
r
r r
r
pq
q Bq

and by denoting by
r
r
a = ( B 1 p)

r r
b=p

r r
c =q

r
r
d = (B q)

= r r
pq

1
= r
r
q B q

we get

r r
r r
D = B 1 + a b + c d = B C

D 1 = ( B C ) 1 = C 1 B 1
r r
r r
where C = [1 + a b + c d ] . The inverse of C can be obtained via subsection (a) with
= 1 . Moreover, we have:

(1) = 1 ,
( 2) =

1
1
1

= r r r
r
r r =
r r = r r
( + a b) (1 + a b) p q (1 + r 1 r ( B 1 pr ) pr ) ( p q + p B 1 p)
pq

r
r 1 r r
d A 1 c = (d c )

r r r r

r r (a d )(b c )
( + a b )

r r
r
r r r
1
= (( B q ) q ) + r r r
(( B 1 p) ( B q ) )( p q )
1 r
( p q + p B p)
r
r r r
r
r ( p ( B T B ) q ) ( p q )
= q B q +
r r r
r
( p q + p B 1 p)

1
1
= r
r
r
r
r r r
1 r
r ( p ( B T B ) q ) ( p q )
1 r
(1 + d A c ) q B q
1 + r

+
q
B
q

r
r r r
r

q Bq
( p q + p B 1 p )

r r r
1 r
( p q + p B p)
= r
r r r
( p ( B T B ) q ) ( p q )
r r r
r
( p q + p B 1 p )
1
1
( 2 ) (3) = r r r 1 r r
r r r = r
r r r
T
T
( p q + p B p) ( p ( B B ) q ) ( p q ) ( p ( B B ) q ) ( p q )

( 3) =

r r

( 2 ) ( 3) (a d ) = r

( p (B

= r
( p ( B T
r r

( 2 ) ( 3) (b c ) = r

( p (B

r r r r

r
1
1 r
r r r (( B p) ( B q ))
B) q) ( p q )
r
r
1
1
T
r r r ( p (B B) q ) = r r
( p q)
B) q) ( p q )
r r
1
1
r r r ( p q) = r
r
T
B) q ) ( p q )
( p (B B) q )

( 2 ) (3) (b c )(a d ) = r

( p (B

University of Castilla-La Mancha


Ciudad Real - Spain

r r r
1
1 r
r r r (( B p) ( B q ))( p q ) = 1
B) q ) ( p q )
Draft

By: Eduardo W. V. Chaves (2014)

1 TENSORS

79

The equation in (1.93) becomes:

[
[

] [

r r r
r
r r
r
r r r
C 1 = 1 + ( 2 ) (a b ) + ( 3) c + ( 2 ) (b c )a d + ( 2 ) (a d )b
r r r r
r r
r r
r r r r
C 1 = 1 + ( 2 ) a b + ( 3) c d + ( 3) ( 2 ) (a d ) c b + ( 3) ( 2 ) (b c ) a d + L
r r r r r r
L + ( 3) (22 ) (b c )(a d ) a b
r r r r r r
r r
r r r r
C 1 = 1 + ( 2 ) + ( 3) (22 ) (b c )(a d ) a b + ( 3) c d + ( 3) ( 2 ) (a d ) c b + L
r r r r
L + ( 3) ( 2 ) ( b c ) a d

}[

Note that: { ( 2 ) + (3) (22 ) (b c )(a d )}= ( 2 ) {1 + (3) ( 2 ) (b c )(a d )}= ( 2 ) {1 1} = 0 , thus
r r r r

r r r r

r r r r
r r
r r r r
C 1 = 1 + ( 3) c d + ( 3) ( 2 ) (a d ) c b + ( 3) ( 2 ) (b c ) a d

r r r r
r r
r r r r
C 1 = 1 + ( 3) c d + ( 3) ( 2 ) (a d ) c b + ( 3) ( 2 ) (b c ) a d
r r r
r
r
r
( p q + p B 1 p)
1 r r
=1 + r
r r r [q ( B q ) ] + r r [q p ] + L
T
( p q)
( p (B B) q ) ( p q )
r
1
1 r
L+ r
r ( B p) ( B q )
T
( p (B B) q )

With that, we can obtain:


D 1 = C 1 B 1
r r r
r

r
r
1 r r
( p q + p B 1 p )
= 1 + r
r r r [q ( B q ) ] + r r [q p ] + L
T
( p q)
( p (B B) q ) ( p q )

= B 1

r 1
1
1 r
L+ r
r ( B p) ( B q ) B
T
( p (B B) q )

r r r
1 r
r
r
1 r r
( p q + p B p)
1
1
+ r
r r r [q ( B q )] B + r r [q p ] B + L
T
p
q

(
)
( p (B B) q ) ( p q )
r
1
1 r
1
L+ r
r ( B p) ( B q ) B
T
( p (B B) q )

Note that:

{[qr ( B qr )] B } = [qr ( B qr )] B = [q ( B qr ) ]B
r
r
= [q ( B B ) q ) ]
[( B pr ) ( B qr )] B = ( B pr ) ( B B ) qr
1

ij

ik

1
kj

1
kj

= q i ( B kp q p ) B kj1 = q i ( B kp B kj1 q p )

ij

If now we consider the symmetry of B , i.e. B = B T , we obtain:


D

=B

= B 1
= B 1

r r r
r
( p q + p B 1 p ) r r
[q q ] + r 1r
+
r r r r
( p q) ( p q)
( p q)
r r r
1 r
( p q + p B p) r r
[q q ] + r 1r
+
r r r r
( p q) ( p q)
( p q)
r r r
1 r
( p q + p B p) r r
+
[q q ] r 2 r
r r 2
( p q)
( p q)

[qr ( pr B )] + ( pr1qr ) [( B
1

{[qr ( pr B )] + [( B
1

[qr ( B
r

p) q ]
r

p) q ]
r

p)]
r

sym

Note that, due to the symmetry of B , it holds that p B 1 = B 1 p = s , and B T B = 1 .

University of Castilla-La Mancha


Ciudad Real - Spain

Draft

By: Eduardo W. V. Chaves (2014)

SOLVING PROBLEMS BY MEANS OF CONTINUUM MECHANICS

80

b.2) A tensor is non-singular if det (D ) 0 . By using the equation obtained previously we get:

r r
r r
D = B 1 + a b + c d

( [

])

r r
r r
r r
r r
det (D) = det B 1 + a b + c d = det ( B )det 1 + a b + c d

Note that det ( B ) > 0 , since B is a positive definite tensor. Then, the condition under which
r r
r r
D is a non-singular tensor is det[1 + a b + c d ] 0 . By using the determinant
expression obtained in Problem 1.86 we get:

r r
r r
r r
r r r r
r r r r
det (1 + a b + c d ) = 3 + 2 (c d ) (a d )( b c ) (a b )(c d )
r r
r r r
r r r
r
r
r r r r r r
where = 1 , a b = ( B 1 p ) p = p B 1 p , a d = ( B 1 p ) ( B q ) = p q , b c = p q
r r r
r
r
r
r r
r
1 r
c d = q ( B q ) = q B q , (c d ) = r
r q B q = 1 ,
q B q

[(a d )(b c ) (a b )(c d ) ] = r r r


r r r r

r r r r

r r r r
r
r
1
1 r r
r ( p q )( p q ) ( p B p )( q B q )
pq q B q
1

Thus:

r r
r r
r r r r
r
r
1
1 r r
det 1 + a b + c d = r r r
r ( p q )( p q ) ( p B p )(q B q ) 0
( p q )(q B q )
r
r
r r
r r
r
r
Then, the conditions are: p 0 , q 0 , ( p q ) 0 , i.e. p and q can not be orthogonal

vectors. Another condition that must be met is:


r r r r
r
r r
r
( p q )( p q ) ( p B 1 p )(q B q ) 0
142
4 43
4 144424443
>0

>0

Note that by the fact that B is positive definite tensor, the scalar (q B q ) > 0 is always
r r
r
r
positive for any vector q 0 . The same apply to ( p B 1 p ) > 0 , since, if the tensor is
positive definite so is its inverse. Note also that D is a positive definite tensor if
r r
r
r r
r
r r
( p q ) 2 > ( p B 1 p )(q B q ) and ( p q ) > 0 . These two conditions can be replaced by
r r
r
r r
r
( p q ) > ( p B 1 p )(q B q ) .

Problem 1.88
Let A = A () and be a second-order tensor and a scalar respectively, show that:
dA

dA
A 1
= A Tr

d
d

(1.94)

Solution:
In Problem 1.82 and in Problem 1.79, we have demonstrated, respectively, that:
III

= det ( A ) = A =

1
[Tr( A )]3 + 2 Tr(A 3 ) 3Tr( A 2 ) Tr(A )
6

III A A 1 = A 2 AI A + II A 1

where I A = Tr (A ) , II A =

(1.96)

1
[Tr ( A )]2 Tr ( A 2 ) .
2

University of Castilla-La Mancha


Ciudad Real - Spain

(1.95)

Draft

By: Eduardo W. V. Chaves (2014)

1 TENSORS

81

Note also that the following derivatives are true:


d [I A ] d [Tr ( A )] d [A kk ] d [A ik ik ] d [A ik ]
dA
dA
=
=
=
=
ik =
: 1 = Tr
d
d
d
d
d
d
d

d (A 2 )
d Tr ( A 2 )
dA

dA
= Tr
= 2 Tr A
= Tr 2A

d
d
d

d
d Tr ( A 3 )
dA

= 3Tr A 2
d
d

Taking the derivative of (1.95) with respect to we obtain:

}
[

d ( III A ) 1 d
[Tr( A )]3 + 2 Tr(A 3 ) 3Tr( A 2 ) Tr(A )
=
d
6 d
d Tr ( A 3 )
d Tr ( A 2 )
d [Tr ( A )]
1
2 d [Tr ( A )]
Tr ( A ) 3 Tr ( A 2 )
= 3[Tr ( A )]
+2
3

dt
d
d
d
6

1
dA
dA
2 dA
dA
2
Tr ( A ) 3Tr ( A 2 ) Tr
6 Tr A
3[Tr ( A )] Tr + 6 Tr A

d
d
6
d

dA
1
dA
dA

2
Tr ( A ) + [Tr ( A )] Tr ( A 2 ) Tr
Tr A
= Tr A 2

d
d
2
d

or
d ( III A )
dA

dA
dA
= Tr A 2
Tr A
I A + II A Tr

d
d
d

Taking the scalar product of the equation in (1.96) with


III A A 1

(1.97)

dA
, we can obtain:
d

dA
dA
dA
dA
dA
= A 2 AI A + II A 1
= A2
A
I A + II A
d
d
d
d
d

and the trace of the above equation is given by:


dA
dA
dA
2 dA

Tr A 1
A
I A + II A

III A = Tr A

d
d
d
d

dA
dA

dA
= Tr A 2
Tr A
I A + Tr
II A

d
d

(1.98)

Comparing equations (1.97) and (1.98), we can conclude that:


d ( III A )
dA
dA

A 1
= III A Tr A 1
= III A Tr
d
d

1.1.11 Isotropic and Anisotropic Tensors


Problem 1.89
Let C be a fourth-order tensor, whose components are
C ijkl = ij kl + ik jl + il jk

University of Castilla-La Mancha


Ciudad Real - Spain

Draft

(1.99)

By: Eduardo W. V. Chaves (2014)

SOLVING PROBLEMS BY MEANS OF CONTINUUM MECHANICS

82

where ij are the second-order unit tensor components, and , , are scalar.
a) What kind of symmetry has the tensor C ? b) What conditions must be met to guarantee
the symmetry of C ?
Solution:
The tensor has major symmetry whether C ijkl = C klij holds. Taking into account the equation
in (1.99), we conclude that the tensor has major symmetry since
C klij = kl ij + ki lj + kj li = C ijkl

We check now if the tensor has minor symmetry, e.g. C ijkl = C ijlk
C ijlk = ij lk + il jk + ik jl C ijkl

We can easily verify this fact by adopting i = 2 , j = 1 , k = 1 , l = 2 , thus


C ijkl = C 2112 = 21 12 + 21 12 + 22 11 =
C ijlk = C 2121 = 21 21 + 22 11 + 21 12 =

Then, the tensor C has minor symmetry if and only if = , with that we obtain:
C ijkl = ij kl + ( ik jl + il jk )

Note that ij kl has major and minor symmetry, while the tensors ik jl , il jk are not
sym
symmetric. Note also that ( ik jl + il jk ) = 2I ijkl
.

Problem 1.90
Let C be a fourth-order tensor, whose components are given by:
C ijkl = ij kl + ( ik jl + il jk )

(1.100)

where , are constant real numbers. Show that C is isotropic.


Solution:
Applying the transformation law for the fourth-order tensor components:
C ijkl = a im a jn a kp a lq C mnpq

(1.101)

and by replacing the relation C mnpq = mn pq + ( mp nq + mq np ) into the above equation,


we obtain:

Cijkl = aim a jn akp alq mn


= aim a jn akp alq mn

pq

pq

+ mp nq + mq np

+ aim a jn akp alq mp nq + aim a jn akp alq mq np

= ain a jn akq alq + aip a jq akp alq + aiq a jn akn alq

= ij kl + ik

jl

)]

+ il

jk

)= C

(1.102)

ijkl

which proves that C is an isotropic tensor, i.e. the C -components do not change for any
transformation basis.

University of Castilla-La Mancha


Ciudad Real - Spain

Draft

By: Eduardo W. V. Chaves (2014)

1 TENSORS

83

Problem 1.91
Let C be a symmetric isotropic fourth-order tensor which is represented by its components
as follows:

C ijkl = ij kl + ik jl + il jk
C = 1 1 + 2 I

(indicial notation)
(tensorial notation)

where and are scalars, 1 is the second-order unit tensor, I is the symmetric fourthorder unit tensor, i.e. I I sym .
a) Given a symmetric second-order tensor , obtain which is given by the following
algebraic operation = C : . Express the result in indicial and tensorial notation.
b) Show that and have the same eigenvectors, i.e. the same principal directions.
c) If are the eigenvalues (principal values) of , obtain the eigenvalues of .
Solution:
a)
Tensorial notation:

Indicial notation
ij = C ijkl kl

= C :

= (1 1 + 2 I ) :

)]

= ij kl kl + ik jl kl + il jk kl

= 1 1
: + 2 I{
:
{
Tr ( )

= ij kl + ik jl + il jk kl
= ij kk + ij + ji

sym

( )

= ij kk + 2 ijsym

= Tr ( )1 + 2

= ij kk + 2 ij

where we have considered the symmetry of the tensor = T .


b) and c) Starting from the definition of eigenvalue-eigenvector of a tensor :
n = n

and by substituting the value of obtained previously we can obtain:

(Tr()1 + 2 ) n = n
Tr ( )1 n + 2 n = n
2 n = n Tr ( )n

Tr ( )n + 2 n = n
2 n = ( Tr ( ) )n

Tr ( )
n
n =

n = n

Note that the last equation is the definition of eigenvalue-eigenvector of . With that we
conclude that and have the same eigenvectors (they are coaxial). And the eigenvalues of
can be obtained as follows:
=

Tr ( )
2

If we denote by (1) = 1 , (2) = 2 , (3) = 3 and (1) = 1 , ( 2) = 2 , (3) = 3 . The explicit


form of the above relationship is given by:

University of Castilla-La Mancha


Ciudad Real - Spain

Draft

By: Eduardo W. V. Chaves (2014)

SOLVING PROBLEMS BY MEANS OF CONTINUUM MECHANICS

84

1
0

0
2
0

1
where it is also true that 0
0

0
1
1

0=
0
2
0
3

0
2
0

1 0 0
Tr ( ) 0 1 0

2
0 0 1
3
0
0

1 0 0
1
= Tr ( ) 0 1 0 + 2 0

0 0 1
0
3

0
2

0
0

0
2
0

0
0
3

Problem 1.92
a) Obtain the inverse of the fourth-order tensor C = 2 I + 1 1 where I I sym is the
symmetric fourth-order unit tensor, 1 is the second-order unit tensor, and > 0 and are
scalars. b) Obtain the determinant of C . In addition, if we consider that =

E
,
(1 + )(1 2 )

E
, find the possible values of E and that guarantee that the tensor C is positive
2(1 + )
definite. c) Obtain also the reciprocal of the expression = C : in function of > 0 , ,
where and are symmetric second-order tensors.

Solution:
a) We use the equation obtained in (1.83):
( D + A B ) 1 =

D 1

( + B : D 1 : A )

[(D

: A ) (B : D 1 )

Denoting by D = I , A = B = 1 , = 2 , = , we obtain:
C 1 = ( 2 I + 1 1) 1 =

1 1
I
(I 1 : 1) (1 : I 1 )
1
2
2 ( 2 + 1 : I : 1)

Remember that it holds that I 1 = I , (I 1 : 1) = I : 1 = 1 . Then we obtain the scalar value of


1 : I 1 : 1 = 1 : I : 1 = 1 : 1 = Tr (1) = 3 . We also express in indicial notation:
sym
1 : I 1 : 1 = 1 : I : 1 = ij I ijkl

kl

=
=

ij

1
( ik
2

1
(
2

jj

jl

jj

+ il

jk

kl

1
( ij ik jl
2

kl

+ ij il jk

kl

)=3

Resulting that:
C 1 = ( 2 I + 1 1) 1 =

I
(1 1)
2
2 ( 2 + 3 )

Let us check whether C : C 1 = I sym I holds or not:


1

C : C 1 = ( 2 I + 1 1) :
I
(1 1)
2 ( 2 + 3 )
2

2
2

2
C : C 1 = (
I:I
I : (1 1) +
(1 1) : I
(1 1) : (1 1)
2
2 ( 2 + 3 )
2
2 ( 2 + 3 )

According to Problem 1.27 it fulfills that I : I = I , I : (1 1) = (1 1) : I = 1 1 , and


(1 1) : (1 1) = 3(1 1) . With that we get:

University of Castilla-La Mancha


Ciudad Real - Spain

Draft

By: Eduardo W. V. Chaves (2014)

1 TENSORS

85

2
3 2

(1 1) = I
+

C : C 1 = I +
( 2 + 3 ) 2 2 ( 2 + 3 )
124
44444424444444
3
=0

b) We can directly use the equation (1.37), (see Problem 1.50):

det I sym + A B = 3 + 2 A : B

and by denoting by = 2 , = , A = B = 1 we conclude that:


det (2 I + 1 1) = (2 ) 3 + (2 ) 2 1 : 1 = (2 ) 3 + (2 ) 2 3 = (2 ) 2 (2 + 3 )

The tensor C is definite positive if the eigenvalues are positive numbers, i.e.:
E
>0
2(1 + )
E
E
E
2 + 3 > 0 2
+3
=
>0
2(1 + )
(1 + )(1 2 ) (1 2 )

>0 =

Denoting by y1 = (1 + ) 0 , y2 = (1 2 ) 0 , we can conclude that:

E
E
=
>0
2(1 + ) 2 y1

E > 0

y1 > 0

E < 0
y < 0
1

2 + 3 =

E
E
=
>0
(1 2 ) y 2

E > 0

y 2 > 0

E < 0
y < 0
2

The above conditions must fulfill simultaneously. Then, by means of Figure 1.4 we can
conclude that E > 0 and 1 < < 0.5 .
y ( )
y2 = (1 2 ) 0

zone not feasible


1

1
( y 2 > 0 E > 0)
( y 1 < 0 E < 0)

E >0

0 .5

zone not feasible

y1 = (1 + ) 0

( y 1 > 0 E > 0)
( y 2 < 0 E < 0)

Figure 1.4
c)

University of Castilla-La Mancha


Ciudad Real - Spain

Draft

By: Eduardo W. V. Chaves (2014)

SOLVING PROBLEMS BY MEANS OF CONTINUUM MECHANICS

86

= C:

C 1 : = C 1 : C :

C 1 : = I sym : = sym =

= C 1 :
1

1
=
I
1 1 : =
I:
1 1:
2 ( 2 + 3 )
2
2 ( 2 + 3 )
2

1
=

Tr ( )1
2
2 ( 2 + 3 )

(see Figure 1.5). It is interesting to compare this last equation with Problem 1.41.
x3

33

13

23

22
21

11

33

13

32

31

x1

x3

12

= C:

23

32
22

31

x2

11

12

21

x2

x1

= C 1 :

C 1

Figure 1.5
Problem 1.93
) be a second-order tensor, which is known as the elastic acoustic tensor, and is
Let Q e (N
defined as follows:
) =N
Ce N

Q e (N
is the unit vector and C e is the isotropic symmetric fourth-order tensor and given
where N
e
by C e = (1 1) + 2 I , whose components are: C ijkl
= ij kl + ( ik jl + il jk ) . Obtain
the components of the elastic acoustic tensor.

Solution:
Using symbolic notation we obtain:

( ) (C

) =N
Ce N
= N e
Q e (N
i i

pqrs e p

e q e r e s

) (N e )

C e N e e
= N i C epqrs N j ip sj e q e r = N
p
pqrs s
q
r

) are:
Then, the components of Q e (N

Ce N = N
+ ( + ) N
Qe qr = N
p pqrs s
p
pq rs
pr qs
ps qr
s
N

= pq rsN
p s + (N p pr qsNs + N p ps qrNs ) = NqNr + (NrNq + Ns qrNs )
University of Castilla-La Mancha
Ciudad Real - Spain

Draft

By: Eduardo W. V. Chaves (2014)

1 TENSORS

87

is the unit vector, then N N = 1 holds. With that we obtain:


Note that N
s s

Q e qr = qr + ( + )N q N r

) = 1 + ( + )N
N

Q e (N

tensorial

Problem 1.94
Let Q be a symmetric second-order tensor and given by:
) = 1 + ( + )N
N

Q (N
is the unit vector.
where , are scalars, and N
) and determine the restrictions on and so that there is
a) Obtain the eigenvalues of Q (N
) , i.e. Q 1 .
inverse of Q (N

b) Taking into account that =

E
E
, =
, determine the possible values of
(1 + )(1 2 )
2(1 + )

) is a positive definite tensor.


( E , ) for which Q (N
).
c) Obtain the inverse of Q (N

Solution:

a) It was shown in Problem 1.50 that, given the vectors a and b it holds that:

r r
r r
det 1 + a b = 3 + 2 a b

The eigenvalues can be determined by means of the characteristic determinant


det (Q 1) = 0 , where i are the eigenvalues of Q . Then:

N
1 = 0
det 1 + ( + )N

N
=0
det ( )1 + ( + )N

Denoting by = ( ) and = ( + ) we conclude that:

N
=0
det ( )1 + ( + )N
( ) [( ) + ( + )] = 0
2

N
=0
( ) 3 + ( ) 2 ( + )N
{
( ) [( + 2 ) ] = 0

=1

The above characteristic equation has the following solutions:


( ) [( + 2 ) ] = 0
2

solution

1 =
2
( ) = 0

2 =
[( + 2 ) ] = 0 = ( + 2 )
3

In the principal space of Q , the components of Q are:

Qij = 0
0

( + 2 )
0
0

The inverse of Q exits if the determinant of Q is non-zero:


Q = 2 ( + 2 ) 0

+ 2 0

b) A tensor is definite positive if its eigenvalues are greater than zero, then:

University of Castilla-La Mancha


Ciudad Real - Spain

Draft

By: Eduardo W. V. Chaves (2014)

88

SOLVING PROBLEMS BY MEANS OF CONTINUUM MECHANICS

= 2(1 + ) > 0

E
E
E (1 )
+ 2 =
+2
=
>0

(1 + )(1 2 )
2(1 + ) (2 2 + 1)

(1 + ) 0 1

We check that
1
2
(2 + 1) 0 0.5

Denoting by y1 = (1 + ) 0 , y2 = (1 ) 0 , y3 = (2 2 + 1) 0 , we can rewrite the


restrictions as follows:

E > 0

E
y1 > 0

= 2 y > 0 E < 0
1

y < 0

y2 , y3 > 0

E > 0

y2 , y3 < 0
+ 2 = Ey 2 > 0

y3
y 2 > 0, y 3 < 0

E <0

y 2 < 0, y 3 > 0

with which we obtain:


] 1 ; 0.5[ ] 1 ; [

] ; 1[
y ( )

y2 = (1 ) 0

= 1
E<0

E >0

zone not feasible

E > 0

E < 0

y1 = (1 + ) 0

=1
E >0

= 0 .5

y3 = (2 2 + 1) 0

University of Castilla-La Mancha


Ciudad Real - Spain

Draft

By: Eduardo W. V. Chaves (2014)

1 TENSORS

89

) -components in the principal space of Q (N


) are given by:
c) The inverse of the Q (N

Qij = 0
0

( + 2 )
0

inverse

Qij 1

=0

0
1

) 1 are Q 1 = Q 1 =
Then, the eigenvalues of Q (N
1
2

( + 2 )
0

, Q3 1 =

Q 1 =

1
( + 2 )
2

1
. Recall that a tensor
( + 2 )

and its inverse share the same principal space, i.e. they are coaxial tensors. Moreover, we can
) 1 as follows:
express the spectral representation of Q (N
Q 1 =

1 ( a )
a N

( a ) = Q 1N
(1) N
(1) + Q 1N
(2) N
( 2 ) + Q 1N
( 3) N
(3)
N
1
2
3

a =1

(1) N
(1) + N
( 2) N
( 2 ) ) + Q 1N
( 3) N
(3) = Q 1 (1 N
( 3) N
(3) ) + Q 1N
( 3) N
( 3)
= Q11 (N
3
1
3
N
) + Q 1N
N

= Q 1 (1 N
1

( 3) = N
. It is interesting to see Problem 1.77. Then:
where we have considered that N

N
) + Q 1N
N
=
Q 1 = Q11 (1 N
3
=
=

1
NN+

N
)+
(1 N

1
N

N
( + 2 )


1
1
N
= 1 1 1
NN
N

( + 2 )
( + 2 )

+
NN
1
( + 2 )
1

Ce N
1 N
C e 1 N
, where C e 1 = 1 I
Note that Q 1 = N
2

C
evaluate the tensor Qinv = N

e 1

(1 1) . We
2 ( 2 + 3 )

N :

1
1 1 ( + )
ij kl N l
Nl = N
(Qinv ) jk = N i Cijkl
i
ik jl
il jk
2 (2 + 3 )

2 2
1 1

(Ni ik jlN l + N i il jk N l )
N i ij klN l
(Qinv ) jk =
2 2
2 (2 + 3 )
(Qinv ) jk =

N = 1
(NkN j + N lN l jk )
N
j k
4
2 (2 + 3 )
4

(Qinv ) jk =

jk

jk

N jNk
+

4 2 (2 + 3 )


2 +
N jNk
+
4 ( 2 + 3 )

Thus:
Qinv =

2 +

N N
1 +

4
4
(
2
+
3
)

Note that 0 and (2 + 3 ) 0 , and moreover, these conditions are the same as those to
guarantee that C 1 (see Problem 1.92).

University of Castilla-La Mancha


Ciudad Real - Spain

Draft

By: Eduardo W. V. Chaves (2014)

SOLVING PROBLEMS BY MEANS OF CONTINUUM MECHANICS

90

1.12 Polar Decomposition


Problem 1.95
Let us consider that F has inverse ( det ( F ) 0 ), and that can be decomposed as:
F = Q U = V Q
( a ) , and V has the eigenvalues
If U has the eigenvalues a associated with the eigenvectors N
a associated with the eigenvectors n ( a ) , show that:

a = a
( a ) and n ( a ) .
Obtain also the relationship between the eigenvectors N

Solution:
Based on the definition of eigenvalue and eigenvector of the tensor U :
(a) = N
(a)
UN
a

(here, the index does not indicate summation)

and using the definition of F we can obtain the following relationship:


QT F = QT Q U = QT V Q

QT F = U = QT V Q

and considering the definition of eigenvalue-eigenvector of U :


(a ) = N
(a)
UN
a
(a ) = N
(a)
QT V Q N
a
(a ) = Q N
(a)
Q QT V Q N
a
123
1

we obtain
(a) = Q N
(a)
V Q N
a
V n ( a ) = a n ( a )
( a ) . Furthermore, by comparing the two definitions
where we have assumed that n ( a ) = Q N
of eigenvalue-eigenvector of the tensors U and V , we can verify the they have the same
eigenvalues and different eigenvectors and they are related to each other by the orthogonal
(a) .
transformation n ( a ) = Q N

1.13 Spherical and Deviatoric Tensors


Problem 1.96
Let be a symmetric second-order tensor, and s dev be a deviatoric tensor. Prove that
s:

s
= s . Also show that and dev are coaxial tensors.

Solution: First, we make use of the definition of a deviatoric tensor:


= sph + dev = sph + s =

University of Castilla-La Mancha


Ciudad Real - Spain

Draft

I
1+s
3

s=

I
1.
3

By: Eduardo W. V. Chaves (2014)

1 TENSORS

91

Afterwards we calculate:
I

1
3 [ ] 1 [I ]
s
1
=
=

which in indicial notation is:


s ij
kl

ij
kl

1 [I ]
1
ij = ik jl kl ij
3 kl
3

Therefore
s ij

s ij
kl

= s ij ik

jl

kl ij = s ij ik
3

jl

1
s ij kl
3

ij

1
= s kl
3

To show that two tensors are coaxial, we must prove that

dev

kl

s ii = s kl
{

s:

=0

s
=s

= dev :

I
1
3
I
I
I

= 1 = 1 = 1 = dev
3
3
3

dev = ( sph ) = sph =

Therefore, we have shown that and dev are coaxial tensors. In other words, they have the
same principal directions (eigenvectors).

1.14 Miscellaneous Problems


Problem 1.97
1

Consider that J = [det (b )] 2 = ( III b ) 2 , where b is a symmetric second-order tensor, i.e. b = b T .


Obtain the partial derivatives of J and ln(J ) with respect to b .
Solution:
1

( III b ) 2
1
1 III
1
J
1
1
1

1(
b
=
III b ) 2
= ( III b ) 2 III b b T = ( III b ) 2 b 1 = J b 1
=
b
b
2
b
2
2
2

ln III b 2
[ln( J )]
1 III b 1 1

=
= b
=
b
2 III b b
2
b

1.15 Voigt Notation


Problem 1.98
a) Write the equation = C : in Voigt notation, where C = 1 1 + 2 I is the isotropic
symmetric fourth-order tensor, and the tensors and are structured according to Voigt
notation as follows:

University of Castilla-La Mancha


Ciudad Real - Spain

Draft

By: Eduardo W. V. Chaves (2014)

SOLVING PROBLEMS BY MEANS OF CONTINUUM MECHANICS

92

11

22

{ } = 33
12
23

13

11

22

{ } = 33
2 12
2 23

2 13

b) Write the equation = C 1 : in Voigt notation, where the tensor C 1 (see Problem 1.91)
is given by.
C 1 =

I
11
2
2 ( 2 + 3 )

Solution:
We write the equation = (1 1 + 2 I ) : in indicial notation:

)]

ij = ij kl + 2 ik jl + il jk kl = ij kl + ik jl + il jk kl
2

The second-order unit tensor in Voigt notation is:


1
1

1 0 0
1

Voigt
ij = 0 1 0 {} =
0
0 0 1
0

0

Then, the term (1 1)ij = ij kl in Voigt notation becomes:

I ijkl = ij kl

1
1
1
1


1
1
Voigt

I = [1 1 1 0 0 0] =
0
0
0
0


0
0

[]

The symmetric fourth-order unit tensor I ijkl =

I ijkl

I1111
I
2211
I 3311
Voigt
[I ] =
I1211
I 2311

I1311

1 1 0 0 0
1 1 0 0 0
1 1 0 0 0
T
= {}{}
0 0 0 0 0
0 0 0 0 0

0 0 0 0 0

1
ik jl + il jk in Voigt notation is:
2

I1122
I 2222

I1133
I 2233

I1112
I 2212

I1123
I 2223

I 3322
I1222

I 3333
I1233

I 3312
I1212

I 3323
I1223

I 2322
I1322

I 2333
I1333

I 2312
I1312

I 2323
I1323

I1113 1

I 2213 0
I 3313 0
=
I1213 0
I 2313 0

I1313 0

0 0 0 0 0
1 0 0 0 0
0 1 0 0 0

0 0 12 0 0
0 0 0 12 0

0 0 0 0 12

With these, we can conclude that C = 1 1 + 2 I in Voigt notation becomes:

University of Castilla-La Mancha


Ciudad Real - Spain

Draft

By: Eduardo W. V. Chaves (2014)

1 TENSORS

1
1

1
[C ] =
0
0

1 1 0 0 0
1
0

1 1 0 0 0

0
1 1 0 0 0
+ 2
0 0 0 0 0
0
0

0 0 0 0 0

0 0 0 0 0
0

93

0 0 0 0 0 + 2

1 0 0 0 0
+ 2

0 1 0 0 0

+ 2
=
1
0 0 2 0 0 0
0
0
1

0 0 0 2 0
0
0
0

1
0 0 0 0 2 0
0
0

0
0

0
0

0
0

0
0

0
0
0

0
0

thus

= (1 1 + 2 I ) :

Voigt

0 0 0 11
11 + 2

+ 2

0 0 0 22
22
33

+ 2 0 0 0 33

=
0 0 212
0
0
12 0
23 0
0
0
0 0 2 23

0
0
0 0 2 13
13 0
144444444444244444444444
3

{ } = [C ]{ }

b)
= C 1 :

=
I
1 1 : =
I:
1 1:
2 ( 2 + 3 )
2
2 ( 2 + 3 )
2

Tr ( )1
=
2
2 ( 2 + 3 )
ij =

1
ij
kk ij
2
2 ( 2 + 3 )

Note that:

+
1

11
11
( 11 + 22 + 33 ) 11 =
( 22 + 33 )
+ 3 )
+
2
2 ( 2 + 3 )
(
2
3
)
2
(
2

1
22
=
22
( 11 + 22 + 33 ) 22 =
( 11 + 33 )
2
2 ( 2 + 3 )
2 ( 2 + 3 )
( 2 + 3 )

11 =
22

1
33
33
( 11 + 22 + 33 ) 33 =
( 11 + 22 )
2
2 ( 2 + 3 )
2 ( 2 + 3 )
( 2 + 3 )

1
1
1
=
12
12

( kk ){
2 12 = 12
12 =
2
2 ( 2 + 3 )
2

=0

33 =
12

2 23 =
2 13 =

23

13
Restructuring the above in Voigt notation we obtain:

University of Castilla-La Mancha


Ciudad Real - Spain

Draft

By: Eduardo W. V. Chaves (2014)

SOLVING PROBLEMS BY MEANS OF CONTINUUM MECHANICS

94

( 2 + 3 )

11

2 ( 2 + 3 )

22
33 2 ( 2 + 3 )

=
2 12
0
2 23


0
2 13

2 ( 2 + 3 )
2 ( 2 + 3 )
+

2 ( 2 + 3 )
( 2 + 3 )
+

2 ( 2 + 3 )
( 2 + 3 )

0
1

{ } = [C ] 1 { }

0 11

22

33

0 12
23

0 13

1.16 Tensor-Valued Tensor Function. Tensor Fields.


Problem 1.99
r

Let T ( x , t ) be a symmetric second-order tensor, which is expressed in terms of the position


r
( x ) and time (t ) . Also, bear in mind that the tensor components, along direction x3 , are
equal to zero, i.e. T13 = T23 = T33 = 0 .
r

NOTE: We define T ( x , t ) as a field tensor, i.e. the value of T depends on position and time.
r
r
If the tensor is independent of any one direction at all points ( x ) , e.g. if T ( x , t ) is
independent of the x3 -direction, (see Figure 1.6), the problem becomes a two-dimensional
problem (plane state) so that the problem is greatly simplified.

2D

x2

T
Tij = 11
T12

x2

T12
T22

T22
T22

T12
T12

T12

T11

T11

T11

x1
T12
T22

x3

x1

Figure 1.6: A two-dimensional problem (2D).

University of Castilla-La Mancha


Ciudad Real - Spain

Draft

By: Eduardo W. V. Chaves (2014)

1 TENSORS

95

, T12 in the new reference system ( x1 x 2 ) defined in Figure 1.7.


a) Obtain T11 , T22

b) Obtain the value of so that corresponds to the principal direction of T , and also find
an equation for the principal values of T .
c) Evaluate the values of Tij , (i, j = 1,2) , when T11 = 1 , T22 = 2 , T12 = 4 and = 45 . Also,
obtain the principal values and principal directions.
and T12 ,
d) Draw a graph that shows the relationship between and components T11 , T22
and in which the angle varies from 0 to 360 .

Hint: Use the Voigt Notation, and express the results in terms of 2 .
x2

a11
a ij = a 21
0

x1

x 2

a12
a 22

0 cos
0 = sin
1 0

sin
cos
0

0
0
1

x1

Figure 1.7: A two-dimensional problem (2D).


Solution:
a) Here we can apply the transformation law in Voigt notation {T } = [M] {T } , where
T11
T
22
T
{T } = 33 ;
T12
T23


T13

a11 2

2
a 21
2

[M] = a 31
a 21 a11
a a
31 21
a 31 a11

a12 2
a 22 2
a 32 2
a 22 a12
a 32 a 22
a 32 a12

a13 2
a 23 2
a 33 2
a13 a 23
a 33 a 23
a 33 a13

T11
T
22
T
{T } = 33
T12
T23

T13

2a11 a12
2a 21 a 22
2a 31 a 32
(a11 a 22 + a12 a 21 )
(a 31 a 22 + a 32 a 21 )
(a 31 a12 + a 32 a11 )

2a12 a13
2a 22 a 23
2a 32 a 33
(a13 a 22 + a12 a 23 )
(a 33 a 22 + a 32 a 23 )
(a 33 a12 + a 32 a13 )

2a11 a13

2a 21 a 23

2a 31 a 33

(a13 a 21 + a11 a 23 )
(a 33 a 21 + a 31 a 23 )
(a 33 a11 + a 31 a13 )

For the particular case shown in Figure 1.7, the transformation matrix [M] , after eliminate the
role and column associated with the x3 -direction, becomes:
T11 a11
T = a 2
22 21
T12 a 21 a11

University of Castilla-La Mancha


Ciudad Real - Spain

a12

a 22

a 22 a12

T11

2a 21 a 22
T22
a11 a 22 + a12 a 21 T12

2a11 a12

Draft

(1.102)

By: Eduardo W. V. Chaves (2014)

SOLVING PROBLEMS BY MEANS OF CONTINUUM MECHANICS

96

T = A T AT

x2

T22

x 2

T22

T12

T11

T12

T11

T11

T11

T12

T12

x1

T22

x1

T22

x1

T = AT T A

Figure 1.8: Transformation law for (2D) tensor components.


The transformation matrix, a ij , in the plane, can be evaluated in terms of a single parameter,
:
a11
a ij = a 21
a 31

a12
a 22
a 32

a13 cos
a 23 = sin
a 33 0

sin
cos
0

0
0
1

(1.103)

By substituting the matrix components a ij given in (1.103) into (1.102) we obtain:


2
T11 cos
T = sin 2
22

T12 sin cos

sin 2
cos 2
cos sin

2 cos sin T11

2 sin cos T22


cos 2 sin 2 T12

(1.104)

trigonometric identities,
2 cos sin = sin 2 ,
1

cos
2

1
+
cos
2

cos 2 sin 2 = cos 2 , sin 2 =


, cos 2 =
, (1.104) becomes:
2
2

Making

use

of

the

following

1 + cos 2

T11
T = 1 cos 2

22
2

T12
sin

1 cos 2

sin 2
2

T11
1 + cos 2

sin 2 T22
2

T12
sin 2
cos 2

Explicitly, the above components are given by:

1 + cos 2
1 cos 2
T11 +
T22 + T12 sin 2
T11 =
2
2

1 + cos 2
1 cos 2
=
T11 +
T22 T12 sin 2
T22
2
2

sin 2
sin 2
T12 =
T11 +
T22 + T12 cos 2
2

University of Castilla-La Mancha


Ciudad Real - Spain

Draft

By: Eduardo W. V. Chaves (2014)

1 TENSORS

97

Reordering the previous equation, we obtain:

T11 + T22 T11 T22


+
cos 2 + T12 sin 2
T11 =
2
2

T + T22 T11 T22


= 11

cos 2 T12 sin 2


T22
2
2

T T22
T12 = 11
sin 2 + T12 cos 2

(1.105)

b) Recalling that the principal directions are characterized by the lack of any tangential
components, i.e. Tij = 0 if i j , in order to find the principal directions for the plane case, we
let T12 = 0 , hence:
T T22
T T22
T12 = 11
sin 2 + T12 cos 2 = 0 11
sin 2 = T12 cos 2
2
2

2 T12
2 T12
sin 2
=

tan(2 ) =
cos 2 T11 T22
T11 T22

Then, the angle corresponding to the principal direction is:

2 T12

T11 T22

1
2

= arctan

(1.106)

To find the principal values (eigenvalues) we must solve the following characteristic equation:
T11 T
T12

T12
=0
T22 T

T 2 T ( T11 + T22 ) + T11 T22 T122 = 0

And by evaluating the quadratic equation we obtain:


T(1, 2 )

=
=

[ ( T11 + T22 )]

[ (T11 + T22 )]2


2(1)

T11 + T22

[(T11 + T22 )]2

4(1) T11 T22 T122

4 T11 T22 T122


4

By rearranging the above equation we obtain the principal values for the two-dimensional case
as:
T(1, 2 ) =

T11 + T22
T T22
11

2
2

+ T122

(1.107)

c) We directly apply equation (1.105) to evaluate the values of the components Tij , (i, j = 1,2) ,
where T11 = 1 , T22 = 2 , T12 = 4 and = 45 , i.e.:

University of Castilla-La Mancha


Ciudad Real - Spain

Draft

By: Eduardo W. V. Chaves (2014)

SOLVING PROBLEMS BY MEANS OF CONTINUUM MECHANICS

98

1 + 2 1 2
T11 = 2 + 2 cos 90 4 sin 90 = 2.5

1 + 2 1 2
=

cos 90 +4 sin 90 = 5.5


T22
2
2

1 2
T12 =
sin 90 4 cos 90 = 0.5
2

And the angle corresponding to the principal direction is:


2 T12 2 (4)
=
1 2
T11 T22

1
2

= arctan

= 41.4375 )

The principal values of T ( x , t ) can be evaluated as follows:


T(1, 2 ) =

T11 + T22
T T22
11

2
2

+ T122

T1 = 5.5311

T2 = 2.5311

d) By referring to equation in (1.105) and by varying from 0 to 360 , we can obtain


, T12 , which are illustrated in the following graph:
different values of T11 , T22
x1
T1

= 41.437

T2

x1
= 131.437
2

Components

T22

T1 = 5.5311

T22

T12

T11
0
0

50

100

-2

45

T12

-4

T2 = 2.5311

150

200

250

300

350

T11

x1
= 86.437

-6

TS max = 4.0311

University of Castilla-La Mancha


Ciudad Real - Spain

Draft

By: Eduardo W. V. Chaves (2014)

1 TENSORS

99

Problem 1.100
Obtain the principal values (eigenvalues) and the principal directions (eigenvectors) of the
symmetric part of T , whose components in the Cartesian system are given by:
5 1
Tij =

3 4

(i, j = 1,2)

Solution:
The symmetric part of the tensor is given by:
Tijsym =

5 2
1
Tij + T ji =

2
2 4

The principal values:


5

=0

2 9 + 16 = 0

The solution of the quadratic equation is given by:


(1, 2 ) =

9 (9) 2 4 (1) (16)


2 1

1 T1 = 6.5615

2 T2 = 2.4385

We can draw the Mohr circle (2D) of the tensor T sym :


TSsym

(T11sym , T12sym )
2

TII = 2.4385

TI = 6.5615

T Nsym

For the plane case, the principal direction can be obtained by means of the equation:
tan(2) =

2 T12sym
T11sym

sym
T22

22
=4
54

= 37.982

Problem 1.101
Find the gradient of the function f ( x1 , x 2 ) = cos( x1 ) + exp x1x2 at the point ( x1 = 0, x 2 = 1) .
Solution: By definition, the gradient of a scalar function is given by:
xr f =

University of Castilla-La Mancha


Ciudad Real - Spain

f
f
e1 +
e2
x1
x 2

Draft

By: Eduardo W. V. Chaves (2014)

SOLVING PROBLEMS BY MEANS OF CONTINUUM MECHANICS

100

where:

f
= sin( x1 ) + x 2 exp x1x2
x1

f
= x1 exp x1x2
x 2

xr f ( x1 , x 2 ) = sin( x1 ) + x 2 exp x1x2 e 1 + x1 exp x1x2 e 2 xr f (0,1) = [1] e 1 + [0] e 2 = 2e 1

Problem 1.102
r

Let v and be, respectively, vector and scalar, and twice continuously differentiable. Using
indicial notation, show that:
r

a) xr ( xr v ) = 0
b) xr ( xr ) = xr 2
c) xr ( ) = xr + xr
r

d) xr (v ) = xr v + xr v

e) xr ( A B) = xr A : B + A ( xr B)

( A and B are second-order tensors)

Solution:
a) Considering that
r
r
xr v = ijk v k , j e i

and

xr () =

()
el
xl

(1.108)

then
r
r

xr ( xr v ) =
( ijk v k , j e i ) e l =
( ijk v k , j il ) =
( ljk v k , j ) = ljk v k , jl
x l
x l
x l

(1.109)

Note that ljk is an antisymmetric tensor in lj and vk , jl is a symmetric tensor in lj , thus:


ljk v k , jl = 0

(1.110)

b)
xr ( xr ) =

, j

( ,i e i ) e j =
( ,i ij ) =
= , jj
x j
x j
x j

=
x j

x j

2
2
=
= xr
x 2
j

(1.111)

c)

[ xr ( )]i

= ( ) ,i = ,i + ,i = [ xr ]i + [ xr ]i
r
d) The result of xr (v ) is a scalar which can be expressed as follows:
r
xr (v ) = (v i ) ,i = ,i v i + v i ,i
r
r
= ( xr ) v + ( xr v )

(1.112)

e) Considering that ( A B) ij = A ik B kj , [ xr ( A B)]i = ( A B) ij , j = ( A ik B kj ) , j , thus


( A ik B kj ) , j = A ik , j B kj + A ik B kj , j = [ xr A : B ]i + [A ( xr B)]i

University of Castilla-La Mancha


Ciudad Real - Spain

Draft

By: Eduardo W. V. Chaves (2014)

1 TENSORS

101

Problem 1.103
r

Let a and b be vectors. Show that the following identity xr (a + b) = xr a + xr b holds.


Solution:
r r

, we can express xr (a + b) as follows:


x i

Observing that a = a j e j , b = b k e k , xr = e i
(a j e j + b k e k )
x i

e i

a j
x i

e j e i +

r
r
b k
a
b
e k e i = i + i = xr a + xr b
x i
x i x i

Alternative solution: Working directly with indicial notation we obtain:

r
r r
r
xr (a + b) = (a i + b i ), i = a i , i +b i , i = xr a + xr b

Problem 1.104

Find the components of ( xr a) b .


r

Solution: Bearing in mind that a = a j e j , b = b k e k , xr = e i

( i = 1,2,3 ), the following is


x i

true:

a j
a j
r r (a j e j )
a j

e i (b k e k ) =
( xr a) b =
e j e i (b k e k ) = b k ik
e j = b k
e j
x i

x
x
x

i
i
k

Expanding the dummy index k , we obtain:


a j

bk

x k

a j

= b1

x1

+ b2

a j
x 2

+ b3

a j
x 3

Thus,
j =1

b1

a1
a
a
+ b2 1 + b3 1
x1
x 2
x 3

j = 2 b1

a 2
a
a
+ b2 2 + b3 2
x1
x 2
x 3

j = 3 b1

a 3
a
a
+ b 2 3 + b3 3
x1
x 2
x 3

Problem 1.105
Prove that the following relationship is valid:
r
q 1 r r
1 r
= x q 2 q xr T
T
T T
r r
r
where q( x , t ) is an arbitrary vector field, and T ( x , t ) is a scalar field.
r
x

Solution:
r
r 1 r
q
qi qi
1
1
1
=
= q i ,i 2 q i T,i = xr q 2 q xr T (scalar)
T
T
T
T x i T T ,i T
r
x

University of Castilla-La Mancha


Ciudad Real - Spain

Draft

By: Eduardo W. V. Chaves (2014)

SOLVING PROBLEMS BY MEANS OF CONTINUUM MECHANICS

102

Problem 1.106
Show that:
r

a) rot (a) xr (a) = ( xr a) + ( xr a)


r

r r

r r

(1.113)
r

b) xr (a b) = ( xr b)a ( xr a)b + ( xr a) b ( xr b) a
r

(1.114)

2
c) xr ( xr a) = xr ( xr a) xr a

(1.115)

2
d) xr ( xr ) = xr + ( xr ) ( xr )

(1.116)

Solution:

a) The result of the algebraic operation xr (a) is a vector, whose components are given
by:

[r

r
x

r
(a) i

= ijk (a k ) , j

= ijk ( , j a k + a k , j )
= ijk a k , j ijk , j a k
(1.117)
r
r
r
r
= ( x a) i ijk ( x ) j a k
r
r
r
= ( xr a) i ( xr a) i
r
r
r
r
r
r
with that we check the identity: rot (a) = xr (a) = ( xr a) + ( xr a) .
r r
r r
The components of the vector product (a b) are given by (a b) k = kij a i b j . Then:

[r

r
x

r r
(a b) l = lpk ( kij a i b j ) , p

(1.118)

= kij lpk (a i , p b j + a i b j , p )

b) Considering that kij = ijk , the result of ijk lpk = il jp ip jl and by substituting in the
above equation we obtain:

[r

r
x

r r
(a b) l = kij lpk (ai , p b j + a i b j , p )
= ( il

ip jl )(ai , p b j + ai b j , p )

jp

(1.119)

= il jp a i , p b j ip jl a i , p b j + il jp a i b j , p ip jl a ib j , p
= al , p b p a p , p b l + a l b p , p a p b l , p

Note

that

r r
( xr b) a l = a p b l , p .

[( ar ) br ] = a
r
x

[(

l, pb p ,

r
x

a)b]l
r r

= a p, p b l ,

[(

r
x

b)a]l
r r

= al b p, p ,

c) The components of ( xr a) are given by ( xr a) i = ijk a k , j . Then:


123
ci

[r

r
x

r
r
( xr a) q = qli c i ,l
= qli ( ijk a k , j ) ,l

(1.120)

= qli ijk a k , jl

Considering that qli ijk = qli jki = qj lk qk lj , the above equation becomes:

University of Castilla-La Mancha


Ciudad Real - Spain

Draft

By: Eduardo W. V. Chaves (2014)

1 TENSORS

[r

r
x

r
r
( xr a)

103

= qli ijk a k , jl = ( qj lk qk lj )a k , jl = qj lk a k , jl qk lj a k , jl = a k ,kq a q ,ll

Note that [ xr ( xr a)] q = a k , kq and xr 2 a q = a q ,ll .


r

d)
xr ( xr ) = ( ,i ),i = ,ii + ,i ,i = xr + ( xr ) ( xr )
2

(1.121)

where and are scalar functions.


Another interesting identity originating from the above is:
xr ( xr ) = xr + ( xr ) ( xr )
2

(1.122)

xr ( xr ) = xr + ( xr ) ( xr )
2

Subtracting the two previous identities we obtain:


xr ( xr ) xr ( xr ) = xr xr
2

(1.123)

xr ( xr xr ) = xr xr
2

Problem 1.107

Let be a scalar field, and u be a vector field. a) Show that xr ( xr v ) = 0 and


r

r
r
xr ( xr ) = 0 .
r
r
r
r
r r
r r
r
r r
r
r
b) Show that xr xr v v = ( xr v )( xr v ) + xr ( xr v ) v ( xr v ) ( xr v ) ;
r
r
r
r
r
r
r
c) Referring = xr v , show that xr ( xr 2 v ) = xr 2 ( xr v ) = xr 2 .

[(

Solution:

Regarding that: xr v = ijk v k , j e i

r
r

xr xr v =
ijk vk , j e i e l = ijk
vk , j il = ijk
vk , j = ijk vk , ji
xl
xl
xi
r
The second derivative of v is symmetrical with ij , i.e. v k , ji = v k ,ij , while ijk is antisymmetric

( )

( )

with ij , i.e., ijk = jik , thus:


ijk v k , ji = ij1v1, ji + ij 2 v 2, ji + ij 3 v3, ji = 0
Note that ij1v1, ji = 0 since the double scalar product between a symmetric and an
antisymmetric tensor is zero.
Likewise, we can show that:
r

r
r
xr ( xr ) = ijk , kj e i = 0 i e i = 0

b) Denoting by = xr v we obtain:

[(

) ]

r
r
r r
xr xr v v

r
r r
= xr ( v )

Observing the equation in (1.114), it holds that:

r
r r
r r
r r
r r
r r
xr ( v ) = ( xr v ) ( xr )v + ( xr ) v ( xr v )

University of Castilla-La Mancha


Ciudad Real - Spain

Draft

By: Eduardo W. V. Chaves (2014)

SOLVING PROBLEMS BY MEANS OF CONTINUUM MECHANICS

104

Note that xr = xr ( xr v ) = 0 . Then, we can draw the conclusion that:


r
r r
r r
r r
r r
xr ( v ) = ( xr v ) + ( xr ) v ( xr v )
r
r
r r
r
r r
r
r
= ( xr v )( xr v ) + xr ( xr v ) v ( xr v ) ( xr v )

c) Observing the equation in (1.115) we obtain:

r
r
r r
r
r r
2r
xr v = xr ( xr v ) xr ( xr v ) = xr ( xr v ) xr

Applying the curl to the above equation we obtain:

r
r
r
r
r
r
2r
xr ( xr v ) = xr [ xr ( xr v )] xr ( xr )
14442r 444
3
=0

Referring once again to the equation in (1.115) to express the term xr ( xr ) :

r
r
r
r
r
r
r
2r
2r
2r
xr ( xr v ) = xr ( xr ) = xr ( xr ) + xr = xr xr ( xr v ) + xr
144244
3
=0
r
r
2
r
r
= x ( x v )

Problem 1.108
Show that:
r

a) xr (a b) = ( xr a) b + a ( xr b) rot (a) b + a rot (b)


Solution:
r

(1.124)

The expression xr (a b) is a scalar which can be expressed as follows:


r
r
r r
r r r r
xr (a b) = ( ijk a j b k ) ,i = ijk a j ,i b k + ijk b k ,i a j = ( xr a) b + a ( xr b)
1
1
23
r 2r3
r r
( a) k

( b ) j

Problem 1.109

a) Let T be an arbitrary second-order tensor, obtain the symbolic notation of a.1) ( xr T ) ,


r
r
r
r
a.2) ( xr T )T , a.3) ( xr T T ) , and a.4) ( xr T T )T . a.5) Considering c a constant vector,
show that:

r
r
r
r r r
T
xr ( T c) = ( xr T ) c = c xr T
r
r
b) Obtain the symbolic notation of xr ( xr )T .
r
r
r
u
c.1) Consider the second-order tensor F = r + 1 , prove that xr ( xr F )T = 0 , c.2)
x
r
r
T
r
x F = 0 . c.3) Obtain the explicit expression for xr F components.

Solution:
r

a.1) ( xr T ) =

Tqj

e p Tqj (e q e j ) =
e p e q e j = Tqj , p ipq e i e j = ipq Tqj , p e i e j
x p
x p

a.2) ( xr T )T = ipq Tqj , p e j e i = jpq Tqi , p e i e j


University of Castilla-La Mancha
Ciudad Real - Spain

Draft

By: Eduardo W. V. Chaves (2014)

1 TENSORS

a.3) ( xr T T ) =

105

T jq

e p T jq (e q e j ) =
e p e q e j = ipq T jq , p e i e j
x p
x p

a.4) ( xr T T )T = jpq Tiq , p e i e j


where we have considered the definition e j e k = ijk e i .
r

a.5) Let us considere that a = T c = ( Tqj c j )e q = aq e q , thus:


r
r
a q
a q
r
r

ipq e i = ipq
xr ( T c) = xr a =
e p a q e q =
e i = ipq a q , p e i
x p
x p
x p
ipq a q , p e i = ipq ( Tqj c j ) , p e i = ipq Tqj , p c j e i + ipq Tqj c j , p e i = ipq Tqj , p c j e i
{
=0

where we have considered that c is constant, i.e. c j , p =

c j
x p

= 0 jp .

Note that ipq Tqj , p are the components of ( xr T )ij (see (a.1)), thus

r
r
r
r
r
r r
xr ( T c ) = ipq Tqj , p c j e i = ( xr T )ij c j e i = ( xr T ) c i e i = c ( xr T )T i e i
r
r
( xr T ) c = ipq Tqj , p e i e j c k e k = ipq Tqj , p c k e i jk = ipq Tqj , p c j e i
r
b) We have already shown that ( xr ) = ipq qj , p e i e j , thus

r
r
qj , p

e s ipq qj , p e j e i = ipq
e s e j e i = ipq tsj qj , ps e t e i
xr ( xr )T =
xs
xs
= ( iqp )( tjs ) qj , ps e t e i = iqp tjs qj , ps e t e i = qpi jst qj , ps e t e i

Note that:
r
r
qj , p

xr ( xr ) =
e s ipq qj , p e i e j = ipq
e s e i e j = ipq qj , ps tsi e t e j
xs
xs
= its ipq qj , ps e t e j = ( tp sq tq sp ) qj , ps e t e j

= ( tp sq qj , ps tq sp qj , ps )e t e j = ( sj , ts tj , ss )e t e j
r
r
r
r
r
r
u
r
u r
u r
c.1) Note that xr F = xr r + 1 = xr r + xr (1) = xr r = xr J , where
x

x
x
r
r
r
u
u
we have denoted by J = r . Taking into account qj = J qj = q = uq , j into xr ( xr )T
x
x j

we get:

r
r
xr ( xr J )T = iqp tjs J qj , ps e t e i = iqp tjsuq , jps e t e i

Note that uq , jps = uq , pjs = uq , psj , i.e. it is symmetric in js , and the tensor tjs = tsj is
r

antisymmetric in js , so tjsuq , jps = 0tqp , and xr ( xr F ) T = 0 ti e t e i = 0 .


Alternative solution:
Taking into account that

University of Castilla-La Mancha


Ciudad Real - Spain

Draft

By: Eduardo W. V. Chaves (2014)

SOLVING PROBLEMS BY MEANS OF CONTINUUM MECHANICS

106

it ij is
iqp tjs = qt qj qs = it qj ps + ij qs pt + is pj qt is qj pt qs pj it ps qt ij
pt pj ps
then

iqp tjs Fqj , ps = ( it qj


= it qj

ps u q , jps

+ ij qs

ps

pt

+ is pj

qt

is qj

pt

qs pj it ps qt ij u q , jps

+ ij qs pt u q , jps + is pj qt u q , jps is qj pt u q , jps qs pj it u q , jps ps qt ij u q , jps

= it u j , jss + u s ,its + u t , ppi u j , jti it u s , pps u t ,ipp = 0 ti

Note that it u j , jss = it u p , pss = it u p ,ssp = it us , pps , u s ,its = u j ,itj = u j , jti , ut , ppi = ut ,ipp .
r

c.2) We express xr J T in indicial notation:


r
J qj

e p J qj (e j e q ) =
e p e j e q = J qj , p ipj e i e q
xr J T =
x p
x p
= ipj J qj , p e i e q = ipj uq , jp e i e q = 0 ip e i e q

Note that uq , jp = uq , pj is symmetric in jp meanwhile ipj = ijp is antisymmetric in jp .


r

c.3) We express xr J in indicial notation (see (a.1)):


r
xr J = ipq J qj , p e i e j = ipq u q , jp e i e j

Expanding the term ipquq , jp we get:


ipquq, jp = ip1u1, jp

1
424
3

+ ip 2u2, jp
1
424
3

+ ip 3u3, jp
1
424
3

i11u1, j1 + i12u2, j1 + i13u3, j1


+

i 21u1, j 2 + i 22u2, j 2 + i 23u3, j 2


+
i 31u1, j 3

+
+ i 32u2, j 3

+
+ ip 3u3, j 3

thus
u3,12 u2,13 u3, 22 u2, 23 u3,32 u2,33 J 31, 2 J 21,3
r


( xr J )ij = u1,13 u3,11 u1, 23 u3, 21 u1,33 u3,31 = J 11,3 J 31,1
u2,11 u1,12 u2, 21 u1, 22 u2,31 u1,32 J 21,1 J 11, 2

J 32, 2 J 22,3 J 33, 2 J 23,3

J 12,3 J 32,1 J 13,3 J 33,1


J 22,1 J 12, 2 J 23,1 J 13, 2

Note that
J 13, 2 J 12,3
r

T
r
( x J )ij = J 11,3 J 13,1
J 12,1 J 11, 2

= 0ij

J 23, 2 J 22,3 J 33, 2 J 32,3 u1,32 u1, 23 u2,32 u2, 23 u3,32 u3, 23


J 21,3 J 23,1 J 31,3 J 33,1 = u1,13 u1,31 u2,13 u2,31 u3,13 u3,31
J 22,1 J 21, 2 J 32,1 J 31, 2 u1, 21 u1,12 u2, 21 u2,12 u3, 21 u3,12

Note that, if
=

1
(J + J T )
2

University of Castilla-La Mancha


Ciudad Real - Spain

r
r
r
r
1r
1r
xr ( xr )T = xr ( xr J )T + xr ( xr J T )T = 0
1
424
3
2 1442443 2
=0

Draft

=0

By: Eduardo W. V. Chaves (2014)

1 TENSORS

107

r
u
where J = r .
x

Problem 1.110
r

Let a and v be vectors, show that

r
r r
r
r
( xr v ) a = xr v ( xr v )T

Solution:
r

[r

] ar

If we consider ( xr v ) i = ijk vk , j , then ( xr v ) a s = sip ijk vk , j a p . Note also that the


relationship sip ijk = psi jki = pj sk pk sj holds, then

[(r

r
x

r r
v ) a s = sip ijk v k , j a p = ( pj
= (v s , p v p , s ) a p =

Alternative solution:

pk sj )v k , j a p = ( pj
r
r
r
xr v ( xr v ) T a

{[

sk

sk v k , j

pk

sj v k , j )a p

If we denote by l = xr v , then xr v ( xr v ) T = 2( xr v ) skew = 2l skew . Note that the axial


r
r r
vector associated with the antisymmetric tensor ( xr v ) skew = (v xr ) skew is the vector
r 1 r
r
= ( xr v ) (see Problem 1.37). If we recall the property of an antisymmetric tensor
2
r r r
r
( xr v ) skew a = a , we conclude that

] ar = 12 (r

r r r
r
r
1 rr
( xr v ) skew a = a
x v ( xr v ) T
2
r
r
r
r T r
r
r
r
r
x v ( x v ) a = ( x v ) a

r
x

r
r
v) a

Problem 1.111
r

r r

Let u = u( x ) be a vector field. By means of components of u , a) show that


r
2r
xr u = xr ( xr u)
r
r
xr ( xr u) = 0 .

if

r
r
r
r
xr ( xr u) = 0 ,

b)

show

r
r
r
2r
xr u = xr ( xr u)

that

if

Solution:
We have proven in Problem 1.106 that the following is true:
r
r
r
r
2r
xr ( xr a) = xr ( xr a) xr a

indicial

ilq qjk ak , jl = a j , ji ai , jj

Then, we can obtain

r
r
r r
r
2r
xr ( xr u) xr u = xr ( xr u) xr ( xr u)

indicial

ui , jj = u j , ji ilq qjk uk , jl

Then, it is easy to verify that:

r
r
r r
r
2r
xr ( xr u) xr u = xr ( xr u) xr ( xr u)
1442r 44
3

a)

r
2r
xr u = xr ( xr u)

=0

Components:
ui , jj = u j , ji

ui ,11 + ui , 22 + ui ,33 = u1,1i + u 2, 2i + u3,3i

u1,11 + u1, 22 + u1,33 = u1,11 + u 2, 21 + u3,31

u 2,11 + u 2, 22 + u2,33 = u1,12 + u2, 22 + u3,32


u + u + u = u + u + u
3, 22
3,33
1,13
2 , 23
3,33
3,11
University of Castilla-La Mancha
Ciudad Real - Spain

Draft

u1, 22 + u1,33 = u 2, 21 + u3,31

u 2,11 + u 2,33 = u1,12 + u3,32


u + u
3, 22 = u1,13 + u 2 , 23
3,11

(1.125)

By: Eduardo W. V. Chaves (2014)

SOLVING PROBLEMS BY MEANS OF CONTINUUM MECHANICS

108

Note that in the Cartesian System we have:


r
u = u i e i = u1 e 1 + u 2 e 2 + u 3 e 3
r
r
r
r
u
u u
u
u
u
( xr u) rot (u) = (rot (u) )i e i = 3 2 e 1 + 1 3 e 2 + 2 1 e 3
x 2 x 3
x 3 x1
x1 x 2
14
14
14
42r44
3
42r44
3
42r44
3

= (rot (u ) )1
= (rot (u) )2
= (rot (u) )3
r
r
r
r
r
r
r
r
r (rot (u) )3 (rot (u) )2
(rot (u) )1 (rot (u) )3
(rot (u) )2 (rot(u) )1
e 1 +
e 2 +
e 3

xr ( xr u) =

x2
x3
x3
x1
x1
x2

r
r
(rot (u) )3 (rot (u) )2 u2 u1 u1 u3

x x

x2 x3 x3 x1
x
x

2
1

2
3

r
r
r
r
(rot (u) )1 (rot (u) )3 u3 u2 u2 u1
r

xr ( xr u) i =
=

x
x

x
x
x
x
x
x

3
1
3
2
3
1
1
2

r
r
(rot (u

) )2 (rot (u) )1

u1 u3 u3 u2

x x

x1
x2
1 3 x1 x2 x2 x3
u2,12 u1, 22 u1,33 + u3,13

= u3, 23 u2,33 u2,11 + u1, 21


u u u + u
2, 32
1,31 3,11 3, 22
r
r
r
r
If we are considering that xr ( xr u) = 0 then:

u2,12 u1, 22 u1,33 + u3,13 0 u2,12 + u3,13 = u1, 22 + u1,33


r
r
r


xr ( xr u) i = u3, 23 u2,33 u2,11 + u1, 21 = 0 u3, 23 + u1, 21 = u2,33 + u2,11
u u u + u 0 u + u
1,31
2 ,32
2, 32 = u3,11 + u3, 22
1,31 3,11 3, 22

which are the same conditions as those presented in equation (1.125).


b)

r
r
r
r
r
2r
xr ( xr u) xr u = xr ( xr u) xr ( xr u)
142r 43

r
r
r
2r
xr u = xr ( xr u)

=0

Components
u1,11 + u1, 22 + u1,33 = (u2,12 u1, 22 u1,33 + u3,13 )

ui , jj = ilq qjk uk , jl

u2,11 + u2, 22 + u2,33 = (u3, 23 u2,33 u2,11 + u1, 21 )


u + u + u = (u u u + u )
3, 33
1,31
3,11
3, 22
2 , 32
3,11 3, 22
r
r
And if we consider xr ( xr u) = 0 we get:

[ xr ( xr ur )]i = u1,1i + u 2,2i + u 3,3i = 0 i

u1,11 + u 2, 21 + u 3,31 = 0

u1,12 + u 2, 22 + u 3,32 = 0
u + u
2 , 23 + u 3, 33 = 0
1,13

(1.126)

u 3,31 + u 2, 21 = u1,11

u1,12 + u 3,32 = u 2, 22
u + u
2 , 23 = u 3, 33
1,13

If we replace the above equations into (1.126) we prove that the equality holds.

University of Castilla-La Mancha


Ciudad Real - Spain

Draft

By: Eduardo W. V. Chaves (2014)

1 TENSORS

Problem 1.112

109

Let be a second-order tensor field, and a be a vector field. Show the identities:

a)

r
r
r
xr (a ) = : ( xr a) T + a ( xr )

b)

r
r
r r
r
xr ( a) = a xr T xr a

(1.127)

(1.128)

where is the Levi-Civita tensor (third-order tensor).


Solution:
a)

r
a = a i e i jk e j e k = jk a i pij e p e k

xr (a ) =
jk a i pij e p e k e q =
jk a i pij e p
x q
x q
r
xr (a ) = jk a i pij ,k e p = ( pij jk ,k a i + pij jk a i ,k )e p

kq

jk a i pij e p
x k

Note that pij jk , k ai = pij ( xr ) j ai = pij (a)i ( xr ) j = [a ( xr )]p


r

and pij jk ai , k = pij jk ( xr a)ik = pij ( xr a)ik jk = pij ( xr a) T

] = { : [( ar ) ]}
T

r
x

ij

with that we show the equation in (1.127).


r

Note that if a = x the equation (1.127) becomes:


r
r
r
r
r
xr ( x ) = : [( xr x ) T ] + x ( xr ) = : [1 T ] + x ( xr ) = : T + x ( xr )

b)

r
a = jk e j e k a i e i = jk a i pki e j e p

xr ( a) =
jk a i pki e j e p e q =
jk a i pki e j
x q
x q
r
xr ( a) = jk a i pki , p e j = ( pki jk , p a i + pki jk a i , p )e j

pq

jk a i pki e j
x p

Note that

[r

pki jk , p ai = ipk jk , p ai = xr T

pki jk ai , p = kip ai , p jk = kpi ai , p jk

Problem 1.113

{ [

]}

r r
= a xr T j
r
r
r
r
= ( xr a) k jk = xr a

ij a i

{ [

]}

Consider that xr + p = q , where is a second-order tensor field, and p and q are vector
r r
fields. The equation xr + p = q fulfills at any point of the volume V which is delimitated
by surface S . Show that, if the following equation:
r

r*

x pdV + x t

r r
dS = x qdV

is also valid, then = T holds. Consider that t * = n where n is the outward unit normal
to surface S .
Solution:
University of Castilla-La Mancha
Ciudad Real - Spain

Draft

By: Eduardo W. V. Chaves (2014)

SOLVING PROBLEMS BY MEANS OF CONTINUUM MECHANICS

110

r r
r r
r r
x pdV + x t * dS = x qdV

r r
r
r r
x pdV + x ( n )dS = x qdV

Note that ( x t * ) i = ijk x j t *k = ijk x j ( n ) k = ijk x j kp n p = ( x ) ip n p = ( x ) n , with


that we get:
r r
r
r r
x pdV + ( x ) n dS = x qdV

By applying the divergence theorem to the surface integral we get:


r r
r
r r
x pdV + xr ( x )dV = x qdV

It was proven in Problem 1.112 that xr ( x ) = : T + x ( xr ) , and by replacing it


into the above equation we obtain:
r r
r
r r
x pdV + [ : T + x ( xr )]dV = x qdV

r
r r
r
r r
[ x p + : T + x ( xr ) x q]dV = 0

r
r r
r
{ x [( xr ) + p q] + : T }dV = 0
144
42r 444
3

=0

r
: T dV = 0

Note that ( : T ) i = ijk kj = . ikj kj = 0 i , i.e. the tensor is antisymmetric in kj , since the
double scalar product between a symmetric and an antisymmetric tensor is zero, then we
prove that T = is symmetric. We can also prove that by means of components:
1 jk kj = 0

2 jk kj = 0

3 jk kj = 0

123 32 132 23 = 0

32 23 = 0

213 31 23113 = 0

31 13 = 0

312 21 32112 = 0

21 12 = 0

32 23
31 13
21 12

Problem 1.114
a) Show that

r
r
r
r
r
r
r
xr {[ xr ]T x} = { xr [ xr ]T } x ( : ( xr )) 1 + [ xr ]
r
where is a second-order tensor, x is the vector position, is the Levi-Civita tensor (thirdorder tensor), and 1 is the second-order unit tensor. b) Simplify the above equation when
= T .

Solution:
T

r
r

T
r
r

e i
e j pq e p e q x k e k
x {[ x ] x} =
xi
x j

pq
e i
tjp e t e q x k e k
=
x
xi

University of Castilla-La Mancha


Ciudad Real - Spain

Draft

By: Eduardo W. V. Chaves (2014)

1 TENSORS

111

Applying the transpose property we get:


pq

r
r

pq

r

e i
e i
x k tjp stk e q e s
xr {[ xr ]T x} =
tjp e q e t x k e k =
xi
x j

x j
xi

xi

pq

pq

x k tjp stk niq e n e s = niq tjp stk


x k e n e s
x j

x i x j

pq

= niq tjp stk


xi x j

2
pq
= niq tjp stk
x
xi x j k

pq
e e
x k + x k
s

x
x
i
j

pq
e e
+ ki
n
s
x j

2 pq
pq

e e
= niq tjp stk
x k + nkq tjp stk
s

x
x
x
i
j
j

Note that the term nkq tjp stk

pq
x j

= nkq tjp stk pq , j = nqk tjp stk pq , j can be expressed as

follows:
nkq tjp stk pq , j = nqk tjp stk pq , j = ( ns qt nt qs ) tjp pq , j = ns qt tjp pq , j + nt qs tjp pq , j
= ns tjp pt , j + njp ps , j = ptj pt , j
= ptj pt , j

ns

+ njp ps , j

+ njp ps , j = ( ptj pt ) , j ns + njp ps , j


r
r
= ( : ( xr )) ns + [ xr ] ns = ( xr ( : )) ns + [ xr ] ns
r
r
= {( : ( xr )) 1 + [ xr ]}ns = {( xr ( : )) 1 + [ xr ]}ns
ns

and
niq tjp stk

r
r
r
xk = niq tjp stk pq ,ij xk = { xr [ xr ]T } x

xi x j
ns

2 pq

b) If = T we can show that : ( xr ) = xr ( : ) = 0 , then


r
r
r
r
r
r r
xr {[ xr ]T x} = { xr [ xr ]T } x + [ xr ]

Problem 1.115
r

r r

Let v be a vector field in function of x , i.e. v = v ( x ) , whose components are given by:
v1 = x1 5 x 2 + 2 x3

v 2 = 5 x1 + x 2 3 x3
v = 2 x + 3 x + x
1
2
3
3
r
r
a) Obtain the gradient of v ; b) Obtain xr v : 1 ; c) Apply the additive decomposition of the
r
tensor xr v into a symmetric and antisymmetric parts; d) Obtain the axial vector associated
r
with the antisymmetric tensor ( xr v ) skew .

Solution: a)
University of Castilla-La Mancha
Ciudad Real - Spain

Draft

By: Eduardo W. V. Chaves (2014)

SOLVING PROBLEMS BY MEANS OF CONTINUUM MECHANICS

112

xr v

v1

x1
vi v 2
r
components
( xr v ) ij =
=
x j x1

v3
x1

r
v
= r
x

v1

x 3 1 5 2
v 2
= 5
1 3

x 3

1
v 3 2 3
x 3

v1
x 2
v 2
x 2
v3
x 2

b) xr v : 1 = Tr ( xr v ) = 1 + 1 + 1 = 3
r

c) xr v = ( xr v ) sym + ( xr v ) skew =

] [

r
r
r
r
1
1
( xr v ) + ( xr v ) T + ( xr v ) ( xr v ) T
2 44424443 1
2 44424443
1
r
=( xr v ) sym

r
=( xr v ) skew

Then, the components of ( xr v ) sym and ( xr v ) skew are given, respectively, by:
r
( xr v ) ijsym

1 0 0
1 vi v j

=
+
= 0 1 0
2 x j xi
0 0 1

r
( xr v ) ijskew

0 5 2
1 vi v j
0 3
=

= 5
2 x j x i
2 3
0

d) Remember that

r
(W) ij ( xr v ) skew

0
= W21
W31

ij

W12
0
W32

viskew
,j

1 v
= 2
2 x1
1 v
3
2 x1

W13 0
W23 = W12
0 W13

v1

x 2
v1

x3

W12
0
W23

1 v1 v 2

2 x 2 x1

1 v1 v 3

2 x 3 x1
1 v 2 v 3

2 x3 x 2

0
1 v 3 v 2

2 x 2 x3

(1.129)

w3
0
w1

W13 0
W23 = w3
0 w2

w2
w1
0
r
where w1 , w2 , w3 are the components of the axial vector w associated with the
r
antisymmetric tensor W ( xr v ) skew , then, to the proposed problem we have:
w1 = 3

w2 = 2
w = 5
3

w2 0 5 2

w1 = 5
0 3
0
w1
0
0 2 3
r
The axial vector, in the Cartesian basis, is w = 3e 1 + 2e 2 + 5e 3 .
0
w
3
w2

w3

Alternative solution d) Resorting to Problem 1.37 where we have shown that


r

1 r r
(a x ) is the
2

axial vector associated with the antisymmetric tensor ( x a ) skew . Then, the axial vector
associated with the antisymmetric tensor

r skew
r
r
( xr v ) skew = (v ) ( xr )

is the vector

r 1 r
r
w = xr v .
2

University of Castilla-La Mancha


Ciudad Real - Spain

Draft

By: Eduardo W. V. Chaves (2014)

1 TENSORS

e 1
r 1
w=
2 x1
v1
=

e 2

x 2
v2

e 3
v

1 v
= 3 2
x3 2 x 2 x3
v3

113

v
v
v
v
e 1 3 1 e 2 + 2 1
x1 x 2

x1 x 3


e 3

1
(3 (3) )e 1 ((2) (2) )e 2 + (5 (5))e 3 = 3e 1 + 2e 2 + 5e 3
2

Problem 1.116
r

Let l = xr v be a second-order tensor. Considering that D = ( xr v ) sym and W = ( xr v ) skew ,


r
r
show that W D + D W = 2(D W) skew = ( xr v xr v ) skew = ( l l ) skew .
Solution:
l

In Problem 1.35 we have shown that: given an arbitrary second-order tensor


l

skew

sym

+l

sym

skew

= 2( l

skew

it fulfills that

sym skew

Then, W D + D W = 2(D W ) skew holds. Taking into account the definition of symmetry and
Antisymmetry, i.e. D =

1
l +l
2

] , W = 12 [l l ] , we can conclude that:

skew
2
1
= l l + l l
(l + l T ) (l l T )
4
2
skew
skew
1
1
T
T
l l l l
= 1
+ l l l T l T
4
4
4
2
4
4
4
3
2
2

W D + D W = 2(D W ) skew =

=0

1
T
l l (l l )
2

skew

Obs.: Note that the resulting tensor


(l l

l T ]

skew

1
2( l l ) skew
2

skew

r
r
= ( l l ) skew = ( xr v xr v ) skew

l T l T l is symmetric, since:
l T l )T = l l T l T l .
l

Problem 1.117
Consider the scalar J = F det(F ) and an arbitrary second-order tensor given by
l

r
dF
= xr v = F& F 1 , where F&
dt

represents the time derivative of F . Show that the

following is true:
r
d(J ) &
J = J ( xr v )
dt

(1.130)

dA

dA
= A Tr
A 1 holds, where A = A ()
d
d

is an arbitrary second-order tensor and a scalar. Making A = F and = t , we can obtain:

Solution: In Problem 1.87 we have shown that

dF
dt

( )

dJ
dF
= F Tr
F 1 = J Tr F& F 1 = J Tr l = J Tr l
dt
dt

sym

) = J Tr(

r
xv

) = J ( xr vr )

Alternative solution:
In Problem 1.46 we have shown that given a second-order tensor F the relationship
F tpq = rjk Frt F jp Fkq holds, and if we take the time derivative of it we get:
University of Castilla-La Mancha
Ciudad Real - Spain

Draft

By: Eduardo W. V. Chaves (2014)

SOLVING PROBLEMS BY MEANS OF CONTINUUM MECHANICS

114

DF
D
tpq =
( rjk Frt F jp Fkq ) = rjk F&rt F jp Fkq + rjk Frt F& jp Fkq + rjk Frt F jp F&kq
Dt
Dt

(1.131)

According to the problem statement we have l = F& F 1 F& = l F , with that the
following relations F&rt = l rs Fst , F& jp = l js Fsp and F&kq = l ks Fsq hold, and the equation in (1.131)
can be rewritten as follows:
DF
tpq = rjk F&rt F jp Fkq + rjk Frt F& jp Fkq + rjk Frt F jp F&kq
Dt
= rjk l rs Fst F jp Fkq + rjk Frt l js Fsp Fkq + rjk Frt F jp l ks Fsq

We multiply both sides of the above equation by ut v p w q we get:


DF
Dt

tpq u t v p w q = rjk l rs Fst F jp Fkq u t v p w q + rjk Frt l js Fsp Fkq u t v p w q + rjk Frt F jp l ks Fsq u t v p w q
= rjk ( l rs Fst u t )( F jp v p )( Fkq w q ) + rjk ( Frt u t )( l js Fsp v p )( Fkq w q )
+ rjk ( Frt u t )( F jp v p )( l ks Fsq w q )
= rjk ( l rs a s )(b j )(c k ) + rjk (a r )( l js b s )(c k ) + rjk (a r )(b j )( l ks c s )

where we have denoted by a s = Fst ut , b j = F jp v p , c s = Fsq w q . The above equation in tensorial


notation becomes:

] [

r r r r
DF r r r
r r r r
r
r r r
u ( v w ) = ( l a) (b c ) + a ( l b) c + a b ( l c ) = Tr ( l ) a (b c )
Dt

where we have used the property of trace (see Problem 1.48). The above equation can also be
written as follows:

DF r r r
r r r
r
r
r
r r r
u ( v w ) = Tr ( l ) a (b c ) = Tr ( l ){( F u) [( F v ) ( F w ) ]} = Tr ( l ) F u ( v w )
Dt

where we have used the property of determinant (see Problem 1.49), with that we conclude
that

DF
= Tr ( l ) F .
Dt

Problem 1.118
r
Let us consider a vector field represented by the unit vector field b ( x ) , (see Figure 1.9).
r
r
r
Obtain the second-order projection tensor P such that p = P u holds, where u is an

r
r
arbitrary vector and p is orthogonal to the field defined by b ( x ) .
r
b ( x )

Figure 1.9: Vector field.


University of Castilla-La Mancha
Ciudad Real - Spain

Draft

By: Eduardo W. V. Chaves (2014)

1 TENSORS

115

Solution:
The proposed problem can be represented by the following figure:

r
u

r r
r
u b = p = P u

r r
a = u // b
r
b ( x )

And, by considering the vector summation we obtain u = a + p . In addition, the vector a can

r r
r
r
be obtained by means of the projection of u onto the direction b : a = a b = (u b ) b , note
r

also that a = (u b ) b = u (b b ) . With that we can obtain:


r r r
p =ua
r r
r r
= u (u b ) b = u (u b ) b
r
r
= 1 u (b b ) u
r
= 1 (b b ) u
r
= P u

p i = ui ai
= u i (u k b k )b i
= u u b b
k

= ( ik

ik

b k b i )u k

= Pik u k

Thus, we conclude that the projection second-order tensor is given by:


P = 1 b b

The same result could have been obtained by means of vector product. We draw the proposed
problem from another perspective for better visualization:
r
u b

r
b ( x )

r
u
r
b (u b )
r

Taking into account that a (b a) = [(a a)1 a a] b , (see Problem 1.17), we can obtain
r

r
b (u b ) = (b b )1 b b

r r

] ur = [1 b b ] ur = pr .

Then we can present a vector as follows:

r r r
r
r
u = u // b + ub = (b b ) u + 1 (b b ) u

r
r
r
r
where u// b = (b b ) u is the vector parallel to b -direction and ub = 1 (b b ) u is the
perpendicular one.

University of Castilla-La Mancha


Ciudad Real - Spain

Draft

By: Eduardo W. V. Chaves (2014)

SOLVING PROBLEMS BY MEANS OF CONTINUUM MECHANICS

116

Problem 1.119

r r

Given a vector field v ( x ) , show that the following relationship holds:


r r 1
r r
r
( xr v ) v = xr (v 2 ) v ( xr v )
2
r
r
r r
where v = v is the module of v , so v 2 = v v .

Solution:

r r
r
r
1 r 2
1
1
1
x (v ) i = [ xr (v v )]i = (v k v k ) ,i = (v k ,i v k + v k v k ,i ) = v k v k ,i = (v xr v ) i .
2
2
2
2
r
r
At one point of the vector field v , we consider a plane normal to v and recalling that the
r
projection of a second-order tensor onto a direction ( v ) is a vector which does not necessary
r
r r
have the same direction as ( v ), with that we represent the following vectors xr v v and
r
r
v xr v :

Note that

r r
xr v v
r
r
( xr v )

r
v

r
r
v xr v

r r
cv
r
r
r
c ( xr v )

r
r r
r
c = v ( xr v )

Note that, by means of summation of vectors we can obtain:


r r
r r
r r r r
r
r
r r
r r
r
( xr v ) v + c = v ( xr v )

c = v ( xr v ) ( xr v ) v

c = v ( xr v ) v ( xr v ) T
r r
r
r
r
r
c = v (( xr v ) ( xr v ) T ) = v 2( xr v ) skew
r
r
If we consider that w is the axial vector associated with the antisymmetric tensor ( xr v ) skew , it
r
r r r
r
r
r r
fulfills that: ( xr v ) skew v = w v v ( xr v ) skew = v w . In addition, the relationship
r
r
r
r
rot (v ) xr v = 2 w holds. Then,
r
r r
r
r
r r
r
(1.132)
c = v 2( xr v ) skew = v 2 w = v ( xr v )

with that we conclude that:

r r r r
r
r r r
r r
( xr v ) v + c = v ( xr v )

( xr v ) v = v ( xr v ) c
r r 1
r r
r
( xr v ) v = xr (v 2 ) v ( xr v )
2
r
r
r
It is interesting to note that: when ( xr v ) is a symmetric tensor, i.e. ( xr v ) = ( xr v ) sym , the
r
r r
r
r r
r r r
r
following is fulfilled ( xr v ) skew = 0 , c = 0 , ( xr v ) = 0 , and ( xr v ) v = v ( xr v ) has the
r
same direction as v .

University of Castilla-La Mancha


Ciudad Real - Spain

Draft

By: Eduardo W. V. Chaves (2014)

1 TENSORS

117

When ( xr v ) = ( xr v ) skew we have that c = v 2( xr v ) skew = 2v ( xr v ) , (see equation (1.132)).


r
r
r
r r
r
r
With that, v xr v = xr v v holds, and the vector v is perpendicular to the vector ( xr v ) ,
(see Figure 1.10).
r
r
( xr v ) = ( xr v ) skew

r r
xr v v

r r
cv
r
r
r
c ( xr v )

r
v
r
r
( xr v )

r
r
v xr v

r
r r
r
r
r
c = v ( xr v ) = 2v ( xr v )

Figure 1.10
Alternative solution:
r r
r
r
r
r
r
r
r
xr v v = (( xr v ) sym + ( xr v ) skew ) v = ( xr v ) sym v + ( xr v ) skew v
r
r
r
r
r
r
r
r
= ( xr v ) sym v + ( xr v ) skew v + (( xr v ) skew v ( xr v ) skew v )
r
r
r
r
r
r
= (( xr v ) sym v ( xr v ) skew v ) + 2( xr v ) skew v
r
r
r
r
r
r
r
1
= ( xr v + ( xr v ) T ) ( xr v ( xr v ) T ) v + 2( xr v ) skew v
2
r
r
r
r r
r
r
r
1
= (2( xr v ) T ) v + 2( xr v ) skew v = v ( xr v ) + 2( xr v ) skew v
2
r r
r
1
= xr (v 2 ) v ( xr v )
2
r
r
r
r
r
r
r
r
r
skew
Remember that ( xr v ) T = ( xr v ) skew , thus 2( xr v ) skew v = v 2( xr v ) skew = v ( xr v )

Problem 1.120
r r

Let u( x ) be a stationary vector field. a) Obtain the components of the differential du . b)


r r
Now, consider that u( x ) represents a displacement field, and is independent of x3 . With
these conditions, graphically illustrate the displacement field in the differential area element
dx1 dx 2 .

University of Castilla-La Mancha


Ciudad Real - Spain

Draft

By: Eduardo W. V. Chaves (2014)

SOLVING PROBLEMS BY MEANS OF CONTINUUM MECHANICS

118

Solution: According to the differential and gradient definitions, it holds that:


r r
u( x )

r r r
r r r
du u( x + dx ) u( x )
r
r
r
du = ( xr u) dx

x2

r
x

r
dx

r r
r
u( x + dx )

r
r
x + dx

x1
x3

Thus, the components are defined as:

du i =

u i
dx j
x j

u1

du1 x1
du = u 2
2 x
du 3 1
u 3
x1

u1
x 2
u 2
x 2
u 3
x 2

u1

x3 dx
1
u 2

dx
2
x3

u 3 dx3
x3

or:

u1
u
u
dx1 + 1 dx 2 + 1 dx3
du1 =
x1
x 2
x3

u 2
u
u
dx1 + 2 dx 2 + 2 dx3
du 2 =
x3
x1
x 2

u
u
u
du 3 = 3 dx1 + 3 dx 2 + 3 dx 3

x1
x 2
x3

with
du1 = u1 ( x1 + dx1 , x 2 + dx 2 , x3 + dx3 ) u1 ( x1 , x 2 , x3 )

du 2 = u 2 ( x1 + dx1 , x 2 + dx 2 , x3 + dx3 ) u 2 ( x1 , x 2 , x3 )
du = u ( x + dx , x + dx , x + dx ) u ( x , x , x )
3
1
1
2
2
3
3
3
1
2
3
3

As the field is independent of x3 , the displacement field in the differential area element is
defined as:
u1
u1

du1 = u1 ( x1 + dx1 , x 2 + dx 2 ) u1 ( x1 , x 2 ) = x dx1 + x dx 2

2
1

u
u

du = u ( x + dx , x + dx ) u ( x , x ) = 2 dx + 2 dx
2
1
1
2
2
2
1
2
1
2
2
x1
x 2

or:
u1
u1

u1 ( x1 + dx1 , x 2 + dx 2 ) = u1 ( x1 , x 2 ) + x dx1 + x dx 2

2
1

u ( x + dx , x + dx ) = u ( x , x ) + u 2 dx + u 2 dx
2
1
1
2
2
2
1
2
2 1
x1
x 2

University of Castilla-La Mancha


Ciudad Real - Spain

Draft

By: Eduardo W. V. Chaves (2014)

1 TENSORS

119

Note that the above equation is equivalent to the Taylor series expansion taking into account
only up to linear terms. The representation of the displacement field in the differential area
element is shown in Figure 1.11.
u2 +

u 2
dx 2
x 2

u2 +

( x1 + dx1 , x 2 + dx 2 )

( x1 , x 2 + dx 2 )
u1 +

u 2
u
dx1 + 2 dx 2
x1
x 2

u1
dx 2
x 2

u1 +

r
du

dx 2

u1
u
dx1 + 1 dx 2
x1
x 2

u2 +

(u 2 )

( x1 + dx1 , x 2 )

( x1 , x 2 )

x2

u 2
dx1
x1

u1 +

(u1 )

u1
dx1
x1

dx1

x1

144444444444444444424444444444444444443

=
644444444444444444474444444444444444448
x 2 ,u 2
u2 +

u1
dx2
x2

u 2
dx2
x2

dx 2

O
u2

O
u1
u1 +

dx 2
A
O

dx1

u 2
dx1
x1

dx1

u1
dx1
x1

x1 ,u1

Figure 1.11: Displacement field in the differential area element.

University of Castilla-La Mancha


Ciudad Real - Spain

Draft

By: Eduardo W. V. Chaves (2014)

SOLVING PROBLEMS BY MEANS OF CONTINUUM MECHANICS

120

Problem 1.121

Given a second-order tensor field T ( x ) . Show that: if there is no source of the field T ( x ) it
r

fulfills that the divergence of T ( x ) is equal to zero, i.e. xr T = 0 . For the demonstration,
consider the tensor field in a differential volume element dV = dx1 dx 2 dx 3 in the Cartesian
system.
Solution:

Let us set the tensor field T ( x ) in the differential volume element. For this purpose, we start
r
from the definition of the differential of T ( x ) which is defined by means of gradient as
follows:
r
r
r
dT T ( x + dx ) T ( x )
r
r
r
r
r
r
r
r
T ( x + dx ) T ( x ) = ( xr T ) dx T ( x + dx ) = T ( x ) + ( xr T ) dx
r

dT = ( xr T ) dx

The above equation in indicial notation becomes:

r
r
r
Tij ( x + dx ) = Tij ( x ) + Tij , k dx k
r
= Tij ( x ) + Tij ,1 dx1 + Tij , 2 dx 2 + Tij ,3 dx 3

Tij
Tij
r Tij
dx1 +
dx 2 +
dx 3
= Tij ( x ) +
x1
x 2
x 3
r
r
The representation of the field components Tij ( x + dx ) can be appreciated in Figure 1.12.
r

Note that on the face normal to x1 + dx1 act the components Ti1 ( x ) +

Ti1
dx1 , since
x1

according our convention, the first index indicate the direction in which points out and the
second index indicates the normal plane.
r

Once established the field Tij ( x + dx ) in the differential volume element, we apply the total
r
r
balance of the field components Tij ( x + dx ) according to the directions x1 , x 2 , x3 .
r

Total balance of Tij ( x + dx ) in dV according to x1 -direction is equal to zero (there is no


source):

T
T
T
T11 + 11 dx1 dx 2 dx3 + T13 + 13 dx 3 dx1 dx 2 + T12 + 12 dx 2 dx1 dx3 T11 dx 2 dx3
x1
x3
x 2

T13 dx1 dx 2 T12 dx1 dx3 = 0

By simplifying the above equation we obtain:


T
T11
T
dx1 dx 2 dx3 + 13 dx3 dx1 dx 2 + 12 dx 2 dx1 dx3 = 0
x1
x3
x 2

T11 T12 T13


=0
+
+
x3
x 2
x1

Similarly, according to the directions x 2 and x3 we will obtain, respectively:


T21 T22 T23
+
+
=0
x1
x 2
x3

University of Castilla-La Mancha


Ciudad Real - Spain

and

Draft

T31 T32 T33


+
+
=0
x1
x 2
x3

By: Eduardo W. V. Chaves (2014)

1 TENSORS

121

x3

Rear face

T11
T33 +

Rear face

T23 +
T13 +

T21

T33
dx3
x3

T13
dx3
x3

T23
dx3
x3

T32 +

T12

T22

T31 +

T31
dx1
x1

T32

T12 +
T21 +

T
T11 + 11 dx1
x1

T21
dx1
x1

T31
dx 3

T32
dx2
x2

T12
dx2
x2

T22 +

T22
dx2
x2

x2

dx1

T13
T23

x1

Rear face

T33
dx 2

Figure 1.12: Tensor field components in the differential volume element.


Then, we have the following set of equations that must be met simultaneously:
T11 T12 T13
+
+
=0

x
x
x
1
2
3

T21 T22 T23


+
+
=0

x
x
x
1
2
3

T31 T32 T33


+
+
=0

x1
x 2
x3

T11,1 + T12, 2 + T13,3 = 0

T21,1 + T22, 2 + T23,3 = 0

T31,1 + T32, 2 + T33,3 = 0

T1 j , j = 0

T2 j , j = 0

T3 j , j = 0

Tij , j = 0 i

Thus demonstrating that in the absence of source, the divergence is zero:


Tij , j = 0 i

( xr T ) i = 0 i

tensorial

r
xr T = 0

NOTE 1: If we have a tensor field, the order of the source is a minor order of the tensor, e.g.
the source of a vector field is represented by a scalar field.
NOTE 2: If the divergence of a tensor field is positive we have a source, on the contrary if
the divergence is negative we have a sink.

University of Castilla-La Mancha


Ciudad Real - Spain

Draft

By: Eduardo W. V. Chaves (2014)

SOLVING PROBLEMS BY MEANS OF CONTINUUM MECHANICS

122

Problem 1.122
Show that:

[( xr T ) ur ] ar = [ xr ( T ar )] ur

(1.133)

r
r r r
r
where T = T (x ) is a second-order tensor field, u = u( x ) is a vector field, and a an arbitrary
r
vector (independent of ( x ) ).
r r
Solution: Note that the term [( xr T ) u] a is a vector, which in indicial notation becomes:
{[( xr T ) ur ] ar}i = [( xr T ) ur ]ik (ar )k = ( xr T )ikp u p ak = Tik , pu p ak = Tik , pu p ak
(1.134)
r r
Now we express the term [ xr (T a )] u in indicial notation:
r
r
r
gradient
( T a )i = Tik ak

[ xr ( T a )] ij = ( T a )i , j = ( Tik ak ), j
r
[ xr ( T a )] ij = ( Tik ak ), j = Tik , j ak + Tik ak , j = Tik , j ak
{

r
r
or [ xr ( T a )] ij = a ( xr T T )

=0 k , j

r
r
+ [ T ( xr a )]ij = a ( xr T T )
123

ij

(1.135)

ij

= Tik , j ak

=0

r
r
where we have considered that a is independent of ( x ) . If we apply the scalar product
r
between the above equation and u we obtain:
{[ xr ( T ar )] ur}i = [ xr ( T ar )]ij u j = Tik , j a k u j = Tik , p u p a k
(1.136)

If we compare (1.134) with (1.136) we show (1.133).


r

r r

Not that, if a = a (x ) is dependent of x and according to the equation in (1.135) we conclude


that:

[ xr ( T ar )] ij = ( Tik a k ), j = Tik , j a k + Tik a k , j

[ xr ( T ar)] ij = [ar ( xr T T )] ij + [ T ( xr ar )] ij

Problem 1.123
r

Show that if the magnitude of a vector, = (t ) , is constant with time, this implies that is
r
d
orthogonal to
at any time t .
dt

Solution:
r

We start from the definition of the magnitude of a vector, where = holds, thus:

( ) = d (r r ) = d (r ) r + r d (r ) = 2r d (r ) = 0

r
d

dt

dt

dt

dt

dt

r
r d

dt

NOTE: A particular case of this problem is the circular motion in which:


r
v

r
x = constant
r
r dx

r
x
r dx
dt
v=

dt

r
x

University of Castilla-La Mancha


Ciudad Real - Spain

Draft

By: Eduardo W. V. Chaves (2014)

1 TENSORS

123

1.17 Theorems Involving Integrals


Problem 1.124

Check the divergence theorem (Gauss theorem) for the vector field F whose Cartesian
components are given by Fi = xi + ( x32 x 3 ) i 3 . Consider the boundary defined by the
cylinder x12 + x 22 1 , 0 x3 1 .
Solution:
The divergence theorem states that:

r
r
xr F dV = F n dS

where n is the normal to the surface and points outwards.


x3

x12 + x 22 1

S (2)

n ( 2 )

r
r

r
r =1
h =1

S (1)

n (1)

r
x

x2
n (3)

x1

S ( 3)

Figure 1.13
r

Calculation of xr F dV :
V

r
xr F = Fi ,i = x i + ( x 32 x 3 ) i 3

,i

= x i ,i + ( x 32 x 3 ) ,i i 3 = ii + ( x 32 x 3 ) ,3 = 3 + (2 x 3 1) = 2 x 3 + 2

Thus

r
xr F dV = (2 x3 + 2) dV =

x3 =1

A x =(02 x

+ 2)dx3 dA = 3 dA = 3(r 2 ) = 3

where A is the area defined by the circle x12 + x 22 1 .


Calculation of

F n dS
S

University of Castilla-La Mancha


Ciudad Real - Spain

Draft

By: Eduardo W. V. Chaves (2014)

SOLVING PROBLEMS BY MEANS OF CONTINUUM MECHANICS

124

We decompose the boundary in three areas, namely: S (1) , S ( 2) , S (3) , (see Figure 1.13), then
r

F n dS = F n

(1)

S ( 1)

F n

dS (1) +

( 2)

dS ( 2 ) +

S ( 2)

F n

( 3)

dS (3)

S ( 3)

r
The components of F are: F1 = x1 + ( x32 x3 ) 13 = x1 , F2 = x 2 , F3 = x3 + ( x32 x 3 ) 33 = x32 .
r
r
The representation of F in the Cartesian basis is given by: F = x1e 1 + x 2 e 2 + x32 e 3 . The

normal for each surface are defined as follows:


r
n (1) // r

n (1) =

x12

x 22

( x1 e 1 + x 2 e 2 ) ; n ( 2 ) = e 3 ; n (3) = e 3

On the surface S (1) it holds that:


r

F n

(1)

( x e

dS (1) =

S ( 1)

+ x 2 e 2 + x 32 e 3 )

S ( 1)

x12 + x 22
x12

( 1)

dS (1) =

x 22

1dS

1
x12
(1)

x 22

( x1 e 1 + x 2 e 2 )dS (1)

= 2rh = 2

(1 )

where we have considered the cylinder area ( 2rh = 2 ).


On the surface S ( 2) it holds that x3 = 1 :

r
F n ( 2) dS ( 2 ) =

( x e
1

S (2)

+ x 2 e 2 + 1e 3 ) (e 3 ) dS ( 2 ) =

S (2)

1dS

(2)

= r 2 =

S (2)

where we have considered the circle area ( r 2 = ).


On the surface S (3) , it holds that x3 = 0 :

r
F n (3) dS (3) =

S ( 3)

( x e
1

+ x 2 e 2 + 0e 3 ) (e 3 )dS (3) =

S ( 3)

with that: F n dS =
S

0dS

( 3)

=0

S (3)

r
F n (1) dS (1) +

S (1 )

r
F n ( 2 ) dS ( 2 ) +

S (2)

r
F n (3) dS (3) = 3

S ( 3)

Then, proving the divergence theorem: xr F dV = F n dS = 3 .


V

Problem 1.125
Let be a domain bounded by as shown in Figure 1.14. Further consider that m is a
second-order tensor field and is a scalar field. Show that the following relationship holds:

[m : (

r
r
x ( x

[m

University of Castilla-La Mancha


Ciudad Real - Spain

))]d =

[( ) m] n d [(
r
x

r
x

m) xr ]d

ij

, ij ] d = ( , i m ij )n j d [m ij , j , i ] d

Draft

By: Eduardo W. V. Chaves (2014)

1 TENSORS

125

x2

x1

Figure 1.14

Solution: We could directly apply the definition of integration by parts to demonstrate the
above relationship. But, here we will start with the definition of the divergence theorem. That
r
is, given a tensor field v , it is true that:

r
x

v d = v n d indicial
v j , j d = v j n j d

r
Observing that the tensor v can be represented by the result of the algebraic operation
r
v = xr m and the equivalent in indicial notation is v j = , i m ij , and by substituting it in

the above equation we obtain:

j, j

d = v j n j d

[,

[,

ij

[,

m ij

,j

d = , i m ij n j d

m ij + , i m ij , j d = , i m ij n j d

ij

m ij d = , i m ij n j d

[ ,

m ij , j d

The above equation in tensorial notation becomes:

[m : (

r
r
x ( x

))]d =

[( ) m] n d [ (
r
x

r
x

r
x

m)]d

NOTE: Consider now the domain defined by the volume V , which is bounded by the
r
surface S with the outward unit normal to the surface n . If N is a vector field and T is a
scalar field, it is also true that:

N T,
i

ij

dV = N iT ,i n j dS N i , jT ,i dV

r
r
r
N ( xr ( xrT ))dV = ( xrT N ) n dS xrT xr N dV

where we have directly applied the definition of integration by parts.

University of Castilla-La Mancha


Ciudad Real - Spain

Draft

By: Eduardo W. V. Chaves (2014)

SOLVING PROBLEMS BY MEANS OF CONTINUUM MECHANICS

126

Problem 1.126
r

Let b be a vector field, which is defined as b = xr v . Show that:

b n
i

d S = , i b i dV

where = ( x ) represents a scalar field.


r

Solution 1: The Cartesian components of b = xr v are b i = ijk v k , j and by substituting them


in the above surface integral we obtain:

b n
i

dS = ijk v k , j n i dS

Applying the divergence theorem we obtain:

b n
i

dS = ijk v k , j n i dS

= ( ijk v k , j ), i dV

= ( ijk , i v k , j + ijk v k , ji ) dV
V

= (, i ijk v k , j + ijk v k , ji ) dV = , i b i dV
1
424
3
1
424
3
V

bi

Solution 2:

b n
i

dS = (b i ), i dV = (, i b i + b i , i ) dV

note that b i = ijk v k , j b i ,i = ijk v k , ji = ijk v k ,ij = 0

b n
i

dS = , i b i dV = , i ijk v k , j dV

Problem 1.127
Let V be a volume domain which is delimited by surface S . a) Show that:
r

( x n + n x) dS = 2V 1
S

where n is the outward unit vector to surface S . b) Show also that:

r
x

) x dV = ( n ) x
S

dS dV
V

ik , k

x j dV = ik n k x j dS ij dV

and
r

x (

r
x

) dV

r
= x ( n ) dS T dV

University of Castilla-La Mancha


Ciudad Real - Spain

x
i

Draft

jk , k

dV = xi jk n k dS ji dV
S

By: Eduardo W. V. Chaves (2014)

1 TENSORS

127

where is an arbitrary second-order tensor field.


r
dS = n dS

x2

dS

B
r
x

x1
x3

Solution:
a) Considering only the first term of the integrand, we can obtain:
r

( x n ) dS = ( x 1 n ) dS = ( x 1) n dS
S

By applying the divergence theorem we obtain:


r

( x n ) dS = ( x 1) n dS =
S

r
x

( x 1) dV

We will continue the development in indicial notation:

x n
i

dS = x i jk n k dS = (

jk

xi ) ,k dV = (

jk , k x i

jk

xi ,k ) dV

Taking into account that jk ,k = 0 j , xi ,k = ik , we conclude that:

x n
i

dS =

ji

dV =

ji

dV =

( x n ) dS = V 1

jiV

= V1

(1.137)

Similarly, we conclude that (n x ) dS = V 1 . With that the following is true:


S

( x n + n x) dS = 2V 1
S

b) Note that it holds


( x j ik ) ,k = x j ,k ik + x j ik ,k
{
=

x j ik , k = ( x j ik ) ,k ij

jk

r
r
( xr ) x = xr ( x )

with that we can obtain:

University of Castilla-La Mancha


Ciudad Real - Spain

Draft

By: Eduardo W. V. Chaves (2014)

SOLVING PROBLEMS BY MEANS OF CONTINUUM MECHANICS

128

r
x

) x dV = xr ( x ) dV dV
V

ik , k

= ( ik n k ) x j dS ij dV

r
= ( n ) x dS dV

dV = x j ik n k dS ij dV

dV = ( x j ik ) ,k dV ij dV

r
r
( xr ) x dV = ( x ) n dS dV

ik , k

where we have applied the divergence theorem to the first integral on the right side of
equation.
Taking into account that

[( xr ) xr ]T = [ xr ( xr ) ]T
r
r T
x ( xr ) = [ xr ( x )] T
In indicial notation
xi jk ,k = ( xi jk ) ,k ji

we can obtain:
r

x (

r
x

x (

) dV = [ xr ( x )]T

dV T dV

jk , k

dV

jk , k

r
x

) dV = ( x ) n dS T
S

dV = ( xi jk )n k dS ji dV

r
= x ( n ) dS T dV

dV = ( xi jk ) , k dV ji dV

= xi ( jk n k ) dS ji dV

NOTE: If we obtain the trace of the equation (1.137) we can also obtain:

x n
i

( x n ) : 1 dS = ( x n ) dS = V 1 : 1

dS = ji jiV = iiV

(1.138)

If we are dealing with a three dimensional case (3D) the trace ii = 3 , and if we dealing with
two dimensional case (2D) we have that ii = 2 . With that we can conclude that:

x n
i

( x n ) dS = 3V

dS = 3V

(3D)

and

x n

University of Castilla-La Mancha


Ciudad Real - Spain

( x n ) d = 2 A

d = 2 A

Draft

(2D)

By: Eduardo W. V. Chaves (2014)

1 TENSORS

129

x2

A : area of the domain

x1

Figure 1.15: Two dimensional case 2D.


Problem 1.128
Let be a scalar field which is given by:
GM
a

= r

where G and M are scalars and constants, and a is the magnitude of the vector a 0 . a)
r r
Obtain the gradient of . b) Obtain the gradient of for the particular case when a = x and
draw the field xr in the Cartesian space.
Solution:
GM
,i = r
a
x ,i

( xr ),i r

= GM 1 ( ar ) ,i

ar 2
,i

(1.139)

Note that:
1
1
r r 1
r
r r
1 r r
1 r r
( a ) ,i = ( a a ) 2 = (a a ) 2 ( a a ) ,i = ( a a ) 2 ( a k a k ) ,i
2

,i 2
1

r r
1 r r 2
1
(a a ) ( a k ,i a k + a k a k ,i ) = (a a ) 2 ( a k ,i a k ) = r ( a k , i a k )
2
a

or in indicial notation:
r
r
1 r
xr ( a ) = r (a xr a )
a

(1.140)

Then, the equation (1.139) becomes:


1 r
1 1
r
GM
GM r
,i = GM r 2 ( a ) ,i = GM r 2 r (a k ,i a k ) = r 3 (a k ,i a k ) = r 3 (a xr a ) i

a
x ,i
a
a
a
a

( xr ),i r

r
r
a
Moreover, considering that the unit vector according to the direction a is given by a = r ,
a

we can obtain:
r
r
r
GM
r
r
( xr ),i = GM
r 3 (a x a ) i = r 2 (a x a ) i
a

University of Castilla-La Mancha


Ciudad Real - Spain

Draft

(1.141)

By: Eduardo W. V. Chaves (2014)

SOLVING PROBLEMS BY MEANS OF CONTINUUM MECHANICS

130

b) For the particular case when a = x we have:


r
r
1
1
1
( x ) ,i = r ( x k ,i x k ) = r ( ki x k ) = r ( xi ) where r = x = x12 + x 22 + x32
x
x
x

or in tensorial notation:
r
r
1 r
1 r
1 r
xr ( x ) = r ( x xr x ) = r ( x 1) = r ( x ) = x
x
x
x

whereupon
GM
,i = r
x
x i

( xr )i r

= GM 1

xr 2
,i

r
( x ) = GM ( xr )
i
,i
r3

(1.142)

or in tensorial notation:
GM
xr = xr r
x

GM r GM
=
x = r 2 x
xr 3
x

(1.143)
r

Note that the vector field xr is radial, i.e. it is normal to the spheres defined by x and
r

decreases with x = r 2 (see Figure 1.16). The equation (1.143) can also be written as follows:
GM GM GM (r )
=
r = (r )r
= 2 r=
r =
r
r r
r r

(1.144)

x3

Spheres
x

xr

x = 1

r
x

r
b

x1

x2

xr

xr

Figure 1.16

University of Castilla-La Mancha


Ciudad Real - Spain

Draft

By: Eduardo W. V. Chaves (2014)

1 TENSORS

GM
x

NOTE: The function = r

131

represents the gravitational potential which has the

following property b = xr , (see Figure 1.16), where G = 6.67384 10 11

m3
kg s 2

is the

gravitational constant, M is the total mass of the planet. We check the units:

[] = GM
r =

m 3 kg kg m m N m J
( Unit of energy per unit mass)
= 2
=
=
kg
kg
kg s 2 m
s kg
(specific energy)

[br ]= [ ] = xr = mJkg = mNmkg = skgmkg = sm (Unit(unitof force


per unit mass)
of acceleration)
r
x

It is interesting to check that xr b = xr [ xr ] = 0 , (see Problem 1.107).


r

We can obtain b on the Earth surface by means of


r
GM
b = xr = r 2 x
x

where the total mass of Earth is M 5.98 10 24 kg and the approximate radius is
R 6.37 10 6 m , with that we obtain
r
GM
GM
b = r 2 x = 2 x 9.82 x
R
x

and its module is denoted by g = b 9.82

m
.
s2

Adopting that the system x has its origin at the center of mass of the body M , and invoking
r
r
the Newtons second law ( F = ma ), we can obtain the force that act in a body ( m ) due to the
r
gravitational field b = xr :
r
r
r
GMm
F = ma = mb = r 2 x
x

University of Castilla-La Mancha


Ciudad Real - Spain

Draft

(1.145)

By: Eduardo W. V. Chaves (2014)

SOLVING PROBLEMS BY MEANS OF CONTINUUM MECHANICS

132

We can express the above equation in a generic system ( x ) as shown below:


x 2

x1

r
x

r
F (Mm)

x3

r
x (M )

r
F (mM )

r
x (m )

x2

r
r r
x ( M ) + x = x ( m)
r r
r
x = x (m) x (M )
x1

x3

Then, for the system x the force is given by:


r
F ( mM ) =

GMm
r ( m) r ( M )
x x

r
r
( x ( m) x ( M ) )
r
r
x ( m) x ( M )

Newtons law of universal gravitation

(1.146)

where we use the nomenclature F (mM ) to indicate the force in m due to the influence of M .
Note also that in M we have the same force in direction and magnitude, but of opposite
r
sense to F (mM ) .
Problem 1.129
Consider that =

r
1
where r = x = x12 + x 22 + x32 :
r

a) Show that:

r r
2 2 2
xr xr ( x 0) 2 2 + 2 + 2 = 0 Laplace equation
(1.147)
x1
x 2
x3
r r
for r 0 . We use the nomenclature xr ( x 0) to indicate that the origin is not included.

b) Given a closed surface S containing the origin, show that:

( ) n dS = 4
r
x

(1.148)

where n is the outward unit vector to surface.


Solution:
It was obtained in Problem 1.128 that
University of Castilla-La Mancha
Ciudad Real - Spain

Draft

By: Eduardo W. V. Chaves (2014)

1 TENSORS

GM
xr = xr r
x

133

GM r GM
=
x = r 2 x
xr 3
x

(1.149)

Denoting by GM = 1 we obtain:
1 1 r 1
xr = xr r = r 3 x = r 2 x
x x
x

(1.150)

1 r
1
( xr ) i = r 3 x = r 3 xi
x

(1.151)

or in indicial notation:

Calculating the divergence of the previous relationship we obtain:


xr

( xr ) = ,ii

x
= r 3i
x

= xi ,i x 1 = x i ,i x 3 ( xr )
i r 3
i
,i
r3
r3
r

x 4
x
x

x ,i
,i
r

1
x

(1.152)

In Problem 1.128 it was shown that xr ( x ) = r ( x ) , in addition, note that xi ,i = ii = 3 ,


with that we can obtain:
xr

3 r
3 x
3x x
3
3
3

r 4 ri = r 3 + ri 5i
x

(
)
=

x
x
i
i
,i
r3
r 4
r3
x
x

x
x
x
x
x

r 2
3x
3
= r 3 + r 5 =0
x
x

( xr ) =

(1.153)

c) We adopt an arbitrary sphere of radius r , whose surface area is 4r 2 . Then:

1
( ) n dS = r
r
x

1
x n dS = r 2

x n dS =
S

1
r 2
x

dS = r

( Area) =

1
(4r 2 ) = 4
2
r

(1.154)
Note that x n = 1 since for the sphere it holds that x // n .
It is interesting to note that by means of the divergence theorem it fulfills that:

[ ]dV = ( ) n dS
r
x

r
x

r
x

,ii dV

= ,i ni dS

(1.155)

We have demonstrated that xr xr ( x 0) = 0 , but that only apply to x 0 (the origin is


not included). That is, taking into account the result in (1.154), the result in (1.155) has
r r
consistency if at the point x = 0 there is a sink and equal to ( 4 ). With that, it is very
intuitive to conclude that any closed surface that does not contain the origin the following
holds xr n dS = 0 , (see Parker (2003)).

University of Castilla-La Mancha


Ciudad Real - Spain

Draft

By: Eduardo W. V. Chaves (2014)

SOLVING PROBLEMS BY MEANS OF CONTINUUM MECHANICS

134

Problem 1.130
a) Show that:

( ) n dS = 4GM (r )

(1.156)

GM
is the gravitational potential, and M (r ) is the total mass contained into
r
the sphere whose radius is r , and S -surface represents the sphere boundary.

where =

b) Consider a sphere of radius r = a which represents a planet. Obtain the total mass of the
planet in function of the mass density = (r ) .
c) Obtain the gravitational potential for r < a and r a . In this section, consider that the
mass density is uniform in the planet = 0
Solution:
a) In Problem 1.129 we showed that:

( ) n dS = r n dS = 4
1

(1.157)

By multiply both sides of the equation by GM (r ) we obtain:


1
GM (r ) n dS = 4GM (r )
r
S

GM (r )
n dS = 4GM (r )
r

[ ] n dS = 4GM (r )

(1.158)

b)

Spherical planet

r=a
r

(r )

The total mass is obtained as follows:

M = (r )dV
V

University of Castilla-La Mancha


Ciudad Real - Spain

Draft

(1.159)

By: Eduardo W. V. Chaves (2014)

1 TENSORS

135

Note that V = 43 r 3 dV = 43 3r 2 dr = 4r 2 dr . Then:

M = (r )dV =

r =a

(r )4r

dr

(1.160)

r =0

c) Remember that in Problem 1.128 (see equation (1.144)) we have obtained that
r
GM GM GM (r )
= b =
r = (r )r
= 2 r =
r =
r r
r
r r

(1.161)

By using the equation in (1.158) we get:

[ ] n dS = 4GM (r )
S

r
b n dS = (r ) r n dS = (r ) dS = ( r )(4r 2 ) = 4GM (r )
123

=1

(r )r 2 = GM (r )

(r ) =

(1.162)

GM (r )
r2

where M (r ) = V 0 = 43 r 3 0 . Then:

(r ) =

GM (r ) 4G 0
=
r
3
r2

d ( r ) 4G 0
=
r
dr
3

4G 0
rdr
3

(1.163)

2G 0 2
r +C
3

(1.164)

d ( r ) =

By integrating the above equation we obtain:

d =

4G 0
rdr
3

(r ) =

4G 0 r 2
+C
3
2

(1) (r ) =

where we have denoted (1) ( r ) = (r ) for r < a . For values r a the gravitational potential is
given by

GM 4Ga 3 0
=
= (2)
r
3r

for

(1.165)

ra

where M is the total mass of the planet whose value is M = V 0 = 43 a 3 0 . Note that the
potential has to be continuous in r = a , (see Parker (2003)), thus:
4Ga 3 0
2G 0 2
a +C =
3
3a
3
3
2Ga 0 2Ga 0 4 3 2GM 3 3MG
C =
=
=
=
a
a
34
a
4
2a

(1) (r = a ) = ( 2) ( r = a )

(1.166)

With that the equation (1.164) becomes

(1) (r ) =

2G 0 2
2G 0 2 3MG MG 2 3MG MG r 2
3
= 3r
= 2 2
r +C =
r
2
2a
3
3
2a
2a 2a
2a

(1.167)

We summarize the gravitational potential as follows:

(r ) =

(r ) =

University of Castilla-La Mancha


Ciudad Real - Spain

MG r 2
3


for
2
2
2
2a 2a
MG
for
ra
r

Draft

r<a

(1.168)

By: Eduardo W. V. Chaves (2014)

SOLVING PROBLEMS BY MEANS OF CONTINUUM MECHANICS

136

(r )
Planet surface
a

MG
a

inflection point
3MG
2a

Figure 1.17: Gravitational potential vs. radius.

Figure 1.18: Gravitational potential (Ref. Wikipedia: Gravitational potential).

Problem 1.131
a) Show that the orbit of a planet takes place on a plane. b) Prove the Keplers laws of
planetary motion:
b.1) First Law: The orbit of a planet is an ellipse, with the Sun at one of the foci of the ellipse;
b.2) Second Law: The vector position from the Sun to the planet describes one area at a
constant rate;

University of Castilla-La Mancha


Ciudad Real - Spain

Draft

By: Eduardo W. V. Chaves (2014)

1 TENSORS

137

b.3) Third Law: If T (orbital period) represents the time required for the planet to perform a
full elliptical orbit, whose major axis of the ellipse is 2a , the relationship T 2 = a 3 holds,
where is a constant.
Reminder: Expressions related to the ellipse:
x2
r
x

b
f2

f1

x1

Equation of the ellipse: x = r =


Eccentricity: e =

a 2 b2
a2

p
1 + e cos
0 < e < 1 , where a 2 =

p2
holds.
(1 e 2 ) 2

Area enclosed by an ellipse: A = ab


Solution:
M - Mass of Sun
m - Mass of the planet
r
x
x = r
x

r
r dx
v=
,
dt
r
a // x

x2
x3

r
x
Sun

r r r
c = xv

r
dx r
=v
dt

r
r
r d 2 x dv
a= 2 =
dt
dt

r r
F // a

t=0

r
h

x1

Figure 1.19: Orbit of the planet.

University of Castilla-La Mancha


Ciudad Real - Spain

Draft

By: Eduardo W. V. Chaves (2014)

SOLVING PROBLEMS BY MEANS OF CONTINUUM MECHANICS

138

a) To show that the orbit takes place on a plane, we must prove that the vector ( c ) normal to
r
r
the plane which is defined by the vectors x and v does not change with time, i.e. c is
constant.
We recall the equation (1.145) of Problem 1.131:
r
r
r
r
GMm
GM
F = ma = mb = r 2 x
; a = r 2 x
(1.169)
x
x
r r r
Next, we obtain the rate of change of c = x v :
r
r r d r r r r r r
dc d r r
d r
= ( x v ) = ( x ) v + x (v ) = v12
3v + 1
x2
3
a =0
r
r
dt dt
dt
dt
=0
=0
r r r
Thus we have shown that the vector c = x v does not change with time, which implies that

the orbit takes place on a plane.


b.1) First Law

Since the planets orbit is performed on a plane, we take x1 x 2 as the plane of the orbit, then
r
the vector c has the same direction as x3 , (see Figure 1.19).
r

We express c in term of x :
r
r
d( x )
r dx d r
r dx
= ( x x ) =
v=
x + x
dt dt
dt
dt

and
r
r
d( x )
r r r
r
r dx
r d( x )
r 2
r 2
dx
dx

= x
3x + x x
= x x
c = x v = ( x x )
x + x
x 2
1

r
dt
dt
dt
dt
dt
=0

GM

Taking into account that a = r


r
r r r
direction as v , i.e. (a c ) // v :

r r
x , we calculate the vector a c which has the same

r r GM r 2
dx
dx
dx
dx

a c = r 2 x x x
= GM x x
= GM ( x ) x ( x x )
x

dt
dt
dt
dt

dx
= GM
dt
r r r
r r r r r r
where we have used the property a b c = (a c )b (a b)c , (see Problem 1.17). Note
dx
dx
2
also that it holds that x = 0 since x , and x x = x = 1 . Considering that GM is a
dt
dt

constant, the following is true:

r r
dx d (GM x )
a c = GM
=
dt
dt

Since the vector c does not change with time, the following is true:
r r
r
r r dv r d ( v c )
ac =
c =
dt
dt

Thus

University of Castilla-La Mancha


Ciudad Real - Spain

Draft

By: Eduardo W. V. Chaves (2014)

1 TENSORS

139

r r
d (v c ) d (GM x )
=
dt
dt

Integrating over time the above equation we obtain:

r
r r
v c = GM x + h

where h is constant vector of integration and is not dependent of time. Note that h is located
r r
on the plane x1 x 2 , since (v c ) and x are also on the plane x1 x 2 , (see Figure 1.8).
We calculate:
r
r
h x = h x cos = h cos
r

where we have denoted by h = h . Then:


r
c2 = c

r r
r r r
r r r
= c c = ( x v ) c = (v c ) x
r
r
r
r r
r
r
= GM x + h ( x x ) = x GM x x + x h x = x GM + x h cos
r
= x (GM + h cos ) = r (GM + h cos )
r
where we have considered that r = x . Then, we can obtain the following equation of the
2

ellipse:
c2
p
c
GM
r=
=
=
(GM + h cos ) (GM + h cos ) 1 + e cos
GM
2

where we have considered that:


p=

c2
GM

and

e=

h
GM

(1.170)

b.2) Second Law


r 1 r
A = x S
2

S 0

r 1 r
r
dA = x ds
2

x2

S
A
r
x
x1

The rate of change of dA becomes:


University of Castilla-La Mancha
Ciudad Real - Spain

Draft

By: Eduardo W. V. Chaves (2014)

SOLVING PROBLEMS BY MEANS OF CONTINUUM MECHANICS

140

r
r
r
r
r
r 1 r D ( ds )
D(dA) 1 D( x ds ) 1 D( x )
=
=
ds + x
Dt
Dt
Dt
2
2 Dt
2
r
r 1r r 1r
1 D( x )
=
ds + x v = c (constant)
2 1Dt
2
4243 2
r
=0

and its magnitude:


r
D (dA) D(dA) 1 r 1
=
= c = c
Dt
Dt
2
2

NOTE: As a consequence of second law it follows that if the areas of two sectors are equal,
the time required to perform their paths are equal, that is, according to Figure 1.20 as the areas
of the sectors OCD and EFO are equal the times to perform C D and E F are equal.
As result, when the planet is closer to the Sun its velocity is greater than when it is far.
sector EFO

sector OCD
E
D

A
C

Figure 1.20: Orbit of the planet.


b.3) Third Law
If T is the total time for a complete orbit (orbital period), we can obtain:
T

D (dA)
1
1
A=
dt =
c dt = cT
2
2
Dt
0
0

Taking into account the area enclosed by the ellipse: A = ab , we conclude that

1
cT = ab ,
2

thus:
T=

2ab
c

T2 =

4 2 a 2 b 2
c2

(1.171)

Considering the equation of the eccentricity, we can obtain:


e=

a 2 b2
a2

b2 = a 2 a 2e2

and taking into account a 2 =

b 2 = a 2 (1 e 2 )

p2
p
a=
(1 e 2 )a = p into the above equation,
2 2
2
(1 e )
(1 e )

we can obtain:

University of Castilla-La Mancha


Ciudad Real - Spain

Draft

By: Eduardo W. V. Chaves (2014)

1 TENSORS

b 2 = a 2 (1 e 2 )

141

b 2 = ap

p=

b2
a

Whereby the equation (1.171) can be rewritten as follows:


T2 =

4 2 a 2 b 2 4 2 a 2 ab 2 4 2 a 3 p 4 2 3
=
=
=
a = a3
GM
c2
c 2a
c2

where we have considered that

(1.172)

p
1
=
, (see equation (1.170)).
2
GM
c

COMPLEMENTARY NOTE 1

Geometrical Properties of Curves


Let us consider a curve, the function y = y ( x) , we denote by:
First derivative:

dy ( x)
y y, x (tangent of the curve at a point)
dx

Second derivative:

d 2 y ( x)
y y, xx .
dx 2

y ( x)

y
x

s 2 = x 2 + y 2
y

1
x

Figure 1.21
Infinitesimal arc length ds :
According to Figure 1.21 we can obtain:
2

x 2
(x 2 + y 2 )
y
s = x + y = (x + y ) 2 =
x = 1 + x
2
x
x
x
2

Then, we define the differential arc-length element as follows:

University of Castilla-La Mancha


Ciudad Real - Spain

Draft

By: Eduardo W. V. Chaves (2014)

SOLVING PROBLEMS BY MEANS OF CONTINUUM MECHANICS

142

2
2

y
dy

ds = lim 1 + x = 1 + dx 1 + ( y ) 2 dx = 1 + ( y ) 2
x 0

x
dx

ds
= 1 + ( y ) 2
dx

1
2

dx

1
2

Curvature
Curvature measures how quickly the direction of s changes with respect to a change in arc
length s , where s is the unit vector according to the ( y ) -direction. So, we define the vector
curvature as follows:
ds
ds

r
ds
= =
ds

curvature

where ( x) is the curvature of the curve at point x .


Let us consider that there is an angle such as tan( ) =

dy
y and if we differentiate with
dx

respect to x we obtain:
d dy d
d [tan( )] d
d
d
= sec 2 ( )
= 1 + tan 2 ( )
= [tan( )] =
dx dx dx
d
dx
dx
dx

2
d2y
d dy d
d
2
= 1 + ( y) 2
2 y = 1 + tan ( )
= 1 +
dx dx dx
dx
dx
d
y

=
dx [1 + ( y) 2 ]

The curvature can be obtained as follows:


=

where

dx
=
ds

1
1
[1 + ( y) 2 ] 2

d d dx
y
=
=
ds
dx ds [1 + ( y) 2 ]

1
1
2 2
[1 + ( y) ]

y
3
2 2
[1 + ( y) ]

holds.

Note that the curvature of a circumference is constant:


=

d
=
ds

ds = d

ds = d

integratin
g

2r = 2

ds = d
2 1
=
=
2r r

(1.173)

where ( 2r ) is the length of the circumference of radius r .


If we consider Figure 1.21 we can conclude that the curvature of the circumference of radius
r (1) is greater than the circumference of radius r ( 2 ) :
r (1) < r ( 2 )

University of Castilla-La Mancha


Ciudad Real - Spain

1
r

(1)

Draft

>

1
r ( 2)

(1) > ( 2 )

By: Eduardo W. V. Chaves (2014)

1 TENSORS

143

r ( 2)

r (1)

Figure 1.22
An interesting equation we can obtain from the curvature, (see equation (1.173)), is:

ds = d = B A B _ A
A

B _ A

Area = ds = B A B _ A

Area

B
B

Figure 1.23

University of Castilla-La Mancha


Ciudad Real - Spain

Draft

By: Eduardo W. V. Chaves (2014)

SOLVING PROBLEMS BY MEANS OF CONTINUUM MECHANICS

144

For example, let us consider a circumference of radius r , and the variation of angle from A
to B can be obtained as follows:
1
Area = ds = [(2r ) ]
4
1
1

= [(2r ) ] =
4
r
2
= B A B _ A

B _ A =

Figure 1.24
Curvature Vector
r

ds
ds

where s is the unit vector (tangent to the curve), and if we use the unit vector properties we
can conclude that:
s = 1

y ( x)

= s s = 1

d (s ) d (s )
ds
s + s
=2
s =0
ds
ds
ds

d (s s ) d (1)
=
=0
ds
ds
r
s = 0

s
ds
ds

ds
ds

Figure 1.25

University of Castilla-La Mancha


Ciudad Real - Spain

Draft

By: Eduardo W. V. Chaves (2014)

1 TENSORS

145

Geometrical relations among coordinate increments

The transformation matrix from the system x to the system n - s (which is denoted by x ),
see Figure 1.26, is given by:
cos
aij =
sin

sin n 1 n 2
=

cos n 2 n 1

Then, it fulfils that:


dxi = aij dx j
dx1 n 1
=
dx2 n 2

dxi = a ji dx j
T

n 2 0 n 1
=
n 1 ds n 2

n 2 0 n 2 ds

=
n 1 ds n 1ds

With that we can obtain:


dx2
n 1 ds
= dx
n 2 1
ds

n i = 3ij

dx j
ds

where kij is the permutation symbol.

x2

x2

x2

n i = 3ij

x1

dx2

ds

ds

(i, j = 1,2)

dx2
n 1 ds
n i = =

n 2 dx1
ds

dx j

x1

kij - permutation symbol


ds - arc-length

dx1

x1

Figure 1.26

University of Castilla-La Mancha


Ciudad Real - Spain

Draft

By: Eduardo W. V. Chaves (2014)

SOLVING PROBLEMS BY MEANS OF CONTINUUM MECHANICS

146

COMPLEMENTARY NOTE 2

Geometrical Center (Centroid C.Geo.)


Let us consider V = dV the volume delimited by the surface S , and also let us consider the
r

systems x and x , (see Figure 1.27). By means of vector summation we can obtain:
r r r
x = x + x

By integrate over the volume we get:

r r
r
r
r
xdV = ( x + x )dV = xdV + x dV

The volume centroid is the point ( x (V ) ) where the following equation fulfils:
r
r
x dV = 0

r r
x = x (V )

Then, the centroid can be calculated as follows:


r

r
r
r
r
r
xdV = x (V ) dV + x dV = x (V ) dV = x (V ) dV

V
12r3
=0

r
x (V ) =

xdV xdV

dV

x3
S
dV

x3

x2

r
x

x2

r
x

x1

r
x (V )

x1

Figure 1.27
The components of the volume centroid ( x1(V ) , x 2(V ) , x3(V ) ) can be obtained as follows:

x dV
1

x1(V )

dV

University of Castilla-La Mancha


Ciudad Real - Spain

x dV
2

x 2(V )

dV

x dV
3

x3(V )

dV

Volume Centroid

(1.174)

Draft

By: Eduardo W. V. Chaves (2014)

1 TENSORS

147

where

x dV

is the first moment of the volume about the x1 axis;

x dV
2

is the first moment of the volume about the x 2 axis;

x dV

is the first moment of the volume about the x 3 axis.

Note that, if the volume is given by V = V ( A) + V ( B ) then the following is true:


r

xdV xdV

r
x (V ) = V

dV

=V

r
r
Vx (V ) = xdV =

r
r
Vx (V ) = xdV

r
xdV =

V ( A ) +V ( B )

r
xdV ( A) +

V ( A)

r A
r B
r
xdV ( B ) = V ( A) x (V ) + V ( B ) x (V )

V (B)

where we have used the definitions:


r

xdV

r A
( A)
x (V ) = V

dV

xdV

( A)

r B
(B)
x (V ) = V

( A)

dV

V ( A)

(B)

( B)

V (B)

In a similar fashion to the derivation of the equations in (1.174) which define the volume
centroid, we can also define the area centroid as follows:

x dA

dA

x dA

x 2( A)

dA

x dA
3

x1( A)

x3( A)

dA

Area centroid

(1.175)

Line centroid

(1.176)

and the line centroid:

x dL

dL

x dL

x 2( L )

dL

x dL
3

x1( L )

x3( L )

dL

Center of the Scalar Field of a Domain


r

Let us consider the scalar field ( = ( x ) ), by means of vector summation we can write:
r

x = x (V _ ) + x

Integratin
g

(V _ )
xdV = x dV + x dV

r
The center of the scalar field = ( x ) delimited by the domain V is defined by:
r
r
x dV = 0

With that we can conclude that:

University of Castilla-La Mancha


Ciudad Real - Spain

Draft

By: Eduardo W. V. Chaves (2014)

SOLVING PROBLEMS BY MEANS OF CONTINUUM MECHANICS

148

r (V _ )

xdV = x

r
x (V _ ) = V

dV

xdV xdV
dV

=V

V ( )

whose components are:

x dV

x dV

x1(V _ )

dV

x 2(V _ )

dV

x dV

x 3(V _ )

dV

Note that, if the scalar field is uniform inside the volume, the center of the scalar field and the
geometrical center are the same:
r

xdV

r
x (V _ ) = V

dV

xdV
V

dV

xdV

=V

dV

r
= x (V )

Similarly we can define the center of the scalar field into an Area:
r

r
x (A _) =

xdA
A

dA
A

and the center of a scalar field of the curve


r

r
x (L _) =

xdL
L

dL
L

If the scalar field represents the mass density ( ) the center of the scalar is denoted by
Center of Mass (C.M.).
r

xdV

r
x (V _ ) = V

dV

Center of mass (defined by a volume)

(1.177)

University of Castilla-La Mancha


Ciudad Real - Spain

Draft

By: Eduardo W. V. Chaves (2014)

1 TENSORS

149

Center of Vector Field delimited by a Domain

r r

Consider that the body of volume V is subjected to the field b = b( x ) , and by a scalar field
r
( x ) , (see Figure 1.28).
r r
b( x )

x3
x2

dV

x3

r
x

x2
r
x

x1

( x)

r r
x (b )

x1

Figure 1.28
We can obtain:
r r r
r r r
r
r r
r r
x = x (b ) + x x b = x (b ) b + x b
r r r
r r
r r
x bdV = x (b ) bdV + x bdV

The center of the vector field delimited by the volume V is defined by:
r

x bdV = 0

(1.178)

Then
r r

(b )
(b )
(b )
x bdV = x bdV = x bdV = x F

where we have denoted by

r
r
F = bdV

(1.179)

r r r
r r
x (b ) F = x bdV

Note that F is the resultant force and is located at the point x (b ) .


r

If the arbitrary field b is uniform inside of volume V we can obtain:

University of Castilla-La Mancha


Ciudad Real - Spain

Draft

By: Eduardo W. V. Chaves (2014)

SOLVING PROBLEMS BY MEANS OF CONTINUUM MECHANICS

150

r r r (br ) r
r
xdV b = x
b dV

V
r
r

r r r
r

r r r

dV x (V _ ) x (b ) b = 0
dV x (V _ ) b = dV x (b ) b

V
r (V _ ) r (br ) r
x
x
=0
r (V _ ) r (br )
x
=x
r r

(b )
x bdV = x bdV

and if in addition the scalar field is uniform we get:


r
r r
r
x (V _ ) = x (b ) = x (V )

if is uniform x (V _ ) = x (V )
r

if b is uniform x (V _ ) = x (b )
r

if b and are uniform x (V _ ) = x (b ) = x (V )


r r
b( x )

x2

x3

x3

x2

x1

x3

x1

r r
x (b )
x2

x3

r
x (V )

r
x (V _ )

r
( x)

x 2
x1

x1

Figure 1.29

If the scalar field = is the mass density, and b represents the gravitational field on the
proximity of the Earth surface, the equation (1.179) becomes:

University of Castilla-La Mancha


Ciudad Real - Spain

Draft

By: Eduardo W. V. Chaves (2014)

1 TENSORS

151

0
Fi = b i dV =
= 0
gdV mg
V

where m = dV stands for the total mass of the body.


V

Center of null rotation


Consider that:

r r r
r r r
r
r r r r
x = x () + x
x b = x () b + x b
r
r r
r r
r r
x bdV = x ( ) bdV + x bdV

The center of the vector filed where the rotation is null is defined by:
r

x bdV = 0

(1.180)

with that we can get:


r

1
42
4
3
r

()
()
x bdV = x bdV = x bdV

r r
r r
x () F =

r
r
where is the torque that the field b produces into the body and is defined by:
r r
r
= x bdV

(1.181)

If the scalar field represents the mass density ( = ), and b represents the gravitational field,
r r
r r
r r
x ( ) is denoted by Center of Gravity (G). Note also that x ( ) = x (b ) .
r r
Next we will obtain the torque of the vector field (b x ) :
r
r r
r r
r r
r r r
r
r
x = x + x
x (b x ) = x (b x ) + x (b x )

By integrating over the volume the above equation we get:

r
r r
r r
r r
r
r
x (b x )dV = x (b x )dV + x (b x )dV

(1.182)

The center of the vector field (b x ) of null rotation is defined by:

r r
r
r
x (b x )dV = 0

(1.183)

with that the equation in (1.182) becomes:

r r
r
r
r r
r r
r r
r
r r
r
r
x (b x )dV = x (b x )dV = x (b x )dV = x x ( ) F = x

University of Castilla-La Mancha


Ciudad Real - Spain

Draft

(1.184)

By: Eduardo W. V. Chaves (2014)

SOLVING PROBLEMS BY MEANS OF CONTINUUM MECHANICS

152

where we have used the equation (1.181). In Problem 1.17 we have shown that the equation
r r r
r r
r r r
x

(
b

x
)
dV
=
[(
x

x
)
1

x
x ] b holds, so that the above equation can be rewritten as

follows:

r r
r r
r
x (b x )dV = x

r r
r
r r
r r
{ [( x x )1 x x ] b }dV = x

r r
r
{ j O b }dV = x

where we have introduced the second-order pseudo-tensor:


r r
r r
j O = ( x x )1 x x

j O ij = x k x k ij xi x j

whose components are:


j O ij = x k x k ij xi x j =

( x12

x 22

1 0 0 x1 x1
1 0 x1 x 2
0 0 1 x1 x3

x1 x 2
x2 x2

x32 ) 0

x 2 x3

x1 x3
x 2 x3
x3 x3

( x 22 + x32 )
x1 x 2
x1 x3

2
2
= x1 x 2
( x1 + x3 )
x 2 x3
x x
x 2 x3
( x12 + x 22 )
1 3

(1.185)

If = is the mass density, and b is uniform vector field, we can obtain:

r r
r
{ j O b }dV = x

r r

j O dV b = x r

r r r
I O b = x

where I O is the inertia tensor of mass and is defined by:

2
2
( x 2 + x 3 )dV
V
= x1 x 2 dV
V
x1 x 3 dV
V

I O( ) = j O dV

I O( ) ij

whose SI unity is [I O( ) ] =

kg
m

x1 x 2 dV
V

( x + x )dV
x x dV

2
1

2
3

2 3

x1 x 3 dV

V
x 2 x 3 dV
V

( x12 + x 22 )dV

m 2 m 3 = kg m 2 .

Similarly, we define the inertia tensor of area:

2
2
x1 x 2 dA
x1 x3 dA
( x 2 + x3 )dA

A
A
A
( A)
2
2

( x1 + x3 )dA
I O ij = x1 x 2 dA
x 2 x3 dA

A
A

A
x1 x3 dA
( x12 + x 22 )dA
x 2 x3 dA
A
A
A

r
r r
r
r
Note that if we consider the torque (b x ) and vector x = x + x we can obtain:

University of Castilla-La Mancha


Ciudad Real - Spain

Draft

By: Eduardo W. V. Chaves (2014)

1 TENSORS

153

r r
r
r
r r
r r
x (b x )dV = { [( x x )1 x x ] b }dV

r r
r r
r r
r r
r
= { { [( x + x ) ( x + x )]1 [( x + x ) ( x + x )] } b }dV

r r
r r
r r
r r
{ { [( x x ) + ( x x ) + ( x x ) + ( x x )]1
=
r r
r r
r
r
r
r
r
[( x x ) + ( x x ) + ( x x ) + ( x x ) } b }dV
V
r r
r r
r
r
r r
r
r
= { [( x x )1 x x ] b }dV + { [( x x )1 x x ] b }dV +

r r
r
r r
r
r
r
r r
+ { [( x x )1 x x ] b }dV + { [( x x )1 x x ] b }dV

(1.186)
r

Note that, if the field b is uniform then we get:


r r
r r
r
r

r r
r r
{ [( x x )1 x x ] b }dV = [( x x )1 x x ] dV b

V
r r
r
r

r r
= ( x x )1 dV b + x x dV b

r
r r r
r
r
= x dV x 1 b + x x dV b

and
r r
r
r r
r
r
r

r
r
{ [( x x )1 x x ] b }dV = [( x x )1 x x ] dV b

V
r r
r
r
r

r
= ( x x )1 dV b + x x dV b

V
r
r r
r
r
r
= x x dV 1 b + x dV x b

r
r
r
r r
Note that we have considered that b is uniform, hence x (V _ ) = x (b ) = x , and the equation
r
r
x dV = 0 holds. With that the equation in (1.186) becomes:

r r

x (b x)dV = { [( x x )1 x x] b }dV

r r
r r
r
r
r r
r
r
= { [( x x )1 x x ] b }dV + { [( x x )1 x x ] b }dV

(1.187)

If we consider that:

University of Castilla-La Mancha


Ciudad Real - Spain

Draft

By: Eduardo W. V. Chaves (2014)

SOLVING PROBLEMS BY MEANS OF CONTINUUM MECHANICS

154

r r
r
r
r
r r
r r
x (b x )dV = { [( x x )1 x x ] b }dV = { j O b }dV

r
r r
r r
r r
r
r
x (b x )dV = { [( x x )1 x x ] b }dV = { j O b }dV = j O

r r
r
r
r
r r
r
r
x (b x )dV = { [( x x )1 x x ] b }dV = { jO b }dV

{ b }dV

(1.188)

The equation in (1.187) becomes:

r r
r r
r
r
r
r r
r r
r r
r
r
{ [( x x )1 x x ] b }dV = { [( x x )1 x x ] b }dV + { [( x x )1 x x ] b }dV

r
r
r
{ j O b }dV = { jO b }dV + { j O b }dV

r
r
r
r
{ j O b }dV { jO b }dV { j O b }dV = 0

r r

[ j O jO j O ] dV b = 0

(1.189)
r r
Note that the above equation must be true for any uniform vector field b 0 , thus:

[j

jO j O ] dV = 0

(1.190)

j O dV = jO dV + j O dV
V

where the components (see equation (1.185)) of j O , jO , and jO are given by:
( x 22 + x32 )
( x 2 2 + x3 2 )
x1 x 2
x1 x3
x1 x 2
x1 x3

( j O ) ij = x1 x 2
( x12 + x32 )
x 2 x3 , ( jO ) ij = x1 x 2
( x1 2 + x 3 2 )
x 2 x3 ,
x x
x x
x 2 x3
( x12 + x 22 )
x 2 x3
( x1 2 + x 22 )
1 3
1 3

( x 22 + x 32 )
x1 x 2
x1 x 3

( jO ) ij = x1 x 2
( x12 + x32 )
x 2 x3
x x
x 2 x3
( x12 + x 22 )
1 3

University of Castilla-La Mancha


Ciudad Real - Spain

Draft

By: Eduardo W. V. Chaves (2014)

2 Continuum Kinematics
2.1 Description of the Motion, Material Time Derivative,
Velocity, Acceleration
Problem 2.1
A continuum is defined by a square with sides b , subjected to rigid body motion which is
defined by rotating the continuum counterclockwise by an angle of 30 to the origin. Find
the equations of motion. Also obtain the new position of particle D .
r

Hint: Consider the systems x and X to be superimposed.


X 2 , x2
x2

D
30
b
A = A

x1

D
B
30

X 1 , x1

Solution: We apply the rigid body motion equations x = c + Q X = Q X , to c = 0 . The


components of Q are the same as the components of the transformation matrix from the
r
r
x -system to the x -system, i.e.:
cos sin 0
Q ij = sin cos 0
0
0
1

So, the continuum particles are governed by the equations of motion:


x1 cos 30 sin 30 0 X 1


x 2 = sin 30 cos 30 0 X 2
x 0
0
1 X 3
3

A particle which initially was at point D ( X 1 = 0 , X 2 = b , X 3 = 0 ) moves into the following


position:

SOLVING PROBLEMS BY MEANS OF CONTINUUM MECHANICS

154

x1D cos 30 sin 30 0 0 b sin 30


D


x 2 = sin 30 cos 30 0 b = b cos 30
x D 0

0
1 0
0

Problem 2.2
A continuum medium motion, in the material description, is given by:
x1 = exp t X 1 exp t X 2

t
t
x 2 = exp X 1 + exp X 2
x = X
3
3

(2.1)

for t > 0 . Find velocity, acceleration in material and spatial descriptions.


Solution:
Velocity:
V1 = exp t X 1 + exp t X 2
r
r
r r
Dx ( X , t ) componentes
V ( X , t) =

V2 = exp t X 1 exp t X 2
Dt
V = 0
3

(2.2)

Acceleration:
A1 = expt X 1 exp t X 2

A2 = expt X 1 + exp t X 2

A3 = 0

(2.3)

To find the velocity and acceleration components in the spatial description we substitute
the equations of motion:
Eulerian velocity (spatial description)
v1 = x2

v2 = x1

v3 = 0

(2.4)

Eulerian acceleration (spatial description)


a1 = x1 = v2

a2 = x2 = v1

a3 = 0

(2.5)

Problem 2.3
The velocity field of a fluid is given by:
and the temperature field is:

r
v = x1e 1 + x2 e 2 + x3e 3

(2.6)

r
T ( x , t ) = 3 x 2 + x3 t

(2.7)

Find the rate of change of temperature.


Solution:
The rate of change of any property is given by the material time derivative:
r
r
T T
T
T
DT T ( x , t ) T ( x , t )
=
+
+
vj =
v1 +
v2 +
v3
t
x j
t x1
x 2
x3
Dt
DT
= x3 + (0 x1 + 3 x 2 + tx3 ) = x3 + (3x 2 + tx3 )
Dt
University of Castilla-La Mancha
Ciudad Real - Spain

Draft

(2.8)
(2.9)

By: Eduardo W. V. Chaves (2014)

2 CONTINUUM KINEMATICS

155

Problem 2.4
Given the following motion:
xi = X i + 0.2tX 2 1i

(2.10)

and the temperature field (steady):


r
T ( x ) = 2 x1 + x 22

(2.11)

a) Find the temperature field in material description;


b) Find the rate of change of temperature for one particle that in the reference
configuration was at the position (0,1,0) .
Solution:
According to the equations of motion we have:
x1 = X 1 + 0.2tX 2 11 = X 1 + 0.2tX 2
x 2 = X 2 + 0.2tX 2 12 = X 2
x3 = X 3 + 0.2tX 2 13 = X 3

Then:

r
r
r
2
r r
2
T ( x ( X , t )) = 2 x1 ( X , t ) + x 2 ( X , t ) = 2( X 1 + 0.2tX 2 ) + ( X 2 ) = 2 X 1 + ( X 2 + 0.4t )X 2 = T ( X , t )

b) The material time derivative of temperature is given by:

r
DT ( X , t ) & r
T ( X , t ) = 0 .4 X 2
Dt

For the particle ( X 1 = 0; X 2 = 1; X 3 = 0) we have:


T& (( X 1 = 0; X 2 = 1; X 3 = 0), t ) = 0.4 X 2 = 0.4

Problem 2.5

r r

r r

Find the velocity field V ( X , t ) in the material description and the acceleration field A( X , t )
r r
of the particle at time t in function of the rate of change of displacement U ( X , t )
Solution:
r r
r&
D r r
V ( X , t) =
U ( X , t) = U
Dt
r r
r& D 2 r r
D r r
&r&
A( X , t ) =
V ( X , t) = V = 2 U ( X , t) = U
Dt
Dt

(2.12)
(2.13)

Problem 2.6
Consider the following equations of motion in the Lagrangian description:
r
2
x1 ( X , t ) = X 2 t 2 + X 1
x1 1 t
r


Matrix form
x 2 = 0 1
x 2 ( X , t ) = X 3 t + X 2
r

x 0 0
3
x 3 ( X , t ) = X 3
University of Castilla-La Mancha
Ciudad Real - Spain

Draft

0 X 1

t X 2
1 X 3

(2.14)

By: Eduardo W. V. Chaves (2014)

SOLVING PROBLEMS BY MEANS OF CONTINUUM MECHANICS

156

Is the motion above possible? If so, find the displacement, velocity and acceleration fields
in Lagrangian and Eulerian descriptions. Consider a particle P that at time t = 0 was at the
point defined by the triple equation X 1 = 2, X 2 = 1, X 3 = 3 . Find the velocity of P at time
t = 1s and t = 2 s .
Solution:

Motion is possible if J 0 , thus


x1
X 1
x 2
=
X 1
x3
X 1

x1
X 2
x 2
X 2
x3
X 2

x1
X 3 1 t 2
xi
x 2
=0 1
J=
X j
X 3
0 0
x3
X 3
r
The displacement vector field is given by the definition u =

motion (2.79) we obtain:


u1

u 2

u 3

(Xrr , t ) = x ( Xrr, t ) X = [X t + X ] X
(Xr , t ) = x ( Xr , t ) X = [X t + X ] X
(X , t ) = x ( X , t ) X = [ X ] X = 0
1

0
t =1 0
1
r r
x X . Using the equations of

= X 2t 2

= X 3t

(2.15)

which are the components of the displacement vector in the Lagrangian description. Here,
velocity and acceleration can be evaluated as follows:

( )

r
r

d u1 X , t
d
=
X 2t 2 = 2 X 2t
V1 v1 ( X , t ) =
dt
dt

r
r

du 2 X , t
d
= ( X 3t ) = X 3
V 2 v 2 ( X , t ) =
dt
dt

r
r

du 3 X , t
d
= (X 2t ) = 0
V3 v 3 ( X , t ) =
dt
dt

( )

r
dV1

A1 a1 ( X , t ) = dt = 2 X 2

r
dV 2

=0
A2 a 2 ( X , t ) =
dt

r
dV 3

A3 a 3 ( X , t ) = dt = 0

( )

(2.16)

The inverse form of (2.14) provides us the inverse equations of motion (Eulerian
description):
X 1 1 t 2

1
X 2 = 0
X 0
0
3

r
t 3 x1 X 1 ( x , t ) = x1 t 2 x 2 + t 3 x 3
r

t x 2 X 2 ( x , t ) = x 2 tx 3
r
1 x 3 X 3 ( x , t ) = x 3

(2.17)

Then, the displacement, velocity and acceleration fields in Eulerian description can be
evaluated by substituting equation (2.17) into the equations (2.15) and (2.16), i.e.:

(Xr ( xr , t ), t ) = X ( xr , t )t = ( x tx )t = u ( xr , t )
(Xr ( xr , t ), t ) = X ( xr , t )t = x t = u ( xr , t )
(X ( xr , t ), t ) = u ( xr , t ) = 0
r r
r
r
V (X ( x , t ), t ) = 2 X ( x , t )t = 2( x tx )t = v ( x , t )
r r
r
r

V (X ( x , t ), t ) = X ( x , t ) = x = v ( x , t )
r r
r

V (X ( x , t ), t ) = v ( x , t ) = 0
u1

u 2

u 3

Ciudad Real - Spain

(2.18)

University of Castilla-La Mancha

(2.19)

Draft

By: Eduardo W. V. Chaves (2014)

2 CONTINUUM KINEMATICS

A1

A2

A3

(Xr ( xr , t ), t ) = 2 X ( xr , t ) = 2( x
(Xr ( xr , t ), t ) = a ( xr , t ) = 0
(X ( xr , t ), t ) = a ( xr , t ) = 0
r

157

r
tx 3 ) = a1 ( x , t )

(2.20)

Taking into account the Lagrangian description of velocity given in (2.16), the velocity of
particle P ( X 1 = 2, X 2 = 1, X 3 = 3 ) at time t = 1s is given by:
r
r
r
v1 ( X , t ) = 2 X 2 t = 2 m / s ; v 2 ( X , t ) = X 3 = 3m / s ; v 3 ( X , t ) = 0

We can also observe that at time t = 1s the particle P occupies the position:
x1 = X 2 t 2 + X 1 = 3 ;

x 2 = X 3t + X 2 = 4 ;

x3 = X 3 = 3

So, the velocity of the particle P can also be evaluated by (2.19) as:
r
v1 ( x , t ) = 2( x 2 tx 3 )t = 2( 4 + 1 3) 1 = 2m / s
r

v 2 ( x , t ) = x 3 = 3m / s
v ( xr , t ) = 0
3

Note that, the velocities obtained via the Lagrangian or Eulerian description are the same,
since velocity is an intrinsic property of the particle.
We can also provide the velocity of the particle P at time t = 2 s :

( )
( )
( )

r
V1 v1 X , t = 2 X 2 t = 2 2 1 = 4m / s
r

V 2 v 2 X , t = X 3 = 3m / s
r

V3 v 3 X , t = 0

At time t = 2 s the new position of P is:


r
x1 ( X , t ) = X 2t 2 + X 1 = 6

r
r
x2 ( X , t ) = X 3t + X 2 = 7 ;
x3 ( X , t ) = X 3 = 3
r r
As we can verify the Lagrangian description of motion x ( X , t ) describes the trajectory of
P.
;

Trajectory of particle P

r
viP ( x , t = 1s) = [2;3;0]

r
Vi P ( X P , t = 1s) = [2;3;0]
t0
X iP = [2;1;3]

t = 1s

r
Vi P ( X P , t = 2s ) = [4;3;0]

xiP = [3;4;3]
P
xiP = [6;7;3]

t = 2s

r
viP ( x , t = 2s ) = [4;3;0]

University of Castilla-La Mancha


Ciudad Real - Spain

Draft

By: Eduardo W. V. Chaves (2014)

SOLVING PROBLEMS BY MEANS OF CONTINUUM MECHANICS

158

NOTE: Note that, the Eulerian velocity can not be obtained by means of
r r
DX ( x , t ) r r r
= 0 v ( x , t ) . We can verify this by means of the proposed problem:
Dt
r
r
r
r
r
DX i ( x , t ) X i ( x , t ) X i ( x , t )
X i ( x , t )
X i ( x , t )
r
r
r
v1 ( x , t ) +
v 2 ( x, t ) +
v3 ( x , t )
=
+
Dt
t
x 2
x 3
x1

thus:
r
r
r
r
r
r
r
X 1 ( x , t )
X 1 ( x , t )
DX 1 ( x , t ) X 1 ( x , t ) X 1 ( x , t )
r
=
+
v1 ( x , t ) +
v 2 ( x, t ) +
v3 ( x , t )
t
x 2
x 3
Dt
x1

) [

= 2tx 2 + 3t 2 x3 + 1 2( x 2 tx3 )t t 2 x3 + t 3 0 = 0
r
r
r
r
r
X 2 ( x, t )
X 2 ( x , t )
DX 2 ( x , t ) X 2 ( x, t ) X 2 ( x , t )
r
r
r
+
=
v 3 ( x, t )
v 2 ( x, t ) +
v1 ( x , t ) +
x3
x 2
t
Dt
x1

= ( x3 ) + [0 2( x 2 tx 3 )t + 1 x3 t 0] = 0
r
r
r
r
r
X 3 ( x , t )
X 3 ( x , t )
DX 3 ( x , t ) X 3 ( x , t ) X 3 ( x, t )
r
r
r
+
=
v3 ( x, t )
v 2 ( x, t ) +
v1 ( x , t ) +
x3
x 2
t
Dt
x1

= (0 ) + [0 2( x 2 tx 3 )t + 0 x3 + 1 0] = 0
r r r
Remind that u = x X , then:
r r
r r
r r
r r
Dx ( X , t ) D r r
Du( X , t ) r& r
v ( X , t) =
=
u( X , t ) X ( x , t ) =
u( X , t )
Dt
Dt
Dt

Also, it fulfills that:


r r
r r
r r
r& r
r r
Du( x , t ) u( x , t ) u( x , t ) r r
v ( x , t ) = u( x , t )
=
+
r v ( x, t )
Dt
t
x

Problem 2.7
The velocity field of the continuum, in Eulerian description, is given by:
v1 =

x1
1+ t

; v2 =

2 x2
1+ t

; v3 =

3 x3
1+ t

(2.21)
r

a) Obtain the relationship between material and spatial coordinates xi = xi ( X , t ) ;


b) Obtain the acceleration components by means of the spatial motion description.
c) Obtain the acceleration components by means of the Lagrangian motion.
Solution:
a) Considering that vi =

dxi
dt
v1 =

dx1 =

dx1
x
dx
dt
= 1 1 =
dt 1 + t
x1 1 + t

1 + t dt Lnx

(2.22)

= Ln(1 + t ) + Ln(C1 )

(2.23)

x1 = C1 (1 + t )

The initial condition t = 0 x1 = X 1 imply that C1 = X 1


University of Castilla-La Mancha
Ciudad Real - Spain

Draft

By: Eduardo W. V. Chaves (2014)

2 CONTINUUM KINEMATICS

159

x1 = X 1 (1 + t )
dx 2 2 x 2
dx
2dt
=
2 =
dt 1 + t
x2 1 + t

v2 =
1

dx 2 =

(2.24)

1 + t dt Lnx

(2.25)

= 2Ln(1 + t ) + LnC 2

x 2 = C 2 (1 + t ) 2

(2.26)

for t = 0 x 2 = X 2 C 2 = X 2
x2 = X 2 (1 + t ) 2
dx3 3 x3
dx
3dt
=
3 =
dt 1 + t
x3 1 + t

v3 =
1

dx 3 =

(2.27)

1 + t dt Lnx

(2.28)

= 3Ln(1 + t ) + LnC 3

x 3 = C 3 (1 + t ) 3

(2.29)

and t = 0 x3 = X 3 C3 = X 3
x3 = X 3 (1 + t ) 3

(2.30)

The equations of motion are:


(2.31)
x1 = X 1 (1 + t ) ;
x2 = X 2 (1 + t ) 2
;
x3 = X 3 (1 + t )3
r r
b) Knowing v ( x , t ) in the spatial description (Eulerian), we can apply the material time
derivative to obtain:

r r
r r
r r
r r
Dv ( x , t ) r r
v ( x , t )
= a ( x, t ) =
+ v ( x , t ) v ( x , t )
Dt
t
v
v
a i = i + (vi , k )v k = i + (v i ,1v1 + v i , 2 v 2 + v i ,3 v 3 )
t
t

(2.32)
(2.33)

thus,
1
x

+ 1
+ 0 + 0 = 0
(1 + t )
1 + t 1 + t

2x2
2x 2
2 x2

a2 =
+ 0 + 2
+ 0 =
2
2
1+ t 1+ t
(1 + t )
(1 + t )

x1

a1 =

a3 =

(2.34)

3x 3
6 x3

+ 0 + 0 + 3
=
1 + t 1 + t (1 + t ) 2
(1 + t )

3 x3

c) The Lagrangian velocity components are given by:


V1 = X 1

V2 = 2 X 2 (1 + t )

V3 = 3 X 3 (1 + t ) 2

dV2
= 2X2
dt

a3 =

x3 = X 3 + AX 2

(2.35)

thus,
a1 =

dV1
=0
dt

a2 =

dV3
= 6 X 3 (1 + t )
dt

(2.36)

Problem 2.8
Consider the equations of motion:
x1 = X 1

x2 = X 2 + AX 3

University of Castilla-La Mancha

Draft

Ciudad Real - Spain

(2.37)

By: Eduardo W. V. Chaves (2014)

SOLVING PROBLEMS BY MEANS OF CONTINUUM MECHANICS

160

where A is constant. Find the displacement vector components in the material and spatial
descriptions.
Solution:
Displacement vector:
r r r
u= xX

u1 = x1 X 1 = 0

u2 = x2 X 2 = X 2 + AX 3 X 2 = AX 3
u = x X = X + AX X = AX
3
3
3
2
3
2
3

(2.38)

The equations of motion are obtained as follows:


x1 1
x = 0
2
x3 0
1 0
det 0 1
0 A

0 X1
A X 2
A 1 X 3

(2.39)

0
A = 1 A 2
1

(2.40)

0
1

the inverse:
1 A 2
1
0
1 A2
0

1 A
A 1
0

(2.41)

thus,
X1
X = 1
2 1 A2
X 3

1 A

0
0

0 x1

1 A x2
A 1 x3

X 1 = x1

( x2 Ax3 )
X2 =
1 A2

1
X 3 = 1 A2 ( x3 Ax2 )

(2.42)

The displacement vector components, in the spatial description, become:

u1 = x1 X 1 = 0

A( x3 Ax 2 )
1

( x 2 Ax3 ) =
u 2 = x 2 X 2 = x 2
2
1 A2
1 A

A( x 2 Ax3 )
1
( x3 Ax 2 ) =
u1 = x3 X 3 = x3
2
1 A2
1 A

(2.43)

Problem 2.9
Consider the equations of motion:
x1 = X 1

x2 = X 2 + X 3t

x3 = X 3 + X 3t

(2.44)

Obtain the velocity of the particles that are passing at point (0,1,2) at time t1 = 0 s and
t2 = 1 s

Solution:
University of Castilla-La Mancha
Ciudad Real - Spain

Draft

By: Eduardo W. V. Chaves (2014)

2 CONTINUUM KINEMATICS

The velocity field is given by:

161

r r
r r
Dx ( X , t )
V ( X ,t) =
Dt

(2.45)

in components:
V1 = 0 ; V2 = X 3
; V3 = X 3
r r
For t = 0 s we have x = X , then, ( X 1 = 0, X 2 = 1, X 3 = 2)

V1 = 0

V2 = 2

(2.46)

V3 = 2

(2.47)

The particle that is passing at point ( x1 = 0, x 2 = 1, x3 = 2) at time t = 1 s , was at the point


(reference configuration):
x1 = 0 = X 1

x 2 = 1 = X 2 + X 3 ( X 1 = 0; X 2 = 0; X 3 = 1)
x3 = 2 = X 3 + X 3

(2.48)

thus,
V1 = 0

V2 = 1

V3 = 1

(2.49)

Problem 2.10
Given the Cartesian system e i , the particle motion is defined as follows:

ct
ct

+ X 2 cos
x1 = X 1 sin 2
2
2
2

X1 + X 2
X1 + X 2

ct
ct
+ X 2 sin
x 2 = X 1 cos 2
2
2
X +X2
2
1
X1 + X 2
x3 = X 3

(2.50)

where c is a constant.
Obtain the velocity components in spatial and material descriptions.
Solution:
The velocity components in the material (Lagrangian) description are:
r
V1 ( X , t ) =
r
V2 ( X , t ) =

X 22

ct
X 1 cos 2
2

X1 + X 2

ct
X 2 sin
X2 +X2

2
1

X 22

ct
X 1 sin 2
2

X1 + X 2

ct
+ X 2 cos
2
X +X2

2
1

c
X 12

+
c

X 12

r
V3 ( X , t ) = 0

(2.51)

Taking into account (2.50), we can note that the following relationship holds:
x12 + x22 = X 12 + X 22

(2.52)

Then, the velocity components in the spatial (Eulerian) description are:


r
cx
v1 ( x , t ) = 2 2 2
x1 + x2
University of Castilla-La Mancha
Ciudad Real - Spain

r
v2 ( x , t ) =

Draft

c x1
+ x22

x12

r
v3 ( x , t ) = 0

(2.53)

By: Eduardo W. V. Chaves (2014)

SOLVING PROBLEMS BY MEANS OF CONTINUUM MECHANICS

162

r r

The inverse equations of motion, X = X ( x , t ) , are:


ct
ct
cos

sin 2
2
x 2 + x 2 0
x1 + x 2

2
1
X1
x1
X = cos c t sin c t 0 x
2
2 x2 + x2
x2 + x2

2
2
1
X 3 1

x
0
0
1 3

(2.54)

Problem 2.11
The Eulerian velocity field components are:
v1 = x1

v2 =

x2
2t + 3

v3 = 0

(2.55)

Find the parametric equations of the trajectory of the particle which was at ( X 1 , X 2 , X 3 ) in
the reference configuration.
Solution:
To find the path line (trajectory) we must solve the system:
dx1
= x1
dt

dx2
x
= 2
dt
2t + 3

dx3
=0
dt

(2.56)

with the initial conditions


x1 (t = 0) = X 1

x2 (t = 0) = X 2
x (t = 0) = X
3
3
x1

X1
x2

X2

dx1
= dt
x1 0

x
Ln 1 = t
X1

dx 2
dt
=
x2
2t + 3
0

x
Ln 2
X2

(2.57)

x1 = X 1 exp t

( )

= Ln 2t + 3 Ln 3

x2 = X 2

2
t +1
3

(2.58)

x3 = X 3

Then, the equations of motion are given by:


x1 = X 1exp t

; x2 = X 2

2
t + 1 ; x3 = X 3
3

(2.59)

Problem 2.12
Consider the following equations of motion:
x1 = X 1

x3 = X 3
(2.60)
r
and a physical quantity represented by the scalar field q ( x , t ) in the Eulerian description:
r
q ( x , t ) = 2 x1 + x 2 x3 + 1
(2.61)
;

x2 = 2 t X 3 + X 2

a) Obtain the Lagrangian description of the physical quantity;


University of Castilla-La Mancha
Ciudad Real - Spain

Draft

By: Eduardo W. V. Chaves (2014)

2 CONTINUUM KINEMATICS

163

b) Obtain the Lagrangian and Eulerian velocities;


c) Obtain the rate of change of the physical quantity.
d) Obtain the local rate of change of q at the spatial point (1,3,2) .
Solution:
r

r r

a) To obtain the Lagrangian description we have, q ( x , t ) = q ( x ( X , t ), t ) = Q( X , t ) , i.e., we


r
substitute the equations of motion (2.60) into the equation of the variable q ( x , t ) given by
(2.61), i.e.:
r
Q( X , t ) = 2 X 1 + X 2 + ( 2t 1) X 3 + 1

(2.62)

r r
r r
Dx ( X , t )
V ( X , t) =
Dt

(2.63)

b) The velocity vector

Lagrangian description
V1 = 0

V2 = 2 X 3

V3 = 0

(2.64)

The inverse equations of motion:


x1 = X 1

x2 = 2 t X 3 + X 2
x = X
3
3

X 1 = x1

X 2 = x 2 2 t x3
X = x
3
3

inverse

Then, the Eulerian velocity components are given by:


v1 = 0

v2 = 2 x3

v3 = 0

(2.65)

c) The rate of change of the variable is obtain by applying the material time derivative
r
D
Q( X , t ) = 2 X 3
Q& =
Dt

or
q& =

(2.66)

r
r
q ( x , t )
+ xr q v
t4
1
42
3

(2.67)

= 0 ( steady )

q
q
q
q& = 0 + q, i v i = 0 +
v1 +
v2 +
v 3 = [(2)(0) + (1)(2 x 3 ) + ( 1)(0)] = 2 x 3
x 2
x 3
x1

(2.68)

We could have obtained the same result by starting from Q& = 2X 3 and substituting
X 3 = x3 , thus
r r
r
q& ( x , t ) = Q& ( X ( x , t ), t )
r
q& ( x , t ) = 2 x 3

(2.69)
r

d) Note that the physical quantity field is stationary, i.e. q = q ( x ) , then the local rate of
r
q ( x )
change is
= 0 at any spatial point.
t

University of Castilla-La Mancha


Ciudad Real - Spain

Draft

By: Eduardo W. V. Chaves (2014)

SOLVING PROBLEMS BY MEANS OF CONTINUUM MECHANICS

164

Problem 2.13
Given the Lagrangian displacement field:
u1 = ktX 2

u2 = 0

u3 = 0

and the Eulerian temperature field:

r
T ( x , t ) = ( x1 + x 2 ) t

a) Find the rate of change of temperature for a particle that at time t = 1s is passing at point
(1,1,1) .
Solution:

r
r
r
r
dT ( X , t )
T T x
&
&
We can apply the definition T ( x , t ) =
or T ( X , t ) =
+ r
t
t x t

By means of the equation u i = xi X i we can obtain the equations of motion:


u1 = x1 X 1

x1 = X 1 + ktX 2

u 2 = x2 X 2

x2 = X 2

u 3 = x3 X 3

x3 = X 3

The Lagrangian temperature field (material description) can be obtained as follows:

r
r r
T ( x ( X , t ), t ) = ( x1 + x 2 ) t = (( X 1 + ktX 2 ) + ( X 2 ) ) t = X 1t + kX 2 t 2 + X 2 t = T ( X , t )

Then, the material time derivative becomes:

r
T& ( X , t ) = X 1 + 2kX 2 t + X 2

If we want to find the rate of change of temperature of a particle which is passing through
the point x1 = 1, x 2 = 1, x3 = 1 at t = 1s , we have two possibilities, namely: 1) Finding the
position of said particle in the reference configuration and replacing in the above equation.
2) obtaining the expression of the rate of change of temperature in the spatial (Eulerian)
r r
description. To do this, we need the equations of motion X ( x , t ) :
x1 = X 1 + ktX 2

x2 = X 2
x = X
3
3

X 1 = x1 ktx 2

X 2 = x2
X = x
3
3

r r
r
T& ( X ( x , t ), t ) = X 1 + 2kX 2 t + X 2 = ( x1 ktx 2 ) + 2kt ( x 2 ) + ( x 2 ) = T& ( x , t )
r
by simplifying the above we obtain T& ( x , t ) = x1 + ktx 2 + x 2 . Then:

T& ( x1 = 1, x 2 = 1, x3 = 1, t = 1) = (1 k ) + 2k + 1 = k + 2

Alternative solution:
r
T x1 T x 2 T x 3
r
T T x
&

+ r
= ( x1 + x 2 ) +
+
+
T ( x, t ) =
t x t
x 2 t
x 3 t
x1 t
= (x1 + x 2 ) + (tkX 2 + t (0) + (0)(0) ) = x1 + x 2 + tkX 2

Note that x 2 = X 2 , then:


r
T& ( x , t ) = x1 + x 2 + tkx 2

University of Castilla-La Mancha


Ciudad Real - Spain

Draft

By: Eduardo W. V. Chaves (2014)

2 CONTINUUM KINEMATICS

165

Problem 2.14
Let us consider the following equations of motion:
x1 = X 1

x2 = X 2 +

t
X3
2

x3 = X 3 +

t
X2
2

(2.70)

a) Is this motion possible? Justify your answer;


b) Obtain the velocity components in the Lagrangian and Eulerian descriptions;
c) Obtain the path line (trajectory equation).
Solution:
a) Obtaining the Jacobian determinant:
1 0 0
xi
t2
= 0 1 2t = 1
J=F =
4
X j
0 2t 1

(2.71)

so that motion is possible for:


J =1

t2
>0t <2 s
4

(2.72)

b) The Lagrangian velocity components:

V = 0
1

D
t

X2 + X3 =
V2 =
2
Dt

D
t

V3 =
X3 + X2 =
2
Dt

X3
2

(2.73)

X2
2

The inverse equations of motion:


J
x1 1 0 0 X 1
X1
1
x = 0 1 t X inverse

X 2 = 0
2 2
2
J
t
0
x3 0 2 1 X 3
X 3

0
1
2t

0 x1

2t x 2
1 x3

(2.74)

By replacing X i into the Lagrangian velocity expression we obtain the velocity in the
spatial description:
t
x2
2 x tx
2
= 3 22
v1 = 0 ; v 2 =
2
t
4t
2
2
x3

t
x3
2 x tx
2
= 2 23
; v3 =
2
t
4t
2
2
x2

(2.75)

c) The trajectory can be obtained by eliminating t of the equations of motion (2.70):


x1 = X 1

( x3 X 3 ) X 3 = ( x 2 X 2 ) X 2

University of Castilla-La Mancha


Ciudad Real - Spain

Draft

x3 =

X2
X2
x2 2 + X 3
X3
X3

(2.76)

By: Eduardo W. V. Chaves (2014)

SOLVING PROBLEMS BY MEANS OF CONTINUUM MECHANICS

166

Problem 2.15

The velocity field at point x of a steady fluid is given by:


b 2 ( x12 x 22 )
b 2 x1 x 2
r
1 + 2U
v =U
e
e 2 + Ve 3
( x12 + x 22 ) 2
( x12 + x 22 ) 2

(2.77)

where U and V are constants.


r

Show that xr v = 0 and find the Eulerian acceleration field.


Solution:
v
x ( x 2 3x 2 )
x ( x 2 3x 2 )
v
v
r
xr v = v i ,i = 1 + 2 + 3 = 2Ub 2 1 2 1 2 23 + 2Ub 2 1 2 1 2 23 = 0
x1 x 2 x3
( x1 + x 2 )
( x1 + x 2 )

The acceleration:

r
r v
r r
r r
a=
+ xr v v = xr v v
t

The components of the spatial velocity gradient are given by:


x1 (3 x 22 x12 ) x 2 (3 x12 x 22 ) 0
r

2Ub
( xr v ) ij = 2
x 2 (3 x12 x 22 ) x1 (3 x 22 x12 ) 0
2 3
( x1 + x 2 )
0
0
0

r r
The acceleration components are given by a i = ( xr v ) ij (v ) j :
2

2 x1U 2 b 4
2
2 3
( x1 + x 2 )

a i = 2 x 2U 2 b 4
2
2 3
( x1 + x 2 )

Problem 2.16
r

Let ( X , t ) be a scalar field in Lagrangian (material) description. Find the relationship


r
r
between the material gradient of ( X , t ) , i.e. Xr ( X , t ) , and the spatial gradient of
r
r
( x , t ) , i.e. xr ( x , t ) .
r

Solution:
Remember that a Lagrangian variable ( X , t ) can be expressed in the Eulerian
(current) configuration by means of the equations of motion, i.e.:
r

r r

( X , t ) = ( X ( x , t ), t ) = ( x , t ) .
Then, from the scalar gradient definition we obtain:

r
r r
r
r
r
r
X
X

(
,
)
(
( x , t ), t ) x ( x , t )
Xr ( X , t ) =
r
r=
=
r F = xr ( x , t ) F
r
x
x
X
X

In addition we have the inverse form:


xr

r
r
r
r r
r
r
( x , t ) ( x ( X , t ), t ) X ( X , t )
r F 1 = Xr ( X , t ) F 1
( x, t ) =
r
r=
r =
x
x
X
X

University of Castilla-La Mancha


Ciudad Real - Spain

Draft

By: Eduardo W. V. Chaves (2014)

2 CONTINUUM KINEMATICS

167

Problem 2.17
Given the following Eulerian velocity field components:
v1 = 0 ;

v2 = 0 ;

v3 = f ( x1 , x 2 ) x3

a) Find the particle trajectory;


b) Obtain the mass density ( ), knowing that at t = 0 we have = f ( x1 , x 2 ) .
Solution:
dx1
= v1 = 0 x1 (t ) = C1 at t = 0 x1 = X 1 x1 (t = 0) = C1 = X 1 ;
dt
dx 2
= v 2 = 0 x 2 (t ) = C 2 at t = 0 x 2 = X 2 x 2 (t = 0) = C 2 = X 2
dt
dx3
dx3
= v3 = f ( x1 , x 2 ) x3 = f (C1 , C 2 ) x3
=
dt
x3

f (C , C
1

2 ) dt

Ln( x 3 ) = f (C1 , C 2 )t + k

denoting by k = Ln(C 3 ) we obtain:


Ln( x3 ) Ln(C 3 ) = f ( X 1 , X 2 )t
x
ln 3
C3

x
= f ( X 1 , X 2 )t 3 = exp f ( X 1 , X 2 )t x3 = C 3 exp f ( X 1 , X 2 )t
C3

at t = 0 x3 = X 3 x3 (t = 0) = C 3 = X 3
Summarizing:
x1 = X 1

x2 = X 2

x3 = X 3exp f ( X 1 , X 2 )t

(2.78)

The mass density:

with

1 0
Fij = 0 1

Fij =

0
0

xi
X j

= exp f ( X 1 , X 2 )t

? ? exp f ( X 1 , X 2 )t

The values marked by ( ? ) are not necessary to obtain the determinant, then:

0
F

f (X1, X 2 )
exp f ( X 1 , X 2 )t

Note that according to the problem statement, t = 0 , = f ( x1 , x 2 ) , and according to the


equations in (2.78) we conclude that 0 = f ( X 1 , X 2 ) .
Problem 2.18
Calculate the material time derivative
described as follows:

D
for the property when said property is
Dt

Material description: ( X , t ) = X 1t 2 ;

Spatial description: ( x , t ) =

University of Castilla-La Mancha


Ciudad Real - Spain

x1t 2
.
(1 + t )
Draft

By: Eduardo W. V. Chaves (2014)

SOLVING PROBLEMS BY MEANS OF CONTINUUM MECHANICS

168

Solution:

a) Material time derivative of ( X , t ) = X 1t 2 :


r
r
D
( X , t ) & ( X , t ) = 2 X 1t
Dt
r

b) Material time derivative of ( x , t ) =

x1t 2
:
(1 + t )

r
r
r
r
r ( x , t ) ( x , t )
( x , t )
D
vi
( x , t ) =
+ xr v =
+
xi
Dt
t
t
r
r
r
r
( x , t ) ( x , t )
( x , t )
( x , t )
=
+
v1 +
v2 +
v3
t
x 2
x3
x1

r
2

x t ( x , t )
v1 + 0 + 0
= 1 +
t (1 + t ) x1

We need to know the velocity component v1 . We start from the principle that a property is
intrinsic to the particle, then:
r

( X , t ) = X 1t 2

r r

( X ( x, t ), t ) = ( x, t ) =

x1t 2
(1 + t )

X1 =

x1
(1 + t )

The velocity becomes:

r
D
v( X , t) =
X 1t 2 = 2 X 1t e 1
Dt

x
r
v ( x , t ) = 2 1 t e 1
(1 + t )

Returning to the material time derivative we obtain:


r

x t2 t2
r
D
x t 2 ( x , t ) 2 x1t
( x, t ) = 1 +
v1 =
X1
1 2 +
Dt
t (1 + t ) x1
(1 + t ) (1 + t ) (1 + t )
2 x1t
2 x1t
x t2 t2
x
2 1 t =
=
1 2 +
(1 + t ) (1 + t ) (1 + t ) (1 + t ) (1 + t )

We could also have obtained the same result by starting from


and also knowing the equations of motion X 1 =
r
r
D
( X , t ) & ( X , t ) = 2 X 1t
Dt

r
r
D
( X , t ) & ( X , t ) = 2 X 1t
Dt

x1
, i.e.:
(1 + t )

r r
r r
x
r
D
( X ( x, t ), t ) & ( X ( x, t ), t ) & ( x, t ) = 2 1 t
Dt
(1 + t )

Problem 2.19
Consider the following equations of motion in the Lagrangian description:
x1 = X 1t 2 + 2 X 2 t + X 1
x1
Matrix


form
2
x 2 =
x 2 = 2 X 1t + X 2 t + X 2
x
x = 1 X t + X
3
3
3 2 3

t 2 + 1 2t
2
t +1
2t
0
0

0 X 1

0 X 2
1
t + 1 X 3
2

(2.79)

Find the components of the displacement vector in Lagrangian and Eulerian descriptions.

University of Castilla-La Mancha


Ciudad Real - Spain

Draft

By: Eduardo W. V. Chaves (2014)

2 CONTINUUM KINEMATICS

169

Solution:
r

The displacement vector can be obtained as follows u = x X . By replacing the equations


of motion (2.79) we obtain:
u1 = x1 X 1 = X 1t 2 + 2 X 2 t

2
u 2 = x 2 X 2 = 2 X 1t + X 2 t

1
u 3 = x 3 X 3 = 2 X 3 t

which are the displacement components in the Lagrangian description (material).


To obtain the Eulerian displacement we need to obtain the inverse equations of motion
(2.79), which the result is:
2tx 2 x1 (1 + t )

X1 = 3
3t 1 t t 2
x
0
1
2 x1t 2 x 2 (1 + t 2 )
0
x2 X 2 =
3t 3 1 t t 2
3t 3 1 t t 2 x 3

2 x3
1
(t + 2)
X 3 =
2
(t + 2)

(1 + t )
2t
X1

1

2
(1 + t 2 )
2t
X 2 = 3
2
t
t
t

3
1

X
3
0
0

We can also use the definition u = x X , but now we replace the material coordinate to
obtain the displacement vector components in the Eulerian description:
2tx 2 x1 (1 + t )

u1 = x1 X 1 = x1 3
3t 1 t t 2

2 x1t 2 x 2 (1 + t 2 )
u 2 = x 2 X 2 = x 2
3t 3 1 t t 2

2 x3
u 3 = x 3 X 3 = x 3
(t + 2)

Problem 2.20
The following equations describe
the motion of a body (continuum
medium):
x1 = X 1 + 0.2 X 2 t

x2 = X 2
x = X
3
3

X 2 , x2

Reference configuration

B
1

O
D

X 1 , x1

X 3 , x3

Figure 2.1: Reference configuration t = 0 .

University of Castilla-La Mancha


Ciudad Real - Spain

Draft

By: Eduardo W. V. Chaves (2014)

SOLVING PROBLEMS BY MEANS OF CONTINUUM MECHANICS

170

At time t = 0 , the cubic body (with sides 1) has one vertex at the origin of the system
which is indicated by point O, (see Figure 2.1). Obtain the configuration of the body at
time t = 2 s .
Solution:
To obtain the current configuration of the body at time t = 2 s , we will analyze the particle
motion. The particle which occupies position O (origin) at t = 0 has material coordinate:
X1 = 0

X2 = 0

X3 = 0

Substituting in the equations of motion:


x1 = 0

x i ( X 1 = 0, X 2 = 0, X 3 = 0, t ) x 2 = 0
x = 0
3

Then, we conclude that the particle does not change its position during motion.
The particles occupying the line OA , in the initial configuration, have the reference
coordinate ( X 1 , X 2 = 0, X 3 = 0) . In spatial coordinates:
x1 = X 1 + 0.2 X 2 t = X 1

x2 = X 2 = 0
x = X = 0
3
3

That is, all particles belonging to line OA do not move during motion. Similarly, we can
verify that the line ( X 1 , X 2 = 0, X 3 = 1) in the reference configuration ( X 1 , X 2 = 0, X 3 = 1)
does not move:
x1 = X 1 + 0.2 0 2 = X 1

x2 = X 2 = 0

x3 = X 3 = 0

The particles belonging to line CB ( X 1 , X 2 = 1, X 3 = 0) at time t = 2 s will move according


to:
x1 = X 1 + 0.2 1 2 = X 1 + 0.4

x2 = X 2 = 1

x3 = X 3 = 0

Then, all particles belonging to line CB will move 0.4 according to x1 -direction.
The particles belonging to line OC at t = 0 , will move to positions:
x1 = X 1 + 0.2 X 2 t = 0 + 0.2 2 X 2 = 0.4 X 2

x2 = X 2
x = X = 0
3
3

Following the same procedure for the remaining particles, we obtain the final configuration
of the body at time t = 2 s , (see Figure 2.2).

University of Castilla-La Mancha


Ciudad Real - Spain

Draft

By: Eduardo W. V. Chaves (2014)

2 CONTINUUM KINEMATICS

171

x2
0.4

0.4

C 1

C
E

Current configuration at

t = 2s

E 1
A=A

x1

G=G

x3

Figure 2.2: Current configuration at time t .


Problem 2.21
Consider the equations of motion:
x1 = X 1 + t 2 X 2
x1
Matrix


form
2
x 2 =
x 2 = t X 1 + X 2
x
x = X
3
3
3

1
2
t
0

0 X 1

0 X 2
1 X 3

t2
1
0

a) Obtain the trajectory of particle Q which originally ( t 0 ) was at X i = (1,2,1) ;


b) Consider the current configuration at t = 0.5 s . Obtain the velocity and acceleration
; 4 ;1) ;
components of the particle P that was originally at X i = (16
15 15
c) Obtain the equations of motion in the Eulerian description;
d) Obtain the velocity and acceleration components of one particle that at time ( t = 0.5 s ) is
passing at point xi = (1,0,1) .
Obs.: Consider the International System of Units (SI-Units).
Solution:
1)Using the equations of motion and by replacing the material coordinates of the point
X i = (1,2,1) , we obtain:
x 1 = 1 + 2t 2

x2 = 2 + t 2

x3 = 1

The above equations represent the motion of the particle. To obtain the trajectory, we
eliminate the time of the equations of motion, i.e.:
x1 2 x 2 = 3

x3 = 1

which indicates that the particle moves in a straight line defined by ( x1 2 x 2 = 3) on the
plane x3 = 1 . The graphical representation follows

University of Castilla-La Mancha


Ciudad Real - Spain

Draft

By: Eduardo W. V. Chaves (2014)

SOLVING PROBLEMS BY MEANS OF CONTINUUM MECHANICS

172

Particle trajectory

X 3 , x3

( x1 2 x 2 = 3)

x3 = 1

X 2 , x2

X 1 , x1

2) The velocity and acceleration components of the particle P are given by:
V1 = 2tX 2
r
r r
Dx

components
V ( X , t) =
V 2 = 2tX 1
Dt
V = 0
3
A1 = 2 X 2
r
r r
Dv

components
A2 = 2 X 1
A( X , t ) =
Dt
A = 0
3

Then, to the particle originally located at the point X i = (16


; 4 ;1) , at t = 0.5 s , we have:
15 15
4
4
V1 = 2 0.5 15
m/s
= 15

16
16
V 2 = 2 0.5 (15 ) = 15 m / s
V = 0
3

and

8
4
A1 = 2 15
= 15
m / s2

2
16
32
A2 = 2 (15 ) = 15 m / s
A = 0
3

3) The inverse equations of motion can be obtained as follows:


x1 = X 1 + t 2 X 2 X 1 = x1 t 2 X 2

2
2
x2 = t X 1 + X 2 X 2 = x2 t X 1
x = X X = x
3
3
3
3

x1 t 2 x 2
X1 =
1 t4

x 2 t 2 x1
X 2 =
1 t4

X 3 = x3

4) The velocity and acceleration of the particle that at time ( t = 0.5 s ) is passing through the
point xi = (1,0,1) can be obtained by means of velocity and acceleration in Eulerian
description:
Velocity:

University of Castilla-La Mancha


Ciudad Real - Spain

Draft

By: Eduardo W. V. Chaves (2014)

2 CONTINUUM KINEMATICS

173

x 2 t 2 x1
=
2
v
t
1
1 t4

=
2
V
tX
1
2

x1 t 2 x 2

substituti ng
t = 0. 5 s
=

2
=
V
X
t
2

v
t
2
2
1
4
(1, 01)
X1 , X 2
x

1
t
V = 0

3
v 3 = 0

v1 = 15 m / s

16

m/s
v 2 =
15

v 3 = 0

Acceleration:

x 2 t 2 x1
=
2
a
1
1 t4

=
2
A
X
1
2

x1 t 2 x 2

substituti ng
t = 0. 5 s
=

=
2

A
X
2
a
2
2
1
4
(1, 01)
X1 , X 2
x

1
t
A = 0

3
a 3 = 0

2
a1 = 15 m / s

32

m / s2
a 2 =
15

a 3 = 0

We can obtain the initial position of the particle by using the inverse equations of motion
which was obtained in paragraph (3), xi (1,0,1) :

x1 t 2 x 2 1 (0.5 2 )(0) 16
=
=
=
X
1
15
1 t4
1 (0.5) 4

x 2 t 2 x1 0 (0.5 2 )(1)
4

=
=
X 2 =
4
4
15
1 t
1 (0.5)

X = x = 1
3
3

We can verify that it is the same particle P referred to in paragraph 2. It is logical that we
have obtained the same velocity and acceleration using either the material or spatial
description, since the velocity and acceleration are intrinsic of the particle.
Problem 2.22
The acceleration vector field is described by:

r
r
r r
r r
D v v
=
a ( x , t) =
+ xr v v
Dt t

Show that acceleration can also be written as:


r
r
v2
Dv v
=
+ xr
Dt t
2

r
r
r
v2
r v
r
v ( xr v ) =
+

2
t

r
r
v2
r v
r
v rot v =
+

2
t

r r
+ rot v v

Solution:
To prove the above relationship one need only demonstrate that:
v2
r r
xr v v = xr
2

r
r
r
v ( xr v )

Expressing the terms on the right of the equation in symbolic notation we obtain:

University of Castilla-La Mancha


Ciudad Real - Spain

Draft

By: Eduardo W. V. Chaves (2014)

SOLVING PROBLEMS BY MEANS OF CONTINUUM MECHANICS

174

v2
xr
2

r
r

1
i ) e r (v s e s )
v ( xr v ) = e i
(
v
v
v
e

j
j
i

2 x i

x r

Using the definition of the permutation symbol (see Chapter 1) we can express the vector
product as:
v2
xr
2

r
r

v
r
1

i ) rst s e t
v ( xr v ) = e i
(
e
v
v
v

j
j
i

2 x i
x r

v j
v s
1
e k
= e i 2v j
rst itk v i
2
x r
x i

where we have used the equation e i e t = itk e k . In Chapter 1 we also proved that
rst itk = rst kit = rk si ri sk , then:
v2
xr
2

r
v j
r
r
v ( xr v ) = v j

x i

v j
=vj
x i
v j
=vj
x i

v2
xr
2

v s
e k
x r

v
v
e i rk si v i s ri sk v i s
x r
x r

v
v
e i v s s v i k e k
x i
x k
e i ( rk si ri sk )v i

e k

r
v j
r
v
v
r
v ( xr v ) = v j
e i v s s e k + v i k e k

x i
x k
x i

v j
v
v
e i v s s ik e i + v i k e k
= sj v s
x i
x k
x i
v s
v s
v k
e i v s
e i + v i
e k
= vs
x i
x i
x i
r
v e
(v )
vi
= k k vi =
x i
x i
r r
= xr v v

NOTE: It is interesting to see Problem 1.119 (Chapter 1).


Problem 2.23

r r

Consider the equations of motion x ( X , t ) and the temperature field T ( x , t ) given by:
x1 = X 1 (1 + t )

x 2 = X 2 (1 + t )
x = X
3
3

r
T ( x ) = x12 + x 22

Find the rate of change of temperature for the particle P at time t = 1s given that particle
P was at point ( X 1 = 3, X 2 = 1, X 3 = 0) at time t = 0 .
Solution 1:
In this first solution we first obtain the material time derivative of the Lagrangian
r
temperature, so, we have to obtain the temperature in Lagrangian description T ( X , t )
(Lagrangian temperature):

University of Castilla-La Mancha


Ciudad Real - Spain

Draft

By: Eduardo W. V. Chaves (2014)

2 CONTINUUM KINEMATICS

175

r
T ( x ) = x12 + x 22

By substi tuting
the equations of motion

r
T ( X , t ) = X 12 (1 + t ) 2 + X 22 (1 + t ) 2

The material time derivative of the Lagrangian temperature is given by:


r
r
dT
X
(
, t)
DT
T& ( X , t )
=
= 2 X 12 (1 + t ) + 2 X 22 (1 + t )
Dt
dt

By substituting t = 1s , ( X 1 = 3, X 2 = 1, X 3 = 0) , into the above equation we obtain:


r
T& ( X , t ) = 2 X 12 (1 + t ) + 2 X 22 (1 + t ) = 2(3) 2 (1 + 1) + 2(1) 2 (1 + 1) = 40

Solution 2:
In this alternative solution we directly use the definition of material time derivative of the
r
r
r
r
DT T ( x ) T ( x )
&
Eulerian variable, i.e. T ( x , t ) =
=
+
vk ( x, t ) .
t
x k
Dt

From the equations of motion we obtain:


r
v1 ( X , t ) = X 1
x1 = X 1 (1 + t )
r

velocity
x 2 = X 2 (1 + t ) v 2 ( X , t ) = X 2
r
x = X

3
3
v 3 ( X , t ) = 0

The equations of motion in Eulerian description are given by:


x1

X 1 = (1 + t )

x1 = X 1 (1 + t )

x2

inverse of the motion


x 2 = X 2 (1 + t ) X 2 =
(1 + t )
x = X

3
3
X 3 = x3

So, it is possible to obtain the Eulerian velocity as follows:


r r
r
r
x1

V1 ( X ( x , t ), t ) = X 1 ( x , t ) = (1 + t ) = v1 ( x , t )

r r

x2
r
r
= v 2 ( x, t )
V 2 = ( X ( x , t ), t ) = X 2 ( x , t ) =
(1 + t )

r
V3 = v 3 ( x , t ) = 0

Afterwards, the material time derivative of the Eulerian temperature, T ( x , t ) , is given by:
r
DT ( x , t ) & r

T ( x, t ) =
Dt

r
T ( x )
t3
12

= 0 (Stationar y field)

x
x
r
T& ( x , t ) = 2 x1 1 + 2 x 2 2 + 0
1+ t
1+ t

T
T
T
+
v1 +
v2 +
v3
x 2
x 3
x1

2x 2 2x 2
r
2
T& ( x , t ) = 1 + 2 =
( x12 + x 22 )
1+ t 1+ t 1+ t

The position of particle P at time t = 1s is evaluated as follows:

University of Castilla-La Mancha


Ciudad Real - Spain

Draft

By: Eduardo W. V. Chaves (2014)

176

SOLVING PROBLEMS BY MEANS OF CONTINUUM MECHANICS

x1 = X 1 (1 + t ) = 3(1 + 1) = 6

x 2 = X 2 (1 + t ) = 1(1 + 1) = 2
x = X = 0
3
3

Then, by substituting the spatial coordinates in the expression of the material time
derivative of temperature we obtain:
r
2
2
( x12 + x 22 ) =
T& ( x , t ) = T& ( x1 = 6, x 2 = 2, x 3 = 0, t = 1) =
(6 2 + 2 2 ) = 40
1+ t
1+1
r
Alternatively, the expression T& ( x , t ) could also have been obtained as:
r
T& ( X , t ) = 2 X 12 (1 + t ) + 2 X 22 (1 + t )
2
2
r r
x
x
r 2
r 2
T& ( X ( x , t ), t ) = 2[X 1 ( x , t )] (1 + t ) + 2[X 2 ( x , t )] (1 + t ) = 2 1 (1 + t ) + 2 2 (1 + t )
(1 + t )
(1 + t )
r
2
=
( x12 + x 22 ) = T& ( x , t )
(1 + t )

Problem 2.24
Consider the motion:
x i = X i (1 + t )

t > 0

Obtain the velocity field in the spatial description.


Solution:
The velocity is obtained by means of the material time derivative of the equations of
motion:
Vi = x& i =

d
[X i (1 + t )] = X i
dt

(2.80)

To find the velocity in the spatial description we need to obtain the inverse equations of
motion and replace it into the equation (2.80):
xi

x i = X i (1 + t ) X i = (1 + t )

v = X ( xr , t ) = x i
i
i
1+ t

Problem 2.25

Consider the temperature field T ( x ) in the spatial description and the equations of motion:
T = 2( x12 + x 22 )

i {1,2}
xi = X i (1 + t )

Find the rate of change of temperature at time t = 1s for one particle that was at position
(1,1) in the reference configuration.
r

NOTE: We can observe that the temperature field is a steady field, i.e. T = T ( x ) .

University of Castilla-La Mancha


Ciudad Real - Spain

Draft

By: Eduardo W. V. Chaves (2014)

2 CONTINUUM KINEMATICS

177

Solution 1:
In this first solution we obtain the equation for temperature in the material description:
r
T ( x ) = 2( x12 + x 22 )

by replacing the equations


of motion

r
T ( X , t ) = 2 X 2 (1 + t ) 2 + X 2 (1 + t ) 2
1
2

The material time derivative is given by:

r
r
dT
(
, t)
X
DT
T& ( X , t ) =
=
= 2 2 X 12 (1 + t ) + 2 X 22 (1 + t )
Dt
dt

By replacing t = 1s and material coordinate ( X 1 = 1; X 2 = 1) we obtain:


T& ( X 1 = 1; X 2 = 1; t = 1) = 16

Solution 2:
In this alternative solution we use directly the definition of the material time derivative of
Eulerian property:
r
T ( x ) = 2( x12 + x12 )

x i = (1 + t ) X i
r
r
r
DT T ( x ) T ( x ) x k
&
T ( x, t ) =
=
+
Dt
t
x k t
r
r
T ( x )
Note that T ( x ) is not a function of time, so
=0:
t

i {1,2}

i {1,2}

r
T x1
T x 2
T& ( x , t ) = 0 +
+
x1 {
t
x 2 {
t
V1 = X 1

r
x
x
T& ( x , t ) = 0 + 4 x1 1 + 4 x2 2
1+ t
1+ t

V2 = X 2

r
4 x2 4 x2
T& ( x , t ) = 1 + 2
1+ t 1+ t

The particle that at reference configuration was at position (1,1) , at time t = 1s will be at
position xi = (1 + t ) X i = 2 X i , i.e. ( x1 = 2; x 2 = 2 ):
T& ( x1 = 2; x 2 = 2; t = 1) =

4( 2) 2
1+1

4( 2) 2
1+1

= 16

Problem 2.26
Consider the equations of motion:
x1 = X 1 exp t + X 3 (exp t 1)

t
t
x 2 = X 2 + X 3 (exp exp )
x = X
3
3

Obtain the velocity and acceleration components in Lagrangian and Eulerian descriptions.
Solution:
First we obtain the inverse equations of motion:

University of Castilla-La Mancha


Ciudad Real - Spain

Draft

By: Eduardo W. V. Chaves (2014)

SOLVING PROBLEMS BY MEANS OF CONTINUUM MECHANICS

178

x1 = X 1 exp t + X 3 (exp t 1)

t
t
x 2 = X 2 + X 3 (exp exp )
x = X X = x
3
3
3
3

x1 X 1 exp t = x 3 (exp t 1)

x 2 X 2 = x 3 (exp t exp t )
x = X X = x
3
3
3
3

thus:
X 1 = x1 exp t exp t (exp t 1)

2t
t
X 2 = x 2 x 3 (exp 1)exp
X = x
3
3

(2.81)

or
x1

x2 =
x
3

exp t

0
0

X 1
X1
inverse
t
t
X 2 =
1 (exp exp ) X 2

X
X
0
1
3
3
0

(exp t 1)

exp t

0
0

exp t (exp t 1) x1

1 (exp 2t 1)exp t x 2
x
0
1
3
0

a) The velocity components in the material description are given by:


V1 = X 1exp t + X 3 exp t
r

D
V 2 = X 3 exp t + X 3 exp t = X 3 (exp t + exp t )
Vi =
x j ( X , t)
Dt
V = 0
3

(2.82)

b) The acceleration components in the material description are given by:


A1 = X 1exp t + X 3 exp t
r
r

DV i ( X , t )
Ai ( X , t ) =

A2 = X 3 (exp t exp t )
Dt
A = 0
3

(2.83)

To obtain the velocity and acceleration in the spatial description it is sufficient to replace
the values of X 1 , X 2 , X 3 , given by the equation (2.81), into the equations (2.82) and (2.83),
i.e.:
v1 = x1 + x 3

t
t
v 2 = x 3 (exp + exp )
v = 0
3
Velocity in the
spatial descriptio n

Problem 2.27
r

a1 = x1 + x 3

t
t
a 2 = x 3 (exp exp )
a = 0
3
Accelerati on in the
spatial descriptio n

r r

The motion of the continuum, x = x ( X , t ) , is given by the following equations:


x1 = 12 ( X 1 + X 2 )exp t + 12 ( X 1 X 2 )exp t

t
t
1
1
x 2 = 2 ( X 1 + X 2 )exp 2 ( X 1 X 2 )exp
x = X
3
3
0 t constant

Express the velocity components in the material and spatial descriptions.


Solution:
University of Castilla-La Mancha
Ciudad Real - Spain

Draft

By: Eduardo W. V. Chaves (2014)

2 CONTINUUM KINEMATICS

179

The velocity components using material description are:


r

Dx1 ( X , t ) 1
1
= ( X 1 + X 2 )exp t ( X 1 X 2 )exp t
V1 =
Dt
2
2

r
Dx 2 ( X , t ) 1
1

= ( X 1 + X 2 )exp t + ( X 1 X 2 )exp t
V 2 =
Dt
2
2

V3 = 0

(2.84)

To express the velocity components in the spatial description we need the inverse
r r r
equations of motion, i.e. we need to find X = X ( x , t ) :
(exp t

x1
t
(exp
x2 =
x
3

+ exp t )
2
exp t )
2
0

0
X 1

0 X 2


1 X 3

(exp t exp t )
2
(exp t + exp t )
2
0

(exp 2t + 1)exp t
X1

inverse

X 2 = (exp 2t 1)exp t
X 2
0
3

(exp 2t 1)exp t
(exp 2t + 1)exp t
0

0 x1

0 x 2
2 x 3

Then, to obtain the Eulerian velocity we replace the above equations into (2.84), which the
result is:
v1 = x 2

v 2 = x1

v3 = 0

Problem 2.28
Given the motion:
x i = ( X 1 + ktX 2 ) i1 + X 2 i 2 + X 3 i 3

i {1,2,3}

and the temperature field


T = x1 + x 2

Obtain the rate of change of T of a particle that in the current configuration is located at
point (1,1,1) .
Solution:
Considering the equations of motion:
x1 = X 1 + ktX 2

x2 = X 2

x3 = X 3

we replace xi into the temperature field, with that we obtain the temperature field in the
material description:
r
r
T ( x ) = x1 + x 2 T ( X , t ) = X 1 + ktX 2 + X 2

The material time derivative is given by:


r
DT D ( X 1 + ktX 2 + X 2 )
1,1,1)
T& ( X , t ) =
= kX 2 = k x 2 (
T& = k
=
Dt
Dt

Alternative solution:
University of Castilla-La Mancha
Ciudad Real - Spain

Draft

By: Eduardo W. V. Chaves (2014)

SOLVING PROBLEMS BY MEANS OF CONTINUUM MECHANICS

180

The material time derivative for a property expressed in the spatial description is given by:
DT T T x k
T& ( x1 , x 2 , x 3 , t ) =
=
+
Dt
t x k t

Considering T = x1 + x 2 , we obtain:
T T x1 T x 2 T x 3
+
+
+
T& ( x1 , x 2 , x 3 , t ) =
t x1 t
x 2 {
t
x 3 {
t
{
{
=0

=0

=0

=0

T& ( x1 , x 2 , x 3 , t ) = kX 2

we obtain the inverse equations of motion:


X 1 = x1 ktx 2
x1 = X 1 + ktX 2

inverse

X 2 = x 2
x2 = X 2
X = x
x = X
3
3
3
3
T& ( x1 , x 2 , x 3 , t ) = kX 2 = kx 2

For one particle in the current configuration at the position (1,1,1) we have:
T& ( x1 = 1, x 2 = 1, x 3 = 1, t ) = k

Problem 2.29
Given a steady velocity field: it asks readers to give their opinion on whether particle
velocities are constant or not. If not, in which situation is met. Justify the answer.
Solution:

A field ( x , t ) is said to be steady if the local rate of change does not vary over time, so:
r
( x , t )
=0
t

= ( x ) Steady state (stationary) field

(2.85)

For example, let us consider a stationary (steady state) velocity field as shown in Figure 2.3.
Then, as we can verify, the field representation for any time, e.g. t1 and t 2 , does not
change. However, that does not mean that the velocities of the particles do not change
over time. In light of Figure 2.3, we can now focus our attention on the fixed spatial point
r
r
r
x * . At time t1 the particle Q is passing through point x * with velocity v * . Let us also
consider another particle P , which is passing through another point with velocity
r
r
r
v P (t1 ) v * . At time t 2 the particle P is now passing through the point x * . It follows that
r
if we are dealing with a steady state velocity field, then the velocity of particle P at x *
r
r
r
must be v * , i.e. v P (t 2 ) = v * . We can easily contrast this with the material time derivative of
velocity, which is always associated with the same particle, i.e.:
r r
r r
r r r
r r r
r r
Dv ( x , t ) r r
v ( x , t )
a ( x, t ) =
+ xr v v ( x ) = xr v v ( x ) = a ( x )
Dt
t

1
424
3
r

(2.86)

= 0 (Stationay )

The rate of change of velocity (acceleration) will be zero if the velocity field is stationary
r r
r
v ( x , t ) r
= 0 and homogeneous ( xr v = 0 ).

University of Castilla-La Mancha


Ciudad Real - Spain

Draft

By: Eduardo W. V. Chaves (2014)

2 CONTINUUM KINEMATICS

181

We can also verify that, although spatial velocity is independent of time, that does not
mean material velocity is also, since:
r r r r r
r r
v ( x ) = v ( x ( X , t )) = v ( X , t )

t1

(2.87)

r r
v ( x)

r r
r
r
v ( x * , t1 ) = v * = v Q

Particle - Q

Particle- P
r
r
v P v*

r
x*

t2

r r
v ( x)

r r
r
r
v ( x * , t2 ) = v * = v P

Particle - P
r
x*

Figure 2.3: Steady velocity field.

2.2 Deformation/strain Tensors, Homogeneous Deformation


Problem 2.30
A rod, which is considered as a one-dimensional solid, undergoes a uniform stretching and
is given by = exp at where a = constant .
r

r r

a) Obtain the equations of motion x = x ( X , t ) ;


b) Obtain the rate-of-deformation tensor components, i.e. D -components.

University of Castilla-La Mancha


Ciudad Real - Spain

Draft

By: Eduardo W. V. Chaves (2014)

SOLVING PROBLEMS BY MEANS OF CONTINUUM MECHANICS

182

x1

= exp at

Figure 2.4.
Solution:
Using the approach in 1D we have:

ds dx
=
= exp at dx = exp at dX
dS dX
Integrating
dx = exp at dX
x1 = exp at X 1 + C
=

(2.88)

(2.89)

at t = 0 x = X , thus
x = exp 0 X 1 + C X = X + C C = 0

(2.90)

with that we obtain the equations of motion:


x1 = exp at X 1

x2 = X 2

x3 = X 3

v2 = 0

(2.91)

The velocity field components become:


v1 =

dx1
= a X 1 exp at = a x1
dt

v3 = 0

(2.92)

And the rate-of-deformation can be obtained as follows:


v j
1 v
D ij = i +
2 x j xi

a 0 0
Dij = 0 0 0
0 0 0

(2.93)

Problem 2.31
Consider the equations of motion:
x1 = X 1 + 2 X 3

x2 = X 2 2 X 3

x3 = X 3 2 X 1 + 2 X 2

Obtain the Green-Lagrange strain tensor components, i.e. E -components.


Solution 1:
The displacement field components are given by
u1 = x1 X 1 = 2 X 3

u 2 = x 2 X 2 = 2 X 3
u = x X = 2 X + 2 X
3
3
1
2
3

The Green-Lagrange strain tensor can be expressed in function of Lagrangian displacement


as follows:

University of Castilla-La Mancha


Ciudad Real - Spain

Draft

By: Eduardo W. V. Chaves (2014)

2 CONTINUUM KINEMATICS

E ij =

u j u k u k
1 u i
+
+

2 X j X i X i X j

u j
1 u
= i +
2 X j X i

183

1 u k u k
+
2 X i X j

u i
=
X j

sym

1 u k u k
2 X i X j

where the material (Lagrangian) displacement gradient is given by:


u1

X 1
u i u 2
=
X j X 1

u 3
X 1

u1

X 3 0 0 2
u 2
= 0 0 2

X 3

u 3 2 2 0
X 3

u1
X 2
u 2
X 2
u 3
X 2

Note that, in this case, the displacement gradient is an antisymmetric tensor. That is, the
symmetric part is the null tensor, and the remaining term is:
0 0 2 T 0 0 2 2 2 0

1 u u k 1
0 0 2 = 2 2 0
0
0
2
=

E ij = k

2 X i X j 2
2 2 0 2 2 0 0
0 4

Solution 2:
We can directly apply the definition Eij =
x1

X 1
xi x 2
=
Fij =
X j X 1

x3
X 1

1
1
C ij ij = Fki Fkj ij , where:
2
2
x1
X 2
x 2
X 2
x3
X 2

x1

X 3 1 0 2
x 2
= 0 1 2
X 3

x3 2 2 1
X 3

Thus
1 0 2 T 1 0 2 1 0 0 2 2 0

1
E ij = 0 1 2 0 1 2 0 1 0 = 2 2 0
2

2 2 1 2 2 1 0 0 1 0
0 4

Problem 2.32
Consider a homogeneous transformation defined by the following equations:
x1 = X 1 + 2 X 2 + X 3

x2 = 2 X 2

x3 = X 1 + 2 X 3

(2.94)

Show that, for a homogeneous transformation, vectors that are parallel in the reference
configuration remain parallel after deformation.
For the demonstration consider two vectors defined by the vector position of two particles
A and B in the reference configuration:
r
X A = e 1 + e 2

University of Castilla-La Mancha


Ciudad Real - Spain

r
X B = 2e 1 + 2e 2 + e 3

Draft

(2.95)

By: Eduardo W. V. Chaves (2014)

SOLVING PROBLEMS BY MEANS OF CONTINUUM MECHANICS

184

Solution:
The vector connecting the two particles in the reference configuration is given by:
r r r
V = B A = e 1 + e 2 + e 3

(2.96)

and the deformation gradient is:


1 2 1
xi
Fij =
= 0 2 0
X j
1 0 2

(2.97)

We can obtain the vector position of the particle in the current configuration by means of:
r
r
dx = F dX

r
r
x=FX

Homogeneous transformation

(2.98)

thus,
xiA

1 2 1 1 3
= 0 2 0 1 = 2
1 0 2 0 1

xiB

1 2 1 2 7
= 0 2 0 2 = 4
1 0 2 1 4

(2.99)

and the vector that connect these two points is:


r r
r
(2.100)
v = x B x A = 4e 1 + 2e 2 + 3e 3
r
then any vector parallel to V , for example the vector 2e 1 + 2e 2 + 2e 3 , after transformation
r
becomes: 8e 1 + 4e 2 + 6e 3 , which is parallel to v .

Problem 2.33
Consider a pure shear deformation represented by homogenous deformation:
r r
x = X + k t X 2e 1

(2.101)

where e i is the Cartesian basis, and the components of the above equation are:
x1 = X 1 + k t X 2

x2 = X 2

x3 = X 3

(2.102)

Obtain the geometry in the current configuration of Figure 2.5 which is represented by a
rectangle in the reference configuration.
X2
B

X1

Figure 2.5

University of Castilla-La Mancha


Ciudad Real - Spain

Draft

By: Eduardo W. V. Chaves (2014)

2 CONTINUUM KINEMATICS

185

Solution:
The deformation gradient:
1 k t 0
xi
Fij =
= 0 1 0
X j
0 0 1

(2.103)
r

Note that this is a case of homogenous deformation, i.e. x = F X + c with c = 0 .


The Jacobian determinant:
J = F =1

(2.104)

Note that, in this case there is no dilatancy.


The particles which are on the line BC have coordinates ( X 1 , X 2 ,0) in the reference
configuration, with that we obtain:
x1 = X 1 + k t X 2

x2 = X 2

x3 = 0

(2.105)

The particles which are on the line OA , coordinates ( X 1 ,0,0) , in the current configuration
assume the position:
x1 = X 1

x2 = 0

x3 = 0

(2.106)

then, the line OA does not change its position during motion, (see Figure 2.6).
x2
B

x1

Figure 2.6

Problem 2.34
Consider the equations of motion:
x1 = X 1 +

2
X2
2

x2 =

2
X1 + X 2
2

x3 = X 3

(2.107)

a) Show that this deformation is characterized by a homogeneous transformation;


b) Obtain the displacement field components in material and spatial descriptions;
c) Consider the particles that are located according to the equation:
X 12 + X 22 = 2

X3 = 0

Obtain the new configuration of these particles in the current configuration;

University of Castilla-La Mancha


Ciudad Real - Spain

Draft

By: Eduardo W. V. Chaves (2014)

SOLVING PROBLEMS BY MEANS OF CONTINUUM MECHANICS

186

d) Obtain the right Cauchy-Green deformation tensor components ( C ) and the GreenLagrange strain tensor ( E ).
e) Obtain the principal values of C and E .
Solution:
a) The equation of a homogeneous deformation is described by:
xi = Fij X j

(2.108)

where

xi
Fij =
=
X j

2
2

1
2
2
0

1
0

0
1

(2.109)

Note that the equation (2.108) is in accordance with (2.107):

x1
x =
2
x3

2
2

1
2
2
0

1
0

0
X1

0 X 2
1 X 3

(2.110)

Hence, we are dealing with homogeneous deformation. The inverse of (2.110) is obtained
as follows:
X1 2
X = 2
2
X 3 0

2
2
0

0 x1

0 x 2
1 x3

X 1 = 2 x1 2 x 2

X 2 = 2 x1 + 2 x 2
X = x
3
3

(2.111)

b) The displacement field is given by:

2
2
X 2 X1 =
X2
u1 = x1 X 1 = X 1 +
2
2

r r r
2
2

X1 + X 2 X 2 =
X1
u = x X u 2 = x 2 X 2 =
2
2

u 3 = x3 X 3 = 0

which, in spatial coordinates, becomes:

(2.112)

u1 = x1 X 1 = x1 2 x1 2 x 2 = x1 + 2 x 2

u 2 = x 2 X 2 = x 2 2 x1 + 2 x 2 = 2 x1 x 2

(2.113)

u 3 = x 3 X 3 = x3 x3 = 0

c) The particles describing a circle in the reference configuration:

University of Castilla-La Mancha


Ciudad Real - Spain

Draft

By: Eduardo W. V. Chaves (2014)

2 CONTINUUM KINEMATICS

X 12 + X 22 = 2

187

X3 = 0

(2.114)

in the current configuration becomes:

(2 x

2 x2

) + (
2

2 x1 + 2 x 2

=2

(2.115)

which is the same as:


3x12 + 3 x 22 4 2 x1 x 2 = 1 (an ellipse equation)

(2.116)

(see Figure 2.7).


2

Current configuration
1,5
1
0,5

Reference configuration

x2

0
-2

-1

-0,5
-1
-1,5
-2
x1

Figure 2.7
d) The right Cauchy-Green deformation tensor and the Green-Lagrange strain tensor are
given, respectively, by:
C = FT F

E=

1
(C 1)
2

(2.117)

Then, the C -components are:

C ij =

1
2
2
0

2
2
1
0

0
1

2
2

1
2
2
0

1
0

0 =

3
2
2
0

2
3
2
0

(2.118)

And the eigenvalues of (2.118) are:

University of Castilla-La Mancha


Ciudad Real - Spain

Draft

By: Eduardo W. V. Chaves (2014)

SOLVING PROBLEMS BY MEANS OF CONTINUUM MECHANICS

188

C1 =

3
3
+ 2 2.914 ; C 2 = 2 0.086 ; C 3 = 1
2
2

(2.119)

The E -components are:

1
E ij = C ij ij
2

1
=
2

3
2

0
1
1
0
0

1
0 0 1 0 = 2 2
4

0
1 0 0 1

2
3
2
0

2
0

2 2
1
0

0
0

(2.120)

The eigenvalues of E can be obtained as follows:

1+ 2 2
1 =
7

4
2
=0
2 16
1 2 2

2 =
4

(2.121)

1+ 2 2
1 2 2
0.957 ; E 2 =
0.457 ; E 3 = 0
4
4

(2.122)

4
2
2

2
2 =0
1

Then, the three eigenvalues of E are:


E1 =

Problem 2.35
Let us consider the following equations of motion:
x1 = X 1 +

1
X2
2

x2 =

;
r

1
X1 + X 2
2

x3 = X 3

(2.123)

a) Obtain the displacement field ( u ) in the Lagrangian and Eulerian descriptions;


b) Determine the material curve in the current configuration for a material circle defined in
the reference configuration as:
X 12 + X 22 = 2

X3 = 0

c) Obtain the components of the right Cauchy-Green deformation tensor and the GreenLagrange strain tensor;
d) Obtain the principal stretches.
Solution:
The deformation gradient is given by:
2 1 0
xi
1
Fij =
= 1 2 0
X j 2
0 0 2

J = F = 0.75

And by comparing this with the equations of motion in (2.123) we have:


x1
2 1 0 X 1
x = 1 1 2 0 X
2 2
2
x3
0 0 2 X 3

University of Castilla-La Mancha


Ciudad Real - Spain

Draft

x i = Fij X j

By: Eduardo W. V. Chaves (2014)

2 CONTINUUM KINEMATICS

189

So, we can verify that the proposed example is a case of homogeneous deformation in
r r
which c = 0 . The inverse form of the above equation is given by:
X1
4 2 0 x1
X = 1 2 4 0 x
2 3
2
X 3
0
0 3 x3

The displacement field is defined by


Lagrangian displacement become:

4
2

X 1 = 3 x1 3 x 2

2
4

(2.124)
X 2 = x1 + x 2
3
3

X 3 = x3

r
r r
u = x X , after which the components of the

r
1
1

u
X
(
, t ) = x1 X 1 = X 1 + X 2 X 1 = X 2
1

2
2

r
1
1

u i = xi X i u 2 ( X , t ) = x 2 X 2 = X 1 + X 2 X 2 = X 1
2
2

r
u 3 ( X , t ) = x3 X 3 = 0

(2.125)

The components of the Eulerian displacement can be obtained by substituting the Eulerian
description of motion (2.124) into (2.125), the result of which is:
r r
r

1
u1 ( X ( x, t ), t ) = 2 X 2 ( x , t ) =

r r

r
1
u 2 ( X ( x , t ), t ) = X 1 ( x , t ) =
2

r r
u ( X ( x , t ), t ) = u ( xr , t ) = 0
3
3

r
1 2
4
x1 + x 2 = u1 ( x , t )

2 3
3
r
1 2
4
x1 + x 2 = u 2 ( x , t )

2 3
3

(2.126)

The particles belonging to the circle X 12 + X 22 = 2 in the reference configuration will form
a new curve in the current configuration which is defined by:
2

X 12

X 22

2
4
4
2
= 2 x1 x 2 + x1 + x 2 = 2 20 x12 32 x1 x 2 + 20 x 22 = 18
3
3
3
3

which is an ellipse equation (Figure 2.7 shows the material curve in different
configurations).
The components of C and E can be obtained by using the definitions C = F T F and
E=

1
(C 1) :
2
C ij = Fki Fkj

1
E ij = C ij ij
2

2 1 0 2 1 0 1.25 1 0
1
C ij = 1 2 0 1 2 0 = 1 1.25 0
4
0 0 2 0 0 2 0
0 1
1.25 1 0 1 0 0 0.125 0.5 0

1
E ij = 1 1.25 0 0 1 0 = 0.5 0.125 0
2
0 1 0 0 1 0
0
0
0

In the principal space of C its components are given by:

University of Castilla-La Mancha


Ciudad Real - Spain

Draft

By: Eduardo W. V. Chaves (2014)

SOLVING PROBLEMS BY MEANS OF CONTINUUM MECHANICS

190

21

C ij = 0
0

0
23

0
22
0

C ij = 0
0

0
3

0
2
0

where i show the principal stretches. Therefore, to calculate these we need to obtain the
C eigenvalues:
1.25 C
1
C1 = 2.25
= 0 C 2 2.5C + 0.5625 = 0
1
1.25 C
C 2 = 0.25

21

C ij = 0
0

0
22
0

2.25
0
0

0.25 0
= 0
23 0
0
1
0
0

0
0

0
2
0

1.5 0 0

= 0 0.5 0
0 1
3 0
0
0

2.0

material curve
1.5
Reference Conf.
Current Conf.

1.0
0.5
x2

0.0
-2

-1

-0.5
-1.0
-1.5
-2.0
x1

Figure 2.8: Material curve.


Problem 2.36
Show that
Xr

[(detF ) F T ] = 0
r

(2.127)

r
r
dA .
Hint: The Nansons formula da = J F T dA , or da = da n = J F T N

Solution:
Considering the Nansons formula in indicial notation da n i = J Fki1N k dA , with J = det (F )
we apply the surface integral:

University of Castilla-La Mancha


Ciudad Real - Spain

Draft

By: Eduardo W. V. Chaves (2014)

2 CONTINUUM KINEMATICS

191

1
ki N k dA

n da = J F
i

(2.128)

S0

Note that, if we consider a function f , it holds that:

f da =

f ,i dV =

f
dV
xi

Denoting by f = 1 , we obtain:

da = 0 i

Returning to equation (2.128), and applying the divergence theorem to the integral on the
right of the equation we obtain:

n da = 0 = J F
i

1
ki N k dA =

1
ki
k

V0

S0

(J F ),

Xr

[(detF ) F

dV0 =

V0

J Fki1 dV0 = 0 i
X k

(2.129)

r
dV0 = 0

V0

Then, if the above volume integral is valid for the entire volume we can guarantee that is
also valid locally, i.e.:

[(detF ) F T ] = 0
r

Xr

Problem 2.37

r r

sym

(2.130)

r r

sym

Show that E& = F T Xr u& ( X , t)


and b) D = xr u& ( x , t) , where E is the GreenLagrange strain tensor and D is the rate-of-deformation tensor.
Solution:

) [

] [

1
D
D 1 T
1
E&
E=
F F 1 = F& T F + F T F& = ( F T F& ) T + ( F T F& ) = F T F&

2
Dt
Dt 2
2

sym

Note that:

r r
u& ( X , t )
D xi ( X , t )
Dxi ( X , t )
&
[
F&ij =
=
=
= Xr u& ( X , t ) ij
u i ( X , t )] = i
Dt X j X j
Dt
X j
X j

with that we demonstrate that:

D
E&
E = F T F&
Dt

sym

r r sym
= F T Xr u& ( X , t )

b)

] [

r r sym
r
r T
r r
1
1
T
sym
l + ( l ) = xr v + ( xr v ) = ( xr v ( x , t ))
= xr u& ( x , t )
2
2
r r
r r
where we have considered v ( x , t ) = u& ( x , t ) .
D=l

sym

University of Castilla-La Mancha


Ciudad Real - Spain

Draft

By: Eduardo W. V. Chaves (2014)

SOLVING PROBLEMS BY MEANS OF CONTINUUM MECHANICS

192

Problem 2.38
Consider the velocity field:
v1 = 5 x2 + 2 x3

v2 = 5 x1 3 x3

v3 = 2 x1 + 3 x2

Show that this motion corresponds to a rigid body motion.


Solution:
At first we obtain the spatial velocity gradient
v1

x1
r
vi ( x, t ) v 2
l ij =
=
x j
x
1

v3
x1

v1
x 2
v 2
x 2
v3
x 2

(l ), whose components are given by:

v1

x3 0 5 2
v 2
= 5
0 3

x3

0
v3 2 3
x3

(2.131)

Remember that ( l ) can be decomposed into a symmetric ( D ) and antisymmetric ( W )


part, i.e. l = D + W = W . Since D = 0 , there is no strain during motion, i.e. a rigid body
motion.
Problem 2.39
Let us consider the following velocity field:
v1 = 3 x2 + 1x3

v2 = 3x1 5 x3

v3 = 1x1 + 5 x2

Show that this motion corresponds to rigid body motion.


Solution: First we obtain the components of the spatial velocity gradient
v1

x1
r
vi ( x , t ) v 2
l ij =
=
x j
x
1

v3
x1

v1
x 2
v 2
x 2
v 3
x 2

(l ):

v1

x3 0 3 1
v 2
= 3
0 5 = l ijskew

x3

0
v3 1 5
x3

Taking into account that l can be decomposed into a symmetric ( l sym D ) and an
antisymmetric ( l skew W ) part, i.e. l = D + W , we can thus conclude that D = 0 , which is
a characteristic of rigid body motion.
Problem 2.40
The displacement field components are given by:
u 1 = 3 X 12 + X 2
;
u 2 = 2 X 22 + X 3
;
u 3 = 4 X 32 + X 1
r
Obtain the vector dx (current configuration) correspondent to the vector in the reference
r
configuration represented by dX at the point P(1,1,1) .

University of Castilla-La Mancha


Ciudad Real - Spain

Draft

By: Eduardo W. V. Chaves (2014)

2 CONTINUUM KINEMATICS

X 3 , x3

dX 1
dX k = dX 2
dX 3

r
dX

193

X 2 , x2
X 1 , x1

Solution:

To determine the vector dx we need to obtain the deformation gradient F . The


deformation gradient components can be obtained by using the relationship:
u i
Fij = ij +
X j

1
0
1 + 6 X 1

Fij = 0
1+ 4X2
1
1
0
1 + 8 X 3

The deformation gradient components evaluated at the point P(1,1,1) are:


Fij

7 1 0
= 0 5 1
1 0 9

Then, the vector components dx i are given by:


dx i = Fij dX j
dx1 7 1 0 dX 1 7 dX 1 + dX 2
dx = 0 5 1 dX = 5dX + dX
2
3
2
2
dx 3 1 0 9 dX 3 dX 1 + 9 dX 3

Problem 2.41
Consider a continuum in which the displacement field is described by the following
equations:
u1 = 2 X 12 + X 1 X 2

u 2 = X 22

u3 = 0

By definition, a material curve is always formed by the same particles. Let OP and OT be
material lines in the reference configuration, where O( X 1 = 0, X 2 = 0, X 3 = 0) ,
P ( X 1 = 1, X 2 = 1, X 3 = 0) and T ( X 1 = 1, X 2 = 0, X 3 = 0) . Find the material curves in the
current configuration. Also find the deformation gradient.
Solution:
a) The equations of motion can be obtained by means of the displacement field, i.e.:
u i = xi X i

University of Castilla-La Mancha


Ciudad Real - Spain

x1 = u1 + X 1

x2 = u 2 + X 2
x = u + X
3
3
3

Draft

x1 = X 1 + 2 X 12 + X 1 X 2

substituti ng

x2 = X 2 + X 22
the values of u 1 ,u 2 ,u 3
x = X
3
3

By: Eduardo W. V. Chaves (2014)

SOLVING PROBLEMS BY MEANS OF CONTINUUM MECHANICS

194

2.5

material curve

x2

1.5

1
0.5

Current Conf.

Reference Conf.

O0

0.5

1.5

2.5

3.5

4.5

x1

Figure 2.9: Deformation of the material curve OP .

Reference Conf.

x2

0.1
0.08

Reference Conf.

0.06
0.04
0.02
0

O0

T
0.5

1.5

2.5

3.5

x2

Current Conf.
0.1
0.08
0.06
0.04
0.02
0

O0

Current Conf.

T
0.5

1.5

2.5

3.5

x1

Figure 2.10: Deformation of the material curve OT .


Then, to obtain the material curve, one need only substitute the material coordinates with
the particles belonging to the line OP in the equations of motion, (see Figure 2.9). Notice
that the material curve OP in the current configuration is no longer a straight line, but the
line OT is still a straight line in the current configuration (see Figure 2.10).
University of Castilla-La Mancha
Ciudad Real - Spain

Draft

By: Eduardo W. V. Chaves (2014)

2 CONTINUUM KINEMATICS

195

The components of the deformation gradient can be obtained as follows:

F jk

x1

X 1
x
= 2
X
1
x 3
X 1

x1

X 3 (1 + 4 X + X )
X1
1
2
x 2
=
0
1 + 2X 2
X 3

0
0
x 3
X 3

x1
X 2
x 2
X 2
x 3
X 2

0
0
1

Problem 2.42
Starting from the definition
r
J& = J xr v , is valid.

Solution: Considering that Fij =

DF
D
[det (F )] = ij cof Fij , show that the equation
Dt
Dt

( )

x i
, the material time derivative of F det (F ) is given
X j

by:

D
[det ( F )] = D x i ( X , t ) cof Fij = D x i ( X , t ) cof Fij = D (v i )cof Fij
X j
Dt
Dt X j
Dt
X j

r
and considering that v i ( x ( X , t ), t ) , we can state that:

( )

( )

( )

D
[det (F )] = vi x k cof Fij
x k X j
Dt

( )

By referring to the definition of the cofactor: [cof (Fij )]T = (Fij )1 det (Fij ) , we can also state
the following is valid:
D
[det (F )] = vi xk Fij
Dt
xk X j

( )

( )

det Fij =

vi
Fkj F ji
x k

( )

( )

det Fij =

vi
v
ki det Fij = i det Fij
x k
xi

( )

( )

= Jvi ,i

An alternative solution is presented in Problem 1.117 in Chapter 1.


Problem 2.43
r

Let dx be a differential line element in the current configuration. Find the material time
r
derivative of dx .
Solution:
r
r
r
r
r
r r
D r D
D
D
dx =
( F dX ) =
( F ) dX + F
( dX ) = l 1
F2
d3
X = l dx xr v dx
r
Dt
Dt
Dt
Dt
1
424
3
dx
r
0

And, whose components are represented by:


r
v i ( x , t )
D r
dx = v i , k dx k =
dx k

x k
Dt i

Problem 2.44
Let us consider the equations of motion:
x1 = X 1 + 4 X 1 X 2

x2 = X 2 + X 22

x3 = X 3 + X 32

Find the Green-Lagrange strain tensor ( E ).


University of Castilla-La Mancha
Ciudad Real - Spain

Draft

By: Eduardo W. V. Chaves (2014)

SOLVING PROBLEMS BY MEANS OF CONTINUUM MECHANICS

196

Solution: Referring to the E equation:


E=

1
( F T F 1)
2

E ij =

1
( Fki Fkj ij )
2

(2.132)

where the components of F are derived as:


x1

X 1
x k x 2
Fkj =
=
X j X 1

x 3
X 1

x1
X 2
x 2
X 2
x 3
X 2

x1

X 3 (1 + 4 X )
4X1
2
x 2
0
1 + 2X 2
=
X 3

0
0
x 3
X 3

0
1 + 2 X 3
0

And,
0
0 (1 + 4 X 2 )
4X1
(1 + 4 X 2 )

= 4X1
1 + 2X 2
0
0
1 + 2X 2

0
0
1 + 2X 3
0
0
2

(1 + 4 X 2 )
(1 + 4 X 2 ) 4 X 1
0

2
2
0
= (1 + 4 X 2 ) 4 X 1 ( 4 X 1 ) + (1 + 2 X 2 )

X
0
0
(
1
2
)
+
3

Fki Fkj

0
0
1 + 2 X 3

Then substituting the above into the equation in (2.132) we obtain:


(1 + 4 X 2 ) 2 1

(1 + 4 X 2 ) 4 X 1
0

1
2
2
0
E ij = (1 + 4 X 2 ) 4 X 1 ( 4 X 1 ) + (1 + 2 X 2 ) 1

2
2

0
0
(
1
+
2
)

1
X
3

Problem 2.45
Obtain the principal invariants of E in terms of the principal invariants of C and b .
Solution:
The principal invariants of E are given by:
I E = Tr ( E )

II E =

1 2
I E Tr ( E 2 )
2

III E = det ( E )

1
2

Considering E = (C 1) , the principal invariants can also be expressed as follows:


The First Invariant:
1
1
1
1
I E = Tr ( E ) = Tr (C 1) = Tr (C 1) = [Tr (C ) Tr (1) ] = (I C 3)
2
2
2
2

The Second Invariant:


II E =

1 2
I E Tr ( E 2 )
2

where
2

1
1

I E2 = (I C 3) = I C2 6 I C + 9
4
2

University of Castilla-La Mancha


Ciudad Real - Spain

Draft

By: Eduardo W. V. Chaves (2014)

2 CONTINUUM KINEMATICS
2

197

) [ ( )

1
1
1
1

Tr ( E 2 ) = Tr (C 1) = Tr (C 1) 2 = Tr C 2 2C + 1 = Tr C 2 2 Tr (C ) + Tr (1)
4
4
4
2

1
= Tr C 2 2 I C + 3
4

[ ( )
]
The term Tr (C ) can be obtained as follows:
2

C C = C

C12

C ij 2 = 0
0

0 Tr C 2 = C12 + C 22 + C 32
C 32

0
C 22

( )

It is also true that:

I C2 = (C1 + C 2 + C 3 ) = C12 + C 22 + C 32 + 2 C1 C 2 + C1 C 3 + C 2 C 3
1444424444
3
2

II C

C12 + C 22 + C 32 = I C2 2 II C

Therefore we have:
Tr ( E 2 ) =

1 2
I C 2 II C 2 I C + 3
4

Whereupon, the second invariant can also be expressed as:


II E =

1 1 2
1
1
I C 6 I C + 9 I C2 2 II C 2 I C + 3 = ( 2 I C + II C + 3)

2 4
4
4

The Third Invariant:


3

1
1
III E = det ( E ) = det (C 1) = det [(C 1)]
2
2

The term det[(C 1)] can also be expressed as:


C1 1
0
det (C 1) = 0
C2 1
0

0
0

= (C1 1)(C 2 1)(C 3 1)

C3 1

= C1C 2 C 3 C1C 2 C1C 3 C 2 C 3 + C1 + C 2 + C 3 1 = III C II C + I C 1

Then:
III E =

1
( III C II C + I C 1)
8

In short we have:
1
(I C 3 )
2
1
II E = ( 2 I C + II C + 3)
4
1
III E = ( III C II C + I C 1)
8

I C = 2I E + 3

IE =

II C = 4 II E + 4 I E + 3
III C = 8 III E + 4 II E + 2 I E + 1

Problem 2.46
Let = (I C , II C , III C ) be a scalar-valued tensor function, where I C , II C , III C are the
principal invariants of the right Cauchy-Green deformation tensor C . Obtain the
University of Castilla-La Mancha
Ciudad Real - Spain

Draft

By: Eduardo W. V. Chaves (2014)

SOLVING PROBLEMS BY MEANS OF CONTINUUM MECHANICS

198

derivative of with respect to C and with respect to b . Check whether the following
equation is valid F ,C F T = ,b b or not.
Solution:
Using the chain rule of derivative we obtain:

,C =

(I C , II C , III C ) I C
II C
III C
=
+
+
C
I C C II C C
III C C

(2.133)

Considering the partial derivative of the invariants:


I C
II C
III C
=1 ,
= IC 1 C T = IC 1 C ,
= III C C T = III C C 1 , we can obtain:
C
C
C

,C =

(I C 1 C ) + III C C 1
1+
I C
II C
III C

,C =
I C 1
III C C 1
+
C+
I
I
I
I
I
I
I
I

C
C
C
C

(2.134)

It is also true that:


,b =
I b 1
III b b 1
+
b+
II b
III b
I b II b

(2.135)

We apply the dot product of the above equation with F on the left and with F T on the
right, i.e.:

F ,C F T =
I C F 1 F T
F C F T +
III C F C 1 F T
+
II C
III C
I C II C

(2.136)
And considering the following relationships:
F 1 F T = F F T = b
C = F T F F C F T = F F T F F T = b b = b2

And considering the relationship C 1 = F 1 b 1 F we conclude that:


C 1 = F 1 b 1 F F C 1 F T = F F 1 b 1 F F T = b 1 b

Then, the equation in (2.136) can be rewritten as follows:


+
F ,C F T =
I C b
b +
III C b 1 b
II C
III C
I C II C

+
F ,C F T =
I C 1
b+
III C b 1 b
II C
III C
I C II C

It is also valid that:


+
F ,C F T =
I b 1
B+
III b b 1 b
II b
III b
I b II b

F ,C F T = , b b

University of Castilla-La Mancha


Ciudad Real - Spain

Draft

By: Eduardo W. V. Chaves (2014)

2 CONTINUUM KINEMATICS

199

Taking into account the equation (2.135) we can conclude that the equation ,b b = b ,b
is valid, indicating that the tensors ,b and b are coaxial.
Problem 2.47
Show that the Green-Lagrange strain tensor ( E ) and the right Cauchy-Green deformation
tensor ( C ) are coaxial tensors.
Solution:
Two tensors are coaxial if they have the same principal directions. Coaxiality can also be
demonstrated if the relation C E = E C holds.
Starting with the definition C = 1 + 2 E , we can conclude that:
C E = (1 + 2 E ) E = 1 E + 2 E E = E (1 + 2 E ) = E C

Thus, we can prove that E and C are coaxial tensors.


Problem 2.48
Obtain

the

E& = F T D F

relationship

starting

from

the

definition

r
r
D
(ds ) 2 (dS ) 2 = dX 2 E dX . Get also the relationship between
(ds ) 2 and D .
Dt

Solution:

Taking the material time derivative of (ds ) 2 (dS ) 2 = dX 2 E dX we obtain:

D
(ds ) 2 (dS ) 2
Dt

The term

r
D
D r
dX 2 E dX
(ds ) 2
=
Dt
Dt
r&
r
r
r
r
r&
D r r
[dx dx ] = 2d{
=
X E dX + 2dX E& dX + 2dX E d{
X
Dt
=0
=0
r
r
r D r
&
= 2 dx
[dx ] = 2dX E dX
Dt
=

D r
[dx ] can be expressed as follows:
Dt

r
D
D r
[
]
d
d
x
F
X

=
Dt
Dt r

& dX
F
=

= l F dX

D
D x k
[dx k ] =

Dt X i
Dt
D x k

Indicial


=
Dt X i

= k dX i

X i

dX i

D x k
dX i =
dX i

DX i t

with that we conclude that:


r
r
2dX E& dX

University of Castilla-La Mancha


Ciudad Real - Spain

r D r
[dx ]
= 2dx
Dt
r
r
= 2dx l F dX
r
r
= 2 F dX l F dX
r
r
= 2dX F T l F dX

Draft

By: Eduardo W. V. Chaves (2014)

SOLVING PROBLEMS BY MEANS OF CONTINUUM MECHANICS

200

We can apply the additive decomposition of the spatial velocity gradient ( l ) into a
symmetric ( D ) and an antisymmetric ( W ) part:
r
r
2dX E& dX

r
r
= 2 dX F T l F dX
r
r
= 2dX F T (D + W ) F dX
r
r
r
r
= 2 dX F T D F dX + 2 dX F T W F dX
r
r
= 2 dX F T D F dX
r
r
r
r
r
r
Note that dX F T W F dX = dx W dx = W : (dx dx ) = 0 , since W is
r
r
antisymmetric tensor and (dx dx ) is a symmetric tensor. Then, we conclude that:

an

E& = F T D F

D
(ds ) 2 and D as follows:
Dt
r
r
r
r
D
(ds ) 2 = 2dX F T D F dX = 2dx D dx
Dt

With that it is possible to relate

Problem 2.49
Obtain the material time derivative of the Jacobian determinant ( J& ) in terms of ( E& ), ( C& ),
( F& ).
Solution:
We starting from the relationship J& = J Tr (D ) , where D is the rate-of-deformation tensor
which is related to E& by means of the relationship D = F T E& F 1 , then:

) (

J& = J Tr (D) = J Tr F T E& F 1 = J F T E& F 1 : 1

In indicial notation we have:


J
J& = J Fki1 E& kp F pj1 ij = J Fki1 F pi1 E& kp = J ( F 1 F T ) : E& = J C 1 : E& = C 1 : C&
2

The J& can still be expressed in terms of F& . To this end let us consider the following

1
equation E& kp = (F&sk Fsp + Fsk F&sp ) . Then, J& can also be expressed by:
2

J
1
J& = J Fki1 F pi1 E& kp = J Fki1 F pi1 F&sk Fsp + Fsk F&sp = Fki1 F pi1 F&sk Fsp + Fki1 F pi1 Fsk F&sp
2
2
J
J
= si Fki1 F&sk + si F pi1 F&sp = Fks1 F&sk + F ps1 F&sp = JFts1 F&st = JF&st Fts1
2
2
T
T
&
&
= JF : F = JF : F

In short, there are various different ways to express the material time derivative of the
Jacobian determinant:
J 1 &
C :C
= JF& : F T
2
J
= J Tr (C 1 E& ) = Tr (C 1 C& ) = J Tr ( F& F 1 )
2

J& = J Tr (D) = J C 1 : E&

where we have used the trace property: A : B = Tr ( A B T ) = Tr ( A T B ) in which A and B


are arbitrary second-order tensors.

University of Castilla-La Mancha


Ciudad Real - Spain

Draft

By: Eduardo W. V. Chaves (2014)

2 CONTINUUM KINEMATICS

201

Problem 2.50
The displacement field components are given by:
u1 = 0.1 X 22

u2 = 0

u3 = 0

a) Is this motion possible? Justify the answer;


b) Obtain the right Cauchy-Green deformation tensor;

c) Find the current vectors related to the material vectors b = 0.01e 1 and c = 0.015e 2 , said
vectors are at the point P (1,1,0) in the reference configuration;
r

d) Obtain the stretches of the vectors b and c , at the point P (1,1,0) ;


r

e) Find the angle variation defined by the two vectors b and c .


Solution:
a) A motion is possible if the Jacobian determinant is positive. The deformation gradient is
given by:
1 0 0 0 0 .2 X 2
u i
Fij = ij +
= 0 1 0 + 0
0
X j
0 0 1 0
0

0 1 0 .2 X 2
0 = 0
1
0 0
0

0
0
1

The determinant is Fij = J = 1 > 0 . Then, the motion is possible.


b) The right Cauchy-Green deformation tensor is defined as C = F T F , then the
components are given by:
0 0 1 0.2 X 2
1

1
C ij = 0.2 X 2 1 0 0
0
0 1 0
0
r
c) The vector b = 0.01e 1 at the point P(1,1,0)
r
b = F

r
b

0 1
0 = 0.2 X 2
1 0

0 .2 X 2
0
2
2
0.2 X 2 + 1 0
0
1

deforms according to:

b 1 1 0.2 1 0 0.01 0.01


b = 0
1
0 0 = 0
2
b 3 0
0
1 0 0

and the vector c = 0.015e 2 in the current configuration becomes:


c 1 1 0.2 1 0 0 0.003
c = 0
1
0 0.015 = 0.015
2
c 3 0
0
1 0 0

d) The stretch can be obtained as follows:


br =

r
b
r
b

0.012
=1
0.01

and the stretch of c is given as:


cr =

University of Castilla-La Mancha


Ciudad Real - Spain

r
c
r
c

0.003 2 + 0.015 2
= 1.0198 1.02
0.015

Draft

By: Eduardo W. V. Chaves (2014)

SOLVING PROBLEMS BY MEANS OF CONTINUUM MECHANICS

202

Alternative solution: Taking into account that M = M C M and by evaluating C at the


point P we obtain:
1
C ij ( X 1 = 1, X 2 = 1, X 3 = 0) = 0.2 X 2
0

0
0 .2
+ 1 0
0
1
0 .2 X 2
2

X 22

0 .2 0
1

= 0.2 1.04 0
0
0 1

Then, by applying b = b C b and c = c C c we can obtain:


b

0 . 2 0 1
1

= [1 0 0 ] 0.2 1.04 0 0 = 1
0
0
1 0

0.2 0 0
1

= [0 1 0 ] 0.2 1.04 0 1 = 1.04


0
0
1 0

b = 1

c = 1.0198
r

e) In the current configuration the angle between the vectors b and c can be obtained
according to the relation:
r r
b c
cos = r r
b c

cos =

(0.01e 1 + 0e 2 + 0e 3 ) (0.003e 1 + 0.015 e 2 + 0e 3 )


0.01 2

0.003 2 + 0.015 2

0.00003
0.01 0.000234

= 0.196116135

= arccos(0.196116135) 78.69

In the reference configuration the angle between these two vectors is 90 , then angle
variation is:
= 90 78 .69 = 11 .3

Alternative solution: Given two directions in the reference configuration represented by their
unit vectors M and N , the angle formed by these unit vectors in the current configuration
(after motion) is given by:
cos =

M C N
M C N
=
M N
M C M N C N

Denoting by M = b and N = c it fulfills that:


0 .2 0 0
1

b C c = [1 0 0] 0.2 1.04 0 1 = 0.2


0
0
1 0

Then,
cos =

0 .2
b C c
b C c
=
=
= 0.196116135
b c
1 1.04
b C b c C c

University of Castilla-La Mancha


Ciudad Real - Spain

Draft

By: Eduardo W. V. Chaves (2014)

2 CONTINUUM KINEMATICS

203

Problem 2.51
Obtain an equation for mass density in terms of the third invariant of the right CauchyGreen deformation tensor, i.e. 0 = 0 ( III C ) .
Solution:
Starting by the definition:

0 ( X ) = ( x, t) J
and considering that the third invariant III C = det (C ) = det ( F T F ) = J 2 , we obtain
J = III C , then:

0 =

(2.137)

III C

Problem 2.52
At a certain moment, the displacement field of a continuous medium is:
u1 = (a1 1) X 1

u 2 = (a 2 1) X 2 + a1X 1

u 3 = (a 3 1) X 3

where is a constant. Determine a1 , a 2 and a 3 knowing that the solid is incompressible,


that a segment parallel to the X 3 -axis does not stretch and that any element area defined in
the plane X 1 X 3 remains unchanged.
Solution:
r

Based on the definition of the displacement field, i.e. u = x X , we obtain:


u1 = x1 X 1 = (a1 1) X 1

x1 = a1 X 1

u 2 = x 2 X 2 = (a 2 1) X 2 + a1X 1
u 3 = x 3 X 3 = (a 3 1) X 3

x 2 = a 2 X 2 + a1X 1

x3 = a3 X 3

Then, the equations of motion are:


x1 = a1 X 1

x 2 = a 2 X 2 + a1X 1
x = a X
3 3
3

0
x1 a1

x 2 = a1 a 2
x 0
0
3

0 X 1

0 X 2 (homogeneous deformation
a 3 X 3

which is possible to establish that F = a1 a 2 a 3 > 0 .


By means of the incompressibility condition dV = F dV0 F J = 1 , the following
relationship is true:
a1 a 2 a 3 = 1

= [0 0 1] , does not stretch that


By the fact that a segment parallel to the X 3 -axis, e.g. M
i
implies that the stretching according to this direction is unitary, i.e. M = 1 , thus

E M
= 1 + 2E = 1
M = 1 + 2M
33

The components of the Green-Lagrange strain tensor ( E =

University of Castilla-La Mancha


Ciudad Real - Spain

Draft

E 33 = 0

1 T
F F 1 ) are given by:
2

By: Eduardo W. V. Chaves (2014)

SOLVING PROBLEMS BY MEANS OF CONTINUUM MECHANICS

204

a1

a1
1
E ij = 0
2
0

a2
0

0 a1
0 a1
a 3 0

a12 + a12 2 1 a1 a 2
0 1 0 0
0

2
a1 a 2
a2 1
0 0 1 0 =
0
2
a 3 0 0 1
a 32 1
0
0

0
a2
0

thus:
E 33 = a 32 1 = 0

a 3 = 1

Any element area on the plane X 1 X 3 does not change


0
x1 a1

x 2 = a1 a 2
x 0
0
3

0 X 1

0 X 2
a 3 X 3

with N i(1) = [1 0 0] and N i(3) = [0 0 1] we obtain:


n (i1)

0
a1

= a1 a 2
0
0

0 1 a1

0 0 = a1
a 3 0 0

n i(3)

0
a1

= a1 a 2
0
0

0 0 0

0 0 = 0
a 3 1 a 3

Then, the area in the current configuration is obtained as follows:


e 1
r (1) r (3)
n n = a1
0

e 2 e 3
a1 0 = a1e 1 a1 a 3 e 2 + 0e 3
0

a3
r

and its module does not change N (1) N (3) = n (1) n (3) = 1 :
r
r
n (1) n (3) = 1 = (a1 ) 2 + (a1 a 3 ) 2 a12 a 32 2 + a12 a 32 = 1

We have previously obtained that a 32 = 1 , with that we obtain:


a12 a 32 2 + a12 a 32 = 1

a12 2 + a12 = 1 a12 =

1
1
a1 =
2
(1 + )
(1 + 2 )

with that we conclude that:


a1 =

1
(1 + )
2

a 2 = (1 + 2 )

a3 = 1

Problem 2.53
Consider the solid shown in Figure 2.11 which is subjected to a homogenous deformation.
r r

a) Obtain the general expression of the material displacement field U ( X , t ) in function of


the material displacement gradient tensor J .
r r
b) Obtain U ( X , t ) knowing that also holds the following boundary conditions:
r
r
u 2 ( X , t ) = u3 ( X , t ) = 0

X 1 , X 2 , X 3

u1 ( X 1 = 0, X 2 , X 3 , t ) = 0
u1 ( X 1 = L, X 2 , X 3 , t ) =

c) Justify the possible values (positive and negative) that can take .

University of Castilla-La Mancha


Ciudad Real - Spain

Draft

By: Eduardo W. V. Chaves (2014)

2 CONTINUUM KINEMATICS

205

d) Calculate the material and spatial strain tensors and the infinitesimal strain tensor.
x3

x1

x2

Figure 2.11

Solution:

A homogeneous deformation is characterized by F ( X , t ) = F (t ) . In addition, we know


that:
r
r
Homogeneous deformation
F ( X , t ) = 1 + J ( X , t )

F (t ) = 1 + J (t )

where J is the material displacement gradient tensor. Note that the homogenous
deformation is not dependent on the vector position, with that we can obtain:
r r
r
r r
u( X , t )
J (t ) =
r

J (t ) dX = du( X , t )
X
r
where c (t ) is the constant of integration. Then:
r r
r r
u( X , t ) = J (t ) X + c (t )

r r
r r
u( X , t ) = J (t ) X + c (t )

In components:
u1 J 11 X 1 + J 12 X 2 + J 13 X 3 c1


u 2 = J 21 X 1 + J 22 X 2 + J 23 X 3 + c 2
u J X + J X + J X c
32 2
33 3
3
3 31 1

b) From the conditions in paragraph b) we can conclude that:


r

condition 1) u 2 ( X , t ) = u 3 ( X , t ) = 0

X 1 , X 2 , X 3 :

u1 J 11 X 1 + J 12 X 2 + J 13 X 3 c1
J 21 = 0; J 22 = 0; J 23 = 0, c 2 = 0

u 2 = 0 = J 21 X 1 + J 22 X 2 + J 23 X 3 + c 2
u = 0 J X + J X + J X c J 31 = 0; J 32 = 0; J 33 = 0, c3 = 0
32 2
33 3
3
31 1
3

condition 2) u1 ( X 1 = 0, X 2 , X 3 , t ) = 0 :
u1 = 0 J 11 X 1 + J 12 X 2 + J 13 X 3 c1

0
+ 0 {J 12 = 0; J 13 = 0, c1 = 0
u2 =
0
u
0

3

condition 3) u1 ( X 1 = L, X 2 , X 3 , t ) =

University of Castilla-La Mancha


Ciudad Real - Spain

Draft

By: Eduardo W. V. Chaves (2014)

SOLVING PROBLEMS BY MEANS OF CONTINUUM MECHANICS

206

u1 J 11 L 0



u 2 = 0 + 0 J 11 =
L
u = 0 0
3

Hence, the components of the displacement gradient are:

L
J ij = 0

0 0
0 0

0 0

And the displacement field components are:

X1

L
r r
r
r r
components

u i ( X , t ) = 0
u( X , t ) = J (t ) X + c (t )

c) The motion is possible and has physical meaning if F > 0 :



1 + L
components
F (t ) = 1 + J (t ) Fij = 0

0 0

1 0 F = 1 + > 0 > L

L
0 1

The material strain tensor (the Green-Lagrange strain tensor):

E=

1 T
F F 1
2

1 2
+
2
L 2 L
0
E ij =

components

0 0

0 0
0 0

The spatial strain tensor (the Almansi strain tensor):

e=

1
1 F FT
2

components

1 2
1 0 0
+
2
L 2 L

eij =
2
0 0 0

1 + 0 0 0
L

The infinitesimal strain tensor:

L
ij = 0

University of Castilla-La Mancha


Ciudad Real - Spain

Draft

0 0
0 0

0 0

By: Eduardo W. V. Chaves (2014)

2 CONTINUUM KINEMATICS

207

Problem 2.54
The tetrahedron shown in Figure 2.12 undergoes homogeneous deformation ( F = const. )
with the following consequences:
1. The points O , A and B do not move;
2. The solid volume becomes " p" times the initial volume;
p

3. The length of the segment AC becomes

times the initial length;

4. The angle AOC becomes 45 .


a) Justify why we cannot use the infinitesimal deformation theory;
b) Obtain the deformation gradient, and the possible values of " p" and displacement field
in material and spatial descriptions;
c) Draw the deformed solid.
x3
C

x2

A
x1

Figure 2.12.

Solution:
a) The angle AOC = 90 becomes 45 , so we are not dealing with a small deformation,
since in the case of small deformation << 1 , and in this problem we have
<<

0.7854 ;
4

b) We have a case of homogeneous deformation. Then, the equations of motion are given
by:
r r
r
x = F (t ) X + c (t )

x1 F11

x 2 = F21
x F
3 31

F12
F22
F32

F13 X 1 c1

F23 X 2 + c 2
F33 X 3 c3

The point O( X 1 = 0, X 2 = 0, X 3 = 0) does not move:


0 F11

0 = F21
0 F
31

F12
F22
F32

F13 0 c1

F23 0 + c 2
F33 0 c3

c1 0

c 2 = 0
c 0
3

The point A( X 1 = a, X 2 = 0, X 3 = 0) does not move:

University of Castilla-La Mancha


Ciudad Real - Spain

Draft

By: Eduardo W. V. Chaves (2014)

SOLVING PROBLEMS BY MEANS OF CONTINUUM MECHANICS

208

a F11

0 = F21
0 F
31

F12
F22
F32

F13 a

F23 0
F33 0

a aF11


0 = aF21
0 aF
31

F11 = 1

F21 = 0
F = 0
31

The point B( X 1 = 0, X 2 = a, X 3 = 0) does not move:


0 1 F12

a = 0 F22
0 0 F
32

F13 0

F23 a
F33 0

0 aF12

a = aF22
0 aF
32

F12 = 0

F22 = 1
F = 0
32

Gathering the above information, we have:


1 0 F13
Fij = 0 1 F23
0 0 F33

F = F33 > 0

The volume of the solid becomes " p" times the initial volume. The relationship between the initial
(reference) volume and the current (final) volume is given by:
dV = F dV0

dV = F dV

V final = F Vinitial = F33Vinitial

where we have considered the homogeneous deformation case. With this, we conclude that
F33 = p

(The length of segment AC becomes

p
2

times the initial length). As we are dealing with a

homogeneous deformation, a line in the reference configuration will remain a line in the
current configuration.
The point C ( X 1 = 0, X 2 = 0, X 3 = a ) moves to:
x1C 1 0 F13 0
C

x 2 = 0 1 F23 0
x C 0 0 p a

3

x1C aF13
C

x 2 = aF23
x C ap

The length of segment AC in the reference configuration is L AC = a 2 . The vector that


connect the points A A( x1 = a, x 2 = 0, x3 = 0) and C ( x1 = aF13 , x 2 = aF23 , x3 = ap) in the
current configuration is given by:
AC = (aF13 a )e 1 + (aF23 )e 2 + (ap)e 3

and its magnitude is:


AC = l AC = (a ( F13 1)) 2 + (aF23 ) 2 + (ap) 2 = a ( F13 1) 2 + ( F23 ) 2 + ( p) 2

Using the information provided by the problem l AC =


l AC =
a ( F13 1) 2 + ( F23 ) 2 + ( p ) 2 =

p
2

L AC , we obtain:

p
2
p
2

L AC
a 2

( F13 1) 2 + ( F23 ) 2 + ( p ) 2 = p
University of Castilla-La Mancha
Ciudad Real - Spain

Draft

By: Eduardo W. V. Chaves (2014)

2 CONTINUUM KINEMATICS

209

thus
( F13 1) 2 + ( F23 ) 2 + p 2 = p 2

( F13 1) 2 + ( F23 ) 2 = 0

F13 = 1

F23 = 0

Then, the deformation gradient components are:


1
0
p

1 0
Fij = 0 1
0 0

The angle AOC changes to 45 .


dX i(1) = [1 0 0]

dX i( 2 ) = [0 0 1]

dxi(1) = Fij dX (j1)

dx1( 2) 1 0 1 0 1
( 2)

dxi( 2 ) = Fij dX (j 2 )

dx 2 = 0 1 0 0 = 0
dx ( 2) 0 0 p 1 p

3
r
r
dx (1) dx ( 2 )
2
cos( AOC ) = cos(45 ) = r (1) r ( 2 ) =
2
dx
dx

dx1(1) 1 0 1 1 1
(1)

dx 2 = 0 1 0 0 = 0
dx (1) 0 0 p 0 0

3

where dx (1) = 1 , dx ( 2) = 1 + p 2 , dx (1) dx ( 2) = 1 . Then:


1
1+ p2

2
2

p = 1

As the Jacobian determinant must be greater than zero F = p > 0 , this implies that p = 1 :
1 0 1
Fij = 0 1 0
0 0 1

The equations of motion become:


x1 1 0 1 X 1 X 1 + X 3


x 2 = 0 1 0 X 2 = X 2
x 0 0 1 X X

3
3
3

The material displacement field becomes:


r r
r r
u( X , t ) = x X

u1 X 1 + X 3 X 1 X 3


u 2 = X 2 X 2 = 0
u X
X 0
3
3
3

The spatial displacement field becomes:


u1 x3

u 2 = 0
u 0
3

University of Castilla-La Mancha


Ciudad Real - Spain

Draft

By: Eduardo W. V. Chaves (2014)

SOLVING PROBLEMS BY MEANS OF CONTINUUM MECHANICS

210

x3

c)

x1C aF13 a
C

x 2 = aF23 = 0
x C ap a

3

a
a

B = B

x2

A = A

x1

Problem 2.55
Consider the following equations of motion:
x1 = X 1

x2 = X 2 X 3

x3 = X 3 + X 2

a) Obtain the deformation gradient, the right Cauchy-Green deformation tensor, the left
Cauchy-Green deformation tensor, the Green-Lagrange strain tensor and the Almansi
strain tensor. Check whether this case represents a homogeneous deformation.
b) Obtain the right stretch tensor, the spin tensor of polar decomposition and the principal
space of the left Cauchy-Green deformation tensor of the polar decomposition.
c) Obtain the final length of an initial length element equal to 2 which is in the X 3 direction, and the angular distortion of an initial angle 30 which is in the plane X 1 X 2 .
d) Obtain the strain tensor by considering the small deformation regime.
Solution:
a) The deformation gradient ( F )
x1
x1
x1

0
X 1 X 2 X 3 1 0
xi x 2 x 2
x 2
Fij =
= 0 1
=
X j X 1 X 2 X 3

x3 0 1
x3 x3
X 1 X 2 X 3
r
r
In general we have dx = F dX , and if we are dealing with a homogeneous deformation (a
r r
r
particular case of motion) the relationship x = F X + c holds, a fact that can be checked
r r
by means of the equations of motion in matrix form with c = 0 :

0 X1
x1 1 0
x = 0 1 X
2
2
x3 0 1 X 3

University of Castilla-La Mancha


Ciudad Real - Spain

Draft

By: Eduardo W. V. Chaves (2014)

2 CONTINUUM KINEMATICS

211

The right Cauchy-Green deformation tensor ( C = F T F ):


0 1 0
0 1
0
0
1 0

2
0
C ij = Fki Fkj = 0 1 0 1 = 0 1 +
0 1 0 1 0
0
1 + 2

The left Cauchy-Green deformation tensor ( b = F F T )


0 1 0
0 1
0
0
1 0

2
0
bij = Fik F jk = 0 1 0 1 = 0 1 +
0 1 0 1 0
0
1 + 2

1
2

The Green-Lagrange strain tensor ( E = (C 1) ) and the Almansi strain tensor


1
2

( e = (1 b 1 ) ) are defined by their components as follows:


1
0
0 1 0 0
0 0
1

1
1
2
0 0 1 0 = 0 2
E ij = (C ij ij ) = 0 1 +
2
2
2
0
0 0
0
1 + 2 0 0 1

0
1
0
0

1
1
1
eij = ( ij bij1 ) = 0 1 0 0

2
2
1+ 2

0
0
1

0
0

0
0
0

1
0 = 0
2 1 + 2

0
0

1 + 2

0
0
2

1+ 2

We can check the results by the relationship E = F T e F :

0
0 0
1 0
2
1

E ij = 0 1 0
1+ 2
2
0 1
0
0

0 1 0
0
0 0

0 0 1 = 0 2

0 0
0
1
2

1+ 2

0
0
2

b) According to the format of the Cartesian components of C , we can verify that the
original space is already the principal space of C , i.e. the principal directions are
N i(1) = [1 0 0] , N i(1) = [0 1 0] , N i(1) = [0 0 1] . By definition, the right stretch tensor is
given by U = C , and its components are:
1

U ij = 0
0

0
1+ 2
0

inverse

U ij1
0
1 + 2
0

= 0

0
1
1+ 2
0

2
1+

By means of the right polar decomposition ( F = R U R = F U 1 ), we obtain:

University of Castilla-La Mancha


Ciudad Real - Spain

Draft

By: Eduardo W. V. Chaves (2014)

SOLVING PROBLEMS BY MEANS OF CONTINUUM MECHANICS

212

R ij = Fik U kj1

0 1
1 0

= 0 1 0
0 1

1
0
=
1+ 2

2
1+

0
1
1+ 2
0

1+ 2

0
0

1
1

Note that by means of the format of the Cartesian components of b indicate that the
principal directions are [1 0 0] , [0 1 0] , [0 0 1] , but this is not the principal
directions of b related to the polar decomposition. Note that there are two equal
eigenvalues related to the directions [0 1 0] , [0 0 1] , then any direction in the plane
x 2 x 3 is a principal direction.
X 2 , x2

Any direction on the plane


x 2 x 3 is a principal
direction of b

The principal direction n (1) is


unique, associated with the
eigenvalue b1 = 1 .
X 1 , x1

X 3 , x3

Figure 2.13: Principal space of b .


Remember that the polar decomposition is unique, i.e. there is one principal base b for the
( a ) . By means of the relation n ( a ) = R N
( a ) we can
polar decomposition associated with N
obtain the principal base of b for the polar decomposition:

(2)
n (i 2 ) = R N

n (i 3)

( 3)
= R N

1+ 2

1
=
0
1+ 2 0

1+ 2

=
0
1+ 2 0

0 0

1
1 =
2
1 0 1 +

0
1

0 0

1
0 =
2
1 1 1 +

0
1

0
1


0

( a ) holds:
In addition, we can check that the relation R = n ( a ) N
a =1

University of Castilla-La Mancha


Ciudad Real - Spain

Draft

By: Eduardo W. V. Chaves (2014)

2 CONTINUUM KINEMATICS

213

R ij = n i(1) N (j1) + n i( 2 ) N (j2 ) + n i(3) N (j3)


1
1
= 0[1 0 0] +
1+ 2
0
1 0 0
1
= 0 0 0 +
1+ 2
0 0 0

0
1
1 [0 1 0] +

1+ 2

0 0
0 1

0
1
0 +
1+ 2
0

0
[0 0 1]

1

0 0 0
1
0 0 =

1 + 2
0 0 1

1+ 2

0
0

c) By means of the stretch definition according to the direction M , i.e. M =

0
1

r
dx
ds
,
r =
dS
dX

and considering that the stretch is not dependent on line integral (homogeneous
deformation), it holds that:

L final = ds = M dS = M dS = M Linitial

The stretch according to X 3 -direction is given by:


X = C 33 = 1 + 2 E 33 = 1 + 2
3

Then:
2

L final = M dX 2 = 1 + 2 ( Linitial ) = 2 1 + 2
0

As we are dealing with a homogeneous deformation, a line in the reference configuration


remains a line in the current configuration, (see Figure 2.14).
X 3 , x3

A
L final
2

x1A 1 0 0 X 1A
A
A
x 2 = 0 1 X 2
x3A 0 1 X 3A


1 0 0 0 0
= 0 1 0 = 2
0 1 2 2

A
Linitial = 2

X 2 , x2

X 1 , x1

Figure 2.14.

According to Figure 2.14 we can check that:


Linitial = 22 + (2 )2 = 4(1 + 2 )
2

Linitial = 2 1 + 2

To obtain the angle in the current configuration formed by two unit vectors, we can use
the equation:
University of Castilla-La Mancha
Ciudad Real - Spain

Draft

By: Eduardo W. V. Chaves (2014)

SOLVING PROBLEMS BY MEANS OF CONTINUUM MECHANICS

214

cos =

E N
cos + 2 M
M N

(2.138)

where is the angle between the unit vectors M and N in the reference configuration,
and is the angle between the to new unit vectors in the current configuration.
r

Considering that the Green-Lagrange strain tensor is independent of X , we adopt two unit
vectors in the plane X 1 X 2 forming an angle = 30 , e.g. N i = [1 0 0] and
M i = [cos 30 sin 30 0] . With these data we have:
0 0
1

M E N = [1 0 0]0 2
2
0 0

0 cos 30
0 sin 30 = 0
2 0

The stretches:
2M

0
0 1
1

= M C M = [1 0 0] 0 1 +
0 0 = 1
0
0
1 + 2 0

2N

M = 1

0
0 cos 30
1

2
= N C N = [cos 30 sin 30 0] 0 1 +
0 sin 30
0
0
1 + 2 0

= cos 2 30 + (1 + 2 ) sin 2 30 = 1 + 2 sin 2 30

Then, N = 1 + 2 sin 2 30 . Then, we obtain:


cos =

E N
cos + 2 M
cos 30
=
M N
1 + 2 sin 2 30

As we are dealing with a homogeneous deformation, we adopt two lines in the reference
configuration and we obtain the angle formed by these lines in the current configuration.
For example, adopting the lines OB = [cos 30 0 0] and OC = [cos 30 sin 30 0]. And
according to the equations of motion, the point O does not move. Then, we obtain the
new position of the points B and C , (see Figure 2.15):
x1B 1 0
0 X 1B 1 0
0 cos 30 cos 30
B
B

x 2 = 0 1 X 2 = 0 1 0 = 0
x B 0 1 X B 0 1 0 0

3
3
x1C 1 0
0 X 1C 1 0
0 cos 30 cos 30
C
C

x 2 = 0 1 X 2 = 0 1 sin 30 = sin 30
x C 0 1 X C 0 1 0 sin 30

3
3

University of Castilla-La Mancha


Ciudad Real - Spain

Draft

By: Eduardo W. V. Chaves (2014)

2 CONTINUUM KINEMATICS

215

X 3 , x3

sin 30

sin 30

X 2 , x2

cos 30

30

B = B

X 1 , x1

Figure 2.15.
Then the angle formed by the new unit vectors O B and O C is:
O B O C = O B O C cos
cos 2 30 = cos 2 30 cos 2 30 + sin 2 30 + 2 sin 2 30 cos

cos =

cos 30
1 + 2 sin 2 30

d)
0 0 0
ij = 0 0 0
0 0 0

Problem 2.56
A rigid body motion is characterized by the following equation:
r
r r
x = c(t ) + Q(t ) X

(2.139)

r
r
Find the velocity and the acceleration fields as a function of , where is the axial vector
& Q T ).
associated with the antisymmetric tensor ( = Q

Solution:
r

The material time derivative of x = c(t ) + Q(t ) X is given by


r D r r& r& & r
v=
x x =c + Q X
Dt

& QT Q
& = Q . The above equation can also be expressed as:
Let us consider that = Q
r
r r
v = c& + Q X
r r
r r
v = c& + ( x c )

University of Castilla-La Mancha


Ciudad Real - Spain

Draft

By: Eduardo W. V. Chaves (2014)

SOLVING PROBLEMS BY MEANS OF CONTINUUM MECHANICS

216

If is an antisymmetric tensor, it holds that a = a , where (angular velocity vector)


is the axial vector associated with the antisymmetric tensor . Then, the associated
velocity can be expressed as:
r r
r r
r r
r r
v = c& + ( x c ) = c& + ( x c )

(2.140)

Note that Q(t ) is only dependent on time, hence the axial vector (angular velocity)
r r
associated with is also time-dependent, i.e. = (t ) .
Then, its acceleration is given by:
r r &r& &r& && r
a = v& = x
=c + Q X

&& = & Q + Q
& , the above equation can also be expressed as:
By referring to Q
r &r&
r
r
r &r&
&) X =c
& X
a=c
+ (& Q + Q
+ & Q X + Q
r
r r
r
r r
r r
= &c& + & Q X + Q X = &c& + & ( x c ) + ( x c )

we can state that:


r &r& r&
r r
r
r
r r
a =c
+ ( x c) + [ ( x c)]

(2.141)

r r
where & shows the angular acceleration.
r

For a rigid body motion where c = 0 , the velocity becomes v = x whose components
are vi = ipq p x q , and the rate-of-deformation tensor D becomes:
D ij =
=

1 vi v j
+
2 x j xi

1
ipq p qj
2

1 ( ipq p x q ) ( jpq p x q ) 1
x
x
=
= ipq p q + jpq p q
+
2
2
x j
xi
x j
xi

1
1
+ jpq p qi = ipj p + jpi p = ipj p ipj p = 0 ij
2
2

So, once again we have proved that D = 0 for a rigid body motion.
Problem 2.57
r

Given a coordinate system x which is fixed in space, and mobile system x * characterized
r
only by rotation, (see Figure 2.16). Show that the rate of change of a vector b can be
represented by:
r
r
r
r
Db
Db
r r Db
T

=
+

b
=
+

b
Dt

Dt

fixed Dt mobile

mobile

(2.142)

r
r
Db
r

where
represents
the
rate
of
change
of
b
with
respect
to
the
fixed
system
x
,

Dt fijo
r
r
Db

represents the rate of change of b with respect to the mobile system which its
Dt

mvil

r
angular velocity is .

University of Castilla-La Mancha


Ciudad Real - Spain

Draft

By: Eduardo W. V. Chaves (2014)

2 CONTINUUM KINEMATICS

r
b

217

x3

x2*

x3*

x1*
x2

x1

Figure 2.16.
Solution:
By means of components transformation law the following relationships are true:
Components

{br

r
= A b

r
r
b = A T b*

where A is the matrix transformation from the system x to the system x * .


r

The rate of change of the vector b = A T b * can be evaluated as follows:


r
r
r&
D r r& D
b b =
A T b * = A& T b * + A T b *
Dt
Dt

(2.143)

Making an analogy with rate of change of the orthogonal tensor, see chapter 2 from the
textbook, we can state that = A& A T A& T = A T T , where T is an antisymmetric
r
tensor and represents the rate of change of rotate of the system x * with respect to the
r
system x . Then, the equation in (2.143) can be written as follows:
r
r&
r
r&
r
r&
D r r&
b b = A& T b * + A T b = A T T b * + A T b * = A T T b * + b *
(2.144)

Dt
r
r r
r
Recall the antisymmetric tensor property T b * = b * , where is the axial vector

r r
associated with the antisymmetric tensor T , i.e. = (t ) is the angular velocity of the
r
mobile system x * . Then, the equation in (2.144) can also be written as:
r&
r
r&
r
r&
r
b = A T T b * + b * = A T * b * + b * (components)
(2.145)

r&
r&
r
Note that the term A b represents the components of b in the system x * , and also note
r& r&
that A b b * , thus:

r * r
r
A b& = b& * + r * b * (components)

(2.146)

which in tensorial notation becomes:


r
r
Db
Db
r r

+ b (tensorial notation)
Dt = Dt

fijo
mvil

University of Castilla-La Mancha


Ciudad Real - Spain

Draft

(2.147)

By: Eduardo W. V. Chaves (2014)

SOLVING PROBLEMS BY MEANS OF CONTINUUM MECHANICS

218

Problem 2.58
r

a) A continuum is rotating as a rigid body with a constant angular velocity = 3 e 3 :


a.1) Obtain the velocity components in the spatial and material descriptions;
a.2) Obtain the acceleration in the spatial (Eulerian) description;
a.3) If 3 = 3rad / s obtain the vector position, velocity and acceleration at time t = 2.5s
of the particle that in the reference configuration was at (1,1,0) .
b) Taking into account Problem 1.128 where we have obtained the body force vector (per
r

GM
x

unit mass) b = r x where g = b is the acceleration of gravity caused by gravitational


field. Now, if we consider the Earth as a sphere that rotates around its axis with angular
r
velocity = 3 e 3 , obtaining the acceleration of gravity ( g ) at sea level in terms of the
latitude .
X 3 , x3
r
= 3 e 3

r r
r = r e r = re r

3
r
r

e 3
e r

r
x

X 2 , x2

X 1 , x1

Figure 2.17.
Solution:

r r

a.1) By means of the previous problem we conclude that v ( x , t ) = x , or in indicial


notation:
vi = ijk j x k = i1k
{1 x k + i 2 k
{2 x k + i 3k 3 x k = i 3k 3 x k
=0

=0

= i 313 x1 + i 32 3 x 2 + i 33 3 x3 = i 31 3 x1 + i 32 3 x 2
{
=0

Then:
v1 = 1323 x2 = 3 x2
; v2 = 2313 x1 = 3 x1
r r
r r r
Note that the field v ( x , t ) is stationary, i.e. v = v ( x ) .

v3 = 0

(2.148)

For a rigid body motion, the equations of motion are governed by:
r
r
x = Q(t ) X

University of Castilla-La Mancha


Ciudad Real - Spain

Draft

By: Eduardo W. V. Chaves (2014)

2 CONTINUUM KINEMATICS

219

where the orthogonal matrix components are given by the transformation matrix from the
r
r
system x to x , thus:
x1 cos (t ) sin (t ) 0 X 1 cos (t ) X 1 sin (t ) X 2


x 2 = sin (t ) cos (t ) 0 X 2 = sin (t ) X 1 + cos (t ) X 2
x 0

0
1 X 3
X3
3

Considering that =

d(t )
and by integrating we obtain:
dt

d(t ) = dt

(t ) = t

Then, we can rewrite the equations of motion as follows:


x1 cos (t ) sin (t ) 0 X 1 X 1 cos(t ) X 2 sin(t )


x 2 = sin (t ) cos (t ) 0 X 2 = X 1 sin(t ) + X 2 cos(t )
x 0

0
1 X 3
X3
3

(2.149)

To obtain the expression of velocity in the material (Lagrangian) description, we replace the
equations of motion (2.149) into the equations (2.148):
r
v1 ( X , t ) = 3 ( X 1 sin(t ) + X 2 cos(t ))
r

v 2 ( X , t ) = 3 ( X 1 cos(t ) X 2 sin(t ))
r

v3 ( X , t ) = 0

(2.150)

a.2) The Eulerian acceleration can be obtained by means of the definition of material time
r r
derivative of v ( x , t ) , i.e.:
r r
r r
r r
r r
r r r
v ( x , t ) v ( x , t ) x ( X , t )
a ( x, t ) =
+
= xr v v ( x , t )
r
t4
x
t
1
42
3
r
0

where the spatial velocity gradient components are given by:

r r
r
v ( x , t )

r = ( xr v )ij
x ij

v1

x1
v
= 2
x
1

v3
x1

v1
x 2
v 2
x 2
v 3
x 2

v1

x 3 0
v 2
= 3
x 3

v 3 0
x 3

3
0
0

0
0 (antisymmetric)
0

With that, we check that we are dealing with a rigid body motion. Then, the Eulerian
acceleration components are given by:
0 3
r
r r r
r
a i ( x , t ) = [ x v v ( x , t )]i = 3
0
0
0
r r
We can express the acceleration a ( x , t ) = 32 x1 e 1

0 3 x 2 32 x1


0 3 x1 = 32 x 2
0 0 0
32 x 2 e 2 in the cylindrical coordinate,

(see Figure 2.17). Note that:


x1 = r cos ,

x1 = r cos ,

e 1 = e r cos e sin ,

e 2 = e r sin + e cos . Then, the

acceleration in the cylindrical coordinate system becomes:

University of Castilla-La Mancha


Ciudad Real - Spain

Draft

By: Eduardo W. V. Chaves (2014)

SOLVING PROBLEMS BY MEANS OF CONTINUUM MECHANICS

220

r
a = 32 x1 e 1 32 x 2 e 2

= 32 ( r cos )(e r cos e sin ) 32 ( r sin )(e r sin + e cos )


r
= 32 r (cos 2 + sin 2 )e r = 32 re r = 32 r

The latter is known as the centripetal acceleration.


a.3) The particle at position (1,1,0) in the reference configuration describes a circular path
of radius r = 2 on the x1 x 2 -plane, (see Figure 2.18).
X 2 , x2

Particle P at t = 2.5s
r r
v P ( x , t = 2 . 5)

1
r
x

r r
v ( X , t = 0)

Particle P

r
X

X 1 , x1

Trajectory of particle P
Figure 2.18.
r

In the reference configuration ( t = 0 ) it fulfills that X = x . For the particle P we have:


r
v1P ( x , t = 0) = 3 x 2 = 3 X 2 = (3)(1) = 3
P r
v 2 ( x , t = 0) = 3 x1 = 3 X 1 = (3)(1) = 3
P
v3 = 0
32 X 1 9
r


a iP ( x , t = 0) = 32 X 2 = 9
0 0

At time t = 2.5s the position, velocity, and acceleration of the particle P are given by:
x1P X 1 cos(t ) X 2 sin(t ) cos(3 2.5) sin(3 2.5) 0.59136
P

x 2 = X 1 sin(t ) + X 2 cos(t ) = sin(3 2.5) + cos(3 2.5) = 1.28464


x P

X3
0
0

r
v1P ( x , t = 2.5) = 3 x 2 = (3)(1.28464) = 3.85391
P r
v 2 ( x , t = 2.5) = 3 x1 = (3)(0.59136) = 1.77409
P
v 3 = 0
32 x1 5.322
r

a iP ( x , t = 2.5) = 32 x 2 = 11.562
0


University of Castilla-La Mancha
Ciudad Real - Spain

Draft

By: Eduardo W. V. Chaves (2014)

2 CONTINUUM KINEMATICS

221

b) For a particle located on the surface of the Earth, due to rotation, this particle will feel as
if being projected outward according to r -direction, (see figure below). Keep in mind that
the real force is the Centripetal due to the centripetal acceleration. For convenience, we
adopt a fictitious force, centrifugal force, which would be the cause of this apparent
v
outward projection. Associated with this force we have the centrifugal acceleration ( a ctfu )
v
which is equal but opposite to the centripetal acceleration ( a ctpe ).
x3 , z

x3 , z
3

x3

r
b

v
a ctfu

x2 , y

x1 , x

Acceleration of gravity for defined latitude is given by:


r
v
g = a ctfu + b

Remember
v
a ctfu

r
+b =

we have

that
v
a ctfu

given

v
+ 2 a ctfu

r
b =g,

two

r
r
b cos + b

vectors

it

holds

that

, (see Problem 1.02). For this particular case,

r
v
v
a ctpe = a ctfu = 32 r = 32 r . Also check that

r = R cos

and

cos = cos( ) = cos . With that, we obtain:


r
v
g = a ctfu + b =

v
a ctpe

r
r
v
2 a ctpe b cos + b

= ( 32 r ) 2 2( 32 r ) g cos + g 2

= ( 32 R cos ) 2 2( 32 R cos ) g cos + g 2

thus
g = g 2 2 g 32 R cos 2 + 34 R 2 cos 2

Note that at the poles ( = 90 ) we have g Pol = g , and in the line of Ecuador it holds that
g Ecu = g 2 2 g 32 R + 34 R 2 = ( g 32 R ) 2 = g 32 R .

University of Castilla-La Mancha


Ciudad Real - Spain

Draft

By: Eduardo W. V. Chaves (2014)

SOLVING PROBLEMS BY MEANS OF CONTINUUM MECHANICS

222

Problem 2.59
Consider a rod subjected to successive displacements as shown in figure below

B0

L0

L0
L(1)

L( 2 )

L(f1)

L L( 2 )

L(1)
L
( 2)

Show that the engineering strain (Cauchy strain) is not additive to successive increments of
strain, i.e. (1) + ( 2) .
Solution:
The Cauchy strain was obtained as:
C =

L L L 0
=
= 1
L0
L0

Then, the total strain experienced by the body, i.e. from the B0 -configuration to the B configuration is:
C =

L( 2 ) L0 L( 2 )
=
1
L0
L0

In the B -configuration the engineering strain is:


C(1) =

L(1) L0 L(1)
=
1
L0
L0

In the B -configuration considering only the displacement increment u ( 2) , we obtain:


C( 2) =

L( 2) L(1) L( 2)
= (1) 1
L(1)
L

thus
L(1)
L( 2)

L( 2 )
C(1) + C( 2 ) =
1 + (1) 1
1 = C
L
L0
L0

An essential requirement for any strain is to that it can be possible to characterize the real
displacement. In this case the final length is:
L0

L(1)

C(1) dx =
1dx = L(1) L0 = L(1)
L0

0
0
(1)
( 2)
L + L = L
L1
L1
(2)
L

(C2 ) dx = (1) 1 dx = L( 2 ) L(1) = L( 2 )


L

0
0

L0

University of Castilla-La Mancha


Ciudad Real - Spain

Draft

By: Eduardo W. V. Chaves (2014)

2 CONTINUUM KINEMATICS
L0

223

L0

C dx

L
=
1 dx = L L0 = L
L

0 0

The Green-Lagrange strain tensor


Note that the Green-Lagrange strain tensor in the B -configuration is given by:
G =

We

could

E=E

(1)

+F

have
(1)T

( 2)

L2 L20
2 L20

L(1)
. Then:
L0

E = E (1) + F (1)
G =

1 2
1
2

obtained the same expression by using the relationship


F (1) , where for the uniaxial case we have E G , E (1) G(1) ,

E ( 2 ) G( 2) , F (1) (1) =

G(1)

(1) G( 2) (1)

1 L(1)
=
2 L0

L( 2) 2 L2
2
2
0

L L0
=
=
2 L20
2 L20

E ( 2) F (1)

2
L(1)

1 +

L0

1 L( 2)

2 L(1)

2
L(1)

L0

2.3 Polar Decomposition of the Deformation Gradient


Problem 2.60
Let us consider the Cartesian components of the deformation gradient:
5 3 3
Fij = 2 6 3
2 2 4

obtain the tensors U (right stretch tensor), V (left stretch tensor), and R (rotation tensor).
Solution:
Before obtaining the tensors U , V , R , we analyze the deformation gradient F .
The motion is possible if the determinant of F is greater than zero, det ( F ) = 60 > 0 . The
eigenvalues and eigenvectors of F are given by:
F11 = 10 associated with eigenvector m i(1) = [0.6396021491; 0.6396021491; 0.4264014327]

F22 = 3 associated with m i( 2 ) = [ 0.5570860145; 0.7427813527; 0.3713906764]


F33 = 2 associated with m i(3) = [ 0.4082482905; 0.4082482905; 0.8164965809]

It is easy to check that the basis formed by these eigenvectors does not form an orthogonal
basis, i.e. m i(1) m i( 2) 0 , m i(1) m i(3) 0 , m i( 2) m i(3) 0 . We can also verify that if D is the
matrix containing the eigenvectors of F :

University of Castilla-La Mancha


Ciudad Real - Spain

Draft

By: Eduardo W. V. Chaves (2014)

SOLVING PROBLEMS BY MEANS OF CONTINUUM MECHANICS

224

m i(1) 0.6396021491;
0.6396021491;
0.4264014327
( 2)
D = m i = 0.5570860145; 0.7427813527; 0.3713906764
m (3) 0.4082482905; 0.4082482905; 0.8164965809

we find that det (D ) = 0.905 1 , and D 1 D T . However, it holds that:


10 0 0
5 2 2

D 0 3 0 D = 3 6 2 = ( F T ) ij
0 0 2
3 3 4
1

and

5 2 2
10 0 0

1
D 3 6 2 D = 0 3 0
3 3 4
0 0 2

The right Cauchy-Green deformation tensor components, C = F T F , are given by:


33 31 29
C ij = Fki Fkj = 31 49 35
29 35 34

Then the eigenvalues and eigenvectors of C are given by:


= 9.274739
C11

eigenvecto
r

(1) = [0.6861511933; 0.7023576528; 0.1894472683]


N
i

= 3.770098
C 22

eigenvecto
r

( 2 ) = [0.5105143234; 0.2793856273; 0.8132215099]


N
i

= 102.955163
C 33

eigenvecto
r

(3) = [ 0.518239; 0.65470405; 0.550264423]


N
i

These eigenvectors constitute an orthogonal basis, so, it holds that AC1 = ACT , and
det (AC ) = 1 (improper orthogonal tensor):
N (i1) 0.6861511933 0.7023576528 0.1894472683

AC = N i( 2) = 0.5105143234 0.2793856273 0.8132215099


N
(3) 0.518239
0.65470405
0.550264423
i

Furthermore, it holds that:

C11

A 0
0
T
C

C 22
0

0
33 31 29
33 31 29
C11

T
0 AC = 31 49 35 = C ij ; AC 31 49 35 AC = 0
29 35 34
29 35 34
0

C 33

C 22
0

0
0

C 33

In the C principal space we obtain the components of the right stretch tensor, U , as:
1
U = Uij = 0
0

0
2
0

0 C11

0 = 0
3 0

C 22
0

0 3.0454455
0
0

0 =
0
1.9416741
0


C 33
0
0
10.1466824

and its inverse:

U 1 = Uij1

1
= 0

University of Castilla-La Mancha


Ciudad Real - Spain

0
1
2
0

0
0
0

3.0454455

0
0
0 =

1.9416741

1
0
0

10
.
1466824
3

Draft

By: Eduardo W. V. Chaves (2014)

2 CONTINUUM KINEMATICS

225

We can evaluate the components of the tensor U in the original space by means of the
transformation law:
4.66496626 2.25196988 2.48328843
A U AC = 2.25196988 6.00314487 2.80907159 = U ij
2.48328843 2.80907159 4.46569091
T
C

and
0.31528844 0.05134777 0.14302659
A U AC = 2.25196988
0.24442627 0.12519889 = U ij1
0.14302659 0.12519889 0.38221833
1

T
C

Then, the rotation tensor of the polar decomposition is given by the equation R = F U 1 ,
which is a proper orthogonal tensor, i.e. det (R ) = 1 .
R ij =

Fik U kj1

0.10094326 0.05592536
0.9933191

= 0.10658955 0.98826538 0.10940847


0.04422505 0.11463858 0.9924224

The left Cauchy-Green deformation tensor components, b = F F T , are given by:


43 37 28
bij = Fik F jk = 37 49 28
28 28 24

Next, the eigenvalues and eigenvectors of b are given by:


= 9.274739
b11

eigenvecto
r

n i(1) = [0.6212637156 0.7465251613 0.238183919]

= 3.770098
b22

eigenvecto
r

n i( 2 ) = [0.4898263742 0.1327190337 0.8616587383]

= 102.95516
b33

eigenvecto
r

n (i 3) = [ 0.611638389 0.6519860747 0.448121233]

Note that, the tensors b and C have the same eigenvalues but different eigenvectors. If
the eigenvectors of b constitute an orthogonal basis then it holds that Ab1 = AbT , and
det (Ab ) = 1 :
n (i1) 0.6212637156 0.7465251613
0.238183919
( 2)
Ab = n i = 0.4898263742 0.1327190337 0.8616587383
n (3) 0.611638389 0.6519860747 0.448121233

and, it also holds that:

b11

A 0
0
T
b

b22
0

0
b11
43 37 28
43 37 28

T
0 Ab = 37 49 28 = bij ; Ab 37 49 28 Ab = 0
0
28 28 24
28 28 24

b33

b22
0

0
0

b33

Since C and b have the same eigenvalues, it follows that Uij = Vij , i.e. they have the same
components in their respectively principal space. Additionally, it holds that Uij1 = Vij 1 .
The components of the tensor V in the original space can be evaluated by:

University of Castilla-La Mancha


Ciudad Real - Spain

Draft

By: Eduardo W. V. Chaves (2014)

SOLVING PROBLEMS BY MEANS OF CONTINUUM MECHANICS

226

5.3720129 2.76007379 2.41222612


A V Ab = A U Ab = 2.76007379 6.04463857 2.20098553 = Vij
2.41222612 2.20098553 3.6519622
T
b

T
b

and
0.28717424 0.07950684 0.14176921
A V Ab = A U Ab = 0.07950684 0.23396031 0.08848799 = Vij1
0.14176921 0.08848799 0.42079849
1

T
b

T
b

The polar decomposition rotation tensor obtained previously has to be the same as the one
obtained by R = V 1 F .
We could also have obtained the tensors U , V , R , by means of their spectral
representation. That is, if we know the principal stretches, i , and the eigenvectors of C
(i ) ), and the eigenvectors of b ( n (i ) ), it is easy to show that:
(N

3
(a) N
( a ) = N (1)N (1) + N ( 2 ) N ( 2 ) + N (3) N
(3)
U ij = a N
j
i
j
i
j
1 i
2
3

ij
a =1

3
Vij = a n ( a ) n ( a ) = 1 n (i1) n (j1) + 2 n i( 2) n (j2) + 3 n i(3) n (j3)
ij
a =1

3 (a) (a )
(1) + n ( 2 )N ( 2 ) + n (3) N (3)
R ij =
n N = n (i1) N
j
i
j
i
j
ij
a =1

3
( a ) = n (1) N (1) + n ( 2 ) N ( 2 ) + n (3) N (3)
Fij = a n ( a ) N
1 i
j
2 i
j
3 i
j

a =1
ij

F=

(a) N
(a ) =
R N

a =1

n ( a ) n ( a ) R

a =1

(a) N
( a ) = n ( a ) n ( a ) R = R U = V R
= R a N
a

a =1
a =1

As we can verify, the representations of the tensors R and F are not the spectral
representations in the strict sense of the word, i.e., i are not eigenvalues of F , and
(i ) are eigenvectors of F .
neither n (i ) nor N

Problem 2.61
The deformation gradient at one point of the body is given by:
F = 0.2e 1 e 1 0.1e 1 e 2 + 0.3e 2 e 1 + 0.4e 2 e 2 + 0.1e 3 e 3

where e i

(i = 1,2,3) represents the Cartesian basis.

a) Obtain the deformation tensors b and C ;


b) Obtain the eigenvalues and eigenvectors of b and C ;
c) Write the spectral representation of F in function of the eigenvalues of C ( C a ) and
3

( a ) holds, where are the principal stretches, n are the


check if F = a n ( a ) N
a
a =1

are the eigenvectors of C ;


eigenvectors of b , and N

University of Castilla-La Mancha


Ciudad Real - Spain

Draft

By: Eduardo W. V. Chaves (2014)

2 CONTINUUM KINEMATICS

227

d) Obtain the spectral representation and components of: ( R ) spin tensor of the polar
decomposition; the stretch tensors U and V ;
Solution
The deformation gradient components can be represented as follows:
F = Fij e i e j = 0.2e 1 e 1 0.1e 1 e 2 + 0.3e 2 e 1 + 0.4e 2 e 2 + 0.1e 3 e 3

0 .2 0 .1 0
0
Fij = 0.3 0.4
0
0
0.1

a) The left Cauchy-Green deformation tensor ( b = F F T ) components:


T

bij = Fik F jk

0
0.2 0.1 0 0.2 0.1 0
0.05 0.02

= 0 .3 0 .4
0 0 .3 0 .4
0 = 0.02 0.25 0
0
0
0
0.1 0
0
0.1
0
0.01

(2.151)

The right Cauchy-Green deformation tensor ( C = F T F ) components are given by:


0.2 0.1 0
0
C ij = Fki Fkj = 0.3 0.4
0
0
0.1

0
0.2 0.1 0 0.13 0.1
0.3 0.4

0 = 0.1 0.17
0

0
0
0.1 0
0
0.01
b) The eigenvalues and eigenvectors of b and C are obtained as follows;
=C N
(a)
C N
(a)

(2.152)

C C1 = 0

where the index (a ) does not indicate summation. Note that we already know one
eigenvalue of C , i.e. C (3) = 0.01 , (see C -components in (2.152)). Then, the characteristic
determinant becomes:
0.13 C

0 .1

0 .1

0.17 C

=0

(0.13 C )(0.17 C ) 0.01 = 0

The solution of the quadratic equation is:


C (1) = 0.25198

C ( 2 ) = 0.04802

Then:
Cc (1) = 0.25198

C (3) = 0.01

N i(3)

0.633399
= 0.77334

0
0
= 0
1

N (i1)

C ( 2 ) = 0.04802

N i( 2 )

0.77334
= 0.63399

Similarly for the eigenvectors of the tensor b :


b n = b( a ) n ( a )

where the index (a ) does not indicate summation. Then

University of Castilla-La Mancha


Ciudad Real - Spain

Draft

By: Eduardo W. V. Chaves (2014)

SOLVING PROBLEMS BY MEANS OF CONTINUUM MECHANICS

228

b(1) = 0.25198

b(3) = 0.01

0.098538
= 0.995133

0
0
= 0
1

n i(1)

n i(3)

b( 2 ) = 0.04802

n (i 2 )

0.995133
= 0.098538

As expected, the tensors C and b have the same eigenvalues but different eigenvectors.
0
0
0.252

0.048 0
C ij = 0
0
0
0.01

0
0
0.252

0.048 0
bij = 0
0
0
0.01

In addition, the spectral representations of the tensors C and b are given respectively by:
C=

(a ) N
(a)
2a N

a =1

b=

n
2
a

(a)

n ( a )

a =1

where a > 0 are the principal stretches. Considering that 2a = C a are the eigenvalues of
C and of b , the principal stretches are:
(1) = 0.25198 0.501976
F=

c) To check if

( 2) = 0.04802 0.219134

n
a

(a)

(a )
N

(3) = 0.01 = 0.1

holds we calculate the components of

a =1

3
( a ) with the results obtained previously, i.e.:
a n ( a ) N

ij
a =1

3
( a ) = n (1) N (1) + n ( 2 ) N ( 2 ) + n (3) N (3)
a n ( a ) N
j
j
j
1 i
2 i
3 i

a =1
ij

0.06247 0.0762 0
0.76958 0.6309 0

= 0.50197 0.0762 0.06247 0 + 0.219134 0.6309 0.76958 0 +


0

0
0
0
0
0
0 0 0
+ 0.10 0 0
0 0 1
0.2 0.1 0
0 = Fij
= 0.3 0.4
0
0
0.1
3

(a ) .
Checking that F = a n ( a ) N
a =1

d)
R=

(a)
n ( a ) N

components

a =1

University of Castilla-La Mancha


Ciudad Real - Spain

0.832 0.554 0
(R )ij = 0.554 0.832 0
0
0
1

Draft

By: Eduardo W. V. Chaves (2014)

2 CONTINUUM KINEMATICS

229

which can be verified with:


0.76958 0.6309 0 0.06247 0.0762
R ij = 0.0762 0.06247 0 + 0.6309 0.76958

0
0
0 0
0
0.333
3
(
a
)
(
a
)
N

aN
U=
components (U)ij 0.139
a =1
0

V=

n
a

(a )

(a)

components

a =1

0 0 0 0 0.832 0.5547 0
0.832
0
0 + 0 0 0 = 0.5547
0 0 0 1 0
0
1
0.139 0
0.388 0
0
0.1

0.222 0.028 0
(V )ij 0.028 0.5 0
0
0
0.1

Problem 2.62
For a given motion (shear deformation):
x1 = X 1 + kX 2

x2 = X 2

x3 = X 3

where k is a constant. Obtain the tensors: F (deformation gradient), C (the right CauchyGreen deformation tensor), b (the left Cauchy-Green deformation tensor), E (the GreenLagrange strain tensor), U (the right stretch tensor), V (the left stretch tensor) and R (the
spin tensor of the polar decomposition).
Solution:
The deformation gradient components:
x1

X 1
xi x 2
Fij =
=
X j X 1

x3
X 1

x1
X 2
x 2
X 2
x3
X 2

x1

X 3 1 k 0
x 2
= 0 1 0
X 3

x3 0 0 1
X 3

The right Cauchy-Green deformation tensor ( C = F T F ), whose components are:


k
1 0 0 1 k 0 1

C ij = Fki Fkj = k 1 0 0 1 0 = k 1 + k 2
0 0 1 0 0 1 0
0

0
0
1

The left Cauchy-Green deformation tensor ( b = F F T ), whose components are:


2
1 k 0 1 0 0 1 + k

bij = Fik F jk = 0 1 0 k 1 0 = k
0 0 1 0 0 1 0

1 0
0 1

The Green-Lagrange strain tensor, E =

1
(C 1) , whose components are:
2

University of Castilla-La Mancha

Draft

Ciudad Real - Spain

By: Eduardo W. V. Chaves (2014)

SOLVING PROBLEMS BY MEANS OF CONTINUUM MECHANICS

230

1
k
1
E ij = k 1 + k 2
2
0
0

0 1 0 0
0 k
1

0 0 1 0 = k k 2
2
0 0
1 0 0 1

0
0
0

Note that there is only deformation on the x1 x 2 -plane.


Considering the polar decomposition F = R U = V R , we obtain:
C = (V R)T (V R) = R T V T V R = R T V V R = R T V 2 R = R T b R

For simplicity, we will work on the x1 x 2 -plane, with that we represent the rotation
tensor components as follows:
cos sin c s
R ij =
=

sin cos s c

(i, j = 1,2)

where cos 2 + sin 2 = c 2 + s 2 = 1 holds. The relationship C = R T b R becomes:


k c s 1 + k 2 k c s
1

k 1 + k 2 = s c

1 s c


k
(c 2 + c 2 k 2 + 2 sck + s 2 )
( sck 2 s 2 k + c 2 k )
=

2
2
2
(c 2 + s 2 k 2 2 sck + s 2 )
( sck s k + c k )
(c 2 + c 2 k 2 + 2 sck + s 2 ) = 1 (c 2 k 2 + 2 sck + 1) = 1

From the relationship

we obtain

k
c . Then, starting from the relation ( sck 2 s 2 k + c 2 k ) = k and by considering that
2
k
s=
c , we obtain:
2
s=

c=

1
2

k
+1
4

k +4

s=

k
2
2

k
+1
4

k
k2 + 4

thus:
2

2
k +4
k
R ij =
2
k +4
0

k
k2 + 4
2
k2 + 4
0

From the polar decomposition F = R U = V R , we obtain U = R T F and V = F R T ,


whose components are:
2

2
k +4
k
U ij = R ki Fkj =
2
k +4
0

University of Castilla-La Mancha


Ciudad Real - Spain

k
2

k +4
2
k2 + 4
0

0
2
1 k 0 k + 4

k
0 0 1 0 =
2
0 0 1 k + 4
1
0

Draft

k
2

k +4
2+ k2
k2 + 4
0

By: Eduardo W. V. Chaves (2014)

2 CONTINUUM KINEMATICS

Vij = Fik R jk

2
2+k
0
2
k +4
k

0 =
2
k +4
1
0

2
1 k 0 k + 4
k
= 0 1 0
2
0 0 1 k + 4
0

231

k2 + 4
2
k2 + 4
0

k
k2 + 4
2
k2 + 4
0

Problem 2.63
A deformable parallelepiped of dimensions 2 2 1 is in the reference configuration as
indicates in Figure 2.19. This body is subjected to motion:
r r
x ( X , t ) = exp X 2t e 1 + tX 12 e 2 + X 3 e 3

(2.153)

where ( X 1 , X 2 , X 3 ) are the material coordinates, and t represents time.


r

a) Obtain the deformation gradient F , for all X and time t .


b) Obtain the right Cauchy-Green deformation tensor C , and the principal stretches.
c) Obtain the right stretch tensor U and the rotation tensor R . Check that the latter is a
proper orthogonal tensor.
d) Find the volume of the deformed parallelepiped at time t = 1s .
X2

X3

X1

Figure 2.19.
Solution:
a) According to the equation (2.153), the vector position components are x1 = exp X 2t ,
x 2 = tX 12 , x3 = X 3 , then the deformation gradient ( F ) components are given by:
0
xi
Fij =
= 2tX 1
X j
0

t exp X 2t
0
0

0
1

b) The right Cauchy-Green deformation tensor are defined by C = F T F , whose


components are C ij = Fki Fkj :
0

C ij = t exp X 2t

2tX 1

University of Castilla-La Mancha


Ciudad Real - Spain

0
0

0 0

0 2tX 1
1 0

t exp X 2t

Draft

0
0

0 4t 2 X 12

0 = 0
1 0

0
2

t exp
0

2 X 2t

0
1

By: Eduardo W. V. Chaves (2014)

SOLVING PROBLEMS BY MEANS OF CONTINUUM MECHANICS

232

Note that this space is the principal space (principal directions) of C . Considering that i
are the principal stretches, the following relationship is fulfilled:
C = U2 =

(a ) N
(a)
2a N

U=

a =1

(a) N
(a)
N

a =1

As we are working in the principal space of C , we can obtain the principal stretches as
follows:
1 = + 4t 2 X 12

2 = + t 2 exp 2 X 2t

3 = + 1

(a ) N
( a ) is a positive definite tensor by
which are positive numbers, since U = a N
a =1

definition, thus:
1 = 2tX 1

2 = t exp X 2t

3 = 1

c)
2tX 1
U ij = 0
0

0
t exp
0

X 2t

0
0
1

U ij1

1
2tX
1

= 0

0
1
t exp X 2t
0

According to the polar decomposition F = R U R = F U 1 , we can obtain the rotation


tensor ( R ) components as follows:
0

R ij = 2tX 1
0

t exp

X 2t

0
0

0 2tX 1

0 0

1
0

0
1
t exp X 2t
0

0
0 1 0
0 = 1 0 0

0 0 1
1

Note that the orthogonality condition R R 1 = R R T = 1 holds:


R ik R jk

0 1 0 0 1 0 1 0 0
= 1 0 0 1 0 0 = 0 1 0
0 0 1 0 0 1 0 0 1

and the proper condition det (R ) = 1 .


d) To calculate the final volume we use the relationship dV = JdV0 , where J = F is the
Jacobian determinant and is given by:
0
J = 2tX 1
0

t exp X 2t
0
0

0
0 = 2t 2 X 1exp X 2t
1

At time t = 1s we have J = 2 X 1exp X 2 . Then, the volume at time t = 1s is given by:

University of Castilla-La Mancha


Ciudad Real - Spain

Draft

By: Eduardo W. V. Chaves (2014)

2 CONTINUUM KINEMATICS

dV =

JdV0 =

(2 X exp )dX
X2

3 dX 2 dX 1

233

= 4 exp 2 1 25.556

X1 =0 X 2 =0 X 3 =0

V0

NOTE: We can not use the equation V = JV0 because we are not dealing with
homogeneous deformation case.
Problem 2.64
A body is subjected to motion:
x1 = X 1

x 2 = X 2 + kX 3

x3 = X 3 + kX 2

where k is a constant.
a) Obtain the deformation gradient ( F ); the right Cauchy-Green deformation tensor ( C );
the Green-Lagrange strain tensor ( E ).
b) Calculate the displacement field, the magnitude (dx) 2 of sides OA and OB , and
diagonal OC after deformation of figure below.
X3

dX 2

C
dX 3

X2

X1

c) Consider now a square as figure below


x3

C
23

B
B

x2

c.1) Obtain the stretches according to directions OC and BA ; c.2) Obtain the angle 23 in
the current configuration in function of k .
c.3) Apply the polar decomposition of the tensor F in order to obtain U and R .
Solution:
a) C = F T F . The deformation gradient components are:
University of Castilla-La Mancha
Ciudad Real - Spain

Draft

By: Eduardo W. V. Chaves (2014)

SOLVING PROBLEMS BY MEANS OF CONTINUUM MECHANICS

234

1 0 0
xi
Fij =
= 0 1 k
X j
0 k 1
0
0
1 0 0 1 0 0 1

2
C ij = Fki Fkj = 0 1 k 0 1 k = 0 1 + k
2k
0 k 1 0 k 1 0
2k
1 + k 2

The Green-Lagrange strain tensor, E =

1
(C 1) , whose components are:
2

1
0
0 1 0 0
0 0
1
1

2
E ij = 0 1 + k
2 k 0 1 0 = 0 k 2
2
2
0 2k
2k
1 + k 2 0 0 1
0
r r r
b.1) The displacement field, u = x X , whose components are:

0
2k
k 2

u1 = x1 X 1 = 0 ; u 2 = x 2 X 2 = kX 3
; u 3 = x 3 X 3 = kX 2
r
b.2) Calculation of (dx ) 2 = dx 2
r
r
r
r
r
r
(dxr )2 = dxr dxr = F dX F dX = dX F T F dX = dX C dX

Explicitly:

(dx )

= [dX 1

0
0 dX 1
1

2
dX 3 ] 0 1 + k
2k dX 2
0
2k
1 + k 2 dX 3

dX 2

= (dX 1 ) 2 + (dX 2 ) 2 (1 + k 2 ) + (dX 3 ) 2 (1 + k 2 ) + 4k (dX 2 )(dX 3 )

Then, for the diagonal OC we have [0 dX 2 dX 3 ] , with that we obtain:

(dx )2 = (dX 2 ) 2 (1 + k 2 ) + (dX 3 ) 2 (1 + k 2 ) + 4k (dX 2 )(dX 3 )


For the side OA we have [0 dX 2 0] , with that we obtain:

(dx )2 = (dX 2 ) 2 (1 + k 2 )
For the side OB we have [0 0 dX 3 ] , with that we obtain:

(dx )2 = (dX 3 ) 2 (1 + k 2 )
-direction (reference configuration) is given by the
c) The stretch according to the N
C N
.
equation ( N )2 = N

c.1) The stretch according to the OC -direction: N i = 0

( )

OC

= 0

University of Castilla-La Mancha


Ciudad Real - Spain

1
2

0
1
1
2
0 1+ k
2
0
2k

Draft

1
2

0
2k

1 + k 2

1
, is:
2

1
= (1 + k ) 2
2
1

2
By: Eduardo W. V. Chaves (2014)

2 CONTINUUM KINEMATICS

The stretch according to the BA -direction: N i = 0

( )

BA

= 0

0
1
1
2
0 1 + k
2
0
2k

1
2

1
2

235

1
, with that we obtain:
2

0
0

1
2k
= (1 k ) 2

2
1 + k 2 1

c.2) The variation of the angle. We can directly use the equation:
cos =

C N

C N

M
M
=
M N
C M
N
C N

= [0 0 1] , and according to the


where the unit vector according to the OB -direction is M
i
OA -direction is N i = [0 1 0] . With that we obtain:

( )

0
0 0
1

2
= [0 0 1] 0 1 + k
2k 0 = 1 + k 2
0
2k
1 + k 2 1

( )

0
0 0
1

2
= [0 1 0] 0 1 + k
2k 1 = 1 + k 2
0
2k
1 + k 2 0

OB

OA

0
0 0
1

2k 1 = 2k
Mi C ij N j = [0 0 1] 0 1 + k
0
2k
1 + k 2 0

Then:
cos 23 =

C N

2k
M
=
M N
1+ k2

c.3) The polar decomposition of F = R U = V R , where:


C = U2 =

(a ) N
(a )
aN

U= C =

a =1

(a) N
(a)
aN

a =1

Calculation of the eigenvalues of C . Note that according to the format of C -components,


there is only deformation according to x 2 x 3 -plane. In addition, we know one eigenvalue
1 = 1 associated with the direction N i(1) = [1 0 0] . By means of the characteristic
determinant we obtain:
(1 + k 2 )

2k

2k

(1 + k )

) (

2 2 1 + k 2 + 1 k 2

=0

) (

2 2 1 + k 2 + 1 2k 2 + k 4 = 0

=0

The roots are: 2 = 1 + k 2 + 2k = (1 + k ) 2

3 = 1 + k 2 2k = (1 k ) 2

Then, in the principal space of C we have:


University of Castilla-La Mancha
Ciudad Real - Spain

Draft

By: Eduardo W. V. Chaves (2014)

SOLVING PROBLEMS BY MEANS OF CONTINUUM MECHANICS

236

0
1

C ij = 0 (1 + k ) 2
0
0

(1 k ) 2
0
0

1
( 3)
, 3 N i = 0
2

The principal directions are 2 Ni( 2) = 0

1
. Then,
2

1
2

the transformation matrix between the original space and the principal space is:

1
a ij = A = 0

0
1
2
1
2

0
1
2
1

That is, it must meet:


C = A C AT
0
1
0 (1 + k ) 2

0
0

0 1
0 = 0

(1 k ) 2
0

0 1
0
1
0 1+ k2

2k
1 0

0
1
2
1
2

0 1
2k 0

1 + k 2
0

0
1
2
1

0
1
2
1
2

Then, in the principal space of C , we have:


0
1

C ij = 0 (1 + k ) 2
0
0

(1 k ) 2
0
0

+ 1
0

+ (1 + k ) 2
U ij = 0
0
0

2
+ (1 k )
0
0

0
0
1

0
U ij = 0 (1 + k )
0
0
(1 k )

The inverse tensor in the principal space can be obtained as follows:

Uij1

1
= 0

0
1
(1 + k )
0

0
0

(1 k )

The components of U in the original space are given by:

University of Castilla-La Mancha


Ciudad Real - Spain

Draft

By: Eduardo W. V. Chaves (2014)

2 CONTINUUM KINEMATICS

U 1 = A T U 1 A

1
U ij = 0

0
1
2
1

0
1
2
1
2

1
0

0 1
0 0

1
0
(1 k )

0
1
(1 + k )
0

237



0 1
1
= 0
2
1
0
2

0
1
2
1
2

0
1
(1 k 2 )
k
(1 k 2 )

0
k
(1 k 2 )

1
(1 k 2 )

From the polar decomposition we have F = R U R = F U 1 , thus

1 0 0 1

R ij = 0 1 k 0
0 k 1
0

0
1
(1 k 2 )
k
(1 k 2 )

0 1 0 0
k
= 0 1 0
(1 k 2 )

1 0 0 1
(1 k 2 )

Problem 2.65
Given the following motion:
x1 = 1 X 1

x2 = 3 X 3

x3 = 2 X 2

a) Obtain the final volume to a unit cube;


b) Obtain the deformed area to a unit square defined in the X 1 X 2 -plane, and draw the
deformed area;
c) Apply the polar decomposition and obtain the tensors U , V and R
Solution:
a)
x1 1

x2 = 0
x 0
3

0
0
2

0 X 1

3 X 2
0 X 3

1
Fij = 0
0

0
3 (homogenous deformation)
0

0
0
2

The determinant of F is given by F J = 1 2 3 , and the deformed volume:


integrating
dV = F dV0

V final = F Vinitial = 1 2 3

b) Applying the Nansons formula and by considering the particular case (homogeneous
deformation):
r integrating r
r
r
da = JF T dA

a final = JF T Ainitial

where
e 1
r
Ainitial = 1

e 2
0

e 3
0 = e 3 ; Fij1 =
0

University of Castilla-La Mancha


Ciudad Real - Spain

2 3
1
0
1 2 3
0

Draft

0
0
1 2

0 1
1 3 = 0

0
0

0
0
1
3

1
2

By: Eduardo W. V. Chaves (2014)

SOLVING PROBLEMS BY MEANS OF CONTINUUM MECHANICS

238

With that the deformed area vector is:


1

a1
1

a 2 = 1 2 3 0
a

3
0

0
0
1
2

0
0 0
1

0 = 1 2

3
1 0
0

and its magnitude is:


r
a final = ( 1 2 ) 2 = 1 2

X 3 , x3

B (0,0, 2 )

r
a final = 1 2

C (1 ,0, 2 )
O(0,0,0)

B (0,1,0)

r
Ainitial = 1

C (1,1,0)

A(1,0,0)

X 2 , x2

A( 1 ,0,0)
X 1 , x1

where the points A(1,0,0) , B(0,1,0) and C (1,1,0) move according to the equations of
motion:
x1A 1
A
x2 = 0
x A 0
3
x1C 1
C
x2 = 0
xC 0
3

c)

0
0
2

According

0
0
2

0 1 1

3 0 = 0
0 0 0

x1B 1
B
x2 = 0
x B 0
3

0
0
2

0 0 0

3 1 = 0
0 0 2

0 1 1

3 1 = 0
0 0 2

to

the

polar

decomposition

definition

F = R U = V R

where

U = C = F T F and V = b = F F T we obtain:

University of Castilla-La Mancha


Ciudad Real - Spain

Draft

By: Eduardo W. V. Chaves (2014)

2 CONTINUUM KINEMATICS

1
C ij = 0
0
1
bij = 0
0

0
0
3
0
0
2

0 1
2 0
0 0

0 21

3 = 0
0 0

0
0
2

0 1
3 0
0 0

0
0
3

0 21

2 = 0
0 0

0
22
0
0
23
0

0
23
0

0
22

239

1
U ij = 0
0

1
Vij = 0
0

0
2
0
0
3
0

0
0
3
0
0
2

Note that the original space coincides with the principal space of C . Note also that C and
b have the same eigenvalues but different principal directions. To obtain the spin tensor of
the polar decomposition we apply R = F U 1 = V 1 F , thus:
1
R ij = 0
0

0
0
2

0 1
3 0
0
0

0
1
2
0

0
1 0 0
0 = 0 0 1


1 0 1 0
3

Problem 2.66
Consider the equations of motion:
x1 = 3 X 1

x2 = 2 X 2

x3 = 3 X 3 X 2

Obtain the material ellipsoid associated with the material sphere defined in the reference
configuration by X 12 + X 22 + X 32 = 1 (see Figure 2.20). Check that the ellipsoid in the
principal space of the left stretch tensor V has the shape:
x1 2
21

x 2 2
22

x3 2
23

=1

where 1 , 2 , 3 are the principal stretches.


X 2 , x2

X 3 , x3

Material surface
(always constituted by the
same particles)

X 1 , x1

Figure 2.20: Material sphere.

University of Castilla-La Mancha


Ciudad Real - Spain

Draft

By: Eduardo W. V. Chaves (2014)

SOLVING PROBLEMS BY MEANS OF CONTINUUM MECHANICS

240

Solution:
The law of motion and its inverse are given by:
x1

x2 =
x
3

0
0

0 X 1
X1
inverse
0 X 2

X 2 =
X

3 X 3
3

0
2
1

3
3

0
x1

0 x2

3 x3

0
1
2
3
6

0
0

The equations of motion in the spatial description are given by:


X1 =

3
x1
3

X2 =

x2
2

X3 =

3
3
x2 +
x3
6
3

Replacing the above into the equation of the sphere we obtain:


2

X 12

X 22

X 32

=1

3 x2 2 3
3

x1 + +
x2 +
x3 = 1
3 2 6
3

By simplifying the above equation we obtain:


x12 + x 22 + x32 + x 2 x3 = 3

which is the equation of an ellipsoid. We now represent the ellipsoid equation in the
principal space of the left stretch tensor V . Recall that the tensor V and b are coaxial, i.e.
the have the same principal directions), and is also true that:
V = b = F FT

The components of b are


3

bij = 0
0

0 3 0

0 0
2
3 0 1

0
2
1

0
0
0
3

0 = 0
5
3
0 3
3
3

Note that we know already one eigenvalue b1 = 3 associated with the eigenvector
n i(1) = [1 0 0] . Then, the other principal directions are in the plane x 2 x 3 , with that we

obtain

eigenvector
b2 = 6

n i( 2) = 0

2
2

eigenvector
b3 = 2

n i(3) = 0

2
2

2
2

thus:

1
3 0 0

Transformation matrix
a ij = 0
bij = 0 6 0

0 0 2

University of Castilla-La Mancha


Ciudad Real - Spain

Draft

2
2
2
2

0
2
2
2

By: Eduardo W. V. Chaves (2014)

2 CONTINUUM KINEMATICS

1 = 3
0

Vij = 0
2 = 6
0
0

241

0
3 = 2

Then, applying the transformation law from x1 , x 2 , x3 -system to the x1 , x 2 , x3 -system we


obtain:

x1 1

x2 = 0
x
3
0

0
2
2
2
2

0
2
2
2

x1

x 2
x
3

x1 = x1

2
2

x 2 +
x3
x2 =
2
2

2
2
x 2 +
x3
x3 =
2
2

with that, the equation of the ellipsoid in the principal space of V is represented by:
x12 + x 22 + x32 + x 2 x3 = 3

(x1 )

2
2
2
2
2 2
2
+
x 2 +
x3 +
x 2 +
x3 +
x 2 +
x3
x 2 +
x 3 = 3
2

2
2

2
2
2
2

Simplifying the above equation we obtain:


x1 2 x 2 2 x3 2
x 2
x2
x 2
x2 x2 x2
+
+
= 1 2 + 2 2 + 3 2 = 12 + 22 + 32 = 1
3
6
2
1
2
3
( 3)
( 6)
( 2)
X 3 , x3

X 2 , x2

x3

3 = 2
1 = 3

2 = 6

x 2

x1

X 1 , x1

Figure 2.21: The material ellipsoid (deformed configuration).

University of Castilla-La Mancha


Ciudad Real - Spain

Draft

By: Eduardo W. V. Chaves (2014)

SOLVING PROBLEMS BY MEANS OF CONTINUUM MECHANICS

242

x2
x1

x3

x 2

X2

x2
x1

X1

x3

X3

x1

x3

Figure 2.22: The left polar decomposition.

2.4 Infinitesimal Deformation Regime


Problem 2.67
Given the equations of motion
x1 = X 1 + 4 X 1 X 2t

x2 = X 2 + X 22t

x3 = X 3 + X 32t

(2.154)

a) Obtain the velocity field;


b) Obtain the infinitesimal strain field;
c) At time t = 1 s , obtain the infinitesimal strain tensor.
Solution:
a) Velocity:
V1 = 4 X 1 X 2
r
r r

dx
V 2 = X 22
V ( X , t) =
dt

2
V3 = X 3

(2.155)

r
A1 = 0
r r
dV

A2 = 0
A( X , t ) =
dt
A = 0
3

(2.156)

b) Acceleration:

c) Displacement field:
University of Castilla-La Mancha
Ciudad Real - Spain

Draft

By: Eduardo W. V. Chaves (2014)

2 CONTINUUM KINEMATICS

243

u1 = x1 X 1 = X 1 + 4 X 1 X 2 X 1 = 4 X 1 X 2

2
2
u 2 = x2 X 2 = X 2 + X 2 X 2 = X 2

2
2
u3 = x3 X 3 = X 3 + X 3 X 3 = X 3

(2.157)

Then, the infinitesimal strain tensor components are given by:


ij =
u1

X 1
u i u 2
=
x j X 1

u 3
X 1

1 u i u j
+
2 x j xi

(2.158)

u1

X 3 4 X
2
u 2
= 0
X 3

u 3 0
X 3

u1
X 2
u 2
X 2
u 3
X 2

4X1
2X 2
0

0
0
2 X 3

(2.159)

thus:
4 X 2
ij = 2 X 1
0

0
0
2 X 3

(2.160)

0
2
X
23
l
X2X3

l 2

(2.161)

2 X1
2X 2
0

Problem 2.68
Consider the infinitesimal strain tensor:

0
0

X
X
ij = 0 2 2 3

X2
0 3

l2

and the infinitesimal spin tensor:

0
0

ij = 0
0

2
2
0 2 X 2 X 3
2l

2l 2

X 22 X 32

(2.162)

Obtain the displacement field components.


Solution:
The displacement gradient is related to the infinitesimal strain tensor and the infinitesimal
spin tensor as follows:
u i , j = ij + ij

ij =

1
ui, j + ui, j
2

; ij =

1
ui, j ui, j
2

(2.163)

thus:
University of Castilla-La Mancha
Ciudad Real - Spain

Draft

By: Eduardo W. V. Chaves (2014)

SOLVING PROBLEMS BY MEANS OF CONTINUUM MECHANICS

244

2l

2
3X 3
2 X 2 X 3

u i , j = 2 0

2X 2 X 3

X 22

X 32

X 22

(2.164)

u1
=0
u1 = 0
x1
u 2

= 2 (2 X 2 X 3 )
x 2 2l

u 2 =

2l (2 X
2

2X3

)x 2 u 2 =

2l 2

u 3

= 2 (2 X 2 X 3 )
x3
2l

u 3 =

2l 2

(2.165)

(2 X 2 X 3 )x3 u 3 =

X 22 X 3

+ C1 ( X 3 )

X 32 X 2

2l 2

(2.166)

+ C2 ( X 2 )

(2.167)

To determine the constant C1 ( X 3 ) from the result (2.166) we take the derivative of u 3
with respect to X 3 :
u 2

= 2
X 3 2l

2 C1 ( X 3 )
C1 ( X 3 )

2
2
= 3 X 32
X 2 +
= 2 X 2 3X 3
X 3 2l
X 3

C1 ( X 3 ) =

(2.168)

X 33

Similarly we find the constant C2 ( X 2 ) :


u 3

= 2
X 2
2l

2 C 2 ( X 2 )
C 2 ( X 2 )

2
2
= X 22
X 3 +
= 2 X2 + X3
X

2
l
2
2

X3
C2 ( X 2 ) = 2
3

(2.169)

Then, the displacement field is given by:


u1 = 0 ; u 2 =

2l 2

[X

2
2 X3

X 33

; u3 =

X 23
2
X
X
+

3
2
3
2l 2

(2.170)

Problem 2.69
Show that, in the infinitesimal strain regime, the rate of change of the infinitesimal strain
tensor ( & ) is equal to the rate-of-deformation tensor ( D ).
Solution:
Consider the relationship between the rate of change of he Green-Lagrange deformation
tensor ( E& ) and the rate-of-deformation tensor ( D ):
E& = F T D F

(2.171)

In the case of small deformation F 1 holds, in addition it fulfills that E& e& & then:
E& = & = D

University of Castilla-La Mancha


Ciudad Real - Spain

Draft

(2.172)

By: Eduardo W. V. Chaves (2014)

2 CONTINUUM KINEMATICS

245

Problem 2.70
Given the equations of motion
x1 = X 1

x2 = X 2 + X 1 exp 2t 1

x3 = X 3 + X 1 exp 3t 1

(2.173)

Obtain the rate-of-deformation ( D ) and compare with the rate of change of the
infinitesimal strain tensor ( & ).
Solution:
By definition, the rate-of-deformation tensor ( D ) is the symmetric part of the spatial
velocity gradient, i.e.:
1
(l + l T )
r
r
r
and ( x , t ) = sym u (u) sym
2
r
l = xv

D=

(2.174)

By definition, the infinitesimal strain tensor is equal to the symmetric part of the
displacement gradient:
r
D
(2.175)
= (u) sym &
Dt
r r r
The displacement field is given by u = x X . Considering the equations of motion, the

displacement field components become:

u1 = x1 X 1 = X 1 X 1 = 0

2t
2t
u 2 = x 2 X 2 = X 2 + X 1 exp 1 X 2 = X 1 exp 1

3t
3t
u 3 = x 3 X 3 = X 3 + X 1 exp 1 X 3 = X 1 exp 1
r
r Du
The velocity field is given by v =
. Then, the velocity field components, in material
Dt

(
(

)
)

(
(

)
)

coordinates, are:
V1 = 0

V2 = X 1 ( 2exp 2t )

V3 = X 1 (3exp 3t )

(2.176)

Given the inverse equations of motion:


x1 = X 1

2t
x 2 = X 2 + X 1 (exp 1)

3t
x3 = X 3 + X 1 (exp 1)

X 1 = x1

2t
X 2 = x 2 x1 (exp 1)

3t
X 3 = x3 x1 (exp 1)

inverse

(2.177)

we can obtain the velocity field in spatial coordinates:


v1 = 0

v2 = 2 x1exp 2t

v3 = 3 x1exp 3t

(2.178)

The spatial velocity gradient ( l ) components are given by:


0

vi
r
= 2exp 2t
( l ) ij = ( x v ) ij =
x j
3t
3exp

0 0
0 0
0 0

(2.179)

and

University of Castilla-La Mancha


Ciudad Real - Spain

Draft

By: Eduardo W. V. Chaves (2014)

SOLVING PROBLEMS BY MEANS OF CONTINUUM MECHANICS

246

0
1
1
(D) ij = ( l ij + l ji ) =
2exp 2t
2
2
3exp 3t

= exp 2t
3 exp 3t
2

exp 2t

0
0

We also obtain the spin tensor W = l

Wij =

1
l ij l ji
2

0 0
0

0 0 + 2exp 2t
0 0 3exp 3t

T
0 0
0 0

0 0

(2.180)

exp 3t
2

skew

components

= exp 2t
3 exp 3t
2

exp 2t
0
0

exp 3t
2

(2.181)

The infinitesimal strain tensor ( )


Starting from the displacement field:
u1 = 0

2t
u 2 = x1 (exp 1)

3t
u 3 = x1 (exp 1)

(2.182)

the displacement gradient components can be obtained as follows:


0

r
u i
2t
(u)ij = = exp 1
x j
exp 3t 1

(
(

)
)

0 0
0 0
0 0

(2.183)

We can decompose (u) into a symmetric and an antisymmetric part:

(u)ij = ( symu)ij + ( skewu)ij = ( )ij + ()ij


r

(2.184)

The symmetric part:

r
sym u ij

0
0 0
0
0
1

2t
2t
exp 1 0 0 + exp 1 0
=
2
exp 3t 1 0 0 exp 3t 1 0

0
exp 2t 1 exp 3t 1

1
0
0
= exp 2t 1
= ij
2
3t

0
0
exp 1

(
(

)
)

(
(

)
)

0
0
0

(2.185)

We also provide the infinitesimal spin tensor:

( )ij

0
(exp 2t 1) (exp 3t 1)0

1
0
0
= (exp 2t 1)

2
3t

0
0

(exp 1)

(2.186)

Then, the rate of change of is:


University of Castilla-La Mancha
Ciudad Real - Spain

Draft

By: Eduardo W. V. Chaves (2014)

2 CONTINUUM KINEMATICS

(& )ij

247


exp 2t 1 exp 3t 1
0

D
D 1
( )ij = exp 2t 1
0
0
=

Dt
Dt 2
3t

exp 1
0
0

0
exp 2t exp 3t

0
0
= exp 2t

3 exp 3t
0
0

(2.187)

with that we conclude that:


D = &

(2.188)

Problem 2.71
Consider a material body in a small deformation regime, which is subjected to the
following displacement field:
u1 = (2 x1 + 7 x 2 ) 10 3

; u 2 = (10 x 2 x1 ) 10 3

; u 3 = x3 10 3

a) Find the infinitesimal spin and strain tensor;


b) Find the principal invariants of the infinitesimal strain tensor, as well as the
correspondent characteristic equation;
c) Draw the Mohrs circle for strain, and obtain the maximum shear strain;
d) Find the dilatation and the deviatoric infinitesimal strain tensor.
Solution
a) For the displacement gradient we obtain:

(u)ij

u1

x1
u
u
= i = 2
x
x j
1
u 3
x1

u 1
x 2
u 2
x 2
u 3
x 2

u1

x 3 2
7
0
u 2
= 1 10 0 10 3
x 3

0
1
u 3 0
x 3

m
m

In the International System of Units the displacement gradient is dimensionless, i.e.


r

[ur ] = ur = m .
x

As for the infinitesimal spin tensor we obtain:

ij =

skew r

u ij

u j
1 u
= i

2 x j x i

0 4 0

= 4 0 0 10 3


0 0 0

Then for the infinitesimal strain tensor we have:

ij =

sym r

University of Castilla-La Mancha


Ciudad Real - Spain

u ij

u j
1 u
= i +
2 x j xi

Draft

3 0
2

= 3 10 0 10 3


0
0 1

By: Eduardo W. V. Chaves (2014)

SOLVING PROBLEMS BY MEANS OF CONTINUUM MECHANICS

248

b) The principal invariants are defined as I = Tr ( ) ,

II =

III = det( ) , (see Chapter 1). Then, it follows that:

1
[Tr()]2 Tr ( 2 ) ,
2

I = Tr ( ) = (2 10 + 1) 10 3 = 11 10 3
2
3
0 2
3
0 2
3
0

1
2
2
II = [Tr ( )] Tr ( ) = 3 10 0 + 3 10 0 + 3 10 0 10 6 = 1 10 6
2
0
0
1
0
0 1
0
0
1

III = det ( ) = 11 10 9

Then, the characteristic determinant is:

2 10 3
3 10 3
0

3
3
0
10 10
=0
3 10

0
0
1 10

whilst the characteristic equation is:


3 I 2 + II III = 0

3 + 11 10 3 2 + 11 10 6 11 10 9 = 0

c) To draw the Mohrs circle for strain, (see Appendix A - textbook), we need to evaluate
the eigenvalues of . But, if we take a look at the components of we can verify that
= 1 is already an eigenvalue associated with the direction n i = [0 0 1] . So, to obtain
the remaining eigenvalues one only need solve the following system:
2 10 3
3 10 3

=0
3
10 10 3
3 10

1 = 1.0 10 3
2 + 12 10 3 + 11 10 6 = 0
2 = 11.0 10 3

Then by restructuring the eigenvalues such that I > II > III , we obtain:
I = 1.0 10 3

II = 1.0 10 3

III = 11.0 10 3

Then the maximum shear (tangential) strain is evaluated as follows:


S max =

University of Castilla-La Mancha


Ciudad Real - Spain

I III
= 6 10 3
2

Draft

By: Eduardo W. V. Chaves (2014)

2 CONTINUUM KINEMATICS

249

Finally, the Mohrs circle for strain can be depicted as:


S (10 3 )
S max = 12 max = 6

II = 1
III = 11

N (10 3 )

I = 1

d) The volumetric strain (dilatation) - V is:


V = I = Tr ( ) = 12 10 3

The additive decomposition of into a spherical and a deviatoric part is denoted by


= sph + dev , where the spherical part is given by:
ijsph

0
4 0
Tr ( )

=
ij = 0 4 0 10 3
3
0
0 4

And, the deviatoric part is given by:


ijdev

= ij

ijsph

2
3 0 4 0
0
2 3 0

3
= 3 10 0 0 4 0 10 = 3 6 0 10 3
0
0 0 4
0
0 0
0 4

Problem 2.72
At one point, the displacement gradient is represented by its components as follows:
4 1 4
r
(u) ij = 1 4 2 10 3
4 0
6

(2.189)

Obtain:
a)

the infinitesimal strain and spin tensors;

b) the components of the spherical and deviatoric parts of the infinitesimal strain tensor;
c)

the principal invariants of : I , II , III ;

d) the eigenvalues and eigenvectors of the rate-of-deformation tensor.


Solution:
a) The infinitesimal strain tensor ( ) is the symmetric part of the displacement gradient:

University of Castilla-La Mancha


Ciudad Real - Spain

Draft

By: Eduardo W. V. Chaves (2014)

SOLVING PROBLEMS BY MEANS OF CONTINUUM MECHANICS

250

r 1
r
r
= sym u = (u) + (u) T
2

(2.190)

Then:
4 1 4 4
1 4
8 0 0 4 0 0
1
1

ij = 1 4 2 + 1 4 0 = 0 8 2 = 0 4 1
2
2
0 2 12 0 1 6
6 4 2 6
4 0
r
The infinitesimal spin tensor = skewu
4 1 4 4
1 4
0 2 8 0 1 4
1
1

ij = 1 4 2 1 4 0 = 2 0
2 = 1 0
1
2
2
8 2 0 4 1 0
6 4 2 6
4 0

[ 10 ]
3

[ 10 ]
3

b) The tensor can be additively decomposed into a spherical and deviatoric part:
(2.191)

= sph + dev

where the spherical part is given by:

sph

Tr ( )
6
1 = 1 = 21
=
3
3

ijsph

2 0 0
= 0 2 0
0 0 2

[ 10 ]
3

(2.192)

The deviatoric part is given by:


ijdev

4 0 0 2 0 0 2 0 0
= 0 4 1 0 2 0 = 0 6 1
0 1 6 0 0 2 0 1 4

[ 10 ]
3

(2.193)

c) The principal invariants of are:


I = Tr ( ) = 6
II =

[ 10 ]
3

4 1 4 0 4 0
+
+
= 17
1 6 0 6 0 4

III = 4 (4) 6 4 = 100

[ 10 ]

3 2

(2.194)

[ 10 ]

3 3

d) The infinitesimal strain tensor components:


4 0 0
ij = 0 4 1
0 1 6

[ 10 ]
3

(2.195)

Note that 1 = 4 10 3 is one eigenvalue associated with the eigenvector [ 1,0,0] . To


obtain the remaining eigenvalues, we need to solve the characteristic determinant:

University of Castilla-La Mancha


Ciudad Real - Spain

Draft

By: Eduardo W. V. Chaves (2014)

2 CONTINUUM KINEMATICS

4
1
=0
1
6
=

(4 )(6 ) 1 = 0

251

2 2 25 = 0

2
b b 2 4ac 2 (2) 4 1 (25) 2 4 + 4 25
=
=
= 1 26
2a
2 1
2

(2.196)

1 = 6.0990

2 = 4.099

thus:
1 = 4 10 3 ;

2 = 6.0990 10 3 ;

3 = 4.099 10 3

(2.197)

Restructuring we obtain:
I = 6.0990 10 3 ;

II = 4 10 3 ;

III = 4.099 10 3

(2.198)

Problem 2.73
Obtain the infinitesimal strain tensor and the infinitesimal spin tensor for the following
displacement field:
u 1 = x12

u 2 = x1 x 2

u3 = 0

Solution:
The infinitesimal strain tensor
In the small deformation regime, the strain tensor is given by:
E ijL eijL ij =

1 u i u j
+
2 x j
x i

We need to obtain the displacement gradient components:


u 1

x1
u j u 2
=
x k x1

u 3
x1

u1
x 2
u 2
x 2
u 3
x 2

u1

x 3 2 x
1
u 2
x
=
2
x 3
0
u 3
x 3

0
0
0

0
x1
0

with that we can obtain:


E ijL

eijL

ij

u j
1 u
= i +
x i
2 x j

2 x

1 1
= x2
2
0

0
x1
0

0 2 x1
0 + 0
0 0

x2
x1
0

0 2 x1

0 = x 2

0 2
0

x2
2
x1
0

The infinitesimal spin tensor:

University of Castilla-La Mancha


Ciudad Real - Spain

Draft

By: Eduardo W. V. Chaves (2014)

SOLVING PROBLEMS BY MEANS OF CONTINUUM MECHANICS

252

1 u u j
ij = i
2 x j xi

2 x
1 1
= x2
2

0 2 x1
0 0
0 0

0
x1
0

x2
x1
0

0
0

x
0 = 2
2
0 0

x2
2
0
0

Problem 2.74
Figure 2.23 shows the transformation experienced by the square ABCD of unit side.
X 2 , x2

x2
D
C

45
C

B
A = A

X 1 , x1

B
x1

Figure 2.23: Body subjected to rotation.


a) State the equations of motion;
b) Is the theory valid for small deformation? justify the answer;
c) Is the finite deformation valid? Justify.
Solution:
The transformation law between systems x x is given by:

x1
x1 cos sin 0 x1

= 45
x 2 = sin cos 0 x 2 x 2 =
x
x 0
0
1 x3
3
3

2
2
2
2
0

2
2
2
2
0

0
x1

0 x 2

1 x 3

(2.199)

NOTE: Remember that by definition of the transformation matrix a ij from x to x is


given by:
cos( x1 , x1 ) cos( x1 , x 2 ) cos( x1 , x3 ) cos(315 ) cos(225 ) cos(90 )
a ij = cos( x 2 , x1 ) cos( x 2 , x 2 ) cos( x 2 , x 3 ) = cos(405 ) cos(345 ) cos(90 )
cos( x3 , x1 ) cos( x3 , x 2 ) cos( x3 , x3 ) cos(90 ) cos(90 ) cos(0 )

Considering the spatial and material coordinates are superimposed, the equations of
motion are defined by the inverse of the equation in (2.199):

University of Castilla-La Mancha


Ciudad Real - Spain

Draft

By: Eduardo W. V. Chaves (2014)

2 CONTINUUM KINEMATICS

x1

x 2 =
x
3

2
2
2
2
0

0
X 1

0 X 2

1 X 3

2
2
2
2
0

253

2
2
X1 +
X2
x1 =

2
2

2
2

x 2 = 2 X 1 + 2 X 2

For example, the point C in the reference configuration has the material coordinates
X 1C = 1 ,
x 2C =

X 2C = 1 .

After

the

motion

we

x1C =

have

2
2
(1) = 2 ,
(1) +
2
2

2
2
(1) = 0
(1) +
2
2

Displacement field:

u1 = x1 X 1 =

u 2 = x 2 X 2 =

2
2
2
2
X1
X 2 X 1 = X 1
X2
1
2
2
2
2

2
2
2
2
X1 +
X2 X2 =
X 1 + X 2
1
2
2
2

Displacement material gradient:


u1

X 1
u i u 2
=
X j X 1

u 3
X 1

u1
X 2
u 2
X 2
u 3
X 2

u1 2
1

X 3 2
u 2 2
=
X 3 2

u 3
X 3 0

The infinitesimal strain tensor is given by = symu =


2
1

ij = 0
0

1 0
2

0
0

2
2

r
r
1
(u) + (u)T , thus:
2

1 0 0 ij
2
0
0

Note that, for a rigid body motion the strain tensors must be equal to zero, i.e. = 0 (the
infinitesimal strain tensor), E = 0 (the Green-Lagrange strain tensor), e = 0 (the Almansi
strain tensor). Calculating the Green-Lagrange strain tensor components we have:
u j u k u k
1 u
+
E ij = i +
2 X j X i X i X j

University of Castilla-La Mancha


Ciudad Real - Spain

Draft

0 0 0

= 0 0 0


0 0 0

By: Eduardo W. V. Chaves (2014)

SOLVING PROBLEMS BY MEANS OF CONTINUUM MECHANICS

254

Problem 2.75
A square of side b turns counterclockwise of 30 . After turning the square is deformed
such that the base maintains its initial length and the height is doubled (see Figure 2.24).
Calculate the deformation gradient, the right Cauchy-Green deformation tensor, and the
Green-Lagrange strain tensor.
Solution:
X 2 , x2
C

x2
D

2b

30

x1

30

A = A

X 1 , x1

Figure 2.24: Body under rotation/deformation.


Note that we can apply the decomposition of motion: first we apply a pure deformation
and then a rotation is applied:
The motion is governed by the right
stretch tensor of the polar
decomposition:

X 2 , x2

2b

1 0 0
U ij = 0 2 0
0 0 1

where we have applied the


definition of stretch. Note that they
are principal values. We then apply a
rotation, where the components of
R
are the same as the
r
transformation matrix from the x r
system to the x -system:

B B
A = A

X 1 , x1

cos sin 0
R ij = sin cos 0
0
0
1

Then, applying the left polar decomposition F = R U we obtain:

University of Castilla-La Mancha


Ciudad Real - Spain

Draft

By: Eduardo W. V. Chaves (2014)

2 CONTINUUM KINEMATICS

255

cos sin 0 1 0 0 cos 2 sin 0


Fij = R ik U kj = sin cos 0 0 2 0 = sin 2 cos 0
0
0
1 0 0 1 0
0
1

For the proposed problem, we have:


cos 30 2 sin 30 0
Fij = sin 30 2 cos 30 0
0
0
1
r

As we are dealing with a homogenous deformation the equation x = F X + c holds, in


r r
this case with c = 0 . For example, for a particle at point D in the reference configuration
moves to the point:
x1D cos 30 2 sin 30 0 X 1D cos 30 2 sin 30 0 0 2b sin 30
D


D
x 2 = sin 30 2 cos 30 0 X 2 = sin 30 2 cos 30 0 b = 2b cos 30

x D 0
0
1 X 3D 0
0
1 0
0

a fact that can be easily checked by means of Figure 2.24.


By means of definition of the right Cauchy-Green deformation tensor, C = F T F , we can
obtain the Cartesian components:
sin 0 cos 2 sin 0 1 0 0
cos

C ij = Fki Fkj = 2 sin 2 cos 0 sin 2 cos 0 = 0 4 0


0
0
1 0
0
1 0 0 1

1
2

The Green-Lagrange strain tensor, E = (C + 1) , and its components are:


1 0 0 1 0 0 0 0 0



1
E ij = 0 4 0 0 1 0 = 0 1.5 0
2
0 0 1 0 0 1 0 0 0

Note that the original space coincides with the principal space. We could also have
obtained the components of C and E by means of its spectral representations:
C=

(a) N
(a) , E =
2a N

a =1

2
a

(a ) N
( a ) , where are the principal stretches.
1)N
a

a =1

University of Castilla-La Mancha


Ciudad Real - Spain

2 (

Draft

By: Eduardo W. V. Chaves (2014)

256

SOLVING PROBLEMS BY MEANS OF CONTINUUM MECHANICS

University of Castilla-La Mancha


Ciudad Real - Spain

Draft

By: Eduardo W. V. Chaves (2014)

3 Stress
3.1 Force, Stress Tensor, Stress vector
Problem 3.1
Ignoring the curvature of the Earths surface, the gravitational field can be assumed to be
uniform as shown in Figure 3.1, where g is the acceleration caused by gravity (the gravity
of the Earth). Find the resultant force acting on the body B .

x3

x2

x1

Figure 3.1: Gravitational field.

Solution:

All bodies immersed in a force field are subjected to the body force b , and in the special
case presented in Figure 3.1 this is given by:
0
r
b i ( x , t ) = 0
g

m
s2

Hence, the total force acting on the body can be evaluated as follows:
Fi =

r
0

b i ( x , t ) dV =

g dV

[m3 ]

kg m } kg m
We can also verify the F unit: [F] = 3 2 dV = 2 = N ( Newton ) .
m s
s
V

SOLVING PROBLEMS BY MEANS OF CONTINUUM MECHANICS

258

Problem 3.2
The Cauchy stress tensor components at a
point P are given by:
8 4 1
ij = 4 3 0.5 Pa
1 0.5 2
r
a) Calculate the traction vector ( t (n ) ) at P

x3

C (0,0,5)

which is associated with the plane ABC

defined in Figure 3.2


b) With reference to paragraph a).
r

B (0,2,0)

Obtain the normal ( N ) and tangential ( S )


traction vectors at P (see Appendix A of the
textbook, (Chaves(2013)).

x2

A(3,0,0)

x1

Figure 3.2: Plane ABC .


Solution:
First, we obtain the unit vector which is normal to the plane ABC . To do this we choose
two vectors on the plane:

BA = OA OB = 3e 1 2e 2 + 0e 3

BC = OC OB = 0e 1 2e 2 + 5e 3

Then, the normal vector associated with the plane ABC is obtained by means of the cross

product between BA and BC , i.e.:


e 1

r
n = BC BA = 0
3

e 2

e 3

5 = 10e 1 + 15e 2 + 6e 3
0
r
Additionally, the unit vector codirectional with n is given by:
v
n 10
15
6
n = v = e 1 + e 2 + e 3
n 19
19
19
2
2

we can obtain the traction components as:


t (i n )

= ij n j

t1
8 4 1 10
t = 1 4 3 0.5 15 Pa
2 19

t 3
1 0.5 2 6

t1
26
1

t 2 = 8 Pa
19
t 3
29.5

b) The traction vector t (n ) associated with the normal n can be broken down into a
r
r
normal ( N ) and a tangential ( S ) vector as shown in Figure 3.3. Then,
r
r
r
t (n ) = N + S

r
t ( n ) = N n + S s

or
r

where N and S are the magnitudes of N and S , respectively.

University of Castilla-La Mancha


Ciudad Real - Spain

Draft

By: Eduardo W. V. Chaves (2014)

3 STRESS

259

r
t (n )

x3

r
t ( n ) = n
r
r
r
t (n ) = N + S
r
N = N n

r
N

r
S

r
t ( n)

e 3
e 2

e 1

= 2N + 2S

x2

x1

Figure 3.3: Normal and tangential stress vector.


As we have seen in Appendix A in Chaves (2013), the normal component, N , can be
evaluated as follows:
r

N = t ( n ) n = ( n ) n = n n = : (n n ) = t i( n) n i = ( ij n j )n i = n i ij n j = ij (n i n j )

Thus:
N

= t i n i

10
1
= 2 [26 8 29.5] 15 1.54 Pa
19
6

Then the tangential component, S , can be obtained by means of the Pythagorean


Theorem, i.e.:
r
t ( n )

= 2N + 2S

2S = t i( n ) t i( n ) 2N

where
t i( n ) t i( n )

26
1
= 2 [26 8 29.5] 8 4.46
19
29.5

Thus,

S = t i( n ) t i( n ) 2N = 4.46 2.3716 2.0884 Pa

Problem 3.3
The stress state at a point in the continuum is represented by the components of the
Cauchy stress tensor as:
2 1 0
ij = 1 2 0 Pa
0 0 2

University of Castilla-La Mancha


Ciudad Real - Spain

Draft

By: Eduardo W. V. Chaves (2014)

SOLVING PROBLEMS BY MEANS OF CONTINUUM MECHANICS

260

a) Obtain the components of in a new system x1 , x 2 , x3 , where the transformation


matrix is given by:
x3

3 0 4
1
a ij = A = 0 5 0
5
4 0 3

x 2

x3

x1

where
e 3

a11 = cos 1

e 2

e 3

a12 = cos 1
a13 = cos 1

e 1

1
e

e 2

x2

M
x1

b) Obtain the principal invariants of ;


c) Obtain the eigenvalues and eigenvectors of . Also verify if the eigenvectors form a
basis transformation between the original and the principal space;
d) Illustrate the Cauchy stress tensor graphically, i.e. with the Mohrs circle in stress, (see
Appendix A in Chaves (2013));
e) Obtain the spherical ( sph ) and the deviatoric ( dev ) part of . Also, find the principal
invariants of dev ;
oct
f) Obtain the octahedral normal ( oct
N ) and tangential ( S ) components of , (see
Appendix A).

Solution:
a) As we have seen in Chapter 1, the transformation law for the components of a secondorder tensor is given by:
ij = a ik a jl kl

Matrix
form

= A AT

Thus,
3 0 4 1 1 0 3 0 4 2 0 .6 0
1
ij = 2 0 5 0 2 2 0 0 5 0 = 0.6 2 0.8
5
4 0 3 0 0 2 4 0 3 0 0.8 2

These new components ij can be appreciated in Figure 3.4.

University of Castilla-La Mancha


Ciudad Real - Spain

Draft

By: Eduardo W. V. Chaves (2014)

3 STRESS

x3
x3

261

x2
x2

P
x1

x1

= A AT

x3
33
13
13

x3

23

33

23

23

13

22

12

x2

12

11

23

13

22

x2

12

12

11
x1
x1

= AT A
Figure 3.4: Basis transformation.
b) The principal invariants of the Cauchy stress tensor can be calculated as follows:
I = Tr ( ) = ii = 11 + 22 + 33

] (

1
1
( Tr ) 2 Tr ( 2 ) = ii jj ij ij
2
2
2
2
= 11 22 + 11 33 + 33 22 12
13
223

II =

1
ii jj kk 3 ii jk jk + 2 ij jk ki
6
2
2
+ 212 23 13 11 223 22 13
33 12

III = det ( ) = ijk i1 j 2 k 3 =


= 11 22 33

By substituting the values of ij for those in the proposed problem we obtain:


I = 6

University of Castilla-La Mancha


Ciudad Real - Spain

II =

2 0
0 2

2 0
0 2

Draft

2 1
1 2

= 11

III = 6

By: Eduardo W. V. Chaves (2014)

SOLVING PROBLEMS BY MEANS OF CONTINUUM MECHANICS

262

c) The principal stresses ( i ) and principal directions ( n (i ) ) are obtained by solving the
following set of equations:
1
0 n1 0
2
1
2
0 n 2 = 0

0
0
2 n 3 0

To obtain the nontrivial solutions of n (i ) we have to solve the characteristic determinant,


which is a cubic equation for the unknown magnitude :
ij ij = 0

3 I 2 + II III = 0

However, if we look at the format of the Cauchy stress tensor components, we can notice
that we already have one solution as in the x3 -direction the tangential components are
equal to zero, then:
3 = 2 Principal
direction
n1(3) = n (23) = 0 , n 3(3) = 1

To obtain the other two eigenvalues, one only need solve:


2

= (2 ) 1 = 0
2

1 = 1

2 = 3

Then we can express the Cauchy stress tensor components in the principal space as:
1 0 0

ij = 0 3 0 Pa
0 0 2

Additionally, the principal direction associated with 1 = 1 is calculated as follows:


1
0 n1(1) 0
2 1
(1)
(1)
1
n (1) = 0 n1 + n 2 = 0 n (1) = n (1)

2
1
0
1
2

2 n (1) + n (1) = 0
2
0
0
2 1 n (31) 0 1
2

with n 3(1) = 0 and by using the condition n1(1) + n (21) = 1 we obtain:


n1(1) = n (21) =

1
2

1
then n i(1) =

Since is a symmetric tensor, the principal space is formed by an orthogonal basis, so, it
is valid that:
n (1) n ( 2 ) = n ( 3)

n ( 2 ) n ( 3) = n (1)

n ( 3) n (1) = n ( 2)

Thus, the second principal direction can be obtained by the cross product between n ( 3)
and n (1) , i.e.:
n

(2)

= n

( 3)

(1)

e 1
= 0
1

e 2
0
1

e 3
1
1
1 =
e1 +
e2
2
2
0

which can also be checked by the following analysis:

University of Castilla-La Mancha


Ciudad Real - Spain

Draft

By: Eduardo W. V. Chaves (2014)

3 STRESS

263

The Principal direction associated with 2 = 3 :


1
0 n1( 2 ) 0
2 3
( 2)
( 2)
n ( 2 ) = 0 n1 + n 2 = 0 n ( 2 ) = n ( 2 )
1
2
3
0

1
2
2 n ( 2 ) n ( 2 ) = 0

2
0
0
2 3 n 3( 2 ) 0 1
2

With n 3(3) = 0 and using the condition n1(3) + n (23) = 1 we obtain:


n1( 2 ) = n (22 ) =

then n i( 2 ) =

1
2

As we have seen in Chapter 1, the eigenvectors of a symmetric tensor form the


transformation matrix D , from the original system to the principal space, i.e.
= D D T , thus:

0
0
1 = 1
0
2 = 3
0 =

0
3 = 2
0

2
1

2
1

1
0

2 1 0 2
1
0 1 2 0

2
0 0 2

1
0

2
1
2
0

d) The graphical representation of a second-order tensor can be obtained from the


description in Appendix A. To do this we have to restructure the eigenvalues of so that
I > II > III , thus:
I = 3

II = 2

III = 1

Then the three circumferences are defined by:


Circle 1

Circle 2

Circle 3

1
( II + III ) = 1.5
2
1
(center )C 2 = ( I + III ) = 2.0
2
1
(center )C 3 = ( I + II ) = 2.5
2

(center )C1 =

;
;
;

1
( II III ) = 0.5
2
1
(radius ) R 2 = ( I III ) = 1.0
2
1
(radius ) R3 = ( I II ) = 0.5
2
(radius ) R1 =

Then, we can illustrate the Cauchy stress tensor at P by means of Mohrs circle in stress as
shown in Figure 3.5.

University of Castilla-La Mancha


Ciudad Real - Spain

Draft

By: Eduardo W. V. Chaves (2014)

SOLVING PROBLEMS BY MEANS OF CONTINUUM MECHANICS

264

S max =

S max = 1

1
( I III ) = 1.0
2

R2
R1
C3
III = 1

C1

R3

II = 2

I = 3 = N max

Figure 3.5: Mohrs circle in stress at the point P .


e) As defined in Chapter 1 of the textbook, a second-order tensor can be broken down
additively into a spherical and a deviatoric part, i.e.:
Tensorial notation

Indicial notation

ij = ijsph + ijdev

= sph + dev
= m1 +

1
= kk ij + ijdev
3
= m ij + ijdev

dev

(3.1)

A schematic representation of these components in the Cartesian basis can be appreciated


in Figure 3.6 and the value of the scalar m is evaluated as follows:
m =

11 + 22 + 33 1 + 2 + 3 1
I
1
6
=
= kk = Tr ( ) = = = 2
3
3
3
3
3 3

Then the spherical part becomes:


ijsph

2 0 0
= m ij = 2 ij = 0 2 0
0 0 2

And, the deviatoric part can be evaluated as follows:


ijdev

11 12 13 m 0
= 12 22 23 0 m
13 23 33 0
0
13 (211 22 33 )

1
(2 22
=
12
3

13

0
0
m
12
11 33 )
23

23

11 22 )
13

1
3

(2 33

Thus,
ijdev

University of Castilla-La Mancha


Ciudad Real - Spain

1
0 0 1 0
2 2

= 1
22
0 = 1 0 0
0
0
2 2 0 0 0

Draft

By: Eduardo W. V. Chaves (2014)

3 STRESS

265

Now let us remember from Chapter 1 that and dev are coaxial tensors, i.e., they have
the same principal directions, so we can use this information to operate in the principal
space of to obtain the eigenvalues of dev = sph . With that we obtain:
ijdev

1
= 0
0

m
0

3 0

0
2

0
0

0 1 0 0
0 = 0 1 0
m 0 0 0

0
m
0

Then the invariants of dev are given by:


I dev = Tr ( dev ) = 0

II dev = 1

III dev = 0

Traditionally, in engineering, the invariants of the deviatoric stress tensor are represented
by:
J1 = I dev = 0
J 2 = II

dev

J 3 = III dev

1 2
I 3 II
3
1
=
2 I 3 9 I II + 27 III
27
=

x3

33
23

13
13

23
12

11

22
12

x2

14414444442444444443
x3

x3

dev
33

23

13
m

13

x2

23
12

dev
22
12

dev
11

x1

x2

x1

sph

dev

Figure 3.6: The spherical and deviatoric part of .

University of Castilla-La Mancha


Ciudad Real - Spain

Draft

By: Eduardo W. V. Chaves (2014)

SOLVING PROBLEMS BY MEANS OF CONTINUUM MECHANICS

266

f) The octahedral normal and tangential components, (see Appendix A in Chaves (2013)),
can be expressed as:
oct
N =

oct
S

oct

1
(1 + 2 + 3 ) = 1 ii = I = m
3
3
3

1
2
2 I 2 6 II =
=
J2 =
3
3

( ) + ( ) + ( )
dev 2
1

dev 2
2

dev 2
3

Then, by substituting the values of the proposed problem we obtain:


oct
N = m = 6

oct =

2
2
J2 =
3
3

Problem 3.4
At point P the Cauchy stress tensor components are:
1 2 3
ij = 2 4 6 MPa
3 6 1

Find:

(3.2)

a) the traction vector t related to the plane which is normal to the x1 -axis;
r

b) the traction vector t associated with the plane whose normal is (1,1,2) ;
r

c) the traction vector t associated with the plane parallel to the plane 2 x1 2 x 2 x3 = 0 ;
d) the principal stress at the point P ;
e) the principal directions of at the point P .
Solution:
a) In this case, the unit vector is (1,0,0) . Then, the traction vector is given by:

t i(n)

1 2 3 1 1
= 2 4 6 0 = 2
3 6 1 0 3

(3.3)

b) The unit vector associated with the direction (1,1,2) is:


1
1
1
n i =
6
2

(3.4)

thus,

t i(n)

1 2 3 1
5
1
1

=
2 4 6 1 =
10
6
6
3 6 1 2
1

University of Castilla-La Mancha


Ciudad Real - Spain

Draft

(3.5)

By: Eduardo W. V. Chaves (2014)

3 STRESS

267

c)
2
1 2 3 2
5
n i = 1 2 t i(n ) = 1 2 4 6 2 = 1 10
3

3
3
1
3 6 1 1
7

(3.6)

d) Solving the characteristic determinant


1
2
2
4
3

3
6

=0

(3.7)

we obtain:
1 = 10 ; 2 = 0 ; 3 = 4

(3.8)

e) The principal directions are:


Associated with 1 = 10
9n1 + 2n 2 + 3n 3 = 0

2n1 6n 2 + 6n 3 = 0
3n + 6n 9n = 0
2
3
1

n i(1)

3
= 6
5

(3.9)

Similarly, we obtain:
n (i 2 )

2
1

( 3)
= 1 ; n i = 2
0
3

(3.10)

Normalization of the principal directions:


3
n (1)
n i(1) = r i = 1 6
70
n (1)
5

2
n ( 2)
n i( 2 ) = r i = 1 1
5
n ( 2)
0

1
n ( 3)
n i(3) = r i = 1 2
14
n ( 3)
3

Problem 3.5
r

Show that S = t (n) (1 n n ) , where t (n) is the traction vector resulting from

projecting the second-order tensor onto the n -direction, and S is the tangential stress
vector associated with the plane.
Solution 1:

r
r
r r
r
r
S = t (n) t (n) n n = t (n) t (n ) n n = t (n) (1 n n )

Solution 2:
We can also solve the problem just using the components of the equation
r
r
S = t (n) [ : (n n )]n :

S i = t i(n) (n k n l kl ) n i = t i(n) n in k t (kn ) = t (kn ) ik n in k t (kn) = t (kn) ( ik n in k )

which in tensorial notation becomes


r
r
S = t (n) (1 n n )
University of Castilla-La Mancha
Ciudad Real - Spain

Draft

By: Eduardo W. V. Chaves (2014)

SOLVING PROBLEMS BY MEANS OF CONTINUUM MECHANICS

268

Problem 3.6
The stress state at one point P of the continuous medium is given schematically by:
x3

1
22

4
1

x2

x1

Obtain the value of the component 22 of the Cauchy stress tensor such that there is at
least one plane passing through P in which is free of stress;
Obtain the direction of the plane.
Solution:
r

We seek to find a plane whose direction is n such that t (n ) = 0 . We can relate the Cauchy
stress tensor to the traction vector by means of the equation:
r
t (n) = n

thus:
t1(n ) 0 1
(n )
t 2 = 1 22
t (n ) 4 1
3

4 n1 0

1 n 2 = 0
0 n3 0

Resulting in the following system of equations:


1

n2 + 4n3 = 0 n3 = 4 n 2

n1 + 22n 2 + n3 = 0

1
4n1 + n2 = 0 n1 = n 2
4

By combining the above equations we obtain:


n1 + 22n 2 + n3 = 0

1
1
n2 + 22n 2 n2 = 0
4
4

1
1
+ 22 n 2 = 0
4
4
r

1
1
1
+ 22 = 0 22 = .
4
4
2

Then, for n 0 , we have:

To determine the direction of the plane we start by the restriction n i n i = 1 , then:

University of Castilla-La Mancha


Ciudad Real - Spain

Draft

By: Eduardo W. V. Chaves (2014)

3 STRESS

269
2

nini = 1 n12 + n22 + n32 = 1


n2 =

2 2
3

n1 = n3 =

1
1
2
n2 + n2 + n2 = 1
4

2
6

Thus, the direction of the normal to the plane, when it meets t (n ) = 0 :


1
2
4
n i =
6
1

3.2 Eigenvalues and Eigenvectors of the Stress Tensor


Problem 3.7
The stress field in the medium is represented by:
1
ij = 0
2 x2

0
1
4 x1

2 x2
4 x1
1

(3.11)

where xi are the Cartesian coordinates.


a) Obtain the traction vector acting at the point ( x1 = 1, x2 = 2, x3 = 3) associated with the
plane x1 + x 2 + x3 = 6 ;
b) Obtain the projection of the traction vector according to the normal and tangential
direction to the plane x1 + x 2 + x3 = 6 ;
Solution:
a) The unit vector which is normal to the plane x1 + x 2 + x3 = 6 is:

1
n i = 1 1
3
1

(3.12)

r
t (n) = n

(3.13)

1 0 4
ij ( x1 = 1, x 2 = 2, x3 = 3) = 0 1 4
4 4 1

(3.14)

The traction vector t (n) :

thus,

t i(n)

1 0 4
1
5
1
1

= 0 1 4
1 =
5
3
3
4 4 1
1
9

(3.15)

b) The normal stress associated with this plane is

University of Castilla-La Mancha


Ciudad Real - Spain

Draft

By: Eduardo W. V. Chaves (2014)

SOLVING PROBLEMS BY MEANS OF CONTINUUM MECHANICS

270

1
r (n )
1
1
N = t n =
[5 5 9] 1 = 1 (5 + 5 + 9) = 19
3
3
3
3
1

(3.16)

and the tangential stress is


r r
2S = 2N + t (n) t (n)

(3.17)

5
r (n ) r (n )
1
t t =
[5 5 9] 5 1 = 131
3
3
3
9

(3.18)

thus
2

32
19 131
2S = +
=
3
9
3

(3.19)

Problem 3.8
Given a continuum where the stress state is known at one point and is represented by the
Cauchy stress tensor components:
1 1 0
ij = 1 1 0 Pa
0 0 2

(3.20)

a) Find the principal stresses and the principal directions (eigenvectors).


Solution:
To obtain the principal stresses i = i and principal directions n (i ) we must solve the
following system of equations:
1
1
1
1

0
0

0 n1 0
0 n 2 = 0
2 n 3 0

(3.21)

for nontrivial solutions of n (i ) , which is equivalent to solve:


ij ij = 0

But if we look at the format of the matrix (3.20), we can note that we have one solution,
since the tangential components in the x3 -direction are zero, then:
direction
1 = 2
n1(1) = n (21) = 0 , n3(1) = 1

To obtain the remaining solutions it is sufficient to solve:


1
1
= (2 ) = 0
1
1

We can easily verify that the roots of the above equations are:
2 = 2 and 3 = 0

University of Castilla-La Mancha


Ciudad Real - Spain

Draft

By: Eduardo W. V. Chaves (2014)

3 STRESS

271

We express the stress tensor components in the principal space as follows:


2 0 0
ij = 0 2 0 Pa
0 0 0

b) The principal directions


b.1) To obtain the principal direction associated with the solution 2 = 2 , we substituting
this solution into the equation (3.21):
1
0 n1( 2 ) 0
1 2
n ( 2 ) = 0
1
1
2
0

0
0
2 2 n 3( 2 ) 0

n1( 2 ) + n (22 ) = 0
( 2)
n1 n (22 ) = 0

Solving the system we obtain n 3( 2) = 0 , n1( 2) = n (22) and by using the restriction
2

n1( 2 ) + n (22 ) = 1 we obtain: n1( 2 ) = n (22 ) =

. n ( 2 ) =

b.2) For the solution 3 = 0 , we obtain:


n1( 3) + n (23) = 0

n1( 3) + n (23) = 0
(3)
2n 3 = 0

1
0 n1( 3) 0
1 0

1
1 0
0 n (23) = 0

0
0
2 0 n 3( 3) 0

Solving the system we obtain n (33) = 0 , n1(3) = n (23) and using the restriction n1(3) + n (23) = 1 ,
we obtain: n1(3) =

1
1
1
, n (23) =
. n (3) =
2
2
2

1
2

0 .

As we have seen, the eigenvectors form a matrix transformation ( A ) between the two
systems, i.e. = A A T , thus:

0
0
1 = 2
0
2 = 2
0 =

0
3 = 0
0

1
2
1
2

1
2
1

0
1

1 1 0
1
0 1 1 0
2

0 0 2 1
0

0
1
2
1

Problem 3.9
A prismatic dam is subjected to water pressure. The dam has thickness equal to b and
height equal to h , (see Figure 3.7). Obtain the restrictions of the Cauchy stress tensor
Cartesian components on the faces BC , OB and AC .

University of Castilla-La Mancha


Ciudad Real - Spain

Draft

By: Eduardo W. V. Chaves (2014)

SOLVING PROBLEMS BY MEANS OF CONTINUUM MECHANICS

272

x2

a - mass density of water


g - acceleration of gravity
C

a
a g (h x 2 )
h

x1

Figure 3.7
Solution:
The face BC has normal vector n (i BC ) = [0 1 0] . Considering that this face has no
traction vector, we conclude that:
t i( BC )

= 0 i = ij n j

11

21
31

12
22
32

13 0 12 0
23 1 = 22 = 0
33 0 32 0

what is the same as i 2 = 0 and because of the symmetry of we have 2i = 0 .


The face OB has as normal vector n (i BC ) = [ 1 0 0] . Considering that in this face the
traction vector components are t i(OB ) = [ a g (h x 2 ) 0 0] , we conclude that:
t i(OB )

a g ( h x 2 )
= n
0
=
ij j

11

21
31

12
22
32

13 1 11 a g (h x 2 )

0
23 0 = 21 =

0
33 0 31

which is the same as i1 = a g (h x 2 ) i1 .


The face AC has normal vector n (i BC ) = [1 0 0] . Considering that in this face there is no
traction vector, we conclude that:
t i( AC )

= 0 i = ij n j

11

21
31

12
22
32

13 1 11 0
23 0 = 21 = 0
33 0 31 0

which is the same as i1 = 0 and because of the symmetry we have 1i = 0 .

University of Castilla-La Mancha


Ciudad Real - Spain

Draft

By: Eduardo W. V. Chaves (2014)

3 STRESS

273

3.3 Other measure of stress


Problem 3.10
Prove that the following relationship are valid:
P = J dev F T + J m F T

S = JF 1 dev F T + J m C 1

where P and S are the first and second Piola-Kirchhoff stress tensors, respectively, C is
the right Cauchy-Green deformation tensor, F is the deformation gradient, J is the
Jacobian determinant, and the scalar m is the mean normal Cauchy tress. Also prove that
the following relationships are true:
P : F = S : C = 3J m

Solution:
First of all we prove that P : F = S : C :
P : F = Pij Fij = ( Fik S kj ) Fij = S kj ( Fik Fij ) = S kj ( F T F ) kj = S kj (C )kj = S : C

Secondly, by referring to the definition P = J F T , and the different components of


by = sph + dev , we obtain:
P = J ( dev + m 1) F T = J dev F T + Jm 1 F T = J dev F T + Jm F T

Thirdly, by taking into account the definition S = JF 1 F T , and by breaking down


into = sph + dev , we obtain:
S ij = JFik1 kp F jp1

S = JF 1 F T
= JF 1 ( dev + m 1) F T

1
= JFik1 ( dev
kp + ( m ) kp ) F jp

= JF 1 dev F T + JF 1 m 1 F T

1
1
1
= JFik1 dev
kp F jp + JFik ( m ) kp F jp

= JF 1 dev F T + J m ( F T F ) 1

1
1 1
= JFik1 dev
kp F jp + J ( m ) Fik F jk

= JF 1 dev F T + J m C 1

1
1
= JFik1 dev
kp F jp + J ( m ) C ij

Then by applying the double scalar product between S and C we can obtain:
S : C = ( JF 1 dev F T + J m C 1 ) : C = JF 1 dev F T : C + J m C 1 : C

where the term JF 1 dev F T : C becomes:


JF 1 dev F T : C

= ( JF 1 dev F T ) : {
C

( JF 1 dev F T ) ij ( F T F ) ij

1
= ( Fip1 dev
pk F jk )( Fqi Fqj )

F T F

= J qp qk dev
pk
dev
= J dev
pk pk = J kk
dev
=J
:1
1
424
3

=0

Tr ( dev ) =0

Thus:
S : C = J m C 1 : C = J m Tr (C 1 C ) = J m Tr (1) = 3 J m
University of Castilla-La Mancha
Ciudad Real - Spain

Draft

By: Eduardo W. V. Chaves (2014)

SOLVING PROBLEMS BY MEANS OF CONTINUUM MECHANICS

274

Now, by taking the double scalar product between P and F we obtain:


P : F = J dev F T : F + J m F T : F

Then by analyzing the term J dev F T : F we can conclude that:


dev
J dev F T : F = ( J dev F T ) ij ( F ) ij = J ikdev F jk1 Fij = J ikdev ik = J
:1 = 0
1
424
3

Tr ( dev ) =0

Thus,
P : F = J m F T : F = J m Tr ( F T F T ) = J m Tr (1) = 3 J m

3.4 Maximum shear stress, Mohr circle in stress


Problem 3.11
Obtain the maximum shear stress at a point in which the stress state is given by:
x2

30 MPa

20 MPa
x1

x3

Figure 3.8
Solution:
Note that the axes xi are
principal directions. We draw the
Mohrs circle with the principal
stresses:

max (MPa)

I = 30 MPa , II = 20MPa and


III = 0 .

20

30

N (MPa)

Figure 3.9

max =

University of Castilla-La Mancha


Ciudad Real - Spain

30 0
2

= 15 MPa

Draft

(3.22)

By: Eduardo W. V. Chaves (2014)

3 STRESS

275

Problem 3.12
Consider the stress state at a point represented by the infinitesimal element shown in
Figure 3.10.
a) Draw the Mohrs circle;

x2

b) Obtain the maximum normal stress,


and indicate the plane in which occurs;

20 MPa

c) Obtain the maximum shear stress.

5 MPa
x1

10 MPa

x3

Figure 3.10
Solution:
S
max = 15

N max = 10MPa
S max =

20

10 (20)
= 15MPa
2

N (MPa)

10

Problem 3.13
Determine for which values of * are possible
the following stress state at the point P :

Case a) N = 4

=2

Case b) N = 4

=1

Case c) N = 7

=0

Figure 3.11
Solution

University of Castilla-La Mancha


Ciudad Real - Spain

Draft

By: Eduardo W. V. Chaves (2014)

SOLVING PROBLEMS BY MEANS OF CONTINUUM MECHANICS

276

The pair ( N ; ) is feasible if it belongs to the gray zone of the Mohrs circle including the
circumferences, (see Figure 3.12).

Figure 3.12: Mohrs circle in stress.

Case a)
Case b)

Case c)

1
2

Figure 3.13: Mohrs circle.

University of Castilla-La Mancha


Ciudad Real - Spain

Draft

By: Eduardo W. V. Chaves (2014)

3 STRESS

277

Case a): In this case the pair ( N = 4; = 2) belongs to the circumference formed by the
principal stresses 2 and 6 , thus * can assume any value, (see Figure 3.14).

Case a)

2
1

Figure 3.14: Mohrs circle.


Case b) In this case the solution is:
*( 2) * *(1)

(3.23)

where *( 2) and *(1) are identified in Figure 3.15.

Limit cases

( 4,1)

( 4,1)

1
2

* ( x ) 6

* ( 2)

* (1) 6

Figure 3.15: Mohrs circle.


Starting from the circumference equation:
( x xC ) 2 + ( y y C ) 2 = R 2

University of Castilla-La Mancha


Ciudad Real - Spain

Draft

(3.24)

By: Eduardo W. V. Chaves (2014)

SOLVING PROBLEMS BY MEANS OF CONTINUUM MECHANICS

278

For the case

*(1)

, we have: x = 4; x C =

( *(1) + 2)
2

y = 1; y C = 0; R =

( *(1) 2)
2

Substituting these values into the circumference equation we obtain:


2

( x xC ) + ( y yC ) = R

For the case

*( 2 )

(* 2)
(* + 2)
4 (1)
+ (1 0)2 = (1)
*(1) = 4.5

2
2

, we have: x = 4; xC =

(6 + *( 2 ) )
2

; y = 1; y C = 0; R =

(6 *( 2 ) )
2

substituting these values into the circumference equation we obtain:


2

( x xC ) + ( y yC ) = R

(6 *( 2 ) )
(6 + *( 2 ) )
4
+ (1 0 )2 =
*( 2) = 3.5

2
2

thus:
(3.25)

3.5 * 4.5

Case c) In this case the only possible solution is that N is a principal stress, then
(3.26)

* = 7

* = 7

Figure 3.16: Mohrs circle.


Problem 3.14
Obtain the maximum normal and tangential stress and draw the corresponding Mohrs
circle in stress for the following stress states:
a)

0
ij = 0
0 0 0

University of Castilla-La Mancha


Ciudad Real - Spain

(3.27)

Draft

By: Eduardo W. V. Chaves (2014)

3 STRESS

b)

279

2 0 0
ij = 0
0
0
0

(3.28)

Solution:
a) The principal values. If we check the format of the Cauchy stress tensor components, we
can observe that the value (3) = 0 is already an eigenvalue. Then, to obtain the remaining
eigenvalues, it is sufficient to solve:


= ( ) 2 2 = 0 = = 0

(1) = 0
( ) 2 2 = 0 2 2 + 2 2 = 0 (2 + ) = 0
( 2) = 2

(3.29)
(3.30)

N max = 2

max =

max =

Figure 3.17:
b)

N max =
max =

(2) 3
=
2
2

Figure 3.18.

University of Castilla-La Mancha


Ciudad Real - Spain

Draft

By: Eduardo W. V. Chaves (2014)

SOLVING PROBLEMS BY MEANS OF CONTINUUM MECHANICS

280

Problem 3.15
Make the representation of the Mohrs circle for the following cases:
1) Unidimensional case (traction);
2)Unidimensional case (compression);
3) Bidimensional case (traction)
4) Triaxial case
5) State of pure shear
Solution:
1) Unidimensional case (traction)

x
I
I
0

0 0
0 0
0 0

2) Uniaxial compression

x
0
0
0
II

0
0

0
0
0

II

3) Biaxial case
II

I
II
I
0

University of Castilla-La Mancha


Ciudad Real - Spain

0
II
0

0
0
0

Draft

By: Eduardo W. V. Chaves (2014)

3 STRESS

281

4) Triaxial case

III

I
0

II

0
II
0

0
0
III

III

II

5) State of pure shear


S
0 0
0 0

0 0 0

3.5 Feature of the stress tensor


Problem 3.16
The Cauchy stress tensor components at point P are given by:
5 6 7
ij = 6 8 9 GPa
7 9 2

(3.31)

Obtain:
a) the mean stress;
b) The deviatoric and volumetric part of the tensor .
Solution:
The mean stress
m =

kk 5 + 8 + 2
=
=5
3
3

(3.32)

b)

University of Castilla-La Mancha


Ciudad Real - Spain

Draft

By: Eduardo W. V. Chaves (2014)

SOLVING PROBLEMS BY MEANS OF CONTINUUM MECHANICS

282

ijsph

m
= 0
0

0
m
0

ij = ijsph + ijdev

0 5 0 0
0 = 0 5 0
m 0 0 5
ijdev = ij ijsph

ijdev

0 6 7
= 6 3 9
7 9 3

(3.33)

Problem 3.17
Consider the Cauchy stress tensor components, in the Cartesian base (e 1 , e 2 , e 3 ) :
5 3 2
ij = 3 1 0
2 0 3

(3.34)

Given the transformation law between the systems x and x' :


x'1

x' 2

x '3

x1

3
5

4
5

x2

3
5

x3

4
5

where the system x' is constituted by the basis (e '1 , e ' 2 , e ' 3 ) .
r

a) Obtain the traction vector t ( e'2 ) associated with the plane whose normal is e ' 2 . Express
the result in the Cartesian system (e '1 , e ' 2 , e ' 3 ) according to the format:

r
t (e'2 ) = ( )e 1 + ( )e 2 + ( )e 3

(3.35)

b) Obtain the spherical and deviatoric parts of the Cauchy stress tensor.
Solution:
a) As defined in the textbook (see Chaves (2013)), the first row of the transformation
matrix is formed by the direction cosines between the x'1 -axis with x1 , x2 and x3 , thus:
3 0 4
1
A = 0 5 0
5
4 0 3

(3.36)

and the transformation law for the second-order tensor components is:
thus:

University of Castilla-La Mancha


Ciudad Real - Spain

' = A A T

Draft

(3.37)

By: Eduardo W. V. Chaves (2014)

3 STRESS

53 0 45

ij = 0 1 0
4 0 3
5
5
9

1

t i(e' 2 ) = 5
5
12

5 3 2 53
3 1 0 0

2 0 3 45

283
4

9 9 2
1
1 0 = 9 5 12
5
2 12 31
0 35
0

(3.38)

r
9
12
t (e'2 ) = e 1 + (1)e 2 + e 3
5
5

(3.39)

since:

11

21
31

t 1 ( e'1 )
13
( e ' )
23 = t 2 1
( e ' )
33 t 3 1

12
22
32

t1

( e ' 2 )
( e ' 2 )

t2
( e ' )
t3 2

t1

( e '3 )

( e '3 )

t2

( e ' )
t3 3

(3.40)

b)
I
ij + ijdev
3
I = 5 +1+ 3 = 9

ij = ijsph + ijdev =

(3.41)
(3.42)

3 0 0
= 0 3 0
0 0 3
2 2 3 2
5 3 3

sph
= ij ij = 3 1 3
0 = 3 2 0
2
0 3 3 2 0 0
ijsph

ijdev

(3.43)

(3.44)

Problem 3.18
The stress state in an continuous medium is given by the Cauchy stress tensor Cartesian
components:
0
ij = Cx 3
0

Cx 3
0
Cx1

0
Cx1
0

where C is a constant. Consider that the body is free of body force.


a) Calculate the traction vector at the point P (4,4,7) associated with the plane whose
2
3

2
3

1
3

normal vector is given by n = e 1 + e 2 e 3 .


b) Represent the Mohrs circle in stress at the point P .
Solution:
a) The traction vector is given by:
r
(n )
t (n) = n ; t i = ij n j

0
ij ( x1 = 4; x 2 = 4; x3 = 7) = Cx3
0

University of Castilla-La Mancha


Ciudad Real - Spain

Cx3
0

Cx1

Draft

0 0
Cx1 = 7C
0 0

(3.45)
7C
0
4C

0
4C
0

(3.46)

By: Eduardo W. V. Chaves (2014)

SOLVING PROBLEMS BY MEANS OF CONTINUUM MECHANICS

284

2
1
n j = 2
3
1

(3.47)

thus, we obtain:
0
r (n )

t i = ij n j = 7C
0

0 2
14C
1 1

4C 2 = 18C
3
3
8C
0 1

7C
0
4C

(3.48)

b)
0
0 7

ij = C 7 0 4
0 4 0

(3.49)

The eigenvalues (principal stresses) are obtained by solving:


0 0 0
0 7

C 7 0 4 0 0 = 0
0 4 0 0 0

C
0
0 7

C 7 0 4 C 0

0 4 0

C
0

0 =0

(3.50)

Considering that = C we obtain:


0
7

C 7 4 = C 3
0 4
3 + 16 + 49 = 0

0
7
7 4 = 0

0 4

2 + 65 = 0

(3.51)
(3.52)

= 65

With that we obtain = C 65


Then, the Mohrs circle is represented by:
S

C 65

III = C 65

University of Castilla-La Mancha


Ciudad Real - Spain

I = C 65

Draft

By: Eduardo W. V. Chaves (2014)

3 STRESS

285

Problem 3.19
The stress state at a point of the body is given by the Cauchy stress tensor components as
follows:
x3

r
t ( e 3 ) = 8e 1

r
t ( e 2 ) = 6e 1

e 3
e 1

e 2

x2

r
t ( e1 ) = 6e 2 + 8e 3

x1

Figure 3.19:
a) Obtain the deviatoric part of the stress tensor;
b) Obtain the principal stresses ( I , II , III ) and the principal directions;
c) Draw the Mohrs circle in stress;
d) Obtain the maximum shear stress at the point;
e) Find the traction vector associated with the plane whose normal vector is
6
n = 0.75e 1 + 0.25e 2
e3 ;
4

f) Obtain the normal and tangential stress vector associated with the plane described in
paragraph (e).
Solution:
According to Figure 3.19 we can obtain the Cauchy stress tensor components as follows:
0 6 8
ij = 6 0 0
8 0 0

a)
ij = ijsph + ijdev

The spherical part is ijsph =

I
ij = 0 ij since I = 0 . Then, the deviatoric part is given by:
3
ijdev

ijsph

0 6 8
ij = 6 0 0
8 0 0

b) The eigenvalues can be obtained by means of the characteristic determinant:

University of Castilla-La Mancha


Ciudad Real - Spain

Draft

By: Eduardo W. V. Chaves (2014)

SOLVING PROBLEMS BY MEANS OF CONTINUUM MECHANICS

286

6
6
8

8
0 =0

3 + 100 = 0

2 + 100 = 0

whose solutions are 1 = 0 , 2 = 10 , 3 = 10 , (principal stresses). The principal directions are:


eigenvector
1 = 0

n i(1) = [0 0.8 0.6]

eigenvector
2 = 10

n i( 2 ) = [ 0.707 0.424 0.566]


eigenvector
3 = 10

n (i 3) = [0.707 0.424 0.566]

I = 10 , II = 0 , III = 10

c) The Mohrs circle can be appreciated as follows:


S
max = 10

II = 0

III = 10

I = 10

d) We can directly obtain the maximum shear stress by means of the Mohrs circle:
max =

I III
= 10
2

e) Considering t i (n) = ij n j , we can obtain the traction vector associated with the plane whose
normal vector is n = 0.75e 1 + 0.25e 2

6
e3 :
4

t 1 (n ) 0 6 8 0.75 3.39898
(n )


t 2 = 6 0 0 0.25 4.5
t (n ) 8 0 0
6


3
4

f)
r
t (n)

r
S

r
N
s

University of Castilla-La Mancha


Ciudad Real - Spain

Draft

By: Eduardo W. V. Chaves (2014)

3 STRESS

287

The magnitude of N can be obtained by the projection N = t (n) n = t i (n) n i , thus:


r
N

0.75
(n )
= t i n i [ 3.39898 4.5 6] 0.25 5.09847

6
4

The vector N is given by:


r
r
N = N n = 3.82385e 1 1.27462e 2 + 3.12216e 3
r
r
r
In addition, the relationship t (n) = N + S holds, with that the tangent stress vector is

obtained as follows:

r
r
r
S = t (n) N ( 3.39898 + 3.82385)e 1 + (4.5 + 1.27462 )e 2 + (6 3.12216 )e 3
(0.42487 )e 1 + (5.77462 )e 2 + (2.87784 )e 3

and its module as:


r
S

(0.42487 )2 + (5.77462)2 + (2.87784)2


r

NOTE: We could also have used the equation S

41.808713 = 6.465966
2

r
module of S .

r r
r
= t (n) t (n) N

to obtain the

Problem 3.20
The Cauchy stress tensor field of a continuous medium is represented by:
3 x1
r
ij ( x ) = 21

31

5 x 22
3x 2
32

2 x3
0

a) Obtain the body force (per unit volume) to ensure the balance of the continuum.
b) For a particular point ( x1 = 1, x 2 = 1, x3 = 0 ):
b.1) Draw the Mohrs circle. Obtain the maximum normal and tangential stress
component.
b.2) Obtain the traction vector associated with the plane whose normal is
1
ni =
3

1
3

b.2.1) Obtain the normal and tangential in this plane.


Solution:
a) Due to the symmetry of the Cauchy stress tensor ( = T ) we have:
3 x1
r 2
ij ( x ) = 5 x 2
0

University of Castilla-La Mancha


Ciudad Real - Spain

Draft

5 x 22
3x 2
2 x3

2 x3
0

By: Eduardo W. V. Chaves (2014)

SOLVING PROBLEMS BY MEANS OF CONTINUUM MECHANICS

288

+ 12, 2 + 13,3 = b1
3 + 10 x 2 + 0 = b1
r r components 11,1

r
x + b = 0
21,1 + 22, 2 + 23,3 = b1 0 + 3 + 2 = b 2
0 + 0 + 0 = b

31,1 + 32, 2 + 33,3 = b1

with that we obtain:


10 x 2 3
b i = 5

(Force per unit volume)

(3.53)

b) For the particular point ( x1 = 1, x 2 = 1, x3 = 0 ) we have:


3 5 0
ij = 5 3 0
0 0 0

where we can verify that 3 = 0 is one principal value. For the other eigenvalues, it is
sufficient to solve:
3
5
=0
5
3

(3 ) = (5) 2

3 = 5

1 = 8

2 = 2

Restructuring the eigenvalues:


I = 8 , II = 0 , III = 2

b.1) The Mohrs circle is drawn in figure below:


S
max = 5

II = 0

III = 2

I = 8

By means of the Mohrs circle we can obtain the maximum shear stress max = 5 and the
maximum normal stress N max = I = 10 .

e) Considering that t i (n) = ij n j , we can obtain the traction vector associated with the plane
1
1
1
whose normal vector is n =
e1 +
e2 +
e3 :
3

University of Castilla-La Mancha


Ciudad Real - Spain

Draft

By: Eduardo W. V. Chaves (2014)

3 STRESS

289

t 1 (n )
3 5 0 1
8
(n ) 1
1

5 3 0 1 =
8
t 2 =
3
3
t (n )
0 0 0 1
6
3

b.2) The normal stress component is obtained as follows:


N = ti

(n )

n i =

1
3

1
[8 8 0] 1 = 16
3
3
1

To obtain the tangential component we apply directly S


r
t (n)

r r
r
= t (n) t (n) N

, where

8
r (n ) r (n )
1 1
(n ) (n )
= t t = ti ti =
[8 8 0] 8 = 128 . Then:
3
3 3
0
r
S

r r
r
= t (n) t (n) N

128 16
128
=
3 3
9

S =

128
3

Problem 3.21
The stress state at one point of the body is given by means of the spherical and deviatoric
part of the Cauchy stress tensor as follows:
ijsph

1 0 0
= 0 1 0
0 0 1

ijdev

0 6 8
= 6 0 0
8 0 0

a) Obtain the Cauchy stress tensor components;


b) Find the principal stresses ( I , II , III ) and principal directions;
c) Obtain the maximum shear stress;
d) Draw the Mohrs circle in stress for the cases: d.1) the Cauchy stress tensor ( ij ), d.2)
the spherical part ( ijsph ) and; d.3) the deviatoric part ( ijdev );
Solution:
ij =

a)

ijsph

ijdev

1 0 0 0 6 8 1 6 8
= 0 1 0 + 6 0 0 = 6 1 0
0 0 1 8 0 0 8 0 1

In Problem 3.19 we have obtained the principal values of ijdev whose values are the same
as for the proposed problem. As the tensor and its deviatoric part have the same principal
directions, i.e. they are coaxial, we can automatically obtain the principal stresses:
ij =

ijsph

ijdev

0 9 0 0
1 0 0 10 0

0 = 0 1 0
= 0 1 0 + 0 0
0 0 1 0 0 10 0 0 11

The principal directions are the same as those provided in Problem 3.19.
d) Mohrs circle in stress
University of Castilla-La Mancha
Ciudad Real - Spain

Draft

By: Eduardo W. V. Chaves (2014)

SOLVING PROBLEMS BY MEANS OF CONTINUUM MECHANICS

290

S
max = 10

+
dev
II = 0

dev
III = 10

dev
= 10
I

dev
N

I = II = III = 1

Deviatoric part

Spherical part

14444444444444442444444444444444
3

S
max = 10

III = 9

II = 1

I = 11

Note that the spherical part contribution deviates (translate) the Mohrs circle of the
deviatoric part according the N -axis, and does not alter the value of the maximum shear
stress.
Problem 3.22
At one point P in the continuum medium, The Cauchy stress tensor is represented by
its Cartesian components as follows:
1 1 0
ij = 1 1 0 MPa ,
0 0 2

a) Obtain the principal stresses and principal directions at the point P ;


b) Obtain the maximum shear stress;
c) Draw the Mohrs circle for the cases: c.1) the Cauchy stress tensor ( ij ), c.2) the
spherical part ( ijsph ) and; c.3) the deviatoric part ( ijdev );
d)

i.) Find the traction vector associated with the plane whose normal vector is
r
n = 1.0e 1 + 1.0e 2 + 0e 3 ;
ii.) Obtain the normal and tangential stress on the plane.

f) Obtain the eigenvalues and eigenvectors of the deviatoric part ( dev ).


University of Castilla-La Mancha
Ciudad Real - Spain

Draft

By: Eduardo W. V. Chaves (2014)

3 STRESS

291

Solution:
a) (See Problem 3.8). The eigenvalues are I = 2 , II = 2 , III = 0
b) and c)
ijdev

= ij

ijsph

1 0 0
1 0 0
2 0 0
4
2

= 0 2 0 0 1 0 = 0 1 0
3
3
0 0 2
0 0 1
0 0 0

max = 1

N
III = 1.333

I , II = 0.667

I = II = III = 1.333

Deviatoric part

Spherical part

14444444444444442444444444444444
3
S

max = 1

III = 0

I , II = 2

d) The traction vector is obtained by t (n) = n , we need to normalize the normal vector

r
n
1
1
e1 +
e 2 + 0e 3 . Thus:
to the plane, i.e. n = r =
n
2
2

t 1(n ) 1 1 0
2
1
(n )
1 1

t 2 = 1 1 0
1 = 2 2
2

(
n
)
t 0 0 2
0
0

University of Castilla-La Mancha


Ciudad Real - Spain

Draft

By: Eduardo W. V. Chaves (2014)

SOLVING PROBLEMS BY MEANS OF CONTINUUM MECHANICS

292

Problem 3.23
The Cauchy stress tensor components at one point are:
0
0
29

ij = 0 26 6 Pa
0
6
9

Decompose the stress tensor in a spherical and a deviatoric part, and obtain the principal
stresses and principal directions of the deviatoric part.
Solution:
Consider the additive decomposition of the stress tensor into a spherical and deviatoric
part:
ij = ijdev + ijsph

The deviatoric part is given by


ijdev

11 m
= 12
13

12
22 m
23

13
23
33

where the mean stress is:


m =

( 29 26 + 9)
1
ii =
=4
3
3

thus:
ijdev

0
0 25
0
0
29 4

= 0
26 4
6 = 0 30 6 Pa
0
6
9 4 0
6
5

The spherical part components are:


ijhyd

ijsph

4 0 0
= 0 4 0 Pa
0 0 4

To verify the above operations, the following relationship must be verified:


ij =

ijdev

ijsph

0
0 4 0 0 29
0
0
25

= 0 30 6 + 0 4 0 = 0 26 6 Pa
0
6
5 0 0 4 0
6
9

To obtain the eigenvalues we solve the characteristic determinant of the deviatoric part:
ijdev ij = 0
3 J 2 J 3 = 0

By solving the above cubic equation we obtain the following principal values:
1dev = 25 Pa
dev
2 = 6 Pa
dev = 31Pa
3

University of Castilla-La Mancha


Ciudad Real - Spain

Draft

By: Eduardo W. V. Chaves (2014)

3 STRESS

293

Problem 3.24
Decompose the Cauchy stress tensor:
12
ij = 21
31

4
9
32

0
2 MPa
3

into a spherical and a deviatoric part.


Obtain the principal invariants of the deviatoric part
Obtain the normal octahedral stress and the mean stress at this point.
Solution:
Due to the symmetry of the Cauchy stress tensor:
0
12 4

ij = 4 9 2 MPa
0 2 3

The mean stress m = oct =

I 12 + 9 + 3 24
=
=
= 8.
3
3
3

The spherical and deviatoric parts are:


ijsph

8 0 0
= 0 8 0
0 0 8

ijdev

= ij

ijsph

0 8 0 0 4 4
0
12 4

9 2 0 8 0 = 4 1 2
= 4
0 2 3 0 0 8 0 2 5

The principal invariants of the deviatoric part are:


I dev J1 = 4 + 1 5 = 0 , as expected, since the trace of any deviatoric tensor is zero.

II dev =

2 5

0 5

or using the definition: J 2 =

4 4
4 1

= 41 = J 2

) (

1 2
1
I 3 II = 24 2 3 151 = 41
3
3

III dev J 3 = det ( dev ) = 44

Problem 3.25
The stress state at one point is represented by the Cauchy stress tensor components:
a b
ij = a c
b c

where a , b and c are constants and is the value of stress. Determine the constants a ,
b and c such that the traction vector is zero on the octahedral plane.
Solution:

University of Castilla-La Mancha


Ciudad Real - Spain

Draft

By: Eduardo W. V. Chaves (2014)

SOLVING PROBLEMS BY MEANS OF CONTINUUM MECHANICS

294

1
A octahedral plane has the following unit vector: n i =
[1 1 1] . The traction vector on
3
r (n )
this plane is defined by t = n , whose components are:

t 1(n ) a b
+ a + b 0 a + b = 1
1
(n )
1 1

a + + c = 0 a + c = 1
1 =
t 2 = a c

3
t (n ) b c 3 1
b + c + 0 b + c = 1

by solving the above system we obtain b =

1
1
1
, c=
, a= .
2
2
2

Problem 3.26
At one point P in the continuous medium the Cauchy stress tensor is represented by its
Cartesian components as follows:
57
ij = 21
31

a)
b)
c)
d)

0
50
32

24
0 MPa ,
43

Obtain the principal stresses and principal directions at the point P ;


Obtain the maximum tangential and normal stress at this point;
Draw the Mohrs circle in stress;
r
Obtain the traction vector t (n) on the octahedral plane of the Haigh-Westergaard
space. Obtain the normal octahedral stress and the tangential octahedral stress.

Solution:
Considering the symmetry of the Cauchy stress tensor we obtain:
57 0 24
ij = 0 50 0 MPa
24 0 43

Note that the stress 22 = 50 is already a principal stress and is associated with the principal
direction n ( 2) = [0 1 0] . To find the other principal stresses we solve the following
system:
57
24
=0
24
43

1 = 25
2 100 + 1875 = 0
3 = 75

Using the definition of eigenvalue-eigenvector, we can obtain the following eigenvectors:


1 = 25

n (1) = [m 0.6 0 0.8]

3 = 75

n (3) = [ 0.8 0 0.6]

Mohrs circle in stress:


Restructuring the principal stresses such that I > II > III we have:
I = 75 , II = 50 , III = 25

University of Castilla-La Mancha


Ciudad Real - Spain

Draft

By: Eduardo W. V. Chaves (2014)

3 STRESS

295

b, c) The Mohrs circle is drawn in figure below:


S
max =

max = 25

II = 50

III = 25

75 25
= 25
2

I = 75 = N max

d) The Haigh-Westergaard space is formed by principal stress directions, then the traction
r
vector in this space is given by t (n) = n , whose normal vector to the octahedral plane is
1

1
:
3

given by n i =

r
t (n) = n

t 1(n)
(n)
t 2 =
t ( n )
3

components

75 0 0
1
75
0 50 0 1 1 = 1 50

3
3
0 0 25
1
25

and its module is given by:


r
t (n)

1
8750

75 2 + 50 2 + 25 2 =
3
3
r
The normal octahedral stress is given by oct = t (n) n :
2

oct

r
t (n) = 54.00617

1
=
[75 50 25] 1 = 50
3 3
1
1

We could have applied directly the definition of octahedral normal stress:


oct =

I
75 + 50 + 25
= m =
= 50
3
3

The tangential octahedral stress can be obtained by means of the Pythagorean theorem:
oct =

r
t (n )

oct

8750
50 2 = 20.4124
3

We could also have applied the definition:


oct =

1
1
2 I 2 6 II =
2 150 2 6 6875 = 20.41241
3
3

where I = 150 , II = 75 50 + 75 25 + 50 25 = 6875 .


e) The spherical part:

University of Castilla-La Mancha


Ciudad Real - Spain

Draft

By: Eduardo W. V. Chaves (2014)

SOLVING PROBLEMS BY MEANS OF CONTINUUM MECHANICS

296

ijsph

50 0 0
Tr ( )
=
ij = m ij = 0 50 0
3
0 0 50

ijsph

57 0 24 50 0 0 7 0 24
= 0 50 0 0 50 0 = 0 0 0
24 0 43 0 0 50 24 0 7

and the deviatoric part:


ijdev

= ij

f) Considering that the tensor and its deviatoric part are coaxial tensors, we can use the
principal space to obtain the deviatoric part:
ijdev

= ij

ijsph

0
75 0 0 50 0 0 25 0

= 0 50 0 0 50 0 = 0 0
0
0 0 25 0 0 50 0 0 25

3.6 Stress state in two dimension (2D)


Problem 3.27
Consider the following stress state:
5

4
2
y

6
x

Figure 3.20:
Obtain the state of plane stress at the point ij .
Solution:
In the state of plane stress ij (i, j = 1,2) we need two planes to define the stress state at the
point:
x
ij =
xy

University of Castilla-La Mancha


Ciudad Real - Spain

Draft

xy
y

(3.54)

By: Eduardo W. V. Chaves (2014)

3 STRESS

297

According to Figure 3.20 we verify that:


5

x = 4
xy = 2

xy = 2
y

y = 6
x

Figure 3.21:
Then:
4 2
ij =

2 6

(3.55)

Problem 3.28
Consider a composite material, which is made up of matrix and fiber along direction of
45 such as shows in Figure 3.22. This composite material can break if the shear stress
along the fiber exceeds the value 3.8 10 6 Pa ( N / m 2 ) .
For the normal stress x = 2.8 10 6 Pa , obtain the maximum value of y for which the
material does not break.
y

45

45

Figure 3.22: Composite material (fiber-matrix).

University of Castilla-La Mancha


Ciudad Real - Spain

Draft

By: Eduardo W. V. Chaves (2014)

SOLVING PROBLEMS BY MEANS OF CONTINUUM MECHANICS

298

Solution:
We need to obtain the traction vector on the plane defined by = 45 , and the tangential
components can directly be obtained by means of:
xy ( ) =
xy ( = 45 )

x y

sin 2 + xy cos 2
2
2.8 10 6 y
=
sin( 90 ) = 3.8 10 6 Pa
2

y 4.8 10 6 Pa (compression)

(see Problem 1.98).


Problem 3.29
The stress acting on two planes passing through the point P are shown in Figure 3.23.
Obtain the value of the shear stress on the plane a a and the principal stresses at this
point.

y
b

80 Pa
45
x

60

60 Pa

a
b

Figure 3.23: Stress state at one point, according to the planes a and b .
Solution:
To obtain the stress state at a point in the two dimensional case, we need to determine x ,
y , xy , as indicated in Figure 3.24.
Considering Figure 3.24, we can directly obtain x and xy by means of the projection of
the traction vector 60 Pa , (see Figure 3.24(b)), i.e.:
x = 60 cos( 30 ) = 51 .962 Pa
xy = 60 cos( 60 ) = 30 Pa

To obtain y we employ the equations:

University of Castilla-La Mancha


Ciudad Real - Spain

Draft

By: Eduardo W. V. Chaves (2014)

3 STRESS

x ( ) N =
xy S ( ) =

x + y
2
x y

299

x y
2

cos 2 + xy sin 2

sin 2 xy cos 2

By substituting the numerical values in the above equations we obtain:


( = 45 ) =
( = 45 ) =

51.962 + y
2
51 .962 y
2

51 .962 y
2

cos( 90 ) + 30 sin( 90 ) = 80 Pa

sin( 90 ) 30 cos( 90 )

The first equation give us the value of y : y = 48.038 Pa .


y

xy
xy

80 Pa
45

xy

80 Pa

x
x

60

45

xy

60
xy
60 Pa

60 Pa
b

a)

b)

Figure 3.24: Stress state at a point, according to the planes a and b .


Once y is determined, we can obtain the component (= 45) :
( = 45 ) = 1.96 Pa

The principal stresses can be obtained by means of the components x , y , xy , such as


indicated in the equations:
(1, 2 ) =

(1, 2 ) =

x + y
2

x y

2

+ 2xy

51.962 + 48 .038
51.962 48.038

2
2

= 80.1Pa
+ 30 2 1
2 = 19.9 Pa

or by means of the characteristic determinant:


x
xy
=0
xy
y

University of Castilla-La Mancha


Ciudad Real - Spain

Draft

51.962
30
=0
30
48.038

By: Eduardo W. V. Chaves (2014)

SOLVING PROBLEMS BY MEANS OF CONTINUUM MECHANICS

300

Problem 3.30
Given a stress state x = 1Pa , xy = 4 Pa and y = 2 Pa . Draw a graph of angle vs.
stresses ( x , y , xy ), where is the rotation angle of the coordinate system (see Figure
3.25).
y

y = 2 Pa

xy = 4 Pa

xy = 4 Pa
x

x = 1Pa

xy
y

Figure 3.25: Stress state at one point.


Solution:
We calculate the values x , y , xy by using the equations:
x =

x + y

xy =
y =

2
x y

2
x + y
2

x y
2

cos 2 + xy sin 2

sin 2 + xy cos 2
y x
2

cos 2 xy sin 2

We can calculate the angle corresponding to the principal direction by means of the
equation:
tan 2 =

2 xy
x y

2 ( 4 )
= 8 ( = 41.437 )
1 2

and the principal stresses:


1, 2 =

x + y
2

x y

2

+ 2xy

1 = 5.5311P

2 = 2.5311Pa

Considering the transformation law, we can obtain the values of x , y , xy for different
values of . Making vary from 0 to 360 we can represent the stresses x , y , xy in
function of the angle, (see Figure 3.26). We can observe that when = 41 .437 we have a
principal direction, then the tangent stress is zero ( xy = 0 ) and the principal stresses are
I = 5.5311Pa and II = 2.5311Pa .

University of Castilla-La Mancha


Ciudad Real - Spain

Draft

By: Eduardo W. V. Chaves (2014)

3 STRESS

301

x
1

= 41.437

x
= 131.437
2

Stresses

1 = 5.5311

xy

x
0
0

50

100

-2

45

xy

-4

150

200

250

350

2 = 2.5311

300

= 86.437

-6

max = 4.0311

Figure 3.26: Stress components in function of the angle .


Problem 3.31
a) Consider the stress field ij (i , j = 1,2) in the Cartesian system x1 x 2 x 3 , and the
following equations:
t
2

m11 = 11 x3 dx3

t
2

m12 = 12 x 3 dx3

t
2

t
2

t
2

m22 = 22 x 3 dx3
t
2

Obtain the component transformation law of mij (i, j = 1,2) to a new system x1 x 2 x 3
which is formed by a rotation around the x3 -axis (see Figure 3.27).
Solution:
Due to the symmetry of ij = ji , we can conclude that m12 = m21 . The transformation
matrix from x1 x 2 x 3 to x1 x 2 x 3 is given as follows:
cos sin 0
a ij = sin cos 0
0
0
1

University of Castilla-La Mancha


Ciudad Real - Spain

Draft

2D

cos sin
A=

sin cos

By: Eduardo W. V. Chaves (2014)

SOLVING PROBLEMS BY MEANS OF CONTINUUM MECHANICS

302

x3 = x3
x2

x3 =

t
2
x2

x3 =

t
2

x1

x1

Figure 3.27.
By using the Voigt notation, we get:

m11

{m } = m22 =
m
12

t
2


11

22 x3 dx3 =

12

t
2

t
2


11

22 x3 dx3 =

12

t
2

t
2

t
2

11
[M] 22 x3dx3 = [M]

12

t
2

11

22 x3 dx3

12

t
2

with that, we can conclude that:



m11
m11

{m } = m 22 = [M]m 22 = [M]{m}
m
m
12
12

(3.56)

where [M] is the transformation matrix for a second-order tensor when it is in the Voigt
notation, (see Problem 1.98), and is given by:
a11 2

[M] = a 21 2
a a
21 11

a12

a 22

a 22 a12

cos 2

2
2a 21 a 22
= sin
a11 a 22 + a12 a 21 sin cos

2a11 a12

sin 2
cos
2

cos sin

2 cos sin

2 sin cos
cos 2 sin 2

Also considering that [M]1 = [N ]T , we obtain {m} = [N ]T {m }, where


a11 2
[N ] = a 212
2a a
21 11

a12

a 22

2a 22 a12

cos 2

2
a 21 a 22
= sin
a11 a 22 + a12 a 21 2 sin cos

a11 a12

sin 2
cos
2

2 cos sin

sin cos
cos 2 sin 2
cos sin

The same result (3.56) could have been obtained by consider mij as a second-order tensor
in two dimensional case (2D), and by means of the transformation law of a second-order
tensor we obtain:
mij = a ik a jl m kl
m
11

m12

(i, j = 1,2)

or

cos sin m11


m12
=
m 22 sin cos m12

University of Castilla-La Mancha


Ciudad Real - Spain

Draft

m = Am A T

m12 cos sin


m 22 sin cos

(3.57)

By: Eduardo W. V. Chaves (2014)

4 The Fundamental
Equations of Continuum
Mechanics
Problem 4.1
Prove that Reynolds transport theorem is valid in the following equation:
D
D
dV =
JdV 0
Dt V
Dt V

(4.1)

where V is the volume in the current configuration, V0 is the volume in the reference
configuration, J is the Jacobian determinant and is a scalar field that describes the
physical quantity of a particle per unit volume at time t .
Solution:
r
r
D
DJ
D
D
D
JdV 0 = J
+
+ J xr v dV 0 =
+ xr v dV (4.2)
dV 0 = J
Dt V
Dt
Dt
Dt
Dt

V0
V0
V
0

Problem 4.2
Show that
r
DPijL ( x , t )
r
D
PijL ( x , t ) dV =
dV
Dt V
Dt
V

(4.3)

where PijL ( x , t ) is a continuum property per unit mass, which can be a scalar, a vector or
higher order tensor.
Solution:
It was proven in the textbook, (see Chaves (2013)), that:
D
r
r
r v p
D
( x , t )dV = ( x , t ) + ( x , t )
dV
x p
Dt V
Dt
V

Then by making = PijL , and by considering it in the above equation we obtain:

SOLVING PROBLEMS BY MEANS OF CONTINUUM MECHANICS

304

D
D
PijK dV = ( PijK ) + PijK
Dt V
Dt
V

D
=
PijK + PijK
Dt
V

v p

dV =
x p
V
D
v

+ k
x k
Dt
1
4243

v
D
D
+ PijK k
PijK + PijK
x k
Dt
Dt

dV

dV

=0
mass continuity equation

Thus, we can conclude that:


DP
D
PijK dV = ijK dV
Dt V
Dt
V

Problem 4.3
Prove that the following relationship is valid:
r

a=

r
r r

( v ) + xr ( v v )
t

(4.4)

Solution: Based on the Reynolds transport theorem:


r
D

dV =
dV + (v n ) dS
Dt V
t
V
S
r
and if we consider that = v we obtain:

r
r
r
r
( v )
D
v dV =
dV + v (v n ) dS
Dt V
t
V
S

Then, the above equation in indicial notation becomes:


( v i )
( v i )
D
D
vi dV =
dV + v i (v k n k ) dS
v i dV =
dV + ( v i v k )n k dS
Dt V

t
Dt

t
123
V
S
V
V
S

= ai

Additionally, by applying the divergence theorem to the surface integral we obtain:

dV =

( v i )
( v i )

+ ( v i v k ) ,k dV
dV + ( vi v k ) ,k dV =
t
t

V
V

which in tensorial notation is:


r
r
r r
( v )
+ xr ( v v ) dV
a dV =
t

V
V

r
r ( v )
r r
a=
+ xr ( v v )
t

Problem 4.4
Let us consider the following velocity field:
vi =

xi
1+ t

for t 0

a) Find the mass density field;


b) Prove that this motion satisfies x1 x 2 x3 = 0 X 1 X 2 X 3 .
Solution: a) By applying the mass continuity equation we obtain:
University of Castilla-La Mancha
Ciudad Real - Spain

Draft

By: Eduardo W. V. Chaves (2014)

4 THE FUNDAMENTAL EQUATIONS OF CONTINUUM MECHANICS

Dt

v k
=0
x k

Dt

305

v
d
= k
dt
x k

and by using the given velocity field, we find that:


v i

1 x i
3
=
= ii =
x i 1 + t x i 1 + t 1 + t

Thus,
d
3
=
dt
1+ t

3dt
1+ t

Then by integrating the both sides of the above equation we obtain:

3dt
1+ t

ln = 3 ln(1 + t ) + C

The constant of integration C is obtained by means of the above equation if we refer to


r
the initial condition t = 0 , in which ( x , t = 0) = 0 , thus
ln 0 = 3 ln(1 + 0) + C C = ln 0
1
ln = 3 ln(1 + t ) + ln 0 = ln
3
(1 + t )

+ ln 0 = ln
(1 + t ) 3

Thus, we can conclude that:

(1 + t )3

b) Then by using the velocity definition we obtain:


vi =

dx i
x
= i
dt 1 + t

dx i
dt
=
xi 1 + t

Additionally, by integrating the both sides of the above equation we obtain:

dx i
dt
=
xi
1+ t

lnx i = ln(1 + t ) + K i

(4.5)

Then by applying the initial condition, i.e. at time t = 0 xi = X i , we obtain:


lnX i = ln(1 + 0) + K i

K i = lnX i

Additionally, by substituting the value of K i into the equation (4.5) we obtain:


lnx i = ln(1 + t ) + ln X i

ln( x i ) = ln[ X i (1 + t ) ]

Hence we can conclude that x i = X i (1 + t ) , which gives us x1 = X 1 (1 + t ) , x 2 = X 2 (1 + t ) ,


0
, we obtain:
x 3 = X 3 (1 + t ) , and if we consider that =
3

(1 + t )

(1
12
+3
t )(1
12
+3
t )(1
12
+3
t)= 0
x1
X1

x2
X2

University of Castilla-La Mancha


Ciudad Real - Spain

x1 x 2 x 3 = 0 X 1 X 2 X 3

x3
X3

Draft

By: Eduardo W. V. Chaves (2014)

SOLVING PROBLEMS BY MEANS OF CONTINUUM MECHANICS

306

Problem 4.5
The equations of motion of a body are given, in Lagrangian description, by:
x1 = X 1 + tX 3

x 2 = X 2 + tX 3
x = X t ( X + X )
3
1
2
3

where is a constant scalar. Find the mass density in the current configuration ( ) in
terms of the mass density of the reference configuration ( 0 ) , i.e. = ( 0 ) .
Solution:
We can apply the equation 0 = J , where J is the Jacobian determinant and is given by:
x1
X 1
x 2
=
X 1
x 3
X 1

x i
J= F =
X j

Thus, we obtain =

0
J

x1
X 2
x 2
X 2
x 3
X 2

x1
X 3
1
x 2
= 0
X 3
t
x 3
X 3

0
1
t

t
t = 1 + 2(t ) 2
1

0
1 + 2(t ) 2

Problem 4.6
Given the velocity field:
v1 = ax1 bx 2
;
where a , b and c are constants.

v 2 = bx1 ax 2

v3 = c x12 + x 22

a) Check if the mass continuity equation is fulfilled;


b) Is the motion isochoric?
Solution:
The mass continuity equation:
r
D
+ ( xr v ) = 0
Dt

where:

r
xr v = v i ,i = v1,1 + v 2, 2 + v 3,3 = a a + 0 = 0
r
The motion is isochoric (incompressible medium), since xr v = 0

Problem 4.7

Consider a continuous medium and one property ( x , t ) assigned by density, i.e. unit of
the property per unit volume. Obtain the rate of change of the property that is described
by a control volume in conjunction with a control surface.
Solution:
Remember that the rate of change of a property is always associated with the same
particles. By means of the material time derivative we can obtain the rate of change of a

University of Castilla-La Mancha


Ciudad Real - Spain

Draft

By: Eduardo W. V. Chaves (2014)

4 THE FUNDAMENTAL EQUATIONS OF CONTINUUM MECHANICS

307

property when this property is in Eulerian description. Then, the total rate of change of
r
( x , t ) in the volume V that is bounded by the surface S is given by:
r
r
r
D
D
D
D

( x, t )dV =
( x, t ) + ( x, t ) (dV )
( dV ) = dV
Dt V
Dt
Dt
Dt

V
V

r
r
r
D

( x, t ) + ( x, t ) xr v dV
= dV
Dt

(4.6)

r
r
r
D
= ( x , t ) + ( x , t ) xr v dV
Dt

We apply the definition of the material time derivative to

r
D
( x , t ) :
Dt

r
r
r
r
D
D
( x, t )dV = ( x, t ) + ( x, t ) xr v dV
Dt V
Dt

V
r
r
r
r
( x , t ) r r

= ( x, t ) +
r v ( x , t ) + ( x , t ) xr v dV
t
x

V
r
r
r
r
( x , t ) r

= ( x , t ) dV +
r v + ( x , t ) xr v dV
x
t

V
V

(4.7)

r
r

= ( x , t ) dV + [ xr (v ) ]dV
t

V
V

We can apply the divergence theorem to the second integral on the right side of the
equation to obtain:
flux of through

suface S
6
44744
8
r
r
r
( x , t )
D
( x, t )dV =
dV + ({
v ) n dS
Dt V
t4
42
3
V 1
S flux of

(4.8)

local

r
( x , t )
the term
is local, the volume integral of the right side of the equation is a control
t
r
volume and the integral surface is a control surface, since the variable (v ) is in Eulerian
r
description. The term (v ) represents the flux of the property .

When there is no source or sink of the property is true that

r
D
( x, t )dV = 0 . And, note
Dt V

also that when the property is the mass density ( = ) the equation (4.7) becomes the mass
continuity equation.
r
r
r
r
D
D
( x, t )dV = ( x, t ) + ( x, t ) xr v
Dt V
Dt
V

r
r

r
dV = 0 = t ( x , t ) + x ( v ) dV = 0 (4.9)

If the above equation is valid for the entire volume then it is valid locally, so
r
r
r
D
( x, t ) + ( x, t ) xr v = 0 Mass continuity equation
Dt

(4.10)

or
University of Castilla-La Mancha
Ciudad Real - Spain

Draft

By: Eduardo W. V. Chaves (2014)

SOLVING PROBLEMS BY MEANS OF CONTINUUM MECHANICS

308

r
r

( x, t ) + xr ( v ) = 0
t

Mass continuity equation

r
(v )

control volume
S
V

r
( x , t )
t

r
x

(4.11)

r
r
q n = [(v ) n ] n

control surface

Figure 4.1: Control volume and control surface.

material volume

control surface

control volume

t=0
v0

XP

X*

control surface

control volume

material volume
t1

r
v( x * , t1 )
xP

x*

material volume

control surface

control volume

t2
r
v( x * , t 2 )
xP

x*

Figure 4.2: Material volume vs. control volume.


University of Castilla-La Mancha
Ciudad Real - Spain

Draft

By: Eduardo W. V. Chaves (2014)

4 THE FUNDAMENTAL EQUATIONS OF CONTINUUM MECHANICS

309

Problem 4.8
Show that
r
r

+ ( xr ) v = ( ) + xr ( v )
t
t

+ , i vi = ( ) + ( vi ) ,i

t
t
r
where is a scalar field, is the mass density field, and v is the velocity field.

(4.12)

Solution:

+
+ , i ( v i ) + ( v i ) , i =
+ , i v i + + ( v i ) , i
( ) + (v i ) ,i =
t
t
t
t
t

=
+ , i v i
t

where we have used the mass continuity equation

+ ( vi ) ,i = 0 .
t

4.1 Equations of Motion. Equilibrium Equations


Problem 4.9
Find the equilibrium equations in engineering notation by means of the differential volume
element equilibrium ( dxdydz ). For this purpose consider that the Cauchy stress tensor field
in the differential volume element varies as indicated in Figure 4.3.
Solution:
To obtain the equilibrium equations we apply the force equilibrium condition in the
volume element. First, we evaluate the equilibrium force according to the x -direction:

=0

xy

x
dx dydz x dydz + xy +
dy dxdz
x
y

xy dxdz + xz + xz dz dxdy xz dxdy = 0


z

b x dxdydz + x +

Then by simplifying the above equation we obtain:

b x dxdydz +
b x +

xy
x

dxdydz +
dxdydz + xz dxdydz = 0
x
y
z

x xy xz
+
+
=0
x
y
z

University of Castilla-La Mancha


Ciudad Real - Spain

Draft

By: Eduardo W. V. Chaves (2014)

SOLVING PROBLEMS BY MEANS OF CONTINUUM MECHANICS

310

Rear face
z
dz
z

z +

Rear face

yz

yz +

xy
y

yz +

bz

xz +

bx

xy +

x + x dx
x

yz
y

y +
xy +

xy
x

dz

dy

by

xz
dx
x

yz

xz

dz

xz
dz
z

xz +

xy

xy
y

y
y

dy

dy

dx

dx

xz

yz

Rear face

dy

Figure 4.3: The stress field in the differential volume element.

The equilibrium force according to the y -direction,

Fy = 0 , can be expressed as follows

yz

dy dxdz y dxdz + yz +
dz dxdy

y
z

xy

yz dxdy + xy +
dx dydz xy dydz = 0
x

b y dxdydz + 22 +

Then by simplifying the above equation we obtain:

b y +

xy
x

y
y

yz
x z

Finally, the equilibrium according to the z -direction,

University of Castilla-La Mancha


Ciudad Real - Spain

Draft

=0

Fz = 0 , is given by:

By: Eduardo W. V. Chaves (2014)

4 THE FUNDAMENTAL EQUATIONS OF CONTINUUM MECHANICS

311

dz dxdy z dxdy + xz + xz dx dzdy


z
x

yz

xz dzdy + yz +
dy dxdz yz dxdz = 0
y

b z dxdydz + z +

Additionally, by simplifying the above equation we obtain:

b z +

xz yz z
+
+
=0
x
y
z

Then, the equilibrium equations in engineering notation become:


x xy xz
+
+
+ b x = 0

y
z
x
xy y yz
+
+
+ b y = 0

y
x z
x

z
xz + yz +
+ b z = 0
z
y
x

Problem 4.10
Let be the Cauchy stress tensor field, which is represented by its components in the
Cartesian basis as:
11 = x12 ;

22 = x 22 ;

12 = 21 = 2 x1 x 2 ;

33 = x12 + x 22
23 = 32 = 31 = 13 = 0

Considering that the body is in equilibrium, find the body forces acting on the continuum.
Solution:

By applying the equilibrium equations, xr + b = 0 , we obtain:

ij , j + b i = 0 i

11 12 13
+
+ b1 = 0
+

x 2
x 3
x1
21 22 23
+
+ b2 = 0
+

x 2
x 3
x1
31 32 33
+ b3 = 0
+
+

x1
x 3
x 2

2 x1 + 2 x1 + b 1 = 0

2 x 2 + 2 x 2 + b 2 = 0
b = 0
3

Thus, to satisfy the equilibrium equations the following condition must be met:
4 x1 = b 1 b 1 = 4 x 1

4 x 2 = b 2 b 2 = 4 x 2

b 3 = 0

b = 4( x1 e 1 + x 2 e 2 )

Problem 4.11
Given the velocity field:
v1 = x1 x3

v 2 = x 22 t

v3 = x 2 x 3t

and the Cauchy stress tensor field:

University of Castilla-La Mancha


Ciudad Real - Spain

Draft

By: Eduardo W. V. Chaves (2014)

SOLVING PROBLEMS BY MEANS OF CONTINUUM MECHANICS

312

x 2 x3
x 22

x 2 x1
ij = x 2 x3
0

0
x 2
x32

x2

where is a constant. Obtain the body force (per unit volume) to guarantee the principle
of conservation of the linear momentum.
Solution:
From the principle of conservation of linear momentum we obtain the equations of
motion:
r
r
r
xr + b = v& = a

b = a xr

The acceleration field:

r r
r r
r v ( x , t ) v ( x , t ) r r
a=
+
r v ( x, t )
t
x

ai =

v i v i
+
vj
t
x j

where
0
v i 2
= x2
t
x 2 x3

x3
v i
= 0
x j
0

0
2x2 t
x3t

x1
0
x 2 t

Then
0 x3
vi vi
ai =
v j = x22 + 0
+
t x j
x2 x3 0

x1 x1 x3 0 x1 x32 + x1 x2 x3t

0 x22t = x22 +
2 x23t

2 2
2
2

x2t x2 x3t x2 x3 x3 x2 t + x2 x3t

0
2 x2t
x3t

x1 x32 + x1 x2 x3t

2
3
=
x2 + 2 x2t

x2 x3 + x3 x22t 2 + x22 x3t 2

The divergence of the Cauchy stress tensor is given by:

ij , j

11 12 13
+
+
= ( x 2 x3 )

x
x
x
1
2
3

= 21 + 22 + 23 = ( 2 x 2 )
x 2
x 3
x1
31 32 33
+
+
= ( 2 x 3 1)

x1
x 2
x 3

ij , j

x 2 x3
= 2 x 2
2 x 3 1

with that the body force (per unit volume) becomes:


r

b = a xr
b i = a i ij , j

University of Castilla-La Mancha


Ciudad Real - Spain

x1 x 32 + x1 x 2 x 3 t
x 2 x3

2
3
x2 + 2x2 t
b i =
2x2
x 2 x 3 + x 3 x 22 t 2 + x 22 x 3 t 2
2 x 3 1

Draft

By: Eduardo W. V. Chaves (2014)

4 THE FUNDAMENTAL EQUATIONS OF CONTINUUM MECHANICS

313

Problem 4.12
The stress field in the medium is represented by:
1
ij = 0
2 x2

2 x2
4 x1
1

0
1
4 x1

(4.13)

where xi are the Cartesian coordinates.


a) Neglecting body forces, is the body in balance?
Solution:
The equilibrium equations:

r r
xr + {
b=0

indicial

r
=0

ij , j = 0i

(4.14)
(4.15)

expanding,
i1,1 + i 2, 2 + i 3,3 = 0 i

11,1 + 12, 2 + 13,3 = 0

21,1 + 22, 2 + 23,3 = 0

31,1 + 32, 2 + 33,3 = 0

(4.16)

where we have used:

11

= 0; 12, 2 = 12 = 0; 13,3 = 13 = 0
x3
x1
x 2
23
21
22
=
= 0; 22, 2 =
= 0; 23,3 =
=0
x 2
x 2
x 3

= 31 = 0; 32, 2 = 32 = 0; 33,3 = 33 = 0
x3
x 2
x 3

11,1 =
21, 2
31,3

(4.17)

Problem 4.13
Given a body in equilibrium in which the Cauchy stress tensor field is represented by its
components:
11 = 6 x13 + x 22

; 12 = x 32

22 = 12 x13 + 60
33 = 18 x 23 + 6 x33

; 23 = x 2
; 31 = x12

Obtain the body force vector (per unit volume) at the point ( x1 = 2; x 2 = 4; x3 = 2 ).
Solution:
The equilibrium equations:
r r
xr + b = 0

University of Castilla-La Mancha


Ciudad Real - Spain

Draft

(4.18)

By: Eduardo W. V. Chaves (2014)

SOLVING PROBLEMS BY MEANS OF CONTINUUM MECHANICS

314

11 12 13

+
+
+ b1 = 0 b1 = 11 12 13

x 2
x3
x1
x 2
x3
x1
21 22 23
21 22 23
+
+ b2 = 0 b2 =

x 2
x3
x1
x 2
x3
x1
31 32 33
31 32 33
x + x + x + b 3 = 0 b 3 = x x x
2
3
1
2
3
1
b1 = 18 x12 0 0

b 2 = 0 0 0
b = 2 x 1 18 x 2
1
2
3

18 x12

0
bi =

2 x 1 18 x 2
1
2

(4.19)

(4.20)

At the point x1 = 2; x 2 = 4; x3 = 2 we obtain:


72
b i = 0
77

(Force per unit volume)

(4.21)

Problem 4.14
The Cauchy stress tensor field is represented by its components as follows:
x12 x 2

ij = k (a 2 x 22 ) x1

1 3
( x 2 3a 2 x 2 )
0
3

0
2ax32
(a 2 x 22 ) x1

(4.22)

where k and a are constants.


r

Obtain the body force field b (per unit mass) required for the stress field is in balance.
Solution:
11 12 13
+
+
+ b1 = 0 b1 = 2 x1 x 2 k + 2 x1 x 2 k = 0

x 2
x3
x1
k
21 22 23
+
+
+ b 2 = 0 b 2 = k (a 2 x 22 ) (3 x 22 3a 2 ) = 0

3
x 2
x 3
x1
31 32 33
x + x + x + b 3 = 0 b 3 = 4kax3
2
3
1

(4.23)

Then:
0
4kax3
0
bi =

1

Problem 4.15

(Force per unit mass)

(4.24)

Let us assume that the body force is b = ge 3 , where g is a constant and consider the
Cauchy stress tensor field components:

University of Castilla-La Mancha


Ciudad Real - Spain

Draft

By: Eduardo W. V. Chaves (2014)

4 THE FUNDAMENTAL EQUATIONS OF CONTINUUM MECHANICS

x2
ij = x3
0

x3
0

0
x 2
p

x2

315

(4.25)

Find p such that satisfies the equilibrium equations. Consider that is a constant and that
the mass density field is homogeneous, i.e. it is independent of the vector position.
Solution:
The equilibrium equations:

r r
xr + b = 0

11 12 13
+
+
+ b1 = 0

x 2
x 3
x1
21 22 23
+
+
+ b2 = 0

x 2
x 3
x1
31 32 33
x + x + x + b 3 = 0
2
3
1

(4.26)

0 + 0 + 0 + b = 0 b = 0
1
1

0
+
0
+
0
+
=
0

b
b

2
2 =0

0 + 33 + b3 = 0

x3

33 (p )
p
= b3
=
=
x 3
x3
x3

g
p
=1 +

x3

dp = 1 +
dx
3

(4.27)

(4.28)

g
g

p = 1 +
x
x 3 p = 1 +

Verification:

+ 1 +
g = + + g g = 0

(4.29)

Problem 4.16
Show that for the following Cauchy stress field:
11 = x 22 + ( x12 x 22 ) ; 12 = 2x1 x 2
;
2
2
2
2
2
22 = x1 + ( x 2 x1 ) ; 33 = ( x1 + x 2 )

23 = 13 = 0

satisfies the equilibrium equations with zero body forces.


Solution:
The equilibrium equations:
ij , j + b i = 0 i
{

ij , j = 0 i

i1,1 + i 2, 2 + i 3,3 = 0 i

=0i

University of Castilla-La Mancha


Ciudad Real - Spain

Draft

By: Eduardo W. V. Chaves (2014)

SOLVING PROBLEMS BY MEANS OF CONTINUUM MECHANICS

316

11 12 13
+
+
=0

x 2
x 3
x1

22 23
21 +
+
=0
x 2
x 3
x1
31 32 33
=0
+
+

x 3
x 2
x1

i = 1 11,1 + 12 , 2 + 13,3 = 0

i = 2 21,1 + 22 , 2 + 23, 3 = 0
i = 3 31,1 + 32 , 2 + 33,3 = 0

thus:
11,1 + 12 , 2 + 31,3 = 2 x1 2x1 = 0

12 ,1 + 22 , 2 + 23, 3 = 2 x 2 + 2x 2 = 0

13,1 + 23, 2 + 33,3 = 0

with that we prove that the body is in balance.


Problem 4.17
Consider a body in equilibrium in which the Cauchy stress field is:
x1 + x 2
r
ij ( x ) = 12
0

12
x1 2 x 2

0
0
x 2

Find 12 , knowing that 12 is a function of x1 and x 2 , i.e. 12 = 12 ( x1 , x 2 ) . It is also


known that the medium is free of body forces and the traction vector associated with the
r
plane x1 = 1 is given by t (n) = (1 + x 2 )e 1 + (5 x 2 )e 2 .
Solution:
As the body is in equilibrium, it must satisfy the equilibrium equations:
ij , j + b i = 0 i
{

ij , j = 0 i

i1,1 + i 2, 2 + i 3,3 = 0 i

=0i

thus
11 12 13

+
= 1 + 12 + 0 = 0
+

x 2
x 2
x 3
x1

22 23 12
+
=
2+0=0
21 +
x 2
x 3
x1
x1
31 32 33
=0+0+0=0
+
+

x 3
x 2
x1
r
Now considering that for the plane x1 = 1 , t (n) = (1 + x 2 )e 1 + (5 x 2 )e 2 holds, we have:

12
1 + x 2

ij ( x1 = 1, x 2 ) = 12 1 2 x 2
0
0

(n )

12
1 + x 2

= ij ( x1 = 1, x 2 )n j = 12 1 2 x 2
0
0

University of Castilla-La Mancha


Ciudad Real - Spain

Draft

0
0
x 2

0 1 1 + x 2
0 0 = 5 x 2
x 2 0 0

(4.30)

By: Eduardo W. V. Chaves (2014)

4 THE FUNDAMENTAL EQUATIONS OF CONTINUUM MECHANICS

317

t (n) = ij ( x1 = 1, x 2 ) n j

1 + x2
12 ( x1 = 1, x 2 ) 0 1
1 + x2
1 + x 2

12 ( x1 = 1, x 2 )
1 2x2
0 0 = 12 ( x1 = 1, x 2 ) = 5 x 2
0

x 2 0
0
0
0

By means of the equilibrium equations:


12
=2
x1

12

= 2x1

12 ( x1 , x 2 ) = 2 x1 + C ( x 2 )

Using the information given in (4.30) we can obtain the constant of integration:
12 ( x1 = 1, x 2 ) = 5 x 2 = 2 + C ( x 2 )

C ( x2 ) = 3 x2

thus:
12 ( x1 , x 2 ) = 2 x1 x 2 + 3

Problem 4.18
The stress state in an continuous medium is given by the Cauchy stress tensor Cartesian
components:
0
ij = Cx 3
0

Cx 3
0
Cx1

0
Cx1
0

where C is a constant. Consider that the body is free of body force.


a) Show whether the body is in balance;
Solution:
a) The continuous medium is in equilibrium if the following equation holds:
r r
+ b = 0 ; ij,j + bi = 0 i (the equilibrium equations)
0
ij = Cx 3
0

Cx 3
0

Cx1

0
Cx1
0

(4.31)
(4.32)

For the proposed problem we have bi = 0i , thus:


ij,j

i = 1 0 + 0 + 0 = 0
i1 i 2 i 3

=
=
+
+
i = 2 0 + 0 + 0 = 0
x j
x1
x 2
x3
i = 3 0 + 0 + 0 = 0

ij

(4.33)

ij,j = 0 i then the body is in equilibrium.

Problem 4.19
Considering the principle of conservation of angular momentum, show that:

[r

( x (a b) (a b) x dV =

University of Castilla-La Mancha


Ciudad Real - Spain

Draft

[ ( xr tr

r
r
t * x dS

By: Eduardo W. V. Chaves (2014)

SOLVING PROBLEMS BY MEANS OF CONTINUUM MECHANICS

318

where

r r
r
r
r r
x - is the vector position; ( x , t ) is the mass density; a ( x , t ) is the acceleration; b( x , t ) is
r r
the body force (per unit mass); t * ( x , t ) is the prescribed traction vector (surface force) on
surface S .

Solution:
The principle of conservation of angular momentum states that:

r
r
r
r
r
r r
r
D
( x t * )dS + ( x b)dV =
( x v )dV = ( x a )dV
Dt V
V
V

Then, we apply the cross product of the above equation with an arbitrary vector z , which
r
is independent of x , and we obtain:
r
r
r
r
z ( x a )dV = z

r
r r
r
r
( x t * )dS + z ( x b)dV

r
r
r
z ( x a )dV =

r
r r
r
r
r
z ( x t * )dS + z ( x b)dV

We have shown in Chapter 1 that given three vectors a , b , c , the relationship


r r
r r r r r
r
a (b c ) = (b c c b) a holds. Then, the above equation can be rewritten as
follows:

r
r r r
r
r
r
r r r
r r
r r
r
( x a a x ) z dV = ( x t * t * x ) z dS + ( x b b x ) z dV

r
r r r r r
r r r r r
r r
r r
( x a a x ) z dV ( x b b x ) z dV = ( x t * t * x ) z dS

r
r
r r
r r
r r
r r
r r
x (a b) (a b) x z dV = ( x t * t * x ) z dS

r
r

r r r
r
r r
r r
r
r
x (a b) (a b) x dV z = ( x t * t * x ) dS z
V

with that, we conclude that:

[r

*
*
x (a b) (a b) x dV = ( x t t x) dS

Problem 4.20
1) Considering the definition of the mean stress tensor ( ):

V = dV
V

and based on the principle that the continuum is in static equilibrium, show that:
=

1
2V

[r

x b + b x dV +

1
2V

r
r r
r
( x t * + t * x ) dS

2) Considering that the volume can be decomposed by V = V (1) V ( 2) , (see Figure 4.4).

University of Castilla-La Mancha


Ciudad Real - Spain

Draft

By: Eduardo W. V. Chaves (2014)

4 THE FUNDAMENTAL EQUATIONS OF CONTINUUM MECHANICS

319

The continuum is subjected to pressure p (1) on surface S (1) , and to pressure p ( 2) on


surface S ( 2) . Considering the continuum is free of body forces, show that:
=

(V

(1)

1
( p (1)V (1) p ( 2)V ( 2 ) )1
( 2)
V )

n (1)
S (1)

p (1)

V (1)
S (2)

V ( 2)

n ( 2 )
p (2)

Figure 4.4
Solution:

Taking into account the equilibrium equations xr + b = a = 0 (the principle of


conservation of linear momentum) for the entire continuum, it must fulfill that:

r
r
r
x xr + b dV = 0

r
r
r
r
x xr dV + x b dV = 0

(4.34)

In Chapter 1 (see Problem 1.127) we have shown that the following holds:
r

r*

( ) x dV = ( n ) x dS dV = t

x ( ) dV = x ( n ) dS

r
x dS dV

(4.35)

r r
dV = x t * dS T dV

(4.36)

where we have considered the prescribed traction vector t * = n . By replacing (4.36) into
the equation (4.34), we obtain:

r
r
r
r
x xr dV + x b dV = 0

r
r
r r
r
x t * dS T dV + x b dV = 0

r
r r
r
dV = x t * dS + x b dV

University of Castilla-La Mancha


Ciudad Real - Spain

(4.37)

Draft

By: Eduardo W. V. Chaves (2014)

SOLVING PROBLEMS BY MEANS OF CONTINUUM MECHANICS

320

Then, the following is true:

r r
r
r
dV = t * x dS + b x dV

(4.38)

r
r r
r
Note that the tensors x t * and x b are not symmetric. This means that the equation

in (4.34) does not take in account the principle of conservation of angular momentum, i.e.
the symmetry of the Cauchy stress tensor. To guarantee the symmetry of we do:
r r
r
1 r r

r
r
+ T
1 r
dV = t * x dS + b x dV + x t * dS + x b dV
2 S
2
2 S

V
V
V

r r r r
1
1 r r* r* r
sym dV =
x b + b x dV +
x t + t x dS
2
2
V
V
S

(4.39)

Considering the definition of the mean stress tensor, we conclude:


r r
r

1 r r
r
r
+ T
1 r
dV = t * x dS + b x dV + x t * dS + x b dV
2
2 S

2 S
V
V
V

r r r r
1
1 r r* r* r
x b + b x dV +
sym dV =
x t + t x dS
2
2
V
V
S

(4.40)

r r r r
1
1 r r* r* r
x b + b x dV +
V =
x t + t x dS
2V
2S

1
2V

[r

x b + b x dV +

1
2V

[ xr tr

r
r
+ t * x dS

In addition, if we consider that the body is free of body force, the above equation becomes:
=

1
2V

[x t

r*

r
r
+ t * x dS

(4.41)

For the particular case shown in Figure 4.4 we have V = V (1) V ( 2) , S = S (1) + S ( 2) ,
r (1)
r ( 2)
t * = p (1) n (1) , t * = p ( 2) n ( 2) . In this case, the equation in (4.41) becomes:
=

2(V

(1)

r r

r
r
r
r r
r
1
x t * + t * x dS (1) + x t * + t * x dS ( 2 )
( 2)
V ) S (1)

S ( 2)

r
r
r
1
(1) r
(1) + n (1) x dS (1) + p ( 2 ) x n ( 2) + n ( 2 ) x dS ( 2 )
x
p

2(V (1) V ( 2 ) ) S (1)

S (2)

r (1) (1) r
r ( 2) (2) r
1
(1)
(1)
( 2)
( 2)
x
x
x
x

p
n
n
dS
p
n
n
dS

2(V (1) V ( 2 ) )

S ( 1)
S ( 2)

We have shown in Chapter 1 that is true

( x n + n x) dS = 2V 1 ,

where n is the

outward unit normal to surface S (see Problem 1.127). For this example, n ( 2) is the

University of Castilla-La Mancha


Ciudad Real - Spain

Draft

By: Eduardo W. V. Chaves (2014)

4 THE FUNDAMENTAL EQUATIONS OF CONTINUUM MECHANICS

inward unit normal to surface S ( 2) , then, we have

[x n
r

( 2)

321

r
+ n ( 2 ) x dS ( 2 ) = 2V ( 2 ) 1 ,

S (2)

with that we obtain:

r
r
r
r
1
p (1) x n (1) + n (1) x dS (1) + p ( 2 ) x n ( 2 ) + n ( 2) x dS ( 2 )
(2)
2(V V )

S (1 )
S (2)

1
1
=
p (1) 2V (1) 1 p ( 2 ) 2V ( 2) 1 = (1)
p (1)V (1) p ( 2)V ( 2 ) 1
(1)
(2)
( 2)
2(V V )
(V V )

(1)

Problem 4.21
r

Starting from u& = : D xr q + r , show that the energy equation can also be written
as follows:
r r
r
r
D
1 2
u + v = xr (v ) + b v xr q + r
Dt
2
D
1 2

u + v = (v j ji ) ,i + b i v i q i ,i + r
Dt
2

(4.42)

or

r r
r
r

1 2
1 2 r
u + v + xr u + v v = xr (v ) + b v xr q + r
t
2
2

1
1
u + v 2 + u + v 2 vi = (v j ji ) ,i + b i vi qi ,i + r
t
2
2 ,i

(4.43)

or

r r
r
r

1 2
1 2 r

u + v + xr u + v v = xr (v ) + b v xr q + r
2
2
t


1
1

u + v 2 + u + v 2 vi = (v j ji ) ,i + b i vi qi ,i + r

2
2 ,i
t

(4.44)

where is the mass density, u is specific internal energy, v is magnitude of the velocity
r

r r

( v 2 = v = v v ), is the Cauchy stress tensor, b is the body force (per unit mass), q is
the flux vector, r is the radiant heat constant (also called the heat source).
2

Solution:
Taking into account the energy equation:
r

u& = : D xr q + r

u& = ij D ij qi ,i + r

where D is the rate-of-deformation tensor which is the symmetric part of the spatial
r
velocity gradient ( D = ( xr v ) sym l sym ). Note also that : D = : l sym = : l since the
double scalar between symmetric ( = T ) and antisymmetric tensor ( l skew ) is zero, i.e.
: l skew = 0 , thus
r
ij D ij = ij ( l ) ij = ij ( xr v ) ij = ij vi , j

Note also that ( ij vi ) , j = ij , j vi + ij vi , j ij vi , j = ( ij vi ) , j ij , j vi = ijD ij , thus the energy


equation becomes:
University of Castilla-La Mancha
Ciudad Real - Spain

Draft

By: Eduardo W. V. Chaves (2014)

SOLVING PROBLEMS BY MEANS OF CONTINUUM MECHANICS

322

u& = : D xr q + r
r
r
r
u& = xr (v ) ( xr ) v xr q + r

u& = ijD ij qi ,i + r
u& = ( ij vi ) , j ij , j vi qi ,i + r

Taking into account the equations of motion we can obtain that:


r
&r& = vr&
xr + b = u
r
r
xr = v& b
r r
r
r r
( xr ) v = v& v b v

&&i = v&i
ij , j + b i = u
ij , j = v&i b i
ij , j vi = v&i vi b i vi

With that the energy equation can also be written as follows:


r

u& = xr (v ) ( xr ) v xr q + r

r r
r
r
r r
u& = xr (v ) ( v& v b v ) xr q + r
r r
r
r r
r
u& + v& v = r (v ) + b v r q + r
x

Note that

u& = ijD ij qi ,i + r
u& = ( ij vi ) , j ( v&i vi b i vi ) qi ,i + r
u& + v&i vi = ( ij vi ) , j + b i vi qi ,i + r

r r
r r
r r
r r
D r r
1 D r r
1 D 2
(v v ) = (v& v ) + (v v& ) = 2(v& v ) (v& v ) =
(v v ) =
(v ) . Thus, the
Dt
2 Dt
2 Dt

energy equation becomes

r r
r
r r
r
u& + v& v = xr (v ) + b v xr q + r
r r
r
r
1 D 2

u& +
(v ) = xr (v ) + b v xr q + r
2 Dt

r r
r
r
D
1 2

u + v = xr (v ) + b v xr q + r
Dt
2

which in indicial notation becomes


u& + v&i vi = ( ij vi ) , j + b i vi qi ,i + r
Du 1 D 2
(v ) = ( ij vi ) , j + b i vi qi ,i + r

+
Dt 2 Dt

D
1 2

u + v = ( ij vi ) , j + b i vi qi ,i + r
Dt
2

showing the equation in (4.42). The equation in (4.43) can easily be obtained if we consider
r
D

+ ( xr ) v , i.e.:
& =
Dt
t
r r
r
r
D
1
u + v 2 = xr (v ) + b v xr q + r
Dt
2
r r
r

r

1
1 r
u + v 2 + xr u + v 2 v = xr (v ) + b v xr q + r
t
2
2

the material time derivative

In Problem 4.21 we showed that

1
2

r
r

+ ( xr ) v = ( ) + xr ( v ) holds, and if we
t
t

consider that = u + v 2 we show the equation in (4.44).

University of Castilla-La Mancha


Ciudad Real - Spain

Draft

By: Eduardo W. V. Chaves (2014)

4 THE FUNDAMENTAL EQUATIONS OF CONTINUUM MECHANICS

323

4.2 Flux Problems


Problem 4.22
1) Consider a continuum motion in which the stress power is equal to zero. Also, consider
r
that the heat flux is given by q = K (T ) xr T , which is known as Fouriers law of thermal
conduction, where K (T ) is a second-order tensor called the thermal conductivity tensor (the
u (T )
, where c is the specific heat capacity at a
T

thermal property of the material), and c =

constant deformation (the thermal property of the material) and is expressed in units of
J
. Taking into account all previous considerations, find the
K
energy equation for this process. Then also provide the unit of K (T ) in the International

joule per kelvin, i.e. [c] =

System of Units (SI).


2) Consider the stress power is equal to zero, and that there is a continuous medium with
no internal heat source. Also consider that there is a heterogeneous material where
r
K = K ( x ) is an arbitrary second-order tensor (not necessarily symmetrical). a) Show that
the thermal conductivity tensor is semi-definite positive, b) Check in which scenario the
r
skew part of K ( x ) does not affect the outcome of the heat conduction problem. c) Taking
into account that the material is isotropic, in what format is K ?
Solution: For this problem we know that the stress power is equal to zero, : D = 0 . It then
follows that, the energy equation becomes:

u& =
c

r
r
u T
=
:3
D xr q + r = xr q + r
1
2
T t
=0

r
T
= xr q + r
t

T
= xr
t

[ K (T ) xr T ] + r

or
xr [K (T ) xr T ] + r = c

T
t

The above equation is called the heat flux equation which is applied to the thermal
conduction problem.
DT T
.
NOTE 1: If there is no mass transport it fulfills T&
=
Dt

Then if we take into account the following units: [q] =


r

J
W
T K
= 2 , xr T r = , we
2
x m
m s m

can ensure that the units are consistent if the following is met:

[qr ]

[K ] [ xr T ]

W
W K
J
J
m 2 s = m 2 = s m K = m K m

W
J
=
.
s m K m K

thus, we can draw the conclusion that [K ] =

University of Castilla-La Mancha


Ciudad Real - Spain

Draft

By: Eduardo W. V. Chaves (2014)

SOLVING PROBLEMS BY MEANS OF CONTINUUM MECHANICS

324

NOTE 2: As we will see later, when the stress power is equal to zero, we can decouple the
thermal and mechanical problem. That is, we can study these problems separately.
2) a) We start from the heat conductivity inequality:
r
r
q xr T = (K ( x ) xr T ) xr T 0
r
xr T K ( x ) xr T 0

or

q i T,i = ( K ij T, j )T,i 0
T,i K ij T, j 0
r

Remember that the arbitrary tensor A is semi-definite positive if it holds that x A x 0


r r
r
for all x 0 thereby demonstrating that K ( x ) is a semi-definite positive tensor. Then, as a
r
result the eigenvalues of K ( x ) are all real values greater than or equal to zero, i.e. K 1 0 ,
r
K 2 0 , K 3 0 . Also remember that since K ( x ) is not symmetric, the principal space of
r
K ( x ) does not define an orthonormal basis. Moreover, it is noteworthy that: the
r
antisymmetric part of K ( x ) does not affect the heat conduction inequality since:

r
xrT K ( x ) xrT = xrT K sym + K skew

] T = T K
r
x

r
x

sym

xrT + xrT K skew xrT 0

xrT K sym xrT + K skew : ( xrT xrT ) 0

Notice that K skew : ( xr T xr T ) = 0 , since the double scalar product between an


antisymmetric tensor ( K skew ) and a symmetric one ( xr T xr T ) is equal to zero, then:
r
0 xr T K ( x ) xr T = xr T K sym xr T 0
r
That is, the above inequality is always true whether K ( x ) is symmetric or not.

b) For the proposed problem the only remaining governing equation is the energy
r
r
Du
equation:
u& = : D xr q + r = xr q , where u is the specific internal
Dt
energy, : D is the stress power, and r is the internal heat source per unit volume. Then:

u& = q i ,i = (K ij T, j ) ,i = K ij ,i T, j + K ij T, ji = ( xr K T ) ( xr T ) + K : xr ( xr T )

= ( xr K T ) ( xr T ) + K sym + K skew : xr ( xr T )

= ( xr K T ) ( xr T ) + K sym : xr ( xr T ) + K skew : xr ( xr T )
= ( xr K T ) ( xr T ) + K sym : xr ( xr T )

where we have considered the symmetry of [ xr ( xr T )]ij = T,ij = T, ji . If the material is


r

homogeneous the implication is that the K field does not depend on ( x ) , so K ij ,i = 0 j . In


this scenario the heat equation reduces to:

u& = K sym : xr ( xr T )
Therefore, when the material is homogeneous, the antisymmetric part of K does not affect
the outcome.
c) The feature of isotropic materials is that their properties (at one material point) do not
change if the coordinate system is changed. It follows then that K must be an isotropic
tensor. An isotropic second-order tensor has the format of a spherical tensor, (see Chapter
1), then the tensor K must be of the type: K = K1 , where K is a scalar:
1 0 0
K ij = K 0 1 0
0 0 1

University of Castilla-La Mancha


Ciudad Real - Spain

Draft

By: Eduardo W. V. Chaves (2014)

4 THE FUNDAMENTAL EQUATIONS OF CONTINUUM MECHANICS

325

Problem 4.23
Consider a thermal conduction problem, (see Problem 4.22), in a wall with thickness equal
to h in which the temperature at the outer face ( x1 = 0 ) is equal to 38 C and the
temperature in the interior face ( x1 = h ) is equal to 21 C , (see Figure 4.5). Obtain the heat
flow for case defined by: stationary problem, the temperature field according to x 2 and x3 directions is homogeneous, there is no heat source, and the material is isotropic and
homogeneous.
x2
T ( A) = 38 C

Data:
h = 0.04m

T ( B ) = 21 C

K = 0.19

W
mK

(Interior)

(Exterior)

x1

r
q
h

Figure 4.5
Solution:
As we saw in Problem 4.22 the governing equation for this problem is the equation

T
T
. If we consider the stationary problem we have
= 0 . If
t
t
there is no heat source this implies that r = 0 . With these simplifications the governing
equation becomes xr [K xr T ] = 0 , in addition, if the material is homogenous, the tensor
r
with the thermal properties K do not vary with x , then xr [K xr T ] = K : xr [ xr T ] = 0 ,
xr [K xr T ] + r = c

which in indicial notation is [K ij T , j ],i = K ij ,i T , j + K ij T , ji = K ij T , ji = 0 . By expanding this


123
=0

equation we obtain:
K 11

2T
2T
2T
2T
2T
2T
+
K
+
K
+
K
+
K
K
+
+
12
13
21
22
23
x 2 x1
x3 x1
x1x 2
x3 x 2
x12
x 22
+ K 31

2T
2T
2T
+ K 32
+ K 33 2 = 0
x1x3
x 2 x3
x3

(4.45)

If the temperature field according to x 2 and x3 -directions is homogenous, this implies that
the temperature gradient components according to these directions are equal to zero, i.e.
T
T
=
= 0 . For an isotropic material, the thermal conductivity tensor components, (see
x 2 x3

Chapter 5 of the textbook), are given by:

University of Castilla-La Mancha


Ciudad Real - Spain

Draft

By: Eduardo W. V. Chaves (2014)

SOLVING PROBLEMS BY MEANS OF CONTINUUM MECHANICS

326

K 0 0
K ij = 0 K 0
0 0 K

With these considerations the equation (4.45) becomes:


K 11

2T
=0
x12

By integrating the equation K


K

2T
=0
x12

11 =K
K

2T
=0
x12

(4.46)

2T
= 0 we obtain:
x12

integrating

T
+ q1 = 0
x1

q1 = K

dT
dx1

which is the Fouriers law of thermal conduction. Note that for this case q1 is a constant, i.e. it is
independent of x1 . By integrating once more we obtain:

dT =

q1
dx1
K

T ( x1 ) =

q1
x1 + C
K

Applying the boundary condition, x1 = 0 T = T ( A) , we obtain the constant of integration


C = T ( A) . With that we obtain T ( x1 ) =
T ( x1 = h) = T ( B ) =

q1
x1 + T ( A) . In addition, for x1 = h we have
K

q1
h + T ( A)
K

q1 = K

(T ( B ) T ( A) )
h

In this case (unidimensional case), the temperature gradient is the slope of the line defined
by the temperature, which varies linearly in the wall, (see Figure 4.5).
By replacing the problem data (see Figure 4.5), we obtain the heat flux:
q1 = K

W
J
(T ( B ) T ( A) )
W (21 38)( K )
= 0.19
= 80.75 2 = 80.75 2

h
m
m s
mK 0.04(m)

Note that the temperature conversion form degrees Celsius to Kelvin is given by
K = C + 273.15 , then the temperature variation ( T ) either in degrees Celsius or in Kelvin
is the same. Note also that the heat flux flows from the higher temperature to the lower
temperature region.

University of Castilla-La Mancha


Ciudad Real - Spain

Draft

By: Eduardo W. V. Chaves (2014)

4 THE FUNDAMENTAL EQUATIONS OF CONTINUUM MECHANICS

327

NOTE: Let us suppose now that we have two walls with different properties as shown in
Figure 4.6.
T ( A)
T (B )
T (C )

K (1)

x1

r
q

K ( 2)
h (2)

h (1)

Figure 4.6
Note that the equation q1 = K (1)

(T ( B ) T ( A) )
is still valid. This also applies to the material
h (1)

(T (C ) T ( B ) )
. To obtain the heat flux we apply the compatibility in
h (2)
temperature on the face B , i.e.:
2 : q1 = K ( 2)

(T ( B _ 1) T ( A) )
h (1)
(T (C ) T ( B _ 2 ) )
q1 = K ( 2)
h ( 2)

q1 = K (1)

q1 h (1)

T ( B _ 1) = T ( A)

T ( B _ 2) = T (C ) +

K (1)
q1 h ( 2)
K ( 2)

T ( B _ 1) = T ( B _ 2 )
T ( A)

q1 h (1)
K (1)

= T (C ) +

q1 h ( 2 )
K ( 2)

thus:
q1 =

(T (C ) T ( A) )
h (1) h ( 2 )

K (1) + K ( 2 )

Problem 4.24
Next, we assume that at a material point there are two types of material that are
represented by a physical quantity per unit volume in such a way that c = c f + c s , and the
r r
r
following holds v = v f + v s , (see Figure 4.7). Considering an isothermal process, an
incompressible medium, and that the property c s does not affect the velocity of the
material f and that the c f -field is homogeneous, and there is no source of the material
f . Show that:
r
c s
Q s xr (v f c s ) + xr (D xr c s ) =
t

University of Castilla-La Mancha


Ciudad Real - Spain

Draft

Convection-diffusion
equation

(4.47)

By: Eduardo W. V. Chaves (2014)

SOLVING PROBLEMS BY MEANS OF CONTINUUM MECHANICS

328

where the flux of the property s is given by q ( D ) = D xr c s .


Control volume

r
vf

cs

r
v

dV

r
vs

Figure 4.7: Heterogeneous medium.


Solution:
Starting from the continuity equation for this physical quantity, we obtain:
Q=

+ xr
t

(v )

Q=

r
r
(c f + c s )
+ r (c f + c s )(v f + v s )
x
t

(4.48)

with Q = Q s + Q f . Thus:

r
r
(c f + c s )
+ r (c f + c s )(v f + v s )
t
x
f
s
r
r
r
(c + c ) f r f
Qs + Q f =
+ r c v + c f v s + csv f + csv s
x
t
f
s
r
r
r
r
c
c
Qs + Q f =
+
+ xr c f v f + c f v s + c s v f + c s v s
t
t
f
c
r c s
r
r
r
+ xr (c s v f ) + xr c f v s + c s v s
Qs + Q f =
+ xr (c f v f ) +
t
t
Qs + Q f =

(4.49)

r
c f
+ xr (c f v f ) = 0 and Q f = 0
t
hold, which is the continuity equation of the physical quantity c f with which the equation

If we assume that there is no ( f )-material source, then


in (4.49) becomes:
Qs =

Qs =

r
r
r
c s
+ xr (c s v f ) + xr c f v s + c s v s
t

r
r
r
c s
+ xr (c s v f ) + xr (c s v s ) + xr (c f v s )
t

(4.50)
(4.51)

r
r
r
r
c s
(4.52)
+ xr (c s v f ) + xr (c s v s ) + xr c f v s + c f xr v s
t
r
If the physical quantity c f does not change with x , then the gradient of c f becomes
r
xr c f = 0 . In addition if we consider the medium ( s ) to be incompressible we obtain
r
xr v s = 0 . These simplifications indicate that the material ( s ) does not affect the velocity
field of the material ( f ). So, if the amount of the material ( s ) is significant, this approach
Qs =

is no longer valid. Then, with these approximations we obtain:

University of Castilla-La Mancha


Ciudad Real - Spain

Draft

By: Eduardo W. V. Chaves (2014)

4 THE FUNDAMENTAL EQUATIONS OF CONTINUUM MECHANICS

329

r
r
r
r
c s
c s
(4.53)
+ xr (c s v f ) + xr q ( D )
+ xr (c s v f ) + xr (c s v s ) =
t
t
r
r
Notice that the term (c s v s ) q ( D ) represents the flux caused by the ( s )-material
r
r
concentration, the diffusive term. The term (c s v f ) q (C ) is related to mass transport, the
r
convective term. Considering that q ( D ) = D xr c s the equation (4.53) becomes:
Qs =

r
r
c s
+ xr (c s v f ) + xr q ( D )
t
r
c s
Qs =
+ xr (c s v f ) + xr (D xr c s )
t
r
c s
Q s xr (c s v f ) + xr (D xr c s ) =
t

Qs =

(4.54)

with that we have demonstrated the equation in (4.47).


Problem 4.25
Consider a water reservoir with
sediment concentration, (see Figure
4.8). The sediment concentration
(concentration density) is given by
( kx3t )

c( x3 , t ) = C t exp
where C and

, per unit volume,


k are positive
constants. a) Obtain the total mass of
sediment in the reservoir; b) Obtain
the sediment flux knowing that the
flux is only a function of x3 and time
r r
t , i.e. q = q( x3 , t ) .

x3

x2

b
x1
a

Figure 4.8: Reservoir with sediments.


Solution:
To obtain the total mass we have to solve the integral:

M = c s dV =
V

h b a

C t exp

( kx3t )

0 0 0

( kx3t )
C
= ab
exp

dx1 dx 2 dx3 = ab C t exp

( kx3t )

dx3

C abC
C
= ab
exp ( kht ) + =
exp ( kht ) 1
k
k
k

To obtain the flux, we can apply the continuity equation of the concentration:

University of Castilla-La Mancha


Ciudad Real - Spain

Draft

By: Eduardo W. V. Chaves (2014)

SOLVING PROBLEMS BY MEANS OF CONTINUUM MECHANICS

330

Q=

r
c s
+ xr q
t

r
c s
xr q = q i ,i =
t

(4.55)

where we have considered that there is no source of the sediment, i.e. Q = 0 . For this
problem, the flux is not dependent on x 2 and x1 . With this condition we have
q1,1 = q 2, 2 = 0 . Then:
qi ,i = q1,1 + q2, 2 + q3,3 =

where

c s
q1 q2 q3
+
+
=
x1 x2 x3
t

q3
c s
=
x3
t

(4.56)

c s
( kx3t )
( kx3t )
( kx3t )
C t exp
= C exp
C t k x 3 exp
and by replacing into the
=
t
t

equation (4.56) we obtain:

dq 3
c s
( kx3t )
( kx3t )
=
= C exp
+ C t k x3 exp
t
dx3

dq 3 =
q3 =

[ C exp

( kx3t )

+ C t k x3 exp

( kx3t )

]dx

C k x3 t
C
C
( kx3t )
( kx3t )
( kx3t )
exp
exp

exp
+ K3
kt
kt
kt

q 3 = C x3 exp

( kx3t )

(4.57)

+ K3
{
=0

The flux vector in the Cartesian basis is given by q = C x3 exp

University of Castilla-La Mancha


Ciudad Real - Spain

Draft

( kx3t )

e 3 .

By: Eduardo W. V. Chaves (2014)

4 THE FUNDAMENTAL EQUATIONS OF CONTINUUM MECHANICS

331

4.3 Rigid Body Motion


Problem 4.26
Find the linear and angular momentum for a solid subjected to rigid body motion.
x3

r
F(n )

r
F( 2 )

Rigid body

Bt

r
x

x3

x1

G - mass center

r
F(1)

x2

x1

x2

r
v

Figure 4.9
Solution:
According to Problem 2.56 in Chapter 2, we obtained the velocity for rigid body motion
as:
r r r
r r
v = c& + ( x c )

where is the axial vector (angular velocity) associated with the antisymmetric tensor W
(the spin tensor).
Linear momentum:

r
r r
r
r
r r
r r
r r
L = v dV = c& + ( x c ) dV = c& dV + x dV c dV

r
r
r
r r
= c& dV + x dV c dV

By definition

x dV = mx

is the first moment of inertia, where m is the total mass, and

r
x k is the vector position of the center of mass G . The first moment of inertia is equal to
r r
r
zero if the Cartesian system originates at the center of mass, so, x dV = mx = 0 .

r
r r
r r
L = m c& + ( x c )
r
= mv

(Linear momentum for rigid body motion)

(4.58)

where v = c& + ( x c ) is the velocity of the center of mass.


Angular momentum:

University of Castilla-La Mancha


Ciudad Real - Spain

Draft

By: Eduardo W. V. Chaves (2014)

SOLVING PROBLEMS BY MEANS OF CONTINUUM MECHANICS

332

r
r
r
H O = ( x v ) dV =

[ xr (cr& + r ( xr cr ))] dV

Thus
r
r r
r
r r
r
r r
H O = x c& dV + x ( x ) dV x ( c ) dV

(4.59)

r
r
r r
r
r r
= x dV c& + x ( x ) dV x dV ( c )
V

Next, we discuss the second integral of the previous equation.

It was proven in Chapter 1 that given three vectors a , b , c , the relationship


r r
r
r r r
r r r
r r
holds,
thus
when
a=c
it
holds
that
a (b c ) = (a c )b (a b)c
r r
r
r r r
r r r
a (b a) = (a a)b (a b)a , so,

r r r

r r r

x ( x) dV = [( x x) ( x ) x ] dV ,

with

which we obtain:

[x

[
= [x

x k i x p p x i dV = x k x k p pi x p p x i dV = x k x k pi x p x i p dV

x k pi x p x i dV p = I O ip p

or in tensorial notation:
r

r r

x ( x ) dV = [( x x ) 1 ( x x )] dV = I O
V

r r

where I O = [( x x ) 1 ( x x )] dV is the inertia tensor with respect to the origin O . As


V

we can observe, I O is a second-order pseudo-tensor, since it depends on the reference

system, and the components I O ij = [x k x k ij xi x j ] dV can be expressed explicitly as:


V

I O 11 = [( x1 x1 + x 2 x 2 + x 3 x 3 ) 11 x1 x1 ] dV = x 22 + x 32 dV

I O 22 = x12 + x 32 dV

I O 33 = x12 + x 22 dV

I O 12 = [( x1 x1 + x 2 x 2 + x 3 x 3 ) 12 x1 x 2 ] dV = [x1 x 2 ] dV = I O 12

I O 13 = [x1 x 3 ] dV = I O 13

I O 23 = [x 2 x 3 ] dV = I O 23

where I O 11 , I O 22 , I O 33 , are moments of inertia of the body relative to the reference point O ,
and I O 12 , I O 13 , I O 23 , are the products of inertia of the body relative to the reference point
O . Thus,

University of Castilla-La Mancha


Ciudad Real - Spain

Draft

By: Eduardo W. V. Chaves (2014)

4 THE FUNDAMENTAL EQUATIONS OF CONTINUUM MECHANICS

2
2
x2 + x3 dV
V
= [x1 x 2 ] dV
V
[x1 x3 ] dV
V

I Oij

[x1 x 2 ] dV

[x1 x3 ] dV
I O 11
V

[x 2 x3 ] dV = I O12
V
I O13
x12 + x22 dV

[ + ] dV
[x x ] dV [

x12

x32

2 3

333

I O 12
I O 22

I O 23

I O13

I O 23
I O 33

(4.60)
Returning to the equation in (4.59) we can state that:

r
r
r
r r
r
r r
H O = x dV c& + x ( x ) dV x dV ( c )
V
V

r r&
r
r r&
r
r r
r r
r
= m x c + I O m x ( c ) = m x c ( c ) + I O
r r r
Then by adding and subtracting the term m x x in the above equation we obtain:
r r r r
r r r
r r
r
r
r
r r
r
H O = m x c& c + I O = m x c& + ( x c ) m x ( x ) + I O
r r
r r
r r r
r r
r r
r r
r
r
= m x v m ( x x) 1 ( x x) + I O = m x v + m ( x x) ( x x) 1 + I O
r r
r
= m x v + I
r r r
= m x v + HG

{[

(4.61)

r r
r r
where I = I O + m[( x x ) ( x x ) 1] is the inertia pseudo-tensor, which is related to the

reference system at the center of mass. By means of this equation we can calculate the
inertia tensor in any reference system if we know the inertia tensor at the center of mass:
IO ij = Iij m[ xi x j ( x12 + x22 + x32 ) ij ] . Explicitly, these components can be expressed as:
IO11 = I11 + m( x22 + x32 ) ; IO12 = I12 m( x1 x2 )
IO 22 = I22 + m( x12 + x32 ) ; IO 23 = I23 m( x2 x3 ) Steiners theorem
IO 33 = I33 + m( x12 + x22 ) ; IO13 = I13 m( x1 x3 )

(4.62)

Note that, the above equations represent the parallel axis theorem (Steiners theorem) from
Classical Mechanics, which in matrix notation is given by:
I O ij

I11

= I12
I
13

I12
I 22
I 23

x 22 + x 32
I13

I 23 + m x1 x 2
x x
I33
1 3

x1 x 2
x12

x 32

x 2 x3

x1 x 3

x 2 x 3 Steiners theorem
x12 + x 22

(4.63)

NOTE: If we have two bodies B (1) and B ( 2) we can conclude that


r
HO =

(x

V (1 )

(1)

r
v ) dV (1) +

(x

( 2) r

v ) dV ( 2 )

V (2)

r
r
r
r
r
r
= m x (1) v (1) + I (1) (1) + m ( 2 ) x ( 2 ) v ( 2 ) + I ( 2 ) ( 2 )
r
r
r
r
r
r
= m (1) x (1) v (1) + m ( 2 ) x ( 2 ) v ( 2 ) + I (1) (1) + I ( 2 ) ( 2 )
r
r
r
= m ( sys ) x ( sys ) v ( sys ) + I ( sys ) ( sys )
(1)

University of Castilla-La Mancha


Ciudad Real - Spain

Draft

By: Eduardo W. V. Chaves (2014)

SOLVING PROBLEMS BY MEANS OF CONTINUUM MECHANICS

334

where (1) and ( 2 ) stand for properties of the bodies B (1) and B ( 2) respectively. If the
r
r
r
r
two bodies are attached they have the same angular velocity (1) = ( 2 ) = ( sys ) = , so, we
can conclude that:
r
HO = m

r
r
r
x ( sys ) v ( sys ) + [ I (1) + I ( 2 ) ]
r
r
r
and if the system Ox is at the center of mass of the system ( v (sys ) = 0 ) we get:
r
r
r
H O = I ( sys ) = [ I (1) + I ( 2 ) ]
( sys )

Problem 4.27
Consider a parallelepiped whose dimensions are a b c (see Figure 4.10), in which the
r
mass density filed, ( x ) , is homogeneous. Obtain the inertia tensor with respect to system
in the center of gravity.
Solution:
We use the equation (4.60):

[x1 x3 ] dV
I O12 I O13
I O11
V
V

2
2
x1 + x3 dV [x2 x3 ] dV = I O12 IO 22 I O 23
IOij
V
V
I O13 I O 23
IO 33
x12 + x22 dV
[x2 x3 ] dV
V
V

r
Note that, for this problem, the mass density is independent of x (homogeneous material),

2
2
x2 + x3 dV
V
= [x1 x2 ] dV
V
[x1 x3 ] dV
V

[x1 x2 ] dV

and moreover it fulfills that:

m = dV =
V

dV = V = abc

Then, the moment of inertia I O11 becomes:

I O11 = x22 + x32 dV =

c
2

b
2

a
2

[x

2
2

+ x32 dx1dx 2 dx3 =

c b a
2
2
2

Similarly, we can obtain I O 22 =

m 2
(a + c 2 )
12

abc 2
m
(b + c 2 ) = (b 2 + c 2 )
12
12

I O 33 =

m 2
(a + b 2 ) .
12

We leave to the reader show that I O12 = I O 13 = I O 23 = 0 . Recall that the inertia tensor give
us information on how the mass is distributed according to the adopted system, and note
that the mass is equally distributed according to the plane x1 x2 , thus [x1 x 2 ] dV = 0 .
V

Note also that the adopted axes are principal axes of inertia:

I Oij

m 2
2
0
0

12 (b + c )

m 2

=
0
(a + c 2 )
0
12

m 2

2
0
0
(a + b )

12

University of Castilla-La Mancha


Ciudad Real - Spain

Draft

By: Eduardo W. V. Chaves (2014)

4 THE FUNDAMENTAL EQUATIONS OF CONTINUUM MECHANICS

335

x3

x2

x1

Figure 4.10: Parallelepiped.


NOTE: A list of inertia tensor for several solids can be found in Wikipedia
http://en.wikipedia.org/wiki/List_of_moments_of_inertia
Problem 4.28
Consider three thin rods of length a and mass m , (see Figure 4.11). Obtain the inertia
r
tensor for this system at Ox .
x2

r
r
x (1) = 0
r
a
a
x ( 2 ) = e 1 + e 2
2
2
r ( 3) a
a
e 1 + e 3
x =
2
2

rod 2

a
2

x2

a
2

a
2

x1
x2

r
x ( 2)

a
2

O
r
x ( 3)

a
2

x1

rod 1

x1

a
2

x3
x3

rod 3

Figure 4.11: System compound by three rods.

University of Castilla-La Mancha


Ciudad Real - Spain

Draft

By: Eduardo W. V. Chaves (2014)

SOLVING PROBLEMS BY MEANS OF CONTINUUM MECHANICS

336

Data: The inertia tensor for the thin rod is given by:
x2
a
2

a
2

I gij

g
x1

0 0 0
ma 2
=
0 1 0
12
0 0 1

x3

Solution:
We

use

)
(1)
( 2)
( 3)
r
r
r
r
I O( sys
x = I Ox + I Ox + I Ox

IO ij = Iij m xi x j ( x12 + x22 + x32 ) ij

I O ij

I11

= I12
I
13

I12
I 22
I 23

(see

Problem

4.26

(NOTA

1)),

where

(the Steiners theorem):

x 22 + x 32
I13

I 23 + m x1 x 2
x x
I33
1 3

x1 x 2
x12

x 32

x 2 x3

x1 x 3

x 2 x3
x12 + x 22

(4.64)

Rod 1 - I O(1X)r
Mass center vector position: ( x1(1) = 0, x2(1) = 0, x3(1) = 0)
(1)
r)
(I Ox
ij

0 0 0
ma 2
=
0 1 0
12
0 0 1

Rod 2 - I O( 2Xr)
a
2

Mass center vector position: ( x1( 2 ) = , x 2( 2 ) =

a ( 2)
, x 3 = 0)
2

By apply the equation (4.64) we get:


I11

( 2)
(I Oxr ) ij = I12
I
13

I12
I 22
I 23

x 22 + x 32
I13

I 23 + m x1 x 2
x1 x 3
I 33

a 2
2
+ 0
2

1 0 0
ma 2
+ m a a
0
0
0
=

12
2 2
0 0 1

University of Castilla-La Mancha


Ciudad Real - Spain

x1 x 2
x12 + x 32
x2 x3

x1 x 3

x 2 x3
x12 + x 22

a a

2 2
2
a
2
+0
2
0

Draft

4 3 0
ma 2

0
=
3 3 0
12

0
0 7
2
2
a a
+
2 2
0

By: Eduardo W. V. Chaves (2014)

4 THE FUNDAMENTAL EQUATIONS OF CONTINUUM MECHANICS

337

Rod 3 - I O(3X)r
Mass center vector position: ( x1( 2 ) =
I11

( 2)
(I Oxr ) ij = I12
I
13

I12
I 22
I 23

a ( 2)
a
, x 2 = 0, x 3( 2 ) = )
2
2

x 22 + x 32
I13

I 23 + m x1 x 2
x1 x 3
I 33

x1 x 3

x 2 x3
x12 + x 22

x1 x 2
x12 + x 32
x2 x3

2 a 2
0
(0 ) +
2

1 0 0
2
2
ma 2
a a
+ m
+
0
1
0
0
=

12
2 2

0 0 0
a a
0


2 2

a a


2 2
4 0 3
ma 2

0
=
0 7 0
12

3 0 3
2

a
2
+0

Then, we can calculate


(1)r
( 2r)
( 3r)
r )
(I O( sys
x ) ij = (I Ox ) ij + (I Ox ) ij + (I Ox ) ij

0 0 0
4 3 0
4 0 3
8 3 3
ma 2
ma 2
ma 2
ma 2

=
0 1 0 +
3 3 0 +
0 7 0 =
3 11 0

12
12
12
12
3 0 3
0 7
0 11
0 0 1
0
3

Problem 4.29
Obtain the principle of conservation of linear momentum and angular momentum for a
solid subjected to rigid body motion.
Solution: We can start from the definition of the principle of conservation of linear
momentum which states that:
r

r&

F = Dt v dV = L
V

Then we use the equation of linear momentum obtained in Problem 4.26, L = m v , to


obtain:
r

r&

r&

F = Dt v dV = L = m v = m a
V

Then we have:
r

F = m a

Now let us consider the principle of conservation of angular momentum which states:
r

r&
r
r
D
D r
HO HO
( x v )dV =
Dt V
Dt

By which we obtain:
r

M
University of Castilla-La Mancha
Ciudad Real - Spain

r&
= HO

Draft

or

r&
= HG

By: Eduardo W. V. Chaves (2014)

SOLVING PROBLEMS BY MEANS OF CONTINUUM MECHANICS

338

where the equation of angular momentum H O was obtained in Problem 4.26. The set of
equations
equivalent:

F = m a

and

r&
= H G inform us that the following systems are

r&
HG

r
F( 2 )

r
F(n )
G

r
ma

r
F(1)

G - center of mass

Figure 4.12
NOTE: If we are dealing with rigid body motion, the governing equations are:
r

F =ma

and

r&
= HO

Governing equations for rigid body


motion

(4.65)

Problem 4.30
Consider the beam with the following load and boundary conditions:
P

L
2

HA

L
2

VA

VB

Figure 4.13: Isostatic beam.


Obtain the support reactions VA , VB , H A .
Solution:
Although in the beam there is deformation (small deformation regime) and stress, for
purposes of support reaction calculation of an isostatic beam we can consider as a rigid
body case and the necessary equations (see equations (4.65)) are:

r
r r
F=ma =0

F
F
F

University of Castilla-La Mancha


Ciudad Real - Spain

=0

=0

=0

F
F

Draft

= H A + P cos = 0 H A = P cos

= V A + V B P sin = 0 V A = V B + P sin

By: Eduardo W. V. Chaves (2014)

4 THE FUNDAMENTAL EQUATIONS OF CONTINUUM MECHANICS

r
r
r&
MA = HA =0

M
M
M

=0

=0

=0

with which we can obtain V A = V B + P sin =

= V B L P sin

339

L
P sin
= 0 VB =
2
2

P sin
. Note that we have 3 equations and
2

3 unknowns (isostatic system). If we have a system in which there are more unknowns than
equations (hyperstatic system), this procedure is no longer valid since the reactions will
depend on the beam deformation and this depends on the beam stiffness.
Problem 4.31
Find the kinetic energy related to rigid body motion in terms of the inertia tensor, (see
Problem 4.26 and Problem 4.29).
r

Solution: The rigid body motion velocity can be expressed as v = c& + ( x c ) . Then, the
kinetic energy becomes:
r r
1
1
(v v )dV =
2V
2V

][

r& r
r r r r
r r
c + ( x c ) c& + ( x c ) dV

r
r r r
Using the following vector sum x = x + x , where x is the mass center vector position,
r
and x is the particle vector position with respect to the system that has its origin in the

K (t ) =

center of mass, the energy equation becomes:

[(

][

r r
r r
r r
r r r
r
1
c& + (( x + x ) c ) c& + (( x + x ) c) dV
2V

r r
r r
r r
r r
r r
r r
1
=
c& + ( x c ) + ( x ) c& + ( x c) + ( x ) dV
2V

r r r
r r
Note that v = c& + ( x c) is the center of mass velocity, thus:

K(t ) =

] [(

K (t ) =

2V

{ [vr + (r xr )] [vr + (r xr )] }dV

or:
K (t ) =

r r
r r r
r r r
r r
r r
1
1
1
1
v v dV +
v ( x ) dV +
( x ) v dV +
( x ) ( x ) dV
2V
2V
2V
2V

Then by simplifying the above equation we obtain:


K (t ) =

r r
r r r
r r
r r
1
1
v v dV + v ( x ) dV + ( x ) ( x ) dV
2V
2V
V

Next, we discuss separately the terms of the previous equation:


1)

r r
1
1 r
v v dV = v
2V
2

dV = 2 mv

r r

2) v ( x ) dV = v x dV = v ( m {
x ) = 0
V

University of Castilla-La Mancha


Ciudad Real - Spain

r
=0

Draft

By: Eduardo W. V. Chaves (2014)

SOLVING PROBLEMS BY MEANS OF CONTINUUM MECHANICS

340

Note that, the system x is located at the center of mass ( G ), hence the center of mass
r
vector position related to the system x is zero.
r

3) ( x ) ( x ) dV
V

[( x) ( x)] dV =

ijk j xk ipq p xq

dV = ( jp kq jq kp ) j xk p xq dV

= j ( jp kq xk xq jq kp xk xq ) p dV
V

= j ( jp xk xk xp xj ) p dV = j ( jp xk xk xj xp ) dV p

V
V

= j I jp p

or in tensorial notation as:


r

r r

[( x) ( x)] dV = [( x x) 1 ( x x)] dV = I
V

where I is the inertia pseudo-tensor related to the system located at the center of mass,
(see Problem 4.26).
Then if we bear in mind all the above considerations, the kinetic energy equation for rigid
body motion becomes:
K (t ) =

r r
r r r
r r
r r
1
1
1
v v dV + 2 v ( x ) dV + ( x ) ( x ) dV
2V
2
2V
1V44424443

=0

1
2

1r
2

Kinetic energy for rigid


body motion

K(t ) = mv 2 + I

(4.66)

Additionally, if we take into account that:

2
2
x 2 + x3 dV
V
Iij = [x1 x 2 ] dV
V
[x1 x3 ] dV
V

[x1 x 2 ] dV

[x1 x3 ] dV
I11
V

[x 2 x 3 ] dV = I12
V
I13
x1 2 + x 2 2 dV

[x + x ] dV
[x x ] dV
[
1

2 3

I13

I 23
I 33

I12

I 22
I 23

we obtain an explicit equation for the kinetic energy as:


1
1
1
1
K(t ) = mv 2 + k Ikj j = mv 2 + [1 2
2
2
2
2
=

1
2

I11

3 ] I12
I
13

I12

I22
I23

I13 1

I23 2
I33 3

1 2 1
mv + I1112 + I2222 + I3332 2 I1212 2 I1313 2 I2323
2
2

K(t ) = mv 2 +

1
I1112 + I 22 22 + I 33 32 2 I12 1 2 2 I13 13 2 I 23 2 3
2

University of Castilla-La Mancha


Ciudad Real - Spain

Draft

(4.67)

By: Eduardo W. V. Chaves (2014)

4 THE FUNDAMENTAL EQUATIONS OF CONTINUUM MECHANICS

341

Problem 4.32
Consider the inertia pseudo-tensor, I O , with respect to the system x1 x 2 x3 , (see Figure
4.14). a) Make the physical interpretation of the inertia tensor. b) Given another
orthonormal system, represented by x1* x 2* x3* . Obtain the inertia tensor components in this
new system. c) Show that the inertia tensor is positive definite tensor. For a solid in
DI O &
I O is equal to zero.
Dt

motion, find in which situation the term

x3

x2*

x3*

x1*
O

x2

x1

Figure 4.14
Solution:
The inertia pseudo-tensor depends on the adopted coordinate system, and by definition is
given by:
r r
r r
I O = [( x x ) 1 ( x x )] dV

I O ij = x k x k ij xi x j dV

or in components

2
2
x 2 + x3 dV
V
I ij = [x1 x 2 ] dV
V
[x1 x3 ] dV
V

[x1 x 2 ] dV

[x1 x 3 ] dV

V
[x 2 x3 ] dV
V

x12 + x 22 dV

[x + x ]dV
[x x ] dV [
2
1

2
3

2 3

a) The inertia tensor give us the information as the mass into the body is distributed under
the adopted system.
The term [x1 x 2 ] dV indicates how the mass is distributed along the plane x1 x 2 . Then,
V

if the material is homogeneous, i.e. the mass density field is independent of x , and x1 x 2
is a plane of symmetry, i.e. the mass is distributed equally with respect to plane x1 x 2 , the
term

[x x ] dV
1 2

is equal to zero. With this, we conclude that: if the planes x1 x 2 ,

x1 x 3 , x 2 x3 , are planes of symmetry, the inertia matrix is a diagonal matrix.

Let us consider a student attached to a disc with outstretched arms, each hand holding a
r
weight (see Figure 4.15 initial system). The disk rotates with angular velocity (i ) and the
University of Castilla-La Mancha
Ciudad Real - Spain

Draft

By: Eduardo W. V. Chaves (2014)

SOLVING PROBLEMS BY MEANS OF CONTINUUM MECHANICS

342

inertia tensor according to the system x is given by I O(i ) . If we consider a system without
energy dissipation, what will happen when the student moves the arms inwardly as shown
in Figure 4.15 final system? As we are dealing with a conservative system, the angular
momentum is conserved too, i.e.:
r
r
H O( i ) = H O( f )
r
r
I (Oi ) ( i ) = I O( f ) ( f )

Since for the final system the mass is more concentrated according to the rotation axis than
r
r
to the initial system the inequality I O( f ) < I (Oi ) holds and as consequence ( f ) > (i ) .
x3

Initial system

x3
r
(i )

Final system
r
( f )

I O( f )

I O(i )

Figure 4.15:
b) Let us assume that the given systems (see Figure 4.14) are related by the transformation
law xi* = Aij x j , where Aij is the orthogonal matrix, then it follows that xi = A ji x *j . Thus
being able to express I O ij as follows:

I O ij = x k x k ij xi x j dV = ( x k* x k* )Aip pq A jq Aip x *p A jq x q* dV
V

= Aip ( x k* x k* ) pq x *p x q* A jq dV = Aip ( x k* x k* ) pq x *p x q* dV A jq
V
V

{[

]}

= Aip I *O ij A jq

Note that x k x k = A ks x s* A kt x t* = x *s x t* A ks A kt = x *s x t* st = x *s x s* = x t* x t* = x k* x k* .

Abusing a little bit of notation, we also use tensorial notation, but bear in mind that we are
working with tensor components, and we are not doing an orthogonal transformation.

University of Castilla-La Mancha


Ciudad Real - Spain

Draft

By: Eduardo W. V. Chaves (2014)

4 THE FUNDAMENTAL EQUATIONS OF CONTINUUM MECHANICS

343

r r
r r
r r
r
r
IO = [( x x ) 1 ( x x )] dV = ( x * x * )A T 1 A (A T x * A T x * ) dV

r r
r
r
= ( x * x * )A T 1 A (A T x * x * A ) dV
V

{[

r r
r
r
= A T ( x * x * )1 ( x * x * )

]} A dV

r r
r
r
= A T ( x * x * )1 ( x * x * ) dV A = A T I*O A
V

I O = A T I *O A

I O ij = A

*
ip I O ij

A jq

Inertia tensor components after a base


change (rotation)

(4.68)

Then, it is also true I *O = A I O A T , which are the inertia tensor components in the
system x1* x 2* x3* . Note that the equation (4.68) is the same component transformation law
for a second-order tensor, where A is the transformation matrix from the x1 x 2 x3 -system
to x1* x 2* x3* -system.
c) For a positive definite tensor, by definition, its eigenvalues are greater than zero.
We will start from the kinetic energy obtained in Problem 4.31, i.e.:
1
2

K(t ) = mv 2 +

1
I1112 + I 22 22 + I 33 32 2 I12 1 2 2 I13 13 2 I 23 2 3
2

The kinetic energy is a scalar and always a positive number, and only in two situations the
kinetic energy is zero, namely: when there is no mass or when the body is at rest. We adopt
a system such that the origin is at the center of mass and the adopted axes are axes of
symmetry (inertia principal system) and that the body is rotating around the origin (center
of mass). In this situation the kinetic energy becomes:
1
K(t ) = [1 2
2

0 1
I1 0
1

3 ] 0 I2 0 2 = I112 + I222 + I332 > 0


2
0
0 I 3
1
4424433

Eigenvalues of the
Inertia tensor

1
2

In addition, if we have a motion such that 2 = 3 = 0 , we have K(t ) = I112 , then, the

only way that the kinetic energy is always positive is when I1 > 0 holds. Similarly, we can
conclude that I 2 > 0 and I 3 > 0 . Hence, the inertia tensor is a positive definite tensor.

d) As the inertia pseudo-tensor is dependent on the adopted system, for the following
situations the inertia tensor to a solid in motion does not change with time:
1) If the adopted system is attached to the solid.
2) If the solid is rotating along a axe of symmetry, for example if a cylinder is rotating along
the prismatic axe, then during motion the mass distribution is not changing with respect to
the adopted system, (see Figure 4.16).

University of Castilla-La Mancha


Ciudad Real - Spain

Draft

By: Eduardo W. V. Chaves (2014)

SOLVING PROBLEMS BY MEANS OF CONTINUUM MECHANICS

344

reference system fixed in space

Figure 4.16
Problem 4.33
Consider a homogeneous cylinder of radius r and height h = 3r with total mass equal to
m , (see Figure 4.17). Find the inertia tensor in the system Ox1 x 2 x3 . The system Ox1 x 2 x3 is
given by the rotation of the system Ox1x 2 x3 of 45 along the axis x1 . The systems
Gx1 x 2 x3 and Ox1x 2 x3 have the same orientation.
Hint: For the reference system Gx1 x 2 x3 we know the inertia tensor components and are
given by:

I G ij

1
2
2
0
12 m(3r + h )

1
0
=
m(3r 2 + h 2 )
12

0
0

2 0 0
mr 2

0 =
0 2 0

0 0 1
1 2
mr

2
0

x3
r

x3
x3

x1

x2

r
rG

h = 3r

x2
45
x2

x1, x1

Figure 4.17

University of Castilla-La Mancha


Ciudad Real - Spain

Draft

By: Eduardo W. V. Chaves (2014)

4 THE FUNDAMENTAL EQUATIONS OF CONTINUUM MECHANICS

345

Solution:
First of all we obtain the inertia tensor in the system Ox1x 2 x3 by means of the Steiner
theorem, (see equation (4.62) in Problem 4.26). After that, we obtain the components due
to a rotation by means of the equation (4.68) in Problem 4.32.
By means of the equations in (4.63):
I11

I O ij = I12
I
13

I12
I 22
I 23

x 22 + x32
I13

I 23 + m x1 x 2
x x
I33
1 3

x1 x3

x2 x3
x12 + x 22

x1 x 2
x12

x32

x2 x3

(4.69)

where ( x1 , x 2 , x 3 ) are the coordinates of the center of mass with respect to the system
r
3
Ox1x 2 x3 , and by consider the vector rG = x1e 1 + x 2 e 2 + x 3 e 3 = 0e 1 + re 2 + re 3 , we can
2

obtain:
2
3 2
r + ( 2 r )

2 0 0

mr 2

I O ij =
0 2 0 + m
0

2
0 0 1

( r )( 3 r )

2

( r )( 2 r )

0
34 0
mr 2

=
0
0 13 6

4
0 6 6

02 + r 2

0
3 2
2
0 + ( 2 r )

Considering the transformation matrix between the systems Ox1x 2 x3 and Ox1 x 2 x3 :
0
0
1

A = 0 cos 45 sin 45
0 sin 45 cos 45

and applying the equation (4.68) we obtain:


I O ij = A I O A = Aip I O ij A jq
T

Problem 4.34

0
34 0
mr 2
=
0
7 7
8
0 7 31

Taking into account the angular momentum H O = m x v + I = m x v + H G , find the


rate of change of the angular momentum in such a way that we do not need to calculate at
each instant of time the inertia tensor.

University of Castilla-La Mancha


Ciudad Real - Spain

Draft

By: Eduardo W. V. Chaves (2014)

SOLVING PROBLEMS BY MEANS OF CONTINUUM MECHANICS

346

r
- angular velocity of the body

r
HG

r
- angular velocity of the system x *

x3

x2*

x3*

r
HO

x1*

x3
G

r
x

x1

x2

x 2

G - center of mass

x1

Figure 4.18
Solution: Applying the material time derivative we obtain:

r
r
r r r
r r
r Dv
r&
r&
D r
D
D
D r
Dx r
m x v + HG =
m xv +
HO HO =
HG = m
v +m x
+ HG
Dt
Dt
Dt
Dt
Dt
Dt
r r
r r r&
= m v12
3
v + m x a + HG
r

[ ]

=0

Then, we obtain:
r r r&
r&
D r
HO HO = m x a + HG
Dt

(4.70)
r

where a is the acceleration of the center of mass. Next, we discuss the term H& G . We
adopt the mobile system x1 x 2 x3 but with fixed orientation in space which is parallel to the
r
fixed system x1 x 2 x3 , (see Figure 4.18). By expressing the components of I and in the
system x1 x 2 x3 , we obtain:
r
r
H G = I

r&
r&
D r
& r

H G H G = I + I
Dt
r
Note that, as the solid rotates with respect to the system x the inertia tensor changes,
r
since the mass distribution is changing with respect to the system x . Then, at each time
rate of change

step we have to calculate the inertia tensor. This procedure is very laborious. To solve this
r
problem, we adopt a new system x * , which has origin at the center of mass, (see Figure
4.18). By means of the component transformation law, the following is true:
(components)

r
r
r
r
H G* = A H G
H G = A T H G*
;
r *
r
r
r
= A T *
= A ;
*
T

;
I O = A T I O* A
I O = A I O A

where A is the transformation matrix from the x -system to x * -system.


University of Castilla-La Mancha
Ciudad Real - Spain

Draft

By: Eduardo W. V. Chaves (2014)

4 THE FUNDAMENTAL EQUATIONS OF CONTINUUM MECHANICS

347

The rate of change of H G = A T H G* becomes:

r&
r
r
r&
D r
D
A T H G* = A& T H G* + A T H G*
H G H G =
Dt
Dt

(4.71)

By analogy with the rate of change of the orthogonal tensor, (see Chapter of the textbook),
we can conclude that = A& A T A& T = A T T , where T is the antisymmetric
r
tensor and represents the rate of change of rotation of the system x * with respect to the
r
system x . Then, we can express (4.71) as follows:
r&
r
r&
r
r&
H G = A T T H G* + A T H G* = A T T H G* + H G* (components)
(4.72)

r
r r
Resorting to the antisymmetric tensor property such that T H G* = H G* (see NOTA

r
r r
3), where is the axial vector associated with the antisymmetric tensor T , i.e. = (t )
r
is the angular velocity of the rotating system x * . Proving that (4.72) can still be written as
follows:
r&
r
r&
r
r&
r
H G = A T T H G* + H G* = A T * H G* + H G* (components)

(4.73)

where
r*
r&
D * r * DI * r *
H G* =
I =
+ I * D
Dt
Dt
Dt

The term
1)

DI *
is equal to zero when one of the two possibilities holds:
Dt

r r
r
DI *
= 0 if the system x * is attached to the solid. In this case, the equation =
Dt

holds, i.e. the mobile system velocity is equal to the angular velocity of the solid.
2)

DI *
= 0 if the solid rotates around a prismatic axis, (see Figure 4.16 in Problem 4.32).
Dt

NOTE 1: The equation in (4.73) can be rewritten as follows:


r&
r
r&
r
H G = A T * H G* + H G*

(components)
(4.74)
r&
r*
r& *
r * r * r& *
T r*

A H G = A A H G + H G = H G + H G

r
r
r
Note that the term A H& G are the components of H& G in the system x * , and note also
r
r
that A H& G H& G* , then:

r * r
r
A H& = H& * + r * H * (components)
G
G
G

(4.75)

we can also express the above equation in tensorial notation:


r
DH G

Dt

r
DH G

r r

=
Dt + H G

r
f

University of Castilla-La Mancha


Ciudad Real - Spain

Draft

(tensorial notation)

(4.76)

By: Eduardo W. V. Chaves (2014)

SOLVING PROBLEMS BY MEANS OF CONTINUUM MECHANICS

348

r
r
DH G
represents the rate of change of H G with respect to the fixed system,
where

Dt f
r
r
DH G

Dt represents the rate of change of H G with respect to the rotating system with an
r

r
angular velocity .

NOTE 2: The equation in (4.76) is valid for any vector (see Figure 4.19), i.e. the rate of
r
r
change of the vector b respect to the fixed system x is equal to the rate of change of the
r
r
vector b respect to the rotating system x * plus the vector product between angular
r
velocity of the system ( which is associated to the antisimetric tensor T ) and the vector
r
b:

r
r
r
r Db
Db
Db
r r
T

=
+ b =
+ b
Dt

fixed Dt rotating
Dt rotating

(4.77)

r r D
D
D
.
=
=
+
1
2r3 Dt
Dt f Dt r
r

Note also that

=0

r
b

x3

x2*

x3*

x1*
x 2
x1

Figure 4.19
NOTE 3: Note that the equation (4.77) is the convective rate, (see Chapter on The
C
r

Objectivity of Tensors in the textbook), which is defined by a = a& + l

a , where

C
r r
r r
r
= D + W , then a = a& + l T a = a& + (D + W) T a . Recall from Chapter 2 (Chaves (2013))
1
that W = R U& U 1 U 1 U& R T + R& R T holds. And if we are considering rigid solid
2
C
r r
r
motion we have D = 0 , U& = 0 , and W = = R& R T , with that we obtain a = a& + T a .

NOTE 4: Let us expose a simple example to obtain T . Let us assume that the e i -system
is rotating according to the e i -system (see Figure 4.20), and to obtain T we procedure as
follows. The transformation matrix from e i to e i is given by:

University of Castilla-La Mancha


Ciudad Real - Spain

Draft

By: Eduardo W. V. Chaves (2014)

4 THE FUNDAMENTAL EQUATIONS OF CONTINUUM MECHANICS

cos
A = sin
0

d (cos )

dt
d (sin )
d (A )
&
A =
dt
dt

d (sin )
dt
d (cos )
dt
0

sin
T
&
&
= A A = cos
0

cos

0 &

T = & 0
0 0

sin
0

0 0

0 = 3
0 2

3
0

sin
cos
0

0
0
1

0
&
sin
0 = & cos

0 0

0 cos
0 sin
0 0

2
1

349

sin
cos
0

(4.78)

& cos 0 sin

& sin 0 = & cos


0

cos
sin
0

0
0
0

0
0 1 0 0 & 0

0 = & 1 0 0 = & 0 0
0 0 0 0 0 0
1

0
i = 0
&

r
where is the axial vector associated with the antisymmetric tensor T .

3 = &

e 3
e 1

e 2

e 3

e 2
e 1

Figure 4.20

University of Castilla-La Mancha


Ciudad Real - Spain

Draft

By: Eduardo W. V. Chaves (2014)

SOLVING PROBLEMS BY MEANS OF CONTINUUM MECHANICS

350

NOTE 5: Inertial forces


Let us consider the system OX 1 X 2 X 3 (see Figure 4.21) which is fixed in space. This system
is denoted by inertial reference frame. To this system the Newtons law is applied, and if there
is a falling body it is true that:
r
r
F = mA

Let us consider also that an observer (attached to the system ox1 x 2 x3 ) is moving (for
simplicitys sake we will just consider translation). Since the system ox1 x 2 x3 is moving we
denote it by non-inertial reference frame. By means of vector summation, (see Figure 4.21), we
can obtain:
r r r
X =c+ x

The material time derivative of the above equation becomes:


r& r r
X = c& + x&

D
Dt

&r& &r& &r&


+x
X =c

r &r& r
+ &x&
A=c

and if we multiply by mass ( m ) we get:


r
&r& + m&xr&
mA = mc

r
r
&r& = Fr mc
&r&
m&x& = mA mc

r r
&r&
ma = F mc

Note that, for the observer it appears the additional force (m&c&) to the Newtons law.
This additional force is a fictitious force or pseudo force which is denoted by inertial force. In
addition, inertial forces appear if the observers system is rotating, e.g. centripetal force.

r
mA

X2

r
x

r
X

x2

r
c

x1

X1
X3

Figure 4.21

University of Castilla-La Mancha


Ciudad Real - Spain

Draft

By: Eduardo W. V. Chaves (2014)

4 THE FUNDAMENTAL EQUATIONS OF CONTINUUM MECHANICS

Problem 4.35

351

Show that the acceleration at a fixed system a f can be expressed as:


r
r
r r
r
r r
a f = ar + 2( v r ) + ( x )
r

(4.79)

where a r and v r are, respectively, the acceleration and the velocity of a particle with
r
respect to an observer that is rotating with the system x * , (see Figure 4.19). Consider also
r r
r
that = is the angular velocity of the system x * , which is constant with time.
Solution:
We use directly the equation in (4.77) to obtain the velocity:
r
r
r r
Dx
Dx
=

+ x
Dt f Dt r

r
r r r
v f = vr + x

We apply the same definition to the above equation in order to obtain the acceleration, i.e.:
r
r r r
r r r
Dv f
r
r r r
D[v r + x ]
D[v r + x ]
=
+ [v r + x ]

Dt
Dt
f
r
Dt f
r
r r
r
r r r
r r
Dv D[ x ]
+ v r + ( x )
af = r +

Dt r Dt r
r
r
r
r
r r Dx
r r r
r r
Dv D

+ v r + ( x )
af = r +
x

Dt r Dt r
Dt r
r
r
r& r r r r r r
r r
a = a + x + v + v + ( x )
f

r
r
r& r
r r
r
r r
x + 2( vr ) + ( x )
a f = ar +
r r
As we are assuming angular velocity constant & = 0 , i.e. the angular acceleration is zero,

with that we obtain the equation in (4.79). Then, we can conclude:


r
r r& r
r r
r
r r
a f = ar +
x + 2( vr ) + ( x )

(4.80)

Note that to obtain the above equation we have not used any principle of conservation.
The above equation is just relating the acceleration in a fixed system in function of
parameters defined in the rotating system.
r

r r r

r r r

NOTE 1: Using the identity a (b c ) = (a c )b (a b )c , (see Problem 1.17), we can


r

r r r

r r r

r r r

conclude that ( x ) = ( x ) ( ) x = ( x ) x . Note that, if = 3e 3 ,


(see Figure 4.22), and also if we adopt the system ( e r , e , e 3 ) and taking into account that
2

r r
r
r r
r r r
r 2r
r 2r
r = 0 we obtain the following equation ( r ) = ( r ) r = r , which is

the

centripetal

acceleration,

(see

Problem

2.58).

Earth

rotates

at

rate

r
r r
2 rad
rad
rad
. Note that the term ( x ) is very small
=
0.727 10 4
day 86400 s
s
r r
compared with the term 2( vr ) .

3 = 2

University of Castilla-La Mancha


Ciudad Real - Spain

Draft

By: Eduardo W. V. Chaves (2014)

SOLVING PROBLEMS BY MEANS OF CONTINUUM MECHANICS

352

x3 , z

r
= 3e 3

e 3

r
r

x3

r
x

e r

x2 , y

x1 , x

Figure 4.22
r

NOTE 2: The term 2( v r ) , which was established by Gustave-Gaspard Coriolis in


1835, is associated with the fictitious force called Coriolis force. Next, we represent
r r
2( v r ) in the system e i :
3

N
e 2

Latitude

e 3
e 1

r
x

e 3 , e 2

e 2 , e 1

Equator

e 1 , e 3

S
Figure 4.23
Then, the transformation law from e i to e i is given by:
e 1 0
1
0 e 1


e2 = sin 0 cos e 2
e cos 0 sin e
3
3

University of Castilla-La Mancha


Ciudad Real - Spain

Draft

1
0
0

B = sin 0 cos
cos 0 sin

(4.81)

By: Eduardo W. V. Chaves (2014)

4 THE FUNDAMENTAL EQUATIONS OF CONTINUUM MECHANICS

3e 3

353

r
= 3 cos( )e 2 + 3 sin( )e 3

N
e 2

e 3

S
Figure 4.24
r

The term 2( v r ) can be obtained as follows:


e 1
e 2
e 3
r r
2( vr ) = 0 3 cos( ) 3 sin( )
vr1
vr 2
vr 3
= 2e 1 [3 cos( )vr 3 3 sin( )vr 2 ] 2e 2 [ 3 sin( )vr1 ] + 2e 3 [ 3 cos( )vr1 ]
= 2[3 cos( )vr 3 3 sin( )vr 2 ]e 1 + 2[3 sin( )vr1 ]e 2 2[3 cos( )vr1 ]e 3

(4.82)

The term f = 23 sin( ) is known as Coriolis parameter. To small value of vr 3 the above
equation reduce to:
r
r r
Dv r
= 2( v r ) = [23 sin( )vr 2 ]e 1 + [ 23 sin( )vr1 ]e 2 = [ f vr 2 ]e 1 + [ f vr1 ]e 2

Dt r
Dv r 1
Dt = f vr 2

Dv r 2 = f v
r1
Dt

Figure 4.25: Coriolis effect (Ref.: Wikipedia Coriolis effect).

University of Castilla-La Mancha


Ciudad Real - Spain

Draft

By: Eduardo W. V. Chaves (2014)

SOLVING PROBLEMS BY MEANS OF CONTINUUM MECHANICS

354

NOTE 3: Deflection of vertically falling body


A very simple application of the Coriolis effect is presented next. Let us consider an
observer on the surface of the Earth. Let us consider also that a body of mass m is freefalling from rest with the following initial conditions: at t = 0 . ( x3 = h) , ( x1 = 0) ,
d
x3 = v3 = 0) , (v1 = 0) , (v2 = 0) . As the body is falling we will calculate the deflection of
dt
the body, i.e. we will obtain x1 related to the observer which is attached to a system which
(

is rotating with the Earth. We will adopt the system used in Figure 4.23.
r

The Newtons Second Law ( F = ma f ) (apply to an inertial reference frame), then


r
r
r r
r
r r
F = m[ar + 2( v r ) + ( x )]

r
r r
ar = ge 2( vr )

r
r
r r
r r
ma r = F 2m( v r ) = mge 3 2m( v r )

2[3 cos( )vr 3 3 sin( )vr 2 ] 23 cos( )vr 3


r

2[3 sin( )vr1 ]


(a r ) i =
0
=

v
g
g

[
]

2
cos(
)
3
r1

where the acceleration a f is given by (4.80), and we are considering that the term
r
r r
r r
( x ) is very small when compared with the term 2( vr ) whose components are
given by (4.82). Then
d 2 x1
2
ar1 dt2 23 cos( )vr 3
r

d x
(ar )i = ar1 = 22 =
0

a dt2

r1 d x3
2
dt

(4.83)

Note that
d 2 x3
dx
= g integratin
g 3 = gt + C1

vr 3 = gt
2
dt
dt
dx
t2
t2
3 = gt integratin
g x3 = g + C2

x3 = g + h
dt
2
2

where we have considered the initial conditions, i.e. at t = 0 (vr 3 = 0)


t = 0 ( x3 = h)

C2 = h . Note that x3 = g

C1 = 0 , and

t
gt
.
+h=0h=
2
2

Considering the acceleration vr 3 = gt into the first component of (4.83) we can obtain:
d 2 x'1
dx '1
t2
integratin g
=

v
=

gt

+ C1 = vr1
2
cos(
)
2
cos(
)
2
cos(
)
3
r3
3
3
2
dt
dt 2

where the constant of integration is obtained with the initial condition


at (t = 0) {vr1 = 0

C1 = 0

dx '1
t3
= vr1 = 3 g cos( )t 2 integratin
g x'1 = 3 g cos( ) + C2
dt
3
1
3

Note also that C2 = 0 , with that we obtain: x'1 = 3 g cos( )t 3

University of Castilla-La Mancha


Ciudad Real - Spain

Draft

By: Eduardo W. V. Chaves (2014)

4 THE FUNDAMENTAL EQUATIONS OF CONTINUUM MECHANICS

As the body is falling from height h we can state that h =

355

1 2
gt t =
2

2h
, with that the
g

above equation becomes:


3

g 2h 2
1
x'1 = 3 g cos( )t 3 = 3 cos( )
3
3 g

NOTE 4: Acceleration due to sphericity


Local system e 1 (east)- e 2 (north)- e 3 (radially upward)

e 2

r
r

Latitude

W
e 3
e 1

r
x

e 3

e 1

r
r
r = x cos

e 3
e 2

Equator

N North
S South
E East
W West

e 1

S Pole
r d
r
d
= x cos
( e1 )
v r1 = r
dt
dt

e 3
vr 2

NP

e 2

r
r
r
x

e 3

e 2

NP

SP
vr 2

r
r

v r1

e 1

r d
= x
( e 2 )
dt

e 1

Figure 4.26

University of Castilla-La Mancha


Ciudad Real - Spain

Draft

By: Eduardo W. V. Chaves (2014)

SOLVING PROBLEMS BY MEANS OF CONTINUUM MECHANICS

356

Previously we have obtained the transformation matrix from e i to e i , (see equation


(4.78)):
cos
A = sin
0

sin
cos
0

0
0
1

(4.84)

and the transformation matrix from e i to e i (see equation (4.81)) is given by:
1
0
0

B = sin 0 cos
cos 0 sin

(4.85)

Then the transformation matrix from e i to e i is given by:


1
0 cos
0

C = BA = sin 0 cos sin


cos 0 sin 0

sin
cos
0

0 sin
0 = sin cos
1 cos cos

cos
sin sin
cos sin

0
cos
sin

The rate of change of C is given by:

& cos
& sin
0

d (C ) & &
C = ( cos cos + & sin sin ) ( & cos sin & sin cos ) & sin
dt
( & sin cos & cos sin ) ( & sin sin + & cos cos ) & cos

After the algebraic operation = C&C T takes place we obtain:

0
& sin & cos


v
& = r r1 sin
0
= C&C T = & sin
& cos
x cos
0
&


v r1
xr cos cos

0
vr1 tan vr1

1
0
= r vr1 tan
vr 2
x
vr1
0
vr 2

v r1
sin
r
x cos
0
vr 2
r
x

v
r r1 cos
x cos

vr 2
r

which is an antisymmetric matrix, as expected. Notice that according to Figure 4.26 the
v
d
d v r 2
= r r1
= r hold.
and &
following relationships &
dt

x cos

dt

We apply the definition (see equation (4.76))


r
Dv

Dt

r r
Dv
=
+ vr

f Dt r

r r
r
Note also that T v r = v r holds, so:

University of Castilla-La Mancha


Ciudad Real - Spain

Draft

By: Eduardo W. V. Chaves (2014)

4 THE FUNDAMENTAL EQUATIONS OF CONTINUUM MECHANICS

vr1 tan

0
r
1
v r = r vr1 tan
x
vr1
T

0
vr 2

vr1vr 2 tan + vr1vr 3


r
r

1 2
a f = a r + r vr1 tan + vr 2 vr 3
x

vr21 vr22

vr1vr 2 tan + vr1vr 3


v r1 v r1

1 2

vr 2 vr 2 = r vr1 tan + vr 2 vr 3
x

vr21 vr22
0 vr 3

vr1vr 2 tan + vr1vr 3


r
r

1 2
a r = a f r vr1 tan + vr 2 vr 3
x

vr21 vr22

357

(4.86)

NOTE 5: Coriolis + Curvature acceleration


The acceleration related to the Coriolis terms (see Eq. (4.82) and (4.80)) and curvature is
given by:
r
r
r r
r r
r r
a f = a r + 2( v r ) + T vr + ( x )

(4.87)

vr1vr 2 tan( ) + vr1vr 3


2[3 vr 3 cos( ) 3 vr 2 sin( )]
r r

1 2
T r
2( v r ) + v r =
2[3 vr1 sin( )]
+ r vr1 tan( ) + vr 2 vr 3

2[3 vr1 cos( ) ]


vr21 vr22

(4.88)

where

Problem 4.36
Consider the rigid body in motion in which there are no forces acting on the body and also
consider a torque-free motion. a) Show the Eulers equations of motion:
& 1 = 2 3 (I 2 I3 )
I1

& 2 = 13 (I3 I1 )
I 2
I
3 & 3 = 12 (I1 I 2 )

Eulers equations of motion

(4.89)

where Ii are the principal moment of inertia related to the system G xyz whose origin is at
r
the center of mass G , i are the components of the body angular velocity ( ), and
&i

Di
denotes the time derivative of the angular velocity.
Dt

b) Show that the kinetic energy is constant.


Solution:
The governing equations for a rigid body motion (see Problem 4.29) are:

r
r
F = ma

and

r
r&
MG = HG

If the body is free of forces and torque we have that:


r

F =0

and

r r&
= 0 = HG

r
Next we will evaluate the term H& G .

We will consider a mobile system Gxyz attached to the body, (see Figure 4.27), so, in this
r r
situation we have that = .
University of Castilla-La Mancha
Ciudad Real - Spain

Draft

By: Eduardo W. V. Chaves (2014)

SOLVING PROBLEMS BY MEANS OF CONTINUUM MECHANICS

358

r
- angular velocity of the body

r
HG

r
- angular velocity of the system x

x3

x1 , x2 , x3 - principal axes of inertia

=
x2

x3

x1
G

x1

x 2

G - center of mass

Figure 4.27
r
In Problem 4.34 we have obtained an efficient equation in order to calculate H& G , (see
r r
equation (4.76)), and by considering = we get:

r
DH G

Dt

r
r
r

DH G
DH G
r
r r
=
+ H G =
+
H Gxyz

Dt
Dt
f
r

For this problem we have:


I 1
(I Gxyz ) ij = 0
0

0
I2
0

0
0
I 3

1
r

( ) i = 2

3

The angular momentum:


r
r
H Gxyz = I Gxyz
r
( H Gxyz ) 1 I 1
r

( Hr Gxyz ) 2 = 0
( H

Gxyz ) 3 0

components

0
I2
0

r
r
( H Gxyz ) i = (I Gxyz ) ij ( ) j

0 1 I 1 1

0 2 = I 2 2
I 3 3 I 3 3

The rate of change of the angular momentum:


Note that, since the system G xyz is attached to the body the mass distribution respect to
this system does not change during motion, so, I Gxyz does not change as well, i.e.
I& Gxyz = 0 . With that we have:

r
DH G

Dt

r
DH Gxyz
=
Dt

r&

H Gxyz

Gxyz

University of Castilla-La Mancha


Ciudad Real - Spain

components

Draft

r
( H&
) &
& 1 I1
&1
r Gxyz 1 I11 + I1

&

&
& 2 = I 2
& 2
( H Gxyz ) 2 = I 2 2 + I 2
I& + I
r&
& I
&
( H Gxyz ) 3 3 3 3 3 3 3

By: Eduardo W. V. Chaves (2014)

4 THE FUNDAMENTAL EQUATIONS OF CONTINUUM MECHANICS

e 1
r r
H Gxyz = 1
I 1 1

e 2
2

e 3
3

I 2 2

I 3 3

359

= ( 2 I 3 3 3 I 2 2 )e 1 + ( 3 I 1 1 1 I 3 3 )e 2 + (1 I 2 2 2 I 1 1 )e 3
= 2 3 (I 3 I 2 )e 1 + 1 3 (I 1 I 3 )e 2 + 1 2 (I 2 I 1 )e 3

Components:

r r
H Gxyz

2 3 (I 3 I 2 )

= 1 3 (I 1 I 3 )
(I I )
1 2 2 1

With that we can calculate


r
DH G

Dt

r
r

r r
= H& Gxyz +
H Gxyz = 0

whose components are:

{ }
r&
H Gxyz

r r
+ H Gxyz

} {}
i

r
= 0

& 1 2 3 (I 3 I 2 ) 0
I1



& 2 + 1 3 (I 1 I 3 ) = 0
I 2
I


3 & 3 1 2 (I 2 I 1 ) 0

& 1 = 2 3 (I 2 I 3 )
I 1

& 2 = 1 3 (I 3 I1 )
I 2
I
3 & 3 = 1 2 (I 1 I 2 )

b) The kinetic energy for rigid body motion (see Problem 4.31 equation (4.66)) is given by:
1
2

1r
2

K(t ) = mv 2 + I
Since the origin of the adopted system is at G (mass center) we have v = 0 , with that we
obtain:
1
1
K(t ) = k I kj j = [1
2
2

I1
3 ] 0
0

0
I2
0

0 1
1
0 2 = I1 12 + I 2 22 + I 3 32
2
I 3 3

And the rate of change of the kinetic energy becomes:

] [

1 D
1
D
& 1 + 2 2 I 2
& 2 + 23I 3
&3
K(t ) = K& (t ) =
I112 + I 2 22 + I 332 = 21 I1
2 Dt
2
Dt
& 1 + 2I 2
& 2 + 3I 3
&3
= 1 I1

If we consider the Eulers equation (4.89) the above equation becomes:


K& (t ) = 1 I1& 1 + 2I 2 & 2 + 3I3& 3 = 1 23 (I 2 I3 ) + 213 (I3 I1 ) + 312 (I1 I 2 )
= 1 2 3 (I 2 I3 + I3 I1 + I1 I 2 )
=0

with that we show that the kinetic energy is constant for any problem which is governed by
Eulers equations of motion.

University of Castilla-La Mancha


Ciudad Real - Spain

Draft

By: Eduardo W. V. Chaves (2014)

SOLVING PROBLEMS BY MEANS OF CONTINUUM MECHANICS

360

Problem 4.37
Obtain a simplified form of the rigid body governing equations for the particular case:
a) Rigid body rotation around a fixed axis without forces.
Solution:
We will consider the fixed system OX 1 X 2 X 3 and we adopt the rotation axis the X 3 -axis,
(see Figure 4.28) and the mobile system Ox1 x2 x3 attached to the body.

x1

X 3 , x3

X1

= = 3 e 3 = 3 E 3

3
r
- angular velocity of the body

x2

- angular velocity of the system x

X2

system OX 1 X 2 X 3 => orthonormal base (E 1 , E 2 , E 3 )


system Ox1 x2 x3 => orthonormal base (e 1 , e 2 , e 3 )
Figure 4.28
If the body is free of forces the governing equations becomes:
r

F =0

and

r&
= HO

r
where H& O can be calculated by means of

r
r&
DH O
HO
Dt

DH Oxr

=
r Dt
OX

r
r

DH Oxr
r
+ H Oxr =
r
Dt
Ox

r r
+
H Oxr
r
Ox

The angular momentum:


r
r
r
r
components
H Oxr = I Oxr

( H Oxr ) i = (I Oxr ) ij ( ) j
r
( H Oxr ) 1 I O 11
I O 12 I O 13 0 I O 13 3
r r


I O 22
I O 23 0 = I O 23 3
( Hr Ox ) 2 = I O 12
( H r ) I
I O 33 3 I O 33 3
I O 23
O 13
Ox 3

And its rate of change:


r
( H& r ) I
&
O13 3
r& Ox 1

& 3
( H Oxr ) 2 = I O 23
r
( H& r ) I
&
Ox 3 O 33 3

University of Castilla-La Mancha


Ciudad Real - Spain

Draft

By: Eduardo W. V. Chaves (2014)

4 THE FUNDAMENTAL EQUATIONS OF CONTINUUM MECHANICS

361

And we need to calculate the vector H Oxr :


r r
H Oxr =

e 1
0

e 2
0

I O 13 3

I O 23 3

e 3
3 = I O 23 32 e 1 I O 13 32 e 2
I O 33 3

thus

Applying

&3
& 3 I O 2332 I O 2332 IO13
I O13
r&


& 3
& 3 + I O1332 = I O1332 I O 23
( H O ) i = I O 23
I

&3
0
IO 33
O 33 & 3

r
r&
M O = H O we get the following set of equations:

M
M
M

O1
O2
O3

M
M
M

X
Y
Z

&3
= I O 2332 IO13
2
&3
= IO133 I O 23
&3
= I O 33

where & = stands for angular acceleration.


NOTE: If the body is prismatic and if we adopt the prismatic axis the same as the rotating
axis the above equations reduce to:

M
M
M

O1
O2
O3

M
M
M

=0

=0

&3
= I O 3

since the system Ox1 x2 x3 is principal axes of inertia, e.g.


z
x

I O 1

I Gxyz = 0
0
IO1 = I O 2

0
IO 2
0

0
I O 3

Problem 4.38
A rigid body consists of two masses m at each extremity of the weightless rod whose
length is 2l . The rod is inclined about respect to the vertical line and rotates with
angular velocity as indicated in Figure 4.29.
a) Find the angular momentum
of the body;
r
b) Find the torque ( M ) in order to maintain the rotation.

University of Castilla-La Mancha


Ciudad Real - Spain

Draft

By: Eduardo W. V. Chaves (2014)

SOLVING PROBLEMS BY MEANS OF CONTINUUM MECHANICS

362

l
m

Figure 4.29
Solution: We apply the governing equations for a rigid solid motion, (see Problem 4.29). We
will adopt a fixed system OXYZ at the space and a mobile system Oxyz which is attached
to the body, (see Figure 4.30.
Y

r
= J

Zz

l
m
x

Figure 4.30
The inertia tensor I (system Oxyz ) is given by:
IOxyz

0
0

= 0 2ml 2
0
0

0
0
2ml 2

The angular velocity (system Oxyz ):


r
= cos()i + sin()j + 0k

University of Castilla-La Mancha


Ciudad Real - Spain

Draft

By: Eduardo W. V. Chaves (2014)

4 THE FUNDAMENTAL EQUATIONS OF CONTINUUM MECHANICS

363

where is the module of .


r

The angular momentum H O :


r
r
HO = I
0
0 cos()
0
H Ox 0

2
2
H Oy = 0 2ml
0 sin() = 2ml sin()
H Oz 0

0
2ml 2
0
0
r
H O = 0i + 2ml 2 sin() j + 0k
r
The torque M can be evaluated as follows:
r r&
r&
r r
M = H O = ( H O ) Oxyz + H O

r
r&
r r
We can observe that (H O ) Oxyz = 0 and = hold, then:

i
j

k
r r&

r r
M = H O = H O = cos()
sin()
0

0
2ml 2 sin() 0

r r&
2
2
2
M = H O = cos()2ml sin()k = ml sin(2)k
Y

r
= cos()i + sin()j + 0k
r
H O = 0i + 2ml 2 sin() j + 0k

r
HO

X
Zz

l
m
x

Figure 4.31
Solution using the system OXYZ
The transformation matrix from OXYZ to Oxyz is given by:

University of Castilla-La Mancha


Ciudad Real - Spain

Draft

By: Eduardo W. V. Chaves (2014)

SOLVING PROBLEMS BY MEANS OF CONTINUUM MECHANICS

364

cos 2 sin 2 0

sin () cos() 0

A = sin cos 0 = cos() sin () 0

2

0

0
1
0
0
1

The inertia tensor for the system OXYZ is:


I OXYZ = A T I Oxyz A
I OXYZ

0
sin () cos() 0 0

= cos( ) sin ( ) 0 0 2ml 2


0
0
1 0
0

0 sin () cos() 0
0 cos() sin () 0
2ml 2 0
0
1

2ml 2 cos 2 ( )
2ml 2 sin ( ) cos()
0

2
2
2
= 2ml sin ( ) cos()
2ml sin ()
0

0
0
2ml 2

The angular momentum becomes:


r
r
H OXYZ = A T H Oxyz
0
H Ox

H = 2ml 2 sin()
Oy

H Oz

0
2
0
H OX sin () cos() 0
2ml cos() sin()

H OY = cos() sin () 0 2ml 2 sin() = 2ml 2 sin 2 ()

H OZ 0

0
1
0
0

The torque:

M = 2 ml 2 sin(2)K

Problem 4.39
A gyroscope consists of an outer gimbal, inner gimbal and a rotor with mass m (see Figure
4.32). The outer gimbal can rotate about the Z -axis defining the angle (precession
angle), the inner gimbal can rotate about the y -axis defining the angle (nutation angle),
the rotor can rotate about the z -axis defining the angle (rotation angle). The angles
( , , ) are called Euler angles.
Obtain the governing equations for the gyroscope.

University of Castilla-La Mancha


Ciudad Real - Spain

Draft

By: Eduardo W. V. Chaves (2014)

4 THE FUNDAMENTAL EQUATIONS OF CONTINUUM MECHANICS

365

- precession angle
Z

Precession axis

- nutation angle
- rotation angle

&
z - spin axis

outer gimbal

inner gimbal
y

&
&

x
X

Figure 4.32
Consider the inertia tensor components of the rotor according to the system Oxyz as
follows:
I 0 0
(I Oxr )ij = 0 I 0
0 0 I

Solution:
We will adopt the orthonormal basis of the fixed system OXYZ by ( I , J , K ), and for the
mobile system Oxyz we will adopt the orthonormal basis ( i , j , k ).
Angular velocity of the rotor is given by (see Figure 4.33):
r
+ & j + & k
= & K
= [ & sin( ) i + +& cos( )] k ] + & j + & k
= & sin( ) i + & j + [& + & cos( )] k

University of Castilla-La Mancha


Ciudad Real - Spain

Draft

By: Eduardo W. V. Chaves (2014)

SOLVING PROBLEMS BY MEANS OF CONTINUUM MECHANICS

366

& cos( )

& K

& sin( )
x

Figure 4.33
The governing equations for a rigid solid motion are given by:
r

F =ma

and

r&
= HO

r
where H& O can be calculated by means of

r
r
r&
DH O
DH Oxr
r r
=
+ H Oxr
H O

Dt OXr Dt Oxr

Angular momentum:

r
r
r
r
componente

s
H Oxr = I Oxr
( H Oxr )i = (IOxr )ij () j
r
( H Oxr )1 I 0 0 & sin( ) I& sin( )

r r

&
I&
( Hr Ox ) 2 = 0 I 0
=

( H r ) 0 0 I [& + & cos( )] I [& + & cos( )]

Ox 3

and its rate of change can be obtained by:


r
( H& r )
&& sin( ) + & cos( )& ]
I& sin( ) I[

r& Ox 1 D

I&
I&&
( H Oxr ) 2 =
=

( Hr& r ) Dt I [& + & cos( )] I D [& + & cos( )]


Ox 3

Dt

Note that due to the symmetry of the rotor, the inertia tensor does not change with time
with respect to the system Oxyz .
r r
r
We need to calculate the vector H Oxr , where is the angular velocity of the rotating
system Oxyz . Note that the mobile system can rotate about the K -axis and about the j axis, and cannot rotate about the k -axis, (see Figure 4.33), then, the angular velocity of the
mobile system can be given by:

University of Castilla-La Mancha


Ciudad Real - Spain

Draft

By: Eduardo W. V. Chaves (2014)

4 THE FUNDAMENTAL EQUATIONS OF CONTINUUM MECHANICS

367

= & K + & j
= [& sin( ) i + & cos( )] k ] + & j
= & sin( ) i + & j + & cos( ) k
i
j
r
H Oxr = & sin( ) &
I& sin( ) I&

k
& cos( )
I [& + & cos( )]

= {& I [& + & cos( )] & I& cos( )}i {I& sin( ) [& + & cos( )] + & I& sin( ) cos( )}j
+ {I& & sin( ) + I& & sin( )}k

& I [& + & cos( )] & I& cos( )


r

&
( H Oxr ) i = I sin( ) [& + & cos( )] & 2 I sin( ) cos( )

thus
r
DH r
r&
Ox
( H O ) i =
Dt

Applying


r r
+ ( H Oxr ) i
r
Ox i

&&

& &

& I [& + & cos( )] & I& cos( )


I [ sin( ) + cos( )]

2
=
I&&
+ I& sin( ) [& + & cos( )] & I sin( ) cos( )
I D [& + & cos( )]


Dt

&&
& &
& & &
I[ sin( ) + 2 cos( )] + I [ + cos( )]

= I& sin( ) [& + & cos( )] & 2 I sin( ) cos( ) + I&&

D & &
I
[ + cos( )]

Dt

r
r&
M O = H O we can obtain the following set of equations:

M
M
M

O1
O2
O3

M
M
M

x
y
z

&& sin( ) + 2& & cos( )] + I & [& + & cos( )]


= I[
= I [&& & 2 sin( ) cos( )] + I& sin( ) [& + & cos( )]
D & &
=I
[ + cos( )]
Dt

(4.90)

NOTE: Particular case: Steady precession.


&& = 0 and
In this case the variables , & and & are constant. With that we get & = 0 ,
&& = 0 , so
r
r
( H Oxr ) 1 I & sin( )
( )1 & sin( )
r

r & components r


0
0
;
; = K ( ) 2 =
( Hr Oxr ) 2 =
(r ) & cos( )
( H r ) I [& + & cos( )]
3
Ox 3

University of Castilla-La Mancha


Ciudad Real - Spain

Draft

By: Eduardo W. V. Chaves (2014)

SOLVING PROBLEMS BY MEANS OF CONTINUUM MECHANICS

368

r
() 1 x & sin( )

r
0

( ) 2 = y =
r & &
(

) 3 z [ + cos( )]

The equations in (4.90) become:

M
M

O1
O2

M
M

M
M

M O3

=0

= I & 2 sin( ) cos( ) + I& sin( ) [& + & cos( )]


14
4244
3
= z

Mz = 0
O1
O2
O3

M
M
M

=0

= [I z I & cos( )] & sin( )

=0

& = constant

&
r

= & K

z k

& k

x = & sin( )

y
r

Figure 4.34: Steady precession


Rigid Solid Motion Reference
BEER, F.P. & JOHNSTON, E.R. (1987). Vector Mechanics for Engineers: Dynamics. Seventh
Edition. 2 Volumes. McGraw-Hill Science/Engineering/Math; 4 edition.
BEER, F.P.; JOHNSTON, E.R. & CLAUSEN, W.E. (2004). Instructor's and Solutions Manual to
Accompany Vector Mechanics for Engineers - Dynamics. Seventh Edition. 2 Volumes. McGraw
Hill Higher Education; Seventh edition (2004)

University of Castilla-La Mancha


Ciudad Real - Spain

Draft

By: Eduardo W. V. Chaves (2014)

5 Introduction to:
Constitutive Equations,
IBVP Statement, and
IBVP Solution Strategies
Problem 5.1
Describe the constitutive equation and the free variables for simple thermoelastic materials
when we are considering the specific Helmholtz free energy .
Solution:
The constitutive equations for a simple material are in function of the following free
variables:
Constitutive equation for energy = ( F , T )
( F , T )
F
( F , T )
Constitutive equation for entropy ( F , T ) =
T
r
r
rT )
(
F
,
T
,
q
=
q

Constitutive equation for heat conduction 0


0
X

Constitutive equation for stress P( F , T ) = 0

The free variables are F -deformation gradient, T -temperature, Xr T -temperature


gradient, (see Chaves 2013 Chapter 6). The constitutive equations can also be expressed
as follows

= ( E , T )
( E , T )
S = 0

E
;
( E , T )
(E ,T ) =
T
r
r
q 0 = q0 ( E , T , Xr T )

= (F , T )
( F , T ) T
F
=

F
( F , T )
(F , T ) =
T
r
1 r
q = J q0 ( F , T , Xr T ) F T
r
= J 1F q0 ( F , T , Xr T )

Problem 5.2
Consider an elastic material in which the energy density (per unit volume) is known and is
given by:

SOLVING PROBLEMS BY MEANS OF CONTINUUM MECHANICS

372

1
2

( I E , II E ) = ( + 2 )I E2 2 II E
where and are material constants. I E = I E (E ) and II E = II E (E ) are, respectively, the
first and second principal invariants of the Green-Lagrange strain tensor. Obtain the
constitutive equations for this problem. Obtain also the explicit expression for the
constitutive equations in terms of , , I E and II E .
Formulary
I E = I E ( E ) = Tr ( E )

II E = II E ( E ) =

1
( TrE ) 2 Tr ( E 2 )
2

I E
=1
E
II E
= Tr ( E )1 E T
E

Solution:
According to the problem, the energy is only a function of the Green-Lagrange strain
tensor. We know that the general expressions for the constitutive equations for a simple
thermoelastic material are:

= ( E , T )
( E , T )
S = 0

E
( E , T )
(E ,T ) =
T
r
r
q 0 = q0 ( E , T , Xr T )

Considering the expression of the given energy, we can conclude that the problem is
independent of temperature, since the energy expression is not a function of temperature.
Then, the remaining constitutive equation is the one related to stress, i.e.:
S = 0

( E ) ( I E , II E ) ( I E , II E ) I E ( I E , II E ) II E
=
=
+
E
E
I E
E
II E
E

= ( + 2 )I E (1) + ( 2 ) Tr ( E )1 E T
2

Simplifying the above equation, and taking into account that E T = E , I E = Tr (E ) , we


obtain:
S = I E 1 + 2 E

Problem 5.3
Consider the specific Gibbs free energy G(S, T ) = ( E , T )

S : E as constitutive

equation for energy for thermoelastic materials. Obtain the remaining constitutive
equations for thermoelastic materials, based on the principle that G(S, T ) does not depend
on the temperature gradient.
Solution:
University of Castilla-La Mancha
Ciudad Real Spain

Draft

By: Eduardo W. V. Chaves (2015)

5 INTRODUCTION TO: CONSTITUTIVE EQUATIONS, IBVP, AND IBVP SOLUTION STRATEGIES

373

We start from the Clausius-Duhem inequality in terms of specific Helmholtz free energy in
the reference configuration:

1r
S : E& 0 & + T& q0 Xr T 0
T

(5.1)

Taking into account the specific Gibbs free energy we obtain the rate of change:
& (S, T ) = & ( E , T ) 1 S& : E 1 S : E&
G

& (S, T ) + 1 S& : E + 1 S : E&


& ( E , T ) = G

and by replacing the above equation into the inequality (5.1) we can obtain:
&
1r
1 &
1
S : E& 0 G
S:E+
S : E& + T& q0 Xr T 0
(S , T ) +
0
0

T
r
& (S , T ) S& : E T& 1 q
r
0G
0
0 XT 0
T

(5.2)

Note that S& : E = E : S& holds. The above inequality suggests that for a variation of Gibbs
free energy we must have the following relationships: Strain for variation of stress,
Entropy for a variation of temperature, and heat conduction in terms of temperature
gradient.
& (S , T ) can also be expressed as follows:
The term G

DG(S, T ) &
G(S, T ) & G(S , T ) &
T
G(S, T ) =
:S +
T
Dt
S

and by replacing the above equation into the equation in (5.2) we can obtain:
r
& (S, T ) E : S& T& 1 q
r
0G
0
0 XT 0
T
1r
G(S, T ) &
G(S, T ) &
:S 0
T E : S& 0T& q0 Xr T 0
0
S
T
T
r
1
G(S, T )

G(S , T )

0
+ E : S& 0
+ T& q0 Xr T 0
T
S

(5.3)

The above inequality must be satisfied for any admissible thermodynamic process. Let us
r
r
now consider the process such that T& = 0 (isothermal process), and q 0 = 0 (adiabatic
process), then the above entropy inequality becomes:
G(S, T )

0
+ E : S& 0

(5.4)

Note that the above inequality must also be met for any thermodynamic process. Then if in
the current process the condition in (5.4) is met, we can apply another process such that
S& = S& , in which the entropy inequality (5.4) is violated. Thus, the only way in which the
inequality in (5.4) is satisfied is when:

G(S, T )
+E =0
S

E = 0

G(S, T )
S

Then if we take into account the above equation into the inequality (5.3), we obtain:

University of Castilla- La Mancha


Ciudad Real Spain

Draft

By: Eduardo W. V. Chaves (2015)

SOLVING PROBLEMS BY MEANS OF CONTINUUM MECHANICS

374

1r

G(S, T )

G(S, T )
0
+ E : S& 0
+ T& q0 Xr T 0
S
T

(5.5)
G(S, T )
& 1 r
r
0
+ T q0 X T 0
T
T

r
Now let us consider a process where Xr T = 0 (uniform temperature field), then the

inequality becomes:
G(S , T )

0
+ T& 0

Starting from this point, we could apply another process where T& = T& , in which the
entropy inequality is violated. Thus, the only way in which the above inequality is satisfied
is when:
G(S, T )
+=0
T

G(S, T )
T

Then, the constitutive equations are:


Constitutive equation for energy G = G(S, T )
G(S , T ) g(S , T )
=
S
S
G(S, T )
Constitutive equation for entropy =
T
r
r
Constitutive equation for heat conduction q0 = q0 ( Xr T )

Constitutive equation for strain E = 0

(5.6)

where g(S, T ) = 0 G(S, T ) . Note that the free variables are (S , T ) .


Problem 5.4
Show that for an isothermal adiabatic process and with no rate of change of stress the
specific Gibbs free energy cannot increase.
Solution:
We start directly from the inequality in (5.3):
r
& (S, T ) E : S& T& 1 q
r
0G
0
0 XT 0
T

(5.7)
r

Taking into account the isothermal adiabatic process we have T& = 0 , q0 = 0 , and with no
rate of change of stress the equation S& = 0 holds. With that the inequality in (5.7)
becomes:
& (S, T ) 0
0G

(5.8)

Note that 0 > 0 is always positive, then to satisfy the above inequality the condition
& (S, T ) 0 must hold.
G

University of Castilla-La Mancha


Ciudad Real Spain

Draft

By: Eduardo W. V. Chaves (2015)

5 INTRODUCTION TO: CONSTITUTIVE EQUATIONS, IBVP, AND IBVP SOLUTION STRATEGIES

375

Problem 5.5
Find the governing equations for a continuum solid which has the following features:
Isothermal and adiabatic processes; an infinitesimal strain regime and a linear elastic
relationship between stress and strain.
b) Once the linear elastic, stress-strain relationship has been established, find the equation
in which ( ) is a tensor-valued isotropic tensor function.
Solution:
When we have isothermal and adiabatic processes, temperature and entropy play no role.
In an infinitesimal strain regime, the following is satisfied:
r

Strain tensors: E e = sym u


Stress tensors: P S
F 1

Xr xr

0 . If we take this approach, mass density is no

longer unknown ( & = 0 ).

Then, taking into account the fundamental equations:


The Fundamental Equations of Continuum Mechanics
(Current configuration)
The Mass Continuity Equation
(The principle of conservation of mass)
The Equations of Motion
(The principle of conservation of linear
momentum)
Cauchy Stress Tensor symmetry
(The principle of conservation of angular
momentum)
The Energy Equation
(The principle of conservation of energy)
The Entropy Inequality
(The principle of irreversibility)

r
D
+ ( xr v ) = 0
Dt

(5.9)

r
r
xr + b = v&

(5.10)

= T

(5.11)
r

u& = : D xr q + r
r

1
T

& ( x, t ) + : D

(5.12)

1
1 r
u& 2 q xr T 0
T
T

(5.13)

the remaining equations for the proposed problem are:


1) The equations of motion

r
r
+ b = v&

2) The energy equation

0 u&( X , t ) = S : E& Xr q 0 + 0 r ( X , t )

where u& is the specific internal energy, and the relationship


where is the specific Helmholtz free energy: Note also that

u& = : &
Du
D
[ + T ] = & holds,
=
Dt Dt

& = & e = : &


where e is the strain energy density, in which & e = & + & = & . Then if we bear in
mind the entropy inequality, we can observe that the proposed problem is characterized by

University of Castilla- La Mancha


Ciudad Real Spain

Draft

By: Eduardo W. V. Chaves (2015)

SOLVING PROBLEMS BY MEANS OF CONTINUUM MECHANICS

376

a process without any energy dissipation (an elastic process), i.e. all stored energy caused by
will recover when = 0 .
3) For this problem, the constitutive equations described in Problem 5.1 become:

= ( )
S=

( ) e ( )
=
= ( )

Energy ( ) and stress are only functions of strain. Then, if we calculate the rate of change
of the Helmholtz free energy, i.e. & ( ) =

& = & e = : & , we obtain:

( ) &
: , and by substituting it into the equation

( ) & & e ( ) &


: =
: = : &

e ( )

Thus, we can conclude that the energy equation is a redundant one, i.e. if the stress is
known the energy can be evaluated and vice-versa. So, we can summarize the governing
equations for the problem proposed as follows:
The equations of motion:

r
r
&r& (3 equations)
+ b = v& = u

The constitutive equations for stress:


( ) =

Kinematic equations:

(5.14)

e ( )
(6 equations)

r
= symu (6 equations)

The unknowns of the proposed problem are: (6), u (3) and (6), making a total of 15
unknowns and 15 equations, so the problem is well-posed. Then, to achieve the unique
solution of the set of partial differential equations given by (5.14) one must introduce the
initial and boundary conditions, hence defining the Initial Boundary Value Problem (IBVP) for
the linear elasticity problem. The initial and boundary conditions for this problem are:
The displacement boundary condition, on S u :
r r
r r
u( x , t ) =u* ( x , t )

r
r
ui ( x , t ) = u*i ( x , t )

(5.15)

r
jk n k = t *j ( x, t )

(5.16)

r
r
u i ( x , t = 0) = u 0 i ( x )
r
u& 0 i ( x ) = v 0 i

(5.17)

The stress boundary condition, on S :


r r
r
( x , t ) n = t * ( x , n , t )

The initial conditions ( t = 0 ):


r r
r
u( x , t = 0) = u0
r r
r r
r r
u0 ( x , t )
= u& 0 ( x , t ) = v0 ( x )
t
t =0

In the particular case when we have a static or quasi-static problem, the equations of
r r
motion become the equilibrium equations ( + b = 0 ), and the initial conditions
become redundant.

University of Castilla-La Mancha


Ciudad Real Spain

Draft

By: Eduardo W. V. Chaves (2015)

5 INTRODUCTION TO: CONSTITUTIVE EQUATIONS, IBVP, AND IBVP SOLUTION STRATEGIES

Su
dV

r
u*

377

r r
t * ( x)

r r

b( x )
n

Figure 5.1: Solid under external actions.


In subsection 1.6.1 The Tensor Series (Chapter 1-textbook, Chaves (2013)), we have seen
that we can approach a tensor-valued tensor function by means of the following series:
1
1 ( 0 )
1
2 ( 0 )
( 0 ) +
: ( 0 ) + ( 0 ) :
: ( 0 ) + L
0!
1!
2!

1
( 0 )
2 ( 0 )
0 +
: ( 0 ) + ( 0 ) :
: ( 0 ) + L
2

( )

Then, by considering the application point 0 = 0 and ( 0 ) = 0 = 0 , and also taking


into account that the relationship - is linear, higher order terms can be discarded, thus:
( ) =

( 0 )
2 e ( 0 )
: =
: = Ce :

where C e =

ij =

ij
kl

kl =

2 e ( 0 )
e
kl = Cijkl
kl
ij kl

2 e ( )
is a symmetric fourth-order tensor which is known as the elasticity

tensor, which contains the material mechanical properties.


Note that, the energy equation has to be quadratic with which we can guarantee that the
relationship - is linear, since ( ) =

e ( )
. We can also use series expansion to

represent the strain energy density as follows:


2 e ( 0 )
1 e ( 0 )
1
: ( 0 ) + ( 0 ) :
: ( 0 ) + L

1!
2!
2 e ( 0 )
1
: ( 0 ) + L
= e0 + 0 : ( 0 ) + ( 0 ) :

2
1 2 e ( 0 )
1
: = : Ce :
= :
2

2
1
0!

e ( ) = e ( 0 ) +

where we have also considered that 0 = 0 e0 = 0 , 0 = 0 .


NOTE 1: Although the energy equation is a redundant one, at the time of establishing an
analytical or numerical method to solve the problem, we will always start from energy
principles, hence the importance of studying the energy equation in a system.
NOTE 2: Analyzing C e :
e
kl and due to the symmetry of ij = ji
Note that, according to the equation ij = C ijkl
e
e
and kl = lk , the tensor C e has minor symmetry, i.e. C ijkl
= C ejikl = C ijlk
= C ejilk . Note also
that:

University of Castilla- La Mancha


Ciudad Real Spain

Draft

By: Eduardo W. V. Chaves (2015)

SOLVING PROBLEMS BY MEANS OF CONTINUUM MECHANICS

378

Ceijkl =

2 e ( ) 2 e ( )
=
= C eklij (major symmetry)
ij kl
kl ij

NOTE 3: To better illustration of the problem established here, let us consider a particular
case (a one-dimensional case) where the stress and strain components are given by:
0 0
ij = 0 0 0
0 0 0

0 0
e
ij = 0 0 0 11 = C1111
11 = E
0 0 0

In this case, the stress-strain linear relationship becomes = E (Hookes law) and the
1
2

1
2

strain energy density is given by e = = E , and

e ( )

2 e
=
= E , (see Figure 5.2).

Current state

()
1
2

e = E

Stored energy
1
e =
2

E
1

0 = 0

e0 = 0

0 = 0

Figure 5.2: Stress-strain relationship (one-dimensional case).


NOTE 4: Here it should be pointed out that in the case of elastic processes the
constitutive equation ( ) is only dependent on the current value of , i.e. it is
independent of the deformation history.
b) The tensor-valued tensor function ( ) is isotropic if the following is satisfied:

e (kl ) = e ( kl )

ij (kl ) = ij (kl )

Then, taking into account that the relationship - is given by ij ( ) = Ceijkl kl (indicial
notation), we can conclude that:
ij (kl ) = ij (kl )

e
e
Cijkl
kl = Cijkl
kl

e
Cijkl
= Ceijkl

That is, the fourth-order tensor C e is isotropic. An isotropic symmetric fourth-order


e
tensor has the form C ijkl
= ij kl + ( ik jl + il jk ) or C e = 1 1 + 2 I , (see Chapter
1), and here the parameters and are known as Lam constants. As we have seen in
Chapter 1, a symmetric isotropic fourth-order tensor is a function of two variables ( , ).
We will see that it is possible to express C e in terms of other parameters, e.g. ( E , ),
( , G ), where E is the Youngs modulus (or longitudinal elastic modulus), is the Poissons ratio,
is the bulk modulus, and G = is the shear modulus (or transversal elastic modulus).

University of Castilla-La Mancha


Ciudad Real Spain

Draft

By: Eduardo W. V. Chaves (2015)

5 INTRODUCTION TO: CONSTITUTIVE EQUATIONS, IBVP, AND IBVP SOLUTION STRATEGIES

379

NOTE 5: Figure 5.3 shows the stress-strain relationship for an isotropic material. Note
that, for an isotropic linear elastic material in an infinitesimal strain regime the constitutive
equation for stress becomes () = (1 1 + 2 I) : = Tr()1 + 2 :
e ( )
( ) =
4
3
14424

linear

isotropic

( ) = C e :

( ) = Tr ( )1 + 2

Elastic

It should be emphasized here that due to the fact that the C e -components are independent
of the coordinate system, the tensors and share the same principal space
(eigenvectors), (see Figure 5.3).
22

12

e
ij = C ijkl
kl

22

12

11

x1

ij = a ip a jq pq
ij = a ip a jq pq

11

e
ij = C ijkl
kl

22

22

12

12
11

x1

22

11

11

e kl
ij = C ijkl

22

Principal space

Isotropic material

11

e
e
Ceijkl = Cijkl
= Cijkl

e (kl ) = e ( kl )

x1

Figure 5.3: Stress-strain relationship (isotropic material).

University of Castilla- La Mancha


Ciudad Real Spain

Draft

By: Eduardo W. V. Chaves (2015)

SOLVING PROBLEMS BY MEANS OF CONTINUUM MECHANICS

380

NOTE 6: We denote the complementary strain energy density by e ( ) which is a function of


, (see Figure 5.4), and is given by:
e
2 e ( 0 )
1 ( 0 )
1
: ( 0 ) + ( 0 ) :
: ( 0 ) + L


1!
2!
2 e ( 0 )
1
e
= 0 + 0 : ( 0 ) + ( 0 ) :
: ( 0 ) + L

2
2 e ( 0 )
1
1
1
1
= :
: = :De : = : Ce :

2
2
2

1
0!

e ( ) = e ( 0 ) +

Note that if we are dealing with linear elastic material e ( ) = e () holds, and
=

e ( )
.

a) Linear elastic material.

1
Complementary strain energy density - e () = E 1
2

()

( ) = ( )
e

Stored energy
1
Strain energy density - e () = E
2

0 = 0

0 = 0

e + e =
b) Non-linear elastic material.
Complementary strain energy density - e ()

()

Strain energy density - e ()

e () e ()

0 = 0

0 = 0

Figure 5.4: Complementary strain energy density (one dimensional case).


NOTE 7: Note that e () = e () tensorial

e ( ) = g = : e ( ) = 0 G( ) ,
where g( ) = 0 G( ) is the Gibbs free energy density (per unit volume) with reversed
sign, (see equations in (5.6) in Problem 5.3).
University of Castilla-La Mancha
Ciudad Real Spain

Draft

By: Eduardo W. V. Chaves (2015)

5 INTRODUCTION TO: CONSTITUTIVE EQUATIONS, IBVP, AND IBVP SOLUTION STRATEGIES

381

NOTE 8: Taking into account the constitutive equation for stress for an isotropic linear
elastic material ( ) = Tr ( )1 + 2 and considering the additive decomposition of the
tensor into a spherical and deviatoric parts, i.e. = sph + dev =

Tr ( )
1 + dev , we can
3

obtain:

Tr ( )
( ) = Tr ( )1 + 2 = Tr ( )1 + 2 ( sph + dev ) = Tr ( )1 + 2
1 + dev
3

= +
Tr ( )1 + 2 dev = Tr ( )1 + 2 dev = sph + dev
3

33
23

13
13
11

23
12

12

23
12

11

+
m

22
12

12

dev
22

dev
33

23

12

ijdev = 2 ijdev

13

23

dev
11

Tr ( ) ij = 3 Tr ( ) ij

33

23

13
13

22

ij = Tr ( ) ij + 2 ij

13

dev
33

23

13
13

23
12

dev
22
12

dev
11

Figure 5.5: Additive decomposition of the constitutive equation.


Recall that, if we are dealing with small deformation regime, the volume ratio (dilatation) is
given by:
v =

dV dV 0 V
=
= 11 + 22 + 33 = Tr ( ) = I
dV0
dV0

And if we take the trace of ( ) = +

2
dev
Tr ( )1 + 2 we can obtain:
3

: 1 = +
Tr ( )1 : 1 + 2 dev : 1
3

2
2
Tr ( )

Tr ( ) = 3 +

= m = +
Tr ( )
v
3
3
3

University of Castilla- La Mancha


Ciudad Real Spain

Draft

By: Eduardo W. V. Chaves (2015)

SOLVING PROBLEMS BY MEANS OF CONTINUUM MECHANICS

382

where we have considered that 1 : 1 = 3 and Tr ( dev ) = 0 . If we are dealing with a


compression stress state ( p > 0 ) we have:
0
p 0

ij = 0 p 0
0
0 p

3 m = Tr ( ) = 3 p < 0

p = +
v = v
3

For these reason, the parameter is called bulk modulus (or modulus of compression), (see
Figure 5.6), and is given by:
=+

2
3

(5.18)

Just as the spherical part of the tensor ( sph = Tr ( )1 ) is associated with the volume
change, the deviatoric part ( dev = 2 dev ) is associated with the shape change, and the
parameter = G defines the stiffness to the shape change, where G is known as shear
modulus or transversal elastic modulus, (see NOTE 9).
xy

y
p

xy
p
x
E -Youngs modulus

-Bulk modulus

G -Shear modulus

xy

v -volumetric
strain

=G

xy

Figure 5.6: Some material mechanical properties.


NOTE 9: In the laboratory the parameters (, ) are not the more appropriated to be
obtained. Next we try to rewrite the constitutive equation in terms of other parameters.
Recall that the reverse form of the constitutive equation ( ) = Tr ( )1 + 2 was
obtained in Problem 1.98 which is:
=

1
1

Tr ( )1 indicial
ij =
ij
(11 + 22 + 33 ) ij
2
2 ( 2 + 3 )
2
2 ( 2 + 3 )

11

21
13

12
22
23

13
11
1

23 =
21
2
13
33

University of Castilla-La Mancha


Ciudad Real Spain

12
22
23

13
1 0 0
(11 + 22 + 33 )

0 1 0
23

2 (2 + 3 )
0 0 1
33
Draft

(5.19)

By: Eduardo W. V. Chaves (2015)

5 INTRODUCTION TO: CONSTITUTIVE EQUATIONS, IBVP, AND IBVP SOLUTION STRATEGIES

383

Notice also that the normal stress components 11 , 22 , and 33 only produce normal
strain components. Let us consider a particular case in which we only have the normal
stress 11 , 22 = 0 , 33 = 0 , then:
11

21
13

12
22
23

13
11
1

23 =
0
2
0
33

0 0
1 0 0
(11 )

0 0
0 1 0

2 (2 + 3 )
0 0 1
0 0

with that the normal strain components are:


11 =

( + )
1

11 =
11
11
(11 ) 11 =

2
2 (2 + 3 )
2 2 (2 + 3 )
(2 + 3 )

(11 ) 22 =
11
2 (2 + 3 )
2 (2 + 3 )

=
(11 ) 33 =
11
2 (2 + 3 )
2 (2 + 3 )

(5.20)

22 =

(5.21)

33

(5.22)

From the equation of 11 given by the equation in (5.20) we can obtain:


( + )
11
11 =
(2 + 3 )

where we have denoted by E =

11 =

(3 + 2 )
11
( + )

11 = E11

(3 + 2 )
, which is known as Youngs modulus, or
( + )

longitudinal elastic modulus.


As expected, due to the material isotropy, the influence of 11 upon 22 and 33 is the
same, and we can also obtain:
22 =

(3 + 2 )

11 = 11
11 =
11 =

2 (2 + 3 )
2 (2 + 3 ) ( + )
2( + )

33 =

(3 + 2 )

11 = 11
11 =
11 =

2 (2 + 3 )
2 (2 + 3 ) ( + )
2( + )

where we have denoted by =


Poissons

ratio

2( + )

can

, which is known as Poissons ratio. And the

2( + )
assume 1.0 < < 0.5 ,

(see

Problem

1.92).

Note

2
and if we replace it into the equation of E we can obtain:
(1 2 )

+ 2
3
(3 + 2 )
(1 2 )

E=
=
=
( + )
2

(1 2 )

2
+ 2
3
(
1
2
)

=
2
+ 1

(1 2 )

that

+ 2

(1 2 )

+ 1

(1 2 )

6 + 2(1 2 )

(1 2 )
=
= 2 (1 + )
2 + (1 2 )

(1 2 )

thus:
University of Castilla- La Mancha
Ciudad Real Spain

Draft

By: Eduardo W. V. Chaves (2015)

SOLVING PROBLEMS BY MEANS OF CONTINUUM MECHANICS

384

G==

E
2(1 + )

and

2
E
=
(1 2 ) (1 + )(1 2 )

The physical interpretation of comes next, (see Figure 5.6). Let us suppose a stress state
in which is acting just the component 12 , with that and according to the equation in (5.19)
we obtain the only strain not equal to zero:
12 = 21 =

1
12
2

12 = 212
{
{

xy

xy = xy = G xy

G=

xy

We can also express the bulk modulus in function of ( E , ) :


= +

2
3E + E (1 2 )
E (1 + )
E
2
E
E
=
+
=
=
=
3 (1 + )(1 2 ) 3 [2(1 + )] 3(1 + )(1 2 ) 3(1 + )(1 2 ) 3(1 2 )

So, we can obtain the relationships between these mechanical properties:


G==
f (G; E )
f (G; )
f (G; )

f (G; )
f ( E; )
f ( E; )
f ( ; )

f ( ; )

E=

GE
9G 3E

G (E 2G )
3G E

9G
3 + G

E 2G
2G
3 2G
2(3 + G )

G (3 + 2G )
+G

+ G

2( + G )

2G (1 + )

2G (1 + )
3(1 2 )

2G
1 2

E
3(1 2 )

(9 3E )
9 E
E
(1 + )(1 2 )

9 ( )
3

3(1 2 )

3
1+

3E
9 E
E
2(1 + )
3( )
2

3(1 2 )
2(1 + )

2
3

2G
3

3 E
6

We leave the reader to show:


Tensorial notation

Indicial notation
ij = kk ij + 2 ij

= Tr ( )1 + 2

E
E
Tr ( )1 +

(1 + )(1 2 )
(1 + )

=
Tr ( )1 + 2
3

ij =

E
E
kk ij +
ij
(1 + )(1 2 )
(1 + )

ij =
kk ij + 2 ij
3

(5.23)
(5.24)
(5.25)

and

University of Castilla-La Mancha


Ciudad Real Spain

Draft

By: Eduardo W. V. Chaves (2015)

5 INTRODUCTION TO: CONSTITUTIVE EQUATIONS, IBVP, AND IBVP SOLUTION STRATEGIES

Tensorial notation

385

Indicial notation

1
Tr ( )1 +

2 (3 + 2 )
2

ij =

1
kk ij +
ij
2 (3 + 2 )
2

(5.26)

1+

Tr ( )1 +
E
E

ij =

1+
kk ij +
ij
E
E

(5.27)

2 3
1
kk ij
ij =
ij
2
18

(5.28)

2 3
1
Tr ( )1
=

2
18

and that the elasticity tensor for isotropic material can be written as follows:
C e = 1 1 + 2 I
Ce =

E
E
1 1 +
I
(1 + )(1 2 )
(1 + )
Elasticity tensor

(5.29)

C e = 1 1 + 2 I 1 1
3

and:
Ce

De =

1 1 +
I
2 (3 + 2 )
2

(1 + )

1 1 +
I
E
E
1
1 1

1 1 +
I 1 1
De =

9
2 3

Ce De =
Ce

Elastic compliance tensor

In the International System of Units we have [G ] = [ ] = [] = [ ] = [ E ] = Pa , and is a


dimensionless quantity.
Problem 5.6
In tensile testing we have evaluated the following points:
Point
1
2
3
4
5

( Pa ) (10 3 )
6.67
13.3
20
24
22

0.667
1.33
2
3
3 .6

Calculate Youngs modulus ( E ) and define the stress-strain curve limit points.
Solution: First, we verify that the first three points maintain the same proportionalities:
E=

(1) ( 2 ) ( 3)
20
= ( 2 ) = ( 3) =
= 10 000 Pa = 10 kPa
(1)
2 10 3

The stress-strain curve can be appreciated in Figure 5.7, in which we define the following
points: e - the proportionality point; Y - the yield point; u - the ultimate strength
point; and r - the rupture strength point.
University of Castilla- La Mancha
Ciudad Real Spain

Draft

By: Eduardo W. V. Chaves (2015)

SOLVING PROBLEMS BY MEANS OF CONTINUUM MECHANICS

386

(Pa ) 30
u

25

20
15

3; 24

3.6; 22

2; 20

1.33; 13.3

10
0.667; 6.67

E
0; 0

0
0

0 . 2%

0.5

1.5

2.5

3.5

4
3

(10 )

Figure 5.7: Stress-strain curve.


Problem 5.7
Show that the strain energy density, for an isotropic linear elastic material, can be written as
follows:
1
2

e ( ) = ( + 2 ) I 2 2 II

a)

(5.31)

or
b)

e ( ) =

( + )
1
I 2
II
2 (3 + 2 )
2

e ( ) =

[Tr ( )]2 + 1
42
dev : dev
4 43
4
2
14243

(5.33)

1
I 2 +
6(3 + 2 )
144244
3

(5.34)

(5.32)

or
c)

purely volumetric
energy

purely distortional
energy

or
d)

e ( ) =

purely volumetric
energy

1
J2
2
123

purely distortional
energy

where I = Tr ( ) is the trace of (infinitesimal strain tensor), I = Tr ( ) is the first


invariant of the Cauchy stress tensor , and II dev = J 2 is the second invariant of the
deviatoric Cauchy stress tensor. Note that, for linear elastic material the relationship
e ( ) = e () holds, (see Figure 5.4).

University of Castilla-La Mancha


Ciudad Real Spain

Draft

By: Eduardo W. V. Chaves (2015)

5 INTRODUCTION TO: CONSTITUTIVE EQUATIONS, IBVP, AND IBVP SOLUTION STRATEGIES

387

Solution:
1
2

a) Taking into account the strain energy e = : and ( ) = Tr ( )1 + 2 , (see


equation (5.23)), we can obtain:
1
1
2
2
1
1
1
2
= Tr ( ) 1
2
:3
1 + : = [Tr ( )] + : = I 2 + :
2
2
2
Tr ( )

e = : = : [Tr ( )1 + 2 ]

(5.35)

Taking into account the definition of the second invariant and the symmetry of we can
obtain:

] [

] [

] [

1
1
1
1
[ Tr ( )] 2 Tr ( 2 ) = I 2 Tr ( ) = I 2 Tr ( T ) = I 2 :
2
2
2
2
2
: = I 2 II
II =

(5.36)

Then, the equation in (5.35) can be rewritten as follows:


1
2

1
2

1
2

e = I 2 + : = I 2 + ( I 2 2 II ) = ( + 2 ) I 2 2 II
1
2

b) Taking into account the strain energy e = : and


=

1
Tr ( )1 +
, (see equation (5.26)), we can obtain:
2 (3 + 2 )
2

1
1
1

:
Tr ( )1 +
e = : =
2
2 2 (3 + 2 )
2
1

[Tr ( )]2 + 1 :
2
:3
1+
: =
=
Tr ( ) 1
4
4 (3 + 2 )
4
4 (3 + 2 )
Tr ( )
=

(5.37)

:
I 2 +
4 (3 + 2 )
4

According to the equation (5.36) we can conclude that : = I 2 2 II , with that the
above equation becomes:

e=
=

1
1

( I 2 2 II )
I 2 +
: =
I 2 +
4 (3 + 2 )
4
4 (3 + 2 )
4
( + )
1
I 2
II
2 (3 + 2 )
2

1
2

c) Taking into account the strain energy e = : and =

2
Tr ( )1 + 2 , (see
3

equation (5.25)), we can obtain:

2
1
1
e = : = : Tr ( )1 + 2
2
2
3

2
1

2
=
2
:3
1 + : = [Tr ( )] + :
Tr ( ) 1
2

University of Castilla- La Mancha


Ciudad Real Spain

Tr ( )

Draft

(5.38)

By: Eduardo W. V. Chaves (2015)

SOLVING PROBLEMS BY MEANS OF CONTINUUM MECHANICS

388

If we consider that a second-order tensor can be split additively into a spherical and
deviatoric parts, i.e. = sph + dev =

Tr ( )
1 + dev , the expression : can be written as:
3

Tr ( )
Tr ( )

: =
1 + dev :
1 + dev
3
3

Tr ( )
Tr ( ) dev
Tr ( )
=
1 : dev +
: 1 + dev : dev
1 :1 +
3
3
3
=

[Tr()]2
3

(5.39)

+ dev : dev

where we have applied that 1 : 1 = 3 , 1 : dev = dev : 1 = Tr ( dev ) = 0 (the trace of any
deviatoric tensor is zero). With that the equation in (5.38) becomes:
[Tr ( )]2



+ dev : dev
e = [Tr ( )]2 + : = [Tr ( )]2 +

[Tr()]2 + dev : dev


2

d) To show the equation (5.34) we will use the strain tensor defined in (5.26),
=

1
. Then, the strain energy can be written as:
Tr ( )1 +
2 (3 + 2 )
2

1
1
1
e = : =
Tr ( )1 +
:
2
2 2 (3 + 2 )
2

1
[Tr ( )]2 + 1 :
=
Tr ( ) 1
12
:3
+
: =
4 (3 + 2 )
4
4 (3 + 2 )
4
Tr ( )

Note that : =

[Tr( )]2
3

(5.40)

+ dev : dev holds (see equation (5.39)). Taking into account the

equation of the second invariant of a second-order tensor we can obtain:


2
2
1
1
[ Tr ( dev )] 2 Tr ( dev ) =
Tr ( dev )

2
2
T
1
1
1 dev
=
Tr ( dev dev ) =
Tr ( dev dev ) =
: dev
2
2
2

II dev =

where we have used that: the trace of the deviatoric tensor is zero Tr ( dev ) = 0 , the
symmetry of the tensor dev = dev , and trace property Tr ( A B T ) = A : B . Then, we can
obtain:
T

: =

[Tr( )]2 + dev : dev = [Tr( )]2 2 II


3

dev

[Tr( )]2 + 2J
3

By substituting the above equation into the equation in (5.40), we can obtain:

University of Castilla-La Mancha


Ciudad Real Spain

Draft

By: Eduardo W. V. Chaves (2015)

5 INTRODUCTION TO: CONSTITUTIVE EQUATIONS, IBVP, AND IBVP SOLUTION STRATEGIES

e=

389

[Tr( )]2 + 1 :
4 (3 + 2 )
4

1 [Tr ( )]
2
[
]
=
+ 2J 2
Tr ( ) +

4 (3 + 2 )
4
3

1
1
[Tr ( )]2 +
=
+
J2
2
4 (3 + 2 ) 12

1
[Tr( )]2 + 1 J 2
6(3 + 2 )
2

Problem 5.8
Write in Voigt notation: a.1) the strain energy density and, a.2) the constitutive equations in
stress for an isotropic linear elastic material: a.2.1) in terms of ( , ) and, a.2.2) in terms of
( E , ) where =

E
E
and =
. b) Write the infinitesimal strain tensor
(1 + )(1 2 )
2(1 + )

in Voigt notation such as { } = [ L(1) ]{u } where {u } is the displacement field, obtain the

matrix [ L(1) ] .
c) Write the equations of motion in Voigt notation.
Solution:
a.1) The strain energy density ( e () -scalar) can be expressed as follows:
1
2

1
2

1
2

1
2

e ( ) = : C e : = : = : = ij ij
where we have used = C e : . Note that
ij ij = 1 j 1 j + 2 j 2 j + 3 j 3 j
123
123
123
3131
21 21
1111
+
+
+
3232
22 22
1212
+
+
+
3333
23 23
1313

thus
1
2

1
2

e () = ij ij = (1111 + 22 22 + 33 33 + 212 12 + 2 23 23 + 21313 )


and

1
2

e () = ij ij =

1
[11 22
2

33 12

23

11

22
33 1
T
13 ]
= { } { }

2
2
12

2 23

213

Then, the tensors and in Voigt notation are stored as follows:

University of Castilla- La Mancha


Ciudad Real Spain

Draft

By: Eduardo W. V. Chaves (2015)

SOLVING PROBLEMS BY MEANS OF CONTINUUM MECHANICS

390

11
11


22
22
33
33
{ } = ; { } =

212
12
2 23
23


213
13

a.2.1) The constitutive equation for stress in Voigt notation is:

11 + 2

+ 2

22
33

+ 2

=
= C e : Voigt
0
0
12 0
23 0
0
0

0
0
13 0

0
0

0
0

0 11
0 22
0 33
{ } = [C ] { }

0 212
0 2 23

213

0
0
0

(5.41)
More detail about this formulation is provided in Problem 1.97 in Chapter 1 where we
have also obtained
=

Tr ( )1
2
2 ( 2 + 3 )

and

( 2 + 3 )

11

2 ( 2 + 3 )

22
33 2 ( 2 + 3 )

=
212
0
2 23

213
0

a.2.2) Note that

2 ( 2 + 3 )
+
( 2 + 3 )

2 ( 2 + 3 )

2 ( 2 + 3 )

2 ( 2 + 3 )
+
( 2 + 3 )

0
1

0 11

22
0
33

0 12
23

0 13

(5.42)

{ } = [ C ]1 { }

E
E
E
(1 )
+2
=
(1 + )(1 2 )
2(1 + ) (1 + )(1 2 )
E
=

(1 + )(1 2 )
(1 2 )
E
E
=
=
2(1 + ) (1 + )(1 2 )
2

+ 2 =

then, the equation (5.41) can be rewritten as follows:

University of Castilla-La Mancha


Ciudad Real Spain

Draft

By: Eduardo W. V. Chaves (2015)

5 INTRODUCTION TO: CONSTITUTIVE EQUATIONS, IBVP, AND IBVP SOLUTION STRATEGIES

(1 )

11

(1 )


(1 )
22

33
E
0
0
0
=

(
1
)(
1
2
)
+

12

0
23
0
0

13
0
0
0

0
0
(1 2 )
2

0
0

0 11

0 22

0 33
212
0 2 23

(1 2 ) 213

2
0

(1 2 )
2

391

(5.43)

Note that

+ =

E
E
E
+
=
(1 + )(1 2 ) 2(1 + ) 2(1 + )(1 2 )

(2 + 3) =

E
E
E
E2
+3
=
2

2(1 + ) 2(1 + )
(1 + )(1 2 ) 2(1 + )(1 2 )

E
2(1 + )(1 2 ) 1
+
=
=
E
(2 + 3) 2(1 + )(1 2 )
E2
E

(1 + )(1 2 )
=
=
E
2 (2 + 3) (1 + )(1 2 )
E2
1

2(1 + ) 1
= 2(1 + )
E
E

Then, the equation (5.42) becomes:


11
1


22

33 1
=

212 E 0
0
2 23

213
0

0
0

1
0
0

11

0
0
0 22
0
0
0 33

2(1 + )
0
0 12
0
2(1 + )
0 23

0
0
2(1 + ) 13
0

(5.44)

b) According to the definition ij = 12 (u i , j + u j ,i ) we can obtain:

11
ij = 21
31

12
22
32

u1

x1
13
1 u
u
23 = 1 + 2
2 x
x1
33 2
u
1 u
1 + 3
2 x3 x1

University of Castilla- La Mancha


Ciudad Real Spain

Draft

1 u1 u 2

+
2 x 2 x1
u 2
x 2
1 u 2 u 3

+
2 x3 x 2

1 u1 u 3

+
2 x3 x1
1 u 2 u 3

+
2 x3 x 2

u 3

x3

By: Eduardo W. V. Chaves (2015)

SOLVING PROBLEMS BY MEANS OF CONTINUUM MECHANICS

392


u1
x
x
1
1

u2 0
11 x

2


22 u3 0

{ } = 33 = u x3u =
212 1 + 2
2 23 x2 x1 x2

u2 u3
+
0
213
x3 x2
u1 u3
x + x x
1
3
3

x2
0

x1

x3
0


u1
x3
u2
0 u
3


x2


x1

{ } = [ L(1) ]{u }

NOTE: If we adopt the engineering notation, i.e. x1 = x , x 2 = y , x3 = z , u1 = u , u 2 = v ,


u 3 = w , 11 = x , 22 = y , 33 = z , 2 12 = xy , 2 23 = yz , 2 13 = xz , the above equation
becomes:
u
x x
v
0
11 x

y
22 y w
0

{ } = 33 = z = u z v =
212 xy +
2 23 yz y x y

v w
213 xz +
0
z y
u + w
z x z

0
y

u
0

z v { } = [ L(1) ]{u }
(5.45)

0 w

x

z y

x
r
r
&r& , (see equation (5.14)), in
c) Let us consider the equations of motion, + b = v& = u
&& i and its explicit form:
indicial notation ij , j + b i = u
0

&& i
ij , j + b i = i1,1 + i 2, 2 + i 3,3 + b i = u
&&1
11,1 + 12, 2 + 13,3 + b1 = u

&& 2
21,1 + 22, 2 + 23,3 + b 2 = u

&&
31,1 + 32, 2 + 33,3 + b 3 = u 3

11 12 13
&&1
+
+
+ b1 = u

x
x
x
1
2
3

21 22 23
&& 2
+
+
+ b 2 = u

x
x
x
2
3
1
31 32 33
&& 3
+
+
+ b 3 = u

x1
x 2
x 3

Then, if we consider the stress tensor in Voigt notation, the above set of equations
becomes:

University of Castilla-La Mancha


Ciudad Real Spain

Draft

By: Eduardo W. V. Chaves (2015)

5 INTRODUCTION TO: CONSTITUTIVE EQUATIONS, IBVP, AND IBVP SOLUTION STRATEGIES

x1
0

x2

x2

x1

x3

11

x3 22 b u
&&
33 1 1

&&2
0 + b 2 = u

12
&&

b3 u3
23
x1
13

x3

x2

393

(5.46)

[ L(1) ]T { } + {b } = {u&&}

Problem 5.9
Consider an isotropic homogeneous linear elastic material described in Problem 5.5.
Obtain the governing equation so as to result in a system of three equations and three
unknowns, namely: u1 , u 2 , u 3 , (Displacement Formulation established by Navier (1827)).
Solution:
As we have seen in Problem 5.5, the governing equations, for an isotropic linear elastic
material in small deformation regime, are:
Tensorial notation

Indicial notation

The equations of motion:

The equations of motion:

r
r
&r& (3 equations)
+ b = v& = u

&& i (3 equations)
ij , j + b i = u

The constitutive equations for stress:

The constitutive equations for stress:

= Tr ( )1 + 2 (6 equations)

ij = kk ij + 2 ij (6 equations)

The kinematic equations:

(5.47)

The kinematic equations:

r
= sym u (6 equations)

ij =

1 u i u j
+
2 x j
x i

(6 equations)

which results in a system with 15 equations and 15 unknowns (u i , ij , ij ) .


The divergence of the Cauchy stress tensor ( ) can be obtained by means of the
constitutive equations for stress, i.e.:
ij = kk ij + 2 ij

ij , j = ( kk ij + 2 ij ) , j

ij , j = , j kk ij + kk , j ij + kk ij , j + 2 , j ij + 2 ij , j
{
{
{
=0 j

=0i

ij , j = kk , j ij + 2 ij , j

=0 j

(5.48)

ij , j = kk ,i + 2 ij , j

Note that, if the mechanical properties and are constants throughout the medium, i.e.
r

if they do not vary with x (homogeneous material) we can obtain , j

,j

= 0 j and
x j

= 0 j . We can also express the terms kk ,i and ij, j in function of displacements.


x j

For this, we use the kinematic equations:

University of Castilla- La Mancha


Ciudad Real Spain

Draft

By: Eduardo W. V. Chaves (2015)

SOLVING PROBLEMS BY MEANS OF CONTINUUM MECHANICS

394

ij =

1 u i u j
+
2 x j
xi

1
1
u i , j + u j ,i divergence
ij , j = u i , jj + u j ,ij
2
2

Note that
2ui

=
x j x j x j
u j ,ij

kk =

2u j
x j xi

u i

x j

r
r
r
u i , jj [ (u)]i 2 u (Laplacian of the vector u )
i

[ ]

2u j
x i x j

1 u k u k

+
2 x k
x k

x i

u j

x j

r
u j , ji [ ( u)]i

u k
gradient
=
u k ,k

kk ,i = u k ,ki = u j , ji

x
k

With that the equation in (5.48) can be rewritten as:


ij , j = kk ,i + 2 ij , j = u j , ji + 2

1
u i , jj + u j , ji = ( + )u j , ji + u i , jj
2

&& i (equations of motion), we obtain:


By replacing the above equation into ij , j + b i = u
ij , j + b i = &u& i
&& i
( + )u j , ji + u i , jj + b i = u

Thus resulting in 3 equations and 3 unknowns ( u1 , u 2 , u 3 ):


&&i
( + )u j , ji + ui , jj + b i = u
r
r
r
&r&
( + )[ ( u)] + [ (u)] + b = u

Naviers equations

(5.49)

Note that () 2 is the Laplacian of .


NOTE 1: The above equations are known as the Naviers equations also known as NavierLam equations. The explicit form of the equation (5.49) is presented as follows:
&& i
( + )u j , ji + u i , jj + b i = ( + )(u1,1i + u 2, 2i + u 3,3i ) + (u i ,11 + u i , 22 + u i ,33 ) + b i = u

&&1
( + )(u1,11 + u 2, 21 + u 3,31 ) + (u1,11 + u1, 22 + u1,33 ) + b1 = u

&& 2
( + )(u1,12 + u 2, 22 + u 3,32 ) + (u 2,11 + u 2, 22 + u 2,33 ) + b 2 = u

&&
( + )(u1,13 + u 2, 23 + u 3,33 ) + (u 3,11 + u 3, 22 + u 3,33 ) + b 3 = u 3

or:

2u
2 u1 2 u1
u1 u 2 u 3
&&1
+ b1 = u

+ 21 +
+
+
+
( + )
x
x1 x1 x 2 x3
x 22
x32

2u 2 2u 2 2u 2
u1 u 2 u 3

&&

(
)

+
+
+
+

x 2 + x 2 + x 2 + b 2 = u 2
x
x
x

2
1
2
3

1
2
3

2
2
2
( + ) u1 + u 2 + u 3 + u 3 + u 3 + u 3 + b = u
&& 3
3
2
2
x

x 2

x
x
x

x
x

3
1
2
3
2
3
1

University of Castilla-La Mancha


Ciudad Real Spain

Draft

By: Eduardo W. V. Chaves (2015)

5 INTRODUCTION TO: CONSTITUTIVE EQUATIONS, IBVP, AND IBVP SOLUTION STRATEGIES

395

NOTE 2: The above set of equations in matrix form becomes [ A]{u} = {p} , where:

D2
2
2
( + ) 2 + 2
Dt
x1

[A] =
( + )
x

2 x1

2
(

)
+

x3 x1

( + )

2
x1x2

D2
2
2

Dt 2
x22
2
( + )
x3 x2

( + )

x1x3

2
( + )
,
x2 x3

D2
2
2
( + ) 2 + 2
Dt
x3
( + )

b1
u1

{u} = u 2 , and {p} = b 2 .


u
b
3
3
r

Note that 2 = ( ) ( ) =

2
2
2
2
2
2

= 2 + 2 + 2 . The matrix
+
+
=
xk xk x1x1 x2 x2 x3 x3 x1 x2 x3

[ A] can also be written as follows:

D2
2
2
( + ) 2 + 2
Dt
x1

[A] =
( + )
x2 x1

(
+
)

x3x1

2
x1
2
= ( + )
x x
22 1

x x
3 1

2
x1x2
2
x22
2
x3x2

( + )

2
x1x2

D2
2
+ 2 2
2
Dt
x2
2
( + )
x3x2

( + )

x1x3

2
( + )

x2 x3

D2
2
2
( + ) 2 + 2
Dt
x3
( + )

x1x3
1 0 0
D 2
2
2

+
2 0 1 0

x2 x3 xk xk
Dt

0 0 1
2

x32

Using the indicial and tensorial notations the above equation can be written as follows:
A ij = ( + )

r
r

D2

2
D2
+
2 ij and A = ( + )[( ) ( )] + 2 2 1
xi x j xk xk
Dt
Dt

Then, we can also express the Naviers equation as follows:


r
r
r

D2 r
2
1 u = b

)[(

)
(

)]

Dt

or

D2

2
+
2 ij u j = b i
( + )
xi x j xk xk
Dt

The above equation could have been easily obtained by means of the equation in (5.49),
i.e.:

University of Castilla- La Mancha


Ciudad Real Spain

Draft

By: Eduardo W. V. Chaves (2015)

SOLVING PROBLEMS BY MEANS OF CONTINUUM MECHANICS

396

( + )u j , ji + u i , jj + b i = &u&i
&& k ik
( + )u k ,ki + u k , jj ik + b i = u
&& k ik = b i
( + )u k ,ki + u k , jj ik u
&& k ) ik = b i
( + )u k ,ki + ( u k , jj u

ik = b i

2
2

D
( + )
+
2 ik u k = b i
xk xi xk xk
Dt

( + )

2u k
u k
D 2u k
+

xk xi xk xk
Dt 2

NOTE 3: We have proven in Problem 1.105 (Chapter 1) that the following is true:
r
r r
r
r
( a) = ( a) 2 a
indicial

ilq qjk a k , jl = a j , ji a i , jj
Then, we can obtain

r
r r
r
r
r
(u) 2 u = ( u) ( u)

indicial

u i , jj = u j , ji ilq qjk u k , jl

with which the equation (5.49) can also be written as follows:


&&i
( + )u j , ji + ui , jj + bi = u

&&i
( + )u j , ji + (u j , ji ilq qjk u k , jl ) + bi = u
&&i
( + 2 )u j , ji ilq qjk uk , jl + bi = u

and the equivalent in tensorial notation:

r
r
r
&r&
( + )[ ( u)] + [ (u)] + b = u
r
r
r r
r
r
&r&
( + )[ ( u)] + ( u) ( u) + b = u
r
r
r r
r
&r&
( + 2 )[ ( u)] ( u) + b = u
r
r
r r
r
&r&
( + 2 )[ ( u)] ( u) + b = u

[
[

]
]

(5.50)

&& i
( + 2 )u j , ji ilq qjk u k , jl + b i = u

In the Cartesian System we have:


r
u = u i e i = u1 e 1 + u 2 e 2 + u 3 e 3

r r
r
r
u
u
u
u
u
u
( u) rot (u) = (rot (u) )i e i = 3 2 e 1 + 1 3 e 2 + 2 1 e 3
x 2 x3
x3 x1
x1 x 2
14
14
14
42r44
3
42r44
3
42r44
3
r
r
r
r r (rot (u) )3 (rot (u) )2

( u) =
x 2
x 3

= (rot (u) )1

= (rot (u) )2
r
r

(rot (u) )1 (rot (u) )3


e 1 +

x
x1
3

= (rot (u) )3

r
r

(rot (u) )2 (rot (u) )1


e 2 +
e 3

x1
x 2

r
r
(rot (u) )3 (rot (u) )2 u 2 u1 u1 u 3

x x
x3
2 1 x 2 x3 x3 x1
x 2r

(rot (u) )
r
r r
(rot (u) )3 u 3 u 2 u 2 u1
1

( u) i =
=

x1
x 3
x 3 x 2 x 3 x1 x1 x 2
r
r
(rot (u

) )2 (rot (u) )1 u1 u 3
u 3 u 2

x1 x3 x1 x 2 x 2 x3
x1
x 2

University of Castilla-La Mancha


Ciudad Real Spain

Draft

By: Eduardo W. V. Chaves (2015)

5 INTRODUCTION TO: CONSTITUTIVE EQUATIONS, IBVP, AND IBVP SOLUTION STRATEGIES

397

NOTE 4: If we are dealing with heterogeneous material, the equations in (5.48) must be
treated as follows:
ij = kk ij + 2 ij

ij , j = ( kk ij + 2 ij ) , j
ij , j = ( kk ) , j ij + (2 ij ) , j = ( kk ) ,i + ( 2 ij ) , j

Taking into account that 2 ij = u i , j + u j ,i and kk = u k ,k , the above equation becomes:


ij , j = ( kk ) ,i + (2 ij ) , j

ij , j = (u k , k ) ,i + (u i , j + u j ,i ) , j

whereby
ij , j + b i = &u& i

(u k ,k ) ,i + (u i , j + u j ,i ) , j + b i = &u& i

(5.51)

Note that

r
r
u k , k = Tr (u) = ( u) , and
u&
u&
u&
u&
Du& i u& i u& i
&& i =
u
=
+
v j = i + i v1 + i v 2 + i v 3 , and its components
t x j
t
x1
x 2
x 3
Dt

u& 1 u& 1

u&
u&
+
v1 + 1 v 2 + 1 v3

x1
x 2
x3
t

u& 2 u& 2
u& 2
u& 2

&& i =
+
v1 +
v2 +
v3
u
x1
x 2
x 3
t
u&

u&
u&
u&
3 + 3 v1 + 3 v 2 + 3 v3
t

x1
x 2
x3

(u i , j + u j ,i ) , j =
(u i , j + u j ,i )
x j

(u i ,1 + u1,i ) +
(u i , 2 + u 2,i ) +
(u i ,3 + u 3,i )
x1
x 2
x3

2 (u1,1 ) +
(u1, 2 + u 2,1 ) +
(u1,3 + u 3,1 )

x 2
x3
x1

2 (u 2, 2 ) +
(u i , j + u j ,i ) , j =
(u 2,1 + u1, 2 ) +
(u 2,3 + u 3, 2 )
x 2
x3
x1

2 (u 3,3 )
(u 3,1 + u1,3 ) +
(u 3, 2 + u 2,3 ) +

x1

x 2
x3
The three equations in (5.51), ( i = 1,2,3 ), are explicitly given by:

2 (u1,1 ) +
(u1, 2 + u2,1 ) +
(u1,3 + u3,1 ) + b1 = u&&1
[( u)] +

x
x
x
x
1
2
3
1

r

[( u)]+ (u2,1 + u1,2 ) + 2 (u2,2 ) + (u2,3 + u3,2 ) + b 2 = u&&2

x1
x2
x3
x
2

[( ur )] + (u3,1 + u1,3 ) + (u3,2 + u2,3 ) + 2 (u3,3 ) + b3 = u&&3

x1
x2
x3
x3

or

University of Castilla- La Mancha


Ciudad Real Spain

Draft

By: Eduardo W. V. Chaves (2015)

SOLVING PROBLEMS BY MEANS OF CONTINUUM MECHANICS

398

( u) + 2 (u1,1 ) +
(u1, 2 + u2,1 ) +
(u1,3 + u3,1 ) + b1 = u&&1

x2
x3
x1

r

( u) + 2 (u2, 2 ) +
(u2,1 + u1, 2 ) +
(u2,3 + u3, 2 ) + b 2 = u&&2

x1
x3
x2

r

( u) + 2 (u3,3 ) +
(u3,1 + u1,3 ) +
(u3, 2 + u2,3 ) + b3 = &u&3

x
x
x2
1
3

NOTE 5: Wave equations


If we apply the divergence to the equation (5.50) we obtain:

r
r
r r
r
&r&
( + 2 ) [ ( u)] ( u) + b = u
1442443
=0
r
r
&r&
( + 2 ) [ ( u)] + b = u
r
r
&r&
( + 2 ) 2 ( u) + b = u

r
r
&r& = ( + 2 ) 2 ( u
) + b
u

r
(5.52)
r
r
( + 2 ) 2
D

(
)
(
)

u
u
b
Dt 2

r
D 2 ( + 2 ) 2

b
Dt 2

2
r
D

= 2 2 + b
2
Dt
r
&& = 2 2 + b
r r
r
where we have considered that = u and ( v ) = 0 (see Problem 1.106). In indicial

notation becomes:

&&i ,i
( + 2 )u j , jii ilq qjk u k , jli + bi , i = u
&&i ,i
( + 2 )u j , jii + bi , i = u
&&i ,i =
u

( + 2 )

D 2 u
2 i
Dt xi

u j , jii + bi ,i

( + 2 ) 2
=

xi xi

u j

x j

(5.53)

bi
+
xi

2
D 2
b
2
+ i

=
2
xi

x
Dt
i
i

where
=

( + 2 )

P-wave velocity

(5.54)
r

If the body forces do not change in space we have that b = 0 , thus the equation in
(5.52) becomes:
D 2
= 2 2
Dt 2

P- wave equation

(5.55)

P-waves have no rotation.


r
Now if we apply the curl ( ) to the equation (5.50) we obtain:
University of Castilla-La Mancha
Ciudad Real Spain

Draft

By: Eduardo W. V. Chaves (2015)

5 INTRODUCTION TO: CONSTITUTIVE EQUATIONS, IBVP, AND IBVP SOLUTION STRATEGIES

399

r
r r
r r
r r
r &r&
r
( + 2 ) [ ( u)] ( u) + b = u
r r
r r
r r
r &r&
( u) + b = u
r r
r r
r r
D2 r r
( u) + b = 2 ( u)
Dt
r
2
r
r
r r
r
D
( ) + b =
Dt 2
(5.56)
r
r
r
r r
r
r r
D 2
D 2
2 = ( )
( ) =

Dt 2
Dt
2r
r r r
D

= ( )
2

Dt
2r
r
r r
D

= 2 ( )
2
Dt
r
r r r
r r r
where we have considered = u , and that the b -field is conservative thus b = 0 .
r
r
r
r
Note that [ ( u)] = [ ] = 0 (see Problem 1.106), and

[
[

]
]

=
r

Shear wave velocity


r

(5.57)
r

Note that 2 = ( ) ( ) 2 = ( ) , since = ( u) = 0


r

holds for any vector, so ( ) = ( ( u)) = 0 , (see Problem 1.107). With that the
equation in (5.56) becomes:
r
r
D 2
= 2 2
2
Dt

Shear wave equation


(S-wave equation)

(5.58)

Shear waves have no change in volume.


In the case of = 0 the equation (5.55) becomes the acoustic wave equations:
D 2
= c 2 2
2
Dt

Acoustic wave equation

(5.59)

Speed of propagation

(5.60)

with

c=

Note that the displacement field was split up into: u = + where = 0 . We can
r
r r
r
r
prove this by means of the identity ( a) = ( a) 2a . If we consider the vectors
r r
r r
r
r
r
r
r
u = 2a and = a , and the scalar = a , we obtain u = + , with that we
obtain:
r r
r r r
r
r r
r
r r
r
u = + ( ) = and u = + ( ) = ( )
If we consider =

E
E
and =
we can obtain:
2(1 + )
(1 + )(1 2 )

( + 2 )

University of Castilla- La Mancha


Ciudad Real Spain

( + 2 )

E
E
+2
(2 2 )
(1 + )(1 2 )
2(1 + )
=
E
(1 2 )
2(1 + )

Draft

By: Eduardo W. V. Chaves (2015)

SOLVING PROBLEMS BY MEANS OF CONTINUUM MECHANICS

400

With that we conclude that the ratio of P- to S-wave velocities depends only on Poissons
ratio.

a)

b)

Figure 5.8: Displacement occurring from a harmonic plane P-wave (a) and S-wave (b). Pwave has no rotation and S-wave no volume change.
r

NOTE 6: In the previous note (NOTE 5) it was shown that the displacement field u can
r r
r
be split up into u = + , which is applied to any vector field, i.e. given a vector field
r
F , the following is true:

r
r r Helmholtz theorem or Helmholtz
F = +

(5.61)
decomposition
r
which is known as Helmholtz theorem, where is a scalar potential field and is a vector
r
r
r
potential field, in which the relationships div ( ) = 0 and ( ) rot ( ) = 0
r
r
hold. Note also that the SI units of [ ] = [ ] = m 2 , since [u] = m .
r r
r
Then, by substituting u = + , ( ui = ,i + ipq q, p ), into the Naviers equations given
by (5.49) we can obtain:
r
r
r
&r&
( + )[ ( u)] + [ (u)] + b = u
r
r r
r r
&r&
( + ) ( ( + )) + ( ( + )) + b = u

] [

r
r r
r r
&r&

( + ) ( ( )) + ( ( )) + ( ( )) + ( ( )) + b = u
1
4
2
4
3
1
42
43
1
4
4
2
4
4
3

r
=0
( 2 )
= ( ( ))

(5.62)

r
r
r r
r
( + 2 ) ( ( )) + ( 2 ) + b = && + &&
424
3
1
= 2

r
r
r r
r
( + 2 ) ( 2 ) + ( 2 ) + b = && + &&

] [

The above algebraic manipulations in indicial notation are given by:


&&i
( + )u j , ji + ui , jj + bi = u

&&i
( + )( , j + jpq q , p ), ji + ( ,i + ipq q , p ), jj + bi = u

(5.63)

&&i
( + )( , jji + jpq q , pji ) + ( ,ijj + ipq q , pjj ) + bi = u

Note that jpq q , pji = jpq q ,ijp = jpq q , ijp = pjq q , ijp = 0i , since
antisymmetric in jp and q ,ijp is symmetric in jp . Note also that

University of Castilla-La Mancha


Ciudad Real Spain

Draft

jpq = pjq

is

By: Eduardo W. V. Chaves (2015)

5 INTRODUCTION TO: CONSTITUTIVE EQUATIONS, IBVP, AND IBVP SOLUTION STRATEGIES

401

ipq q , pjj = ipq q , jjp = ipq ( q, jj ), p = ipq ([ 2 ]q ), p = ipq ([ 2 ]q ), p = [ ( 2 )]i


and , jji = ,ijj . With the above considerations the equation (5.63) becomes:
&& q , p
( + )( , jji + jpq q , pji ) + ( ,ijj + ipq q , pjj ) + bi = &&, i + ipq
&& q , p
( + 2 )( , jji ) + (ipq q , pjj ) + bi = &&, i + ipq

NOTE 7: Galerkin Vector


The displacement field can be expressed as follows:
r
r
r
u = c( 2g) ( g)

ui = cgi , pp g p , pi

(5.64)

where g is the Galerkin vector with the SI unit [g] = m3 , and c is a constant to be
determined which is dimensionless. If the Galerkin vector for a problem is known the
problem is solved.
Let us consider a static linear elastic problem, then the Naviers equations (5.49) can be
&&i = 0 i , and taking into account that
expressed as follows ( + )u j , ji + ui , jj + bi = u
displacement field (5.64) we can obtain:
( + )u j , ji + ui , jj + bi = 0 i
( + )(cg j , pp g p , pj ), ji + (cgi , pp g p , pi ), jj + bi = 0 i

(5.65)

( + )(cg j , ppji g p , pjji ) + (cgi , ppjj g p , pijj ) + bi = 0 i

Note that g j , ppji = g p , pjji = g p, pijj , then the above equation becomes:
( + )(cg j , ppji g p , pjji ) + (cgi , ppjj g p , pijj ) + bi = 0 i

(5.66)

[( + )(c 1) ]g p , pjji + cgi , ppjj + bi = 0 i

The constant c can be obtained by taking the term between brackets equal to zero, i.e.:
[( + )(c 1) ] = 0

c=

+ 2
= 2(1 )
+

(5.67)

Then, the displacement field (5.64) becomes:


r
r + 2 2 r
u=
( g) ( g)
+

+ 2
g g
+ i , pp p , pi
+ 2 2 (gi ) (g p , p )
=

xi
+ x p x p

ui =

r
r
+ 2
1
( 2 g)
( g)
= 2
2

2 ( + )

( g)
+ 2 2
(gi )
xi
+

(5.68)

r
+ 2
1 ( g)
2

= 2
(gi )
2 xi
2 ( + )

And the Naviers equation (5.66) in terms of Galerkin vector becomes

University of Castilla- La Mancha


Ciudad Real Spain

Draft

By: Eduardo W. V. Chaves (2015)

SOLVING PROBLEMS BY MEANS OF CONTINUUM MECHANICS

402

[( + )(c 1) ]g p , pjji + cgi , ppjj + bi = 0 i


144
42444
3

gi , ppjj =

=0

gi , ppjj =

2
x p x p

1
b i
c

( + )
b
( + 2 ) i
2 (gi )
= 2 2 (gi ) = 4 (gi ) = ( + ) bi

x j x j
( + 2 )

Thus
r
r
( + ) r
1
b =
b
4 (g) =
( + 2 )
2 (1 )

4 (gi ) =

( + )
1
b
=
( + 2 ) 2 (1 ) i

(5.69)

Note that in the absence of body force, each component of the Galerkin vector ( g i ) is
biharmonic function, i.e. 4 (g i ) g i ,kkjj = 0 i .
r

The infinitesimal strain tensor ( = (u) sym ) in terms of Galerkin vector becomes:
r + 2
r
r
r
+ 2 2 r
( g) ( g) =
u =
( 2g) [ ( g)]
+
+

r
+ 2 2 r
=
(g) [ ( g)]
+

r
r
r
r
Note that [ 2 (g)]sym = 2 [(g) sym ] , {[ ( g)]}sym = [ ( g)] . Then,
sym

r
r
r
+ 2 2 r
+ 2 2 r sym
(g) [ ( g)] =
[(g) ] [ ( g)]
= (u) sym =
+
+

r
r
+ 2
1
2 [(g) sym ]
[ ( g)]
= 2
2
2 ( + )

r
r
(1 ) 2
1
[(g) sym ]
[ ( g)]
= 2
2

(5.70)

In indicial notation we have:


ij =
=

+ 2

1 + 2
1
(ui , j + u j , i ) =
gi , pp g p , pi +
g j , pp g p , pj
2 +
2
,i
, j +

+ 2

1 + 2
gi , ppj g p , pij +
g j , ppi g p , pji

2 +
+

1
+ 2
+ 2
[
(
[(gi, j + g j,i )], pp g p, pij
gi , jpp + g j , ipp )] (g p , pji + g p , pij ) =
2( + )
2
2( + )
+ 2
[gi, j + g j ,i ], pp (g p, p ),ij
=
2( + )

The stress tensor field for isotropic linear elastic material ( = Tr()1 + 2 ) in terms of
Galerkin vector becomes:

University of Castilla-La Mancha


Ciudad Real Spain

Draft

By: Eduardo W. V. Chaves (2015)

5 INTRODUCTION TO: CONSTITUTIVE EQUATIONS, IBVP, AND IBVP SOLUTION STRATEGIES

403

+ 2
[
(gk , k + gk , k )], pp (g p, p ), kk = + 2 (gk , kpp + gk , kpp ) g p, pkk
2( + )
2( + )

+ 2

g
=
1gk , kpp =
(
)

+
k , kpp
+

Tr ( ) = kk =

Note that gk , kpp = g p , pkk . Then,


+ 2

gk , kpp ij + 2
gi , j + g j ,i
ij = Tr( ) ij + 2 ij =
+

2( + )

(
g
)

p
,
p
,
ij
, pp

+ 2
gk , kpp ij +
gi , j + gi , j ], pp (g p, p ),ij
[
2( + )
2( + )

= 2

= 2 gk , kpp ij + (1 ) gi , j + g j , i

, pp

(g p , p ), ij

(5.71)

The above equation in tensorial notation becomes:

r
r

+ 2 2 r sym
[ 2 ( g)] 1 +
(g) (( g))
( + )

2( + )

= 2

(5.72)

r
r
r
= 2 [ 2 ( g)] 1 + 2(1 ) 2 (g) sym ( ( g))

where we have considered that

E
2(1 + )(1 2 )
=
=
2( + ) 2(1 + )(1 2 )
E
+ 2 1 2(1 + )(1 2 ) E (1 )
=
= (1 )
E
2( + ) 2
(1 + )(1 2 )
NOTE 8: Loves Strain Function
The Loves strain function is a particular case of Galerkin vector in which
g1 = 0

g1 = 0

g1 = L

where L is the Loves strain function. With that, the equation in (5.69) becomes
4 ( L) =

( + )
1
b 3 =
b 3
( + 2 )
2 (1 )

(5.73)

where we have considered that b1 = b 2 = 0 . The Loves strain function can be applied to
axially symmetric problem.
Taking into account that
2 (g1 ) 0
r

, 2 (gi ) = 2 (g2 ) = 0 , the displacement field


g = gi , i = g1,1 + g2, 2 + g3,3 = L,3
x3
2 (g ) 2 L
3

(5.68) becomes:

University of Castilla- La Mancha


Ciudad Real Spain

Draft

By: Eduardo W. V. Chaves (2015)

SOLVING PROBLEMS BY MEANS OF CONTINUUM MECHANICS

404

r
+ 2
1 ( g)
2

(gi )
ui = 2
2 xi
2 ( + )

2L

x1x3

2L
ui =

x2 x3

+ 2 2
2L
L

x3x3
+

(5.74)

Problem 5.10
a) Obtain the stress field correspondent to the Galerkin vector:
r
g = 2 x14 e 1 + x24 e 2 + (8 x13 x3 4 x23 x3 )e 3
{
{
1442443
= g1

=g2

= g3

b) Obtain the body force density.


Solution:
The stress in terms of Galerkin vector is given by (5.71), i.e.:
ij = 2 { gk , kpp ij + (1 )[gi , j + g j ,i ], pp (g p , p ),ij }
r
r
r
= 2 { [ 2 ( g)] 1 + 2(1 ) 2 [(g) sym ] ( ( g))}ij

(5.75)

The gradient of the Galerkin vector is given by:


g1 g1 g1

0
0
x1 x2 x3 8 x1
r

g2 g2 g2
3

(g)ij = gi , j =
0
4 x2
0
=

x1 x2 x3
2
2
3
3

24 x1 x3 12 x2 x3 8 x1 4 x2
g
3 g3 g3
x1 x2 x3
r
r
Note that ( g) = Tr(g) = 8x13 + 4 x23 + (8 x13 4 x23 ) = 0
r
r
Applying the Laplacian to (g) we obtain 2 (g) , and in indicial notation becomes:
2 (g i , j )
r
[ (g)]ij = (g i , j ) ,kk =
= g i , jkk
x k x k
2

2 (g i , j )
x1x1

2 (g i , j )
x 2 x 2

48 x1
r sym
1
{ [(g) ]}ij = 0
2
48 x3

0
24 x2
12 x3

48x1

= 0
48x
3

0
24 x 2
24 x 3

48 x1

+ 0
48 x1 24 x2 0
0
0

24 x3

University of Castilla-La Mancha


Ciudad Real Spain

x 3 x 3

0
24 x2

48 x1

= 0
24 x
3

2 (g i , j )

48 x1 24 x 2
0

0
24 x2
0

48 x1 24 x2

48 x3
24 x3

48 x1 24 x2
24 x3
12 x3

Draft

By: Eduardo W. V. Chaves (2015)

5 INTRODUCTION TO: CONSTITUTIVE EQUATIONS, IBVP, AND IBVP SOLUTION STRATEGIES

405

Note that [ 2 (g)]sym = 2 [(g) sym ] . Then, the equation for stress (5.75) becomes:
r
r
r
r
ij = 2{ [ 2 ( g)] 1 + 2(1 ) 2 [(g) sym ] ( ( g))}ij = 2 { 2(1 ) 2 [(g) sym ]}ij

48 x1
r

ij = 2 { 2(1 ) 2 [(g) sym ]}ij = 2 2(1 ) 0

24 x3

96 x1

= 2 (1 ) 0
48 x3

0
48x2
24 x3

0
24 x2
12 x3

48 x1 24 x2

24 x3
12 x3

48(2 x1 + x2 )
48x3
24 x3

b) According to the equation in (5.69) the body force density ( b ) and the Galerkin vector
r
( g ) are related to each other by
2 (gi ) ( + )

=
b
x j x j ( + 2 ) i

( + 2 ) 4
( + 2 ) 4
(gi ) = 2 (1 ) 4 (gi )
b i =
(gi ) = 2

( + )
2
(
+
)

4 (gi ) =

2
xk xk

(5.76)

in which
g1 = 2 x14

g2 = x24

g3 = 8 x13 x3 4 x23 x3

2 (gi ) 2 (gi ) 2 (gi ) 2 (gi )


=
+
+
x j x j x1x1 x2x2 x3x3

i = 1

2 (gi )
= ai = i = 2
x j x j

i = 3

2 ( 2 x14 ) 2 (2 x14 ) 2 ( 2 x14 )


+
+
= 24 x12 = a1
x1x1
x2x2
x3x3
2 ( x24 ) 2 ( x24 ) 2 ( x24 )
+
+
= 12 x22 = a2
x1x1 x2x2 x3x3
2 (g3 ) 2 (g3 ) 2 (g3 )
+
+
= 48 x1 x3 24 x2 x3 = a3
x1x1 x2x2 x3x3

i = 1

2 2 (gi ) 2 (ai )
4
(gi ) =
=
= i = 2
xk xk x j x j xk xk

i = 3

2 (24 x12 ) 2 (24 x12 ) 2 ( 24 x12 )


+
+
= 48 x1
x1x1
x2 x2
x3x3
2 (12 x22 ) 2 (12 x22 ) 2 (12 x22 )
+
+
= 24 x2
x1x1
x2x2
x3x3
2 (a3 ) 2 (a3 ) 2 (a3 )
+
+
=0
x1x1 x2x2 x3x3

Then, the equation in (5.76) becomes:


48 x1

bi = 2 (1 ) 4 (gi ) = 2 (1 )24 x2
0

University of Castilla- La Mancha


Ciudad Real Spain

Draft

By: Eduardo W. V. Chaves (2015)

SOLVING PROBLEMS BY MEANS OF CONTINUUM MECHANICS

406

Problem 5.11
a) Show that:
r
r
xr ( xr )T = 0

qjk til ij , kl = 0 qt

Indicial

(5.77)

where ijk is the permutation symbol, and is the infinitesimal strain tensor.
b) Show also that:
ij , kl + kl ,ij il , jk jk ,il = O ijkl

(5.78)

c) Express the explicit form of the equations (5.77).


Solution:
u

u
1
1
j
The infinitesimal strain tensor is given by ij =
+ i = (u j ,i + u i , j ) , and if we

2 xi x j 2
r

take the derivative with respect to ( x ) we can obtain:


ij
x k

ij ,k =

1
(u j ,ik + u i , jk )
2

Note that u i , jk = u i ,kj is symmetric in jk , and if we multiply by the antisymmetric tensor in


jk , i.e. qjk = qkj , we can obtain: u i , jk qjk = 0 iq , thus
1
2

1
2

1
424
3
21

1
2

qjk ij ,k = (u j ,ik + u i , jk ) qjk = u j ,ik qjk + u i , jk qjk = u j ,ik qjk


= 0 iq

once again we take the derivative with respect to ( x ) and we can obtain:
( qjk ij ,k )
xl

= qjk ij ,kl =

1
u j ,ikl qjk
2

Note that u j ,ikl = u j ,kil = u j ,kli is symmetric in il and til = tli is antisymmetic in il . With
that, if we multiply both sides of the equation by til we obtain the equation in (5.77), i.e.:
1
2

til qjk ij ,kl = u j ,ikl til qjk = 0 jkt qjk = 0 qt

Q.E.D.
b) Now, if we multiply both sides of the above equation by tab qmn , we obtain:
tab qmn til qjk ij ,kl = 0 qt tab qmn = O abmn

Remember that the relationships tab til = ai bl al bi and qmn qjk = mj nk mk nj


hold, thus:
tab qmn til qjk ij , kl = O abmn
( ai bl al bi )( mj nk mk nj ) ij , kl = O abmn
( ai bl mj nk ai bl mk nj al bi mj nk + al bi mk nj )ij , kl = O abmn

Then we can obtain am, nb an , mb bm, na + bn , ma = O abmn , which is the same as:
am,bn + bn,am an,mb mb,an = O ambn

University of Castilla-La Mancha


Ciudad Real Spain

Draft

Q.E.D.

By: Eduardo W. V. Chaves (2015)

5 INTRODUCTION TO: CONSTITUTIVE EQUATIONS, IBVP, AND IBVP SOLUTION STRATEGIES

407

Note that, if we multiply the above equation by bn we can obtain:


am,bn bn + bn,am bn an ,mb bn mb,an bn = O ambn bn
am,bb + bb,am ab,bm mb,ba = 0ambb

[ xr ( xr )]am + [ xr [ xr [Tr ( )] ] ]am [ xr ( xr )]am [ xr ( xr )]ma = 0ambb


[ xr ( xr )]am + [ xr [ xr [Tr ( )] ] ]am = [ xr ( xr )]am + [ xr ( xr )]ma

[ ]

2xr am + [ xr [ xr [Tr ( )] ] ]am = [ xr ( xr )]am + [ xr ( xr )]ma

which in tensorial notation becomes:


xr ( xr ) + xr [ xr [Tr ( )] ] = xr ( xr ) + [ xr ( xr )] = 2[ xr ( xr )]
T

sym

(5.79)

c) Note that the equations in (5.77) have 6 independent equations since 0 qt is symmetric.
For the case when q = 1, t = 1 we can obtain 1 jk 1il ij ,kl and by expanding the index l we
obtain:
1 jk 1il ij ,kl = 1 jk 1i1 ij , k1 + 1 jk 1i 2 ij ,k 2 + 1 jk 1i 3 ij ,k 3 = 1 jk 1i 2 ij ,k 2 + 1 jk 1i 3 ij ,k 3
Expanding the index i the above equation becomes:
1 jk 1il ij ,kl = 1 jk 1i 2 ij ,k 2 + 1 jk 1i 3 ij ,k 3 = 1 jk 132 3 j , k 2 + 1 jk 123 2 j ,k 3 = 1 jk 3 j , k 2 + 1 jk 2 j ,k 3
and by expanding the remaining indices we can obtain:

1 jk 1il ij ,kl = 1 jk 3 j ,k 2 + 1 jk 2 j , k 3 = 123 32,32 132 33, 22 + 123 22,33 + 132 23, 23
= 32,32 + 33, 22 + 22,33 23, 23 = 33, 22 + 22,33 2 23, 23 = 0
=

2 33
x 22

2 22
x32

2 23
2
=0
x 2 x3

note that 23, 23 = 32,32 .


We leave the reader with the following demonstrations:
when q = 2, t = 2

2 jk 2il ij ,kl = 31,31 + 33,11 + 11,33 13,13 = 33,11 + 11,33 213,13 = 0


=

2 33
x12

2 11
x32

2 13
=0
x1x3

when q = 3, t = 3

3 jk 3il ij ,kl = 11, 22 12,12 21, 21 + 22,11 = 11, 22 + 22,11 212,12 = 0


=

2 11
x 22

2 22
x12

2 12
=0
x1 x 2

when q = 1, t = 2

1 jk 2il ij , kl = 12,33 + 13, 23 + 32,31 33, 21 = 13, 23 + 23,13 33,12 12,33 = 0


=

23 13 12 2 33
2 12
2 33
2 23
2 13

=0

+
=

+
x2 x3 x1x2
x2 x3 x1x3 x1x2 x3x3 x3 x1

when q = 2, t = 3
University of Castilla- La Mancha
Ciudad Real Spain

Draft

By: Eduardo W. V. Chaves (2015)

SOLVING PROBLEMS BY MEANS OF CONTINUUM MECHANICS

408

2 jk 3il ij ,kl = 11,32 + 13,12 + 21,31 23,11 = 13,12 + 12,13 23,11 11, 23 = 0
=

2 23
2 13
2 11
2 12

+
x1 x 2 x1 x3 x1x1 x 2 x 3 x1

13 12 23

+
x1
x3
x 2

2 11

=0
x 2 x 3

and when q = 1, t = 3
1 jk 3il ij ,kl = 12,32 13, 22 22,31 + 23, 21 = 12, 23 13, 22 22,13 + 23,12 = 0
=

2 23
2 13
2 22
2 12

=
+

x 2 x3 x 2 x 2 x1x 3 x1x 2 x 2

12 13 23

x1
x 2
x3

2 22

=0
x1x3

By regrouping the previous 6 equations we can obtain:

2 33 2 22
2 23
S
=
+

2
=0
11
x 2 x3
x 22
x32

2
2
2
S 22 = 33 + 11 2 13 = 0

x1 x3
x12
x32

2 11 2 22
2 12

S
=
+

2
=0
33
x1 x 2
x 22
x12

2
S = 23 + 13 12 33 = 0
12

x3 x1
x 2
x3 x1x 2

23 13 12 2 11

=0
+
+
S 23 =
x1 x1
x 2
x 3 x 2 x3

2
S = 23 13 + 12 22 = 0
13

x 2 x1
x 2
x3 x1 x3

Compatibility equations for (3D)

(5.80)

The above equations in Voigt notation become:

2
S11 x 2
3
S
2

22

S 33 x 22
=

S12 0
S 23

2
S13

x 2 x 3

2
x32
0
2
x12

2
x 22
2
x12
0

2
x1x 2

2
x1x3

2
x 2 x3

2
x1x 2
2
12 2
x3
2
1
2
x1 x3
2
1
2
x 2 x3

0
2
x1x3
2
12 2
x1
2
1
2
x1x 2
1
2

x1x3 11 0


22 0
0
0
33 =
2
212 0

2
x 2 x 3 2 23 0


2 2 13 0
1

2
x1 x 2
2
12 2
x 2
0

(5.81)

{S} = [ L( 2 ) ] { } = {0}

NOTE 1: The equations in (5.80) are known as the compatibility equations. The
compatibility equations guarantee that the displacement field is unique and continuous (see
Figure 5.9). In other words, the 6 components of the strain tensor are not independent and
cannot be arbitrary.

University of Castilla-La Mancha


Ciudad Real Spain

Draft

By: Eduardo W. V. Chaves (2015)

5 INTRODUCTION TO: CONSTITUTIVE EQUATIONS, IBVP, AND IBVP SOLUTION STRATEGIES

409

(Current configuration?)
2

2
1

6
7

(Reference configuration)

The compatibility equations are


not satisfied
1

The compatibility
equations are satisfied

3
6
9

2
5
8

4
7

(Current configuration)

Figure 5.9
NOTE 2: When using numerical method for obtaining the solution, e.g. finite element
method, the way to ensure the compatibility equations is by means of the continuity of the
displacement field. With regards the finite element method, when we assembly the
elements (tie nodes), in general, we are ensuring that the compatibility equations are
satisfied.
r

NOTE 3: When the displacement field is independent of one direction, e.g. u = u( x1 , x 2 ) ,


the compatibility equations reduce to:
S 33 =

2 11
x 22

2 22
x12

2 12
=0
x1x 2

Compatibility
equations for (2D)

(5.82)

since i 3 = 3i = 0 . The above equation in Engineering notation becomes:


Sz =

2 x
y 2

2 y
x 2

2 xy
xy

=0

Compatibility
equations for 2D
(Engineering notation)

(5.83)

NOTE 4: To understand the compatibility condition let us consider an example in two


dimensional case (2D), where we have the scalar field = ( x1 , x2 ) and we know the

= x1 + 3x2 and
= x12 , we can see clearly that this scalar field is
x1
x2

following derivatives:
incompatible since

= x1 + 3x 2 = F1
x1

= x12 = F2
x 2


( x1 + 3 x 2 )
2

=
=
= 3
x 2
2
2
x1 x 2 x1

x 2 x1 x1x 2
( x12 )
2

1
442443

=
=
= 2 x1

incompatible
x1 x 2 x1x 2
x 2

x 2

The scalar field = ( x1 , x2 ) will be compatible if and only if:

= F1 ( x1 , x2 )
x1
compatible iff F1 F2
=

x2 x1
= F2 ( x1 , x2 )

x2

University of Castilla- La Mancha


Ciudad Real Spain

Draft

(5.84)

By: Eduardo W. V. Chaves (2015)

SOLVING PROBLEMS BY MEANS OF CONTINUUM MECHANICS

410

If we consider the Greens theorem (see Chapter 1 in the textbook) which states:
r r
r
r
F
F
components

F d = (

F) e 3 dS F1dx1 + F2 dx2 = 2 1 dS3

x1 x2
r
and also considering the equation in (5.84), we conclude that: if F = xr , is compatible
r r
r
r r
r
r
if and only if F d = ( xr F) e 3dS = 0 xr F = 0 .

r
x

d = d p

r
dS = dSe 3

x2

x3

e 3

x1

Figure 5.10: Greens theorem.


r

NOTE 5: Let us consider that F = ( xr ) a = a ( xr )T , where is a second-order


r
r
tensor field and a is an arbitrary vector independent of x (constant). Note also that the
following relations are true:
r
r r
r
r r
r
r
r
r r
(a) F d = ( xr ) a d = a ( xr )T d = a ( xr )T d

(b)

r
r
{ [(

r
r
r
( xr F) dS =

and

r
x

r
x

]}

r
r
) a dS =

r
r r
= a xr ( xr )T

r
r r
{ [a (
r
x

r r
dS = a

r
{

r
x

)T

]} dSr

r
( xr )T

r
x

} dS
T

In indicial notation becomes


r
r
r
r
r
r
r
(a) Fi (d )i = ( xr )ij a j (d )i = a j ( xr )ij (d )i = a j ( xr )ij (d )i

(b)

[ [

r
r
r
r
r
( xr F) i (dS ) i = ijk Fk , j (dS ) i = ijk a p ( xr )T

] ]

kp , j

r
(dS ) i

r
r
r
= ijk a p , j ( xr )T kp + a p ( xr )T kp , j (dS ) i
{

=0 pj

r
r
r
r
= ijk a p ( xr )T kp , j (dS ) i = a p ijk ( xr )T kp , j (dS ) i

University of Castilla-La Mancha


Ciudad Real Spain

r
x

= a p ijk psq qk ,s
= ap

,j

r
r
(dS ) i = a p ijk psq qk ,sj (dS ) i

r
( xr )T

Draft

ip

r
r
( dS ) i = a

r
x

r
( xr )T

} dS
T

By: Eduardo W. V. Chaves (2015)

5 INTRODUCTION TO: CONSTITUTIVE EQUATIONS, IBVP, AND IBVP SOLUTION STRATEGIES

411

It would be worth to review Problem 1.109, in which we have shown that the following
r
r
relationship ( xr ) = ksq qp , s e k e p holds, thus ( xr )T = psq qk , s e k e p . Also in
r

Problem 1.109 we have shown that xr ( xr )T = ipq tsj qj , ps e t e i , which is


r

equivalent to xr ( xr )T = psq ijk qk ,sj e i e p .


And by consider the Stokes Theorem (see Chapter 1 in the textbook) we conclude:
r r
r
r
r

F d = (

r
x

F ) dS

14444244443

r
a

r
r
r
r
r
( xr )T d = a xr ( xr )T

} dS

r
r
( xr )T d =

r
r
xr ( xr )T

} dS

Then, for a compatible field it must fulfill:

{
r

r
x

r
( xr )T

=0

r
r
xr ( xr )T = 0

r
r
r
x
Now let us consider that A = F a where F is the gradient deformation, F = r , and by
X

apply the Stokes theorem we can obtain:


r r
r
r
r

A d = ( xr A ) dS

r
r
r
r
r
( F a) d = ( xr ( F a)) dS

1444444444444442444444444444443

r r
r
a F T d = a

{
r

r
x

} dS

(5.85)

{
r

r
x

} dS
T

Then, for a compatible field it must fulfill:

{
r

r
x

=0

r
xr F = 0

More detail about these algebraic manipulations is provided in Problem 1.109.


NOTE 6: Note that if:

T d = {
r

r
x

TT

} dS
T

(5.86)

Then, for a compatible field it must fulfill:

{
r

r
x

TT

=0

r
xr T T = 0

And if we use indicial notation we can obtain:

r
xr T T = ipqT jq , p e i e j = 0ij e i e j

By multiply by ikt we can obtain:


ipq iktT jq , p = ikt 0ij = 0 ktj

University of Castilla- La Mancha


Ciudad Real Spain

Draft

By: Eduardo W. V. Chaves (2015)

SOLVING PROBLEMS BY MEANS OF CONTINUUM MECHANICS

412

Considering ipq ikt = ( pk qt pt qk ) the above equation can be written as follows:


ipq ikt T jq , p = 0 ktj
T jq , p ( pk qt pt qk ) = 0 ktj

(5.87)

T jt ,k T jk ,t = 0 ktj

In other words, the above equation is a necessary and sufficient condition that the
r
integrands of T d be exact differentials.

NOTE 7: In this note we will demonstrate the compatibility equations, for small
deformation regime, using the demonstration described by E. Cesro (see Sokolnikoff
(1956), Love(1944)).
r

Let us consider the material point P 0 ( x 0 ) in which the displacement ui0 ( x 0 ) and the
r

infinitesimal spin tensor ij0 ( x 0 ) are known. Next we will determine the displacement at
r
any other point P ( x ) in terms of , (see Figure 5.11).

at point P 0 :
P0

r r
r
u( x 0 ) u0 - displacement
r
( x 0 ) 0 - spin tensor
x2

r
x

r
x0

at point P :

r r
r
u( x ) u - displacement

x1

x3

Figure 5.11.
Consider the displacement differential element
r
r
r
r
r
r
r
r
r
du = ( xr u) dx = (( xr u) sym + ( xr u) skew ) dx = ( + ) dx = dx + dx ,

which in indicial notation becomes dui = ij dx j + ij dx j , and by integrating it along the


path ( P 0 P ) we can obtain:
P

dui = ij dx j + ij dx j

P0

P0

ui

P
P0

P0

P0
P

P0
P

P0
P

P0

P0

r
r
ui ( x ) ui0 ( x 0 ) = ij dx j + ij dx j

= ij dx j + ij dx j
P0

(5.88)

r
r
ui ( x ) = ui0 ( x 0 ) + ij dx j + ij dx j

Note that
University of Castilla-La Mancha
Ciudad Real Spain

Draft

By: Eduardo W. V. Chaves (2015)

5 INTRODUCTION TO: CONSTITUTIVE EQUATIONS, IBVP, AND IBVP SOLUTION STRATEGIES


P

ij dx j = ij jk dxk = ij

( x j x j )
xk

dxk = [ ij ( x j xj )]

P
P

ij

413

( x j xj )dxk

where we have applied the integration by parts. The above equation can also be expressed
as follows:
P

ij dx j

= [ ij ( x j xj )]

P0

P
P0

ij

P0

( x j xj )dxk

k
P

r
r
= [ij ( x )( xj xj )] [ ij0 ( x 0 )( x 0j xj )] ij ,k ( x j x j )dxk

P0
P

[ ij0 ( x 0j

x j )] ij ,k ( x j xj )dxk
P0

Then, the equation (5.88) can be rewritten as follows:


P

P0

P0

r
r
ui ( x ) = u i0 ( x 0 ) + ij dx j + ij dx j

P0

P0

r
r
ui ( x ) = u i0 ( x 0 ) + ij dx j [ ij0 ( x 0j xj )] ij ,k ( x j xj ) dxk

(5.89)

r
Considering that is the axial vector associated with the antisymmetric tensor the
following is true ij = q qij = q iqj , then ij ,k = q ,k qij = q ,k iqj .

We can prove that

q
xk

r
= [ xr ]qk , (see Problem 5.12, Eq. (5.100)). Then, we can say

that

r
ij ,k ( x j xj )dxk = q ,k iqj ( x j xj )dxk = [ xr ]qk iqj ( x j xj )dxk
r
r r
= {[ xr ]T ( x x )}ki dxk
r
r r
r
= {{[ xr ]T ( x x )}T dx}i

(5.90)

Then, the equation (5.89) becomes


P

P0

P0

r
r
ui ( x ) = u i0 ( x 0 ) + ij dx j [ ij0 ( x 0j xj )] ij ,k ( x j x j )dxk

P0

P0

r
r
r
r r
r
ui ( x ) = u i0 ( x 0 ) + ij dx j [ ij0 ( x 0j xj )] {{[ xr ]T ( x x )}T dx}i

(5.91)

or in tensorial notation:

P
r
r
r0 P
r
r0 r
r r
r
0
u = u + dx [ ( x x )] {[ xr ]T ( x x )}T dx , thus

P0

P0

P
r
r
r
r
r
r r
r
u = u0 [ 0 ( x 0 x )] + [ {[ xr ]T ( x x )}T ] dx

(5.92)

P0

Note that the line integral (from P 0 to P ) must be path-independent, hence the line
integral vanish to a closed path, i.e.:

University of Castilla- La Mancha


Ciudad Real Spain

Draft

By: Eduardo W. V. Chaves (2015)

SOLVING PROBLEMS BY MEANS OF CONTINUUM MECHANICS

414

r r
r
r r
[ {[ xr ]T ( x x )}T ] d = 0

(5.93)

r r
r
r r
[ T {[ xr ]T ( x x )}]T d = 0

And by applying the Stokes theorem (see equation (5.85)), we conclude that:
r
r
r
r
T
r r T r
r r
T
T
T
T

[ {[ xr ] ( x x )}]

r
x

} dS = 0

[ {[ xr ] ( x x )}]

(5.94)

In Problem 1.114 we have proven that

r
r
r
r
r
r r
xr {[ xr ]T x} = { xr [ xr ]T } x + [ xr ]

Then, taking into account the above equation and T = , the equation (5.94) becomes:
r
{

r r
r
T
r r
[ T {[ xr ]T ( x x )}] dS = 0

r
{
[

r
x

r
x

r
r
r r
T ] [ xr {[ xr ]T ( x x )}]

dS = 0

r
r
r
r
r r
[ xr ] [{ xr [ xr ]T } ( x x ) + [ xr ]]

r
{

r
x

r
r r
[ xr ]T } ( x x )

r r
dS = 0

(5.95)

dS = 0

with that we can conclude that

{ {r

r
r r T
[ xr ]T } ( x x )
=0
r
r
r
r
{ xr [ xr ]T } ( x x ) = 0

r
x

Since the vector ( x x ) must be arbitrary we can conclude that xr [ xr ]T = 0 .


Note that, if we take into account that

q
xk

1
qst sk tk
2
xs
xt

(see equation in (5.100)),

the equation in (5.90) can be rewritten as follows:


ij ,k ( x j xj )dxk = q ,k iqj ( x j xj )dxk =

1
qst sk tk
xs
2
xt

iqj ( x j xj )dxk

1
qst qij ( sk ,t tk ,s )( x j xj )dxk
2
1
= ( si tj sj ti )( sk ,t tk ,s )( x j xj )dxk
2
1
= ( si tj sk ,t si tj tk ,s sj ti sk ,t + sj ti tk ,s )( x j xj )dxk
2
1
= ( ik , j jk ,i jk ,i + ik , j )( x j xj )dxk
2
= (ik , j jk ,i )( x j xj )dxk
=

(5.96)

Then, the equation in (5.89) can also be written as follows:

University of Castilla-La Mancha


Ciudad Real Spain

Draft

By: Eduardo W. V. Chaves (2015)

5 INTRODUCTION TO: CONSTITUTIVE EQUATIONS, IBVP, AND IBVP SOLUTION STRATEGIES


P

P0

P0

415

r
r
ui ( x ) = ui0 ( x 0 ) + ij dx j [ ij0 ( x 0j xj )] ij ,k ( x j xj )dxk

P0
P

P0

r
r
ui ( x ) = ui0 ( x 0 ) [ ij0 ( x 0j xj )] + ik dxk ( ik , j jk ,i )( x j xj )dxk

r
r
ui ( x ) = ui0 ( x 0 ) [ ij0 ( x 0j xj )] + [ ik ( ik , j jk ,i )( x j xj )]dxk

(5.97)

P0
P

r
r
ui ( x ) = ui0 ( x 0 ) [ ij0 ( x 0j xj )] + Tik dxk

P0

where Tik = [ik ( ik , j jk ,i )( x j xj )] . Since the displacement must be independent of the


path of integration, the integrands Tik dxk must be exact differentials (see equation (5.87)).
Hence, applying a necessary and sufficient condition that the integrands be exact
differentials we can obtain T jt ,k T jk ,t = 0 ktj , in which:
T jt = [ jt ( jt , p pt , j )( x p xp )] and T jk = [ jk ( jk , p pk , j )( x p xp )] ,

thus
T jt ,k T jk ,t = 0 ktj
[ jt ( jt , p pt , j )( x p xp )],k [ jk ( jk , p pk , j )( x p xp )],t = 0 ktj
jt ,k ( jt , p pt , j ) ,k ( x p xp ) ( jt , p pt , j )( x p xp ) ,k
jk ,t + ( jk , p pk , j ) ,t ( x p xp ) + ( jk , p pk , j )( x p xp ) ,t = 0 ktj
jt ,k ( jt , pk pt , jk )( x p xp ) ( jt , p pt , j )

pk

jk ,t + ( jk , pt pk , jt )( x p xp ) + ( jk , p pk , j )
jt ,k ( jt , p pt , j )

pk

jk ,t + ( jk , p pk , j )

pt

= 0 ktj

pt

+ ( jk , pt pk , jt jt , pk + pt , jk )( x p xp ) = 0 ktj
jt ,k jt ,k + kt , j jk ,t + jk ,t tk , j
+ ( jk , pt pk , jt jt , pk + pt , jk )( x p xp ) = 0 ktj

with that we can obtain:


( jk , pt pk , jt jt , pk + pt , jk )( x p xp ) = 0 ktj

(5.98)

Since the vector ( x p xp ) is arbitrary we can conclude that:


jk , pt pk , jt jt , pk + pt , jk = 0 jkpt

(5.99)

which matches the equation in (5.78).


Problem 5.12

Given the infinitesimal strain tensor , and the displacement field u , (a) show that:
11
r
u i
= 12 + 3
( J ) ij ( xr u) ij =
x j
13 2

12 3
22
23 + 1

13 + 2
23 1
33

where i are the rotation vector components.


University of Castilla- La Mancha
Ciudad Real Spain

Draft

By: Eduardo W. V. Chaves (2015)

SOLVING PROBLEMS BY MEANS OF CONTINUUM MECHANICS

416

b) Show also that:


k
x p

r
r
1
=
[ xr ]kp [ xr
2

23 13 12 23 22


x3
x3 x2 x3 x2
31 11 13 12 23

=
x1
x1 x3
x3 x3

12
11 22 12
12

x3 x x x x
2
1
2
1

1
1
kij ij , p = kij ip jp
x j
2
2
xi

23

x1

= 31
x
1
12
x1

23
x2
31
x2
12
x2

r
]kp = [ xr ]kp
33 23

x
x

3
2
13 33

x1
x3
23 13

x2
x1

(5.100)
r
k
where ( xr ) kp =
, and is the infinitesimal spin tensor. And the relationship
x p
r
x = xr holds, (see Problem 1.109).

r
r

Solution:
r

a) The displacement gradient J xr u can be split additively into a symmetric and an


antisymmetric part:
r

J xr u =

] [

r
r
r
r
r
r
1
1
( xr u) + ( xr u) T + ( xr u) ( xr u) T = ( xr u) sym + ( xr u) skew = +
3 14243
2 44424443 1
2 44424443 1424
1
r
= ( xr u) sym

r
= ( xr u) skew

where the symmetric part = ( xr u) sym represents the infinitesimal strain tensor, and the
r
antisymmetric part = ( xr u) skew represents the infinitesimal spin tensor (rotation tensor).
r
If we consider that is the axial vector associated with the antisymmetric tensor we
conclude that:
0 12
ij = 21 0
31 32

13 0
23 = 12
0 13

12
0

23

13 0
23 = 3
0 2

3
0

2
1
0

with that
11
r
u i
= 12
( xr u) ij =
x j
13

12
22
23

13 0
23 + 3
33 2

3
0

12 3 13 + 2
2 11

1 = 12 + 3
22
23 1
33
0 13 2 23 + 1

b) Recall from chapter on Tensors that an antisymmetric tensor ( ) and its axial vector
r
( ) are related to each other, in indicial notation, by means of ij = k kij or
r
1
k = kij ij . If we use the last equation to obtain the gradient of we can obtain:
2

k , p = kij ij = kij ij , p
1
2

, p

1
2

By expand the dummy indices i, j , we can obtain the following terms:


1
2

1
2

k , p = kij ij , p = ( k1212, p + k1313, p + k 21 21, p + k 23 23, p + k 3131, p + k 3232, p )

University of Castilla-La Mancha


Ciudad Real Spain

Draft

By: Eduardo W. V. Chaves (2015)

5 INTRODUCTION TO: CONSTITUTIVE EQUATIONS, IBVP, AND IBVP SOLUTION STRATEGIES

417

Note that the rows of k , p ( k = 1,2,3 ) are given by:

k, p =

1
kij ij , p
2

1
1

(k = 1) 1, p = 2 (123 23, p + 13232, p ) = 2 ( 23, p 32, p ) = 23, p

1
1

( 21313, p + 23131, p ) =
(13, p + 31, p ) = 31, p
(k = 2) 2, p =
2
2

1
1

(k = 3) 3, p = 2 (312 12, p + 321 21, p ) = 2 (12, p 21, p ) = 12, p

where we have used the antisymmetric tensor definition ij = ji . Taking into account
the above equation we can conclude that:

k, p

23,1 23, 2
1

kij ij , p = 31,1 31, 2


=
2
12,1 12, 2

23

23,3
x1

31,3 = 31
x1

12,3
12
x1

23
x2
31
x2
12
x2

23

x3
31
x3
12
x3

(5.101)

To obtain the derivative of ij with respect to x p we will start from the definition

r
ij = ( xr u) skew

ij

1 ui u j

2 x j xi
ij

1
= (ui , j u j ,i ) , thus:
2

ij , p =

x p

1
1
(ui , j u j ,i ), p = (ui , jp u j ,ip )
2
2

The value of the above equation is not altered if we add and subtract the term

1
u p,ij , thus:
2

1
1
1
(ui , jp u j ,ip + u p ,ij u p , ij ) = (ui , jp + u p , ij ) (u j , ip + u p ,ij )
2
2
2
1
1
1

= (ui , pj + u p ,ij ) (u j , pi + u p , ji ) = (ui , p + u p ,i ) (u j , p + u p , j )


2
2
2
2
, i
, j

ij , p =

= ip , j jp ,i =

ip
x j

jp
xi

Substituting the above equation into the equation (5.101) and by expanding the dummy
indices i, j we can obtain:
1
1
kij ( ip , j jp ,i ) =
( kij ip , j kij jp ,i )
2
2
1
=
( k12 1 p ,1 + k13 1 p ,3 + k 21 2 p ,1 + k 23 2 p ,3 + k 31 3 p ,1 + k 32 3 p , 2
2
k 12 2 p ,1 k13 3 p ,1 k 21 1 p , 2 k 23 3 p , 2 k 31 1 p ,3 k 32 2 p ,3 )

k, p =

Note that the rows of k , p ( k = 1,2,3 ) can be obtained as follows:

University of Castilla- La Mancha


Ciudad Real Spain

Draft

By: Eduardo W. V. Chaves (2015)

SOLVING PROBLEMS BY MEANS OF CONTINUUM MECHANICS

418

k, p

(k = 1) 1, p = 2 ( 123 2 p ,3 + 132 3 p , 2 123 3 p , 2 132 2 p ,3 ) = 3 p , 2 2 p ,3

( k13 1 p ,3 + k 31 3 p ,1 + k13 3 p ,1 k 31 1 p ,3 ) = 1 p ,3 3 p ,1
= (k = 2) 2, p =
2

(k = 3) 3, p = 2 ( 312 1 p ,1 + 321 2 p ,1 312 2 p ,1 321 1 p , 2 ) = 2 p ,1 1 p , 2

Then:

k, p

( 31, 2 21,3 ) ( 32, 2 22,3 ) ( 33, 2 23,3 )

= (11,3 31,1 ) (12,3 32,1 ) (13,3 33,1 )


( 21,1 11, 2 ) ( 22,1 12, 2 ) ( 23,1 13, 2 )

(5.102)

Then, taking into account the equation (5.101) and (5.102) we can conclude that:

k,p =

1
1
kij ij , p = kij ip jp
2
2
xi
x j

23

x1

= 31
x
1
12
x1

23 13 12 23 22 33 23


x3 x2
x3
x3 x2 x3 x2
31 11 13 12 23 13 33

x1 x3
x1 x3
x1
x3 x3

12



12 11 22 12 23 13

x3
x2 x1
x2
x1 x2 x1

23
x2
31
x2
12
x2

where we have used the symmetry of , i.e. ij = ji .


NOTE: Let us suppose that we know the infinitesimal strain tensor which is given by
11
ij = 12
13

12
22
23

8 x1
13
x2
23 =
2
33 3 2
2 x1 x 3

x2
2
x1
0

3 2
x1 x 3
2

3
x1

(5.103)

with the following boundary conditions:


3t
r r
u i ( x = 0, t ) = 0
0

and

r r
( x = 0, t ) = 0

r r

( x = 0, t ) = 0

To this example, we can obtain:

k, p

13 12 23 22 33 23


x 3 0
x 3 x 2
x3 x 2
x 2




= 11 13 12 23 13 33 = x1 x 3
x1
x1 x 3
x1 x 3
x 3

0


12 11 22 12 23 13
x 2
x 2 x1
x 2 x1
x1

0
0
3
2

3 2
x1
2

Note also that the following holds:

University of Castilla-La Mancha


Ciudad Real Spain

Draft

By: Eduardo W. V. Chaves (2015)

5 INTRODUCTION TO: CONSTITUTIVE EQUATIONS, IBVP, AND IBVP SOLUTION STRATEGIES

k, p

x1

= 2
x
1

3
x1

1
x 2
2
x 2
3
x 2

1

x 3 0
2
= x1 x 3
x 3

3 0

x 3

419

3 2
x1
2

0
3
2

with that and by means of the integration we can obtain the components i :

1
= 0
x1

1
= 0 1 = C1 (t )
x 2

1
= 0

x3

2
= x1 x3
x1

2
3 2
=0
x1 x 3 + C 2 (t )
2 =
x 2
2

2 3 2
=
x1

x 3
2

3
=0
x1

3 3
3
= 3 = x 2 + C 3 (t )
2
2
x 2

3
=0

x 3
r
r r r
By applying the boundary condition ( x = 0, t ) = 0 , we can conclude that C i (t ) = 0 . Then:

i = 2 = x12 x3
2

3 3

x2

And the infinitesimal spin tensor becomes:


0 12
ij = 21 0
31 32

13 0
23 = 3
0 2

3
0

0
2
3
1 = x2
2
0 3 2
x x
2 1 3

3
x2
2
0
0

3 2
x1 x3
2

The displacement field can be obtained by means of


11 12
ui
= 12 22
x j
13 23

8 x1
x
2
=
2
3 x2 x
2 1 3

13 0 12
23 + 21 0
33 31 32
x2
2
x1
0

University of Castilla- La Mancha


Ciudad Real Spain

13
23
0

3 2
x1 x3 0
2
3
0 + x2
2
3
3 x2 x
x1
2 1 3
Draft

3
x2
2
0
0

3 2
x1 x3
2
8x
1
0 = x2
2
3x1 x3
0

2 x2
x1
0

0
0
x13

By: Eduardo W. V. Chaves (2015)

SOLVING PROBLEMS BY MEANS OF CONTINUUM MECHANICS

420

or
u1

x1
u i u 2
=
x j x1

u 3
x1

u1
x 2
u 2
x 2
u 3
x 2

u1

x 3 8 x
1
u 2
x
=
2
x 3 2
3x1 x 3
u 3
x 3

2x2
x1
0

0
0
x13

with that we can obtain

u1
= 8 x1
x1

u1
= 2 x 2 u1 = 4 x12 x 22 + K 1 (t )
x 2

u1
=0

x 3

u 2
= x2
x1

u 2
= x1 u 2 = x1 x 2 + K 2 (t )
x 2

u 2
=0

x 3

u 3
= 3 x12 x 3
x1

u 3

3
=0
u 3 = x1 x3 + K 3 (t )
x 2

u1
= x13

x 3

The constants of integration can be obtained by means of the boundary condition:


4 x12 x 22 + K 1 (t )
r

u i ( x , t ) = x1 x 2 + K 2 (t )
x 3 x + K (t )
3
1 3

r r
x =0

K 1 (t ) 3t
r r
u i ( x = 0, t ) = K 2 (t ) = 0
K 3 (t ) 0

Then, the displacement field becomes:


4 x12 x 22 + 3t
r

u i ( x, t ) =
x1 x 2

x
x
1 3

It is interesting to verify that the displacement field is compatible, since the infinitesimal
strain tensor field, (see Eq. (5.103)), fulfills the compatible equations (see equations in
(5.80)). We leave the reader to verify this fact.
Problem 5.13
Consider a cantilever beam in which the infinitesimal strain tensor is given by
x2 x 3
ik ( x1 , x3 ) = 0
0

0 0
0 0
0 0

= x 2 x 3 e 1 e 1

(5.104)

where x2 = x2 ( x1 ) is the curvature of the beam which is constant on cross section, (see
Figure 5.12). a) Check whether the compatibility equations are fulfilled or not. b) Obtain
the displacement field.

University of Castilla-La Mancha


Ciudad Real Spain

Draft

By: Eduardo W. V. Chaves (2015)

5 INTRODUCTION TO: CONSTITUTIVE EQUATIONS, IBVP, AND IBVP SOLUTION STRATEGIES

x3

r
r
x0 = 0

421

11 = x2 x3

x2

cross section
x1

u3 ( x1 = 0)
=0
x1

M x2
x1

Figure 5.12: Cantilever beam.


Solution:
a) We can use the compatibility equations in (5.80) to verify that:
2 33 2 22
2 23
+

=0 X
2
2
x2 x3
x32
x2
2
2
2
2 ( x2 x3 )
( x2 )
33 + 11 2 13 =
=
=0 X
2
2
2
x1
x1x3
x3
x3
x3

2 ( x2 x3 )
2 11 2 22
2 12
+

=
=0 X
2
2
x1x2
x12
x22
x2

2
23 + 13 12 33 = 0 X
x x
x2
x3 x1x2
3 1

2 ( x2 x3 )
( x2 )

2 11
23 + 13 + 12
=
=
=0 X

x2 x3
x2
x2
x3 x2 x3
x1 x1

2
23 13 + 12 22 = 0 X
x2 x1
x2
x3 x1x3

Note that, the compatibility equations are satisfied if x2 is not a function of x 2 or x3 .


r

We can also verify the compatibility equations by means of xr ( xr ) T = 0 , i.e.:


r




( xr ) =
e i (11e 1 e 1 ) =
e 1 +
e2 +
e 3 [ x2 x3e 1 e 1 ]
x
x
x
xi

2
3

1
( x2 x 3 )
( x2 x3 )
( x2 x3 )
e 1 e 1 e 1 +
e 2 e 1 e 1 +
e 3 e 1 e 1
=
424
3
x3 1
x2
x1 12r 3
1
424
3
=e 2
=0
=0

= x 2 e 2 e 1
r
( xr )T = [ x2 e 2 e 1 ]T = x2 e 1 e 2

University of Castilla- La Mancha


Ciudad Real Spain

Draft

By: Eduardo W. V. Chaves (2015)

SOLVING PROBLEMS BY MEANS OF CONTINUUM MECHANICS

422

r
r



e 1 +
e2 +
e 3 [ x2 e 1 e 2 ]
xr ( xr )T =
x2
x3
x1
( x2 )
( x2 )
( x2 )
=
e 1 e 1 e 2 +
e 2 e 1 e 2 +
e 3 e 1 e 2
1
2
3
424
3
x1
x2
x3 1
r
123
123 =e 2
=0
=0

=0

=0 X

b) The displacement field can be obtained by means of the equation in (5.97), i.e.:
P

r
r
ui ( x ) = ui0 ( x 0 ) [ij0 ( x 0j xj )] + [ ik ( ik , j jk ,i )( x j xj )]dxk

(5.105)

and by applying the boundary conditions, (see Figure 5.12), we can obtain:
r
x

r
x

r
ui ( x ) = Tik dxk = [ ik ( ik , j jk ,i )( x j xj )]dxk

r
0
r
x

r
0

(5.106)

r
x

= [ ik ik , j ( x j xj ) + jk ,i ( x j xj )]dxk =
r
0

r
0

[Tik(1)

Tik( 2 )

+ Tik(3) ]dxk

where Tik(1) = ik ,
Tik( 2 ) = ik , j ( x j xj ) = ik ,1 ( x1 x1 ) + ik , 2 ( x2 x2 ) + ik ,3 ( x3 x3 ) = ik ,1 ( x1 x1 ) + ik ,3 ( x3 x3 )
x2 ,1 x3 ( x1 x1 ) + x2 ( x3 x3 ) 0 0
=
0
0 0

0
0 0

where x2 ,1 x2
x1

and
Tik(3) = jk ,i ( x j xj ) = 1k ,i ( x1 x1 ) + 2 k ,i ( x2 x2 ) + 3k ,i ( x3 x3 ) = 1k ,i ( x1 x1 )
x2 ,1 x3 ( x1 x1 ) 0 0

0
0 0
=
x ( x1 x1 )
0 0
2

Then,
x2 x3
Tik = 0
0

0 0 x2 ,1 x3 ( x1 x1 ) + x2 ( x3 x3 ) 0 0 x2 ,1 x3 ( x1 x1 ) 0 0

0 0
0
0 0 +
0
0 0
0 0
0
0 0 x2 ( x1 x1 )
0 0

x2 x3
0 0

0
0 0
Tik =
x ( x1 x1 ) 0 0
2

x2 x3
0 0 dx1 x2 x3 dx1

0
0 0 dx2 =
0
and Tik dxk =

x ( x1 x1 ) 0 0 dx3 x ( x1 x1 )dx1
2
2

Then, by substituting the above equation into the equation (5.106) we can obtain:
University of Castilla-La Mancha
Ciudad Real Spain

Draft

By: Eduardo W. V. Chaves (2015)

5 INTRODUCTION TO: CONSTITUTIVE EQUATIONS, IBVP, AND IBVP SOLUTION STRATEGIES

r
0

x
x

x2 x3 dx1 x3 x2 dx1

r
u1 r

0

0

0
0
= xr
u2 = xr

3 x ( x1 x1 )dx1 x ( x1 x1 )dx1
2
2
0r
0r

r
x

r
ui ( x ) = Tik dxk

423

Note that, if we consider that is constant, we can obtain:


r

x1

x2 3 dx1
r
x2 x3 ( x1 ) 0 x2 x3 x1
u1




0
0

=
=
u2 = xr 0
1
2
x
2
x1


u
x
3 x ( x1 x1 )dx1 ( 1 x x ) x2
2
x2
1 1
2
2


0r
0

Then
u1
= x3 x2
x1

2u3
= x2
x1x1

Note that for the neutral line (line at x3 ) there is not u1 -displacement, there is only
deflection ( u 3 -displacement).
The stress field (with = 0 ), (see Problem 5.5 NOTE 9), can be obtained
1 0 0

11

0 0 1
0 0
0 0
0 0

E
E
E
E
0
ij =
kk ij +
ij =
11 0 1 0 +
(1 + )(1 2 )
(1 + )
(1 + )(1 2 )
(1 + )
0
0 E11
11 (1 )
E

0
=
11 0 = 0
(1 + )(1 2 )

0
0 11 0

0 0
0 0
0 0

The resultant force on a cross-section can be obtained as follows:

F = 11dA = E11dA = E x2 x3 dA = E x2 x3 dA = 0
A
A
A
A 23
1
=0

Note that the first moment of the area about the x 2 -axis ( x3 dAx 2 ) is equals cero, since
A

the system is located at the geometrical center.


The bending moment on the cross-section can be obtained as follows:

M x2 = 11 x3 dA = E11 x3 dA = E x2 x32 dA = E x2 x32 dA = E x2 I x2


A
A
A
A 23
1
=Ix 2

where I x2 = x32 dA is the second moment of the area about the x 2 -axis. With that we can
A

conclude that on the cross-section the following is true:


x2 =

M x2
EI x2

University of Castilla- La Mancha


Ciudad Real Spain

11 ( x3 ) = E11 = E x2 x3 =

Draft

M x2
I x2

x3

By: Eduardo W. V. Chaves (2015)

SOLVING PROBLEMS BY MEANS OF CONTINUUM MECHANICS

424

Problem 5.14
Consider the infinitesimal strain tensor field
x2 x3

ik ( x1 , x3 ) = 0
0

x2 x3
0
x2 x3
0
= x2 x3e 1 e 1 x2 x3e 2 e 2 x2 x3e 3 e 3
0

(5.107)

where x2 is a function of x1 , i.e. x2 = x2 ( x1 ) and (Poissons ratio) is a constant. a)


Check whether the compatibility equations are fulfilled or not. b) In the case that the
compatibility equations are not satisfied, what should be met to ensure the continuity of
the displacement field?
Solution:

We can verify the compatibility equations by means of xr ( xr ) T = 0 , i.e.:


r

e i ( x2 x3e 1 e 1 x2 x3e 2 e 2 x2 x3e 3 e 3 )
( xr ) =
xi



=
e 1 +
e2 +
e 3 [ x2 x3e 1 e 1 x2 x3e 2 e 2 x2 x3e 3 e 3 ]

x
x
x
2
3

1
( x2 x3 )
( x2 x3 )
=
e 1 e 2 e 2
e 1 e 3 e 3
1
4
2
4
3
1
424
3
x1
x1
=e 3

= e 2

( x2 x3 )
x
1
4224
3
=0

+
= x3

( x2 x3 )

x2
x1

x3

e 2 e 1 e 1
1
424
3
= e 3

( x2 x3 )

e 2 e 3 e 3
424
3
x2 1
14243 =e 1
=0

e 3 e 1 e 1
1
424
3

( x2 x3 )

=e 2

e 3 e 2 + x3

x2
x1

x3

e 3 e 2 e 2
1
424
3
= e 1

e 2 e 3 + x2 e 2 e 1 + x2 e 1 e 2

thus
r
x2
x2
( xr )T = x3
e 2 e 3 + x3
e 3 e 2 + x2 e 1 e 2 + x2 e 2 e 1
x1
x1

r
r
r



xr ( xr )T =
e 1 +
e2 +
e 3 [( xr )T ]
x2
x3
x1
x2
x2



e 1 +
e2 +
e 3 [x3
e 2 e 3 + x3
e 3 e 2
=
x1
x1
x2
x3
x1
+ x2 e 1 e 2 + x2 e 2 e 1 ]
= x3

2 x2
x12

e 3 e 3 x3

2 x2

University of Castilla-La Mancha


Ciudad Real Spain

x12

e 2 e 2 +

Draft

x2
x1

e 3 e 1 +

x2
x1

e 1 e 3

By: Eduardo W. V. Chaves (2015)

5 INTRODUCTION TO: CONSTITUTIVE EQUATIONS, IBVP, AND IBVP SOLUTION STRATEGIES

r
r

T
S ij = [ xr ( xr ) ]ij =

0
x3

x2

2 x2
x12
0

x1

x2

x1

0
0 ij

2 x2
x3

x12

425

Note that the compatibility equations are not satisfied. One possibility to guarantee the
r
r
continuity of the displacement field ( [ xr ( xr ) T ] = 0 ), related to the strain field
(5.107), is when x2 is a constant, another possibility is when = 0 , (see Problem 5.13).
Note also that the above equation could be obtained by means of the equation in (5.80), i.e:

2 33 2 22
2 23
+

2
=0
S11 =
x2 x3
x22
x32

2
2
2
2 x2
S 22 = 33 + 11 2 13 = x3
0

x1x3
x12
x32
x12

2 x2

2 11 2 22
2 12
+
2
= x3
0
S 33 =
x1x2
x22
x12
x12

2
S = 23 + 13 12 33 = 0
12

x3 x1
x2
x3 x1x2

23 13 12 2 11

+
+
=0
S 23 =
x2
x1 x1
x3 x2 x3

2
S = 23 13 + 12 22 = x2 0
13

x1
x2 x1
x2
x3 x1x3

Problem 5.15
a) Show that the governing equations for a homogeneous isotropic linear elastic material
(see equations in (5.47)), can be replaced by six equations and six unknowns ( ij ), (Stress
Formulation), i.e.:
Indicial notation
ij , kk +

2( + )

&&i ), j
kk ,ij
ll , kk ij = 2 ( u
(2 + 3 )
(2 + 3 )

Tensorial notation

sym

2 ( bi ), j

sym

2xr +

2( + ) r r
&r&) sym 2 r ( b) sym
x [ x [Tr ( )]]
2xr [Tr ( )]1 = 2 xr ( u
x
(2 + 3)
(2 + 3)

where

2xr

(5.108)
xr ( xr ) and

University of Castilla- La Mancha


Ciudad Real Spain

2xr

[Tr( )] xr [ xr [Tr( )]] .

Draft

By: Eduardo W. V. Chaves (2015)

SOLVING PROBLEMS BY MEANS OF CONTINUUM MECHANICS

426

b) or by:
Indicial notation
ij , kk +

2( + )

&&k ), k ij + 2 ( u
&&i ), j
kk ,ij =
( b k ) , k ( u
(2 + 3 )
(2 + )

Tensorial notation
2xr +

sym

2 ( bi ) , j

] [

sym

r
r
2( + ) r r

&r&) 1 + 2 r ( u
&r&) sym 2 r ( b) sym
x [ x [Tr ( )]] =
xr ( b) ( u
x
x
(2 + 3)
(2 + )

(5.109)
E
E
and =
, express the equations (5.108) and
(1 + )(1 2 )
2(1 + )
(5.109) in function of ( E , ) .
r
Hint: The kinematic equations = sym u can be replaced by:

c) Considering that =

ij , kl + kl ,ij il , jk jk ,il = O ijkl

(5.110)

(see Problem 5.11).


Solution: a) We obtain the inverse of the constitutive equation in stress ( = C e : ):
Ce

: = Ce

: C e : = I sym : = sym =

= Ce

For isotropic materials, (see equation (5.26)), the strain tensor can be obtained as follows:
=

1
1

Tr ( )1 indicial

ij =
ij
ss ij .
2
2 (2 + 3 )
2
2 (2 + 3 )
r

If we consider a homogeneous material, the mechanical properties do not vary with x , i.e.

,i

= 0 i and ,i
= 0 i , then:
x i
xi

2 ij

ij , kl =
ij
ss ij =
ij ,kl
ss , kl ij
x k xl
2 (2 + 3 )
2 (2 + 3 )
2
,kl 2

(5.111)

Moreover, if we multiply the equation in (5.110) (kinematic equations) by jk we can


obtain:
ij , kl jk + kl ,ij jk il , jk jk jk ,il jk = O ijkl jk
ik ,kl + kl ,ik il , kk kk ,il = 0 il

(5.112)

Note that, according to equation (5.111) the following is true:


ik ,kl =

1
1
ik ,kl
ss ,kl ik =
ik ,kl
ss ,il
2
2 (2 + 3 )
2
2 (2 + 3 )

kl ,ik =

1
1
kl ,ik
ss ,ik kl =
lk ,ki
ss ,il
2
2 (2 + 3 )
2
2 (2 + 3 )

il ,kk =

1
il ,kk
ss ,kk il
2
2 (2 + 3 )

University of Castilla-La Mancha


Ciudad Real Spain

Draft

By: Eduardo W. V. Chaves (2015)

5 INTRODUCTION TO: CONSTITUTIVE EQUATIONS, IBVP, AND IBVP SOLUTION STRATEGIES

kk ,il =
=

427

1
1
3

kk ,il
ss ,il kk =
kk ,il
ss ,il
{
2
2 (2 + 3 )
2
2 (2 + 3 )
=3

2
1
3
3
ss ,il =
ss ,il
ss ,il
ss ,il =

2
2 (2 + 3 )
2 (2 + 3 )
2 2 (2 + 3 )

2 ij
x k x l

ij ,kl =

1
ij ,kl
ss ,kl ij
2
2 (2 + 3 )

With that the equation in (5.112) becomes:


ik ,kl + kl ,ik il ,kk kk ,il = 0 il
1
2

2
2

ik ,kl
ss ,il + lk ,ki il , kk +
ss ,kk il
ss ,il = 0 il
(2 + 3 )
(2 + 3 )
(2 + 3 )

2
2

ss ,il + lk , ki il ,kk +
ik ,kl
+
ss , kk il = 0 il
(2 + 3 )
(2 + 3 ) (2 + 3 )
2( + )

ik ,kl
ss ,il + lk , ki il ,kk +
ss , kk il = 0 il
(2 + 3 )
(2 + 3 )

2( + )

ss ,il il , kk +
ss , kk il = ik , kl lk , ki
(2 + 3 )
(2 + 3 )

(5.113)

&& i we can obtain:


From the equations of motion ij , j + b i = u
&& i ) ,k
ij , jk + ( b i ) , k = ( u

with that the following is true:


&& i ) ,l
ik ,kl + ( b i ) ,l = ( u

&& i ) ,l
ik ,kl = ( b i ) ,l ( u

&& l ) ,i
lk , ki + ( b l ) ,i = ( u

&& l ) ,i .
lk ,ki = ( b l ) ,i ( u

And note that ik ,kl lk , ki = ( b i ) ,l ( u&& i ) ,l + ( b l ) ,i ( u&& l ) ,i = 2[( b i ) ,l ]sym 2[( u&& i ) ,l ]sym
By replacing the above equation into the equation (5.113) we can obtain:
2( + )

ss ,il il , kk +
ss , kk il = 2 ( b i ) ,l
(2 + 3)
(2 + 3 )

&& i ) ,l
2 ( u

sym

sym

Restructuring the above and considering that ( l = j ) we obtain:


ij ,kk +

2( + )

&& i ) , j
kk ,ij
ll ,kk ij = 2 ( u
(2 + 3 )
(2 + 3 )

sym

2 ( b i ) , j

sym

(5.114)

Q.E.D.
which is the same as (5.108). The above equation could have been obtained by means of
equation in (5.79):
xr ( xr ) + xr [ xr [Tr ( )]] = xr ( xr ) + [ xr ( xr )]

where =

(5.115)

Tr ( )1 , and is also true that


2
2 (2 + 3)

University of Castilla- La Mancha


Ciudad Real Spain

Draft

By: Eduardo W. V. Chaves (2015)

SOLVING PROBLEMS BY MEANS OF CONTINUUM MECHANICS

428

xr ( xr ) = xr xr

Tr ( )1
2 (2 + 3)

2
;

1 r
=
x ( xr )
xr xr [ Tr ( )]1
2
2 (2 + 3)
Tr ( ) =

1
Tr ( )
(2 + 3 )

xr =

1 r
1 r
x
xr ( Tr ( )1) =
x
xr ( Tr ( ))
2
2 (2 + 3)
2
2 (2 + 3)

xr [ xr [Tr ( )]] =

1
xr [ xr [Tr ( )]] ;
(2 + 3 )

Note that [ xr ( Tr ( )1)]i = ( kk ij ) , j = kk , j ij + kk ij , j = kk , j ij = kk ,i = [ xr ( Tr ( ))]i ,


r

&& we can obtain:


and if we consider xr + b = u

1 r
1 &r&
x
xr ( Tr ( )) =
xr ( Tr ( ))
( u b )
2
2 (2 + 3)
2
2 (2 + 3)
r

1 r &r&
xr ( xr ) =
x [( u b)]
xr [ xr ( Tr ( ))]
2
2 (2 + 3)

xr =

with that we can obtain:


xr ( xr ) + [ xr ( xr )] = 2[ xr ( xr )]
T

sym

r
2
2
&r& b)]}sym
{ xr [( u
{ xr [ xr ( Tr ( ))]}sym
2
2 (2 + 3)
r
2
2
&r& b)]}sym
{ xr [( u
xr [ xr ( Tr ( ))]
=
2
2 (2 + 3)
=

{ xr [ xr ( Tr ( ))]}ij = kk ,ij = kk , ji = { xr [ xr ( Tr ( ))]} ji (symmetric). Taking into account the

above equations into the equation (5.115) we can obtain:


xr ( xr ) + xr [ xr [Tr ( )]] = xr ( xr ) + [ xr ( xr )]

1 r

1
x ( xr )
xr xr [ Tr ( )]1 +
xr [ xr [Tr ( )]] =
2
2 (2 + 3)
(2 + 3)
r
2
2
&r& b)]}sym
{ xr [( u
xr [ xr ( Tr ( ))]
2
2 (2 + 3)

xr ( xr )

(2 + 3)

( xr xr [ Tr ( )])1 +

r
&r& b)]}sym
2{ xr [( u
2xr

(2 + 3)

2xr [ Tr ( )] 1 +

2
xr [ xr [Tr ( )]] =
(2 + 3)

2
xr [ xr ( Tr ( ))]
(2 + 3)

r
2( + ) r r
&r& b)]}sym
x [ x [Tr ( )]] = 2{ xr [( u
(2 + 3)

which matches the equation in (5.114) or (5.108).


b) Starting from the equation (5.114):
ij ,kk +

2( + )

&& i ) , j
kk ,ij =
ll ,kk ij + 2 ( u
(2 + 3 )
(2 + 3 )

University of Castilla-La Mancha


Ciudad Real Spain

Draft

sym

2 ( b i ) , j

sym

(5.116)

By: Eduardo W. V. Chaves (2015)

5 INTRODUCTION TO: CONSTITUTIVE EQUATIONS, IBVP, AND IBVP SOLUTION STRATEGIES

429

Our goal now is to obtain an expression for ll, kk . If we multiply equation (5.110) by
jk li we obtain:
ij ,kl jk li + kl ,ij jk li il , jk jk li jk ,il jk li = O ijkl jk li
ij , ji + ji ,ij ii , jj jj ,ii = 2 ij ,ij 2 ii , jj = 0

(5.117)

ij ,ij ii , jj = 0

If we use the inverse of the constitutive equation (see equation (5.111)), we can obtain:
ij ,ij =
ii , kk

1
1

ij ,ij
ss ,ij ij =
ij ,ij
ss ,ii
2
2 (2 + 3 )
2
2 (2 + 3 )

(5.118)

2
1

ii , kk
=
ii ,kk
ss ,kk ii =
2
2 (2 + 3 )
2 (2 + 3 )

With that the equation in (5.117) becomes:


ij ,ij ii , jj = 0

1
ii ,kk = 0
ij ,ij
ss ,ii
2
2 (2 + 3 )
2 (2 + 3 )

ii ,kk = 0
ij ,ij
+
(2 + 3 ) (2 + 3 )

(5.119)

2 +
ii , kk
ij ,ij =
(2 + 3 )

The above equation can also be written in terms of


2 +
ii , kk
2 + 3

ij , ij =

2 + r
x [ xr [Tr ( )]]
2 + 3

xr ( xr ) =

ij , ij =
xr ( xr ) =

1
ii , kk
1+
1 r
x [ xr [Tr ( )]]
1+

(5.120)

&& i we can obtain:


Now, by means of the equations of motion ij , j + b i = u
&& i ) ,i
ij , ji + ( b i ) ,i = ( u

&& i ) ,i ( b i ) ,i
ij , ji = ( u

With that the equation in (5.119) becomes:


2 +
ii ,kk
ij ,ij =
(2 + 3 )
2 +
&& i ) ,i ( b i ) ,i =
( u
(2 + 3 ) ii ,kk

ii ,kk = ll ,kk =

(5.121)

(2 + 3 )
(2 + 3 )
&& k ) ,k ( b k ) ,k =
&& k ) , k
( u
( b k ) , k ( u
2 +
2 +

Replacing equation (5.121) into (5.116), we obtain:

University of Castilla- La Mancha


Ciudad Real Spain

Draft

By: Eduardo W. V. Chaves (2015)

SOLVING PROBLEMS BY MEANS OF CONTINUUM MECHANICS

430

ij ,kk +

2( + )

&&i ), j
kk ,ij =
ll ,kk ij + 2 ( u
(2 + 3)
(2 + 3)

ij ,kk +

2( + )

(2 + 3)
&&k ),k ij + 2 ( u
&&i ), j
kk ,ij =
( b k ) ,k ( u
(2 + 3)
(2 + 3) 2 +

sym

2 ( b i ), j

ij ,kk +

sym

sym

2 ( b i ) , j

2( + )

&&k ),k ij + 2 ( u
&&i ), j
kk ,ij =
( b k ) ,k ( u
(2 + 3)
(2 + )

sym

2 ( b i ) , j

sym

sym

(5.122)

Q.E.D.
Thus obtaining the equation in (5.109)
c) After some algebraic manipulations we can obtain:
E
1
(1 2 )
(1 2 )

,
,
=
=
=
E
(2 + 3)
E
(2 + 3)
(1 + )(1 2 ) (1 + )

(2 + 3 )

E
2( + )
(1 2 )
(1 2 )
(1 2 )
1

,
,
=
=2
+2
=
E
(1 + )
2(1 + ) 2(1 + ) (2 + 3)
2(1 + ) (1 + )

( 2 + ) = 2

E
E
E (1 )
,
+
=
2(1 + ) (1 + )(1 2 ) (1 + )(1 2 )

E
(1 + )(1 2 )

2 + 1
,
,
=
=
=
(2 + ) (1 + )(1 2 ) E (1 )
(1 ) 2 + 3 1 +

whereby the equation (5.108) becomes:


ij , kk +

&&i ), j
kk ,ij
ll , kk ij = 2 ( u
(1 + )
(1 + )

Tensorial notation
2xr +

sym

2 ( bi ) , j

sym

1
&r&) sym 2 r ( b) sym
xr [ xr [Tr ( )]]
2xr [Tr ( )]1 = 2 xr ( u
x
(1 + )
(1 + )

(5.123)
and the equation (5.109) becomes:
ij , kk +

&&k ), k ij + 2 ( &u&i ), j
kk ,ij =
( b k ) , k ( u
(1 + )
(1 )

Tensorial notation
2xr +

] [

sym

2 ( bi ), j

sym

r
r
1

&r&) 1 + 2 r ( u
&r&) sym 2 r ( b) sym
xr [ xr [Tr ( )]] =
xr ( b) ( u
x
x
(1 + )
(1 )

(5.124)
NOTE 1: For a static problem the above equation becomes:
ij , kk +

2xr

kk ,ij =
( bk ), k ij 2 ( bi ), j
(1 + )
(1 )

] [

sym

r
r sym
1

xr [ xr [Tr ( )]] =
xr ( b) 1 2 xr ( b)
+
(1 + )
(1 )

Michells equations

(5.125)

which are known as Michells equations, which were obtained by Michell in 1900.

University of Castilla-La Mancha


Ciudad Real Spain

Draft

By: Eduardo W. V. Chaves (2015)

5 INTRODUCTION TO: CONSTITUTIVE EQUATIONS, IBVP, AND IBVP SOLUTION STRATEGIES

431

If the body forces do not vary with x , the Michells equations (5.125) reduce to:
ij , kk +
2xr

1
kk ,ij = 0ij
(1 + )

1
+
xr [ xr [Tr ( )]] = 0
(1 + )

Beltramis equations

(5.126)

which are known as Beltramis equations, which were obtained by Beltrami in 1892, (see
Sokolnikoff (1956) first edition in (1946)).
If we take into account that ij , kk =

2ij

= 2xr ij , ( bi ), j

sym

1 ( bi ) ( b j )
+
=
, and
2 x j
xi

xk xk
r
( b k ), k ij = [ xr ( b)] ij , the Michells equations can be rewritten explicitly as follows:

ij , kk +

1
kk ,ij =
( bk ), k ij 2 ( bi ), j
(1 + )
(1 )

2xr ij +

sym

r
( b i ) ( b j )

1 2 [ Tr ( )]

=
+
[ xr ( b)] ij
x j

(1 + ) xi x j
(1 )
x
i

Then, the above six equations are:


r
2
1 2 [ Tr ( )]
( b1 )

r +

[ xr ( b)] 2
=
x 11
2
(1 + )
(1 )
x1
x1

r
1 2 [ Tr ( )]
( b 2 )

r ( b)] 2
2xr 22 +

[
=
x
2
(1 + )
(1 )
x2
x2

2
r
2r + 1 [ Tr ( )] = [ r ( b)] 2 ( b3 )
x 33
x
2

(1 + )
(1 )
x3
x3

2
2r + 1 [ Tr ( )] = ( b1 ) + ( b 2 )
x
x 12 (1 + ) x1x2
x1
2

2
( b 2 ) ( b3 )
1 2 [ Tr ( )]
r +

+
=
x 23

(
1
)

x
x
x
x

2
3
3
2

2xr 13 + 1 [ Tr ( )] = ( b1 ) + ( b3 )

(1 + ) x1x3
x1
x3

(5.127)

&& k = 0 k ) , the equation in (5.121) becomes:


NOTE 2: For a static problem (u
ll , kk =

(2 + 3)
(1 + )
( b k ) , k =
( b k ) , k
2 +
(1 )

r
(1 + ) r
x ( b)
(1 )
r
(1 + ) r
2xr [Tr ( )] =
x ( b)
(1 )
xr { xr [Tr ( )]} =

(5.128)

The above equation can also be obtained by means of equation (5.125) with ( i = j ), i.e.: we
are obtaining the trace of (5.125):

University of Castilla- La Mancha


Ciudad Real Spain

Draft

By: Eduardo W. V. Chaves (2015)

SOLVING PROBLEMS BY MEANS OF CONTINUUM MECHANICS

432

ii , kk +

1
kk ,ii =
( b k ) , k ii 2 ( b i ) ,i
{
(1 + )
(1 )
=3

1
ii , kk =
1 +
2 ( b k ) , k
(1 + )
(1 )

(2 + )
(2 + )
ii , kk =
( b k ) , k

(1 + )
(1 )

]
(5.129)

ii , kk =

(1 + )
( b k ) , k
(1 )

Note that ii ,kk = kk ,ii and ( b k ) ,k = ( b i ) ,i . The above equation in tensorial notation
becomes:
( 2xr ) : 1 +

r
r sym
1
{ xr [ xr [Tr ( )]]} : 1 = xr ( b) 1 : 1 2 xr ( b) : 1
(1 + )
(1 )

(5.130)

Note that
( 2xr ) : 1 { xr ( xr )}: 1 = xr

[ xr [Tr( )]] 2xr [Tr( )]

{ xr [ xr [Tr( )]]}: 1 = xr [ xr [Tr( )]] 2xr [Tr( )]


1 :1 = 3
r sym
r
xr ( b)
: 1 = xr ( b)

with that the equation (5.130) becomes:

] [

r
r
3
1
2xr [Tr ( )] =
xr ( b) 2 xr ( b)
(1 + )
(1 )
r
(1 + ) r
2xr [Tr ( )] =
x ( b )
(1 )
2xr [Tr ( )] +

NOTE 3: For the two-dimensional elasticity case, the stress formulation is provided in
Problem 6.32.
Problem 5.16
Consider a static linear elastic problem, and also that the mass density ( ) and the
r

mechanical properties ( , ) are homogeneous fields, and that the specific body force b is
a conservative and homogeneous field. Show that the Cauchy stress tensor, the
infinitesimal strain tensor, and the displacement components are biharmonic functions.
Solution:

&& becomes
Taking into account the static problem, the equations of motion xr + b = u
r

the equilibrium equations xr + b = 0 , and by applying the divergence to it we can


obtain:
Tensorial notation

Indicial notation

r r
xr + b = 0

ij , j + bi = 0 i

r
xr ( xr ) + xr ( b) = 0
1424
3

ij , ji + ( bi ),i = 0
123

xr ( xr ) = 0

ij ,ij = 0

University of Castilla-La Mancha


Ciudad Real Spain

(5.131)

=0

=0

Draft

By: Eduardo W. V. Chaves (2015)

5 INTRODUCTION TO: CONSTITUTIVE EQUATIONS, IBVP, AND IBVP SOLUTION STRATEGIES

433

where we have considered that the body force density ( b) does not change with x
(homogeneous field).
If we take into account the equation in (5.120) we can conclude that:
Tensorial notation

Indicial notation

1 r
xr ( xr ) =
x [ xr [Tr ( )]] = 0
1+
xr [ xr [Tr ( )]] 2xr [Tr ( )] = 0

1
ij ,ij =
ii ,kk = 0
1+
2 ii
= 2xr ( ii ) = 0
ii ,kk =
xk xk

(5.132)

with that we show that [ Tr ( )] is harmonic function. Then it is easy to show that [ Tr ( )]

is also harmonic function, since [ Tr ( )] = 3 +

2
[ Tr ( )] (see Problem 5.5 NOTE 8):
3

2
2xr [Tr ( )] = 2xr 3 +
[ Tr ( )] = 0
3

2xr [ Tr ( )] = 0

(5.133)

If we apply the Laplacian to the Beltramis equations (5.126) we can obtain:


Tensorial notation

Indicial notation

1
2xr +
xr [ xr [Tr ( )]] = 0
(1 + )
1
2xr { xr [ xr [Tr ( )]]} = 0
2xr 2xr +
(1 + )
2xr 2xr +

1
xr xr [ 2xr [ Tr ( )] ] = 0
14243
(1 + )
=0

2xr 2xr 4xr = 0

1
kk ,ij = 0ij
(1 + )
1
ij , kkpp +
kk ,ijpp = 0ij
(1 + )
1
ij , kkpp +
(kk , pp ),ij = 0ij
(1 + ) 123
ij , kk +

(5.134)

=0

ij , kkpp =

xk xk

2 (ij )

=0
x p x p

2xr 2xr (ij ) 4xr (ij ) = 0ij

With that we show that the Cauchy stress tensor is biharmonic function, where the
operator 4xr 2xr 2xr is known as the bilaplacian. We can show that the infinitesimal strain
tensor is also biharmonic function, i.e.: 4xr = 0 . Taking into account the above equation
and the constitutive equation in stress for isotropic linear elastic material
= Tr ( )1 + 2 we obtain:
4xr = 0
4xr (Tr ( )1 + 2 ) = 4xr (Tr ( )1) + 4xr (2 ) = 4xr ( Tr ( ))1 + 2 4xr ( ) = 0
14243
=0

4xr

=0

To show that the displacement components are biharmonic function, we will start with the
&&i = 0 i , and if we
Naviers equations (5.49) for a static case, ( + )u j , ji + ui , jj + bi = u
apply the Laplacian to it we can obtain:
( + )u j , jikk + ui , jjkk + ( bi ), kk = 0i
1
424
3

(5.135)

= 0i

where we have considered that , and ( bi ) do not change with x . Note also that

University of Castilla- La Mancha


Ciudad Real Spain

Draft

By: Eduardo W. V. Chaves (2015)

SOLVING PROBLEMS BY MEANS OF CONTINUUM MECHANICS

434

r
ui , jjk = ui , kjj = 2xr (ui , k ) = { 2xr [ xr u]}ik

and if we consider the infinitesimal strain tensor:


1
1
(ui , j + u j ,i ) trace

kk = (uk , k + uk , k ) = uk , k
2
2
r
trace
kk ,ij = uk , kij
kk , pp = uk , kpp = 0 tensorial

2xr [ Tr ( )] = 2xr [ Tr ( xr u) ] = 0
ij =

where we have used the equation (5.133). Then, the equation in (5.135) can be written as
follows:
( + )u j , jikk + ui , jjkk = 0i

( + )(u j , jkk ), i + ui , jjkk = 0i


123

=0

4xr (u1 )
ui , jjkk =

4xr (ui )

= 0i

=0
4
xr (u 2 ) = 0
4r
x (u3 ) = 0

(5.136)

with that we show that the displacement components are biharmonic functions.
Problem 5.17
a) Given a scalar field such as:
11 =

22 =

x 22

12 = 21 =

x12

2
x1 x 2

(5.137)

Show that
Indicial notation
, iijj = 0

Tensorial notation

{ [ ( )]} = 0

(i, j = 1,2)

2 2 = 0

4
4 4
4 +2 2 2 + 4 =0
x1
x1 x2 x2

(5.138)

4 = 0

Consider a linear elastic material, a static problem, and with no body forces. Consider also
that the Cauchy stress tensor is only dependent of x1 and x 2 , i.e. = ( x1 , x 2 ) .
b) Show whether the equilibrium are satisfied or not.
Solution:
a) In Problem 5.15 (see equation (5.129)) we have shown that:
ii , kk =

(1 + )
( b k ) , k = 0
(1 )

where we have considered that ( b k ) ,k = 0 . For the proposed problem we have i, k = 1,2 ,
with which:
ii ,kk = 0

11
x12

22
x12

ii ,11 + ii , 22 = 0
+

11
x 22

22
x 22

11,11 + 22,11 + 11, 22 + 22, 22 = 0

=0

Using the definition given by (5.137), we can conclude that:

University of Castilla-La Mancha


Ciudad Real Spain

Draft

By: Eduardo W. V. Chaves (2015)

5 INTRODUCTION TO: CONSTITUTIVE EQUATIONS, IBVP, AND IBVP SOLUTION STRATEGIES

2 11
x12

2 22
x12

2 11
x 22

2 22

435

=0

x 22

2 2 2 2 2 2 2 2
+
+
+
=0
x12 x 22 x12 x12 x 22 x 22 x 22 x12

4
4
4
+
2
+
=0
x14
x12 x 22 x 24

Q.E.D.

b) For the bidimensional case (2D), the equilibrium equations (without body forces) reduce
to:
i , j =1, 2 )
(

ij , j = 0 i

i1,1 + i 2, 2 = 0 i

11 12
x + x = 0
1
2

21 + 22 = 0
x1
x 2

Using the definition (5.137), we can obtain:


11 12
x + x = 0
1
2

21 + 22 = 0
x1
x2

2
3
3
2
=0
X

2
2
2
x1 x2 x2 x1 x2 x1x2 x1 x2

2
3
3
2
+ = + = 0 X
x x x
x2 x12
x12 x2 x2x12
1
1
2

With this, we show that the expressions of stresses given by (5.137) satisfy the equilibrium
equations.
NOTE: In the literature, is known as the Airy stress function, (see Problem 6.33), and the
SI unit of is [] = N ( Newton) .
Problem 5.18
Let us consider that the Cauchy stress tensor field can be obtained as follows:
r
r
= xr ( xr P )T

(5.139)

where the second-order tensor P has the following Cartesian components:


1 0
Pij = 0 2
0
0

0
0
3

[ P ] = N ( Newton)

a) Obtain the explicit components of the stress tensor in function of i . b) Check whether
the body is in equilibrium by considering the static state and without body force.
Solution:
a) In Problem 1.109 we have shown that the following is true:

r
r
qt = [ xr ( xr P ) T ] qt = qjk til Pij , kl

Note also that the explicit equations for qt , (given by (5.139)), are the same the one used
to obtain the components Sij in equation (5.80) (see Problem 5.11), so

University of Castilla- La Mancha


Ciudad Real Spain

Draft

By: Eduardo W. V. Chaves (2015)

SOLVING PROBLEMS BY MEANS OF CONTINUUM MECHANICS

436

2 P33 2 P22
2 P23
+

2
11 =
x2 x3
x22
x32

2
2

P33 P11
2 P13
22 =
+

2
x1x3
x12
x32

2
2
2
= P11 + P22 2 P12
33
x1x2
x22
x12

2
= P23 + P13 P12 P33
12

x3 x1
x2 x3 x1x2

P23 P13 P12 2 P11

23 =
x + x + x x x

x
2 3
1
1
2
3

2
13 = P23 P13 + P12 P22

x2 x1
x2
x3 x1x3

(5.140)

Taking into account that P11 = 1 , P22 = 2 , P33 = 3 and P12 = P23 = P13 = 0 , the stress
components become:

2 3 2 1
2 1

+
(5.141)
x 2
x2 x3
x32

1
2 1 2 2
2 1
2 +

x
x2 x3
x12
2
r
r
&r& , (see equation (5.14)), in
b) We start from the equations of motion + b = v& = u
&& i and by considering the static state ( u
&&i = 0i ) and without
indicial notation ij , j + b i = u
2 3 2 2

2
2

x
x

3
2

2 3
qt =

x1x2

2 2

x1x3

2 3
x1x2

2 2
x1x3

body force ( bi = 0i ) the equations of motion become the equilibrium equations, namely:
ij , j = i1,1 + i 2, 2 + i 3,3 = 0i
11,1 + 12, 2 + 13,3 = 0

21,1 + 22, 2 + 23,3 = 0

31,1 + 32, 2 + 33,33 = 0

11 12 13
+
+
=0

x2
x3
x1
21 22 23
+
+
=0

x2
x3
x1
31 32 33
+
+
=0

x1
x2
x3

And by substituting the stress components given by Eq. (5.141) we can conclude that
11 12 13
2 3 2 2 2 3 2 2

=0
+
+
+
=
X

x2
x3
x1 x22
x32 x2 x1x2 x3 x1x3
x1

2 3 2 3 21 21
21 22 23

+
+
=

x x + x x 2 + x 2 x x x = 0 X

x
x
x
x
2
3
1
1 2
2
3
2 3
1
3
1

2
2
2
2
31 + 32 + 33 = 2 1 + 1 + 2 = 0 X
x1
x1 x1x3 x2 x2 x3 x3 x22
x3
x2
x12

University of Castilla-La Mancha


Ciudad Real Spain

Draft

By: Eduardo W. V. Chaves (2015)

5 INTRODUCTION TO: CONSTITUTIVE EQUATIONS, IBVP, AND IBVP SOLUTION STRATEGIES

437

NOTE 1: In the literature i are known as stress functions. In the particular case when
3 = and 1 = 2 = 0 we fall back into the two-dimensional problem discussed in
Problem 5.17, where = ( x1, x2 ) is the Airy stress function. In this case the Eq. (5.141)
becomes:
2

2
x2
2
qt =
x1x2
0

2
x1x2
2
x12
0

NOTE 2: Note that the stress field can also be expressed by other function . In this case
the P -components are:
0 3 2
1
3 0 1
Pij =
2
2 1 0

[ P ] = N ( Newton)

By substituting these components into the equation in (5.140) we can obtain:

2 1

x2x3


1 + 2 3
qt =

2 x3 x1 x2 x3
1


1 2 + 3

2 x2 x1 x2 x3

1 1 2 3

+
2 x3 x1 x2 x3
2 2
x1x3

1 1 2 3

+
+
2 x1 x1 x2 x3

1 1 2 3

+
2 x2 x1 x2 x3
1 1 2 3

+
+
2 x1 x1 x2 x3

2 3

x1x2

(5.142)
We leave the reader to check whether the equilibrium equations are satisfied or not.
Let us suppose that 3 = 3 ( x2 , x3 ) = ( x2 , x3 ) and 1 = 2 = 0 , with that the stress field
becomes:

1
3
qt =
x
2

3
3
1
3

x
2 x3

1 3

2 x3 x3
0
0

where we have considered that = ( x2 , x3 ) =

1 3

2 x2 x3 0


0
=
x3


0
x2

x3
0
0

x2
0

1 3
. In the literature is known as the
2 x3

Prandtls stress function, (see 6.3 Torsion NOTE 3).

University of Castilla- La Mancha


Ciudad Real Spain

Draft

By: Eduardo W. V. Chaves (2015)

SOLVING PROBLEMS BY MEANS OF CONTINUUM MECHANICS

438

Additional NOTE: The components of xr ( xr P ) (not symmetric) were obtained in


Problem 1.109 and are given by:
r
r
[ xr ( xr P )]tj = Psj ,ts Ptj , ss

then
Psj ,ts Ptj , ss = P1 j ,t1 + P2 j ,t 2 + P3 j ,t 3 ( Ptj ,11 + Ptj , 22 + Ptj ,33 )
(t = 1, j = 1)

P11,11 + P21,12 + P31,13 ( P11,11 + P11, 22 + P11,33 ) = (1, 22 + 1,33 )

(t = 2, s = 2)

P12, 21 + P22, 22 + P32, 23 ( P22,11 + P22, 22 + P22,33 ) = ( 2,11 + 2,33 )

(t = 3, j = 3)

P13,31 + P23,32 + P33,33 ( P33,11 + P33, 22 + P33,33 ) = ( 3,11 + 3, 22 )

(t = 1, j = 2)

P12,11 + P22,12 + P32,13 ( P12,11 + P12, 22 + P12,33 ) = 2,12

(t = 2, j = 3)

P13, 21 + P23, 22 + P33, 23 ( P23,11 + P23, 22 + P23,33 ) = 3, 23

(t = 1, j = 3)

P13,11 + P23,12 + P33,13 ( P13,11 + 13, 22 + P13,33 ) = 3,13


M

Thus

21 21
2 2
2 3

2 +
2
x1x2
x1x3
x3

x2
2
2
2
2

r
r

1
2
2
3

[ xr ( xr P )]tj =
+

2
x1x2
x2x3
x32

x1

2
2
2
2

3
1
2

2 3 +

x1x3
x2 x3
x22
x1

Note that

r
r
r
r
[ xr ( xr P )T ] [ xr ( xr P )]

In the case when P is a spherical tensor, e.g. P = 1 , in which = ( x1 , x2 , x3 ) , we can


conclude that
P = 1

University of Castilla-La Mancha


Ciudad Real Spain

r
r
r
r
[ xr ( xr P )T ] = [ xr ( xr P )]

Draft

By: Eduardo W. V. Chaves (2015)

5 INTRODUCTION TO: CONSTITUTIVE EQUATIONS, IBVP, AND IBVP SOLUTION STRATEGIES

439

Problem 5.19
Consider the governing equation for the linear elastic problem described in Problem 5.5.
r
Obtain an equivalent formulation such as the unknowns are displacement u and stress
(Mixed Formulation). Use Voigt notation.
Solution:
Taking into account the governing equations for the elastic linear problem:
Tensorial notation
The equations of motion:
r
r
&r& (3 equations)
+ b = v& = u
The constitutive equations for stress:

Voigt notation
The equations of motion:
(1) T
[ L ] { } + {b } = {u&&} (3 equations)
The constitutive equations for stress:

( ) = C e : (6 equations)

{ } = [C ] { } (6 equations)

The kinematic equations:

(5.143)

The kinematic equations:

r
= sym u (6 equations)

{ } = [ L(1) ] {u } (6 equations)

where the equations in Voigt notation were obtained in Problem 5.8, where

x1
[ L(1) ]T = 0

x2

x3

x2

x1
0

x3

x2

x3
0


x1

To eliminate the strain from the governing equations, we replace the kinematic equation
into the constitutive equations for stress, i.e.:
{ } = [C ] { }

{ } = [C ] [ L(1) ]{u }

[C ]1{ } = [C ]1[C ] [ L(1) ]{u }


1
424
3
=[1 ]
[C ]1{ } [ L(1) ]{u } = {0 }

whereby the system (5.143) becomes:

[ L(1) ]T { } + {b } = { u&&}

1
(1)
[C ] { } [ L ]{u } = {0 }

which is also possible to write the above set of equations as follows:


[0 ] [ L(1) ]T {u } {b } + {u&&}

{0 }
[ L(1) ] [C ]1 { }

NOTE 1: The above formulation is known as Mixed Formulation. It is interesting to note


that in the formulations either in displacement or in stress, (see Problem 5.9 and Problem
5.15), we have second derivative of the unknowns, meanwhile in the mixed formulation we
deal only with the first derivative of the unknowns, and moreover this formulation does
not deal with the derivative of the mechanical properties.

University of Castilla- La Mancha


Ciudad Real Spain

Draft

By: Eduardo W. V. Chaves (2015)

SOLVING PROBLEMS BY MEANS OF CONTINUUM MECHANICS

440

NOTE 2: We can summarize that, the linear elastic problem, considering an isotropic
homogenous linear elastic material, is governed by the set of partial differential equations:
Tensorial notation
The equations of motion:
r
r
&r& (3 equations)
+ b = v& = u
The constitutive equations for stress:

Voigt notation
The equations of motion:
(1) T
[ L ] { } + {b } = {u&&} (3 equations)
The constitutive equations for stress:

( ) = C e : (6 equations)

{ } = [C ] { } (6 equations)

The kinematic equations:

(5.144)

The kinematic equations:

r
= sym u (6 equations)

{ } = [ L(1) ] {u } (6 equations)

making a total of 15 equations and 15 unknowns, namely (u i , ij , ij ) .


This set of equations can also be represented by:
1) Displacement Formulation (see Problem 5.9):
&& i
( + )u j , ji + u i , jj + b i = u

Naviers equations

r
r
r
&r&
( + )[ ( u)] + [ (u)] + b = u

(5.145)

in which we have 3 equations and 3 unknowns (u i ) .


2) Stress Formulation (see Problem 5.15):
Indicial notation
ij , kk +

2( + )

&&i ), j
kk ,ij
ll , kk ij = 2 ( u
(2 + 3 )
(2 + 3 )

sym

Tensorial notation
2xr +

2 ( bi ), j

sym

2( + ) r r
&r&) sym 2 r ( b) sym
x [ x [Tr ( )]]
2xr [Tr ( )]1 = 2 xr ( u
x
(2 + 3)
(2 + 3)

(5.146)
in which we have 6 equations and 6 unknowns ( ij ) .
3) Mixed Formulation (see Problem 5.19)
[ L(1) ]T { } + {b } = {u&&}

1
(1)
[C ] { } [ L ]{u } = {0 }

(3 equations)
(6 equations)

(5.147)

in which we have 9 equations and 9 unknowns ( u i , ij ).

University of Castilla-La Mancha


Ciudad Real Spain

Draft

By: Eduardo W. V. Chaves (2015)

5 INTRODUCTION TO: CONSTITUTIVE EQUATIONS, IBVP, AND IBVP SOLUTION STRATEGIES

441

Problem 5.20
Let us consider two systems made up by the same linear elastic material but with different
load conditions as shown below:
r

System I

Surface force - t *

Specific body force - b


S

Su
r
u*

dV

r r
t * ( x)

r r
b( x )

Stress field -
Strain field -

System II
Su
r
u*

Displacement field - u

Surface force - t *
S

B
dV

r r
t * ( x)

r
b

Specific body force - b


Stress field -
Strain field -
n

Displacement field - u

Figure 5.13: Two systems under external actions.

Show the Bettis theorem also known as Bettis reciprocal theorem:

: dV = : dV

Bettis theorem

(5.147)

Solution:
Taking into account the constitutive equation for stress, = C e : , in indicial notation:
e
ij = C ijkl
kl

And by multiplying both sides of the equation by the field we can obtain:
ij ij = ij C eijkl kl

of C
Major
Simmetry

e
ij ij = ij C ijkl
kl = kl C eklij ij

e
where we have applied the major symmetry of the elasticity tensor ( C ijkl
= C eklij ). Since the

both systems are made up by the same material the relationship = C e : holds. With that
the above equation becomes:
ij ij = ij C eijkl kl = kl C eklij ij = kl kl

Tensorial
notation

: = :

If now we integrate over the whole volume we can obtain the Bettis theorem:

: dV = : dV

University of Castilla- La Mancha


Ciudad Real Spain

Draft

(5.148)

By: Eduardo W. V. Chaves (2015)

SOLVING PROBLEMS BY MEANS OF CONTINUUM MECHANICS

442

e
NOTE 1: The above equation is only valid if C ijkl
= C eklij holds, i.e. if C e has major
e
symmetry. In other words, the condition C ijkl
= C eklij enforces the existence of the stored-

energy function ( e ), such as:


e
C ijkl
=

2 e ( ) 2 e ( )
=
= C eklij
ij kl
kl ij

NOTE 2: The Bettis theorem is the start point to obtain the formulation of the Boundary
Element Method.
NOTE 3: The Bettis theorem can also be expressed in another form which we show
below.

2 x j

1 u
Recall that ij = i +

ij =

u j 1
= (u i , j + u j ,i ) , which is also valid for the system II, i.e.
xi 2

1
( ui , j + u j ,i ) . Then:
2

dV = dV
ij ij

ij ij

1
1
ij (ui , j + u j ,i )dV =
ij ( ui , j + u j ,i )dV
2V
2V

ij i , j dV

(5.149)

= ij ui , j dV
V

where ij u i , j = ij u j ,i and ij ui , j = ij u j ,i hold due to the symmetry of and ,


respectively. Also note that:
( ij u i ), j = ij , j u i + ij u i , j

ij u i , j = ( ij u i ), j ij , j u i

( ij ui ), j = ij , j ui + ij ui , j

ij ui , j = ( ij ui ), j ij , j ui

With that the equation in (5.149) becomes:

u
ij

= ij ui , j dV

i , j dV

( u ),

( u ),

ij

ij , j u i dV = ( ij ui ), j ij , j ui dV

ij

(5.150)

dV ij , j u i dV = ( ij ui ), j dV ij , j ui dV

Applying the divergence theorem to the first one integral on both sides of the equation, we
can obtain:

u n dS
ij

ij , j u i dV

= ij ui n j dS ij , j ui dV
S

(5.151)

t i u i dS ij , j u i dV = t i ui dS ij , j ui dV
S

where we have applied the definition n = t and n = t . The above equation in


tensorial notation becomes:

r
t

University of Castilla-La Mancha


Ciudad Real Spain

udS ( ) udV = t u dS ( ) u dV
S

Draft

(5.152)

By: Eduardo W. V. Chaves (2015)

5 INTRODUCTION TO: CONSTITUTIVE EQUATIONS, IBVP, AND IBVP SOLUTION STRATEGIES

443

If we resort to the equations of motion is satisfied that:


r
&r&
+ b = u
r
&r&
+ b = u

r r
&&
= (b u )
r r
&&)
= (b u

with that the equation in (5.152) becomes:

r r r
r r
r r r
r r
&&
&&) u dV
t udS + (b u ) udV = t u dS + (b u

Bettis theorem

(5.153)

Note that, if we consider S = S u + S we have:

r r
t udS =

r r
t u dS =

r r
r r
t * udS + t u* dS

r r
r r
t * u dS + t u * dS

Su

(5.154)

Su

For the particular case when the system is in equilibrium and in the absence of body force,
the equation (5.153) becomes:

r r
r r
t udS = t u dS

(5.155)

In addition, if we have concentrated forces instead of surface force, the above equation
becomes:
r
r
r
r
F loc u loc = F loc u loc

Fi loc u loc
= Filoc uiloc
i

(5.156)

Problem 5.21
Let us consider two systems as described in Figure 5.13. Show the Principal of Virtual
Work which states that:
r r r
r* r
u dS + (b u&&) u dV = : dV

144444244444
3
Total external virtual work

Principle of Virtual Work

V
14243

(5.157)

Total internal
virtual work

where u = u* on S u is known (prescribed).


Solution:
We can prove the Principle of Virtual Work by starting directly from the relationship:

ij ij dV

1
ij ( ui , j + u j ,i )dV = ij ui , j dV
2V
V

Note that ( ij ui ), j = ij , j ui + ij ui , j

ij ij dV

ij ui , j = ( ij ui ), j ij , j ui , thus:

= ij ui , j dV = ( ij ui ), j ij , j ui dV
V

ij ij dV = ( ij ui ), j dV ij , j ui dV
V

University of Castilla- La Mancha


Ciudad Real Spain

(5.158)

(5.159)

Draft

By: Eduardo W. V. Chaves (2015)

SOLVING PROBLEMS BY MEANS OF CONTINUUM MECHANICS

444

by applying the divergence theorem to the first volume integral on the right side of the
equation, we can obtain:

dV = ( u ),
ij ij

ij i

dV ij , j ui dV = ij uin j dS ij , j ui dV
V

(5.160)

t *i ui dS ij , j ui dV

where we have applied the definition n = t * . The above equation in tensorial notation
becomes:

: dV =

r
r r
t * u dS ( ) u dV

(5.161)

&& = (b u
&&) , with
If we use the equations of motion we can obtain + b = u
that the equation in (5.161) becomes:

r r r
r r
&&) u dV = : dV
t * u dS + (b u

144444244444
3
Total external virtual work

V
14243
Total internal
virtual work

which is known as the Principle of Virtual Work. Note that, for the demonstration, we
have not used the major symmetry of C e .
For the particular case when the system is in equilibrium and in the absence of body force,
the above equation becomes:

r r r
t * u ( x )dS = : dV

(5.162)

In addition, if we have concentrated forces instead of surface force, the above equation
becomes:
Tensorial notation
r
F loc

Voigt notation

r
u loc = : dV

{F loc }T {u loc } = { }T { } dV

(5.163)

where the direction of uiloc -component is the same as the Filoc -component direction,
where {F loc } = {F1 , F2 ,..., Fn }T , {u loc } = {U1 , U2 ,..., Un }T .
F2

F1

F3

u
,

REAL

VIRTUAL
Figure 5.14

University of Castilla-La Mancha


Ciudad Real Spain

Draft

By: Eduardo W. V. Chaves (2015)

5 INTRODUCTION TO: CONSTITUTIVE EQUATIONS, IBVP, AND IBVP SOLUTION STRATEGIES

445

NOTE 1: The Principle of Virtual Work states: A structure is in equilibrium, under a


system of external forces, if and only if the total external virtual work equals the total internal
r
virtual work for every virtual displacement field ( u ).
NOTE 2: The Principle of Virtual Work is used to discretization techniques of the
problem such as the Finite Element Technique, in which the fundamental unknown is the
displacement.
NOTE 3: It is easy to show that the equation in (5.157) is also valid for rate of change of
r

&
virtual fields u , & , i.e.:

r r r&
r r&
&&) u dV = : & dV
t * u dS + (b u

S
V
1
44444
244444
3

V
14243

Principle of Virtual Work

(5.164)

Total internal
virtual work

Total external virtual work

Also it is valid for a variation of the virtual field u , i.e.:

r r
r
r
r
&&) u dV = : dV
t * u dS + (b u

1444442444443
Total external virtual work

V
14243

Principle of Virtual Work

(5.165)

Total internal
virtual work

NOTE 4: We can also define the Principle of complementary virtual work as follows:

r r r
r r
&&
t u*dS + (b u ) udV =

S ur

: dV

V
14243

14444
4244444
3

Total internal
complementary virtual work

Total external complementary virtual work

Principle of Complementary
Virtual Work

(5.166)

with n = t * on S . Considering a static case without body forces and that the external
action is characterized by concentrated forces, the principle of complementary virtual work
becomes:
r
loc r loc
F
u43
142

Total external complementary virtual work


(due to concentrated forces)

: dV

V
14243

Total internal
complementary virtual work

Principle of Complementary
Virtual Work (static case
without body forces and with
concentrated forces)

(5.167)

NOTE 5: Note that, if we are using the Principle of Virtual Work the fundamental
unknowns are displacements (strains), if we are using the Principles of Complementary
Virtual Work the fundamental unknowns are forces (stresses), and if we are using the
Bettis reciprocal theorem the fundamental unknowns are displacements and forces
simultaneously (see equation (5.152)).
Problem 5.22
Consider a sub-domain ( ) made up by a homogeneous, isotropic linear elastic material.
Consider also that at some points of the sub-domain boundary there are concentrated
forces {F ( e ) } {F loc } , and that the displacement field into the sub-domain is
r
r
approximated by {u( x )} = [ N ( x )]{u(e ) } where {u( e ) } {uloc } are the displacements at the

University of Castilla- La Mancha


Ciudad Real Spain

Draft

By: Eduardo W. V. Chaves (2015)

SOLVING PROBLEMS BY MEANS OF CONTINUUM MECHANICS

446

points where concentrated forces are applied. Prove that the governing equations for a
linear elastic problem in static equilibrium can be replaced by:
{F (e ) } = [ K ( e ) ]{u(e ) }

[ K ( e ) ] = [B]T [C ] [B] dV

with

(5.168)
r

where [C ] is the elasticity tensor in Voigt notation, and obtain an expression for [B( x )] .
Hint: Use the Principle of Virtual Work.
Solution:
We can start directly from the equation in (5.163), which is equivalent to:
r
r r
F u = : dV = : ( sym u ) dV

r r
r
u F = ( sym u ) : dV

(5.169)

The above equation in Voigt notation becomes:


r r
r
u F = ( sym u ) : dV Voigt

{u (e ) }T {F ( e ) } = { }T { } dV

(5.170)

Note that, the above equation is already considering the equilibrium equation (see equations
(5.161)-(5.163)). The constitutive equation in stress, in Voigt notation is given by
r
r
{ ( x )} = [C ]{ ( x )} , where the strain tensor field is given by the kinematic equations

r
r
( x ) = sym u . In Problem 5.8 we have obtained the symmetric part of the displacement

field gradient, ij = 12 (u i , j + u j ,i ) , in Voigt notation, i.e.:


u1
x
x
1
1

u2

0
11

x2

22 u3 0
r

33 x3
{ ( x )} =
= u u =
212 1 + 2
2 23 x2 x1 x2

u
u
213 2 + 3 0

x3 x2
u1 + u3

x
3 x1 x3

x2
0

x1

x3
0

u
1
x3
u
2
0 u
3


x2

x1

r
r
{ ( x )} = [ L(1) ]{u( x )}

then taking into account that {u( x )} = [ N ( x )] {u(e ) } the above equation becomes:
r
r
r
r
{ ( x )} = [ L(1) ] {u( x )} = [ L(1) ] [ N ( x )]{u(e ) } = [B( x )] {u( e) }

where
r
r
[B( x )] = [ L(1) ] [ N ( x )]

(5.171)

The stress field can be expressed as follows:

{ ( xr )} = [C ] { ( xr )} = [C ] [B( xr )] {u(e) }

We can adopt the same displacement field approach to approximate the virtual
displacement field, with which we can obtain:

{u ( xr )} = [ N ( xr )]{u (e) }
University of Castilla-La Mancha
Ciudad Real Spain

Draft

r
r
{ ( x )} = [B( x )] {u ( e ) }
By: Eduardo W. V. Chaves (2015)

5 INTRODUCTION TO: CONSTITUTIVE EQUATIONS, IBVP, AND IBVP SOLUTION STRATEGIES

447

Then, the equation in (5.170) becomes:

{u ( e) }T {F ( e ) } = { }T {} dV =
V

r
r
{[B( x)]{u } } [C ][B( x)]{u
(e)

(e)

} dV

or:
r
r
{u ( e ) }T {F ( e ) } = {u ( e ) }T [B( x )]T [C ][B( x )]{u( e ) } dV

(5.172)

Note that neither {u (e ) } nor {u (e) } depend on x , then:

r
r
{u ( e) }T {F ( e ) } = {u ( e ) }T [B( x )]T [C ][B( x )] dV {u(e ) }

Since the vector {u (e ) } is arbitrary, we can conclude that

r
r
{F ( e ) } = [B( x )]T [C ][B( x )] dV {u(e ) }

(5.173)

{F ( e ) } = [ K ( e) ]{u( e) }

NOTE: [ K (e ) ] is known as the stiffness matrix of the sub-domain (finite element), and the
r

matrix [ N ( x )] from the relationship {u( x )} = [ N ( x )]{u(e ) } is known as the shape function
matrix. The shape functions are functions defined into the domain that allows us to obtain
r
{u( x )} at any point of the domain through the nodal values of the function {u (e ) } . For
example, let us suppose that the sub-domain ( ) is characterized by a quadrilateral then
r
we can obtain the value of the scalar field T ( x ) by means of its nodal values {T (e ) } as
follows:
T (e) _ 1
(e) _ 2
r
r
r T

(e)
T ( x ) = [ N ( x )]{T } = [ N ( x )] ( e ) _ 3
T

(e) _ 4
T

T (e ) _ 2
T (e ) _ 3

r
T ( x)

T (e ) _ 1

T (e ) _ 4

Problem 5.23
a) Consider a sub-domain ( ) made up by a homogeneous, isotropic linear elastic material.
Consider also that at some points of the sub-domain boundary there are concentrated
forces {F ( e ) } {F loc } (nodal forces), and that the displacement field is approximated by
r
r
{u( x )} = [ N ( x )]{u(e ) } where the nodal displacements {u ( e ) } {u loc } are the displacements
at the points where concentrated forces are applied. Prove that the governing equations for
a linear elastic problem can be replaced by:
&& ( e ) } = {F ( e ) }
[ K ( e ) ]{u( e ) } + [ M ( e ) ]{u

University of Castilla- La Mancha


Ciudad Real Spain

Draft

(5.174)

By: Eduardo W. V. Chaves (2015)

SOLVING PROBLEMS BY MEANS OF CONTINUUM MECHANICS

448

where [ K ( e ) ] = [B]T [C ] [B] dV (stiffness matrix), [ M ( e ) ] = [ N ]T [ N ] dV (mass matrix),


V

[C ] is the elasticity tensor in Voigt notation.

b) Show that the equation in (5.174) represents a conservative system.


Hint: Use the Principle of Virtual Work and consider the following approximations for the
fields:
r
r
{u( x )} = [ N ( x )]{u(e ) } (displacement field)

{u& ( xr )} = [ N ( xr )]{u& (e ) } (velocity field)

r
&& r
&& (e )
{u( x )} = [ N ( x )]{u } (acceleration field)
r
r
r
(1)
(e)
(e)
{ ( x )} = [ L ] [ N ( x )]{u } = [B( x )]{u } (strain field)

r
r
(e)
{ ( x )} = [C ]{ ( x )} = [C ][B]{u } (stress field)

r
r
r
{t * ( x )} = [Nt ( x )]{ f r(e ) } (vector traction field)
t

r
r
(e)
b r
{b( x )} = [N ( x )]{ f br } (body force field)

(5.175)

Use the same approximations for the respective virtual fields.


Solution:
The Principle of Virtual Work states that:

r r r
r r
&&) u dV
: dV = t * u dS + (b u

V
14243
Total internal
virtual work

144444V2444443
Total external virtual work

We rewrite the above equation using Voigt notation:

{} {} dV = {t} {u} dS + {b} {u} dV {u&&} {u} dV


T

&&} dV
= {u} {t} dS + {u} {b} dV {u}T {u
T

(5.176)

Using the adopted approximations, (see Eq. (5.175)), we can obtain the following terms:

{} {} dV = {} {} dV = {[B]{u }} {[C ][B]{u }}dV =


T

(e)

(e)

= {u (e ) }T [B]T [C ][B]{u( e ) } dV = {u ( e ) }T [B]T [C ][B] dV {u( e ) }

= {u ( e ) }T [ K (e ) ]{u( e ) }

University of Castilla-La Mancha


Ciudad Real Spain

Draft

By: Eduardo W. V. Chaves (2015)

5 INTRODUCTION TO: CONSTITUTIVE EQUATIONS, IBVP, AND IBVP SOLUTION STRATEGIES

{u}T {t} dS =

{[ N ]{u }} [N ]{ f
(e)

r
t

r( e ) } dS
t

449

= {u (e ) }T [ N ]T [Nt ]{ f tr( e ) } dS

r
r
= {u ( e ) }T [ N ]T [Nt ] dS { f tr(e ) } = {u ( e ) }T [Gt ]{ f tr( e ) } = {u ( e ) }T {Frt( e ) }

{u}T {b} dV =

{[ N ]{u }} [N ]{ f
(e)

r
b

= {u (e ) }T [ N ]T [Nb ] dV { fbr( e ) }

r
r
= {u (e ) }T [Gb ]{ fbr(e ) } = {u ( e ) }T {Fbr(e ) }

{u} {u&&} dV = {[ N ]{u }} [ N ]{u&&


T

r( e ) } dV
b

(e)

(e)

&& ( e ) } dV
} dV = {u ( e ) }T [ N ]T [ N ]{u

(e)

&& } = {u ( e ) }T [ M ( e ) ]{u
&& (e ) }
= {u ( e ) }T [ N ]T [ N ] dV {u

Taking into account the above relationships into the equation (5.176) we can obtain:
&& (e ) }
{u ( e ) }T [ K ( e ) ]{u( e ) } = {u (e ) }T {Ftr( e ) } + {u ( e ) }T {Fbr( e ) } {u (e ) }T [ M ( e ) ]{u

&& ( e ) }
{u ( e ) }T [ K ( e ) ]{u(e ) } = {u ( e ) }T {Ftr(e ) } + {Fbr(e ) } [ M ( e ) ]{u

Since the virtual displacement {u (e ) } is arbitrary we conclude that:


&& ( e ) }
[ K (e ) ]{u( e ) } = {Ftr(e ) } + {Fbr( e ) } [ M ( e ) ]{u
&& (e ) } = {Fr( e ) } + {Fr( e ) }
[ K (e ) ]{u( e ) } + [ M ( e ) ]{u
t
b
&& (e ) } = {F ( e ) }
[ K (e ) ]{u( e ) } + [ M ( e ) ]{u

Q.E.D.

b) To show that the above system is conservative we will consider the discretization of
time where the current time we denote by t and the next time by t + t , where t is the
time increment. In any time the above equation is true so:
&& ( e ) }t = {F ( e ) }t
[ K (e ) ]t {u( e ) }t + [ M (e ) ]t {u
(e)
(e)
(e)
&& ( e ) }t + t = {F (e ) }t + t = {F ( e ) }t
[ K ]t + t {u }t + t + [ M ]t + t {u
&& ( e ) }t + t = [ K ( e ) ]t {u( e ) }t + [ M ( e ) ]t {u
&& ( e ) }t
[ K (e ) ]t + t {u( e ) }t + t + [ M ( e ) ]t + t {u

where the force {F (e ) } is constant with time. Notice that, if the vector {F (e ) } is constant
with time we can obtain:

(
(

)
)

D
&& (e ) } = D {F (e ) } = {0}
[ K (e ) ]{u(e ) } + [ M (e ) ]{u
Dt
Dt
(e)
(e)
( e ) && ( e )
[ K ]{u } + [ M ]{u } = constant in time

NOTE 1: The equation in (5.174) is a forced harmonic motion. Let us consider the onedimensional case where [ K (e ) ] represents the spring constant k , [ M (e) ] represents the
mass m , and the displacement and acceleration are represented by u and u&& respectively,
(see Figure 5.15). With that the equation in (5.174) without applied force becomes:
ku + mu&& = 0

University of Castilla- La Mancha


Ciudad Real Spain

Draft

ku = mu&&

By: Eduardo W. V. Chaves (2015)

SOLVING PROBLEMS BY MEANS OF CONTINUUM MECHANICS

450

u
t

b) Mechanical model (mass+spring)

b) Displacement vs. time

Figure 5.15: Mechanical model for simple harmonic motion.


Note that the energy is conserved. Considering the internal energy for the spring ( 12 uku )
and the kinetic energy ( 12 u&mu& = 12 mv 2 ) for the particle of mass m , and by apply the energy
equation we can obtain:
DK DU DW DQ
+
=
+
=0
Dt
Dt 1Dt
Dt
4243
=0

D 1

D 1
uku = 0
u&mu& +
Dt 2

Dt 2
mu&&u& + kuu& = 0
(mu&& + ku )u& = 0

s =W

(5.177)

mu&& + ku = 0

where K is the kinetic energy, U is the internal energy. The equation mu&& + ku = 0 is
denoted by the simple harmonic motion, (see Figure 5.15).
NOTE 2: If we do the experiment using the mechanical model described in Figure 5.15
we will observe that the motion in reality is not conservative, i.e. there is dissipation of
energy. In other word, there is damping of the system until the rest is achieved, (see Figure
5.17). This phenomenon occurs due to the internal mechanisms of the structures. In
practice, this damping intrinsic of the structures can be dealt by means of the parameter d
(damping) multiply by velocity, (see Figure 5.16).
k

FS = ku
FI = mu&&
d

u (displacement)

F (t )

FD = du&

Figure 5.16: Mechanical model (mass+dashpot+spring).

University of Castilla-La Mancha


Ciudad Real Spain

Draft

By: Eduardo W. V. Chaves (2015)

5 INTRODUCTION TO: CONSTITUTIVE EQUATIONS, IBVP, AND IBVP SOLUTION STRATEGIES

451

Taking into account the discrete mechanical model described in Figure 5.16 and by
applying the force equilibrium we can obtain:
FI + FD + FS = F (t )

mu&& + du& + ku = F (t )

And the equation in (5.174) can be rewritten in order to take the damping effect as follows:
&& ( e ) } = {F ( e ) }
[ K ( e ) ]{u( e ) } + [ D (e ) ]{u& ( e ) } + [ M (e ) ]{u

(5.178)

where [ D (e ) ] is the damping matrix. Note that to solve the equation (5.178) we need to
integrate over time.
u

Figure 5.17: Displacement vs. time Mechanical model with damping.


Reference (Structural Dynamics)
CLOUGH, R.W. & PENZIEN, J. (1975). Dynamic of Structures. McGraw-Hill Companies.
TEDESCO, J.M.; MCDOUGAL, W.G. & ROSS, C.A.(1998). Structural dynamics: theory and
applications. Addison Wesley Longman, Inc.
Problem 5.24
For a equilibrium system let us consider the total potential energy defined as follows:
r r
r r
r
(u) = e ( ) dV t * udS ( b) udV

The total potential energy

(5.179)

where

U int = e ( ) dV =
V

2 : dV

(5.180)

and

University of Castilla- La Mancha


Ciudad Real Spain

Draft

By: Eduardo W. V. Chaves (2015)

SOLVING PROBLEMS BY MEANS OF CONTINUUM MECHANICS

452

r r
r r
U ext = t * udS + ( b) udV

(5.181)

Also let us consider that the first variation of , denoted by , equals cero for a
r
stationary value of . Show that, if = 0 is equivalent to a stationary value , so (u)
assume a minimum value.
r r

Obs.: Consider that during the deformation process, the external actions ( t * , b ) do not
vary, and also consider a linear elastic material.
Solution:
The first variation ( ) can be obtained as follows:
1
r r
r r
: dV t * udS ( b) udV
=

2
S
V

V
r
r
r
r
1
= : dV t * udS ( b) udV
2
V
S
V

(5.182)

r
r
r
r
1
( : ) dV t * udS ( b) udV
2
S
V

Note that:

1
1
1
e ( ) = ( : ) = ( : + : ) = (C e : ) : + :
2
2
2
1
1
1
= (C e : ) : + : = : C e : + : = [ : + : ]
2
2
2
= :

] [

(5.183)

e
:

where we have considered =

e
(see Problem 5.5). For small deformation regime we

can also write the above equation as follows:


e ( ) =

r
r
r
e
: = : = : ( symu) = : ( sym u) = : (u)

(5.184)

where we have used the property A sym : B = A sym : (B sym + B skew ) = A sym : B sym . Then, the
equation in (5.182) becomes:
r
r
r
r
1
( : ) dV t * udS ( b) udV
2
V
S
V
r
r* r
r
= : dV t udS ( b) udV

r
r
r
r
= e dV t * udS ( b) udV

The expression (u + u) can be obtained as follows (see equation (5.179)):

University of Castilla-La Mancha


Ciudad Real Spain

Draft

By: Eduardo W. V. Chaves (2015)

5 INTRODUCTION TO: CONSTITUTIVE EQUATIONS, IBVP, AND IBVP SOLUTION STRATEGIES

r r
r r
r
r
r
r
(u + u) = e ( + ) dV t * (u + u)dS ( b) (u + u)dV

453

(5.185)

By using the Taylor series to approach e ( + ) we can obtain:

e ( + ) = e () +
Note that

1
e ( )
2 e ( )
: + :
: + ...
2

(5.186)

2 e ( )
e ( )
: = e (see equation (5.183)) and C e =
(see Problem 5.5),

with which the equation in (5.186) becomes:

1
e ( )
2 e ( )
: + :
: + ...
2


1
e ( ) + e + : C e :
2

e ( + ) = e () +

and by replace the above equation into the equation (5.185) we can obtain:
r r
r r
r
r
r
r
(u + u) = e ( + ) dV t * (u + u)dS ( b) (u + u)dV

= e ( ) dV + e dV +
V

2 : C

: dV

r r
r r
r
r
t * (u + u)dS ( b) (u + u)dV

r r
r r
= e ( ) dV t * udS ( b) udV +

r
r
r
r
1
+ e dV t * udS ( b) udV +
: C e : dV
2
V
S
V
V

(5.187)
Note that:

r r
r r
r
(u) = e ( ) dV t * udS ( b) udV

and

r
r
r
r
= e dV t * udS ( b) udV = 0

Taking into account the previous equations into the equation in (5.187) we can conclude
that:
r
r
r
1
(u + u) = (u) + +
: C e : dV
2
V

r
r
r
1
(u + u) (u) = +
: C e : dV
2
V

r
r
r
1
(u + u) (u) =
: C e : dV
2
V

University of Castilla- La Mancha


Ciudad Real Spain

Draft

By: Eduardo W. V. Chaves (2015)

SOLVING PROBLEMS BY MEANS OF CONTINUUM MECHANICS

454

where we have considered = 0 . Note that the term : C e : > 0 is always positive for
any valor of since C e is a positive definite tensor (see Chapter 1). Then, we guarantee
that:
r
r
r
r
r
r
1
= (u + u) (u) =
: C e : dV > 0
(u + u) > (u)
2
V
r
= 0
(u) is a minimum
So,

NOTE 1: For a system characterized by a linear elastic problem, the equilibrium point
corresponds to the minimum value of , (see Figure 5.18). This is known as the principle of
minimum potential energy.
NOTE 2: When the external action is characterized by concentrated forces and in the
absence of body forces, the equation (5.179) becomes:
r
1
(u) = U int + U ext =
: dV {F loc }T {uloc }
2
V

The total potential energy

(5.188)

(u )
F

(u ( 2 ) )
(u )

Deformation corresponding to
the equilibrium

=0
u

(u ( 3 ) )

u (2)

u ( 3)

Figure 5.18
NOTE 3: By means of equation (5.188), it is easy to show the Castiglianos Theorem Part I:
r
(u)
loc

{u }

{F loc } =

{F loc }T {u loc }
U int
U ext
U int
+
=

=0
{u loc }
{u loc }
{u loc }
{u loc }
U int
{u loc }

where {F loc } = {F1 , F2 ,..., Fn }T , {u loc } = {U1, U2 ,..., Un }T . Note that the term U int must be in
function of {uloc } .

University of Castilla-La Mancha


Ciudad Real Spain

Draft

By: Eduardo W. V. Chaves (2015)

5 INTRODUCTION TO: CONSTITUTIVE EQUATIONS, IBVP, AND IBVP SOLUTION STRATEGIES

455

NOTE 4: For better illustration of the proposed problem, we will consider a rod of length
L and cross-sectional area A . Consider also that the stress and strain fields are
homogeneous and given by:
0 0
ij = 0 0 0
0 0 0

0 0
e
ij = 0 0 0 11 = C1111
11 = E
0 0 0

Consider also that the displacement field is approached by a linear function


( u ( x) = a1 + a2 x ), and that on the extremities of the rod, we have the forces F (1) , F ( 2) , and
the nodal displacements U (1) , U ( 2) , (see Figure 5.19).

(3D )

V = AL (volume)

F = dA = A
(1)

F ,U
(1D )

(1)

( 2)

( 2)

,U

Figure 5.19: Rod under axial force.


The goal now is to express the total potential energy in terms of U(1) ,U( 2) . Note that, due to
the concentrated forces we have:
U ext = {F loc }T {uloc } = {F (1)

U(1)
F ( 2 ) } ( 2 ) = F (1)U(1) + F ( 2 )U( 2 )
U

(5.189)

For this case, (see Problem 5.5 - NOTE 3), the linear stress-strain relationship is given by
1
1
= E , and the strain energy density by e = = E . Then, the total internal energy
2
2

is given by:
U int =

2 : dV

1D

U int =

note that 11 =

1
1
1
dV =
E dV =
E 2 dV
2V
2V
2V

u1 u ( x)
=
= , thus
x1
x

U int =

1
1
u ( x)
E 2 dV =
E
dV
2V
2 V x

(5.190)

Our goal now is to express the displacement field in terms of their nodal values U(1) ,U( 2) .
To do this we use the approach adopted u ( x) = a1 + a2 x , where:

University of Castilla- La Mancha


Ciudad Real Spain

Draft

By: Eduardo W. V. Chaves (2015)

SOLVING PROBLEMS BY MEANS OF CONTINUUM MECHANICS

456

= a1 + a2 L

u ( x = 0) = U(1) = a1
u ( x = L ) = U( 2 )

U(1) 1 0 a1
( 2) =

U 1 L a2

Next we evaluate the coefficients a1 and a 2 . To do this, we obtain the reverse form of the
above relationship, i.e.:
U(1) 1 0 a1
(2) =

U 1 L a2

a1 1 L 0 U(1)
=
( 2)
a2 L 1 1 U

reverse

a1 = U(1)

1 (2)
(1)
a 2 = U U
L

with which we can obtain the displacement field in terms of its nodal values:
u ( x) = a1 + a2 x = U(1) +

1 (2)
x
x

U U(1) x = 1 U(1) + U( 2 )
L
L
L

(5.191)

(1)
r

x x U
u ( x) = 1 ( 2 ) = [N ( x )]{u( e ) }
L L U

and the equation in (5.190) becomes:


U

int

2
1
E 1 (2)
E
u ( x)
( 2) 2
(1)
2U(1)U( 2 ) + U(1) dV
=
E
dV =
U U dV = 2 U

2 V x
2VL
2L V

Note that U(1) and U( 2) are not dependent of x , then:


U int =

2
2
2
E (2) 2
E
2U(1)U( 2 ) + U(1) dV = 2 U( 2 ) 2U(1)U( 2 ) + U(1) V
U
2

2L
2L
V

(5.192)

2
2
EAL ( 2) 2
EA ( 2 ) 2
=
2U(1)U( 2 ) + U(1) =
2U(1)U( 2 ) + U(1)
U
U
2
2L

2L

Then, the total potential energy (see equation (5.188)) is given by equations (5.189) and
(5.192), i.e.:

r
2
EA ( 2 ) 2
(u) = U int U ext =
2U(1)U( 2 ) + U(1) F (1)U(1) + F ( 2 )U( 2 ) = (U(1) , U( 2 ) )
U

2L

As we look for a stationary state, the following must be true:


(U (1) , U ( 2 ) )

U (1)

(1)
( 2)
(U , U )

U ( 2 )

) = 0

) = 0

2
EA ( 2 ) 2
2U (1) U ( 2) + U (1) F (1) U (1) + F ( 2 ) U ( 2)
U
(1)

U 2 L
EA
=
2U ( 2 ) + 2U (1) F (1) = 0
2L
EA (1)
U U ( 2 ) F (1) = 0
=
L
2
EA ( 2 ) 2
=
2U (1) U ( 2) + U (1) F (1) U (1) + F ( 2 ) U ( 2)
U
( 2)

U 2 L

(
(

EA
2U ( 2 ) 2U (1) F ( 2 ) = 0
2L
EA ( 2 )
U U (1) F ( 2 ) = 0
=
L

Rearranging the above equations in matrix form we can obtain:

University of Castilla-La Mancha


Ciudad Real Spain

Draft

By: Eduardo W. V. Chaves (2015)

5 INTRODUCTION TO: CONSTITUTIVE EQUATIONS, IBVP, AND IBVP SOLUTION STRATEGIES


(1)
(1)
EA 1 1 U F

L 1 1 U( 2 ) F ( 2 )

[ K ( e ) ]{u( e ) } = {F ( e ) }

457

(5.193)

Note that [ K (e ) ] has no inverse, since det[ K (e ) ] = 0 , which indicates that the problem has
infinity solution since we have not imposed any restriction in motion. To solve the
problem we have to introduce the boundary conditions.
Note that the matrix [ K (e ) ] of the above equation could have been obtained by means of
the equation (5.173) (see Problem 5.22), and for this particular case we have [C ] = E , then,
the equation (5.191) becomes:
r
r

x x 1 1
[B( x )] = [ L(1) ][ N ( x )] =
1 =

x L L L L

thus
1
L
r
r
1 1 1
1 1
[ K (e ) ] = [B( x )]T [C ][B( x )] dV =
E dV = E 2
dV

1
1
L
L
L
1

V
V
V

L

E
L2

1 1
E 1 1
EA 1 1
1 1 dV = 2 1 1 V =

L 1 1
L

NOTE 5: It is interesting to note that, initially we had a continuum problem (infinity of


material points) represented by its governing equations and we had transformed this
continuum problem into a set of discrete equations (5.193). In other words, we had applied
a numerical technique to solve the problem. The main goal of any numerical technique is
transformed the continuum governing equations into a set of discrete equations, between
these techniques we can quote: Finite Differences, Finite Element, Boundary Element,
Finite Volume, etc.
r

NOTE 6: Analyzing [ N ( x )]
T

r
r
r

x x
Note that the shape functions are [ N ( x )] = [N1 ( x ) N 2 ( x )]T = 1 . If we draw
L L

these functions inside the domain we can obtain:


N1

N1 ( x = 0) = 1
x

L
N1 ( x = L ) = 0
N 2 ( x = 0) = 0
x
N 2 ( x) =
L
N 2 ( x = L) = 1

N2

N1 ( x ) = 1

N1 ( x ) + N 2 ( x ) = 1

Figure 5.20: Shape function (linear approach)

University of Castilla- La Mancha


Ciudad Real Spain

Draft

By: Eduardo W. V. Chaves (2015)

SOLVING PROBLEMS BY MEANS OF CONTINUUM MECHANICS

458

The adopted approximation for [ N ( x )] will depend on the problem. For the previous
problem we have that the strain is constant into the domain, so, it is sufficient to adopt a
linear approximation for displacement since by definition =
r

u ( x)
. As consequence we
x

need only two points on the boundary to define [ N ( x )] . If a problem requires a quadratic
r
function for displacement approximation, so, we will need three points to define [ N ( x )] ,
and so on.
NOTE 7:

Principle of the Stationarity of Potential Energy


In this problem we have establish the principle of the stationarity of Potential Energy, (see
equation in (5.179)):
r r
r r
r
(u) = e ( ) dV t * udS ( b) udV

(5.194)

1
2

where we have considered e () = : . The functional is stationary if and only if


r
ur (u) = 0 .

Hellinger-Reissners Variational Principle


In Problem 5.5 (see NOTE 7) have established that
tensorial

e ( ) = : e ( ) = 0 G( ) = g( )
e () = e ()
e ( ) = : g( )

(5.195)

where g ( ) is the Gibbs free energy density with reversed sign.


g() - Complementary strain energy

e () - Strain energy

Figure 5.21: Strain energies.


By replacing e ( ) = : g( ) into the functional (5.194) we can obtain:
r r
r r
r
(u) = e ( ) dV t * udS ( b) udV

r r
r r
r
HR (u, ) = : g( ) dV t * udS ( b) udV

University of Castilla-La Mancha


Ciudad Real Spain

Draft

(5.196)

By: Eduardo W. V. Chaves (2015)

5 INTRODUCTION TO: CONSTITUTIVE EQUATIONS, IBVP, AND IBVP SOLUTION STRATEGIES

459

Note that : = : ( sym u) = : (u) . Then, we obtain:


r r
r r
r
r
HR (u, ) = : (u) g( ) dV t * udS ( b) udV

(5.197)

The functional (5.196) is stationary for variation of u vanishing on S ur if and only if


satisfies the equilibrium equations, and is stationary for variation of if and only if they
satisfy the constitutive equation (strain-stress).
r
r
r
r
r
r
ur HR (u, ) = : (u) dV t * udS ( b) udV = 0

= ij (u)i , j dV t*i (u)i dS ( b)i (u)i dV = 0


V

= ij (u)i n j dS ij , j (u)i dV t *i (u)i dS ( b)i (u)i dV = 0


S

= ij , j + ( b)i (u)i dV +
V

[ n
ij

S
j

(5.198)

t *i (u)i dS = 0

In the volume we obtain the equilibrium equations: ij , j + ( b) i = 0 i .


On surface S we obtain the boundary condition in stress: ij n j t *i = 0 i
r
r
HR (u, ) = : ( sym u) g( ) dV = 0

r
g( )
: dV = 0
= ( sym u) :

(5.199)

r
g( )

: dV = 0
= ( sym u)

V

In the volume we obtain the constitutive equation for strain: ( sym u)

g( )
=0.

Hu-Washizus Variational Principle


The Hu-Washizus principle is a generalization of the Hellinger-Reissners principle, in
r
which the functional, in addition of the independent fields (u, ) , is also dependent of field:

r r
r r
r
r
r r
HW (u, , ) = e ( ) : ( symu) ( b) u dV ( n ) (u* u)dS t * udS
V

S ur

(5.200)
and is stationary if and only if:
r
ur HW (u, , ) = 0
r r
u = 0 on Sur

r
HW (u, , ) = 0

University of Castilla- La Mancha


Ciudad Real Spain

Equilibrium equations

Kinematic Equations

Boundary condition on Sur

Draft

By: Eduardo W. V. Chaves (2015)

SOLVING PROBLEMS BY MEANS OF CONTINUUM MECHANICS

460

r
HW (u, , ) = 0

Constitutive equations for stress

That is:

r
ur HW (u, , ) = 0

[ : ( ur ) ( br ) ur ] dV tr ur dS
r
r
r
r
r
r
(u, , ) = [( ) (u) ( b) u] dV t udS
r
r
r
r
r
(u, , ) = [( ) ( b)] (u) dV t udS

r
ur HW (u, , ) =

sym

ur HW

ur HW

r
HW (u, , ) = 0
r
r
r r
HW (u, , ) = : ( symu) dV ( n ) (u* u)dS = 0

S ur

r
r
HW (u, , ) = ( symu) : dV
V

[n (u

r*

r
u) : dS = 0

S ur

r
HW (u, , ) = 0
r
HW (u, , ) = e ( ) : ( ) dV = 0

r
e ( )

e ( )

HW (u, , ) =
: : ( ) dV =
: dV = 0

V
V

NOTE 8: Discretization of the Fields


The variation of the Hu-Washizus principle can be written as follows

r r
r r
r
HW = e ( ) : ( symu) ( b) u dV t * udS = 0
V

r
r
r
r
r
= e ( ) : dV : ( symu) dV ( b) u dV t * udS = 0

r
r
r
r r
= : e ( ) dV + : ( symu ) dV u ( b)dV u t *dS = 0

r
r
r
r r
= : dV + : ( symu ) dV u ( b)dV u t *dS = 0

In the implementation of finite element methods we often use Voigt notation when we are
dealing with symmetric matrix. Using Voigt notation the above equation becomes:

University of Castilla-La Mancha


Ciudad Real Spain

Draft

By: Eduardo W. V. Chaves (2015)

5 INTRODUCTION TO: CONSTITUTIVE EQUATIONS, IBVP, AND IBVP SOLUTION STRATEGIES

461

HW = {}T {} dV + {}T { symu }dV {u}T {b} dV {u}T {t *} dS = 0


V

(5.201)

{}T {} dV + {}T { symu }dV = {u}T {b} dV + {u}T {t *} dS


V

Consider as approximation for displacement, strain, and stress fields, respectively, (see
Jirsek (1998)), as follows:
{u} [N ]{d} + [N c ]{d c }

{} [B]{d} + [G]{e}

{} [S]{s}

and

{u} [N ]{d} + [N c ]{d c }

{} [B]{d} + [G]{e}

{} [S]{s}

(5.202)

where the matrices [N ] and [B] contain the displacement interpolation functions and their
derivatives (strain interpolation matrix), respectively. [N c ] and [G] are matrices containing
some enrichment terms for displacement and strain respectively. [S] is a stress
interpolation matrix. {d} , {d c } , {e} and {s} collect the degrees of freedom corresponding
to nodal displacement, enhanced displacement modes, enhanced strain modes, and stress
parameters, respectively. If we consider the variation of the Hu-Washizus principle:

{} {} dV + {} {
T

V
144244
3

sym

u dV = {u}T {b} dV + {u}T {t *} dS

V 44424443
1

(5.203)

S
144444
424
44444
3

we can obtain:

(1) {}T {} dV =
V

{[B]{d}+ [G]{e}} {} dV = {[B]{d}} {} dV + {[G]{e}} {} dV


T

= {d} [B]T {} dV + {e}


T

[G] {} dV
T

(2) {}T symu dV =


V

{} {
T

sym

V
144424443

2.1

University of Castilla- La Mancha


Ciudad Real Spain

u dV + {}T symu dV

Draft

V
1444
424444
3

2.2

By: Eduardo W. V. Chaves (2015)

SOLVING PROBLEMS BY MEANS OF CONTINUUM MECHANICS

462

(2.1)

{} {
T

sym r

u dV =

{[S]{s}} { ([N]{d} + [N ]{d }) ([B]{d} + [G]{e})}dV


T

sym

[S] { ([N]{d} + [N ]{d })}dV {s} [S] {([B]{d} + [G]{e})}dV

= {s}

sym

[S] { ([N]{d})}+ [S] { ([N ]{d })}dV {s} [S] {([B]{d} + [G]{e})}dV

= {s}

sym

sym

[S] [B]{d} + [S] [B ]{d }dV {s} [S] [B]{d} + [S] [G]{e}dV

= {s}

[S] [B ]{d }dV {s} [S] [G]{e}dV

= {s}

= {s}

[S] { [B ]{d } [G]{e} }dV


T

where we have considered { sym ([N ]{d})}= [B]{d} and { sym ([N c ]{d c })}= [B c ]{d c } .

r
(2.2) {}T sym u dV =

sym

u {}dV

sym

([N ]{d} + [N c ]{d c }) ([B]{d} + [G]{e}) } {[S]{s}}dV


T

{ ([N ]{d}) + ([N ]{d }) ([B]{d} + [G]{e}) } {[S]{s}}dV


sym

sym

{ [B]{d} + [B ]{d } ([B]{d} + [G]{e}) } {[S]{s}}dV


T

{ [B ]{d } [G]{e} } {[S]{s}}dV = { {d } [B ] {e} [G] }{[S]{s}}dV


T

= {d c } [B c ]T {[S]{s}}dV {e} [G]T {[S]{s}}dV


T

(3) {u}T {b} dV + {u}T {t *} dS = {d} { f ext } + {d c } { f c }

Taking into account the previous terms, the equation in (5.203) becomes:

{d}T [B]T {} dV + {e}T [G]T {{} {[S]{s}}} dV + {s}T [S]T { [B c ]{d c } [G]{e} }dV +
V

+ {d c } [B c ]T {[S]{s}}dV = {d} { f ext } + {d c } { f c }


T

(5.204)

University of Castilla-La Mancha


Ciudad Real Spain

Draft

By: Eduardo W. V. Chaves (2015)

5 INTRODUCTION TO: CONSTITUTIVE EQUATIONS, IBVP, AND IBVP SOLUTION STRATEGIES

463

Since {u} , {} and {} are variables of the independent fields, so, we can say that:

T
T
{d} [B]T {} dV = {d} { f ext }

T
T
{e} [G] {{} {[S]{s}}} dV = {0}

T
T
{s} [S] [B c ]{d c } [G]{e} dV = {0}
V

{d }T [B ]T {[S]{s}}dV = {d }T { f } = {0}
c
c
c
c
V

(5.205)

If we consider { f c } = {0}, the above equations can be rewritten as follows:

[B]T {} dV = { f ext }
V

T
[G] {{} {[S]{s}}} dV = {0}
V

T
[S] [B c ]{d c } [G]{e} dV = {0}
V

[B ]T {[S]{s}}dV = {0}
c

(5.206)

Taking into account that the stress-strain relationship is given by the following expression:
{} = [C ] {} = [C ] {[B]{d} + [G]{e}}

(5.207)

and by substituting into the equation in (5.206) we can obtain:

[B]T [C ][B] dV {d} + [B]T [C ][G] dV {e} = { f ext }


V
V

T
T
T
[G] [C ][B] dV {d} + [G] [C ][G] dV {e} [G] [S] dV {s} = {0}
V
V
V

T
T
[S] [B c ] dV {d}c [S] [G] dV {e} = {0}
V
V

[B ]T [S] dV {s} = {0}


c

University of Castilla- La Mancha


Ciudad Real Spain

Draft

(a)
(b)
(5.208)
(c)
(d)

By: Eduardo W. V. Chaves (2015)

SOLVING PROBLEMS BY MEANS OF CONTINUUM MECHANICS

464

Rewriting the above equation in matrix form we can obtain:

0
dV
[S]T [B c ]

[B]T [C ][B] [B]T [C ][G]


0
T
T
T
[G] [C ][B] [G] [C ][G] [G] [S]
T

0
0
[S] [G]

[B c ]T [S]
0
0

{d} { f ext }
{e} {0}

{
}
s
{
}
0

{d c } {0}

(5.209)

Let us suppose that we do not introduce any displacement enhancement terms, thus
{d c } = {0} [B c ] = [0] , with that the equation in (5.208)(c) becomes:

[S] [G] dV {e} = {0}


T

(5.210)

Thus, piecewise constant stress functions {} will require [S] = [1] (unit matrix). The
compatibility conditions (5.210) now read:

[G] dV = [0]

(5.211)

Discontinuity on displacement and strain fields Applying the Principle of Virtual


Work
As we have seen before, virtual work is the work done by real force acting through virtual
displacements. A virtual displacement is any displacement consistent with the constraints
of the structure, i.e. which satisfies the boundary conditions.
The principle states that the virtual work of the internal forces must be equal to the virtual
work of the external forces:

: dV =

V
14243
Total internal
virtual work

r r r
r r
&&) u dV
t * u dS + (b u

S
V
1
44444
244444
3
Total external virtual work

(5.212)

Wext

Wint

for all the admissible virtual displacements u .


Let us consider a discretized system where we can say that all the forces are applied in the
nodes of the finite element (CST-Constant Strain Triangle) (see Figure 5.22).

University of Castilla-La Mancha


Ciudad Real Spain

Draft

By: Eduardo W. V. Chaves (2015)

5 INTRODUCTION TO: CONSTITUTIVE EQUATIONS, IBVP, AND IBVP SOLUTION STRATEGIES

Fy3

u 2y

4y
Fx2

(a) nodal forces

4x

u 1y

Fx1

u 3x

Fy2

Fy1

u 3y

Fx3

465

u 1x

u x2

(b) nodal displacements

Figure 5.22: Discontinuous CST finite element.


The nodal forces and nodal displacements can be expressed as follows:

{F}

{F}

Fx1

Fy1
Fx2

Fy2

Fx3
Fy3

0
0

u1x
1
u y
u 2
x
u 2y
= 3
ux
u 3y
4
x
4
y

{a e }
{ e }

(5.213)

Hence, the external virtual work becomes:

{F}
{ *e }T
{F}

Wext = {a*e }T {F} + { *e }T {F} = {a*e }T

(5.214)

We consider the strain field and the virtual strain field are compound by two parts:
~
{} = {} + {}

~
{ * } = { * } + { * }

(5.215)

thus the internal virtual work becomes:

~
~ T
Wint = {}T {} d V = {}T { * } + { * } d V = { *} + { *} {} d V

(5.216)

Symmetric formulation

The discretization for the first approximation is:


{} = [B]{a} + [Ge ]{ e } = {[B ]
123 1
424
3
{ }

{~ }

[Ge ]}

{a e }
;
{ e }

{a }
{ }= [1B2
]{a3} + [1
G ]{ } = {[B] [G ]}

424
3
{ }
*

*
e

{ }
*

{~ }
*

*
e

*
e
*
e

(5.217)

Notice that we have used the same approximation function [B] , [Ge ] for virtual and real
strains. Then, the stress field can be written as follows:
{} = [C ]{} = [C ]{[B]

University of Castilla- La Mancha


Ciudad Real Spain

Draft

[Ge ]}

{a e }

{ e }

(5.218)

By: Eduardo W. V. Chaves (2015)

SOLVING PROBLEMS BY MEANS OF CONTINUUM MECHANICS

466

By replace the approximations (5.217) and (5.218) into the equation (5.216), the internal
virtual work becomes:
Wint =

~ T
{ } + { * } {} d V = {[B ]
V

{
V

{a * }
= e*
{ e }
V

= {a*e }T

*
[Ge ]}{a e*} [C ]{[B]
{ e }

[Ge ]}

{a e }
dV
{ e }

{a e }
dV
{ e }

{[B] [Ge ]}T [C ]{[B] [Ge ]}

{ *e }T

] [G[B]] [C ]{[B]
T

(5.219)

{a e }

{ e }

[Ge ]} d V

By apply Wext = Wint (see Eq. (5.214) and (5.219)) we can obtain
[B]T [C ][B ]

T
[Ge ] [C ][B]
e

{F}
=
{F} V

[B]T [C ][Ge ] dV {a e }

[Ge ]T [C ][Ge ] { e }

(5.220)

and considering the traction vector continuity, i.e. {F} = {0} , we obtain:
[B]T [C ][B] [B]T [C ][Ge ]
{F}
dV
=
T
T
{0} Ve [Ge ] [C ][B ] [Ge ] [C ][Ge ]
14444442444444
3

{a e }

{ e }

(5.221)

[K e ]

Anti-symmetric formulation

Now consider the real and virtual strain approximation by:


{a e }
{} = [B] {a e } + [Ge ]{ e } = [[B] [Ge ]]

1
424
3 1
424
3
{ e }
{~ }

{ }

{ * } = [B]{a*e } + [G*e ]{ *e } = [B]

(5.222)

{a* }
[G*e ] e*
{ e }

where we are considering different approximation functions for virtual and real strains i.e.
[Ge ] [G*e ] .
Using equation (5.216), and discretization (5.222) we can obtain:

Wint = {a*e }T

{ *e }T

} [B] [C ]{[B] [G ]} dV
[G ]
T

Ve

* T
e

{a e }

{ e }

(5.223)

Considering Wext = Wint and considering the traction vector continuity, we can obtain:
[B]T [C ][B] [B]T [C ][Ge ]
{F}
=
*T
dV

* T
{0} Be Ge [C ][B] Ge [C ][Ge ]
14444442444444
3

[ ]

[ ]

{a e }

{ e }

(5.224)

[K e ]

According to Jirsek(1998) there are three major classes of these models:

University of Castilla-La Mancha


Ciudad Real Spain

Draft

By: Eduardo W. V. Chaves (2015)

5 INTRODUCTION TO: CONSTITUTIVE EQUATIONS, IBVP, AND IBVP SOLUTION STRATEGIES

467

SOS (Statically Optimal Symmetric) formulation cannot properly reflect the


kinematics of a completely open crack but it gives a natural stress continuity
condition;

KOS (Kinematically Optimal Symmetric) formulation describes the kinematic


aspects satisfactorily but leads to an awkward relationship between the stress in the
continuous part of the element and the tractions across the discontinuity line.
These findings motivate the development of the nonsymmetric;

SKON (Statically and Kinematically Optimal Nonsymmetric) formulation, which


combines the strong points of each of the symmetric formulations.

Reference
JIRSEK, M. (1998). Finite elements with embedded cracks. LSC Internal Report 98/01,
April.

University of Castilla- La Mancha


Ciudad Real Spain

Draft

By: Eduardo W. V. Chaves (2015)

SOLVING PROBLEMS BY MEANS OF CONTINUUM MECHANICS

468

Problem 5.25
Consider a rod of length L and cross-sectional area A which undergoes deformation
because of its own weight, (see Figure 5.23 (a)). The rod is fixed at the top and is in static
equilibrium. Use the total potential energy to obtain an analogous equation as the one
obtained in Problem 5.24 in NOTE 4, i.e. obtain an equivalent equation
[ K (e ) ]{u( e ) } = {F (e ) } associated with this problem. Obtain also the displacement field.
Hypothesis: Homogeneous isotropic linear elastic material, small deformation regime.
u=0

y , x2
z, x3

U(1) = 0

L
2

EA

dV
r
b

U( 2 )

L
2

g

bi = 0
0

x, x1

a) 3D

U(3)
x

b) 1D
Figure 5.23

Solution:
To find out which displacement approach we must adopt, we will analyze the equilibrium
r r
equations ( + b = 0 ):
ij , j + b i = i1,1 + i 2, 2 + i 3,3 + b i = &u&i = 0 i
11,1 + 12, 2 + 13,3 + b1 = 0

21,1 + 22, 2 + 23,3 + b 2 = 0

31,1 + 32, 2 + 33,3 + b 3 = 0

11 12 13
+
+
= b1

x
x
x
1
2
3

21 22 23
+
+
= b 2

x
x
x
1
2
3

31 32 33
+
+
= b 3

x1
x2
x3

and for this problem we have:


0 0
ij = 0 0 0
0 0 0

0 0
e
ij = 0 0 0 11 = C1111
11 = E
0 0 0

Then, the equilibrium equations reduce to:


University of Castilla-La Mancha
Ciudad Real Spain

Draft

By: Eduardo W. V. Chaves (2015)

5 INTRODUCTION TO: CONSTITUTIVE EQUATIONS, IBVP, AND IBVP SOLUTION STRATEGIES

11
x = g
1
0 = 0
0 = 0

notation
Engineerin
g

469

= g
x

(5.225)

Note that the term g is constant in the rod, and according to the above equilibrium
equation, the stress must be a linear function in x . And if we consider that = E ,
also requires a linear function in x , and as a consequence the displacement u must be a
quadratic function in x since =

u
.
x

Then, the displacement field will be approached by the quadratic function


( u ( x) = a1 + a 2 x + a 3 x 2 ), hence we will need three points to be able to define this function.
We will adopted the points: x = 0 , x =

L
and x = L . With that we can obtain:
2

L
L
L
(2)
u ( x = ) = U = a1 + a2 + a3
2
2
4
( 3)
2
u ( x = L) = U = a1 + a2 L + a3 L

u ( x = 0) = U(1) = a1

U(1) 1 0
(2) L
U = 1
U(3) 2
1 L

0 a1
L2
a 2
4
L2 a3

Taking the reverse form of the above equation we can obtain:


L
0
0 U
a1
(2)
1
a 2 = 2 3L 4 L L U
a L 2
4 2 U (3)
3

(1)

a1 = U (1)

3 (1) 4 ( 2 ) 1 (3)

U + U U
a 2 =
4
L
L

2 (1) 4 ( 2 ) 2 (3)

a 3 = 2 U 2 U + 2 U
L
L
L

With that the displacement field in terms of U (1) , U( 2) , and U (3) is given by:
3 (1) 4 ( 2 ) 1 (3) 2 (1) 4 ( 2) 2 (3) 2
u = a1 + a2 x + a3 x 2 = U(1) +
U + U U x + 2 U 2 U + 2 U x
L
L
L
L

4
L

by simplify we can obtain:


3x 2 x 2 (1) 4 x 4 x 2 ( 2 ) x 2 x 2 (3)
+ 2 U +
2 U +
+ 2 U
u ( x) = 1
L
L
L
L

L
L
= N1U(1) + N 2U( 2 ) + N 3U(3)
U(1)

= [N1 ( x) N 2 ( x) N 3 ( x)]U( 2 )
U(3)

(5.226)

= [ N ( x)]{u(e ) }

where N 1 ( x) , N 2 ( x) and N 3 ( x) are the shape functions.


The goal now is to express the total potential energy in terms of U (1) , U ( 2) and U (3) .

University of Castilla- La Mancha


Ciudad Real Spain

Draft

By: Eduardo W. V. Chaves (2015)

SOLVING PROBLEMS BY MEANS OF CONTINUUM MECHANICS

470

The term U ext (see equation (5.181)) becomes:


U ext =

r r
r r
r r
t * udS + ( b) udV = ( b) udV = gu ( x) Adx = gA u ( x)dx

L
3x 2 x 2
= gA 1 + 2
L
L

(1) 4 x 4 x 2
U +

L
L2

( 2) x 2 x 2
U +

L + L2

( 3)
U dx

After the integration takes place we can obtain:


2 L ( 2 ) L ( 3)
L
U ext (U (1) , U ( 2 ) , U (3) ) = gA U (1) +
U + U
3
6

(5.227)

The term U int for this problem is the same as the one given by the equation in (5.190), i.e.
U

int

1
1
1
u ( x)
u ( x)
=
E 2 dV =
E
AE
dV =
dx
2V
2 V x
20
x

(5.228)

where
u 3 x 2 x 2 (1) 4 x 4 x 2 ( 2) x 2 x 2
=
+ 2
1 + 2 U + 2 U +
L
x x
L
L
L
L
L
3 4 x (1) 4 8 x ( 2) 1 4 x (3)
=
+ 2 U + 2 U + + 2 U
L L
L L
L L

( 3)
U

thus
2

int

EA 3 4 x (1) 4 8 x ( 2 ) 1 4 x (3)
=
+ 2 U + 2 U +
+
U dx

2 0 L
L
L L2
L L

By solving the above integral we can obtain:


U int (U (1) , U ( 2 ) , U (3) ) =

2
2
EA (1) 2
+ 16U ( 2 ) + 7U (3) + 2U (1) U (3) 16U (1) U ( 2 ) 16U ( 2 ) U (3)
7U

6 L

(5.229)
The total potential energy is given by:
(U (1) , U ( 2 ) , U (3) ) = U int U ext
=

2
2
EA (1) 2
7U
+ 16U ( 2 ) + 7U (3) + 2U (1) U (3) 16U (1) U ( 2 ) 16U ( 2 ) U (3)

6 L
2 L ( 2 ) L ( 3)
L
U + U
gA U (1) +
3
6
6

As we are looking for the stationary state, the following must be fulfilled:
(U (1) , U ( 2 ) , U (3) )
=0

U (1)

(U (1) , U ( 2 ) , U (3) )
=0

U ( 2 )

(U (1) , U ( 2 ) , U (3) )
=0

U (3)

gAL
EA
7U (1) 8U ( 2 ) + U (3)
=0
3L
6
2 gAL
EA
8U (1) + 16U ( 2 ) 8U (3)
=0
3L
3

gAL
EA (1)
=0
U 8U ( 2) + 7U (3)
3L
6

Rearranging the above set of equations in matrix form we can obtain:

University of Castilla-La Mancha


Ciudad Real Spain

Draft

By: Eduardo W. V. Chaves (2015)

5 INTRODUCTION TO: CONSTITUTIVE EQUATIONS, IBVP, AND IBVP SOLUTION STRATEGIES


(1)
1
7 8 1 U
( 2 ) gAL
EA

8 16 8 U =
4
3L
6
( 3)

1 8 7 U
1

[ K (e ) ]{u(e ) } = {F (e ) }

471

(5.230)

Note that [ K (e ) ] has no inverse, since det[ K (e ) ] = 0 . To solve the problem we have to
introduce the boundary conditions. According to the problem statement, the displacement
at x = 0 is equal to zero, i.e. U (1) = 0 . We apply this boundary condition by eliminate the
first line and column of the system (5.230), in other words we eliminate the terms
associated with the degree-of-freedom U (1) , i.e.:
(1)
1
7 8 1 U
( 2) gAL
EA

8 16 8 U =
4
3L
6
( 3)

1 8 7 U
1

or

0 U(1)
0
1 0
( 2 ) gAL
EA

0 16 8 U =
4
3L
6
( 3)

0 8 7 U
1

EA 16 8 U ( 2 ) gAL 4
=

3L 8 7 U (3)
6 1

EA 16 8

3L 8 7

EA 16 8 U ( 2 ) gAL EA 16 8 4

3L 8 7 U (3) = 6 3L 8 7 1

U ( 2 ) gAL EA 16 8 4

( 3) =
3L 8 7 1
6
U

( 2)
U gAL L 7 8 4

( 3) =


6 16 EA 8 1 1
U
U ( 2 ) gL2 3
( 3) =

U 8E 4

Now, if we substitute the values of U (1) , U( 2) , and U (3) into the displacement field (see
equation (5.226)) we can obtain:
3x 2 x 2
u = 1 + 2
L
L

(1) 4 x 4 x 2 ( 2 ) x 2 x 2 (3)
U +
U +

L
L + L2 U
L2

2
2
2
3 x 2 x 4 x 4 x 3 gL x 2 x 2 4 gL2
+
= 1 + 2 0 + 2
+ 2
L
L L
L 8E L
L 8 E

By simplify the above equation we can obtain:


u (Q ) =

g
2E

(2 Lx x 2 )

(5.231)

which is also the exact solution for the proposed problem. Then, the strain and stress field
can be obtained as follows:
(Q ) =

u ( x) g
g
(2 Lx x 2 ) =
( L x)
=
x 2 E
x
E

(5.232)

and
( Q ) = ( Q ) E = g ( L x )

(5.233)

If we replace the nodal displacement into the total potential energy we can obtain:

University of Castilla- La Mancha


Ciudad Real Spain

Draft

By: Eduardo W. V. Chaves (2015)

SOLVING PROBLEMS BY MEANS OF CONTINUUM MECHANICS

472

(Q ) =

1 ( g ) 2 AL3
6
E

NOTE 1: The shape function [N ( x )]


Note that the shape functions are:
3x 2 x 2
N 1 ( x) = 1
+ 2
L
L
4x 4x 2
N 2 ( x) =
2
L
L

N 1 ( x = 0) = 1

L
N1 ( x = 2 ) = 0
N ( x = L) = 0
1

N 2 ( x = 0) = 0

L
N 2 ( x = 2 ) = 1
N ( x = L) = 0
2

N 3 ( x = 0) = 0

L
N 3 ( x = 2 ) = 0
N ( x = L) = 1
3

x 2x 2
+ 2
N 3 ( x) =
L
L

Note also that N 1 ( x) + N 2 ( x) + N 3 ( x) = 1 holds.


If we draw these functions inside the domain we can obtain:

N2
N1

3x 2 x 2
N
1
=

+ 2
1
L
L

4x 4x
2
N 2 =
L
L

x 2x2
+ 2
N3 =
L
L

N3

1
x=0

x=

x=L

L
2

N1 ( x ) + N 2 ( x) + N 3 ( x ) = 1

Figure 5.24: Shape functions (1D) quadratic function.


NOTE 2: Analytical solution (exact) direct integration
We start from the equilibrium equation

= g , (see equation (5.225)), and by integrate


x

we can obtain:

= g integratin
g = gx
x

= gx + C1

The constant of integration can be obtained at x = 0 . In this situation the total force at
x = 0 is given by F = gV = gAL , and the stress can be obtained by
F gAL
=
= gL .
Then,
the
constant
A
A
( x = 0) = C1 = gL . Hence, the stress field becomes:

( x = 0) =

of

integration

becomes

= gx + gL = g ( L x)

University of Castilla-La Mancha


Ciudad Real Spain

Draft

By: Eduardo W. V. Chaves (2015)

5 INTRODUCTION TO: CONSTITUTIVE EQUATIONS, IBVP, AND IBVP SOLUTION STRATEGIES

473

Using the constitutive equation, the strain field can be obtained as follows:
= E

Taking into account the relationship =

g ( L x)
=
E
E

u
(kinematic equation), and by integrate we can
x

obtain:
=

u
x

integrating

g ( L x)
u = x =
x
E

u ( x) =

x2
+ C2
Lx

E
2

At x = 0 there is no displacement, so, u ( x = 0) = 0 C 2 = 0 , then, the displacement field,


(see Figure 5.25), becomes:
u ( x) =

x 2 g
=
(2 Lx x 2 )
Lx

E
2 2 E

=L
x

u ( x = L) =

( x = 0) =

u=
L + L

g
2E

gL2

(5.234)

2E

( x = 0) = gL

gL
E

(2 Lx x 2 )
=

g
E

( L x)

= g ( L x )

a) Displacement (x-direction)

c) Stress

b) Strain

Figure 5.25
NOTE 3: Note that for simple problems the analytical solution is very easy to be obtained,
and this solution serves as an indicator of whether the numerical solution used is the most
appropriate. Note that the analytical solution (equation (5.234)), is the same as the
numerical solution (5.231) in which we have used the quadratic function to approach the
displacement field, and if we consider a cubic function to approach the displacement field
the solution must be the same. Let us check this fact:
Displacement field (cubic function): u ( x) = a1 + a2 x + a3 x 2 + a4 x 3 = a2 x + a3 x 2 + a4 x 3 .
Note the, at x = 0 there is no displacement, so, u ( x = 0) = a1 = 0 .
Then, u ( x) = a2 x + a3 x 2 + a4 x 3

University of Castilla- La Mancha


Ciudad Real Spain

u
= a2 + 2a3 x + 3a4 x 2
x

Draft

By: Eduardo W. V. Chaves (2015)

SOLVING PROBLEMS BY MEANS OF CONTINUUM MECHANICS

474

Internal Potential in function of (a 2 , a 3 , a 4 ) :


L

int

AE u ( x)
AE
(a2 + 2a3 x + 3a4 x 2 ) 2 dx
=

dx =
2 0 x
2 0

AEL
(27 L4 a42 + 45 L3 a4 a3 + 30 L2 a2 a4 + 20 L2 a32 + 30 La2 a3 + 15a22 )
30

External Potential in function of (a 2 , a 3 , a 4 ) :


L

1
1
1

U ext = gA u ( x)dx = gA (a 2 x + a 3 x 2 + a 4 x 3 )dx = gA L2 a 2 + L3 a 3 + L4 a 4


3
4
2

x
0

The total potential energy in function of (a 2 , a 3 , a 4 ) :


(a2 , a3 , a4 ) = U int U ext
=

AEL
(27 L4 a42 + 45 L3 a4 a3 + 30 L2 a2 a4 + 20 L2 a32 + 30 La2 a3 + 15a22 )
30
1
1
1

gA L2 a2 + L3 a3 + L4 a4
2
3
4

As we are looking for the stationary state, the following must be fulfilled:
(a2 , a3 , a4 )
=0

a2

(a , a , a )
2
3
4
=0

(a , a , a )
2
3
4

=0
a4

gAL2
EAL
=0
(30 L2 a4 + 30 La3 + 30a2 )
30
2

EAL
gAL3
=0
(45 L3 a4 + 40 L2 a3 + 30 La2 )
30
3

EAL
gAL4
=0
(54 L4 a4 + 45 L3 a3 + 30 L2 a2 )
30
4

Simplifying and rearranging the above set of equations in matrix form we can obtain:
30
E
30 L
30
2
30 L

30 L
40 L2
45L3

30 L2 a2
6L
a2
2 gL
g 2 Solve 1

3
45L a3 =
4 L
a3 =
g

2
2E
3L3
a4
0
54 L4 a4

Then, the displacement field ( u ( x) = a2 x + a3 x 2 + a4 x 3 ) becomes:


u ( x) = a2 x + a3 x 2 + a4 x 3 =

1
g
(2 gLx gx 2 ) =
(2 Lx x 2 )
2E
2E

which is the same solution as the one provided by the quadratic function used to approach
the displacement field.
Next, let us adopt a linear function to approach the displacement field u ( x) = a2 x , then
Internal Potential:
L

int

AE
AEL 2
AE u ( x)
(a2 ) 2 dx =
a2
=

dx =
2 0 x
2 0
2

External Potential:
U

ext

= gA u ( x)dx = gA (a 2 x)dx =
x

University of Castilla-La Mancha


Ciudad Real Spain

1
gAL2 a 2
2

Draft

By: Eduardo W. V. Chaves (2015)

5 INTRODUCTION TO: CONSTITUTIVE EQUATIONS, IBVP, AND IBVP SOLUTION STRATEGIES

475

The total potential energy:


(a 2 ) = U int U ext =

AEL 2 1
a 2 gAL2 a 2
2
2

The equilibrium point:


(a 2 )
=0
a 2

EALa 2

gAL2
2

=0

a2 =

( L) =

u ( L ) gL
=
2E
x

gL
2E

Then
u ( L ) = a2 x =

gL
2E

( L ) = E =

1
gL
2

Note that, for this case a linear function is not a good approach.
The total ( L ) =

AEL 2 1
1 ( g ) 2 AL3
a2 gAL2 a2 =
2
2
8
E

NOTE 4:
Next, we will establish the stiffness matrix [ K (e ) ] of the rod element by considering the
linear approximation for the displacement field. Adopting the linear function
u ( x) = a1 + a 2 x we can obtain:

= a1 + a2 L

u ( x = 0) = U(1) = a1
u ( x = L ) = U( 2 )

U(1) 1 0 a1 Reverse a1 1 L 0 U(1)


( 2) =
a = L 1 1 ( 2)

U
2
U 1 L a2

Then, the displacement field becomes:


x
x
1
1

u ( x) = a1 + a 2 x = U (1) + U (1) + U ( 2 ) x = 1 U (1) + U ( 2 )


L
L
L

x x U (1)
= 1 ( 2)
L L U

{ }

= [N ] u ( e )

whose equation has already been obtained in Problem 5.24 (NOTE 4). Then, the stiffness
matrix will be the same, but the nodal forces will be not the same.
Internal Potential:
L

U int =

2
AE 1 (1) 1 ( 2 )
AE (1) 2
AE u ( x)
U 2U (1)U ( 2 ) + U ( 2)
U + U dx =

dx =

2 0 x
2 0 L
L
2L

External Potential:
U

ext

x
1
= gA u ( x)dx = gA 1 U (1) + U ( 2 ) dx = gAL(U (1) + U ( 2 ) )
L
L
2

x
0

The total potential energy:


(U(1) , U( 2 ) ) = U int U ext =

2
AE (1) 2
1
U 2U(1)U( 2 ) + U( 2 ) gAL(U(1) + U ( 2 ) )
2
2L

The equilibrium point:

University of Castilla- La Mancha


Ciudad Real Spain

Draft

By: Eduardo W. V. Chaves (2015)

SOLVING PROBLEMS BY MEANS OF CONTINUUM MECHANICS

476

(U (1) , U ( 2 ) )
(1)

U
(U (1) , U ( 2 ) )
U

(2)

=0

1
1
AE
AE (1)
( 2U (1) 2U ( 2 ) ) gAL = 0
(U U ( 2 ) ) = gAL
2L
2
2
L

=0

1
1
AE
AE ( 2 )
( 2U ( 2 ) 2U (1) ) gAL = 0
(U U (1) ) = gAL
2L
2
2
L

Simplifying and rearranging the above set of equations in matrix form we can obtain:
1
EA 1 1 U(1) 1
1 1 ( 2 ) = gAL 1
L

U 2

[ K (e ) ]{u(e ) } = {F ( e ) }

As we have seen in the above NOTE, there is an error when we use the linear
approximation for the displacement field. Next, we will divide the domain in sub-domain.
To establish the displacement field we will adopt a generic element, where the initial point
is x (i ) and the final point is x ( f ) , (see Figure 5.26 (a)).

U(1) = 0
x (i )

(EA)

U(i )
e

(1)

x( f )

L(1) =

L
2

L( 2 ) =

L
2

U( 2 )

U( f )

(EA) ( 2)

2
U(3)

a) Generic element

b) Discretization in 2 elements
Figure 5.26

Taking into account the linear approximation u ( x) = a1 + a 2 x we can obtain:


U(i ) 1 x (i ) a1
u ( x = x (i ) ) = U(i ) = a1 + a2 x (i )

( f ) =

( f )
u ( x = x ( f ) ) = U( f ) = a1 + a2 x ( f )
U 1 x a2
x ( f ) x (i ) U(i )
a
1
Inverse
1 = ( f )

(i )
1 U( f )
a 2 ( x x ) 1

Then, the displacement field becomes:


u ( x) = a1 + a 2 x

u ( x) =

(x

(f)

1
1
( x ( f ) x)U (i ) + ( f )
( x x (i ) )U ( f )
(i )
(i )
x )
(x x )

and its derivative:


u ( x)
1
1
= (f)
U (i ) + ( f )
U( f )
(i )
(i )
x
(x x )
(x x )

University of Castilla-La Mancha


Ciudad Real Spain

Draft

By: Eduardo W. V. Chaves (2015)

5 INTRODUCTION TO: CONSTITUTIVE EQUATIONS, IBVP, AND IBVP SOLUTION STRATEGIES

477

Let us divide the domain into 2 sub-domain (2 finite elements), (see Figure 5.26 (b)), where:
Element e = 1 :

= U ( 2)

x (i ) = 0
L
x( f ) =
2
(i )
U = U (1)
U( f )

(1) 2 x (1) 2 x ( 2 )
u = 1 U + 1 U
L
L

(1)
u = 2 U (1) + 2 U ( 2 )
x
L
L

(2)
2 x ( 2 ) 2 x ( 3)
u = 2 U + 1U
L

(2)
u = 2 U ( 2 ) + 2 U (3)
x
L
L

Element e = 2 :

(f)

U (i ) = U ( 2)

U ( f ) = U (3)
L
2
=L

x (i ) =

Internal Potential:
U

int

L
2

u (1)
1
1
u ( x)
AE
=
( AE ) (1)
dx =
20
20
x
x

L
( 2)

1
( 2 ) u
dx +

AE
(
)

x
2L

dx

( AE )
L

(1)

U (1) 2 2U (1) U ( 2 ) + U ( 2 ) 2 + ( AE )

( 2)

U ( 2) 2 2U ( 2 ) U (3) + U (3) 2

External Potential:
U

ext

L
2

= g Au ( x)dx = g A u dx + g A( 2)u ( 2 ) dx
x

(1) (1)

L
2

1
1
1
gA(1) LU(1) + gL( A(1) + A( 2) )U( 2) + gA( 2) LU(3)
4
4
4

The total potential energy


(U (1) , U ( 2 ) , U (3) ) = U int U ext
=

2
2
( AE ) (1) (1) 2
( AE ) ( 2 ) ( 2) 2
2U (1) U ( 2 ) + U ( 2 ) +
2U ( 2 ) U (3) + U (3)
U
U

L
L
1
1
1

gA (1) LU (1) + gL( A (1) + A ( 2 ) )U ( 2 ) + gA ( 2) LU (3)


4
4
4

The equilibrium point

University of Castilla- La Mancha


Ciudad Real Spain

Draft

By: Eduardo W. V. Chaves (2015)

SOLVING PROBLEMS BY MEANS OF CONTINUUM MECHANICS

478

(U ( a ) )
U (1)
(U ( a ) )
U ( 2 )
(U ( a ) )
U (3)

=0

2( AE ) (1) (1)
1
(U U ( 2) ) gLA (1) = 0
L
4

=0

2( AE ) ( 2 ) ( 2 )
2( AE ) ( 2) ( 2 )
1
(U U (1) ) +
(U U (3) ) gL( A (1) + A ( 2 ) ) = 0
4
L
L

=0

2( AE ) ( 2 ) (3)
1
(U U ( 2 ) ) gLA ( 2 ) = 0
L
4

Rearranging the above set of equations in matrix form we can obtain:


(1)

[ A(1) ]
U 1

(1)
(2)
( 2)
(
2
)
(
1
)
(
2
)
[( AE )
+ ( AE ) ] [( AE ) ] U = gL [ A + A ]
4

[ A( 2 ) ]
[( AE )( 2 ) ]
[( AE )( 2 ) ] U(3)

[( AE )(1) ]
2
[( AE )(1) ]
L

[( AE )(1) ]

[ K ]{u} = {F }

Note that the above matrix could have been obtained directly if we consider the stiffness
matrix of the element and the nodal force vector of the element e :

[k ] = ( EAL )
(e)

(e)

(e)

(1)

(1)

Element e = 2 , ( L( 2) =

[k ] = 2(EAL )
( 2)

{f } = 12 g (LA)
(e)

(e)

L
):
2

Element e = 1 , ( L(1) =

[k ] = 2( EAL )

1 1
1 1

1 1
1 1

{f } = 14 gLA

(1) 1

{f }= 14 gLA

( 2 ) 1

(1)

{u } = U

{u }= U

(1)

( 2)
U
(1)

L
):
2

(2)

1 1
1 1

( 2)

(2)

( 2)

( 3)

Then the global stiffness matrix [K ] can be obtained by adding the contribution of each
stiffness matrix of the element into [K ] , and the same to the global nodal vector. This
process is called the assemble process.
Considering the ( EA) (1) = ( EA) ( 2) = EA the set of discrete equations becomes:
(1)
1
1 1 0 U

2 AE
1

(2)
1 2 1 U = gLA2

4
L
1
0 1 1 U (3)

Applying the boundary condition and solving the system the nodal displacement can be
obtained:
(1)
0
1 0 0 U

2 AE
1

( 2)
0 2 1 U = gLA2

4
L
1
0 1 1 U(3)

Solve

U(1)
0
( 2 ) gL2
U =
3
U(3) 8 E 4

which matches the exact solution.

University of Castilla-La Mancha


Ciudad Real Spain

Draft

By: Eduardo W. V. Chaves (2015)

5 INTRODUCTION TO: CONSTITUTIVE EQUATIONS, IBVP, AND IBVP SOLUTION STRATEGIES

479

The procedure we have developed is the basis of the Finite Element Method which
basically consists of:
Adopt an approach to the unknown field;
Split (discretize) the domain into sub-domain (finite element);
Set the stiffness matrix of each sub-domain and the nodal force vector;
Assemble the global stiffness matrix of the structure;
Apply the boundary condition;
Solve the system.
For more detail about Finite Element Method the reader is referred to Zienkiewicz &
Taylor (1994), Bathe(1996).

University of Castilla- La Mancha


Ciudad Real Spain

Draft

By: Eduardo W. V. Chaves (2015)

SOLVING PROBLEMS BY MEANS OF CONTINUUM MECHANICS

480

Problem 5.26
Show that:

r r
r&
r
r
r* r
&r&( Xr , t ) u& dV = P : r u& dV
( X , t ) u dS 0 + 0 b( X , t ) u
0
0
X

V0

(5.235)

V0

where u is the virtual displacement field, and P is the first Piola-Kirchhoff stress tensor.
Reference
configuration

Current
configuration

S0

V0

B0

r r
r
t * ( X , t ) = t *0

dV0
r r
u( X , t )

r r
t* ( x, t )

dV

r r
u( x , t )

r r
r
0b( X , t ) = 0b0

r r

b( x , t )
Solution:
r

r r

Although the variables t * ( X , t ) and b( X , t ) are not intrinsic variables of the reference
r r
r
configuration like the variables 0 , S 0 , V0 , for simplicity, we denote t * ( X , t ) = t *0 and
r r
r
b( X , t ) = b 0 .

Remember also (see Chapter 2 of the textbook) that:

r
r
r
r
& i ( X , t)
x
t
x
t
(
,
)
(
,
)
u

D
i
i
Fij F&ij =
=
=
= u& i , J ( X , t )
Dt
t X j
X j 142
t 4
X j
3
x&i

r& r
r& r

u
( X , t)
&
or F = l F = Xr u( X , t ) =
r
X
r r
and l = F& F 1 = Xr u& ( X , t ) F 1
r r
r r
F& 1 = F 1 l = F 1 Xr u& ( X , t ) F 1 = F 1 xr u& ( x , t )

Taking into account the above relations, it is also valid that:


r& r
r& r
r& r
F& = Xr u ( X , t ) y F& 1 = F 1 Xr u ( X , t ) F 1 = F 1 xr u ( x , t )

With that we can obtain:


&

P : F dV = P

V0

(PiJ u& i ) , J = PiJ , J u& i + PiJ u& i , J

iJ

r
F&iJ dV0 = PiJ u& i , J ( X , t ) dV0

V0

V0

PiJ u& i , J = (PiJ u& i ) , J PiJ , J u& i

thus:

University of Castilla-La Mancha


Ciudad Real Spain

Draft

By: Eduardo W. V. Chaves (2015)

5 INTRODUCTION TO: CONSTITUTIVE EQUATIONS, IBVP, AND IBVP SOLUTION STRATEGIES

481

r
r
r
P : F& dV0 = PiJ u& i , J ( X , t ) dV0 = (PiJ u& i ( X , t )) , J PiJ , J u& i ( X , t ) dV0

V0

V0

&

V0

P : F dV = (P

iJ

V0

V0

&

P : F dV = P

iJ

V0

r
r
u& i ( X , t )) , J dV0 PiJ , J u& i ( X , t ) dV0

V0

r
r
u& i ( X , t )n J dS 0 PiJ , J u& i ( X , t ) dV0

S0

V0

where we have applied the divergence theorem. The above in tensorial notation becomes:
r

P : F dV = (P n ) u& ( X , t ) dS (
&

V0

S0

r
X

P) u& i ( X , t ) dV0

V0

Remember that the equations of motion in the reference configuration are given by:
r
&r&( Xr , t )
Xr P + 0 b 0 = 0 u

r
&r&( Xr , t )
Xr P = 0 b 0 u

r& r

and taking into account that F& = Xr u ( X , t ) and t *0 = P n we can obtain:


r

P : F dV = (P n ) u& ( X , t ) dS (
&

V0

S0

r
X

V0

P) u& i ( X , t ) dV0

r
r& r
r &r& r
&r&( Xr , t ) u& ( Xr , t ) dV
P : Xr u ( X , t ) dV0 = t *0 u
( X , t ) dS 0 + 0 b 0 u
i
0

V0

S0

V0

Reminder: Recall from Chapter 5 of the textbook that the stress power can be expressed
in different ways, namely:
1
1
S : C& dV0 =
P : F& dV
w int (t ) = P : F& dV0 = S : E& dV0 = P : F& dV0 =
2
J
V
V
V
V
V
=

P : F& dV = : D dV = {
J : D dV0 = : D dV0
V

V0

V0

NOTE 1: Remember that neither P nor F& are in any configuration, but the scalar P : F&
is in the reference configuration.
NOTE 2: Taking into account the above. The total external virtual work can also be
expressed as follows:
sym
r& r
r& r
r& r
: D dV = : xr u ( x , t ) dV = : xr u ( x , t ) dV = P : F& dV0 = P : Xr u ( X , t ) dV0

V
V
V
V
V

r r sym
= xr u& ( x , t)
(see Problem 2.37). Note that, due to
sym
r& r
r& r
the symmetry of the relationship : xr u ( x , t ) = : xr u ( x , t ) holds.

where we have used that D = l

sym

NOTE 3: From a Variational Principle point of view, (see Holzapfel (2000)), the equation
in (5.235) is also valid for a variation of the virtual field:

r r
r
r
r
r r
&r&( Xr , t ) u dV = P : r u dV
t * ( X , t ) u dS 0 + 0 b( X , t ) u
0
0
X

V0

University of Castilla- La Mancha


Ciudad Real Spain

V0

Draft

(5.236)

By: Eduardo W. V. Chaves (2015)

SOLVING PROBLEMS BY MEANS OF CONTINUUM MECHANICS

482

Problem 5.27
a) Show that: given a symmetric second-order tensor A = A sym we can split up into
A = A P + A S where A P = P P : A , A S = P S : A , with P P = (b b ) (b b ) and
P S = I sym (b b ) (b b ) , where b is a unit vector according to certain direction, and

I sym is the symmetric part of the fourth-order unit tensor. b) Show that the constitutive
equation for stress = C e : can be written as follows:
P C PP
S = SP
C

PP
ijP Cijkl
S = SP
ij Cijkl

C PS
:
C SS

PS
kl
Cijkl

SS
Cijkl kl

(5.237)

where
C PP = P P : C e : P P

PP
P
P
Cijkl
Cepqst Pstkl
= Pijpq

C PS = P P : C e : P S

PS
P
S
Cijkl
Cepqst Pstkl
= Pijpq

C SP = P S : C e : P P

SP
S
P
Cijkl
Cepqst Pstkl
= Pijpq

C SS = P S : C e : P S

(5.238)

SS
S
S
Cijkl
Cepqst Pstkl
= Pijpq

Solution:
a) We start using the Cartesian system where it fulfills:
A = A ij (e i e j ) = A i1 (e i e 1 ) + A i 2 (e i e 2 ) + A i 3 (e i e 3 )

= A 11 (e 1 e 1 ) + A 21 (e 2 e 1 ) + A 31 (e 3 e 1 ) + A 12 (e 1 e 2 ) + A 22 (e 2 e 2 ) + A 32 (e 3 e 2 )
+ A 13 (e 1 e 3 ) + A 23 (e 2 e 3 ) + A 33 (e 3 e 3 )

Components:
A 11
A ij = A 21
A 31

A12
A 22
A 32

A13 0
A 23 = A 21
A 33 A 31

A 12
A 22

A13 A11
A 23 + 0
A 33 0

A 32

0 0
0 0 = A ij + A ij
0 0

Note also that the normal component A 11 = A (Ne1 ) (according to e 1 -direction) can also be
obtained by A11 = A : (e 1 e 1 ) = (e 1 e 1 ) : A , so the tensor A = A11 (e 1 e 1 ) can be
written as follows:
A = (e 1 e 1 ) A 11 (e 1 e 1 ) A 11 = (e 1 e 1 ) (e 1 e 1 ) : A

thus
A = A A = A (e 1 e 1 ) (e 1 e 1 ) : A = I sym : A (e 1 e 1 ) (e 1 e 1 ) : A

= I sym (e 1 e 1 ) (e 1 e 1 ) : A

Although we showed for the unit vector e 1 the above is valid for any unit vector, i.e.:
A P = (b b ) (b b ) : A = P P : A
A S = I sym (b b ) (b b ) : A = P S : A

Note that A (Nb ) = (b b ) : A = b A b is the normal component according to b -direction,

i.e. parallel to b . It is interesting to review Problem 1.118.

University of Castilla-La Mancha


Ciudad Real Spain

Draft

By: Eduardo W. V. Chaves (2015)

5 INTRODUCTION TO: CONSTITUTIVE EQUATIONS, IBVP, AND IBVP SOLUTION STRATEGIES

483

b) We apply the above definition to obtain:


= P + S = PP : + PS :
= P + S = PP : + PS :

with that, and considering = C e : , we can obtain:


P = P P : = P P : C e : = P P : C e : ( P + S ) = P P : C e : P + P P : C e : S
= P P : Ce : P P : + P P : Ce : P S :
S = P S : = P S : C e : = P S : C e : ( P + S ) = P S : C e : P + P S : C e : S
= P S : Ce : P P : + P S : Ce : P S :

thus
P P P : C e
S= S
e
P : C

P P : C e P
:
P S : C e S

or
P P P : C e : P P
S= S
e
P
P : C : P

University of Castilla- La Mancha


Ciudad Real Spain

Draft

P P : C e : P S
:
P S : C e : P S

By: Eduardo W. V. Chaves (2015)

SOLVING PROBLEMS BY MEANS OF CONTINUUM MECHANICS

484

Appendix

(1 + )(1 2 )
(1 2 )
E
E
=
=
2(1 + ) (1 + )(1 2 )
2
E
E
E
+2
=
(1 )
+ 2 =
(1 + )(1 2 )
2(1 + ) (1 + )(1 2 )

+=

E
E
E
+
=
(1 + )(1 2 ) 2(1 + ) 2(1 + )(1 2 )

(2 + 3) =

E
E
E
E2
+3
2
=
2(1 + ) 2(1 + )
(1 + )(1 2 ) 2(1 + )(1 2 )

+
E
2(1 + )(1 2 ) 1
=
=
E
(2 + 3) 2(1 + )(1 2 )
E2
(1 + )(1 2 )
E

=
=
E
2 (2 + 3) (1 + )(1 2 )
E2
1

2(1 + ) 1
= 2(1 + )
E
E

1
(1 2 )
,
=
(2 + 3)
E

(2 + 3)

(2 + 3 )

E
(1 2 )

,
=
E
(1 + )(1 2 ) (1 + )

E
(1 2 )
(1 2 )
,
=
E
2(1 + ) 2(1 + )

2( + )
(1 2 )
1

,
=2
+2
=
(2 + 3)
(1 + )
2(1 + ) (1 + )
( 2 + ) = 2

E
E
E (1 )
,
+
=
2(1 + ) (1 + )(1 2 ) (1 + )(1 2 )

(1 + )(1 2 )
E

=
=
(2 + ) (1 + )(1 2 )
E (1 )
(1 )
,

E
2(1 + )(1 2 )
=
= 2
( + ) (1 + )(1 2 )
E
2 + 1
,
=
2 + 3 1 +

University of Castilla-La Mancha


Ciudad Real Spain

Draft

By: Eduardo W. V. Chaves (2015)

6 Linear Elasticity
6.1 Three-Dimensional Elasticity (3D)
Problem 6.1
The cylinder shown in Figure 6.1 is made up of an isotropic linear elastic material, and is
subjected to a strain state (in cylindrical coordinates) as follows:
err = e = a sin
a cos
2
e zz = ez = erz = 0
e r =

(6.1)

where eij are the Almansi strain tensor components.


r

Calculate the traction vector t on the boundary, in cylindrical coordinates.


Assumptions: Small deformation regime and consider the Lam constants , ;
x3

e z

e r

x2

x1

Figure 6.1
Solution:
Small deformation regime: e E

SOLVING PROBLEMS BY MEANS OF CONTINUUM MECHANICS

460

rr
( r , , z ) = r
rz

r

z

a sin
rz
a cos
z =
2
zz 0

a cos
2
a sin
0

(6.2)

= Tr ( )1 + 2

(6.3)

Tr ( ) = 2a sin

(6.4)

thus,
a cos

a sin
0

2
1 0 0

a cos
= 2a sin 0 1 0 + 2
a sin 0

2
0 0 1
0
0
0

0
a cos
2a sin + 2 a sin

0
a cos
2a sin + 2 a sin
(r,r, =

0
0
2a sin
r
The traction vector t :
r
t (n) = n

(6.5)

(6.6)

(6.7)

n = (1,0,0)
t 1(n ) 2a sin + 2 a sin
(n )

a cos
t 2 =

t (n )

0
3

(6.8)

Problem 6.2
The parallelepiped of Figure 6.2 is deformed as indicate by the dashed lines. The
displacement components are given as follows:
u = C1 xyz

v = C 2 xyz

w = C3 xyz

(6.9)

a) Obtain the strain state at the point E , which the current reference of the point is
represented by E whose coordinates are E (1.503; 1.001; 1.997) ;
b) Obtain the normal strain at the point E in the direction of the line EA ;
c) Calculate the angular distortion at the point E that undergoes the angle formed by the
lines EA and EF .
d) Find the volume variation and the average volumetric deformation.

University of Castilla-La Mancha


Ciudad Real Spain

Draft

By: Eduardo W. V. Chaves (2015)

6 LINEAR ELASTICITY

461

z
F = F

G = G

D = D

2m
E
C = C

O = O

1 .5 m

A = A

B = B

1m

Figure 6.2
Solution:
a) The strain state in function of the displacements is given by:
ij =

1 u i u j
+
2 x j x i

(6.10)

which in engineering notation is:

x
1
ij = xy
2
1
2 xz

1
xy
2
y
1
yz
2

u
1
xz
x

1 v u
1
yz = +
2 2 x y

z 1 w u
2 x + z

1 v u
+
2 x y
v
y
1 w v

+
2 y z

1 w u
+

2 x z
1 w v

+
2 y z

(6.11)

To determine the strain state at any point we need a priori to determine the displacement
field.
Calculation of the constants:
By substituting the values given for the point E (1.5; 1.0; 2.0) , we obtain:
u ( E ) = X 1( E ) x1( E ) = 1.503 1.5 = C1 (1.5)(1.0)(2.0) C1 = 0.001
0.001
3
= 1.997 2.0 = C3 (1.5)(1.0)(2.0) C3 = 0.001

v ( E ) = X 2( E ) x2( E ) = 1.001 1.0 = C2 (1.5)(1.0)(2.0) C2 =


w( E ) = X 3( E ) x3( E )

University of Castilla-La Mancha


Ciudad Real - Spain

Draft

(6.12)

By: Eduardo W. V. Chaves (2015)

SOLVING PROBLEMS BY MEANS OF CONTINUUM MECHANICS

462

Then, the displacement field becomes:


Engineering notation

Scientific notation

u = 0.001 xyz
0.001
v=
xyz
3
w = 0.001 xyz

u1 = 0.001 X 1 X 2 X 3
0.001
X1X 2 X 3
3
u 3 = 0.001 X 1 X 2 X 3

u2 =

(6.13)

u
= 0.001 yz = 0.002 = 11
x
v 0.001
y =
=
xz = 0.001 = 22
3
y
w
z =
= 0.001xy = 0.0015 = 33
z
v u 0.001
0.011
=
xy =
+
yz + 0.001xz =
= 2 12
3
3
x y
x =

w u
xz =
+
= 0.001 yz + 0.001xy = 0.0005 = 2 13
x z
w v
0.001
yz =
+ = 0.001xz +
xy = 0.0025 = 2 23
3
y z

The strain field becomes:

yz

1 yz

+ xz
ij = 0.001
2 3

1
( xy yz )
2

1 yz

+ xz

2 3

xz
3
1 xy

xz

2 3

1
(xy yz )
2

1 xy

xz

2 3

xy

The strain state at the point E ( x = 1.5; y = 1.0; z = 2.0) is:

ij

x
1
= xy
2
1
2 xz

1
xy
2
y
1
yz
2

0.011
1

0.00025

xz 0.002
6
2

0.011
1

0.00125
yz =
0.001

z 0.00025 0.00125 0.0015


(6.14)

b) The normal strain component is obtained as follows:


M

M = M

indicial

M = ij M i M j

(6.15)

By expanding the above equation and by considering the symmetry of the strain tensor we
obtain:
M = 11 M 12 + 22 M 22 + 33 M 32 + 212 M 1M 2 + 213 M 1M 3 + 2 23 M 2 M 3

(6.16)

in engineering notation:
M = x M 12 + y M 22 + z M 32 + xy M 1M 2 + xz M 1M 3 + yz M 2 M 3

University of Castilla-La Mancha


Ciudad Real Spain

Draft

(6.17)

By: Eduardo W. V. Chaves (2015)

6 LINEAR ELASTICITY

463

The vector components M i is given by the direction cosines of the direction of the line
EA :
1
M 2 =
;
5

M 1 = 0 ;

2
M 3 =
5

(6.18)

By substituting the corresponding values into the equation (6.17), we obtain:


M = y M 22 + z M 32 + yz M 2 M 3

(6.19)

1
4
2
M = 0.001 + (0.0015) + (0.0025) = 2 10 3
5
5
5

c) For small deformation, the distortion of the angle at the point E formed by the lines
EA and EF , with = 90 ), becomes:
M N =

N
1
1 2M
M N =
2
2
sin


= M N

M N = ij M i N j

components

(6.20)

More details of the above equation is provided in the textbook in Chapter 2 Continuum
kinematics (small deformation regime). Expanding the above expression and considering
the symmetry of the strain tensor we obtain:

M N = 11 M 1 N 1 + 22 M 2 N 2 + 33 M 3 N 3 + 12 M 1 N 2 + M 2 N 1 +

+ 13 M 1 N 3 + M 3 N 1 + 23 M 2 N 3 + M 3 N 2

(6.21)

or in engineering notation:

M N

xy
= x M 1 N 1 + y M 2 N 2 + z M 3 N 3 +
M 1 N 2 + M 2 N 1 +
2
2
yz

+ xz M 1 N 3 + M 3 N 1 +
M 2 N 3 + M 3 N 2
2
2

(6.22)

and by considering the following unit vectors according to EA and EF direcctions


respectively:

M i = 0

1
5

N i = [ 1 0 0]

(6.23)

we can obtain:
M N

1
2
0.011
+ (0.00025)(1)

= 12 M 2 N 1 + 13 M 3 N 1 =
(1)
2
6
5
5

M N
2

= 5.96284793998 10 4

(6.24)

M N = 1.1925696 10 3

Alternative Solution
We can construct an orthogonal basis associated with the unit vectors M and N by means
of the cross product P = M N . Then, we obtain the components of the unit vector P :
e 1
N = 0
P = M
1

e 2
1

e 3
2

5
0

5
0

University of Castilla-La Mancha


Ciudad Real - Spain

2
1
e2
e3
5
5

Draft

Pi = 0

2
5

(6.25)

By: Eduardo W. V. Chaves (2015)

SOLVING PROBLEMS BY MEANS OF CONTINUUM MECHANICS

464

Then, the transformation matrix from the X 1 X 2 X 3 -system to the base constituted by the
unit vectors M , N and P are given by:
M 1

A = a ij = N 1
P
1

M 2
N 2
P
2

M 3 0

N 3 = 1
P3 0

1
5
0
2
5

5
0
1
5

(6.26)

Applying the component transformation law for a second-order tensor components, i.e.
ij = a ik a jl kl or in matrix form = A A T :

0.011
0.00025

2 0.002
0
6

5 0.011

0
0.001
0.00125 1

1 6
0.00025 0.00125 0.0015 0

5

= 1
0

5
0
2
5

1
5
0
2
5

5
0
1
5

(6.27)

Thus:
M N =

2 10 3

ij = 5.96284794 10 4

4
2.5 10

M N
2

5.96284794 10 4
2 10 3
1.75158658 10 3

3
1.75158658 10

1.5 10 3

2.5 10 4

(6.28)

NOTE: Note that this example is not a case of homogeneous deformation, i.e. a straight
edge in the reference configuration is no longer straight in the current configuration. To
obtain the deformed unit vector we must apply the linear transformation m = F M and
n = F N , where F is the deformation gradient.
d) The volume strain by definition is V =

( dV )
where dV is the differential volume.
dV

For small deformation we have:


V =

(dV )
= x + y + z
dV

(dV ) = ( x + y + z )dV

(6.29)

by integrating we obtain the volume variation:


V =

2.0

+ y + z dV = 0.001

1.5

yz +

z = 0 y =0 x = 0

xz

xy dxdydz
3

(6.30)

thus:
V = 1.125 10 3 m 3

University of Castilla-La Mancha


Ciudad Real Spain

Draft

(6.31)

By: Eduardo W. V. Chaves (2015)

6 LINEAR ELASTICITY

465

Then:
V =

(dV )
1.125 10 3
=
= 0.375 10 3
dV
1 .5 1 .0 2 .0

(6.32)

Problem 6.3
The stress state at one point of a structure, which is made up of an isotropic linear elastic
material, is given by:
6 2 0
ij = 2 3 0 MPa
0 0 0

a) Obtain the engineering strain tensor components. Consider the Youngs modulus
( E = 207GPa ) and the shear modulus ( G = 80GPa ).
b) If a cube of side 5cm is subjected to this stress state. Obtain the volume variation.
Solution:
The strain components can be obtained by means of the equations:

)]

1
x y + z = 3.333 10 5
E
1
y = y ( x + z ) = 2.318 10 5
E
1
z = z x + y = 4.348 10 6
E
1
xy = xy = 2.5 10 5
G
1
xy = xy = 0
G
1
xy = xy = 0
G

x =

)]

(6.33)

where the Poissons ratio can be obtained by:


G=

E
2(1 + )

E
207
1 =
1 0.29375
2G
160

Thus:
0
33.24 12.5

0 10 6
ij = 12.5 23.01
0
0
4.257

Alternative solution:
1

In the textbook (Chaves(2013)) we have demonstrated that C e =

(1 + )

I 1 1 , with
E
E

that we obtain
1

(1 + )
(1 + )

(1 + )
= Ce : =
I 1 1 : =
I : 1 1 : =
Tr ( )1
E
E
E
E
E

University of Castilla-La Mancha


Ciudad Real - Spain

Draft

By: Eduardo W. V. Chaves (2015)

SOLVING PROBLEMS BY MEANS OF CONTINUUM MECHANICS

466

In components:
11
(1 + )
ij =
12
E
13

13
1 0 0

23 Tr ( ) 0 1 0
E
0 0 1
23 33
0
6 2 0
1 0 0 33.24 12.5

6
6
= 6.25110 2 3 0 4.2609 10 0 1 0 = 12.5 23.01
0 10 6
0 0 0
0 0 1 0
4.257
0

where we have used

12
22

(1 + )
1
, Tr ( ) = 4.25725 10 6 .
= 6.25 10 6

E
MPa E

In the small deformation regime the volumetric deformation (linear) is equal to the trace of
the strain tensor:
DVL V = I = (33.24 23.01 4.257 ) 10 6 = 5.973 10 6

Then, the volume variation is:


V = V V0 = 5.973 10 6 (5 5 5) = 7.466 10 4 cm 3

Problem 6.4
A parallelepiped of dimensions a = 3cm , b = 3cm , c = 4cm , is made up of an isotropic
homogeneous linear elastic material, which is accommodated in a cavity of the same shape
and dimensions, (see Figure 6.3), whose walls are made up of a very rigid material
(undeformable), (see Ortiz Berrocal (1985)).
Via a rigid plate (dimensions a b ) of negligible weight and negligible friction we apply a
perpendicular compression force equal to F = 200 N which compresses the elastic block.
If = 0.3 is the Poissons ratio and E = 2 10 4 N / cm 2 is the Youngs modulus, calculate:
a) The lateral force exerted by the wall of the cavity on the parallelepiped;
b) The height variation experienced by it, i.e. find c .
F
z
a
y
c
x
b

Figure 6.3

University of Castilla-La Mancha


Ciudad Real Spain

Draft

By: Eduardo W. V. Chaves (2015)

6 LINEAR ELASTICITY

467

Solution:
At any point of the elastic body the stress state is characterized only by normal components
x , y and z . The stress z is given by:
z =

200
200
200 N
=
=
ab
3 3
9 cm 2

(6.34)

Note that, because of the problem symmetry the stresses x and y are equal, then:

)]

1
x y + z = 0
E
x ( x + z ) = 0
x = y =

x =

1
[ x ( x + z )] = 0
E

(6.35)

z
(1 )

thus:
x =

z
0.3 200
200 N
=

=
(1 ) (1 0.3) 9
21 cm2

(6.36)

The force exerted by the wall on the elastic body is given by:
200
3 4 = 114.28 N
21
200
3 4 = 114.28 N
Fx = x b c =
21
Fy = y a c =

The strain z can be obtained as follows:


z =

)]

1
1
1 200
200
z x + y = [ z 2 x ] =

+ 2 0.3
= 8.25 10 4
4
21
E
E
2 10 9

Then, the height variation is given by:


c = z c = 8.25 10 4 4 = 0.0033cm

(6.37)

Problem 6.5
Under the approximation of small deformation theory, the displacement field is given by:
r
u = ( x1 x3 ) 2 10 3 e 1 + ( x 2 + x3 ) 2 10 3 e 2 x1 x 2 10 3 e 3

Obtain the infinitesimal strain tensor, the infinitesimal spin tensor at the point P (0,2,1) .
Solution:
Displacement gradient:
u1

x1
u i u 2
=
x j x1

u 3
x1

u1
x 2
u 2
x 2
u 3
x 2

u1

x3 2( x x )
0
2( x1 x3 )
1
3
u 2
0
2( x 2 + x 3 ) 2( x 2 + x3 ) 10 3
=

x3

0
x1
u 3 x 2
x3

and at the point P(0,2 1) the above becomes

University of Castilla-La Mancha


Ciudad Real - Spain

Draft

By: Eduardo W. V. Chaves (2015)

SOLVING PROBLEMS BY MEANS OF CONTINUUM MECHANICS

468

u i
x j

2( x1 x 3 ) 2 0 2
0
2( x1 x 3 )

=
0
2( x 2 + x3 ) 2( x 2 + x 3 ) = 0 2 2 10 3
x 2
2 0 0
x1
0

In addition we have
ui
= ij + ij
x j

whose components are:


Infinitesimal strain tensor
u j
1 u
ij = i +

2 x j xi

Infinitesimal spin tensor

2 0 2

= 0 2 1 10 3


2 1 0

0 0 0
1 ui u j

= 0 0 1 10 3
ij =
2 x j xi
0 1 0

(6.38)

Problem 6.6
Under the restriction of small deformation theory, the displacement field is given by:
r
u = a ( x12 5 x 22 ) e 1 + (2 a x1 x 2 )e 2 (0) e 3

a) Obtain the linear strain tensor and the linear spin tensor;
b) Obtain the principal strains and principal stresses;
c) Given the shear modulus G , obtain the Youngs modulus E to guarantee the balance at
any point.
Obs.: The body forces are negligible.
Solution:
a) Considering that u1 = a ( x12 5 x 22 ) , u 2 = 2 a x1 x 2 , u 3 = 0 , the displacement gradient
components are given by:
2 x1a
r

u
( xr u)ij = i = 2ax2
x j
0

10ax2
2ax1
0

0
0
0

Decomposing additively the displacement gradient in a symmetric part (the linear strain
tensor - ij ) and an antisymmetric part (the infinitesimal spin tensor- ij ) we obtain:
ui
= ij + ij
x j

where
1 u u j
ij = i +
=
2 x j xi

2 x1a
1
2ax2
2
0

10ax2
2ax1
0

0 2 x1a
0 + 10ax2
0 0

2ax2
2ax1
0

0 2 x1a

0 = 4ax2
0 0

4ax2

0 0

0 = 6ax2
0 0

6ax2

2ax1
0

0
0
0

and
1 u u j
=
ij = i
2 x j xi

2 x1a
1
2ax2
2
0

University of Castilla-La Mancha


Ciudad Real Spain

10ax2
2ax1
0

0 2 x1a
0 10ax2
0 0

Draft

2ax2
2ax1
0

0
0

0
0
0

By: Eduardo W. V. Chaves (2015)

6 LINEAR ELASTICITY

469

b) The principal strains.


2 x1 a

4ax 2

4ax 2

2ax1

2 x1 a

=0

) = (4ax )
2

(2 x a ) (4ax )
2

=0

1 = 2 x1 a 4ax 2
2 x1 a = 4ax 2
2 = 2 x1 a + 4ax 2
0
0
2 x1a 4ax2 0 (principal strains)
0
0

2 x1a + 4ax2
0
ij =

Since the strain and stress share the same principal space we can use the equation
ij = 4 x1a ij + 2 ij to obtain the principal stresses:
0
1 0 0
2 x1a + 4ax2

ij = 4 x1a ij + 2 ij = 4 x1a 0 1 0 + 2
0
2 x1a 4ax2
0 0 1

0
0
0
0
4 x1a + 2 (2 x1a + 4ax2 )

0
0
=
4 x1a + 2 ( 2 x1a 4ax2 )

0
0
4 x1a

0
0
0

c) Starting from the equilibrium equations:

r r
+ {
b = 0 Indicial

ij , j = 0 i
r
=0

and by expanding the above, we obtain:

ij , j = 0i

11,1 + 12, 2 + 13,3 = 0

21,1 + 22, 2 + 23,3 = 0

31,1 + 32, 2 + 33,3 = 0

11 12 13
+
+
=0

x2
x3
x1
21 22 23
+
+
=0

x2
x3
x1
31 32 33
+
+
=0

x1
x3
x2

and considering that kk = 4 x1 a , the stress tensor components ij = kk ij + 2 ij


becomes ij = 4 x1 a ij + 2 ij , thus
11 = 4 x1 a 11 + 2 11 = 4 x1 a + 2 (2 x1 a ) = 4 x1 a ( + )
12 = 4 x1 a 12 + 2 12 = 2 ( 4ax 2 ) = 8 ax 2
13 = 0

or we can use:
1 0 0
11 12 13

ij = 4 x1 a ij + 2 ij = 4 x1 a 0 1 0 + 2 12 22 23
0 0 1
13 23 33
8 ax 2
0
1 0 0
2 x1 a 4ax 2 0 4 x1 a ( + )

2ax1 0 = 8 ax 2
= 4 x1 a 0 1 0 + 2 4ax 2
4 x1 a ( + )
0
0 0 1
0
0
0
0
0
4 x1 a

Then, the first equilibrium equation becomes:


University of Castilla-La Mancha
Ciudad Real - Spain

Draft

By: Eduardo W. V. Chaves (2015)

SOLVING PROBLEMS BY MEANS OF CONTINUUM MECHANICS

470

11 12 13
+
+
=0
x1
x 2
x3

4a ( + ) 8 a = 0

+ = 2 = = G

G (3 + 2G )
, which was obtained by means of the relationships
+G
E
E
and = G =
. Then, we conclude that:
=
(1 + )(1 2 )
2(1 + )

In addition, note that E =

E=

G (3 + 2G ) G (3G + 2G )
= 2.5G
=
+G
G+G

Problem 6.7
The stress state at a point of the continuum is represented by the Cauchy stress tensor
components:
26 6 0
ij = 6
9 0 kPa
0
0 29

Consider an isotropic linear elastic material.


a) Obtain the principal invariants of ;
b) Obtain the spherical and deviatoric parts of ;
c) Obtain the eigenvalues and eigenvectors of ;
d) Draw the Mohrs circle in stress. Obtain the maximum normal and tangential stress.
e) Considering a small deformation regime and taking into account that the elastic
mechanical properties are = 20000kPa and = 20000kPa ( , are the Lam constants).
Obtain the infinitesimal strain tensor;
f) Obtain the eigenvalues and eigenvectors of .
Solution:
a) The principal invariants
I = 12 10 3
II =

0 29

10 6 +

26

29

10 6 +

26 6
6

10 6 = 763 10 6

III = det ( ) = 7830 10 9

The spherical and deviatoric parts ij = ijdev + ijsph :


1
3

The mean stress: m = ii =

( 29 26 + 9)
= 4 10 3 Pa
3

ijhyd

University of Castilla-La Mancha


Ciudad Real Spain

ijsph

4 0 0
= 0 4 0 kPa
0 0 4

Draft

By: Eduardo W. V. Chaves (2015)

6 LINEAR ELASTICITY

ijdev

471

6
0 30 6 0
26 4

= 6
94
0 = 6
5 0 kPa
0
0
29 4 0
0 25

By solving the characteristic equation we obtain the eigenvalues:


I = 29kPa

II = 10kPa

III = 27 kPa :

The eigenvectors:
I = 29kPa

principal direction

II = 10kPa

principal direction

III = 27kPa

n i(1) = [0 0 1]

n i( 2 ) = [0.1644 0.98639 0]

principal direction

n (i 3) = [0.98639 0.1644 0]

S (kPa)

S max =

29 (27)
= 28
2

N (kPa)

III = 27

I = N max = 29

II = 10

ij = Tr ( ) ij + 2 ij

where

reverse
form

ij =

1
Tr ( ) ij +
ij
2 (3 + 2 )
2

= 5 10 9 ( Pa ) 1 , Tr ( ) = 1.2 10 4 ( Pa)
2 (3 + 2 )

1 0 0
26 6 0

8
ij = (5 10 )(1.2 10 ) 0 1 0 + 2.5 10 6
9 0 10 3
0 0 1
0
0 29
0
1 0 0
26 6 0 7.1 1.5

5
5
9 0 = 1.5 1.65
= 6 10 0 1 0 + 2.5 10 6
0 10 4
0 0 1
0
0 29 0
0 6.65
9

As the material is isotropic, the stress and strain have the same principal directions. We
work on the principal space to obtain:

University of Castilla-La Mancha


Ciudad Real - Spain

Draft

By: Eduardo W. V. Chaves (2015)

SOLVING PROBLEMS BY MEANS OF CONTINUUM MECHANICS

472

ij =

ij
Tr ( ) ij +
2 (3 + 2 )
2

0 66.5 0
0
1 0 0
29 0

5
= 6 10 0 1 0 + 2.5 10 0 10
0 = 0 19
0 10 5
0 0 1
0 0 27 0
0 73.5
5

Problem 6.8
Show that the constitutive equations in stress, for an isotropic linear elastic material, can be
represented by the set of equations:
dev = 2 dev

Tr ( ) = 3Tr ( )

where = G is the shear modulus, and is the bulk modulus.


Solution:
= C e : = [1 1 + 2 I] : = Tr ( )1 + 2
= dev + sph = Tr ( )1 + 2 ( dev + sph )
Tr ( )
1 = Tr ( )1 + 2 ( dev + sph )
3
Tr ( ) Tr ( )
1
= Tr ( )1 + 2 dev + 2

3
3
2
Tr ( )

1
= +
Tr ( )1 + 2 dev
3
3

dev +
dev
dev

The trace of the stress tensor:


Tr ( ) = : 1 = [Tr ( )1 + 2 ] : 1 = Tr ( )3 + 2 Tr ( ) = (3 + 2 )Tr ( )

with that we obtain:


2
Tr ( )

dev
1
dev = +
Tr ( )1 + 2
3
3

(3 + 2 )Tr() 1
2

dev
dev = +
Tr ( )1 + 2
3
3

(3 + 2 )Tr() 1 + 2 dev
2

dev = +
Tr ( )1
3
14434444
424444444
3
=0

To the equations dev = 2 dev we must add the constraint:

(3 + 2 )Tr( ) 1 = 0 + 2 Tr( )1 Tr ( ) 1 = 0
2

+
Tr ( )1

3
3
3
3

Tr ( )1 = 3 +
Tr ( )1 = 3 Tr ( )1
Tr ( )1
3

or Tr ( ) = 3 Tr ( ) .

University of Castilla-La Mancha


Ciudad Real Spain

Draft

By: Eduardo W. V. Chaves (2015)

6 LINEAR ELASTICITY

33
23

13
13

12

dev
33

23

12

ij = Tr ( ) ij + 2 ij

23

12

11

12

23

13
13

22
12

dev
22

dev
33

23

12

ijdev = 2 ijdev

13

23

dev
11

Tr ( ) ij = 3 Tr ( ) ij

33

23

13
13

22

11

13

473

23
12

dev
22
12

dev
11

Alternative solution:
Starting from the constitutive equation in stress for an isotropic linear elastic material
= ( ) = Tr ( )1 + 2 , and by considering the linear regime the relationship
= ( ) = ( sph + dev ) = ( sph ) + ( dev ) holds, where:
( sph ) = Tr ( sph )1 + 2 sph
2
Tr ( )
Tr ( )
Tr ( )
sph = Tr
11 + 2
1 = Tr ( )1 + 2
1 = ( +
) Tr ( )1 = Tr ( )1
3
3
3
3

Tr ( )
1 = Tr ( )1
3
Tr ( )1 = 3 Tr ( )1
( dev ) = Tr ( dev )1 + 2 dev = 2 dev
1
424
3
=0

[ ]

Note that Tr ( sph ) = Tr sph

University of Castilla-La Mancha


Ciudad Real - Spain

= Tr[ ] holds.

Draft

By: Eduardo W. V. Chaves (2015)

SOLVING PROBLEMS BY MEANS OF CONTINUUM MECHANICS

474

NOTE: Note that for an isotropic material if we have a purely spherical state of
compression:
p

p>0

0
p 0

ij = 0 p 0
0
0 p

p
p

Tr ( ) = 3 p < 0

E
, we can conclude
3(1 2 )
that: if > 0.5 this implies that < 0 and as consequence Tr ( ) > 0 , i.e. an expansion,

we have that Tr ( ) = 3 Tr ( ) < 0 , and considering that =

which has no physical meaning for a compression state in isotropic materials. With that we
conclude that < 0.5 .
Problem 6.9
A parallelepiped of dimensions a = 0.10m , b = 0.20m , c = 0.30m , (see Figure 6.4), which is
made up of an elastic material with Poissons ration = 0.3 and Youngs modulus
E = 2 10 6 N / m 2 . Said parallelepiped is introduced into a cavity of width b whose walls are
very rigid, so that two opposite faces of the parallelepiped are in contact with the cavity
walls.
Once the parallelepiped is this position the temperature is raised in T = 30 C .
a) Calculate the values of the principal stresses at the point of the parallelepiped.
b) Find the strain components.
Consider that the thermal expansion coefficient of the material is 1.25 10 5 C 1 .
Solution:
As the solid can deform freely according to the directions x and z , hence the normal
stresses are x = z = 0 . The solid is restricted to move according to the y -direction,
hence y = 0 :
y =

1
1
y ( x + z ) + T = y + T = 0
E
E

y = E T

thus:
y = E T = 2 10 6 1.25 10 5 (30) = 750

N
m2

The Cauchy stress tensor components:


0
0
0

ij = 0 750 0 Pa
0
0
0

b)

x = z =

y
E

+ T = 4.875 10 4

The strain tensor components:

University of Castilla-La Mancha


Ciudad Real Spain

Draft

By: Eduardo W. V. Chaves (2015)

6 LINEAR ELASTICITY

475

0
4.875 0

ij = 0
0
0 10 4
0
0 4.875
z

Data:

a = 0.10m
b = 0.20m
c = 0.30m
E = 2 10 6 N / m 2

= 0 .3
T = 30 C

= 1.25 10 5 C 1

c
y

b
x

Figure 6.4

Problem 6.10
Consider a container with very rigid walls, and a squared cross section with dimensions
0.10 0.10m . In the interior of said container is placed a synthetic rubber block whose
dimensions are 0.10 0.10 0.5m , as shown in Figure 6.5(a). The rubber block fits
perfectly into the rigid container.
The elastic characteristics of the rubber are E = 2.94 10 6 N / m 2 and = 0.1 .
Above the rubber is poured 0.004m 3 of mercury, whose mass density is 13580kg / m 3 .
a) Obtain the height H that reach the mercury, (see Figure 6.5(b));
b) The stress state at a generic point of the rubber block.
Hypothesis: 1) the weight of the rubber is negligible. 2) Consider the acceleration of gravity
equal to g = 10m / s 2 , and that between the rubber block faces and the container walls there
is no friction.

University of Castilla-La Mancha


Ciudad Real - Spain

Draft

By: Eduardo W. V. Chaves (2015)

SOLVING PROBLEMS BY MEANS OF CONTINUUM MECHANICS

476

Rigid walls

Rigid walls

Mercury

Rubber
L0 = 0 . 5 m
y

a)

b)
Figure 6.5

Solution:
First we calculate the total force due to the weight of mercury on the rubber:
kg
kgm

m
F = V g = 0.004(m 3 ) 13580 3 10 2 = 543.20 2 N
s
m
s

The normal stress according to the z -direction is given by:


z =

F
543.20
N
=
= 54.320 10 3 2
A
(0.1 0.1)
m

According to the directions x and y the rubber does not deform, hence x = y = 0

x = ( y + z )

y = ( x + z )

1
x ( y + z ) = 0
E
1
y = y ( x + z ) = 0
E

x =

{[

y = ( x + z ) = ( y + z ) + z =

( 2 + )
z =
z = 6035.55 Pa = x
2
(1 )
(1 )

The normal strain according to the z -direction is given by:


z =

1
1
{ 54320 0.1[2(6035.55)]} = 0.0180656
z ( x + y ) =
E
2.94 10 6

b) The length variation of the rubber block is:


L = L0 z = 0.5 ( 0.018656 ) = 0.00903m
University of Castilla-La Mancha
Ciudad Real Spain

Draft

By: Eduardo W. V. Chaves (2015)

6 LINEAR ELASTICITY

477

The height H reached by the mercury is given by:


H = hmer + ( L0 L)

where hmer is given by:


Vmer = b 2 hmer = 0.004

hmer =

0.004
= 0.4m
0.1 0.1

thus:
H = hmer + ( L0 L) = 0.4 + (0.5 0.00903) = 0.891m

Problem 6.11
By means of a material test in the laboratory, it was obtained the following relationships:
31
21
1
z
y +
x = x +
E2
E1
E3
32
1
12
z
x + y +
y =

E2
E1
E3

(6.39)

1
23
13
y + z
x +
z =
E3
E2
E1

where 12 = 0.2 , 13 = 0.3 , 23 = 0.25 , E1 = 1000 MPa , E 2 = 2000MPa , E3 = 1500MPa .


Knowing that the analyzed material is orthotropic, obtain 21 , 31 , 32 .
Solution:
The constitutive matrix for orthotropic material has the following format:
C11
C
12
C
[C ] = 13
0
0

C12

C22
C23
0
0
0

C13

C23
C33
0
0
0

0
0

0
0

C44
0
0

0
C55
0

0
0
0

0
0

C66

Orthotropic symmetry
9 independent constants

(6.40)

By restructuring (6.39) we obtain


1

E
xx 1

12
yy E
zz 1

= 13

xy


2 yz E1

0
2 xz
0
0

University of Castilla-La Mancha


Ciudad Real - Spain

21

E2
1

E2
23

E2
0
0
0

31

E 0
3
32
0

E3
1

0
E3
0
C44
0
0

Draft

0
0

0
0
0
0
C55
0

0
xx

0 yy

zz
0 xy

0 yz

0 xz
C66

(6.41)

By: Eduardo W. V. Chaves (2015)

SOLVING PROBLEMS BY MEANS OF CONTINUUM MECHANICS

478

Then, for orthotropic material it fulfills that:


21 12

E 2 E1

31 13

E3 E1

32 23

E3 E2

with that we obtain

21
E2

31
E3

32
E3

12

13

23

E1
E1
E2

21 =

E 2 12 2000 0.2
=
= 0.4
E1
1000

31 =

E3 13 1500 0.3
=
= 0.45
E1
1000

32 =

E3 23 1500 0.25
=
= 0.1875
E2
2000

Problem 6.12
Given an isotropic linear elastic material whose elastic properties are E = 71 GPa ,
G = 26.6 GPa , find the strain tensor components and the strain energy density at the point
in which the stress state, in Cartesian basis, is represented by:
20 4 5
ij = 4 0 10 MPa
5 10 15

Solution: Poissons ratio can be obtained by means of the equation: G =


=
11 =

E
2(1 + )

E
1 = 0.335
2G

1
[11 ( 22 + 33 )] = 1 9 [20 0.335 (0 + 15)]10 6 = 211 10 6
E
71 10

1
[ 22 (11 + 33 )] = 1 9 [0 0.335 (20 + 15 )]10 6 = 165 10 6
E
71 10
1
1
= [ 33 (11 + 22 )] =
[15 0.335 (20 + 0 )]10 6 = 117 10 6
E
71 10 9

22 =
33

12 =

1+
1 + 0.335
12 =
( 4 10 6 ) = 75 10 6
9
E
71 10

1+
1 + 0.335
13 =
(5 10 6 ) = 94 10 6
E
71 10 9
1+
1 + 0.335
=
23 =
(10 10 6 ) = 188 10 6
9
E
71 10

13 =
23

thus:
211 75 94
ij = 75 165 188 10 6
94
188 117

Then, the strain energy density for a linear elastic material is obtained by the equation:

University of Castilla-La Mancha


Ciudad Real Spain

Draft

By: Eduardo W. V. Chaves (2015)

6 LINEAR ELASTICITY

1
2

1
2

e = : Ce : = :

indicial

479

1
2

e = ij ij

Next, by considering the symmetry of the tensors and , the strain energy density can
be calculated as follows:
1
[1111 + 22 22 + 33 33 + 212 12 + 2 23 23 + 21313 ]
2
1
= [( 211)( 20) + ( 165)(0) + (117 )(15) + 2( 75)( 4) + 2(188)(10) + 2(94)(5) ] = 5637 .5 J / m 3
2

e=

We can also obtain the strain energy density by using the equation:

e =

1
1
1
1
I 2 II dev =
I 2 + J 2
6(3 + 2 )
2
6(3 + 2 )
2

and if we consider that I = 3.5 10 7 ; II = 2.4933 1014 ; 5.3804 10 10 Pa ; = G , we


can obtain e 5638 .03 J / m 3 . Note that any discrepancies in the numerical results of e
are due to numerical approximations.
Problem 6.13
Find the strain energy density in terms of the principal invariants of .
Solution:
1
2

1
2

e = : = : [Tr ( )1 + 2 ] =
=

[Tr ( )]

+ Tr ( T ) =

2
[Tr ( )]2
=
+ Tr ( 2 )
2

Tr ( )
2

[Tr ( )]

2
1 + : =
:3
1
Tr ( )

[Tr ( )]2
2

+ :

+ Tr ( )

We can add and subtract the term [Tr ( )]2 without altering the above outcome:

e=

[Tr ( )]2
2

+ [Tr ( )] + Tr ( 2 ) [Tr ( )] =
2

1
( + 2 )[Tr ( )]2 [Tr ( )]2 Tr ( 2 )
2

Finally, if we consider that the principal invariants of the strain tensor are I = Tr ( ) ,
II =

1 2
I Tr ( 2 ) , we can obtain:
2
1
2

e = ( + 2 )I 2 2 II = e ( I , II )

Problem 6.14
The responses of a liner thermoelastic solid due to two actions are known, namely:
r
r
r (I )
r ( II )
r (I )
r ( II )
I (b ( I ) , t * on S ; u* ; on S ur ; T ( I ) ) and II (b ( II ) , t * on S ; u* ; on S ur ; T ( II ) ) .
Obtain the response of the system formed by I + II and justify, (see Oliver (2000)).

Solution:
As we have a linear regime the following is satisfied:

University of Castilla-La Mancha


Ciudad Real - Spain

Draft

By: Eduardo W. V. Chaves (2015)

SOLVING PROBLEMS BY MEANS OF CONTINUUM MECHANICS

480

r r
r
b = b ( I ) + b ( II )

T = T ( I ) + T ( II )

r
r ( I ) r ( II )
t* = t* + t*

r
r ( I ) r ( II )
u* = u* + u*

The same is true for the fields:


r r
r
u = u ( I ) + u ( II )

= ( I ) + ( II )

= ( I ) + ( II )

Starting from the governing equations of linear thermoelastic equilibrium we have:

The equilibrium equations:

The kinematic equations:

r
r
r
r
r
r
xr + b = xr ( ( I ) + ( II ) ) + (b ( I ) + b ( II ) ) = [ xr ( I ) + b ( I ) ] + [ xr ( II ) + b ( II ) ] = 0

{[
{ [

] [

r
r
r
1 r r (I )
1
x u + ( xr u ( I ) ) T + xr u ( II ) + ( xr u ( II ) ) T
2
2
r ( II )
r (I )
r ( II ) T
+ xr u
+ xr u + xr u

= ( I ) + ( II ) =

] [

]}

r
1
xr u ( I )
2
r
r
r
r
r
rT
T
1
1
= xr u ( I ) + u ( II ) + xr (u ( I ) + u ( II ) ) = xr u + [ xr u] =
2
2
=

] [

]} {

The constitutive equations in stress:

= C e : + MT

where M is the thermal stress tensor


= C e : + MT = C e : ( ( I ) + ( II ) ) + M (T ( I ) + T ( II ) )
= (C e : ( I ) + MT ( I ) ) + (C e : ( II ) + MT ( II ) )
= ( I ) + ( II )

Then, we conclude that all the conditions are met. Then, we can apply the principle of
superposition to linear thermoelastic problem, as expected, since we are in the linear
regime.
Problem 6.15
Let us consider a length rod equal to L = 7.5m , whose diameter is equal to 0.1m , which is
made up of a material whose properties are: E = 2.0 10 11 Pa and = 20 10 6
Initially the rod has a temperature equal to 15 C which later rises to 50 C .

1
.
C

a) Considering that the rod can expand freely, calculate the total elongation of the rod, L ;
b) Now assume that the rod can not expand freely because concrete blocks have been
placed at its ends, (see Figure 6.6(b)). Find the stress in the rod.
Hint: Consider the problem in one dimension.

University of Castilla-La Mancha


Ciudad Real Spain

Draft

By: Eduardo W. V. Chaves (2015)

6 LINEAR ELASTICITY

481

L = L(1) + L( 2 )
L(1)
T

L
L

L( 2)

b)

a)
Figure 6.6: Rod under thermal effect.

Solution: a) To obtain the elongation, we pre-calculate the thermal strain according to the
rod axis direction ij = T ij . Since this is a one-dimensional case, we need only
consider the normal strain component according to the x -direction, 11 = x , then:
11 = x = 20 10 6 (50 15) = 7 10 4

Then, the total elongation, L = L(1) + L( 2) , is obtained by solving the integral:


L

L = x dx = x L = 7 10 4 7.5 = 5.25 10 3 m
0

Note that as the rod can expand freely, it is stress-free.


b) If the ends can not move, there will be a homogeneous stress field equal to:
x = E T = E " x " = 2.0 1011 7 10 4 = 1.4 108 Pa

Note that in the case 2) there is no strain, since L = 0 . Moreover, it is the same as when
the initial length is equal to L + L in which we apply compression stress in order to
obtain a final length equal to L .
Problem 6.16
Consider an isotropic linear elastic material with the following thermo mechanical
properties E = 10 6 Pa (Youngs modulus), = 0.25 (Poissons ratio), = 20 10 6 C 1
(Thermal expansion coefficient).
Consider that that at one point of the solid the stress tensor components are given by:
12 0 4
ij = 0 0 0 Pa
4 0 6

a) Obtain the principal stresses and directions of the stress tensor; Obtain the maximum
shear stress.
b) Obtain the strain related to paragraph (a). And find the principal strains and directions.
c) Obtain the strain energy density.

University of Castilla-La Mancha


Ciudad Real - Spain

Draft

By: Eduardo W. V. Chaves (2015)

SOLVING PROBLEMS BY MEANS OF CONTINUUM MECHANICS

482

d) If the solid undergoes a change in temperature T = 50 C , what is the final strain state
at this point?
e) We can say that we are dealing with a state of plane stress?
Solution:
a) We obtain the eigenvalues by solving the characteristic determinant. Note that we
already know an eigenvalue 2 = 0 which is associated with the direction n i( 2) = [0 1 0] .
Then, to obtain the remaining eigenvalues, it is sufficient to solve:
12

=0

2 18 + 56 = 0

Solving the quadratic equation we obtain:


(1,3) =

18 324 224
2

1 = 14

3 = 4

14 0 0
'ij = 0 0 0 Pa
0 0 4

And the eigenvectors (unit vectors) are given by:


eigenvecto
r

1 = 14
2 = 0

eigenvecto
r

3 = 4

eigenvecto
r

2
n i(1) =
0
5
n (i 2 ) = [0 1 0]
1
n i(3) =
5

1
= [0.8944 0 0.4472]
5
2
= [0.4472 0 0.8944]
5

Making the change of nomenclature such that I > II > III , we have I = 14 , II = 4 ,
III = 0 .
S (Pa )

S max = 7

N (Pa)
III = 0

II = 4
I = 14

We can obtain the maximum normal stress as follows:


S max =

University of Castilla-La Mancha


Ciudad Real Spain

I III
(14) (0)
=
= 7 Pa
2
2

Draft

By: Eduardo W. V. Chaves (2015)

6 LINEAR ELASTICITY

483

b) The Cauchy stress tensor components are given by:


ij = Tr ( ) ij + 2 ij

inverse

ij =

Tr ( ) ij +
ij
2
2 (3 + 2 )
1

Remember that = C e : , and the reciprocal form = C e : .


E
E
= 4 10 5 Pa ,
=G =
= 4 10 5 Pa ,
(1 + )(1 2 )
2(1 + )
Tr ( )

Tr ( ) = 18 ,
= 2.5 10 7 Pa ,
= 4.5 10 6 Pa
2 (3 + 2 )
2 (3 + 2 )

where

1
= 1.25 10 6 ,
2

0
5
1 0 0
12 0 4 10.5

6
6
ij = 4.5 10 0 1 0 + 1.25 10 0 0 0 = 0
4.5 0 10 6

0 0 1
4 0 6 5
0
3

For an isotropic linear material the principal directions of the stress and strain match. The
principal strains can be obtained by means of ij =

Tr ( ) ij +
ij in the
2
2 (3 + 2 )

principal space, i.e.:

1 0 0
14 0 0

6
6
ij = 4.5 10 0 1 0 + 1.25 10 0 0 0 =

0 0 1
0 0 4

The strain energy density is given by e =


space to obtain the strain energy density, i.e.:

0
0
13
0 4.5 0 10 6

0
0
0.5

1
1
: = ij ij . We can use the principal
2
2

0
0
13

ij = 0 4.5 0 10 6
0
0
0.5

14 0 0
ij = 0 0 0
0 0 4

c) With that we can obtain:


1
2

1
2

e = ij ij = ij ij =

1
[11 11 + 3333 ] = 92 10 6 J3
2
m

d) Using the principle of superposition:


ij = ij ( ) + ij (T ) = ij ( ) + T ij

we can obtain:
0
5
0
5
10.5
1 0 0 1010.5

6
6
ij = 0
4.5 0 10 + 20 10 (50) 0 1 0 = 0
995.5
0 10 6
5
0 0 1 5
0
3
0
1003

The principal directions of the infinitesimal strain tensor are the same as the stress tensor
principal directions.
e) We can not say that we are dealing with a state of plane stress, since we do not know
information about how stresses vary in the continuum, i.e. the stress field. Remember that
the state of plane stress is considered when the stress tensor field is independent of one
direction.

University of Castilla-La Mancha


Ciudad Real - Spain

Draft

By: Eduardo W. V. Chaves (2015)

SOLVING PROBLEMS BY MEANS OF CONTINUUM MECHANICS

484

Problem 6.17
Let us consider a bar to which at one end we apply a force equal to 6000 N as shown in
Figure 6.7. Find x , y , z , and the length change of the bar. Let us consider that the bar is
made up of a material whose properties are: Youngs modulus: E = 10 7 Pa ; Poissons ratio:
= 0 .3 .

1m
100 m

1m

y, v

y =

6000
11

F = 6000 N

x, u
z, w

Figure 6.7
Solution: Using the normal strain expressions we can obtain:
1
(0.3)(6000 )

x y + z = y =
= 0.00018
E
E
10 7
y 6000
1
y = y ( x + z ) =
=
= 0.0006
E
E
10 7

x =

z =

)]

z x + y = y = 0.00018
E
E

)]

The total change in cross-sectional dimensions is u = w = 0.00018 1 = 1.8 10 4 m , and


the total change in length is v = 0.0006 100 = 6.0 10 2 m .
Problem 6.18
Let us consider a regular quadrangular prism whose material is characterized by
E P = 27.44 105 N / cm 2 (Youngs modulus) and = 0.1 (Poissons ratio). The side length
of the square section is a = 20cm . In both bases of the prism are placed two plates
perfectly smooth and rigid, they are joined together by four cables of area section
A1 = 1cm 2 and Youngs modulus equals to E1 = 19.6 10 6 N / cm 2 . Initially the length of the
prism is equal to l = 1m , (see Figure 6.8).
On two opposite sides of the prism we apply a compressive pressure p = 7350 N / cm 2 .

University of Castilla-La Mancha


Ciudad Real Spain

Draft

By: Eduardo W. V. Chaves (2015)

6 LINEAR ELASTICITY

485

a) Obtain the stress on the cable C ;


b) Obtain the principal stresses in the prism;
c) Obtain the volume variation experienced by the prism.
a) Reference configuration

b) Current configuration

z
z

l = 1m

a
x

Figure 6.8

Solution:
Verify that the cable and the prism deform in the same way according to the z -direction,
thus:
Pz = Cz

On the cable it fulfills that:


C = EC Cz

Cz =

Cz
EC

The stress field in the prism is given by:

0 0

P
ij = 0 p

0 0

4 Cz AC

a2

The strain in the prism according to the direction z :


Pz =

1
1 4 Cz AC
z x + y =
+ p

2
EP
E P
a

University of Castilla-La Mancha


Ciudad Real - Spain

)]

Draft

By: Eduardo W. V. Chaves (2015)

SOLVING PROBLEMS BY MEANS OF CONTINUUM MECHANICS

486

Applying that Pz = Cz :
Pz = Cz

C
1 4 z AC
+

E P a 2

C
p = z
E C

After some algebraic manipulations we obtain the stress on the cable:


Cz =

EC pa 2
( E P a 2 + 4 EC AC )

N
0.1 19.6 106 7350 20 2
= 4900 2
5
2
6
cm
(27.44 10 20 + 4 19.6 10 1)

The normal stress in the prism according to the z -direction becomes:


Pz =

4Cz AC
4 4900 1
N
=
= 49 2
2
2
a
20
cm

ijP

0
0
0
N

0
= 0 7350
cm 2
0
0
49

The volume variation experienced by the prism is obtained as follows:


V = V V0

where V = I is the linear volumetric deformation (small deformation regime):


V = I = x + y + z =

x + y + z
EP

(1 2 ) = 2.12857 103

and V0 = 4 10 4 cm 3 is the initial prism volume, thus:


V = V V0 = (2.12857 10 3 )(4 10 4 ) = 85.1428cm 3

Problem 6.19
Two parallelepiped of the same material and the same shape a b c are placed on either
side of a rigid flat plate attached thereto by their sides a c . Both parallelepipeds, together
with the plate, are introduced into a cavity such as indicated in Figure 6.9. The walls of the
cavity are flat, rigid and perfectly smooth.
We apply the pressures (force per unit surface area) p1 and p 2 on the upper faces of the
prisms as indicated in Figure 6.9.
Consider the Youngs modulus E and the Poissons ratio .
a) Obtain the principal stresses in both prisms;
b) Obtain the block edge length variations.

University of Castilla-La Mancha


Ciudad Real Spain

Draft

By: Eduardo W. V. Chaves (2015)

6 LINEAR ELASTICITY

487

plate

a
p1

p2

cavity

c
y

Figure 6.9
Solution:
Prism 1:

(x1) = 0

(y1)

(z1) = p1

Prism 2:

(x2) = 0

(y2)

(z2) = p 2

For compatibility of stress:


(y1) = (y2) = y

y (z1)

)]

)]

1 (1)
1
y (x1) + (z1) + (y2 ) (x2) + (z2) = 0
E
E
( 2)
+ y z = 0
y + p1 + y + p 2 = 0

(y1) + (y2 ) = 0

] [

] [

thus
y =

( p1 + p2 )

( p1 + p2 )

Prism 1:

(x1) = 0

(y1) =

Prism 2:

(x2 ) = 0

(y2) =

( p1 + p2 )
2

2
;

(z1) = p1

(z2 ) = p 2

The strains in each prism are given by:


Prism 1:

University of Castilla-La Mancha


Ciudad Real - Spain

Draft

By: Eduardo W. V. Chaves (2015)

SOLVING PROBLEMS BY MEANS OF CONTINUUM MECHANICS

488

(x1) =

)]

(y1)

)]

)]

(x2 ) =

)]

(y2 )

)]

)]

(z1)

1 (1)
[ ( p1 + p2 ) + 2 p1 ]
x (y1) + (z1) =
E
2E
1

= (y1) (x1) + (z1) =


( p1 p2 )
E
2E
1
1 2
= (z1) (x1) + (y1) =
( p1 + p2 ) 2 p1
E
2E

Prism 2:

(z2 )

1 ( 2)

[ ( p1 + p2 ) + 2 p2 ]
x (y2 ) + (z2) =
E
2E
1

= (y2 ) (x2 ) + (z2) =


( p 2 p1 )
E
2E
1
1 2
= (z2 ) (x2 ) + (y2) =
( p1 + p2 ) 2 p2
E
2E

The edge variations:


Prism 1

Prism 2

a
[ ( p1 + p2 ) + 2 p1 ]
2E
b
= (y1) b =
( p1 p2 )
2E
c 2
= (z1) c =
( p1 + p2 ) 2 p1
2E

a ( 2 ) = (x2 ) a =

b (1)

b ( 2 )

c (1)

c ( 2)

[ ( p1 + p2 ) + 2 p2 ]
2E
b
= (y2 ) b =
( p2 p1 )
2E
c 2
= (z2 ) c =
( p1 + p2 ) 2 p2
2E

a (1) = (x1) a =

(6.42)

Problem 6.20
A metallic cube with sides a = 0.20m is immersed in the sea at a depth z = 400m .
Knowing the Youngs modulus of the metal E = 21 1010 Pa , and the Poissons ratio
= 0.3 , calculate the volume variation that is experienced by the cube. Consider the
acceleration of gravity equals to g = 10m / s 2 .
Hypothesis: Although the mass density varies with temperature, salinity, and pressure
(depth), consider that the mass density of seawater equal to = 1027 kg / m 3 and constant.
Solution:
Because of the depth and cube dimensions we can take as a good approximation that the
whole cube is subjected to the same pressure, (see Figure 6.10).

University of Castilla-La Mancha


Ciudad Real Spain

Draft

By: Eduardo W. V. Chaves (2015)

6 LINEAR ELASTICITY

h = 400m

489

Figure 6.10
F
, where A is the area and F can be obtained by
A
means of the Newtons second law F = ma = V g (weight of water column). Then:

The pressure can be obtained by p =

p=

Ah g
kg
kg m
F V g
m
=
=
= gh = 1027 3 10 2 400m = 4.108 10 6 2 2 = 4.108 10 6 Pa
A
A
A
m
s
m s

The stress tensor components in the cube are given by:


0 4.108
0
0
p 0

ij = 0 p 0 = 0
0 MPa
4.108
0
0 p 0
0
4.108

As we have only normal stress components and the material is isotropic, only normal
strains appear:
z = y = x =

)]

1
1
[ 4.108 0.3 ( 4.108 4.108)] 10 6
x y + z =
10
E
21 10

Thus
z = y = x = 7.82 10 6

In small deformation regime the linear volumetric deformation is equal to the trace of the
infinitesimal strain tensor:
V
= DVL V = Tr ( )
V0

V = V0 Tr ( ) = 0.2 3 (2.346 10 5 ) = 1.8768 10 7 m 3

where we have considered that Tr ( ) = 2.346 10 5 .


Problem 6.21
A solid cylinder of radius 0.05m and height 0.25m is made up of a material whose
mechanical properties are: E = 3 10 4 MPa (Youngs modulus) and = 0.2 (Poissons
ratio). Said cylinder is placed between two pistons, which can be considered infinitely rigid,
and all of this is enclosed in a hermetically sealed container as shown in Figure 6.11.

University of Castilla-La Mancha


Ciudad Real - Spain

Draft

By: Eduardo W. V. Chaves (2015)

SOLVING PROBLEMS BY MEANS OF CONTINUUM MECHANICS

490

The container is filled with oil, and by suitable mechanism, the fluid pressure is raised to
the value 15MPa . By operating the mechanical press, we apply a total axial force of
F = 2.35619 10 5 N (piston force+pressure) on the bases of the cylinder.
At a generic point of the body:
a) Obtain the stress tensor components;
b) Obtain the strain tensor components;
c) Obtain the displacement field components ( u , v , w ).
F
y

0.25m

Corte AA
F
0.1m

Figure 6.11: Triaxial compression test.


Solution:
a) The stress tensor components
z =

F
2.35619 10 5
=
= 30MPa
A
(0.05) 2

x = y = 15MPa

Thus:
0
15 0

ij = 0
15
0 MPa
0
30
0

b) For an isotropic linear elastic material, the normal stress only produce normal strain,
then:

University of Castilla-La Mancha


Ciudad Real Spain

Draft

By: Eduardo W. V. Chaves (2015)

6 LINEAR ELASTICITY

491

)]

x = E x y + z

y = y ( x + z )
E

z = E z x + y

)]

By substituting the values of the variables we obtain the following strain tensor
components:
0
2 0

ij = 0 2 0 10 4
0
0 8

c) The displacement field


As we are considering the small deformation regime, the following is fulfilled:
x =

u
x

y =

v
y

z =

w
z

Integrating and obtaining the constants of integration we finally obtain the displacement
field:
u = 2 10 4 x

v = 2 10 4 y

w = 8 10 4 z

Problem 6.22
A hexahedron with sides 0.1m is made up of a material whose mechanical properties are
represented by the Lam constants: = 8333.33MPa , = 12500 MPa .
A deformation is imposed to the material as shown in Figure 6.12, in which all faces
remains planar, the faces AEFB and DHGC become parallelogram and the remaining
faces continue squares:
a) Obtain the displacement field;
b) Obtain the strain field;
c) Obtain the stress field;
d) Obtain the principal strains and stresses in the center of the hexahedron;
e) Obtain the actions performed by the testing machine on the faces ABFE and BCGF .

University of Castilla-La Mancha


Ciudad Real - Spain

Draft

By: Eduardo W. V. Chaves (2015)

SOLVING PROBLEMS BY MEANS OF CONTINUUM MECHANICS

492

z
H

G
tan( ) = 0.001

C = C
D = D

A = A

y
B = B

Figure 6.12: The deformed hexahedron.


Solution:
a) According to Figure 6.12 we can verify that there are only shear strain components.
Moreover we can also verify that there is no displacement in the directions x and z , then
u = 0 , w = 0 . By means a triangle analogy we can obtain the displacement v :
tan( ) = 0.001 =

v
z

v( z ) = 0.001z

The displacement field:


z
u = 0

v( z ) = 0.001z
w = 0

v(z )

b) By considering the strain tensor components:

University of Castilla-La Mancha


Ciudad Real Spain

Draft

By: Eduardo W. V. Chaves (2015)

6 LINEAR ELASTICITY

x
1
ij = xy
2
1
2 xz

1
xy
2
y
1
yz
2

u
1
xz
x

2
1 v u
1
yz = +
2 2 x y

z 1 w u
2 x + z

493

1 v u
+
2 x y
v
y
1 w v

+
2 y z

1 w u
+

2 x z
1 w v

+
2 y z

we can conclude that x = y = z = xy = xz = 0 and the component yz is given by:


yz =

x
1
ij = xy
2
1
2 xz

1
xy
2
y
1
yz
2

v w
= 0.001
+
z y
1

xz
2
0
0
0
1

yz = 0
0
0.0005
2

0 0.0005
0
z

c) The stress field = Tr ( )1 + 2


Considering Tr ( ) = 0 , = 8333.33MPa , = 12500MPa , we obtain:
0
0
0
0
0 0

ij = 2 (12500) 0
0
0.0005 MPa = 0 0 12.5 MPa
0 0.0005
0 12.5 0
0

d) The principal strains:

0.0005
=0
0.0005

2 = 0.0005 2

2 = +0.0005
= 0.0005
3 = 0.0005

Remember that in the small deformation regime, the stress and strain share the same
principal directions, then we work in the principal space to obtain the principal stresses by
using = Tr ( )1 + 2 , i.e.:
0
0
0
0

0 0

0
0 MPa
ij = 2 (12500) 0 0.0005
MPa = 0 12.5
0
0 0
0
0.0005
12.5

e) To obtain the total force acting on one surface, we multiply the surface force by the area
of the corresponding face. The surface force is obtained by means of the traction vector

t (n) = n . For the face ABFE the unit vector is given by n i = [1,0,0] , thus:
t 1 ( ABFE ) 0 0
0 1 0
( ABFE )

= 0 0 12.5 0 = 0
t 2
t ( ABFE ) 0 12.5 0 0 0


3

For the face BCGF , the unit vector is given by n i = [0,1,0] , thus

University of Castilla-La Mancha


Ciudad Real - Spain

Draft

By: Eduardo W. V. Chaves (2015)

SOLVING PROBLEMS BY MEANS OF CONTINUUM MECHANICS

494

t 1 ( BCGF ) 0 0
0 0 0
( BCGF )

= 0 0 12.5 1 = 0 MPa
t 2
t ( BCGF ) 0 12.5 0 0 12.5

If we do the same procedure for the other faces we obtain the representation of the surface
forces on the faces as indicated in Figure 6.13:
z

C = C

D = D
A = A

B = B

Figure 6.13: The surface forces in the hexahedron.


Problem 6.23
Consider the prism as indicated in Figure 6.14, we apply the forces F1 = 10 N and F2 = 2 N
as indicated in said figure. The prism edge lengths are: AB = 4cm , AD =
Consider the following material properties: E = 2.5 10 6

= 0.25 (Poissons ratio), and = 5 10 8

N
cm 2

10
cm , AA = 2cm .
3

(Youngs modulus),

1
(thermal expansion coefficient).
C

a) Obtain the principal stresses; b) Obtain the traction vector on the plane . Is it on that
plane where the maximum shear acts? Justify your answer. c) Obtain the values of the
forces F1 and F2 to be applied to guarantee that in the solid there is no displacement
according to the directions x1 and x2 , when the prism is subjected to a temperature
variation of T = 20 C .

University of Castilla-La Mancha


Ciudad Real Spain

Draft

By: Eduardo W. V. Chaves (2015)

6 LINEAR ELASTICITY

x2

495

A2
A1

F2

F1

F1

x1

60

F2
x3

Figure 6.14
a) The stress field

A1 = 8.0 , A2 = 4

10
3

F1
A
1
ij = 0

F2
A2
0

0
0
0
1.25
N

0 = 0
0.15 0

cm 2

0
0
0

whose values are the principal stresses, since there is no shear stresses.
b)
x2

r
t (n)

D
x1

60

x3

University of Castilla-La Mancha


Ciudad Real - Spain

Draft

By: Eduardo W. V. Chaves (2015)

SOLVING PROBLEMS BY MEANS OF CONTINUUM MECHANICS

496

r
3

1
The unit vector components are: n i = ;
; 0 . Then, the traction vector t (n) is

given by:

r
t (n) = n

t i(n) = ij n j

t (i n)

0
0
1.25
= 0
0.15 0

0
0
0

2 1.0825
1
= 0.075
2
0 0

The normal stress component is:

N = t (n) n = t i(n) n i

N = [1.0825 0.075 0]

2
1
= 0.9
2
0

The tangential stress component is obtained as follows:


r
t (n)

= 2N + 2S

S =

r
t (n)

2N

where
r
t (n)

1.0825
r (n ) r (n )
(n ) (n )
= t t = t i t i = [1.0825 0.075 0] 0.075 = 1.1775
0

Thus:
S =

r
t (n)

2N = 1.1775 0.9 2 = 0.60621778

The Mohrs circle in stress is drawn as follows

III = 0.15

I = 1.25

N ( N / cm 2 )

We verify that for any point in the solid, the maximum tangential stress is on the plane
2
defined by the unit vector n i = ;

2
; 0 and the maximum tangential stress is:
2

University of Castilla-La Mancha

Draft

Ciudad Real Spain

By: Eduardo W. V. Chaves (2015)

6 LINEAR ELASTICITY

max =

497

I III
= 0.7 > S
2

c) We consider the following strain field:


=

1+

Tr ( )1 + T 1
E
E

ij =

1+

ij Tr ( ) ij + T ij
E
E

For the particular case Tr ( ) = 11 + 22 we have:


0 0 0
0 0 0 = 1 +

E
0 0 33

11 0
0
22

0
0

0
1 0 0

0 + T Tr ( ) 0 1 0
E

0 0 1
0

Then, we set the following system:

1+

1+

11 = 0 = E 11 + T E Tr ( ) = E 11 + T E (11 + 22 )

= 0 = 1 + + T Tr ( ) = 1 + + T ( + )
22
22
11
22

22
E
E
E
E

By solving the above set of equation we obtain:


11 = 22 =

E T
N
= 3.33333 2
(1 )
cm

Then, the forces are given by:


F1 = 11 A1 = 26.66666 N

F2 = 22 A2 = 44.44444 N

University of Castilla-La Mancha


Ciudad Real - Spain

Draft

By: Eduardo W. V. Chaves (2015)

SOLVING PROBLEMS BY MEANS OF CONTINUUM MECHANICS

498

6.2 Two-Dimensional Elasticity (2D)


Problem 6.24
a) Define the state of plane stress and the state of plane strain. b) Obtain the relationships
( ) and ( ) for both plane states. c) Give practical examples in which we can apply
these states.
Solution:
a.1) In the case of plane stress one of the dimensions of the structural elements is very
small when compared to the other two, and the load is perpendicular to the direction of
smallest dimension. As a result of this the stress tensor components found in this direction
are equal to zero, e.g. i 3 = 3i = 0 at any point in the structure.
a.2) In the case of plane strain, the structural element has a prismatic axis, in which the
dimension that corresponds to the direction of the prismatic axis is much larger than the
other dimensions. Additionally, the loads applied are normal to the prismatic axis. Under
these conditions the strain components: 13 , 23 and 33 are zero, i.e. i 3 = 3i = 0 at any
point of the structure.
b.1 State of plane stress
In this case, the stress tensor components are:
11
ij = 12
0

12
22

0 x
0 = xy
0 0

xy
y
0

0
0
0

(6.43)

Let us start from the strain equation:


=

Tr ( )1 +

2 (3 + 2 )
2

ij =

Tr ( ) ij +
ij
2 (3 + 2 )
2

and its trace can be obtained as follows:


:1 =

1
3
1
1
Tr ( )1 : 1 +
:1 =
Tr ( ) +
Tr ( ) =
Tr ( )
2 (3 + 2 )
2
2 (3 + 2 )
2
(3 + 2 )

Tr ( ) =

1
Tr ( )
(3 + 2 )

Tr ( ) = (3 + 2 ) Tr ( )

The component 33 is no longer an unknown since:


33 =

Tr ( ) 33 +
Tr ( )
33 =
{
{
2 (3 + 2 )
2 = 0 2 (3 + 2 )
=1

33 =
33

(3 + 2 ) Tr ( ) =
Tr ( ) =
Tr ( )
2 (3 + 2 )
2 (3 + 2 )
2

=
( 11 + 22 + 33 )
2

33 =

(6.44)

( 11 + 22 )
( + 2 )

The stress components ij = Tr ( ) ij + 2 ij become:

University of Castilla-La Mancha


Ciudad Real Spain

Draft

By: Eduardo W. V. Chaves (2015)

6 LINEAR ELASTICITY

499

0
11 12
1 0 0

ij = (11 + 22 + 33 ) 0 1 0 + 2 12 22 0
0
0 0 1
0 33
11
1 0 0
2

( 11 + 22 )]0 1 0 + 2 12
= [(11 + 22 ) +
( + 2 )
0
0 0 1
0
11 12
1 0 0
2

( 11 + 22 ) 0 1 0 + 2 12 22 0
=
( + 2 )
0
0 0 1
0 33

12
22
0

0
0
33

(6.45)

In indicial notation the above equation becomes:

;
ij = ( + 2 ) Tr ( ) ij + 2 ij

1
=
ij
Tr ( ) ij +
ij

2 (3 + 2 )
2

(i, j = 1,2)

with

Tr ( ) = 11 + 22

(6.46)
(i, j = 1,2,3)

(the same as 3D )

or

E
E

with
Tr ( ) = 11 + 22
; (i, j = 1,2)
ij = (1 2 ) Tr ( ) ij + (1 + ) ij

= Tr ( ) + (1 + )
(i, j = 1,2,3)
(the same as 3D )
ij
ij
ij
E
E

(6.47)

The equation in (6.45) can also be written as follows:


4 ( + )

2
11 +
22

( + 2 )

( + 2 )

ij =
2 12

2 12
4 ( + )

22 +
11
( + 2 )
( + 2 )
0

Taking into account the relationships between mechanical parameters we can obtain:
E
E
E
E (1 )
E (1 )
4 ( + )
2
,
, 2 =
, thus:
=
=
=
=
2
2
( + 2 )
(1 + ) (1 + )(1 ) (1 2 )
(1 ) ( + 2 ) (1 )

( + ) (1 )
0
11
22
12

(
1
)
(
)
0
ij =

12
22
11
(1 2 )

0
0
0

Tr ( ) =

E (11 + 22 )
(1 )

Alternative solution: Voigt notation and engineering notation


Taking into account the conditions i 3 = 3i = 0 , and the relation of ( ) :

University of Castilla-La Mancha


Ciudad Real - Spain

Draft

By: Eduardo W. V. Chaves (2015)

SOLVING PROBLEMS BY MEANS OF CONTINUUM MECHANICS

500

1
x E

y
E

z
= E
xy 0

yz 0

zx
0

E
1
E

E
1
E

1
G

1
G

0
1

y 0

xy
0
yz
0

zx

(6.48)

Then, if we remove the columns and rows associated with the zero stresses, the strainstress relationship ( ( ) ) for the plane stress case is given by:
1
x E

y = E
xy
0

E
1
E

0
x

1
x G= E
1
2 (1+ )
0 y y =
E

xy
0
1 xy

G

x

y
2(1 + ) xy
0
0

(6.49)

The reciprocal of the above equation will result in Hookes law ( ( ) ) for the state of
plane stress:
x
E

y = 1 2
xy

0 x
1

1
0 y

1
0 0
xy
2

(6.50)

Note that the normal strain z is not equal to zero, since z is not just dependant on the
normal stress z :
z =

(11 + 22 )
1
1

z x + y = x + y =
Tr ( ) =
(1 )
E
E
E

)]

[ (

)]

(6.51)

Then, the strain tensor components are represented as follows:


x

ij = 12 xy
0

1
2

xy
y
0

0
z

(6.52)

NOTE 1: If we want to be extremely rigorous there is no state of plane stress, in other


words, there is no real structure such as the strain field has the format presented in Eq.
(6.52). Moreover, as we will see later, the compatibility equations (see Problem 5.11) are
not satisfied, in general, if the strain field has the format presented by the equation in
(6.52). But, when we are dealing with small deformation regime and the thickness ( t ) is
small compared to the other dimensions ( L, h ), (see Figure 6.15), the error committed by
using the state of plane stress is small. Keep in mind that the state of plane stress is always
an approximation.

University of Castilla-La Mancha


Ciudad Real Spain

Draft

By: Eduardo W. V. Chaves (2015)

6 LINEAR ELASTICITY

501

b.2 State of Plane Strain


In this case the strain tensor components are given by:
11
ij = 12
0

12
22
0

0 x

0 = 12 xy
0 0

xy

1
2

y
0

0
0

(6.53)

Let us start from then stress equation:


= Tr ( )1 + 2

ij = Tr ( ) ij + 2 ij

and its trace can be obtained as follows:


: 1 = Tr ( )1 : 1 + 2 : 1
Tr ( ) =

Tr ( ) = 3Tr ( ) + 2 Tr ( ) = [3 + 2 ]Tr ( )

+ 22 + 33
Tr ( )
= 11
3 + 2
3 + 2

The component 33 is no longer an unknown since:


ij = Tr ( ) ij + 2 ij 33 = Tr ( ) 33 + 2 33 33 = Tr ( )

(6.54)

Then, the component 33 is defined as follows:


33 = Tr ( ) =
33

3 + 2

3 + 2

33 =

(11 + 22 + 33 )

3 + 2

(11 + 22 )

=
(11 + 22 )
33 1
3 + 2 3 + 2
33 =

(11 + 22 )
2( + )

Then, the strain components ij =

ij become:
Tr ( ) ij +
2 (3 + 2 )
2

0
11 12
1 0 0
1

0
(11 + 22 + 33 ) 0 1 0 +
12 22
ij =
2
2 (3 + 2 )
0
0 0 1
0 33
11
1 0 0

11 + 22 +
(11 + 22 ) 0 1 0 + 12
=
2 (3 + 2 )
2( + )
0 0 1 2 0

0
11 12
1 0 0
1

(11 + 22 ) 0 1 0 +
0
12 22
=
4 ( + )
2
0
0 0 1
0 33

12
22
0

0
0 (6.55)
33

In indicial notation the above equation becomes:

;
ij = 4 ( + ) Tr ( ) ij + 2 ij

= Tr ( ) + 2
(i, j = 1,2,3)
ij
ij
ij

University of Castilla-La Mancha


Ciudad Real - Spain

(i, j = 1,2)

with

Tr ( ) = 11 + 22

(6.56)

(the same as 3D )

Draft

By: Eduardo W. V. Chaves (2015)

SOLVING PROBLEMS BY MEANS OF CONTINUUM MECHANICS

502

or
(1 + )
(1 + )

ij
;
Tr ( ) ij +
ij =
E
E

E
E
ij =
ij
Tr ( ) ij +

(1 + )(1 2 )
(1 + )

(i, j = 1,2)

Tr ( ) = 11 + 22

with

(6.57)

(i, j = 1,2,3)

(the same as 3D)

The equation in (6.55) can also be written as follows:


(2 + )

11 +
22

4 ( + )

4 ( + )

1
ij =
12
2

1
12
2
(2 + )

22 +
11
4 ( + )
4 ( + )

Taking into account the relationships between mechanical parameters we can obtain:

( + 2 )
(1 + )(1 )
(1 + )
(1 + ) 1
,
,
, thus:
=
=
=
4 ( + )
E
4 ( + )
E
2
E

(1 )
11
22
(1 + )
ij =
12

12
(1 ) 22 11
0

Alternative solution: Voigt notation and engineering notation


If we start from the generalized Hookes law and by deleting the columns and rows
associated with the zero strain, i.e.:
x

y
z

xy

yz

zx

E
0
0
0
=
(1 + )(1 2 )
0

0
0

0
0
0

0
0
1 2
2

0
0
0

1 2
2

x
0 y

0 z

0 xy

0 yz

1 2
zx
2
0

(6.58)

we obtain:

1
E

1
y = (1 + )(1 2 )
xy
0
0


x
0 y
1 2
xy
2
0

(6.59)

Then, the stress according to the direction z is given by:

University of Castilla-La Mancha


Ciudad Real Spain

Draft

By: Eduardo W. V. Chaves (2015)

6 LINEAR ELASTICITY

z =

E
x + y
(1 + )(1 2 )

503

(6.60)

0 x

0 y
2 xy

(6.61)

Additionally, the reciprocal of (6.59) is:


x
1+
y = E
xy

1
0

c.1 Example in which the plane stress approximation is suitable


2D
q

Figure 6.15: Deep beam.


c.2 Some examples in which the plane strain approximation is suitable
y
p - pressure

2D
y
x
p

Cross section
per unit length

prismatic axis
Figure 6.16: Cylinder under pressure.

University of Castilla-La Mancha


Ciudad Real - Spain

Draft

By: Eduardo W. V. Chaves (2015)

504

SOLVING PROBLEMS BY MEANS OF CONTINUUM MECHANICS

2D

Figure 6.17: Tunnel.

2D
1

Cross section of the dam


Figure 6.18: Dam.

University of Castilla-La Mancha


Ciudad Real Spain

Draft

By: Eduardo W. V. Chaves (2015)

6 LINEAR ELASTICITY

505

Problem 6.25
Figure 6.19 (a) shows a support device for a machine. Said support apparatus is made up of
a neoprene block of dimensions ( 50 20cm ), and is characterized by the element ABCD of
Figure 6.19(b).
y

a)

b)

1 .1

D
A

1 .2

A
1

20

1 .1

50

Dimensions in centimeters - cm
Figure 6.19
Under the action of vertical and horizontal loads the neoprene deforms as shown in Figure
6.19 (b) (ABCD) in which the displacement field ( u, v) is represented as follows:
u = a1 x + b1 y + c1
v = a2 x + b2 y + c2

where a1 , b1 , c1 , a 2 , b2 , c 2 are constants to be determined.


a) Calculate the strain tensor components and the volumetric deformation at any point;
b) Calculate the stresses at any point;
c) The maximum normal stress;
d) Obtain the unit extension according to the direction of the diagonal AC .
Hypothesis:
1 Isotropic linear elastic material with Youngs modulus equals to 1000 N / cm 2 and the
shear modulus equals to

1
N / cm 2 .
0.0028

2 It is assumed a state of plane strain.


Solution:
u = a1 x + b1 y + c1

v = a 2 x + b2 y + c 2

(6.62)

According to Figure 6.19 we obtain:


u (0;0) = 1 = c1
u (50;0) = 1.1 = 50a1 + 1 a1 = 0.002

(6.63)

u (0;20) = 1.1 = 20b1 + 1 b1 = 0.005

thus
University of Castilla-La Mancha
Ciudad Real - Spain

Draft

By: Eduardo W. V. Chaves (2015)

SOLVING PROBLEMS BY MEANS OF CONTINUUM MECHANICS

506

u = 0.002 x + 0.005 y + 1

(6.64)

For the vertical displacement:


v(0;0) = 0 = c 2
u (50;0) = 0 = 50a 2 a 2 = 0

(6.65)

u (0;20) = 1 = 20b2 b2 = 0.05


v = 0.05 y

(6.66)

u = 0.002 x + 0.005 y + 1

v = 0.05 y

(6.67)

Then:

a)

Strains
x =

u
= 0.002
x

y =

v
= 0.05
y

xy =

u v
= 0.005
+
y x

(6.68)

The linear volumetric deformation (small deformation regime):


DVL = V = x + y + z = I = 0.048

b)
G=

(6.69)

Stresses
E
E
=
1 = 0 .4
2(1 + )
2G

E
(1 ) x + y
(1 + )(1 2 )
= 3571.4286 [(0.6) 0.002 0.4 0.05] = 67.1428

x =

E
(1 ) y + x
(1 + )(1 2 )
= 3571.4286 [(0.6) (0.05) + 0.4 0.002] = 104.2857

y =

xy = G xy =

1
0.005 = 1.785714
0.0028

An alternative solution is to use ij =


x

ij = 12 xy
1 xz
2

1
2

xy
y

1
2

yz

1
2
1
2

(6.70)

E Tr ( )
E
ij +
ij , where:
(1 + )(1 2 )
(1 + )

xz 0.002

yz = 12 (0.005)
z
0

1
2

(0.005) 0

0.05 0
0
0

E
N
E Tr ()
N
= 714.285714 2
= 68.571429 2 ,
(1 + )(1 2 )
(1 + )
cm
cm

University of Castilla-La Mancha


Ciudad Real Spain

Draft

By: Eduardo W. V. Chaves (2015)

6 LINEAR ELASTICITY

507

0.002
1 0 0

ij = 68.5714290 1 0 + 714.285714 12 (0.005)

0 0 1
0

1
2

(0.005) 0

0.05 0
0
0

0
67.1428 1.785714
N

0
1.785714 104.2857
cm 2

0
0
68.571

c)

The principal stresses


(1,2) =

x + y
2

x y

2

+ 2xy

(6.71)
2

67.1428 104.2857
67.1428 + 104.2857
2

+ 5.35714
2
2

= 171.4285 19.328675

(1,2) =

1 = 152.099824 N cm 2

2
2 = 190.757175 N cm

d)

(6.72)

(6.73)

The unit extension

The diagonal ( AC ) in the reference configuration measures:


(6.74)

L0 = AC = 50 2 + 20 2 = 53.852cm

and the deformed diagonal


AC = 50.2 2 + 19 2 = 53.675cm

L = AC AC = 0.177cm

(6.75)

The unit extension is:


=

L 0.177
=
= 0.0033
L0
53.852

(6.76)

Problem 6.26
Consider a soil made up of a linear elastic material. At a point in the soil the volumetric
deformation is V = 2 10 3 , the shear deformation is 12 = 3 10 3 and the normal
strain is 11 = 0 . The soil is subjected to a state of plane strain according to the plane
x1 x 2 .
a) Obtain the Cartesian components of the infinitesimal strain tensor. Obtain the principal
strains, and the directions where they occur.
1
4

b) Assuming that the elastic constants are E = 50MPa and = . Obtain stress tensor
components and the principal stresses. Obtain the maximum normal and shear stresses.
c) Obtain the strain energy density (per unit volume).

University of Castilla-La Mancha


Ciudad Real - Spain

Draft

By: Eduardo W. V. Chaves (2015)

SOLVING PROBLEMS BY MEANS OF CONTINUUM MECHANICS

508

Solution:
a) The infinitesimal strain tensor components are given by:

ij = 3 10 3

0
0

3 10 3
22
0

The volumetric deformation DVL V = I = 11 + 22 + 33 = 2 10 3 22 = 2 10 3 .


with that we obtain:
0

ij = 3
0

3 0
0

plane strain

ij =
2 0 10 3
3
0
0

3
3
10
2

The principal strains


0 3
=0
3 2

2 + 2 3 = 0

1 = 1

2 = 3

thus
1 = 1 10 3

2 = 3 10 3

b)

1 10 3
ij =
0

0
3
3 10

y
1

xy

yy

University of Castilla-La Mancha


Ciudad Real Spain

Draft

By: Eduardo W. V. Chaves (2015)

6 LINEAR ELASTICITY

509

Mohrs circle in strain


S =

(10 3 )
2
( N = 0; S = 3 )
2

I =1

III = 3

N 10 3

( N = 0; S = 2)
( N = 2; S = 3 )

Note that the radius is R = (1 (3)) / 2 = 2 . Then:


tan(2) =

3
1

2 = arctan( 3 )

b) Applying ij = Tr ( ) ij + 2 ij , where =

= 30

E
= 20MPa ,
(1 + )(1 2 )

E
= 20MPa , Tr ( ) = 2 10 3 . Then:
2(1 + )

3 0

2 0 10 3
0
0

40
0
0
0
3 0

3
0 + 40 3 2 0 10
MPa
= 0
40
142
4
3

=10 Pa
0
0
0
0
0
40

0
1 0 0

ij = Tr ( ) 0 1 0 + 2 3
0
0 0 1

Thus:
40
0
40 3

0 kPa
ij = 40 3 120
0
0
40

As the material is isotropic, the stress and strain share the principal space. In addition, the
eigenvalues of and can be related to each other as follows.
By replacing the value of = Tr ( )1 + 2 into the definition of the eigenvalueeigenvector, we obtain:
n = n

University of Castilla-La Mancha


Ciudad Real - Spain

Draft

By: Eduardo W. V. Chaves (2015)

SOLVING PROBLEMS BY MEANS OF CONTINUUM MECHANICS

510

(Tr()1 + 2 ) n = n
Tr ( )n + 2 n = n

Tr ( )1 n + 2 n = n
2 n = n Tr ( )n

Tr ( )
n
n =

2 n = ( Tr ( ) )n
n = n

Then:
=

Tr ( )
2

= 2 + Tr ( )

And the eigenvalues of can be obtained as follows:


(1) I = 2 (1) + Tr ( ) = (40 10 6 ) (1 10 3 ) + (20 10 6 ) (2 10 3 ) = 0
( 2 ) II = 2 ( 2 ) + Tr ( ) = (40 10 6 ) (0) + (20 10 6 ) (2 10 3 ) = 40 10 3 Pa
(3) III = 2 (3) + Tr ( ) = (40 10 6 ) (3 10 3 ) + (20 10 6 ) (2 10 3 ) = 160 10 3 Pa

We can also use the equation = Tr ( )1 + 2 in the principal space:


40
0
0
0
0
0
1 0 0

3
MPa
0 + 40 0 3 0 10
0 kPa
ij = 0
40
142
4
3 = 0 160
0
=103 Pa
0
0 0 0
40
0
0
40

Mohrs circle in stress:

160

40

N (kPa)

1
2

c) The strain energy density is e = : . We can use the principal space in order to
obtain the strain energy density, i.e.:
1
1
2
2
1
m
N m
J
= (0)(1) + (160 10 3 )(3 10 3 ) + (40 10 3 )(0) = 240 Pa = 240 2 = 240 3
2
m
m m
m

e = ij ij = (11 + 2 2 + 3 3 )

where
1
0

0
2
0

0
0
0
= 0 160
0 10 3 Pa

3 0
40
0
0
0

University of Castilla-La Mancha


Ciudad Real Spain

Draft

1
0

0
2
0

0 1 0 0
0 = 0 3 0 10 3
3 0 0 0

m
m

By: Eduardo W. V. Chaves (2015)

6 LINEAR ELASTICITY

511

Problem 6.27
A solid is subjected to state of plane strain, and at one point the infinitesimal strain tensor
components are given by:
3 0
2

ij = 3 10 0 10 3
0
0
0

Consider that the material has an isotropic linear elastic behavior defined by the Youngs
modulus E = 10MPa and Poissons ratio = 0.25 .
a) Obtain the volumetric deformation and the deviatoric part of the strain tensor;
b) Obtain the principal strains and the principal directions;
c) Obtain the Cauchy stress tensor components;
d) Obtain the maximum and minimum normal stress;
e) It is known that the material fails when the tangential stress exceeds the value 40 kPa .
Check if the material fails.
Solution:
a) Volumetric deformation ( V ):
V = I = Tr ( ) = ( 2 10) 10 3 = 12 10 3

Additive decomposition of the strain tensor = sph + dev , where the spherical part is
given by:
ijsph

0
4 0
Tr ( )

=
ij = 0 4 0 10 3
3
0 4
0

and the deviatoric part is:


ijdev

= ij

ijsph

2
0
3 0 4 0
2 3 0

3
= 3 10 0 0 4 0 10 = 3 6 0 10 3
0
0 0 4
0 4
0
0 0

b) The principal strains are obtained by means of the characteristic determinant:


2
3
=0
3
10

2 + 12 + 11 = 0

Solving the quadratic equation:


(1, 2 ) =

(12) (12) 2 4(1)(11)


2(1)

12 10
2

(1) = 1.0

( 2 ) = 11

Then, the principal strains are:


1 = 1.0 10 3

2 = 11.0 10 3

The principal directions can be obtained by solving ( ij ij )n (j ) = 0 i

University of Castilla-La Mancha


Ciudad Real - Spain

Draft

(i, j = 1,2)

By: Eduardo W. V. Chaves (2015)

SOLVING PROBLEMS BY MEANS OF CONTINUUM MECHANICS

512

The principal direction associated with the eigenvalue (1) = 1.0 :


(1)
(1)
(1)
(1)
3
2 (1)
n1(1) 0 n1 + 3n 2 = 0 n1 = 3n 2

3
10 (1) n (21) 0 3n1(1) 9n (21) = 0

restriction n1(1) + n (21) = 1 , with that we obtain (3n (21) ) 2 + n (21) = 1 n (21) =
n1(1) =

1
10

, and

3
10

The principal direction associated with the eigenvalue (1) = 11.0 :


( 2)
( 2)
3
2 (11)
n1( 2) 0 9n1 + 3n 2 = 0
=

10 (11) n (22) 0 3n1( 2 ) + n (22 ) = 0 n (22) = 3n1( 2 )


3

with the restriction n1( 2) + n (22) = 1 , we obtain n1( 2) =

10

, and n (22) =

3
10

We summarize as follows:
1 = 1 10 3
2 = 11 10 3
1 = 0

3
n i(1) =
10

direction
principal

direction
principal

principal direction

n i(3)

1
n (i 2 ) =
10

= [0 0 1]

1
10
3
10

c) The Cauchy stress tensor components are given by:


ij = Tr ( ) ij + 2 ij

where =

E
E
= 4 MPa , = G =
= 4 MPa , Tr ( ) = 12 10 3 :
(1 + )(1 2 )
2(1 + )

3 0
24
0
64
1 0 0
2

3
ij = 4 (12) 0 1 0 + 2 (4) 3 10 0 10 MPa = 24 128
0 kPa

0 0 1
0
0
48
0
0
0

As the material is isotropic, the principal directions of the stress and strain coincide. The
principal stresses can be obtained by working on the principal space
ij = Tr ( ) ij + 2 ij :

0
0
56
1 0 0
1 0 0

3
ij = 4 (12) 0 1 0 + 2 (4) 0 11 0 10 MPa = 0
136
0 kPa

0
0 0 1
0
48
0 0
0

University of Castilla-La Mancha


Ciudad Real Spain

Draft

By: Eduardo W. V. Chaves (2015)

6 LINEAR ELASTICITY

513

d) By considering that I = 48kPa , II = 56kPa , III = 136kPa , the Mohrs circle in


stress can be obtained as follows:
S (kPa)

S max = 44

II = 56
III = 136

I = 48

N (kPa)

The maximum shear stress can be obtained as follows:


S max =

I III
(48) (136)
=
= 44kPa
2
2

Then, the material fails.


Problem 6.28
A strain gauge (or strain gage) is a device used to obtain the strain in only one direction.
Consider a strain rosette that contains three strain gauges where there are 45 internal angles,
(see Figure 6.20). At one point we have calculated the following strain values:
x = 0.33 10 3

x = 0.22 10 3

y = 0.05 10 3

Find the maximum shear stress at the point in question.


Then consider an isotropic linear elastic material with the following mechanical properties:
E = 29000 Pa (Youngs modulus); = 0.3 (Poissons ratio).
a) Obtain the eigenvalues (principal strains) and eigenvectors (principal directions) of the
strain tensor;
b) Obtain the eigenvalues (principal stresses) and eigenvectors (principal directions) of the
stress tensor.
Hint: Consider the state of plane strain.

University of Castilla-La Mancha


Ciudad Real - Spain

Draft

By: Eduardo W. V. Chaves (2015)

SOLVING PROBLEMS BY MEANS OF CONTINUUM MECHANICS

514

y
x

strain gauge

45
45

Figure 6.20: Strain rosette.


Solution:
Firstly, we have to obtain the strain tensor components in the system x, y, z and to do so
we will use the coordinate transformation law in order to obtain the component xy = 212 .
Remember that in two-dimensional cases, the normal component in a new system is given
by (see Problem 1.98 in Chapter 1):
=
11

11 + 22 11 22
+
cos( 2) + 12 sin( 2)
2
2

The above equation was obtained by means of the transformation law, (see Chapter 1 of
the textbook), which in engineering notation becomes:
x =

x + y
2

x y

cos( 2) +

xy
2

sin( 2)

Then, xy can be obtained as follows:


xy =

( + y ) ( x y )

2
x x

cos( 2) = 0.16 10 3

sin( 2)
2
2

thus
0.33 0.08 0
ij = 0.08 0.05 0 10 3
0
0
0

Then, the stress components can be evaluated as follows:

E
(1 2 ) x + y = 12 .0462 Pa
(1 + )(1 2 )
E
y =
(1 2 ) y + x = 3.5692 Pa
(1 + )(1 2 )
E
E
xy =
xy = 1.7846 Pa ; z =
x + y = 4.684 Pa
2(1 + )
(1 + )(1 2 )
x =

Additionally, the maximum shear stress is given by:


max

University of Castilla-La Mancha


Ciudad Real Spain

x + y
=
2

Draft

+ 2xy = 4.5988 Pa

By: Eduardo W. V. Chaves (2015)

6 LINEAR ELASTICITY

515

a) The characteristic equation for the strain tensor (2D) is:


2 0.28 2.29 10 2 = 0

( 10 3 )

Then, by solving the above equation we can find the eigenvalues (principal strains) given
by:
1 = 0.346155 10 3

2 = 0.06615528 10 3

Then, the eigenvectors of the strain tensor are:


associated with 1
Eigenvecto
r

0.9802 0.1979 0
Eigenvecto r associated with 2

0.1979 0.9802 0
associated with 3
Eigenvecto
r


0
0
1

b) Given the stress tensor components, we have:


0
12.0462 1.7846

ij = 1.7846 3.5692
0 Pa
0
0
4.684

We now obtain the characteristic determinant and in turn the eigenvalues (principal
stresses) 1 = 12.40654 , 2 = 3.208843 . Additionally, the eigenvectors of the stress tensor
are:
associated with 1
Eigenvecto
r

0.9802 0.1979 0
Eigenvecto r associated with 2

0.1979 0.9802 0
associated with 3
Eigenvecto
r


0
0
1

As expected, the eigenvectors of stress and strain are the same; since we are working with
isotropic linear elastic material.
b) Alternative solution for the stress tensor components:
Knowing the strain tensor components:
0.33 0.08 0
ij = 0.08 0.05 0 10 3
0
0
0

We apply the constitutive equation: ij = Tr ( ) ij + 2 ij , where the Lam constants are


given by:
E
= 16.7307692 10 3 Pa
(1 + )(1 2 )
E
=
= 11.15384615 103 Pa
2(1 + )

and Tr ( ) = 0.27999972 10 3 0.28 10 3 , with that ij = Tr ( ) ij + 2 ij becomes:

University of Castilla-La Mancha


Ciudad Real - Spain

Draft

By: Eduardo W. V. Chaves (2015)

SOLVING PROBLEMS BY MEANS OF CONTINUUM MECHANICS

516

1 0 0
11 12 13
1 0 0
0.33 0.08 0

ij = Tr ( ) 0 1 0 + 2 12 22 23 = Tr ( ) 0 1 0 + 2 0.08 0.05 0 10 3
0 0 1
13 23 33
0 0 1
0
0
0
0
12.0461 1.784615

0 Pa
= 1.784615 3.5692

0
0
4.6846

As the material is isotropic, the tensors and share the same principal directions, then
we can use the same equation ij = Tr ( ) ij + 2 ij on the principal space, i.e.:
1 0 0
1 0 0

ij = Tr ( ) 0 1 0 + 2 0 2 0
0 0 1
0 0 3
0
0
0
0
1 0 0
0.346155
12.40752

3
0
0
3.20783
0 Pa
= Tr ( ) 0 1 0 + 2
0.0662 0 10 =
0 0 1

0
0
0
0
0
4.6846

Problem 6.29
A strain gauge (or strain gage) is a device used to obtain the strain in only one direction.
Consider a strain rosette that contains three strain gauges arranged according to a equilateral
triangle, (see Figure 6.21), and records the strain values according to the directions x1 , x1
and x1 .
x2

x1

x1

30
60
30

60

60

x1

Figure 6.21

The strain calculated according to the directions x1 , x1 and x1 are respectively:


11 = 4 10 4

= 1 10 4
11

= 4 10 4
11

+ 22 .
Obtain 22 = y , 212 = xy , 22 y . Show that 11 + 22 = 11

Hypothesis: Consider a state of plane strain.


Solution:

University of Castilla-La Mancha


Ciudad Real Spain

Draft

By: Eduardo W. V. Chaves (2015)

6 LINEAR ELASTICITY

517

Using the component transformation law for the second-order tensor and considering the
2D-plane state, we obtain that:
=
11

11 + 22 11 22
+
cos( 21 ) + 12 sin( 21 )
2
2

(6.77)

=
11

11 + 22 11 22
+
cos( 2 2 ) + 12 sin( 2 2 )
2
2

(6.78)

where 1 = 60 and 2 = 120 . Then, by combining the above equations, it is possible to


eliminate 12 , i.e.:
22 =


2
+ 11
11 = 4.66667 10 4
11
3
2

Once the value 22 = 4.66667 10 4 is obtained, we can replace it into the equation (6.77)
and we obtain:
xy = 2 12 =

1
3

(411 11 3 22 ) = 3.46410 10 4

12 = 1.73205 10 4

To obtain 22 , first we obtain the angle of rotation with respect to x1 , which is


3 = 60 +90 = 150 , thus:
22 =

11 + 22 11 22
+
cos( 2 3 ) + 12 sin( 2 3 ) = 0.33333 10 4
2
2

Checking that:
+ 22 = 0.66667 10 4
11 + 22 = 11

As expected, since the trace is an invariant.


Problem 6.30
Consider a dam section in which the displacement is known and given by:
y, v
u ( x, y ) = 4 x 2 y 2 + 2 xy + 2

v ( x, y ) = 4 y 2 x 2 + 2 xy + 5

x, u

This structure consists of a material with the following elastic properties: E = 100 MPa ,
G = 35 .7 MPa , = 0.4 . Assuming that the structure is under a small deformation regimen:
a) Find the stress field, b) For the given displacement field, show whether the equilibrium
equations are satisfied or not.
Solution:
a) We can calculate the strain tensor components as:
x =

u
= 8 x + 2 y
x

y =

v
= 8 y + 2 x
y

xy =

u v
+
=0
y x

which in matrix form is:


University of Castilla-La Mancha
Ciudad Real - Spain

Draft

By: Eduardo W. V. Chaves (2015)

SOLVING PROBLEMS BY MEANS OF CONTINUUM MECHANICS

518

8 x + 2 y
ij =
0

0
0
8 y + 2 x 0
0
0

b) For a dam, as we have seen, we can adopt the approximation of plane strain condition:

x
1
E

y = (1 + )(1 2 )
xy
0

1
0

0
0
1
2


0 .6 0 .4 0 8 x + 2 y
x
y = 357 .1428 0.4 0.6 0 8 y + 2 x MPa

0
0
0
0
.
3

xy

x
4 x 2 y

y = 357.1428 2 x 4 y MPa
xy

0

z =

E
x + y = 357.1428 [( 8 x + 2 y ) + ( 8 y + 2 x ) ]
(1 + )(1 2 )

Then, the equilibrium equations become:


x xy xz
+
+
+ b x = 0

y
z
x
xy y yz
+
+
+ b y = 0

y
z
x

yz z
xz +
+
+ b z = 0
y
z
x

4 + 0 + 0 + 0 0 (fails)

(fails)
0 4 + 0 + 0 0

z
0 + 0 +
+0=0
z

So, the given displacement field does not satisfy the equilibrium equations.

Problem 6.31
A gravity dam of triangular cross section is made up of concrete with specific weight
equal to

5
, where is the specific weight of water. The shape and dimensions of the
2

cross section are indicated in Figure 6.22, and the stress field in the dam (state of plane
strain) is given by:
11 = x 2

22 =

( x1 3 x 2 )
2

12 = x1

1
4

Consider: Poissons ratio: = ; Youngs modulus E .


a) Obtain the graphical representation of the surface force (traction vector) acting on the
face AB due to the ground reaction;
b) Obtain the principal stresses at the points A and B . Starting from the Mohrs circle in
stress, obtain the extreme values of the stresses at the respective points.
c) Obtain the strain field in the dam.

University of Castilla-La Mancha


Ciudad Real Spain

Draft

By: Eduardo W. V. Chaves (2015)

6 LINEAR ELASTICITY

519

NOTE: Although in the literature is known as the specific weight, also known as the
unit weight, in reality is the module of the body force per unit volume, i.e.

[]

r
r
r
r
N
m
= p = b = g , where b is the body force per unit mass b =
= 2 . Remember that
kg s

in the International System of Units (SI) the term specific is related to per unit mass,
which is not the case of , the correct term would be the weight density, since the term
density is related to per unit volume.
x1

O
= g

45

g -acceleration of gravity

- mass density

[ ] =

kg m
N
= 3
3
2
m s
m

x2

Figure 6.22
Solution:
a) The stress and strain fields in the dam:
x 2

ij = x1

x1
0

( x1 3 x 2 ) 0
2

0
33

11
ij = 12
0

12
22
0

0
0
0

We obtain the surface force by means on the traction vector t (n) = n . For the side AB
whose normal unit vector is n i = [0,1,0] , we obtain:

t 1 ( AB ) x 2
( AB )
t 2
= x1
t ( AB )
3
0

x1
0 0 x1

( x1 3 x 2 ) 0 1 = ( x1 3 x 2 )
2

33 0
0
0

The surface force on the base can be appreciated in Figure 6.23.


b) Note that 33 is already a principal stress. Starting from = Tr ( )1 + 2 we can
obtain 33 , i.e.:
ij = Tr ( ) ij + 2 ij 33 = Tr ( ) 33 + 2 33 33 = Tr ( )

The term Tr ( ) can be obtained by means of the double scalar product between
= Tr ( )1 + 2 and the second-order unit tensor, thus:

University of Castilla-La Mancha


Ciudad Real - Spain

Draft

By: Eduardo W. V. Chaves (2015)

SOLVING PROBLEMS BY MEANS OF CONTINUUM MECHANICS

520

: 1 = Tr ( )1 : 1 + 2 : 1
Tr ( ) =

Tr ( ) = 3Tr ( ) + 2 Tr ( ) = [3 + 2 ]Tr ( )

+ 22 + 33
Tr ( )
= 11
3 + 2
3 + 2

Then, the component 33 is defined as follows:


33 = Tr ( ) =

3 + 2

(11 + 22 + 33 )

=
(11 + 22 )
33 1
3 + 2 3 + 2

33

3 + 2

(11 + 22 )

(11 + 22 ) = (11 + 22 )
2( + )

33 =

h
B

3h
2

( AB )

3 + 2

33 =

t1

t2

(according to x1 -direction)

( AB )

(according to x 2 -direction)
x1

O
45

2
A

(n )

x2

Figure 6.23
By replacing the values of 11 and 22 , we obtain:
33 =

(11 + 22 ) = x2 + ( x1 3x2 ) = [x1 5 x2 ] = [x1 5 x2 ]


2( + )
2
2
8

where we have considered =

.
2( + )

The stress state at the point A( x1 = 0; x 2 = h) is given by:

University of Castilla-La Mancha


Ciudad Real Spain

Draft

By: Eduardo W. V. Chaves (2015)

6 LINEAR ELASTICITY

ij( A)

x 2

= x1

( x1 3x 2 )
0
= 0
2

0
( x1 5 x 2 ) 0

8
x1

521

0
3h
2
0


0 1 0

3
0 =0
2

5h
0
0
8

0 h

5
8

Note that this space is already the principal space.


Mohrs circle in stress at the point A :
S ( h)

S max = 0.4375

1.5

N (h)

0.625

The stress state at the point B ( x1 = h; x 2 = h) is given by:

ij( B )

x 2

= x1

x1

( x1 3 x 2 )
2
0


h

0
= h

( x1 5 x 2 ) 0

8
0

(h 3h)
2
0

1 1 0


0
= 1 1 0 h
1

0 0

(h 5h)
2

The principal stresses at the point B ( x1 = h; x 2 = h) are given by:


1
1
=0
1
1

(1 ) 2 1 = 0

(1 ) 2 = 1

( 1 ) = 1

1 = 2

2 = 0
S ( h)

S max = 1

0.5

N (h)

c) We can obtain the expression of the strain field by starting from the equation:
= Tr ( )1 + 2 :

University of Castilla-La Mancha


Ciudad Real - Spain

Draft

By: Eduardo W. V. Chaves (2015)

SOLVING PROBLEMS BY MEANS OF CONTINUUM MECHANICS

522

= Tr ( )1 + 2

2 = Tr ( )1

Remember that we have obtained that Tr ( ) =


=

Tr ( )1
2
2

Tr ( )
, then:
3 + 2

1
1

Tr ( )1 =

Tr ( )1
2
2
2
2 (3 + 2 )

We can also express the above equation in terms of E and :

=G =
E=

E
1
(1 + )

=
2(1 + )
2
E

1
1
(3 + 2 )

=
+
(3 + 2 ) E ( + )

=
=
2 (3 + 2 ) 2 E ( + ) E

Then:
=

Tr ( )1

2
2 (3 + 2 )

or

(1 + )
Tr ( )1
E
E

The trace of is given by:


5
Tr ( ) = 11 + 22 + 33 = ( x 2 ) + ( x1 3 x 2 ) + ( x1 5 x 2 ) = ( x1 5 x 2 )
2
8
8

1
4

With that we obtain the strain tensor components, with = , as follows:


ij =

ij =

5
4E

5
4E

x 2
x1
0
1 0 0

5
( x1 5 x 2 ) 0 1 0
( x1 3 x 2 )
0

x1
E
2
32

0 0 1

0
[
x1 5 x 2 ]
0

1
x1
0
8 ( x1 + 3 x 2 )

x1
(3x1 + 7 x 2 ) 0
8

0
0
0

University of Castilla-La Mancha


Ciudad Real Spain

5
5
ij
( x1 5 x 2 ) ij
4E
32 E

Draft

By: Eduardo W. V. Chaves (2015)

6 LINEAR ELASTICITY

523

Problem 6.32
Let us consider a two-dimensional problem such as into the domain ( ) the infinitesimal
strain field is given by:
N 1

x x

y = 0

xy N 1
y

N 2
x

0
N 1
y
N 1
x

0
N 2
y

0
N 2
y
N 2
x

N 3
x
0
N 3
y

(1)
u
0 v (1)

N 3 u ( 2 )

y v ( 2)
N 3 u (3)

x v (3)

(6.79)

where u (i ) , v (i ) are the displacement at the nodes associated with the directions x and y
respectively, and N1 = N 1 ( x, y ) , N 2 = N 2 ( x, y ) and N 3 = N 3 ( x, y ) are continuous
functions. Check whether the compatibility equation is satisfied or not. Express the
displacement field in terms of [ N ] and {u ( e ) } .
v ( 3)
u ( 3)

v ( 2)

u (2)

v (1)

u (1)

Figure 6.24: Domain .


Solution:
For two-dimensional problem the compatibility equations, (see Problem 5.11 NOTE 3),
reduce to
211 2 22
212
2
+

=0
xy
x22
x12

Engineering notation

2
2
2 x y xy
=0
+ 2
xy
x
y 2

(6.80)

According to (6.79) we can obtain:


2 x 2 N1 (1) N 2 ( 2 ) N3 (3) 3 N1 (1) 3 N 2 ( 2 ) 3 N3 (3)
=
u +
u +
u = 2 u + 2 u + 2 u

x
x
y 2 y 2 x
y x
y x
y x
2 y
x 2
2 xy
xy

2 N1 (1) N 2 ( 2 ) N3 (3) 3 N1 (1) 3 N 2 ( 2 ) 3 N 3 (3)

v +
v +
v =
v +
v +
v
2
y
y
x 2 y
yx 2
yx 2
yx

2 N1 (1) N 2 ( 2 ) N3 (3) N1 (1) N 2 ( 2 ) N 3 (3)

u +
u +
u +
v +
v +
v
xy x
x
x
y
y
y

N1 (1) N 2 ( 2 ) N3 (3)
N1 (1) N 2 ( 2 ) N3 (3)
u +
u +
u +
v +
v +
v
2
2
2
yx
yx
yx
xy 2
xy 2
xy 2

University of Castilla-La Mancha


Ciudad Real - Spain

Draft

By: Eduardo W. V. Chaves (2015)

SOLVING PROBLEMS BY MEANS OF CONTINUUM MECHANICS

524

Then, by substituting the above derivative into the equation in (6.80) we can conclude that
the compatibility equation is satisfied.
The displacement field can be obtained as follows:

u N1 (1) N 2 ( 2 ) N 3 (3)
u
=
u +
u +
u =
N1u (1) + N 2u ( 2 ) + N 3u (3) =
x x
x
x
x
x
v
v N1 (1) N 2 ( 2) N 3 (3)
v =
N1v (1) + N 2v ( 2 ) + N 3v (3) =
v +
v +
=
y =
y
y
y
y
y y

x =

Thus,

u N1
=
v 0

0
N1

N2
0

0
N2

N3
0

u (1)
(1)
v
0 u ( 2)

N 3 v ( 2 )
u (3)
( 3)
v

{u( x, y )} = [ N ]{u( e ) }

Note also that



u

x x
x
v

= 0
y =

y
xy u v
+ y
y x

u ( x, y )

= 0
y v( x, y )

x
y

N1

y 0

0
N1

N2
0

0
N2

N3
0

u (1)
(1)
v
0 u ( 2 )

N 3 v ( 2 )
u (3)
( 3)
v

or in compact form:

{ } = [ L(1) ]{u( x, y )} = [ L(1) ][ N ]{u(e) } = [ B]{u(e )}

University of Castilla-La Mancha


Ciudad Real Spain

Draft

By: Eduardo W. V. Chaves (2015)

6 LINEAR ELASTICITY

525

Problem 6.33
Consider the infinitesimal strain tensor field (2D):
12
x3 x 2
ij =
x3 x 2
12

( i, j = 1,2 )

(6.81)

where x3 = x3 ( x1 ) , i.e. x3 is a function of x1 and is a constant (Poissons ratio).


Consider the state of plane stress, (see Problem 6.24), with no body force.
a) Obtain 12 in order to achieve the equilibrium and obtain the stress field.
a
2

As boundary condition, consider that 12 ( x 2 = ) = 0 .


b) Express the infinitesimal strain tensor and the stress tensor in terms of ( P , E , I x3 ),
where P is the concentrated force at x1 = L , E is the Young modulus, I x3 is the moment
of inertia of the cross-sectional area about the x3 axis, which for a rectangular section:

I x3 = x 22 dA
A

b a
2 2

dx 2 dx3 =

b a
2 2

ba 3
. For boundary condition, consider that at x1 : P = 12 dA
12
A

and that at x1 = 0 x3 =

PL
.
EI x3

y
x
x2 , y

x2 , y

x3 , z
x1 , x

M x3
b

Cross section
Figure 6.25: Fixed-free beam (boundary conditions).
Solution:
Before applying the equilibrium equation we need to obtain the stress field. In Problem
6.24 (State of plane stress) the stress field was obtained as follows:

E x3 x 2
( 11 + 22 ) (1 ) 12
E
ij =
=

( 22 + 11 ) E
(1 2 ) (1 )12
(1 + ) 12

University of Castilla-La Mancha


Ciudad Real - Spain

Draft

12
(1 + )

(6.82)

By: Eduardo W. V. Chaves (2015)

SOLVING PROBLEMS BY MEANS OF CONTINUUM MECHANICS

526

Considering the equilibrium equation without body force, ij , j = 0 i


obtain:
ij , j = 0 i

i1,1 + i 2, 2 = 0 i

(i, j = 1,2) , we can

11 12
x + x = 0
1
2

21 + 22 = 0
x1
x 2

(6.83)

By substituting the stress components given by (6.82) we can obtain:


11 12
x + x = 0
1
2

21 + 22 = 0
x1
x 2

( E x3 x 2 )
E (12 )
+
=0

(1 + ) x 2
x1

E ( 12 ) = 0 (12 ) = 0
(1 + ) x1
x1

(6.84)

Note that 12 does not depend on x1 . From the first equilibrium equation we can obtain:
( E x3 x 2 )

x1

E ( 12 )
=0
(1 + ) x 2

x3
12
(1 + ) x 2 x3 ,1
= (1 + ) x 2
x1
x 2

By integrating in x 2 the above equation we can obtain:


12
= (1 + ) x 2 x3 ,1
x 2

integrating

12 = (1 + ) x3 ,1

x 22
+C
2

(6.85)

The constant of integration can be obtained by means of the boundary condition


a
12 ( x 2 = ) = 0 :
2
1
12 = (1 + ) x3 ,1 x 22 + C
2
(1 + ) x3 ,1 a 2
C =
2
4

a
2

x2 =

12

1
a
= (1 + ) x3 ,1 + C = 0
2
2

Then, the strain 12 , (see Eq. (6.85)) becomes:


12 = (1 + ) x3 ,1

x 22
x 2 (1 + ) x3 ,1 a 2 (1 + ) x3 ,1
+ C = (1 + ) x3 ,1 2
=
2
2
2
4
2

2 a2
x2

(6.86)

With that the infinitesimal strain field becomes:

x3 x 2

ij =
(1 + ) x ,1
a2
3
x 22

2
4

(1 + ) x3 ,1 2 a 2
x2

2
4

x3 x 2

And the stress field:

E x3 x 2
ij =
E
(1 + ) 12

University of Castilla-La Mancha


Ciudad Real Spain

E

Ex 2
12
(1 + )

=
2
E x3 ,1 2 a
0

x
2

4
2

Draft

E x3 ,1 2 a 2
x2
2
4
0

By: Eduardo W. V. Chaves (2015)

6 LINEAR ELASTICITY

527

b) By applying the condition P = 12 dA we can obtain:


A

P = 12 dA =
A

E x3 ,1 2 a 2
E x3 ,1
x2
dA =

2
4
2
A

2 a2
x2
dA

4
A

2
E x3 ,1
E x3 ,1 2
a 2 E x3 ,1
x 2 dA a dA =
Ix
A =
2 A
4 A
2 3
4
2
E x3 ,1
3ba 3 E x3 ,1
Ix
=
=
I x3 3I x3 = E x3 ,1 I x3
2 3
12
2

a2
Ix
ba
3
4

(6.87)

with that we can obtain that:


P = E x3 ,1 I x3

x3 ,1 =

P
EI x3

(6.88)

By integrating we can obtain:


x3 ,1

x3
x1

P
EI x3

integratin
g

x3 =

P
P
x1 + C =
( L x1 )
EI x3
EI x3

where we have applied the boundary condition ( x1 = 0 x3 =

(6.89)

PL
PL
) to obtain C =
.
EI x3
EI x3

With that the stress components become:


11 = E x3 x 2 =

Px 2
( x1 L)
I x3

22 = 0

12 =

P
2 I x3

a2
2

4 x2

(6.90)

or by using Engineering notation:


x = E z y =

Py
( x L)
Iz

y = 0

xy =

P
2I z

a2
2

4 y

Note that, according to the equation of 12 , given by (6.90), is independent of x1 and 12


has parabolic distribution on the cross-section area.
x2

x2 = 2 12 = 0
12 ( x2 )
3P
x = 0 =
12
12 max =
2
2A

12

x1

P = 12 dA
A

Figure 6.26: Tangential stress distribution on the cross-section area.

University of Castilla-La Mancha


Ciudad Real - Spain

Draft

By: Eduardo W. V. Chaves (2015)

SOLVING PROBLEMS BY MEANS OF CONTINUUM MECHANICS

528

The maximum tangential stress is acting at x 2 = 0 :


12 =

P
2 I x3

a2
2

4 x2

x =0

12 max =

Pa 2 3P 3P
=
=
8 I x3 2ab 2 A

where A is cross-section area.


NOTE 1: The displacement field can be obtained starting from the strain definition:

x2
Lx1 1

+ f 1 ( x 2 ) + C1

11 =

u1
Px 2
P
g in x1
=

u1 ( x 2 , x1 ) =
( L x1 ) x 2 integratin
x1
EI x3
EI x3

22 =

u 2
x2
P
P
g in x2
=
( L x1 ) x 2 integratin

u 2 ( x 2 , x1 ) =
( L x1 ) 2 + f 2 ( x1 ) + C 2
x 2
EI x3
EI x3
2

Applying the boundary condition u 2 ( x 2 = 0, x1 = 0) = u1 ( x 2 , x1 ) = 0 , we conclude that


C1 = C 2 = 0 , thus

Px 2
u1 ( x 2 , x1 ) =
EI x3

u ( x , x ) = P
2 2 1
2 EI x3

x2
Lx1 1 + f 1 ( x 2 )

x 22 ( L

(6.91)

x1 ) + f 2 ( x1 )

The tangential strain component can be obtained as follows:


2 12 =

u1 u 2
P
+
=
x 2 x1 EI x3

x2
Lx1 1

f 1 ( x 2 )
P 2 f 2 ( x1 )
+

x2 +

x 2
x1
2 EI x3

(6.92)

Note that we obtained previously that 12 is independent of x1 , so, the following must
hold:
P
EI x3

x 2 f ( x )
Lx1 1 + 2 1 = 0

2
x1

g in x1
integratin

f 2 ( x1 ) =

P
EI x3

f 2 ( x1 )
P
=
x1
EI x3

x2
Lx1 1

Lx12 x13

2 6

Then, the equation in (6.92) becomes


2 12 =

u1 u 2 f 1 ( x 2 )
P 2
a 2 P (1 + ) 2 a 2
=
x2

+
=

x 2 = (1 + ) x3 ,1 x 22

x 2 x1
x 2
2 EI x3
4
4
EI x3

f 1 ( x 2 )
P 2 P (1 + ) 2 a 2
x2

x2 =

2 EI x3
4
x 2
EI x3

f 1 ( x 2 )
P 2 P(1 + ) 2 a 2
x2

=
x2
2 EI x3
4
x 2
EI x3

By integrating the above equation in x 2 we can obtain


f1 ( x2 ) =

P x 23 P (1 + ) x 23 a 2

x2
2 EI x3 3
4
EI x3 3

Then, the displacement field (6.91) becomes:

University of Castilla-La Mancha


Ciudad Real Spain

Draft

By: Eduardo W. V. Chaves (2015)

6 LINEAR ELASTICITY

Px 2
u1 ( x 2 , x1 ) =
EI x3

u 2 ( x 2 , x1 ) = 2 EI
x3

529

x2
P (1 + ) x 23 a 2
Lx1 1 + P x 23

6 EI
3 4 x2
2
EI
x3
x3

x 22 ( L

P
x1 ) +
EI x3

(6.93)

Lx12 x13

2 6

NOTE 2: We will check the compatibility equation for two-dimensional problem, (see
Problem 5.11 NOTE 3),
2 11
x 22

Note that

2 x3
x12

2 22
x12

2 x3
2 x3
2 12
2
= 0 + x 2
2(1 + ) x 2
=0 X
x1 x 2
x12
x12

x3 ,11 = 0 , a fact already verified by the equilibrium equation.

The problem presented previously is only valid if we discard completely the dimension x3 .
The reason follows.
As we are treating the problem by the state of plane stress we do not have stress i 3 but
we have the strain 33 0 , (see Problem 6.24). Then, the strain field becomes:

x3 x 2

(1 + ) x ,1
a2
3
x 22

ij =

2
4

(1 + ) x3 ,1 2 a 2
x2

2
4

x x 2
3

x3 x 2

For the above strain field, the compatibility equations, (see Problem 5.11), are not satisfied,
i.e.:
2 33 2 22
2 23

=0 X
2
2 +
x2 x3
x32
x2
2
2 11
2 13
233 +

=0 X
2
x1x3
x32
x1
2
2
2
11 + 22 2 12 = 0 X
x22
x1x2
x12

2
2
23 + 13 12 33 = ( x2 x2 ) = 0
x2 ,1
x3 x1 x2 x3 x1x2
x1x2

23 13 12 2 11


=0 X
+
+

x1 x1 x2 x3 x2 x3

2
23 13 + 12 22 = 0 X
x2 x1 x2 x3 x1x3

where we have used x3 ,11

2 x3
x12

(it fails)

= 0 , since 12 is independent of x1 , (see Eq. (6.86)), this

implies that x3 ,1 is a constant.

University of Castilla-La Mancha


Ciudad Real - Spain

Draft

By: Eduardo W. V. Chaves (2015)

SOLVING PROBLEMS BY MEANS OF CONTINUUM MECHANICS

530

Problem 6.34
In Problem 5.15 we have shown the Stress Formulation for three-dimensional elasticity.
Obtain the equivalent formulation for two-dimensional elasticity, i.e. considering the state
of plane stress and strain.
Solution:
As seen in Problem 5.5, the governing equations, for an isotropic linear elastic material in
small deformation regime, are:
Tensorial notation

Indicial notation

The equations of motion:

The equations of motion:

r
&r& = ar (2 equations)
+ b = u

&&i = ai (2 equations)
ij , j + bi = u

The constitutive equations for stress:

The constitutive equations for stress:


ij = kk ij + 2 ij (3 equations)

( ) = Tr ( )1 + 2 (3 equations)

The kinematic equations:

(6.94)

The kinematic equations:

r
= sym u (3 equations)

ij =

1 u i u j
+
2 x j x i

(3 equations)

where we have also considered two-dimensional problem ( i, j = 1,2 ). Note that for twodimensional problem we have 8 equations and 8 unknowns namely, ui (2 unknowns), ij
(3 unknowns) and ij (3 unknowns). The kinematic equations can be replaced by the
compatibility equations (see Problem 5.11):
ij , kl + kl ,ij il , jk jk ,il = O ijkl

In the two-dimensional case the compatibility equations (see Problem 5.11 NOTE 3)
reduce to:
S 33 =

2 11
x 22

2 22
x12

2
2
2 12
2 x y xy
Engineering Notation
= 0
S z =
+

= 0 (6.95)
x1 x 2
xy
y 2
x 2

And the equations of motion for two-dimensional case become:


&&i
ij , j + bi = i1,1 + i 2, 2 + i 3,3 + bi = u
2D
&&i

i1,1 + i 2, 2 + bi = u

&&1
11,1 + 12, 2 + b1 = u

&&
21,1 + 22, 2 + b 2 = u2

(i = 1,2)

11 12
&&
x + x + b1 = u1 = a1
1
2

21 + 22 + b = u
&&2 = a2
2
x1
x2

or in engineering notation:
x xy
11 12
&&1
+
+ b x = a x

x + x + b 1 = u
y
1
x
2
Engineering Notation

21 + 22 + b = u
xy + y + b = a
&& 2
2
y
y
x1
x
x 2
y

University of Castilla-La Mancha


Ciudad Real Spain

Draft

By: Eduardo W. V. Chaves (2015)

6 LINEAR ELASTICITY

531

We take the derivative of the first equation with respect to x and the second one with
respect to y :
x xy
+
+ b x = a x

y
x

xy + y + b = a
y
y
x
y

2 x 2 xy

( a x b x )
=
2 +
xy x
x
2
2

xy y
+
=
( a y b y )
xy
2

y
y

2 xy
2 x
=
+ ( a x b x )

x
x 2
xy
2
2 y
xy
=

+ ( a y b y )
xy
y
y 2

(1)
( 2)

By adding the both equations, (1)+(2), we can obtain


2

2 xy
xy

2 y
2 x
(

b
)
+ ( a y b y )
+

x
x
y 2 y
x 2 x

(6.96)

a) The state of plane stress


The constitutive equations for the state of plane stress were obtained in Problem 6.24 and
the strain field (see equation (6.49)) is given by:
x
1
1
y = E
xy
0

1
0

x

0 y
2(1 + ) xy
0

x = E x E y

x + y
y =
E
E

2(1 + )
xy
xy =
E

By replace the above strain components into the compatibility equation (kinematic
equations), and by consider the homogeneous material, we can obtain:
2
2
2 x y xy
=0
+

xy
y 2
x 2

2
2 1
2
1
2(1 + )

xy = 0

x
y
x
y
2
2
E x E
E xy E
y E

2
2
2
1 2 x y 2 x 1 y 2(1 + ) xy

=0
E y 2
E y 2
E x 2
E x 2
E
xy

(6.97)

2
2 y
2 xy
2 x
2 x y

+
2(1 + )
=0
xy
y 2
y 2
x 2
x 2

To consider simultaneously the two equations of motion we use the equation in (6.96):
2 y

2 x

+
+

+
2(1 + )
= (1 + )
+
(
)
(
1
)
(
)

x
x
y
y
2
2
y
x
xy
y

2 xy

and by replace the above equation into the equation (6.97) we can obtain:

University of Castilla-La Mancha


Ciudad Real - Spain

Draft

By: Eduardo W. V. Chaves (2015)

SOLVING PROBLEMS BY MEANS OF CONTINUUM MECHANICS

532

2
2 y
2 xy
2 x
2 x y

2
(
1
+
)
=0
xy
y 2
y 2
x 2
x 2

2
2 y
2 x

2 x
2 x y
b

(
1
)
(
a
)

x
x
2
x
y 2
y 2
x 2
x 2
x

2 y
(1 + )
+ ( a y b y ) = 0
2
y

By simplify the above equation we can obtain:


Stress formulation 2D The state of plane stress
2
2

2 x 2 x y y

+
+
+
= (1 + ) ( a x b x ) + ( a y b y )
2
2
2
2
y
x
y
y
x

(6.98)

For the static or quasi-static case the above equation reduces to:
Stress formulation 2D The state of plane stress (static case)
2
2

2 x 2 x y y

+
+
+
= (1 + ) ( b x ) + ( b y )
2
2
2
2
y
x
y
y
x

(6.99)

a) The state of plane strain


The constitutive equations for the state of plane stress were obtained in Problem 6.24 and
the strain field (see equation (6.61)) is given by:
x
1+
y = E
xy

1
1

0
0

0 x

0 y
2 xy

(1 + )
(1 + )(1 )

x
y
x =
E
E

(1 + )
(1 + )(1 )

x +
y
y =
E
E

2(1 + )
xy
xy =
E

By replace the above strain components into the compatibility equation (kinematic
equations), and by consider the homogeneous material, we can obtain:
2
2
2 x y xy
+

=0
xy
y 2
x 2

2 (1 + )(1 )
(1 + ) 2

y + 2

x
E
E
y 2
x

(1 + )(1 )
(1 + )
x +
y

E
E

2 2(1 + )

xy = 0

xy E

(1 )

(6.100)

2 y
2 y
2 xy
2 x
2 x

+
(
1

2
=0
xy
y 2
y 2
x 2
x 2

To consider simultaneously the two equations of motion we use the equation in (6.96):

2 x
2 y

b
+

2
=
+
(

)
(
)

x
x
y
y
2
y
xy x 2 x
y

2 xy

University of Castilla-La Mancha


Ciudad Real Spain

Draft

By: Eduardo W. V. Chaves (2015)

6 LINEAR ELASTICITY

533

and by replace the above equation into the equation (6.100) we can obtain:
(1 )

2 y
2 xy
2 y
2 x
2 x

(
1
)
2

=0
xy
y 2
y 2
x 2
x 2

(1 )

2 y
2 y 2 x

2 x
2 x
(
1
)


+ ( a x b x )
2
2
2
2
2
x
y
y
x
x
x

2 y
(

b
)

+

=0
y
y
2
y

By simplify the above equation we can obtain:


Stress formulation 2D The state of plane strain
2
2

2 x 2 x y y
1

+
+
+
=
( a x b x ) + ( a y b y )
2
2
2
2
(1 ) x
y
x
y
y
x

(6.101)

For the static or quasi-static case the above equation reduces to:
Stress formulation 2D The state of plane strain (static case)
2
2

2 x 2 x y y
1

+
+
+
=
( b x ) + ( b y )
2
2
2
2
y
(1 ) x
x
y
y
x

(6.102)

NOTE 1: Recall that the body forces can be represented by means of the potential , i.e.
r
r

b = xr , since b is a conservative field. Then, we can write b x =


and b y =
.
x
y
Recall also that in Problem 5.17 we have defined the Airy stress function . If we take

into account the body forces we can write:


x =

2
y 2

y =

2
x 2

xy = yx =

2
xy

(6.103)

thus
x = +

2
y 2

y = +

2
x 2

(6.104)

Substituting the above stress components into the equation (6.99) and by consider the mass
density field homogeneous we can obtain:
2
2
b y
b

2 x 2 x y y

+
+
= (1 + ) ( b x ) + ( b y ) = (1 + ) x +
+

2
2
2
2
y
y
y
y
x
x
x

2
x 2

2 2
+ 2 + 2

y y

2 2
+ 2 + 2

y y

2 2
+ 2 + 2

x x

2 2

2
+ 2 = (1 + ) 2 + 2

y
x
x

2 2
2 2
4
4
4

2
2
(
1
)
=
+
+
+
+
+
2 + 2
x 2 y 2
y
x 2 y 2 y 4
x 4
x

2 2
4
4
4

2
[(
1
)
2
]
=
+

+
+
2 + 2
y
x 2 y 2 y 4
x 4
x

University of Castilla-La Mancha


Ciudad Real - Spain

Draft

By: Eduardo W. V. Chaves (2015)

SOLVING PROBLEMS BY MEANS OF CONTINUUM MECHANICS

534

thus:
Stress formulation 2D The state of plane stress (static case)
2 2
4
4
4

+
+
=

2
(
1
)
2 + 2
x 2 y 2 y 4
y
x 4
x

(6.105)

Now, if the stress components (6.104) into the equation (6.102) we can obtain:
2
x 2

2 2
+ 2 + 2

y y

2 2
+ 2 + 2

y y

2 2
+ 2 + 2

x x

2
b x b y
+ 2 =
+

y
x (1 ) x

2 2
4
4
4
2 2

+
+
+
+

=
2
2
x 2 y 2 (1 ) x 2 + y 2
x 4
x 2 y 2 y 4

2 2
1
4
4
4
2 + 2

+
+
=

2
2
(1 )
y
x 4
x 2 y 2 y 4
x

thus
Stress formulation 2D The state of plane strain (static case)
4
4
4
(1 2 ) 2 2
=

+
+
+

(1 ) x 2 y 2
x 4
x 2 y 2 y 4

(6.106)

Near to the Earth surface the body forces can be considered uniform (homogenous field),
hence

2 2

0 . With that the governing equations for two-dimensional case can be


x
y

represented by means of one equation and one unknown:


Stress formulation 2D (static case and homogenous body forces field)
4
4
4
+
2
+
=0
x 4
x 2y 2 y 4

4xr = 0

,iijj = 0

(i, j = 1,2)

(6.107)

NOTE 2: References
Reference
LAIER, J.E. & BARREIRO, J.C. (1983). Complementos de Resistncias dos Materiais. Publicao
073/92, So Carlos, Universidade de So Paulo, Escola de Engenharia de So Carlos.
SECHLER, E. (1952). Elasticity in Engineering. John Willey & Sons, Inc. new York.
UGURAL, A.C. & FENSTER, S.K. (1981). Advanced strength and applied elasticity. Edward Arnold,
London - U.K.
NOTE 3: Note that, by using the stress function we have reduced the original problem (8
equations and 8 unknowns, (see Eq. (6.94))) to 1 equation (6.107) and 1 unknown ( ).
Recall that the analytical solution (exact) in most practical cases is quite complex and even
impossible to be obtained. So we resort to numerical technique, which consists in: given a
problem we obtain the solution. During the era of G.B. Airy (1862) the only possible
solution was the analytical one, since the numerical techniques were scarce. Then, they used
University of Castilla-La Mancha
Ciudad Real Spain

Draft

By: Eduardo W. V. Chaves (2015)

6 LINEAR ELASTICITY

535

to address the elastic problem through inverse method (Laier&Barreiro (1983)), i.e. for a given
solution of the equation (6.107) they seek which problem represents such solution.
The stress function can be adopted by a polynomial function, (see Figure 6.27):
= K 1 + K 2 x + K 3 y + K 4 x 2 + K 5 xy + K 6 y 2 + K 7 x 3 + K 8 x 2 y + K 9 xy 2
+ K 10 y 3 + K 11 x 4 + K 12 x 3 y + K 13 x 2 y 2 + K 14 xy 3 + K 15 xy 4 + K 16 x 5
4

(6.108)

+ K 17 x y + K 18 x y + K 19 x y + K 20 xy + K 21 y + K 22 x + L

constant term

x2
x3
x4
x5 x4 y

y2

xy

x2 y

x3 y

lineal terms

xy 2
x2 y2

x3 y 2

quadratic terms

y3
xy 3

x2 y3

cubic terms

y4

quartic terms

xy 4

y5

quintuple terms

Figure 6.27: Pascal polynomial for 2D.


For example, let us assume that the Airy stress function is given by the polynomial:
= K 4 x 2 + K 5 xy + K 6 y 2

(6.109)

where K1 , K 2 , and K3 are constants. If we are not considering the body forces the stress
field (6.103) becomes:
2
= K 5
(6.110)
xy
r
Note that the stress field is homogenous, i.e. it is independent of x . For the particular case
when K 4 = K 5 = 0 we obtain the problem represented by a rod subjected to axial force F
x =

2
= 2K 6
y 2

y =

2
= 2K 4
x 2

xy =

at its ends, (see Figure 6.28):


Stress field (Rod subjected to axial force):
F

x = A = 2 K 6

y = 0

xy = 0

K6 =

F
2A

Strain field (Rod subjected to axial force):

University of Castilla-La Mancha


Ciudad Real - Spain

Draft

By: Eduardo W. V. Chaves (2015)

SOLVING PROBLEMS BY MEANS OF CONTINUUM MECHANICS

536

x 2K 6
F

x = E = E = EA

2K 6 F
x

=
=
y =
E
E
EA

xy = 0

y, v

2 D ( x, y )

A (Area)

x, u

x =

F
A

Figure 6.28: Rod subjected to axial force.


Displacement field (Rod subjected to axial force):
u
F
F
integrating in x
x + f 1 ( y ) + C1

u ( x, y ) =
= x =
x
EA
EA

v
F integrating in y
F
y + f 2 ( x) + C 2

v( x, y ) =
= y =
EA
EA
y
v u
= xy = 0
+
x y

where f1 ( y ) is a function of y , f 2 ( x) is a function of x , C1 and C 2 are constants of


integration. By means of the third equation we can obtain:
v u
+
= xy = 0
x y
F
F

y + f 2 ( x) + C 2 +
x + f1 ( y ) + C1 = 0

x EA
y EA

f ( x) f1 ( y )
2
+
=0
x
y
f ( x)
f ( y )
2
= 1
x
y

The only possible solution is when:


f 2 ( x)
= C3
x

and

f 2 ( x)
= C 3
x

where C3 is a constant. Then, the displacement field becomes:

University of Castilla-La Mancha


Ciudad Real Spain

Draft

By: Eduardo W. V. Chaves (2015)

6 LINEAR ELASTICITY

537

u ( x, y ) = EA x + C 3 y + C1

v( x, y ) = F y C x + C
3
2

EA

The constants C1 , C2 and C3 can be obtained by means of the problem boundary


condition. Let us assume that the rod has the boundary condition as indicated in Figure
6.29.
y, v

2 D ( x, y )

A (Area)

x, u

x =

F
A

Figure 6.29: Rod subjected to axial force.


According to the boundary conditions (see Figure 6.29) we can obtain:
u ( x = 0, y = 0) = 0 u ( x = 0, y = 0) = C1 = 0

v( x = 0, y = 0) = 0 u ( x = 0, y = 0) = C 2 = 0
u ( x = 0, y ) = 0 u ( x = 0, y = 0) = C = 0
3

in which the displacement field becomes:


F

u = EA x

v = F y
EA

Problem 6.35
Obtain the displacement field for a problem (without body force) described in Figure 6.30.
As boundary condition consider that at ( x = 0, y = 0, z = 0) (u = 0, v = 0, w = 0) .
Solution:
Let us assume the following Airy stress function:
= K 4 x 2 + K 5 xy + K 6 y 2

(6.111)

with that we can obtain the stresses:


x =

2
= 2 K6
y 2

University of Castilla-La Mancha


Ciudad Real - Spain

y =

2
= 2K4
x 2

Draft

xy =

2
= K5 = 0
xy

(6.112)

By: Eduardo W. V. Chaves (2015)

SOLVING PROBLEMS BY MEANS OF CONTINUUM MECHANICS

538

rigid
p

Figure 6.30
Note that K 5 = 0 since normal stress only produce normal strain, so, xy = 0 xy = 0 .
Note also that according to the Figure 6.30 we can conclude that
x =

2
= 2K6 = p
y 2

K6 =

p
2

For this problem we have that y = 0 and by considering the stress considering the state of
plane stress, (see Problem 6.24), we can obtain the stress field:

x
1
0 x
1
0 x
x

E
E
E


1
0 y =
0 0=
x
y = (1 2 ) 1
2
2

(
1
)
(
1
)

1
1
xy

0
0
0
0
0
0

xy




2
2

E
x (1 2 )
x

(1 2 ) p
E


y = x
= x = p
2

(1 )
0
xy
0


0

Note that from the above equation we can obtain the normal strain x

(1 2 )
p.
E

Let us check the strain field can be obtained as follows:


x
1

1
y = 0 = E
xy = 0
0

1
0

x y
( p ) (p)
x
1
p
1

0 y = y x = (p ) ( p) =
E
E
E

2(1 + ) xy
0
0

(1 2 )

0
0

The normal strain z can be obtained as follows:


z =

( x + y ) (1 2 ) p (1 2 )

(1 + )
=
Tr ( ) =
p=
p

=
(1 )
(1 ) E E (1 )
E
E

Taking into account the definition of the normal strain we can obtain:
University of Castilla-La Mancha
Ciudad Real Spain

Draft

By: Eduardo W. V. Chaves (2015)

6 LINEAR ELASTICITY

u p
(1 2 )
=
E
x
w (1 + )
p
z =
=
E
z

g in x
integratin

x =

g in z
integratin

539

p
(1 2 ) x + C1
E
(1 + ) p
w=
z + C2
E
u=

By applying the boundary condition at ( x = 0, y = 0, z = 0) we conclude that Ci = 0 , thus


p

2
u = E (1 ) x

v = 0

(1 + ) p
w =
z
E

Problem 6.36
Consider the Airy stress function:
= K 10 y 3

(6.113)

where K10 is a constant. Which is the problem governed by the Airy stress function
(6.113)? Obtain the stress, strain and displacement fields. Consider the state of plane stress.
Solution:
If we are not considering the body forces the stress field (6.103) becomes:
x =

2
= 6 K 10 y
y 2

y =

2
=0
x 2

xy =

2
=0
xy

(6.114)

Note that the stress field x = x ( y ) depends only on y . For a given cross section of the
rod we have:
Resultant force on the cross-section:
a
2

F = x dA =

b
2

6K

10 ydzdy

=0

a b
2 2

Bending moment on the cross-section:

M = y x dA = 6 K 10 y 2 dA = 6 K10 y 2 dA = 6 K 10 I z
A

where I z = y 2 dA is the moment of inertia of the cross-sectional area about the z axis.
A

Note that, this is the case of pure bending, (see Figure 6.31). We can also obtain that
M = 6 K10 I z

K10 =

M
.
6I z

Let us analyze the sign of M . According to our sign convention the moment is positive if
the moment vector has the same sense as the axis, e.g. the vector M z is positive if it has
the same sense as the z -axis, (see Figure 6.31). Note also that according to this sign
convention we have x < 0 for values of y > 0 , so, for a positive value of M z we have
K 10 < 0 :

University of Castilla-La Mancha


Ciudad Real - Spain

Draft

By: Eduardo W. V. Chaves (2015)

SOLVING PROBLEMS BY MEANS OF CONTINUUM MECHANICS

540

K 10 =

Mz
6I z

Stress field (Pure bending):


x = 6 K10 y =

Mz
y
Iz

y = 0

xy = 0

Strain field (pure bending):


x 6 K 10 y M z

x = E = E = EI y
z

x
6 K 10 y M z
y
=
=
y =
E
E
EI z

xy = 0

y
x
y

Mz
z
x

Mz
b

Cross section
Figure 6.31
Displacement field (Pure bending):
Mz
Mz
u
integrating in x
yx + f 1 ( y ) + C1
u ( x, y ) =
x = x = EI y
EI
z
z

v
Mz
Mz 2
g in y
y integratin
y + f 2 ( x) + C 2

v( x, y ) =
= y =
EI z
2 EI z
y
v u
= xy = 0
+
x y

Taking into account that:

University of Castilla-La Mancha


Ciudad Real Spain

Draft

By: Eduardo W. V. Chaves (2015)

6 LINEAR ELASTICITY

541

v u
+
= xy = 0
x y

Mz
M z 2

y + f 2 ( x) + C 2 +
yx + f 1 ( y ) + C1 = 0
x 2 EI z

y EI z
f ( x) M z
f ( y )
2
+
x+ 1
=0
x
EI z
y

f 2 ( x) M z
f ( y )
+
x= 1
x
EI z
y

Similarly to the previous example we conclude that:


f 2 ( x) M z

x = C3
x
EI z

and

f 1 ( y )
= C 3
y

By integrate the above equations we can obtain:


f 2 ( x) M z

x = C3
x
EI z

f 1 ( y )
= C 3
y

f1 ( y ) = C 3 y

f 2 ( x) = C 3 x +

Mz 2
x
2 EI z

with that the displacement field becomes:


Mz
Mz

u = EI yx + f 1 ( y ) + C1 = EI yx C 3 y + C1

z
z

Mz 2
Mz
M
M
z
z
v =
y 2 + f 2 ( x) + C 2 =
y 2 + C3 x +
x + C2 =
( y 2 + x 2 ) + C 3 x + C 2

2 EI z
2 EI z
2 EI z
2 EI z

where the constants C1 , C2 and C3 can be obtained by means of the problem boundary
conditions. Let us assume that the rod has the boundary condition as indicated in Figure
6.32, in which one end of the beam has a fixed support (clamped or cantileved) and the
other end is free.

y, v

Mz
u = 0
( x = 0, y = 0)
v = 0

x, u

u = 0
v
and ( x = 0, y ) = 0
( x = 0, y 0)
x
v 0

Deformation

Figure 6.32: Fixed-free beam (boundary conditions).

University of Castilla-La Mancha


Ciudad Real - Spain

Draft

By: Eduardo W. V. Chaves (2015)

SOLVING PROBLEMS BY MEANS OF CONTINUUM MECHANICS

542

With these conditions we can obtain:


u ( x = 0, y = 0) = 0 u ( x = 0, y = 0) = C1 = 0

v( x = 0, y = 0) = 0 u ( x = 0, y = 0) = C 2 = 0
v ( x, y ) =

Mz
( y 2 + x 2 ) + C 3 x + C 2
2 EI z

v( x, y ) M z
x + C3
=
EI z
x

v( x = 0, y = 0)
= 0 = C3
x

Note also that

2 v ( x, y ) M z
, the second derivative of the deflection v( x, y ) is positive.
=
EI z
x 2

Then, the displacement field becomes:


Mz

u ( x, y ) = EI xy

M
z
v( x, y ) =
( y 2 + x 2 )

2 EI z

Note that the defection of the neutral line ( y = 0 ) is given by v =

Mz 2
x .
2 EI z

Problem 6.37
Obtain the stress field for a problem (without body force) which is represented by the Airy
stress function:
= K 5 xy + K 10 y 3 + K 14 xy 3

(6.115)

As boundary condition (B.C.) consider that


a
2

at ( y = ) xy = 0
at x P = xy dA , where A = ab is the area of the rectangular cross section
A

at x = 0 M = PL (Bending moment)
Solution:
For the problem we have

= K 5 y + K14 y 3
x

= K 5 x + 3K 10 y 2 + 3K14 xy 2
y

Then,
x =

2
= 6 K 10 y + 6 K 14 xy
y 2

y =

2
=0
x 2

xy =

2
= ( K 5 + 3K 14 y 2 )
xy

(6.116)
a
2

Applying the boundary condition ( y = ) xy = 0 , we can conclude that


a
a2
xy ( y = ) = K 5 3K 14
=0
2
4

University of Castilla-La Mancha


Ciudad Real Spain

Draft

K 5 = 3K 14

a2
4

By: Eduardo W. V. Chaves (2015)

6 LINEAR ELASTICITY

543

With that the tangential stress becomes

a2

a2
3K 14 y 2 = 3K 14
xy = 3K 14
y 2

By applying the boundary condition at x P = xy dA , we can obtain:


A

a2
a2

y 2 dA = 3K 14
P = xy dA = 3K 14
4
4

A
A

a2

dA y 2 dA = 3K 14
A I z

A
A

a2

3ba 3

P = 3K 14
I z = 3K 14 (3I z I z ) = 6 K 14 I z
ab I z = 3K 14
4

12

P
K 14 =
6I z

Then, the stresses in (6.116) become


x = 6 K 10 y +

P
xy
Iz

y = 0

xy =

P
2I z

a2
2

4 y

(6.117)

The bending moment M acting at the cross-section can be obtained as follows:

P
P
x y 2 dA
xy ydA = 6 K 10 y 2 dA +
M ( x) = x ydA = 6 K 10 y +
Iz A
Iz
A
A
A 23
1
123

=I z

M ( x) = 6 K 10 I z +

=I z

P
xI z
Iz

M ( x) = 6 K 10 I z + Px

The constant K10 can be obtained by means of the B.C.: x = 0 M = PL :


M ( x = 0) = 6 K 10 I z = PL

K 10 =

PL
6I z

Then, the stresses in (6.116) become


x =

Py
( x L)
Iz

y = 0

xy =

P
2I z

a2
2

4 y

(6.118)

Note that this problem was already established in Problem 6.33.

University of Castilla-La Mancha


Ciudad Real - Spain

Draft

By: Eduardo W. V. Chaves (2015)

SOLVING PROBLEMS BY MEANS OF CONTINUUM MECHANICS

544

6.3 Introduction to Torsion


Problem 6.38
Consider the hypothesis (approximations) for the Saint-Venant torsion problem, (see
Figure 6.33):

The body is prismatic (consider as prismatic axis the x1 -axis);


The unit torsion angle (angle of twist per unit length) is constant according to the x1 axis;
The projection of transversal cross section on ( x2 x3 )-plane has rigid body motion
(rotation about the x1 -axis).

Show that this problem can be governed by the equation:


2 u1 = 0

or
or

with

u1 = u1 ( x 2 , x3 )

(6.119)

2u 2u
G 21 + 21 = 0 , where G is the shear modulus
x3
x2

u1
u1

G
x 3 + G
+ x 2 = 0
x 2 x 2
x 3 x3

(6.120)
(6.121)

x1 = x
x, x1

MT

z = x3

z = x3

P2
x1

S2

P2

y = x2

y = x2

x1

S1

r
r

P1

Figure 6.33: Torque applied to a prismatic body.


University of Castilla-La Mancha
Ciudad Real Spain

Draft

By: Eduardo W. V. Chaves (2015)

6 LINEAR ELASTICITY

545

Obs.:
With these hypotheses, in general, the transversal cross section does not remain planar after
deformation.
Considering the prismatic body and by applying the torque at the free end, the body
displaces as indicated in Figure 6.33.
Solution:

Consider the point P1 located on the fixed section S1 whose position vector is r (see
Figure 6.33). Let us consider another cross section S 2 (free to rotate and warping) which
distance from the section S1 is x1 and by projecting the point P1 on the cross section S 2
we obtain the point P2 . After the torque is applied the point P2 moves to P2 as indicated
in Figure 6.34.
x3

x2 = r cos
u2

x3 = r sin

P2

r 2 = x 22 + x32

u3

rx1

x1

P2

x2

Figure 6.34: Motion of the prismatic body cross section.


Geometrically (see Figure 6.34) we can obtain the displacements:
u2 = r x1 sin = x3 x1

u3 = r x1 cos = x2 x1

(6.122)

where x2 = r cos , x3 = r sin , and u2 stands for the displacement according x2 -direction,
and u3 is the displacement according to the x3 -direction. The displacement of the point P2
according to the x1 -direction can be any, thus we summarize:
u1 = u1 ( x 2 , x 3 )

u 2 = x 3 x1

u 3 = x 2 x1

(6.123)

The displacement u1 (warping function) is the warping of the cross section, which is
independent of x1 .

University of Castilla-La Mancha


Ciudad Real - Spain

Draft

By: Eduardo W. V. Chaves (2015)

SOLVING PROBLEMS BY MEANS OF CONTINUUM MECHANICS

546

(Kinematic equations)
The strain-displacement relationships become:
11 =

u1
=0;
x1

12

1 u1 u2

+
2 x2 x1

1 u1

x3 ;
2 x2

22 =

u 2
= 0;
x 2

23

33 =

1 u2 u3

+
2 x3 x2
1
= ( x1 + x1 ) ;
2
=0

13

u 3
= 0;
x3

1 u1 u3

+
2 x3 x1

1 u
= 1 + x2
2 x3

Thus

11 12
ij = 12 22
13 23

13

1 u
23 = 1 x3
2 x2

33
u

1 + x2
x3

u1

x3

x2
0
0

u1

x + x2
3

(6.124)

Note that the compatibility equations, (see Problem 5.11), are satisfied automatically, since
the displacement field is continuous. By means of the compatibility equations we can also
conclude that:

233 2 22
2 23
2
(a)
+

=0
S11 =
x2x3
x22
x32

233 211
213
S22 =
2
(b)
+

=0
x1x3
x12
x32

2
2
2
S = 11 + 22 2 12 = 0
(c)
33 x22
x1x2
x12

2
S = 23 + 13 12 33 = 0
12 x3 x1
x2 x3 x1x2

23 13 12 211

+
+
=0
S23 =
x1 x1
x2 x3 x2x3

S13 = 23 13 + 12 22 = 0

x2 x1 x2 x3 x1x3

(6.125)
(d)
(e)
(f)

From equations (6.125) (d) and (f) we can conclude that:


13 12
=0

x3 x2 x3

13 12
=0

+
x2 x2 x3

12 13

x x = constant
2
3

In fact, and is equal to:


1
1 u1
12 13 1 u1

+ x2 = ( ) =
x3
=

x3 x2 2 x3 x2
2
2 x2 x3

In other words, the compatibility equations are satisfied if the following is true:

University of Castilla-La Mancha


Ciudad Real Spain

Draft

By: Eduardo W. V. Chaves (2015)

6 LINEAR ELASTICITY

12 13

=
x3 x2

notation
Engineerin
g

547

xy
x3

xz
= 2
x2

(Constitutive equations)
The constitutive equations are given by:
ij =

E
E
E
kk ij +
ij =
ij
(1 + )(1 2 )
(1 + )
(1 + )

(6.126)

Note that kk = 0 (trace of is zero). Then, by replace the strain field into the above
equation we can obtain:

1 E u1
E
ij =
ij =
x3

(1 + )
2 (1 + ) x2

1 + x2
x3

u1

x3

x2

13
u

0 = G 1 x3
x2

0
u

1 + x2
x3

u1

x3

x2

0
= 12
13

12
0
0

0
0

0
0

u1

x + x2
3

u1

x
2
x

(6.127)

Let us suppose that we need to obtain the stress components in a new system x1 x 2 x 3 ,
which is formed by a rotation around the x1 -axis (see Figure 6.35). Then the
transformation matrix from x1 x 2 x 3 to x1 x 2 x 3 is given as follows:
a11
a ij = a 21
a 31

a12
a 22
a 32

a13 1 0
a 23 = 0 n 2
a 33 0 s 2

0
n 3

s 3

(6.128)

Recall that the component transformation, (see Problem 1.99), for a second-order tensor is
given by
ij = aik a jl kl

Matricial

= AA T

Voigt

{ } = [M]{ }

where
a11 2

2
a 21
2

[M] = a 31
a 21 a11
a a
31 21
a 31 a11

a12 2
a 22 2
a 32 2
a 22 a12
a 32 a 22
a 32 a12

a13 2
a 23 2
a 33 2
a13 a 23
a 33 a 23
a 33 a13

2a11 a12
2a 21 a 22
2a 31 a 32
(a11 a 22 + a12 a 21 )
(a 31 a 22 + a 32 a 21 )
(a 31 a12 + a 32 a11 )

2a12 a13
2a 22 a 23
2a 32 a 33
(a13 a 22 + a12 a 23 )
(a 33 a 22 + a 32 a 23 )
(a 33 a12 + a 32 a13 )

2a11 a13

2a 21 a 23

2a 31 a 33

(a13 a 21 + a11 a 23 )
(a 33 a 21 + a 31 a 23 )
(a 33 a11 + a 31 a13 )

Then, for this particular transformation (see Eq. (6.128)), we can obtain:
University of Castilla-La Mancha
Ciudad Real - Spain

Draft

By: Eduardo W. V. Chaves (2015)

SOLVING PROBLEMS BY MEANS OF CONTINUUM MECHANICS

548

1
0
11
0 n 2
2
22
33 0 s 22
=

0
0
12
23 0 s 2 n 2

0
0
13

0
n 32
s 32
0
s 3 n 3
0

0
0
n 2

2n 2 n 3
2s 2 s 3

0
s 2

(s 3n 2 + s 2 n 3 )
0

0 11 = 0
0 22 = 0
0 33 = 0
=

n 3 12 n 2 12
0 23 = 0

s 3 13 s 2 12

+ n 3 13

+ s 3 13
0

(6.129)

x3

x3
x3

x2

13
12

x2

= AA T
x2
x3

cross section

13

12

x2

Figure 6.35.
(Equilibrium equations)
Using the equilibrium equations without considering the body forces we can obtain:
11 12 13
+
+
=0

x2
x3
x1
12 22 23
+
+
=0

x3
x2
x1
13 23 33
x + x + x = 0
2
3
1

12 13
+
=0

x3
x2
12
= 0 12 = 12 ( x2 , x3 )

x1
13
= 0 13 = 13 ( x2 , x3 )

x1

(6.130)

Note that the stresses 12 and 13 are not function of x1 , i.e. they do not vary with x1 . By
replace the values of 12 and 13 into the first equation of the equilibrium equations we
obtain:
12 13
+
=0
x 2
x 3

G
x 2

u1

x3
x 2

University of Castilla-La Mancha


Ciudad Real Spain

+ G
x 3

Draft

u1

x + x 2
3

= 0

(6.131)

By: Eduardo W. V. Chaves (2015)

6 LINEAR ELASTICITY

549

2u
2 u1
=0
G 21 +
2

x
3
2

2u1 = 0

(6.132)

u1 = u1 ( x2 , x3 )

where

(6.133)

which is the differential equation of Saint-Venant torsion.


External Equilibrium
According to the problem statement, (see Figure 6.33), and by considering the external
force equilibrium the following must be true:
External moment (Torque):
MT =

13 x2

12 x3 )dA

(6.134)

By using the stress components 12 and 13 given by the equations in (6.127), the above
equation can also be written as follows:
MT =

x2

u
u
(13 x2 12 x3 )dA = G 1 + x2 x2 1 x3 x3 dA

x3

u x u x

u
M T = G 1 x2 + x22 1 x3 + x32 dA = G 1 2 1 3 + x22 + x32 dA
x3
x2
x3 x2

A
A
M T = GJT

(6.135)

where we have introduced the polar moment of inertia:


u x

u x
JT = 1 2 1 3 + x22 + x32 dA
x3 x2

(6.136)

The shearing forces according to the x2 -direction and x3 -direction are equal to cero, so:
u

Qx2 = 12dA = G 1 x3 dA = 0
x2

A
A

Qx3 = 13dA = G 1 + x2 dA = 0
x3

A
A

(6.137)

where we have used the stress components 12 and 13 given by the equations in (6.127).
To complete the boundary conditions for the torsion problem we introduce the boundary
condition defined by the absence of normal stress component on the external surface of
= 0 on the boundary surface. By means of Eq. (6.129) we can
the prismatic body, 13
conclude that:

11

22
33
=

n 2 12
12
23

s 2 12
13

University of Castilla-La Mancha


Ciudad Real - Spain

0
0

0
0
0
0
=

+ n 3 13 0
0
0

+ s 3 13

Draft

(6.138)

By: Eduardo W. V. Chaves (2015)

SOLVING PROBLEMS BY MEANS OF CONTINUUM MECHANICS

550

x3

x2

x3

On boundary surface - :

=0
12

13

x2

cross section

Figure 6.36:
NOTE 1: Prismatic circular cross-sectional rod
Note that, when the cross section is circular there is no warping, since u1 = 0 .
A

MT

x1 =

dx1 = d
=

x1

d
dx1

Figure 6.37: Circular cross-sectional rod subjected to torque.


Then, in this case the strain field on the cross section is given by:
11
ij = 12
13

12

13
x3 x2
0
0

23 = x3
0
0 = x3
2
2
x2
x2
33
0
0

22
23

x3

x2
0
0

0
0

The stress field on the cross section is given by:


11
ij = 12
13

12
22
23

13 0
Gx3 Gx 2
0

23 = Gx3
0
0 = G x 3
x 2
33 Gx 2
0
0

University of Castilla-La Mancha


Ciudad Real Spain

Draft

x3
0
0

x2
0
0

By: Eduardo W. V. Chaves (2015)

6 LINEAR ELASTICITY

z, x 3

551

13

max =

MT
R
JT

12

max

(r )

13

12

y, x 2
R

Figure 6.38: Tangential stresses Moment of torsion.


The moment of torsion is given by:
MT =

13 x 2

12 x3 )dA =

((Gx

2 ) x2

(Gx3 ) x3 )dA = G x 22 + x32 dA

(6.139)

= G r 2 dA = GJ T
A

where JT = r 2 dA =
A

R 4
is the polar inertia moment.
2

According to Figure 6.38 we can conclude that:


(12 ) 2 + (13 ) 2 = 2

(G) 2 ( x 22 + x 32 ) = 2

(G) 2 r 2 = 2
(r ) = Gr

According to the equation (6.139) the relationship G =

MT
holds, with which the above
JT

equation can be rewritten as follows:


(r ) = Gr

( r ) =

MT
r
JT

(6.140)

and the unit torsion angle (angle of twist per unit length) can be obtained as follows:
=

MT
GJT

(r )
Gr

M T (r )
=
= G
r
JT

(6.141)

the term GJT is the torsional rigidity (or torsional stiffness).


The maximum value of (r ) occurs in r = R :
(r = R ) = max = GR

University of Castilla-La Mancha


Ciudad Real - Spain

Draft

(r = R ) = max =

MT
R
JT

By: Eduardo W. V. Chaves (2015)

SOLVING PROBLEMS BY MEANS OF CONTINUUM MECHANICS

552

For the hollow circular section the expressions are the same in which the polar inertia

moment is given by JT = r 2 dA = ( R24 R14 ) , (see Figure 6.39).


A

z, x 3
JT =

4
( R2 R14 )
2

max

(r )
r

R1
y, x 2

R2

Figure 6.39: Tangential stresses hollow circular cross section.


NOTE 2: Prandtls Stress Function
Let us adopt the Prandtls stress function such as:
12 =

x3

13 =

x2

(6.142)

Notice that this function satisfies the equilibrium equation (6.130):


12 13
+
=0
x2
x3

=0
x2 x3 x3 x2

Recall that the governing equation for the linear elastic problem can be represented by the
6 stress components, which is known as stress formulation (see Problem 5.15 and
Problem 5.18). In addition, if we are considering a static problem in which the body force
r
does not vary with x we fall back in Beltramis equations (see Problem 5.15-NOTE 1):
ij , kk +
2xr

1
kk ,ij = 0ij
(1 + )

1
+
xr [ xr [Tr ( )]] = 0
(1 + )

Beltramis equations

(6.143)

Taking into account the stress components (see equation (6.127)):

ij = G 1 x3

x2
u

1 + x2
x3

University of Castilla-La Mancha


Ciudad Real Spain

u1

x3
x2

0
0

Draft

u1

+ x2
x3
0

0
= 12

13
0

12
0
0

13
0
0

By: Eduardo W. V. Chaves (2015)

6 LINEAR ELASTICITY

553

where the trace of stress tensor is cero kk = 0 , the Beltramis equations become:
ij ,kk = ij ,11 + ij , 22 + ij , 22 = 0ij
ij ,kk

ij ,kk

11,11 12,11 13,11 11, 22


= 12,11 22,11 23,11 + 12, 22


13,11 23,11 33,11 13, 22

12, 22
22, 22
23, 22

13, 22 11,33

23, 22 + 12,33
33, 22 13,33

12,33
22,33
23,33

13,33

23,33 = 0ij
33,33

0
(12,11 + 12, 22 + 12,33 ) (13,11 + 13, 22 + 13,33 )

0
0
= (12,11 + 12, 22 + 12,33 )
= 0ij

(13,11 + 13, 22 + 13,33 )


0
0

With that we can obtain:


12,11 + 12, 22 + 12,33 = 0

13,11 + 13, 22 + 13,33 = 0

Note that 12 = 12 ( x2 , x3 ) and 13 = 13 ( x2 , x3 ) (see equation (6.130)), with which the


above equations become:
12, 22 + 12,33 = 0

13, 22 + 13,33 = 0

2 12 2 12
=0
2 +
x32
x2
2
2
13 + 13 = 0
x 2
x32
2

2xr 12 = 0
2
xr 13 = 0

(6.144)

If we consider the stresses defined in (6.142) into the equations (6.144) we can obtain:
2xr 12 = 0
2
xr 13 = 0

2 2
=0

=
+
2xr = 0
xr
x3 x22 x32
x3 x3
(6.145)

2 2
2r
2r
2r

x x = x x = 0 x x = x x 2 + x 2 = 0
2
2
2
2
3
2

As 2xr does not vary with x2 and x3 we can conclude that:


2xr =

2
x 22

2
x 32

= F = constant

(6.146)

Then, any function which satisfies the above equation will fulfill the equilibrium and
compatibility equations.
Starting from the stress components (see equation (6.127)):
u

12 = G 1 x3
x 2

13

u
= G 1 + x 2
x3

u1 12
=
+ x3
x 2
G

u1 13
=
x2
x3
G

(6.147)

and taking the derivative 12 with respect to x3 , and taking the derivative of 13 with
respect to x2 we obtain

University of Castilla-La Mancha


Ciudad Real - Spain

Draft

By: Eduardo W. V. Chaves (2015)

SOLVING PROBLEMS BY MEANS OF CONTINUUM MECHANICS

554

u1
12
1 12
+
=
+ x3 =

x3 x2 x3 G
G x3
13
u1
1 13

=
x2 =

x2 x3 x2 G
G x2

and subtracting the two expressions we can obtain that


1 13
u1
u1 1 12

=
+
+
x3 x2 x2 x3 G x3
G x2
0=

1 12
1 13
+ 2
G x3
G x2

12 13

= 2G
x3
x2

and by replace the stress values 12 =

and 13 =
into the above equation we can
x3
x2

obtain:
12 13

= 2G
x3
x2


2 2
= 2G
+
=
+
x3 x3 x2 x2 x22 x32

and if we take into account the equation in (6.146) we can conclude that:
2 2
+
= 2G = constant = F
x22 x32

xr ( xr ) 2xr = 2G = constant

(6.148)

where 2xr stands for the Laplacian.


To complete the problem statement we have to define the boundary conditions. To the
torsion problem the boundary condition comes defined by the absence of normal traction
vector on the external surface of the prismatic body.
x3
13

n i = 1ij

dx2

ds

0
dx

n i = n 2 = 3
n ds
3 dx2

ds

dx3

dx j

12
ds

x2

kij - permutation symbol


ds - differential arc-length element

Figure 6.40:

University of Castilla-La Mancha


Ciudad Real Spain

Draft

By: Eduardo W. V. Chaves (2015)

6 LINEAR ELASTICITY

555

The traction vector in terms of the Cauchy stress tensor becomes:

t (i n)

= ij n j

t 1(n ) 11
(n )
t 2 = 12
t (n )
3 13

t 1(n ) 0

t (2n) = 12
t (n )
3 13

12
22
23

13 n 1 0

23 n 2 = 0
33 n 3 0

13 0 12 n 2 + 13n 3 0


0
0 n 2 =
= 0

0
0
0 n 3

12
0
0

with that the boundary condition is:


12n 2 + 13n 3 = 0

Taking into account the stresses given by (6.142), the above condition can be rewritten as
follows:
12n 2 + 13n 3 = 0



n2
n3 = 0
x3
x2

dx3 dx2
+
=0
x3 ds x2 ds

d
=0
ds

dx3

x3 ds x2

dx2

=0
ds

(6.149)

With that we conclude that is constant on the boundary and can be assume any value,
with which we adopt zero.
Let us consider that:
F3 ( x2 , x3 ) = 12 =

x3

F2 ( x2 , x3 ) = 13 =

x2

(6.150)

The function is a compatible field if and only if:

= F2 ( x2 , x3 )
x2
compatible iff F2 F3
=

x3 x2

= F3 ( x2 , x3 )

x3

(6.151)

If we consider the Greens theorem (see Chapter 1 of the textbook) we can established
that:
r r
r
r
F2 F3
components
r

F d = (

dS1
F) e1dS F2 dx2 + F3 dx3 =

x3 x2

With which we can conclude:



F
F


dS1 = 0
dx 2 +
dx3 =

+ F3 dx3 = 2 3 dS1
x 2 x 3
x 3
x 2
x 3
x 2
x 3 x 2
r

dx 2 +
dx3 = xr dx = d = 0
x 3
x 2

F dx

University of Castilla-La Mancha


Ciudad Real - Spain

Draft

By: Eduardo W. V. Chaves (2015)

SOLVING PROBLEMS BY MEANS OF CONTINUUM MECHANICS

556

r
dS = dSe 1

x3

x1

e 1

x2

Figure 6.41: Greens theorem.


Now let us obtain the expression for the moment of torsion. Recalling the moment of
torsion equation we can obtain:
MT =

13 x2

12 x3 )dA = ( , 2 x2 + ,3 x3 )dA = ( ,i xi )dA

(i = 2,3)

note that (xi ) ,i = ,i xi + xi ,i = ,i x i + ii( 2 D ) = ,i x i + 2 ,i x i = (xi ) ,i 2 , where we


have applied the trace of the Kronecker delta for 2D, i.e. xi ,i = ii( 2 D ) = 22 + 33 = 2 , thus
M T = ( ,i xi )dA = ((xi ) ,i 2 )dA = (xi ) ,i dA +

(2 )dA

= (xi )n i d +

(2 )dA
A

= 2 A +

(2 )dA
A

If we consider that on the boundary the value of is zero, i.e. = 0 , we obtain that:

M T = 2 dA

(6.152)

The same can be shown by means of tensorial notation (in 2D):


r
r
r
M T = ( ,i xi )dA = [( xr ) x ]dA = [ xr (x )]dA + [ ( xr x )]dA

r
= x n d + 2 dA = 2 A + 2 dA

= 2 dA
A

where we have used the property 2 A = x n d , (see Problem 1.127).

It is easy to show that there exist iso-curves of , (see Figure 6.42). Note that for the
system ( x2 x3 ) on iso-curve of , we have:
=
12

University of Castilla-La Mancha


Ciudad Real Spain

=0
x3

Draft

13 = =

x2

By: Eduardo W. V. Chaves (2015)

6 LINEAR ELASTICITY

557

That is, is equal to the slope of according to the x2 -direction, which has the same
direction as .

x3

x 2

() = 0

( 3)
( 2)

(1)
cross section
Figure 6.42: Iso-curves of .

As additional note, next we will show in a different way that the shearing force on the cross
section is cero, and that the net moment is equal twice the volume formed by ( x2 , x3 ) . We
start from the shearing force ( Qx2 ) according to direction the x2 -direction:

Qx2 = 12 dA =
dA =
x3
A
A

x2( 2 ) x3( 2 )

dx3 dx2 =
x3
x (1 ) x (1 )

x2( 2 ) x3( 2 )

1dx3 dx2
x3

x2(1) x3(1)

(6.153)

(2)
x2( 2 )
x3( 2 ) x3
(1)
Qx2 = x(1)
dx3 dx2 = ( x3( 2 ) ) ( x3(1) ) dx2 = 0
3
x3
x2(1)
x3(1) 1
x2(1)
424
3

=0
x2( 2 )

where we have applied the integration by part. Note that ( x3( 2) ) and ( x3(1) ) are values of
the membrane on the bar surface, as we have shown previously, is constant on the
boundary so, ( x3( 2) ) ( x3(1) ) = 0 . The same procedure can be used to show that Qx3 = 0 .
As we have seen previously, the net moment is obtained as follows:
MT =

13 x2



x2 +
x3 dA
12 x3 )dA =
x2
x3
A

(6.154)

Also note that:


x2( 2 ) x3( 2 )
x2( 2 )

x3 dA =
x3 dx3 dx 2 = x 3

x3
(1) (1) x 3
A
x 2 x3
x2(1)

x3( 2 )
x3(1)

x2( 2 ) x3( 2 )

( x3 )
dx3 dx 2 = dx3 dx 2

x3
x3(1)
x2(1) x3(1)

x3( 2 )

= dA

University of Castilla-La Mancha


Ciudad Real - Spain

Draft

By: Eduardo W. V. Chaves (2015)

SOLVING PROBLEMS BY MEANS OF CONTINUUM MECHANICS

558

where we have used that on any point of the boundary is cero, i.e. ( ) = 0 , so the term

1(2
x3( 2 ) ) x3( 2 ) ( x3(1) ) x3(1) = 0 . Similarly, we can obtain that
3
123
=0

=0

x
A

x 2 dA = dA . With that the

equation in (6.154) becomes M T = 2 dA , (see Eq. (6.152)).


A

NOTE 3: Prandtls membrane analogy (Soap-film analogy)


Consider a homogenous membrane fixed at its extremities, , which is subjected to a
uniform lateral pressure q , [q ] =

N
, (see Figure 6.43). Due to the pressure in the
m2

membrane a stress state S appears. Next, we will define which equation governs such
problem.
x1

see Figure 6.45


Sdx3

u1 ( x2 , x3 )

Sdx3
x2

x3
x3
Sdx2
dx3
dx2
q

Sdx2

x1
x2

Figure 6.43: Membrane under pressure.

University of Castilla-La Mancha


Ciudad Real Spain

Draft

By: Eduardo W. V. Chaves (2015)

6 LINEAR ELASTICITY

559

Note that the slope of the membrane at the point ( x2 , x3 ) is given by the derivative of the
u1
u1
and
, which are tangents to the curve at the point. We
x3
x 2
u
u
denoted by tan( 2 ) = 1 and tan( 3 ) = 1 , and if we are considering small angles the
x 2
x 3
relationships tan( ) sin( ) = hold. In the differential element dx 2 dx3 the variation of

function u1 ( x2 , x3 ) , i.e.

the tangents are indicated in Figure 6.44.

3 +

3
dx 3
x 3

3 +

( x 2 + dx 2 , x 3 + dx3 )

( x 2 , x 3 + dx3 )

2 +

3
3
dx 3 +
dx 2
x 3
x 2

2
dx 3
x 3

2 +
r

2
2
dx 2 +
dx 3
x 2
x 3

d
dx 3

2 +

( 3 )

( x 2 + dx 2 , x3 )

( x 2 , x3 )
x3

2
dx 2
x 2

( 2 )

3 +
dx 2

x2

3
dx 2
x 2

Figure 6.44: Variation of the tangents in the differential element.

If we consider that there is no distortional of the tangents the terms


2
dx 3 = 0 hold, (see Figure 6.45).
x 3

University of Castilla-La Mancha


Ciudad Real - Spain

Draft

3
dx 2 = 0 and
x 2

By: Eduardo W. V. Chaves (2015)

SOLVING PROBLEMS BY MEANS OF CONTINUUM MECHANICS

560

Sdx 2

3 +

3
dx 3
x3
Sdx3

dx3

2 +

2
dx 2
x 2

dx 2
Sdx3

3
Sdx 2

2
2
Sdx3 sin 2 +
dx2 Sdx3 2 +
dx2
x2
x2

Sdx3

2 +
2
Sdx3

2
dx2
x2

Sdx 3 sin( 2 ) Sdx 3 2

Figure 6.45:
By apply the force equilibrium condition according to x1 -direction, we can obtain:

x1

=0

u 2u

u 2u

u
u
qdx2 dx3 + Sdx3 sin 1 + 21 dx2 Sdx3 sin 1 + Sdx2 sin 1 + 21 dx3 Sdx2 sin 1 = 0

x
x
x
2
x3
2 x2

3 x3

u 2u

u 2u
u
u
qdx2 dx3 + Sdx3 1 + 21 dx2 Sdx3 1 + Sdx2 1 + 21 dx3 Sdx2 1 = 0
x2
x3

x3 x3

x2 x2
qdx2 dx3 + Sdx3
qdx2 dx3 +

u1 2u1
u
u 2u
u
+ 2 Sdx3dx2 Sdx3 1 + Sdx2 1 + 21 Sdx2 dx3 Sdx2 1 = 0
x2 x2
x2
x3 x3
x3

2u1
2u
Sdx3dx2 + 21 Sdx2 dx3 = 0
2
x2
x3

2u 2u
qdx2 dx3 + Sdx2 dx3 21 + 21 = 0
x3
x2

University of Castilla-La Mancha


Ciudad Real Spain

Draft

By: Eduardo W. V. Chaves (2015)

6 LINEAR ELASTICITY

561

with which we can conclude that the governing equation of the membrane under pressure
is given by:
2u1 2u1
q
+ 2 =
2
S
x 2
x3

xr ( xr u1 ) 2xr u1 =

q
S

(6.155)

Making an analogy between the above equation and the torsion problem equation (6.148),
we can conclude that u1 = and 2G =

q
= constant . With which we can say that the
S

moment of torsion (see equation (6.152)) is equal to the volume defined by the membrane:

M T = 2 dA = 2 u1 ( x2 , x3 ) dA = 2Vmemb
A

Problem 6.39
a) Using the Prandtls stress function, show that for an elliptical cross section which is
subjected to the torque M T the tangential stresses are:
12 =

2M T
x3
ab 3

13 =

2M T
x2
a 3b

(6.156)

b) Draw the tangential stress distribution in the cross section;


c) Obtain the function u1 ( x2 , x3 ) .
x3

13
12
x2

Figure 6.46: Elliptical cross section.


Solution:
a) The ellipse equation is given by:
x22 x32
+
1 = 0
a2 b2

Since the value of the stress function on the boundary is constant, we can assume that:
x22

= m

x32
1
2
b

(6.157)

where m is a constant to be determined. From the above equation we can obtain:


University of Castilla-La Mancha
Ciudad Real - Spain

Draft

By: Eduardo W. V. Chaves (2015)

SOLVING PROBLEMS BY MEANS OF CONTINUUM MECHANICS

562

2x

= m 22
x 2
a
2x

= m 23
x3
b

2 2m
= 2
x 22
a

2 2m
= 2
x32
b

and by replace the above equations into the equation (6.148) we obtain that:
2
x 22

= 2G = constant = F

x 32

2
2
m 2 + 2 = F
b
a
m=

a 2b 2
2( a 2 + b 2 )

2m
a

2m
b2

=F

2( a 2 + b 2 )
=F
m
2 2

a b

(6.158)

Replacing the value of m into the equation (6.157) we can obtain:


x22

= m

x32

b2

a 2 b 2 x 22 x32
F

2(a 2 + b 2 ) a 2 b 2

(6.159)

Next step: Determine F


By substituting the value of into the moment of torsion given by (6.152) we can obtain:
a 2b 2 x22 x32
x22 x32
a 2b 2

+
+

=
1 dA
1
F
F
M T = 2 dA = 2
dA

2
2 2

(a 2 + b 2 ) A a 2 b 2
b2

2( a + b ) a
A
A
1

1
1
a 2b 2
a 2b 2
1

2
2
F
F 2 I x 3 + 2 I x 2 A
x
dA
x
dA
dA
=
= 2
+

2
3
2
2
2
2
2
(a + b ) a A
b A
b

A
(a + b ) a

where:

I x 3 = x 22 dA - moment of inertia of the cross-sectional area about the x3 -axis;


A

I x 2 = x32 dA - moment of inertia cross-sectional area about x2 -axis;


A

A=

dA - cross-sectional area.
A

For an elliptical cross section it fulfils that:


I x3 =

ba 3
4

I x2 =

b 3 a
4

A = ab

Then, the expression for the moment of torsion becomes:


MT =

1 ba 3

1
1 b 3 a
a 2b 2
a 2b 2
1

=
+
ab
F
F
I
I
A
2
2
2
2 x3
2 x2
2
2
2

4
4
(a + b ) a
b
b
(a + b ) a

a 3b 3
F
2( a 2 + b 2 )

And the value of F can be determined by:


University of Castilla-La Mancha
Ciudad Real Spain

Draft

By: Eduardo W. V. Chaves (2015)

6 LINEAR ELASTICITY

F=

563

2M T (a 2 + b 2 )
a 3b 3

Finally, the stress function (6.159) becomes:

a 2 b 2 x 22 x32

+ 2 1F
2
2 2
b
2( a + b ) a

2
2

x
M x
= T 22 + 32 1
ab a
b

2M T (a 2 + b 2 )
a 2 b 2 x22 x32

a 3b 3
2(a 2 + b 2 ) a 2 b 2

The stresses defined in (6.142) can be expressed by


12 =

=
x3 x3

M T x22 x32

2M T

=
1
x3
+

2
2

b
ab 3
ab a

13 =
=
x2
x 2

(6.160)

M T x22 x32
2M T

=
x2
1
+

2
2
3

b
ab a
a b

b) By means of the above equations we can obtain that:


x3 = 0

x2 = 0

12 = 0

2M T
2M

x2 = a )
(
13 max = 2T
13 = 3 x2
a b
a b

2M T
2M T

x2 = a )
(
12 max =
x
12 =
3 3
ab
ab 2

= 0
13

whose components can be appreciated in Figure 6.47. By means of the Pythagorean


Theorem the resultant tangential stress can be obtained:
2

2M T 2M T
x
= (12 ) + (13 ) =
+ 3 x2
3 3
ab
a b
2

12 max =

2M T
ab 2

2M T
ab

x22 x32
+
b4 a4

x3

13 max =

2M T
a 2 b

x2

Figure 6.47: Tangential stress distribution in the elliptical cross section.


c) We can obtain the angle of twist per unit length by means of equation (6.158):
University of Castilla-La Mancha
Ciudad Real - Spain

Draft

By: Eduardo W. V. Chaves (2015)

SOLVING PROBLEMS BY MEANS OF CONTINUUM MECHANICS

564

F = 2G

2M T (a 2 + b 2 )


M (a 2 + b 2 )
a 3b 3
F
= T

=
=
2G
2G
a 3b 3G

Taking into account the displacement field given by (6.122), we can obtain:

M T (a 2 + b 2 )
x3 x1
u 2 = x3 x1 =

a 3b 3G

2
2
u = x x = M T (a + b ) x x
3
2
1
2 1

a 3b 3G

(6.161)

By considering the above equations and the one in (6.147) we can obtain:
u1 12
=
+ x3
G
x2
u1 13
=
x2
G
x3

u1 12 M T (a 2 + b 2 )
x3
=
+
G
x2
a 3b 3G

u1 13 M T (a 2 + b 2 )
=

x2
G
x3
a 3b 3G

(6.162)

Integrating the above equations we can obtain that:

M (a 2 + b 2 )
u1 = 12 + T 3 3
x3
a b G
G

x2

u1 =

M (a 2 + b 2 )
12
x2 + T 3 3
x3 x2 + f ( x3 )
G
a b G

M (a 2 + b 2 )
x2 x3
u1 = 13 T 3 3
a b G
G

u1 =

13
M (a 2 + b 2 )
x3 T 3 3
x3 x2 + f ( x2 )
G
a b G

By substituting the values of 12 and 13 (see equations (6.160)) into the above equations
we can obtain:
M T (a 2 + b 2 )
M x x
2M T 1
u1 =
+
x
x
x3 x2 + f ( x3 ) = T 33 32 (b 2 a 2 ) + f ( x3 )
2
3 3
3 3
G
a b G
Ga b
ab

M x x
M (a 2 + b 2 )
2M
1
u1 = 3T x2 x3 T 3 3
x3 x2 + f ( x2 ) = T 33 32 (b 2 a 2 ) + f ( x2 )
a b G
Ga b
a b G

Note that the two above equations must be the same at the same point ( x2 , x3 ) , hence
f ( x2 ) = f ( x3 ) = 0 , thus the warping function is given by:
u1 ( x2 , x3 ) =

M T (b 2 a 2 )
x2 x3
G a 3b 3

The above function in the cross-section can be appreciated in Figure 6.48.


For the particular case when a = b , we recover the expressions for the circular cross
section (see Problem 6.38 - NOTE 1). Note also that in this case there is no warping since
u1 ( x2 , x3 ) = 0 .

University of Castilla-La Mancha


Ciudad Real Spain

Draft

By: Eduardo W. V. Chaves (2015)

6 LINEAR ELASTICITY

565

x3
u1 negative (down)

u1 positive (up)

x2

Figure 6.48: Function u1 ( x2 , x3 ) .


By means of stress components given by (6.160) we can show that, in fact, the shearing
forces are equal to cero:

Qx2 = 12 dA =
A

2M T

ab

x3 dA =

2M T
x3 dA = 0
ab 3 A
123

=0

(6.163)

2M T
2M
Qx3 = 13dA =
x2 dA = 3T x2 dA = 0
3
a b
a b A
A
A
123

=0

where

x dA is the first moment of area about


3

x3 , and is equal to cero, since the reference

system is located at the geometrical center, (see Complementary Note 2 at the end of
Chapter 1).
Problem 6.40
Consider a circular cross section with radius R , and that the Prandtls stress function is
given by:

= K ( x22 + x32 R 2 )

(6.164)

Obtain the stress and displacement fields.


Solution:
By means of equation (6.148) we can obtain:
2 2
+
= 2G
x22 x32

4 K = 2G

K =

G
2

(6.165)

thus,

G 2
( x2 + x32 R 2 )
2

(6.166)

And by appling the net moment defined in (6.152) we can obtain:

University of Castilla-La Mancha


Ciudad Real - Spain

Draft

By: Eduardo W. V. Chaves (2015)

SOLVING PROBLEMS BY MEANS OF CONTINUUM MECHANICS

566

M T = 2 dA = G ( x22 + x32 R 2 ) dA = G ( x22 + x32 ) dA R 2 dA

A
A
A
A

R 4

= G (r 2 ) dA R 2 A = G JT R 2 A = G
R 2 R 2
2

(6.167)

R 4
G
2

where J T = r 2 dA =
A

R 4
2

is the polar inertia moment, and the circle area where A = R 2 .

Then
=

2M T
GR 4

The stress field, (see equations in (6.142)), becomes:


2M T

2
2
2
12 = x = 2 x ( x2 + x3 R ) = Gx3 = R 4 x3

3
3

2M T

G
=
=
( x22 + x32 R 2 ) = Gx2 =
x2
13

x2
2 x2
R 4

(6.168)

By using the stress definition (6.127) we can obtain:


u

12 = 1 x3
x2

13 = 1 + x2
x3

Gx3
u1 12
=
+ x3 =
+ x3 = 0
G
x2
G

u1 = 0
u1 13
Gx2
=
x2 =
x2 = 0
x3
G
G

As expected, since for circular cross section there is no warping. This problem was already
discussed in Problem 6.38 - NOTE 1.
Problem 6.41
Apply the torsion theory to obtain the maximum shearing stress ( max ) in a thin rectangular
section described in Figure 6.49. Express the result in terms of ( M T , t , c) . Consider that the
Prandtls stress function, (see Ugural&Fenster (1984)), is given by:
2 t 2
= K x3
2

(6.169)

x3

t
2

x2

t
2

Figure 6.49: Thin rectangular cross section.


University of Castilla-La Mancha
Ciudad Real Spain

Draft

By: Eduardo W. V. Chaves (2015)

6 LINEAR ELASTICITY

567

Solution:
By means of equation (6.148) we can obtain:
2 2
+
= 2G
x 22 x32

2 K = 2G

K = G

(6.170)

thus,
2 t 2
= G x3
2

(6.171)

And by appling the net moment defined in (6.152) we can obtain:


2

t
M T = 2 dA = 2G x32 dA
2
A
A

c t

2
2 2
2 t 2

t
2
= 2G x3 dA = 2G x3 dx3 dx2
2
2
c t
A

22

c
t
c

2
2
2
3
x3 t
t3
t3 2
= 2G x3 dx2 = 2G dx2 = 2G x2
6
3 2 t
6 c
c
c
2
2

2
2
c
2

= G

(6.172)

t 3c
3

By considering that M T = GJT , (see equation (6.135)), we can conclude that JT =


Then, G =

t 3c
.
3

3M T
.
t 3c

The stress field becomes:


2

2 t

6M
12 =
= G
x3 = 2Gx3 = 3 T x3
x3
x3
t c
2

2 t

G
x
=

=0

3
13
x2
x2
2

t
2

The maximum shearing stress occurs at x3 = , thus:


6M t
3M
t
12 ( x 3 = ) = max = m 3 T = m 2 T
2
t c 2
t c

University of Castilla-La Mancha


Ciudad Real - Spain

Draft

By: Eduardo W. V. Chaves (2015)

568

SOLVING PROBLEMS BY MEANS OF CONTINUUM MECHANICS

max =

x3
12 ( x3 )

3M T
t 2c
t
2

x2

t
2

Figure 6.50: Stress distribution.

Torsion References
LAIER, J.E.; BAREIRO, J.C., (1983). Complemento de resistncia dos materiais. Publicao 073/92
So Carlos - USP - EESC.
SECHLER, E. (1952). Elasticity in Engineering. John Willey & Sons, Inc. new York.
SOKOLNIKOFF, I.S. (1956). Mathematic theory of elasticity. New York, McGraw-Hill.
UGURAL, A.C. & FENSTER, S.K. (1981). Advanced strength and applied elasticity. Edward Arnold,
London - U.K.

University of Castilla-La Mancha


Ciudad Real Spain

Draft

By: Eduardo W. V. Chaves (2015)

6 LINEAR ELASTICITY

569

6.4 Introduction to One-Dimensional Elements (1D)


Problem 6.42
Consider the rod element which presents one dimension larger the two others. Obtain the
internal forces in the cross-sectional area of the rod. Adopt the coordinate system indicated
in Figure 6.51 and use engineering notation..
Strain
diagram

z
z

x (z )

y
y

Stress
diagram

x (z )

neutral axis

a) Rod

b) Cross-sectional
Figure 6.51: Rod.

Hypothesis:

Small deformation regime and small rotation;

Homogeneous, elastic, linear and isotropic material;

To obtain the internal forces due to the stress component x consider that any crosssectional area define by a plane remain a plane after deformation takes place.

University of Castilla-La Mancha


Ciudad Real - Spain

Draft

By: Eduardo W. V. Chaves (2015)

SOLVING PROBLEMS BY MEANS OF CONTINUUM MECHANICS

570

Solution:
The internal forces are obtained by integrate over the cross-sectional area of the rod. Then,
in a generic case, on the face of the cross-sectional area (according to the system adopted)
can appear the stresses x , xy and xz .
If we make a cut in the rod according a plane defined by , the stress state at a point is
indicated in Figure 6.52.

xz

xy

x
A - cross-sectional area

Figure 6.52: Stress field on cross-sectional area.


The next step is established how the stress field varies on the cross-sectional area.
As the material is elastic and linear, the stress varies linearly with deformation ( x = E x ).
In addition, as we are dealing with the small deformation regime the relationship x =

u
x

holds. Then, if the displacement field u ( y, z ) on the cross-sectional area define a plane so
the strain and stress do.
We can take the following possibilities:
The cross-sectional area displaces according x direction. In this case the strain is constant on
the cross sectional area, as consequence the
normal stress field on cross-sectional area is also
constant. By integrate the normal stress over the
cross-sectional area we obtain the internal force
N (the axial force) which could be positive
(tensile) or negative (compression).

u (1)
y

Figure 6.53

University of Castilla-La Mancha


Ciudad Real Spain

Draft

By: Eduardo W. V. Chaves (2015)

6 LINEAR ELASTICITY

571

Another possibility for the displacement field on the


cross-sectional area is when the cross-sectional area
rotates about y -axis. In this case the displacement
field is the one showed in Figure 6.54. The strain and
normal stress also vary according to a plane on the
cross-sectional area. Note that, if we integrate the
normal stress over the area we obtain zero as result,
i.e. the resultant force is zero, but there is moment
according to the y -direction, so, the rod is subjected
to pure bending. We denote by M y the bending
moment according to the y -direction. The

u (2)
y

displacement u ( 2) can be caused when the rod is


subjected to a deflection according to z -direction
(displacement w( x) ).

u ( 2)

Figure 6.54
z

Another possibility is when the crosssectional area rotates about the z -axis, (see
Figure 6.55). In this case the resultant force
equals zero, and there is a bending moment
according the z -direction which is denoted
by M z . The displacement u (3) can be caused
when the rod is subjected to a deflection
according to the y direction (displacement
v( x) ).

u ( 3)
y

x
u (3)

Figure 6.55

The combination of the previous cases is also possible. In general, the normal stress field
x on the cross-sectional area is illustrated in Figure 6.56.
If we consider Figure 6.57, we can also express the bending moment M y as follows:

M y = (x2 ) zdA =
A

S z

zdA = S
c
c
A

z
A

dA =

S
Iy
c

(6.173)

where I y = z 2 dA is the inertia moment of area about the y -direction. Taking into
A

account that

S (x2 )
we can also obtain:
=
c
z

(x2 ) ( z ) =

My
Iy

(6.174)

Similarly, we can obtain:


(x3) ( y ) =

University of Castilla-La Mancha


Ciudad Real - Spain

Mz
y
Iz

Draft

(6.175)

By: Eduardo W. V. Chaves (2015)

SOLVING PROBLEMS BY MEANS OF CONTINUUM MECHANICS

572

Taking into account that x = E x , the above equations can be rewritten as follows:
(x2 ) ( z ) =

My
EI y

(x3) ( y ) =

Mz
y
EI z

(6.176)

where EI is the flexural rigidity (or flexural stiffness) of the rod.

x ( y, z ) =

M
N My
+
z z y
A
Iy
Iz

x ( y, z )
x

144444444444444444424444444444444444443

(x1)

(x2 )
y

N = (x1) dA

(x3)

M y = z (x2 ) dA
A

M z = y(x3) dA
A

Mz

My
144424443

( 2)
x dA =

144424443

( 3)
x dA

=0

Figure 6.56: The axial force and bending moments.

University of Castilla-La Mancha


Ciudad Real Spain

Draft

By: Eduardo W. V. Chaves (2015)

6 LINEAR ELASTICITY

573

S
c

x (z)

neutral axis

Figure 6.57: Normal stress distribution on the cross-sectional area.


Shearing forces and moment of torsion
By integrate the tangential stresses (shearing stresses) xy and xz over the cross-sectional
area we obtain the shearing forces Q y and Q z , respectively (see Figure 6.58):

Q y = xy dA

Q z = xz dA

(6.177)

and the moment of torsion (torque) ( M T ) (see Figure 6.38):


MT =

xz y

xy z dA

(6.178)

xz ( y , z )

xy ( y, z )

xz dA

Qz

xy dA

Qy

Figure 6.58: Tangential stresses Shearing forces.

University of Castilla-La Mancha


Ciudad Real - Spain

Draft

By: Eduardo W. V. Chaves (2015)

SOLVING PROBLEMS BY MEANS OF CONTINUUM MECHANICS

574

We summarize in Figure 6.59 the internal forces ( N , Q y , Qz ) and internal moments ( M x ,


M y , M z ) that could appear on the cross section of the rod.
z, w

Mz

y, v

My
Qz

Qy
N

M x MT

x, u

Figure 6.59: Internal forces and internal moments on the cross section of the rod.

University of Castilla-La Mancha


Ciudad Real Spain

Draft

By: Eduardo W. V. Chaves (2015)

6 LINEAR ELASTICITY

575

NOTE 1: The internal forces and internal moments depend on the external actions (loads)
in which the rod is subjected, (see Figure 6.60).

z, w

1) Internal force and moment

1) Load
y, v

N
qz
m

My
Qz

x, u

2) Load
z, w

y, v

2) Internal force and moment


N
qy
m

Mz
Qy

x, u

3) Load
z, w

3) Internal force

y, v
F [N ]

x, u

4) Load
z, w

y, v

4) Internal moment
Nm
mT

m
M x MT

x, u

Figure 6.60: Some example cases of external loads.

University of Castilla-La Mancha


Ciudad Real - Spain

Draft

By: Eduardo W. V. Chaves (2015)

SOLVING PROBLEMS BY MEANS OF CONTINUUM MECHANICS

576

NOTE 2: Warping of the Cross Section


In Problem 6.38 we will show that only when we are dealing with a circular cross section
the section remains planar after the torsion is applied.
The warping of the section appears due to the non-homogeneous tangential stress field on
the cross-sectional area (see Figure 6.62(a)). In circular section there is no warping since the
tangential stress varies linearly according to the radius as show in Figure 6.62(b).

z
xz
xy

Figure 6.61: Tangential stresses (shearing stresses) Moment of torsion.


( y, z )

max

( r )
r

a) Rectangular cross section

b) Circular cross section

Figure 6.62: Distribution of the tangential stress.


NOTE 3: Deflection of the Rod
Initially, let us consider the defection only according to the z -direction (displacement
w( x) ), (see Figure 6.63). By means of Figure 6.63 we can conclude that:
u ( 2 ) = w, x z
(x2 ) ( z )

E (x2 )

(x2 ) ( z ) =

du ( 2 )
d 2w
= 2 z w, xx z
dx
dx

(6.179)

= Ew, xx z

Note that, if we compare the equations (6.180) and (6.179) they have the reversed sign
since the bending moment M z (same direction and sense as z -axis) produces the
displacement field contrary as the one presented in Figure 6.63.
Then, if we consider the defection of the rod according to the y -direction (displacement
v( x) ), (see Figure 6.64) we can obtain:

University of Castilla-La Mancha


Ciudad Real Spain

Draft

By: Eduardo W. V. Chaves (2015)

6 LINEAR ELASTICITY

u ( 3 ) = v, x y
(x3) ( y )

(x3) ( y ) =

E (x3)

577

du (3)
d 2v
= 2 y v, xx y
dx
dx

(6.180)

= Ev, xx y

y = w, x
u

( 2)

My >0

= y z = w, x z

small angles: tan

(after deformation)
n
c
w(x)

dw
w, x
dx

neutral axis

w, xx < 0

z, w

n
c

x, u

(before deformation)

Figure 6.63: Displacement on the transversal cross section due to M y .

u ( 3) =

v
y
x

b) after deformation

m
p

Mz >0

v
x

.
n

v( x)

v
x

v, xx > 0
m
y

a/2

a/2

a) before deformation

Figure 6.64: Displacement on the transversal cross section due to M z .

University of Castilla-La Mancha


Ciudad Real - Spain

Draft

By: Eduardo W. V. Chaves (2015)

578

SOLVING PROBLEMS BY MEANS OF CONTINUUM MECHANICS

If we compare the equations (6.174) and (6.179) we can conclude that:

Iy

(2)
x ( z ) = Ew, xx z
(x2 ) ( z ) =

My

w, xx =

My

(6.181)

EI y

Similarly, if we compare the equations (6.175) and (6.180) we can conclude:


Mz
y
Iz

( 3)
x ( y ) = Ev, xx y
(x3) ( y ) =

v, xx =

Mz
EI z

(6.182)

Note that, to obtain the equations (6.181) and (6.182) we have already considered the
kinematic equations and the constitutive equations. To complete the governing equations of the
IBVP we have introduce the equilibrium equations. As in the cross section of the rod we have
lost the information of the symmetry of the Cauchy stress tensor we have to apply the
Principle of Linear Momentum (equilibrium equations or Summation of forces equal zero)
and the Principle of the Angular Momentum (summation of the moments equal zero). In
another words, we have to apply the summation of forces and moments equal zero in the
differential element of rod dx .

University of Castilla-La Mancha


Ciudad Real Spain

Draft

By: Eduardo W. V. Chaves (2015)

6 LINEAR ELASTICITY

579

NOTE 4: Tangential stress on the cross-sectional area of the rod


Let us suppose that we have several layers of smooth plates as indicated in Figure 6.65 (a).
As the plates can displace freely between layers after the load is applied (see Figure 6.65 (b))
there is no tangential stress between layers. If all plates are united to form a single
monolithic piece (Figure 6.65 (c)), the displacement between layers is limited, and so the
tangential stress appears. This tangential stress is which cause the appearance of the
tangential stress on the cross section (see Figure 6.65 (d)).

a)
transversal cross section
P

b)
transversal cross section

c)

transversal cross section

d)

d
c

a
x

d
c

Figure 6.65: Beam subjected to bending.

University of Castilla-La Mancha


Ciudad Real - Spain

Draft

By: Eduardo W. V. Chaves (2015)

SOLVING PROBLEMS BY MEANS OF CONTINUUM MECHANICS

580

Problem 6.43
Obtain the governing equation for the beam whose the flexural rigidity ( EI y ) is constant.
The beam is subjected to an uniformly distributed load per unit length (q z ) , (see Figure
6.66).
z, w
y, v

Internal forces

N
qz
m

My
Qz

x, u

Load

dx

Figure 6.66: Beam subjected to an uniformly distributed load.


Solution:
Let us consider the differential beam element dx in which the internal forces are indicated
in Figure 6.67.
dx
2
y, v

z, w
My

My +

x, u

A+

Qz

My

M y
x

dx

Qz

Qz +

Qz
dx
x

dx

Figure 6.67: Differential beam element.


Applying the equilibrium of force and of moment (at point B ) in the differential beam
element we can obtain:

=0

Q z

Qz + Qz +
dx + qdx = 0
x

Q z
= q
x

M y

dx
M yB = 0 M y Q z dx + M y +
dx + qdx
=0
x
2

(6.183)
M y
x

= Qz

(6.184)

where we have considered that dxdx 0 .

University of Castilla-La Mancha


Ciudad Real Spain

Draft

By: Eduardo W. V. Chaves (2015)

6 LINEAR ELASTICITY

581

To complete the governing equation we have to introduce the kinematic and constitutive
equations. Note that the equation in (6.181) is already considering these equations, then
w, xx =

My

EI y

M y
x
2M y

= Qz =
=

x 2

M y = EI y w, xx

3w
EI y w, xx = EI y w, xxx EI y 3
x
x

(6.185)

4w
Qz
2
= 2 EI y w, xx = EI y 4 = q
x
x
x

with that we can obtain the beam differential equation:


4w

EI y

x 4

=q

(6.186)

NOTE: Tangential stress on the cross section


z

a)

b
2

xz (z )

A B
z

z
y

QzA

My +

My

Qz +

b)
B

dx

x +

x
dx
x

F2 = x ( z )dA
A

F3 =

transversal cross section

Qz
dx
x

dx
x (z )

M y

( ( z, y)dy )dx

F1 = x + x dx dA
x

neutral axis
Figure 6.68:

University of Castilla-La Mancha


Ciudad Real - Spain

Draft

By: Eduardo W. V. Chaves (2015)

SOLVING PROBLEMS BY MEANS OF CONTINUUM MECHANICS

582

Applying the force equilibrium according the x -direction (see Figure 6.68 (b)) we can
obtain:

=0

F1 F2 F3 = 0

x
dx dA x dA dx ( z, y )dy = 0
x

(6.187)

( z )
dx ( z , y )dy = x
dx dA

If we invoke the equations (6.174) and (6.184) we can obtain:


(x2 ) ( z ) =

My
Iy

(x2) ( z ) M y M y z
Q z
=
z =
= z
x
x I y
x I y
Iy

where we are considering that I y is constant along the beam. Taking into account the
above equation into the equation in (6.187) we can obtain:
( z )
dx ( z , y )dy = x
dx dA

Q z
Q
dx ( z , y )dy = z dx dA = z dx zdA
Iy

Iy

A
A
Qz
( z, y )dy =
zdA
Iy A

with that we conclude that:


Qz

( z, y)dy = I zdA =
y A

Qz z
Iy

where

z = zdA

(6.188)

For the cross section a b and if we considering as a good approximation that

ave ( z )a = ( z, y )dy

where ave (z ) is the average of the tangential stress at (z ) .


( z, y )

A
z

Then we can obtain:

University of Castilla-La Mancha


Ciudad Real Spain

Draft

By: Eduardo W. V. Chaves (2015)

6 LINEAR ELASTICITY

ave ( z ) =

Qz z
Iy

583

ave ( z ) =

Qz z
I ya

References of Mechanics of Materials


BEER, F.P.; JOHNSTON (JR.), E.R.; DEWOLF, J.T. & MAZUREK, D.F. (2012). Mechanics of
Materials. 6th Edition. McGraw-Hill, New York-USA.
Problem 6.44
Show:
a) Mohrs First Theorem
The change in slope of a defection curve between two points is equal the area diagram of
My
EI y

between these two points.

b) Mohrs Second Theorem


Let us consider a beam subjected to bending between two point A and B . Let d be the
distance between the point B and point D , where D is the point intercepted by the
tangent line at the point A and the vertical line through the point B . The Mohrs second
theorem states:
The distance d is equal to the first moment of the diagram

My
EI y

about the axis where

d is measured.

a) Mohrs first theorem


We saw at the end of Chapter 1 (Complementary Note 1 - curvature) that the following
relationships are true:
=

w, xx
d d dx
=
=
ds
dx ds [1 + ( w, x ) 2 ]

where w, x

1
[1 + ( w, x

1
2 2
) ]

w, xx

3
2 2
) ]

[1 + ( w, x

1
r

(6.189)

ds = d = B A B _ A

(6.190)

w
2w
, w, xx 2 , in which w stands for the deflection, is the curvature, and
x
x

ds is the differential arc-length element.

For small curvature it fulfils:


=

1
2w
w 2 w, xx
r
x

ds dx

tan

cos 1

d d

ds
dx

Taking into account the equation (6.181) we can conclude that:


w, xx =

My
EI y

= y =

My
EI y

and if we apply the equation in (6.190) we can obtain:

University of Castilla-La Mancha


Ciudad Real - Spain

Draft

By: Eduardo W. V. Chaves (2015)

SOLVING PROBLEMS BY MEANS OF CONTINUUM MECHANICS

584
B

ds = d = B _ A

My

EI
A

where the expression

My

EI

ds

My

EI
A

dx = d = B _ A

(6.191)

My

ds is the area of the diagram defined by

EI y

z
= B A
d

ds

B
r

wA

My

EI y

wB

A=

My

EI

My

EI y

dx

x
x

Figure 6.69:
b) Mohrs second theorem
By multiply the equation (6.189) by x we can obtain:
B

x =

d
x integratin
g xds = xd
ds
A
A

My
EI y

xdx = xd = ( x ) B ( x ) A d
A

My

dx x = xd = ( x ) B ( x ) A d
EI y
A
A

where

My

EI
A

xdx is the first moment of area of the diagram

University of Castilla-La Mancha


Ciudad Real Spain

Draft

My
EI y

By: Eduardo W. V. Chaves (2015)

6 LINEAR ELASTICITY

585

6.5 The Potential Energy for 1D Structural Element


Problem 6.45
Obtain the total internal energy for a rod element of length L in function of the forces and
moments (see Figure 6.70).
z, w

Mz

y, v

My
Qy

Qz

M x MT

x, u

Figure 6.70: Forces and moments in the rod.


Solution:
The total internal energy is given by:
U int =

1
1
ij ij dV
: dV =
2V
2V

1
(11 11 + 22 22 + 33 33 + 212 12 + 2 23 23 + 213 13 )dV
2V

1
( x x + y y + z z + xy xy + yz yz + xz xz )dV
2V

in the transversal cross section of the rod, the normal stress x can be represented by
x = (x1) + (x2) + (x3) . The total energy associated with the stress (x1) = E (x1) (see Figure
6.56) can be expressed in function of the axial force N :
2

int

(1)
1
1 x
1
=
(x1) (x1) dV =
dV =
2V
2V E
2

N2

EA
2

dAdx =

1
2

N2
dx
EA

(6.192)

Similarly, we can obtain the total energy associated with the normal stress (x2) = E (x2) in
function of the bending moment M y , (see equation (6.174)):
U int =

1
1
(x2 ) (x2 ) dV =
2V
2

Similarly, if we consider

(x3)

My
Iy

My

zdAdx =

EI y

1
2

M y2

EI
0

2
y A

z 2 dAdx =

1
2

M y2

EI

dx

(6.193)

(see equation (6.175)), we can obtain:


L

U int =

University of Castilla-La Mancha


Ciudad Real - Spain

1 M z2
dx
2 0 EI z

Draft

(6.194)

By: Eduardo W. V. Chaves (2015)

SOLVING PROBLEMS BY MEANS OF CONTINUUM MECHANICS

586

The component xz ( z ) = G xz is associated with the shearing force Q z , (see equation


(6.177)), where G is the transversal elastic modulus. In equation (6.188) we have obtained
that:
Qz

( z, y)dy = I zdA =
y A

Qz z
Iy

where

z = zdA

(6.195)

Then, the total energy associated with ( z, y ) = G is given by:


U

int

2
1
1 xz
1 1 2
1 1
dV =
=
dV =
dAdx =
2V
2V G
2 0 GA
2 0 G

L
2
1 Q z z
=
2 0 G I y

1 1 Q z z
dydz dx =

2 0 G I y

dz dx

2
L

dz dx = 1 z Q z dx

2 0 GA

(6.196)
where z is the form factor of the cross-sectional area along z -axis, which is given by:

z = A z
Iy

dz

(6.197)

Similarly, we can obtain:


U

int

2
2
L
1
1 xy
1 yQy
=
xy xy dV =
dV =
dx
2V
2V G
2 0 GA

y = A z
Iy

where

dz

(6.198)

If we consider a circular cross section, the tangential stress field in the cross section, (see
equation (6.140)) is given:
( r ) =

MT
r
J

Then, the total energy due to the tangential stress ( r ) = G ( r ) can be obtained by:
U int =

2
2
L
1
1
1 1 MT
1 M T2
1
(r ) (r )dV =
dV =
r dV =
r 2 dAdx =

2
2V
2V G
2V G J
2 0 GJ A
2

M T2
dx
GJ

(6.199)
where J = r 2 dA is the polar moment of inertia of the circular cross-sectional area. We can
A

obtain an equivalent polar moment of inertia for another shape of the cross section which
is denoted by J Eq . Thus, we can write:
L

int

1 M T2
=
dx
2 0 GJ Eq

(6.200)

Then, the total internal energy of the rod element can be obtained as follows:
U

int

1
=
2

N 2 M y2 M z2 y Q y2 z Q z2
M T2

+
+
+
+
+
EA EI y EI z
GA
GA
EJ Eq
0

dx

(6.201)

NOTE 1: The External Potential Energy


University of Castilla-La Mancha
Ciudad Real Spain

Draft

By: Eduardo W. V. Chaves (2015)

6 LINEAR ELASTICITY

587

Next we provide some equations related to the external potential energy due to external
loads.
Concentrated force: U ext = Pw p , where P - concentrated force and w p - deflection
according to P -direction.
y

neutral axis deflection

wp
x

Figure 6.71: Concentrated force.


L

Distributed load: U ext = q( x) w( x)dx , where q ( x) - distributed load, w p - deflection


0

according to q ( x) -direction.
neutral axis deflection

y
z
w( x)

q ( x)

Figure 6.72: Uniformly distributed load.


L

In the case that q ( x) is uniformly distributed load we obtain U

ext

= q w( x)dx
0

Concentrated moment load: U ext = M yA yA


M Ay
A

yA

Figure 6.73: Concentrated moment at point A .


NOTE 2: Example of application. Let us consider a beam element in which the nodal
forces (shear and moment) and nodal displacement (deflection and rotation) are indicated
in Figure 6.74.

University of Castilla-La Mancha


Ciudad Real - Spain

Draft

By: Eduardo W. V. Chaves (2015)

SOLVING PROBLEMS BY MEANS OF CONTINUUM MECHANICS

588

Let us consider that each node there is two displacements (degree-of-freedom):


deflection ( w ) and rotation ( y =

dw
d 2w
w ). We will use also the notation
w .
dx
dx 2

y1 = w1

w1

y 2 = w2

w2

w1

y1
(e)
{u } =
w2
y 2

a) Displacements
y

z
M y1

Fz1

M y2

Fz 2

Fz1
M
y1
{ f (e) } =

Fz 2
M y 2

b) Internal forces

Figure 6.74: Beam element.


Let us adopt a cubic function to approach the deflection w :
w = ax 3 + bx 2 + cx + d

(6.202)

The first derivative of w becomes:


dw
= w = 3ax 2 + 2bx + c
dx

(6.203)

Then, at the ends of the beam the following fulfills:


x = 0 ( w = w1 )

w1 = d

x = 0 ( w = w1 )

dw
= w1 = c
dx

x = L ( w = w2 )

w2 = aL3 + bL2 + cL + d

x = L ( w = w2 )

w2 = 3aL2 + 2bL + c

University of Castilla-La Mancha


Ciudad Real Spain

(6.204)

Draft

(6.205)

By: Eduardo W. V. Chaves (2015)

6 LINEAR ELASTICITY

589

This results in the following set of equations:


w1 0
w 0
1 =
w2 L3
2
w2 3L

2L
0 1 a
a

1 0 b Reverse b 1 3L2
=

c L4 0
L2 L 1 c
4


2 L 1 0 d
L
d
0
0

L2

2L

2 L3
L4

3L2
0

L2 w1

L3 w1
(6.206)
0 w2

0 w2

where the coefficients are:


a=

2
(w1 w2 ) + 12 (w2 + w1 )
3
L
L

(6.207)

b=

3
(w2 w1 ) 1 (w2 + 2w1 )
2
L
L

(6.208)

c = w1

(6.209)

d = w1

(6.210)

By substituting the values of a , b , c and d into the equation of the deflection (6.202), we
can obtain:
2
2
x 3
x 3

x3 2x 2

x3 x2
x
x
+ x + w2 2
w = w1 2 3 + 1 + w2 2 + 3 + w1 2
L
L
L
L
L
L

(6.211)
Recall that we have adopted as degree-of-freedom the rotation y which is in agreement
with the coordinate system adopted, (see Figure 6.74), and the above equation is in
function of w' (deflection derivative). But, they are related to each other by means of the
equation y = w' . With that, the equation of the deflection becomes:
2
2
x 3
x 3

x 3 2x 2

x3 x2
x
x
w = w1 2 3 + 1 + w2 2 + 3 y1 2
+ x y 2 2
L
L
L
L
L

L
L

(6.212)
and the first derivative becomes:
6x 2 6x
6x 2 6x
3x 2 4 x
3x 2 2 x
w = w1 3 2 + w2 3 + 2 y1 2
+ 1 y 2 2
L
L
L
L
L
L
L

(6.213)
The second derivative:
6x
4
12 x 6
12 x 6
6x 2
w = w1 3 2 + w2 3 + 2 y1 2 y 2 2
L
L
L
L
L
L
L
L

(6.214)
It will be useful to obtain the following integrals:
L

w( x)dx =

L
L
L2
L2
w1 + w2
y1 +
y2
2
2
12
12

University of Castilla-La Mancha


Ciudad Real - Spain

Draft

(6.215)

By: Eduardo W. V. Chaves (2015)

SOLVING PROBLEMS BY MEANS OF CONTINUUM MECHANICS

590
L

w 2 dx =

L3 2
13L 2 13L 2 L3 2
9L
11L2
13L2
w1 +
w2 +
y1 +
y2 +
w1 w2
w1 y1 +
w1 y 2 + L
35
35
105
105
35
105
210

L3
13L2
11L2
w 2 y1 +
w2 y 2
y1 y 2
210
105
70

(6.216)
L

w dx = 5L w
2

2
1

6 2 2L 2 2L 2
12
1
1
w2 +
y1 +
y2
w1 w2 w1 y1 w1 y 2 + L
5L
15
15
5L
5
5
L+

1
1
L
w2 y1 + w2 y 2 y1 y 2
5
5
15

(6.217)
L

12

w dx = L
2

w12 +

12 2 4 2
4
24
12
12
w2 + y1 + y22 3 w1 w2 2 w1 y1 2 w1 y 2 + L
3
L
L
L
L
L
L
L+

12
12
4
w2 y1 + 2 w2 y 2 + y1 y 2
2
L
L
L

(6.218)
L

x w( x)dx =
0

3L
7L
L
L
w1 +
w2
y1 +
y2
20
20
30
20

(6.219)

Let us consider a beam element with the flexural rigidity EI y constant into the beam
element in which is under the uniformly distributed load q , (see Figure 6.75).
z

Figure 6.75: Beam under uniformly distributed load.


The total potential energy is given by:
L

= U int U ext =

EI y
2

w 2 dx qw( x)dx

(6.220)

As we are considering that q is independent of x , the external potential energy becomes:


L

U ext = qw( x)dx = q w( x)dx


0

(6.221)

By substituting the equation (6.215) into the above equation we express U ext in function of
the nodal parameters w1 , w2 , y1 and y 2 , i.e.:
L
L

L
L2
L2
y1 +
y 2
U ext = q w( x) dx = q w1 + w2
2
12
12
2

University of Castilla-La Mancha


Ciudad Real Spain

Draft

(6.222)

By: Eduardo W. V. Chaves (2015)

6 LINEAR ELASTICITY

591

Considering that EI y is constant in the beam element, the internal potential energy
becomes:
U

int

EI y
2

dx

(6.223)

Using the equation (6.218) the above equation becomes:


U int =

EI y
2

w
0

dx =

EI y 12 2 12 2 4 2 4 2 24
12
3 w1 + 3 w2 + y1 + y 2 3 w1 w2 2 w1 y1 L
L
L
2 L
L
L
L
L

12
2

w1 y 2 +

12
2

w 2 y1 +

12
2

w2 y 2 +

y1 y 2
L

(6.224)
Then, the total potential energy (6.220) can be written as follows:
=

EI y 12 2 12 2 4 2
4 2
24
12
12
3 w1 + 3 w2 + y1 + y 2 3 w1 w2 2 w1 y1 2 w1 y 2 + L
L
L
2 L
L
L
L
L
L

L
L2
L2
12
12
4

L + 2 w2 y1 + 2 w2 y 2 + y1 y 2 q w1 + w2
y1 +
y 2
L
2
12
12
L
L

(6.225)
As we are looking for the stationary states the following must hold:

=0
w1

EI y 24
L
24
12
12

3 w1 3 w2 2 y1 2 y 2 q = 0
2 L
2
L
L
L

(6.226)

=0
y1

EI y 8
12
12
4
L2

=0
y1 2 w1 + 2 w2 + y 2 + q
2 L
L
12
L
L

(6.227)

=0
w2

EI y 24
L
24
12
12

3 w2 3 w1 + 2 y1 + 2 y 2 q = 0
2 L
2
L
L
L

(6.228)

EI y 8
12
12
4
L2

w
+
w
+

q
=0

1
2
y2
y1
2 L
L
12
L2
L2

(6.229)

=0
y 2

Restructuring the above set of equations in matrix form we can obtain:


12 EI
y

L3
6 EI
y

L2

12 EI y
L3

6 EI y
L2

6 EI y

12 EI y

L2
4 EI y

L3
6 EI y

L
6 EI y

L2
12 EI y

L2
2 EI y

L3
6 EI y

L2

6 EI y
qL

L2
2
w

2 EI y 1 qL2

L y1 = 12

6 EI y w2 qL

L2 y 2 2
2

qL
4 EI y
12
L

(6.230)

or:

[Ke ] {u }= {f }
(1)

University of Castilla-La Mancha


Ciudad Real - Spain

(e )

Draft

(e)

(6.231)

By: Eduardo W. V. Chaves (2015)

SOLVING PROBLEMS BY MEANS OF CONTINUUM MECHANICS

592

in which:

[Ke ]
(1)

12 EI
y

L3
6 EI
y

L
=
12 EI y
L3

6 EI y
L2

6 EI y

12 EI y

L
4 EI y

L
6 EI y

L
6 EI y

L2
12 EI y

L2
2 EI y

L3
6 EI y

L2

6 EI y

L2
2 EI y
L

6 EI y
L2

4 EI y
L

{f }
(e)

qL

2
qL2

= 12
qL
2
2
qL
12

(6.232)

where [ Ke (1) ] is the stiffness matrix for the beam element, and note that [ Ke (1) ] has no
inverse since det[ Ke (1) ] = 0 . In order to obtain the unique solution of the set of equations
(6.231) we must introduce the boundary conditions. The same equation in (6.231) can be
obtained by means of the Principle of Virtual Work.
Let us consider that the beam is fixed at one end, (see Figure 6.76).
y

deflection

w = 0

Node 1: 1
w1 = 0
x

w 0

Node 2: 2
w2 0

q
L

Figure 6.76: Beam fixed at one end under uniformly distributed load.
By applying the boundary conditions to the equation in (6.230) we can obtain:
1

0
1
0
0

w 0
1
0
y1 qL
=
L2 w2 2
2

4 EI y y 2 qL

12
L

0
0
12 EI y

0
0
6 EI y

L3
6 EI y
L2

(6.233)

Solving the above equation we can obtain:


0
w1 0

4
y1 qL
=
8 EI
y
w2
qL3
y2
6 EI y

University of Castilla-La Mancha


Ciudad Real Spain

Draft

(6.234)

By: Eduardo W. V. Chaves (2015)

6 LINEAR ELASTICITY

593

The above solutions (deflection and rotation) are the same as the exact solution. The
moment at node 1 is given by M y1 = EI y w1 . Using the equation in (6.214) we can
obtains:
6x
4
6x 2
12 x 6
12 x 6
w = w1 3 2 + w2 3 + 2 y1 2 y 2 2
L
L
L
L
L
L
L
L
6
2
w( x = 0) = w1 = w2 2 y 2
L
L
w1 =

qL4
8 EI y

3
6 qL

L2 6 EI

y

2
L

2
5 qL
w1 =
6 2 EI y

Then, the moment becomes:


2
2
5 qL
5 qL
M y1 = EI y w1 = EI y
=
6 2
6 2 EI y

And if we compare with the exact solution M yexact


=
1

(6.235)

qL2
the error is 16.6% .
2

Loading vector of beam element


Once the displacements are obtained (6.234), the internal forces can be obtained by means
of the equation (6.231), i.e.:

{f }= [Ke ] {u }
(e)

(1)

(6.236)

(e)

with which we can obtain:

{f }
(e)

12 EI
y

L3
6 EI
y

= L
12 EI y
L3

6 EI y
L2

6 EI y
2

12 EI y
3

L
4 EI y

L
6 EI y

L
6 EI y

L2
12 EI y

L2
2 EI y

L3
6 EI y

L2

6 EI y

L2 0
2 EI y 0
qL4
L

6 EI y 8 EI y
3

L2 qL

4 EI y 6 EI y
L

qL

2
5qL2

12
=
qL
2
2
qL
12

(6.237)

When we are dealing with the traditional finite element technique, the internal forces of the
beam element are given by {f (e ) }, (see Figure 6.77(a)). The error can be minimized by
dividing the beam element in more elements. From structural analysis, the exact solution is
given by Figure 6.77(c), and we verify that the reactions at the ends of the beam element
can be obtained as follows:

{R }= {f }+ {~f
(e)

University of Castilla-La Mancha


Ciudad Real - Spain

(e )

(e)

} = {f

Draft

(e)

} {f }
(e)

(6.238)

By: Eduardo W. V. Chaves (2015)

SOLVING PROBLEMS BY MEANS OF CONTINUUM MECHANICS

594

M y1 =

a) {f (e ) }

5qL2
12

M y2 =

qL
Fz 1 =
2

Fz 2 =

qL2
12

qL
2

{~ }

b) f ( e ) = {f ( e ) }

M y1 =

qL2
12

M y2 =

qL
Fz 1 =
2

Fz 2 =

qL2
12

qL
2

=
c) Exact solution

M y1 =

qL2
2

M y2 = 0

Fz 1 = qL

Fz 2 = 0

Figure 6.77: Reaction in the beam element.


~
The vector { f ( e ) } = { f

(e)

} , (see equation (6.232)), is known as loading vector. Note that


the vector given in Figure 6.77(b) represents the reactions that appear when the beam is
fixed at both ends (see Figure 6.78), (see Gere&Weaver (1965)). And these reactions are the
same as the one obtained by means of the equation (6.232).

q
Fz 1 =

qL
2

Fz 2 =

qL2
12

M y2 =

M y1 =

qL
2
qL2
12

Figure 6.78: Reaction forces of the beam with the fixed ends.

University of Castilla-La Mancha


Ciudad Real Spain

Draft

By: Eduardo W. V. Chaves (2015)

6 LINEAR ELASTICITY

595

Analytical solution using direct integration


In this sub-section we will obtain the analytical solution (exact) for the problem described
in Figure 6.76. To obtain the analytical solution we will use the direct integration, and we
start from the moment function of the beam, (see Figure 6.79):
M y ( x) = q ( L x)

My >0
z

( L x) q 2
=
( x 2 Lx + L2 )
2
2

( L x)
2

q ( L x)

0 xL

Fz 1 = qL
q

M y1 =

qL2
2

x
M y ( x) =

x
L

q 2
( x 2 Lx + L2 )
2

Figure 6.79: Beam fixed at one end under uniformly distributed load.
Recall the differential equation of the beam in function of the deflection ( w ) (see equation
(6.181)) is given by:
M y = EI y w EI y

d 2w
dx 2

EI y

d 2w q 2
=
( x 2 Lx + L2 )
2
dx 2

By means of direct integration we can obtain:


EI y

d 2w q 2
= ( x 2 Lx + L2 )
dx 2 2

integratin
g EI y

dw q 3
= ( x 3Lx 2 + 3L2 x) + C1
dx 6

The constant of integration C1 can be obtained by means of the boundary condition


dw
w( x = 0) = 0 with which we can obtain C1 = 0 . Then, the first derivative of the
dx

deflection becomes:

q
dw
w =
3L2 x 3Lx 2 + x 3
6 EI y
dx

By integrating once more we obtain the deflection of the beam w( x) :

dw
q
3L2 x 3Lx 2 + x 3
=
dx 6 EI y

integrating

w =

q
6 EI y

3L2 x 2 3Lx 3 x 4

2 3 + 4 + C2

where the constant of integration C 2 can be obtained as follows w( x = 0) = 0 C 2 = 0 .


Then, the deflection of the beam becomes:
w( x) =

qx 2
6 L2 4 Lx + x 2
24 EI y

When x = L we can obtain:

University of Castilla-La Mancha


Ciudad Real - Spain

Draft

By: Eduardo W. V. Chaves (2015)

SOLVING PROBLEMS BY MEANS OF CONTINUUM MECHANICS

596

deflection: w( x = L) =
rotation:

qx 2
qL2
qL4
6 L2 4 Lx + x 2 =
6 L2 4 L2 + L2 =
24 EI y
24 EI y
8 EI y

q
q
qL3
dw
w( x = L) =
3L2 x 3Lx 2 + x 3 =
3L3 3L3 + L3 =
dx
6 EI y
6 EI y
6 EI y

w( x = L ) =

w( x = L) =
1

qL3
6 EI y

qL4
8 EI y

Figure 6.80: Displacements at the free-end of the beam.


BEAM ELEMENT ON ELASTIC FOUNDATION
Let us consider that the beam element is supported by elastic foundation (see Figure 6.81).
z
y
q<0
x
w( x) < 0
Kf

Figure 6.81: Beam element on elastic foundation.


In this situation, the total potential energy is given by:
= U int U ext = (U beam + U spring ) U ext

(6.239)

According to the stationary principle of the total potential energy we can conclude:
d
d {u (e ) }

=0

d
d {u (e ) }

(U

beam

+ U spring U ext = 0

dU beam
d {u ( e ) }

dU spring
d {u (e ) }

dU ext
d {u ( e ) }

=0

(6.240)
or
dU beam dU spring
dU ext
+

=0
d {u( e ) } d {u( e ) } d {u( e ) }

University of Castilla-La Mancha


Ciudad Real Spain

Draft

[ke ]{u }+ dU
d {u
(1)

(e)

spring
(e)

{ }

= f (e)

(6.241)

By: Eduardo W. V. Chaves (2015)

6 LINEAR ELASTICITY

597

where [ke (1) ] is the same matrix showed in equation (6.232). Then, the remained term
dU spring

can be obtained as follows. The internal energy for the spring, (see Figure 6.82), is

d {u (e ) }

given by:
L

U spring =

2K

w 2 dx

(6.242)

F
F

stored energy

1
1
wF = wK f w
2
2

w
Kf
1

Kf

Figure 6.82: Spring element.


Considering that the spring coefficient K f constant in the beam element the equation
(6.242) becomes:
U spring =

Kf
2

w dx
2

(6.243)

Moreover, by means of equation (6.216) we can obtain:


U spring =

K f 13L 2 13L 2 L3 2
L3 2 9 L
11L2

w
w
w
w
w1 y1 + L
+
+

1
2
y
1
y
2
1
2
2 35
35
105
105
35
105
L

13L2
13L2
11L2
L3
w1 y 2
w 2 y1 +
w2 y 2
y1 y 2
210
210
105
70

(6.244)
Then
U spring
w1
U spring
y1
U spring
w2
U spring
y 2

K f 26 L

9L
11L2
13L2
+

+
y2
w
w

y1
1
2
2 35
35
105
210

(6.245)

K f 2 L3

L3
11L2
13L2

y2
w
w

y1
1
2
2 105
105
210
70

(6.246)

K f 26 L

9L
13L2
11L2
+

+
y2
w
w

2
1
y1
2 35
35
210
105

(6.247)

K f 2 L3

L3
13L2
11L2

+
+

y1
w
w

y2
1
2
2 105
210
105
70

(6.248)

University of Castilla-La Mancha


Ciudad Real - Spain

Draft

By: Eduardo W. V. Chaves (2015)

SOLVING PROBLEMS BY MEANS OF CONTINUUM MECHANICS

598

Restructuring the above set of equations in matrix form we can obtain:


11L
13
35
210

11
L
L2

dU spring
105
= K f L 210
9
13L
d {u ( e ) }

420
70

13
L
L2

140
420
dU spring
= Ke ( 2 ) {u ( e ) }
(e)
d {u }

9
70
13L
420
13
35
11L
210

13L
420 w
1
L2

140 y1
11L w2

210
L2 y 2
105

(6.249)

where

[Ke ]
( 2)

13
35

11L
= K f L 210
9

70
13L
420

11L
210
L2
105
13L
420
L2
140

9
70
13L
420
13
35
11L
210

13L
420

L2
140
11L

210
2
L
105

(6.250)

References
GERE, J.M. & WEAVER JR., W. (1965). Analysis of Framed Structures. Van Nostrand Reinhold,
U.S.
SECHLER, E.E. (1952). Elasticity in engineering. John Wiley & Sons, Inc., New York.
LAIER, J.E.; BAREIRO, J.C., (1983). Complemento de resistncia dos materiais. Publicao 073/92
So Carlos - USP - EESC.
UGURAL, A.C.; FENSTER, S.K., (1984). Advanced strength and applied elasticity - The SI version.
Elsevier Science Publishing Co. Inc., New York.

University of Castilla-La Mancha


Ciudad Real Spain

Draft

By: Eduardo W. V. Chaves (2015)

6 LINEAR ELASTICITY

599

Problem 6.46
Consider a beam of length L where the internal forces are schematically described in
Figure 6.83. Apply the Principle of Complementary Virtual Work to the beam, (see
Problem 5.20 NOTE 4).
z, w

Mz

y, v

My
Qz

Qy

M x MT

x, u

Figure 6.83: Internal forces in the beam.


Obs.: Consider the elastic problem case without body forces, and also consider that the
beam is only subjected by concentrated loads.
Solution:
In Problem 5.20 NOTE 4 we have shown that:
r
loc r loc
F
u43
142

: dV

Total external complementary virtual work


(due to concentrated forces)

V
14243

Total internal
complementary virtual work

Principle of Complementary
Virtual Work (static case
without body forces and with
concentrated forces)

(6.251)

with n = t * on S .
The total internal complementary virtual work:

Wint = : dV = ij ij dV = ( 11 11 + 22 22 + 33 33 + 2 12 12 + 2 23 23 + 213 13 )dV


V

= ( x x + y y + z z + xy xy + yz yz + xz xz )dV
V

In the previous problem we have expressed the stresses in terms of internal forces (see
Problem 6.45):
x = (x1) + (x2) + (x3) :

x(1) (x1) dV = x(1)

x( 2) (x2 ) dV =

x(3) (x3) dV

= Mz
0

(x1)
dV =
E
My
Iy

My
EI y

N N
N
dAdx = N
dx
A EA A
EA
0

zdAdx = M y
0

My

EI y2 A

z 2 dAdx = M y
0

My
EI y

dx

Mz
dx
EI z

The component xz ( z ) = G xz is related to the shear force Q z , (see equation (6.177)),


where G is the shear modulus. In equation (6.188) we have obtained:
Qz

( z, y)dy = I zdA =
y A

University of Castilla-La Mancha


Ciudad Real - Spain

Qz z
Iy
Draft

where

z = zdA

(6.252)

By: Eduardo W. V. Chaves (2015)

SOLVING PROBLEMS BY MEANS OF CONTINUUM MECHANICS

600

Then, the strain energy associated with xz ( z ) = G xz is given by:

xz
1
1
dAdx =
dV =
G
GA
G
0
0

dydz dx

2
L
L
L

Q z z
Q
1 Q z z Q z z

dz dx = Q z
=
dz
dx
Q z z z dx
=

G I y I y
G I y
GA

0
0
0

2
dV = xz

(6.253)

where z is the shape factor which is given by:

z = A z
Iy

dz

(6.254)

In the same way we can obtain (see Problem 6.45):

xy xy dV = xy

xy
G

dV = Qy
0

yQ y
GA

dx

y
y = A dA
I
A z

where

(6.255)

The internal complementary virtual work due to the torsion moment becomes:

(r ) (r )dV =

dV =

L
L
MT
M
1 M T M T
2

r
r
dV
=
M
r
dA
dx
=
M T T dx

T
2

G J J
GJ
GJ A
0
0

(6.256)
where J = r 2 dA is the polar inertia moment. When we are dealing with rectangular cross
A

section we can express the equivalent internal complementary virtual work as follows:
L

M
0

MT
dx
GJ Eq

(6.257)

where J Eq is the equivalent inertia moment.


The total internal complementary virtual work:

Wint = ( x x + y y + z z + xy xy + yz yz + xz xz )dV
V

L
My
yQy
M
MT
Q
N
= N
+My
+ M z z + Qz z z + Q y
+ MT
EA
EI y
EI z
GA
GA
GJ Eq
0

dx

And, the Principle of Complementary Virtual Work becomes:


r
loc r loc
u43
F
142

Total external complementary virtual work


(due to concentrated forces)

: dV

V
14243

Total internal
complementary virtual work

L
r
N
My
yQy
r
M
MT
Q
F loc uloc = N
+My
+ M z z + Qz z z + Q y
+ MT
EA
EI y
EI z
GA
GA
GJ Eq
0

University of Castilla-La Mancha


Ciudad Real Spain

Draft

dx

By: Eduardo W. V. Chaves (2015)

6 LINEAR ELASTICITY

601

NOTE: For the next example we apply the principle of complementary virtual work, in
which we consider the beam fixed at one end, (see Figure 6.84).
My >0
z
q

x
x

2
M y ( x) =

qx 2
2

Figure 6.84: Beam fixed at one end under uniformly distributed load.
In this situation the principle of the complementary virtual work becomes:
L
r
My
r
dx
F loc u loc = M y
EI
y
0

If we want to know the deflection of the beam at x = 0 (node 1) we apply a concentrated


virtual load F (1) = 1 at this point. The moment diagram for real and virtual states are
presented in Figure 6.85.
REAL

M y ( x) =

VIRTUAL

qx 2
2

F (1) = 1
M y ( x) = x

1
x

Figure 6.85: Beam fixed at one end under uniformly distributed load.
With that we can state that:
L
r
My
loc r loc
F u = M y
dx
EI y
0

qx 2
L
My
q
2
= My
x3dx
dx = x
dx =
2
EI
EI
EI
y
y
y 0
0
0
L

F (1)U(1)

Taking into account that F (1) = 1 and by solving the integral we can obtain:
University of Castilla-La Mancha
Ciudad Real - Spain

Draft

By: Eduardo W. V. Chaves (2015)

SOLVING PROBLEMS BY MEANS OF CONTINUUM MECHANICS

602

U(1) =

q
2 EI y

x 3 dx =

qL4
8 EI y

which value matches the result in Figure 6.80.


If we want to know the rotation of the beam at x = 0 we apply a concentrated virtual load
(moment) m (1) = 1 at this point. The moment diagram for real and virtual states are
presented in Figure 6.86.
REAL

M y ( x) =

VIRTUAL

qx 2
2

M y ( x) = 1

m (1) = 1
1

1
x

Figure 6.86: Beam fixed at one end under uniformly distributed load.
With that we can state that:
L
r
My
loc r loc
dx
F u = M y
EI y
0

qx 2
L
My
q
2
dx = 1
dx =
= My
x 2 dx
EI y
EI y
2 EI y 0
0
0
L

m (1)(1)

Taking into account that m (1) = 1 and by solving the integral we can obtain:
L

(1) =

q
qL3
x 2 dx =
2 EI y 0
6 EI y

which value matches the result in Figure 6.80.


Problem 6.47
Consider Figure 6.84 and apply the first and second Mohr Theorem (see Problem 6.44) to
obtain the deflection and rotation at the free-end of the beam (node 1 in Figure 6.84).
Solution:
The Mohrs first theorem states: The change in slope of a defection curve between two
points is equal the area diagram of

My
EI y

between these two points.

The slope at the point 2, (see Figure 6.87) is zero ( ( 2) = 0 ). Then,


(2)

( 2) _(1) =

My

EI

(1)

( 2)

dx

University of Castilla-La Mancha


Ciudad Real Spain

( 2) (1) =

(1)

Draft

My
EI y

dx

(1) =

( 2)

My

(1)

EI

dx

By: Eduardo W. V. Chaves (2015)

6 LINEAR ELASTICITY

Moment diagram

M y ( x) =

603

Moment diagram EI y
My

qx 2
2

EI y

qx 2
(parabola)
2 EI y

b=
1

2
x

qL2
2 EI y

x=

3
L (centroid)
4

A Area =

qL3
bL
=
3 6 EI y

z
( 2) _(1) = ( 2 ) (1) = (1)

deflection of the beam

d
2
1
L

Figure 6.87: Beam fixed at one end under uniformly distributed load.
According to Figure 6.87 we can conclude that:
( 2)

(1) =

My

EI

(1)

dx = A =

qL3
6 EI y

The Mohrs second theorem states:


The distance d is equal to the first moment of the diagram

My
EI y

about the axis where

d is measured.

According to Figure 6.87 we can conclude that:


d = Ax =

University of Castilla-La Mancha


Ciudad Real - Spain

qL3 3
qL4
L=
6 EI y 4
8EI y

Draft

By: Eduardo W. V. Chaves (2015)

SOLVING PROBLEMS BY MEANS OF CONTINUUM MECHANICS

604

Problem 6.48
Consider a bar of length L , which has a squared cross section of side a . The elastic
constants of the material is assumed to be known ( E and = 0.25 ).
a) In the case of Figure 6.88(a), calculate the stored energy (strain energy density) in the bar
during the deformation, and also obtain the total strain energy;
b) Determine the stored energy corresponds to the change of volume and to the change of
shape;
c) The same question described in paragraph (a) but considering the case of Figure 6.88(b).
A -Area

P
a

a
M

a)

cross section
b)

Figure 6.88
Solution:
Considering a one dimensional case:
x = E x x =

x
E

x =

with

P
A

(6.258)

We know that the strain energy per unit volume is given by:
1
2

1
2

1
2

one - dimensional
e = :
e = x x = x

x 1 P2
=
E 2 EA 2

(6.259)

Then, the total energy U is given by:

ex (volume) = L A

P2
2 EA 2

(6.260)

P2L
U =
2 EA

The strain energy density (per unit volume) can also be expressed as follows:

e =

1
1
I 2
II dev
6(3 + 2 )
2
144244
3 1424
3

vol

(6.261)

shape

Considering:
x
ij = 0
0

University of Castilla-La Mancha


Ciudad Real Spain

0 0
P
0 0 I = x =
A
0 0

Draft

(6.262)

By: Eduardo W. V. Chaves (2015)

6 LINEAR ELASTICITY

605

Calculation of II dev :
1
I2

II dev = (3 II I 2 ) = = x
3
3
3

(6.263)

Then, the strain energy density associated with the change of volume is:

evol =

1
(1 2 ) 2 (1 2 ) 2
I 2 =
I =
x
6(3 + 2 )
6E
6E

(6.264)

(1 2 ) P 2
(per unit volume)
6 E A2

(6.265)

evol =

The strain energy density associated with the change of shape is:

e shape =

1
1 2(1 + )
II dev =
II dev
2
2 E

(1 + ) x
=

E 3

e shape =

(6.266)

(1 + ) x
(1 + ) P 2
=
E
3
3E A2
2

(6.267)

Checking:
(1 2 ) P 2 (1 + ) P 2
P2
+
=
[(1 2 ) + 2(1 + )]
6 E A2
3E A2 6 EA2
P2
[1 2 + 2 + 2 ]
=
6 EA2
P2
=
= e
2
2 EA

evol + e shape =

For the case of bending, we consider the following relationships:


y =

M y
I

I=

where
y =

a4
12

12 M y
a4

y = E y y =

The strain energy density becomes:


1

1 12 M y y
E

e = y y = 4
2
2 a

University of Castilla-La Mancha


Ciudad Real - Spain

1 12 M y 12 M y 72 M 2 y 2
=
=
4
Ea 4
Ea 8
2 a

Draft

(6.268)

By: Eduardo W. V. Chaves (2015)

606

SOLVING PROBLEMS BY MEANS OF CONTINUUM MECHANICS

University of Castilla-La Mancha


Ciudad Real Spain

Draft

By: Eduardo W. V. Chaves (2015)

11 Introduction to Fluids
11.1 Solved Problems
Problem 11.1
Demonstrate whether the following statements are true or false:
a) If the velocity field is steady, then the acceleration field is also;
b) If the velocity field is homogeneous, the acceleration field is always equal to zero;
c) If the velocity field is steady and the medium is incompressible, the acceleration is always
zero.
Solution:

r r
v ( x , t) r
a) In a steady velocity field we have
= 0 whereby the acceleration field becomes:
t
r
v ( x , t )
a i = v&i = i
+ vi ,k v k = v i ,k v k
t 4
1
42
3
=0i

r r
r r v ( x )
r r r r
r r r r
a = v& =
+ xr v ( x ) v ( x ) = xr v ( x ) v ( x )
144244
3
t
Independent of time

Then, assumption (a) is TRUE.

r r

b) A homogeneous velocity field implies that v ( x , t ) = v (t ) , whereby:


r r
r r
r r
r r
r r& v ( x , t )
v ( x , t )
r
a=v =
+ x v ( x , t ) v ( x, t ) =
1424
3
t
t
=0

Then, assumption (b) is FALSE.

r r

r r

c) A steady velocity field implies that v ( x , t ) = v ( x ) and an incompressible medium means


r r
that xr v ( x , t ) = 0 , so, we can conclude that:
r r
r r r r
r r r r
r r& v ( x )
a=v =
+ xr v ( x ) v ( x ) = xr v ( x ) v ( x )
t

Then, assumption (c) is FALSE.

SOLVING PROBLEMS BY MEANS OF CONTINUUM MECHANICS

548

Problem 11.2
Show the Navier-Stokes-Duhem equations of motion:

v&i = b i p,i + ( * + * )v j , ji + *vi , jj

Navier-Stokes-Duhem

r
r
r
r equations of motion
v& = b xr p + ( * + * ) xr ( xr v ) + * xr 2 v

(11.1)

Solution:
The Navier-Stokes-Duhem equations of motion are a combination of the equations of
r
r
motion xr + b = v& ( ij , j + b i = v&i ) and the constitutive equations:
= p1 + * Tr (D)1 + 2 *D

ij = p ij + * ijD kk + 2 *Dij

(11.2)

Then, the Cauchy stress tensor divergence ( xr ) can be evaluated as follows:

ij , j = ( p ij + * ijD kk + 2 *Dij ) j = p, j ij + * ijD kk

,j

+ 2 *Dij , j

= p,i + * ijD kk , j + 2 *Dij , j = p,i + *D kk ,i + 2 *D ij , j

(11.3)

Note that, in this formulation, we are considering that the material is homogeneous, i.e.
*, j = *, j = 0 .
In addition, by considering 2D ij = vi , j + v j ,i and 2D kk = v k ,k + v k , k = 2v k ,k , we obtain:
2D ij , j = vi , jj + v j ,ij = vi , jj + v j , ji

D kk , j = vk ,kj

(11.4)

whereby the equation in (11.2) becomes:

ij , j = p,i + *D jj ,i + 2 *Dij , j = p, i + *v j , ji + * vi , jj + v j , ji

= p,i + + v j , ji + vi , jj
*

(11.5)

Then, by substituting the equation in (11.5) into the equations of motion ( ij , j + b i = v&i ),
we obtain the Navier-Stokes-Duhem equations of motion for homogeneous materials.
NOTE 1: The explicit form of the equation (11.1) is presented as follows:
( * + * )v j , ji + vi , jj + b i p,i = v&i
( * + * )(v1,1i + v2, 2i + v3,3i ) + * (vi ,11 + vi , 22 + vi ,33 ) + b i p,i = v&i

( * + * )(v1,11 + v2, 21 + v3,31 ) + * (v1,11 + v1, 22 + v1,33 ) p,1 + b1 = v&1


*
*
*
( + )(v1,12 + v2, 22 + v3,32 ) + (v2,11 + u 2, 22 + v2,33 ) p, 2 + b 2 = v&2
*
*
*
( + )(v1,13 + v2, 23 + v3,33 ) + (v3,11 + v3, 22 + v3,33 ) p,3 + b 3 = v&3

or:

University of Castilla-La Mancha


Ciudad Real Spain

Draft

By: Eduardo W. V. Chaves (2013)

11 INTRODUCTION TO FLUIDS

549

*
2v 2v 2v
v1 v2 v3
*

+ * 21 + 21 + 21 + b1 p,1 = v&1
+
+
( + )
x
x1 x1 x2 x3
x3
x2

2v
2 v2 2 v2
v1 v2 v3
*
*
+ b 2 p, 2 = v&2

+ * 22 +
+
+
+
( + )
x
x2 x1 x2 x3
x22
x32
1

2
2
2
( * + * ) v1 + v2 + v3 + * v3 + v3 + v3 + b p = v&
3
,3
3
x 2

x3 x1 x2 x3
x32
x22
1

NOTE 2: We have proven in Problem 1.105 (Chapter 1) that the following is true:
r
r r
r
r
( a) = ( a) 2 a
indicial

ilq qjk a k , jl = a j , ji a i , jj
Then, we can obtain

r
r
r r
r
2r
xr ( xr v ) xr v = xr ( xr v ) xr ( xr v )

indicial

vi , jj = v j , ji ilq qjk vk , jl

with which the equation (11.1) can also be written as follows:


(* + * )v j , ji + *vi , jj + bi p,i = v&i
(* + * )v j , ji + * (v j , ji ilq qjk vk , jl ) + bi p, i = v&i
(* + 2 * )v j , ji *ilq qjk vk , jl + bi p, i = v&i

and the equivalent in tensorial notation:

r
r
r
r
(* + * )[ xr ( xr v )] + * [ xr ( xr v )] + b xr p = v&
r
r
r
r r
r
r
(* + * )[ xr ( xr v )] + * xr ( xr v ) xr ( xr v ) + b xr p = v&
r
r
r
r
r
r
(* + 2 * )[ xr ( xr v )] * xr ( xr v ) + b xr p = v&

In the Cartesian System we have:


r
v = vi e i = v1e 1 + v2 e 2 + v3e 3

r r
v
v
v
r
r
v
v
v
( v ) rot (v ) = (rot (v ) )i e i = 3 2 e 1 + 1 3 e 2 + 2 1 e 3
x2 x3
x3 x1
x1 x2
142
142
142
4 r43
4
4 r43
4
4 r43
4
=(rot (u ) )1

r
r r (rot (vr ) )3 (rot(vr ) )2

( v ) =
x2
x3

=(rot (u) )2
=(rot (u) )3
r
r
r
r

(rot (v ) )1 (rot (v ) )3
(rot (v ) )2 (rot (v ) )1
e 1 +
e 2 +
e 3

x1
x1
x2

x3

r
r
(rot (v ) )3 (rot (v ) )2 v2 v1 v1 v3

x2 x1 x2 x3 x3 x1

x
x
2
3


r
r

r
r r
(rot (v ) )1 (rot (v ) )3 v3 v2 v2 v1

( v ) i =
=
x1
x3
x3 x2 x3 x1 x1 x2
r
r
(rot (v ) )

(rot (v ) )1 v
2
v v3 v2


1 3

x x
x1
x2
1 3 x1 x2 x2 x3

NOTE 3: If we are dealing with heterogeneous material, we have:


ij = p ij + * ijD kk + 2 *D ij
ij , j = ( p ij + * ijD kk + 2 *D ij ) , j = p, j ij + (*D kk ), j ij + 2( *D ij ), j
ij , j = p,i + (*D kk ) ,i + 2( *Dij ), j

Taking into account that 2D ij = vi , j + v j ,i and D kk = vk ,k , the above equation becomes:

ij , j = p,i + (*D kk ),i + 2( *D ij ) , j = p,i + (*vk ,k ),i + * (vi , j + v j ,i ) , j

whereby
University of Castilla-La Mancha
Ciudad Real Spain

Draft

By: Eduardo W. V. Chaves (2013)

SOLVING PROBLEMS BY MEANS OF CONTINUUM MECHANICS

550

ij , j + bi = v&i

p, i + (*vk , k ), i + * (vi , j + v j ,i ) , j + bi = v&i

(11.6)

Note that

r
r
vk ,k = Tr (v ) = ( v )
v
v
v v
Dv&i vi vi
v j = i + i v1 + i v2 + i v3 , and its explicit components are:
v&i =
+
=
x3
x2
t x1
Dt
t x j
v1 v1

v
v
+
v1 + 1 v2 + 1 v3

x2
x3
t x1
v
v
v
v
ai = v&i = 2 + 2 v1 + 2 v2 + 2 v3
x2
x3
t x1
v3 v3

v
v
+
v1 + 3 v2 + 3 v3

x2
x3
t x1

* (vi , j + v j ,i ) , j =
* (vi , j + v j ,i )
x j

* (vi ,1 + v1,i ) +
* (vi , 2 + v2,i ) +
* (vi ,3 + v3,i )
x1
x2
x3

* (v1, 2 + v2,1 ) +
* (v1,3 + v3,1 )
2 * (v1,1 ) +

x3
x2
x1

*
*
*
*
(vi , j + v j ,i ) , j =
(v2,1 + v1, 2 ) +
(v2,3 + v3, 2 )
2 ( v2 , 2 ) +
x3
x2
x1

* (v3,1 + v1,3 ) +
* (v3, 2 + v2,3 ) +
2 * (v3,3 )

x3
x2
x1

The three equations in (11.6), ( i = 1,2,3 ), are explicitly given by:

*
r

2 * (v1,1 ) +
( v ) +
* (v1, 2 + v2,1 ) +
* (v1,3 + v3,1 ) + b1 p,1 = v&1

x
x
x
x
1
2
3
1
*
r
*

2 * ( v2 , 2 ) +
( v ) +
(v2,1 + v1, 2 ) +
* (v2,3 + v3, 2 ) + b 2 p, 2 = v&2

x
x
x
x
1
2
3
2
*
r
*

( v ) +
(v3,1 + v1,3 ) +
* (v3, 2 + v2,3 ) +
2 * (v3,3 ) + b3 p,3 = v&3

x
x
x
x
1
2
3
3

(11.7)

or

*
r

( v ) + 2 * (v1,1 ) +
* (v1, 2 + v2,1 ) +
* (v1,3 + v3,1 ) + b1 p,1 = v&1

x
x
x

2
3
1
*
r

( v ) + 2 * (v2, 2 ) +
* (v2,1 + v1, 2 ) +
* (v2,3 + v3, 2 ) + b 2 p, 2 = v&2

x
x
x

1
3
2
*
r

( v ) + 2 * (v3,3 ) +
* (v3,1 + v1,3 ) +
* (v3, 2 + v2,3 ) + b3 p,3 = v&3

x
x
x

1
2
3

(11.8)

University of Castilla-La Mancha


Ciudad Real Spain

Draft

By: Eduardo W. V. Chaves (2013)

11 INTRODUCTION TO FLUIDS

551

NOTE 4: To obtain the dynamic equations of motion in a rotating fluid on the sphere, we
have to consider the rotation of the Earth and the curvature, and the acceleration for a
fixed system was obtained in Problem 4.35:
r
r
r r
r r
r r
a f = a r + 2( v r ) + T v r + ( x )
r
r
r r
r r
r r
a r = a f 2( v r ) T v r ( x )
r

(11.9)

where (2( v r )) is the Coriolis term, ( T v r ) is the curvature term, ( x ) is the


centrifugal term, and the components of these terms are.
2[3 vr 3 cos( ) 3 vr 2 sin( ) ]
r r

2( v r ) i =
2[3 vr1 sin( )]

2
cos(
)
[
]
v
3 r1

vr1vr 2 tan( ) + vr1vr 3

1 2
T r
( v r ) i = r vr1 tan( ) + vr 2 vr 3
x

vr21 vr22

r
r r
r 2r
( ( r )) i = r

(11.10)

Then, if we want to consider these terms we replace a r into the Navier-Stokes-Duhem


equations.
Problem 11.3
Consider

v&i = b i p,i + ( * + * )v j , ji + * vi , jj

Navier-Stokes-Duhem

r
r
r
r equations of motion
v& = b xr p + ( * + * ) xr ( xr v ) + * xr 2 v

(11.11)

Show the equation of vorticity:


r
r r
* r2 r r

+ 2 xr ( v ) skew
= 0 The equation of vorticity
x
t
r
r
r
r
r
where is vorticity vector and is given by rot (v ) xr v .

(11.12)

Solution:
Taking into account the material time derivative of the Eulerian velocity we obtain:
v&i

r r
r r
r r
r r v ( x , t )
+ xr v ( x , t ) v ( x , t )
a = v& =
t

Dvi vi v i
v
=
+
v j = i + vi , j v j
Dt
t x j
t

(11.13)

The resulting components of the algebraic operation vi , j v j are the components of


r r
( xr v ) v , (see Problem 1.116 in Chapter 1), and it was shown that:

University of Castilla-La Mancha


Ciudad Real Spain

Draft

By: Eduardo W. V. Chaves (2013)

SOLVING PROBLEMS BY MEANS OF CONTINUUM MECHANICS

552

r
r r 1
r r
r r
( xr v ) v = ( xr v ) v + xr (v v )
2
r
r r 1
= ( xr v ) v + xr (v 2 )
2
r r 1
= v + xr (v 2 )
2

(11.14)

Then, the Eulerian acceleration can also be represented by:


r r
r r
r& v ( x , t ) r r 1 r 2
+ v + x (v )
a ( x, t ) = v =
2
t

(11.15)

Taking into account (11.13) and (11.14), the equation in (11.11) becomes:

v&i = b i p,i + ( * + * )v j , ji + * vi , jj
r

v& = b xr p + ( * + * ) xr ( xr v ) + * xr 2 v

r
r
r
v r r 1 r 2
2r

+ v + x (v ) = b xr p + ( * + * ) xr ( xr v ) + * xr v
2

t
r
r 1
( * + * ) r r r * r 2 r r
v r r 1 r 2

+ v + x (v ) b + xr p
x ( x v )
v =0

x
t
2

(11.16)

Then we take the curl of the above equation:


r
r 1
r
v r r 1
( * + * ) r r r * r 2 r r
r
x + v + xr (v 2 ) b + xr p
x ( x v )
v = 0
2

x
t

(11.17)

Note that the following relationships hold:

[
r
[(
r

r r
r r
r
r
r
xr xr (v 2 ) = 0 , xr [ xr p ] = 0 , xr [ xr ( xr v )] = 0 ;

r
xr

r
r r
r r
r
r r
r r
r
v ) v = ( xr v )( xr v ) + xr ( xr v ) v xr v ( xr v )
r r
r r
r r
r r
xr [ v ] = ( xr v ) + [ xr ] v xr v
r
x

[
]

r
r
r
r
r
r
r
2r
2
2 r
xr xr v = xr xr ( xr v ) = xr xr v = xr ;
r
r
r
v r r r
r
x =
x v =
;
t
t t
r
Considering that the field b is conservative, and considering that the curl of any
r r
r
conservative vector field is always zero we obtain xr b = 0 .

Considering the previous expression, the equation (11.17) becomes:


r
r
r r
r r
r r *

2 r
+ ( xr v ) + ( xr ) v xr v
xr = 0

(11.18)

Note that the following relationships hold:


(v i j ), i = vi , i j + vi j , i
(v i j ), j = v i , j j + vi j , j

v i , i j = (vi j ), i vi j , i
vi , j j = (v i j ), j v i j , j = (vi j ), j

(11.19)

which in tensorial notation become:

University of Castilla-La Mancha


Ciudad Real Spain

Draft

By: Eduardo W. V. Chaves (2013)

11 INTRODUCTION TO FLUIDS

r r
r r
r r
( xr v ) = xr [ v ] ( xr ) v
r r
r r
r r
( xr v ) = xr [v ] ( xr )v = xr

553

[v ]
r

(11.20)

where we have applied the definition that the divergence of the curl of a vector is zero, i.e.
r
r
r
xr = xr ( xr v ) = 0 . Taking into account (11.20), the equation (11.18) becomes:
r
r r
r r
r r * r2 r r

=0
+ ( xr v ) + ( xr ) v xr v
x
t
r
r r
r r
r r
r r * r2 r r

=0

+ xr [ v ] ( xr ) v + ( xr ) v xr [v ]
x
t
r
r r
r r * r2 r r

+ xr [ v ] xr [v ]
=0

x
t
r
r
r r r r *

2 r

+ xr [ v v ]
xr = 0

t
r
r r
* r2 r r

+ 2 xr ( v ) skew
=0
x
t

(11.21)

With that we prove the equation of vorticity given by the equation in (11.12).

University of Castilla-La Mancha


Ciudad Real Spain

Draft

By: Eduardo W. V. Chaves (2013)

SOLVING PROBLEMS BY MEANS OF CONTINUUM MECHANICS

554

Problem 11.4

Let us consider a body immersed in a Newtonian fluid. Find the total traction force E
acting on the closed surface S which delimits the volume V . Consider that the bulk
viscosity coefficient to be zero.
r
t (n)

Figure 11.1
Solution: We know that the following holds:

dE i = t i(n) dS

The total traction force is given by the following integral:

E i = t i(n ) dS = ij n j dS = ij , j dV
S

where we have used the relationship ij n j = t i(n ) .


2
3

Then, if the bulk viscosity coefficient is zero, we have: * = 0 * = * (Stokes


condition).

Next, by considering the stress constitutive equation for Newtonian fluids, we obtain:
ij = p ij + * ij D kk + 2 *D ij = p ij

D
2 *

ij D kk + 2 *D ij = p ij + 2 * D ij kk ij
3
14423443
Dijdev

ij = p ij + 2 *D ijdev

Then

E i = ( p ij + 2 *D ijdev )n j dS
S

and by applying the Gauss theorem, we obtain:


Ei =

( p

ij

+ 2 *D ijdev

University of Castilla-La Mancha


Ciudad Real Spain

,j

dV =

( p

,j

Draft

ij + 2 *D ijdev, j )dV = ( p ,i + 2 *D ijdev, j )dV


V

By: Eduardo W. V. Chaves (2013)

11 INTRODUCTION TO FLUIDS

555

where we have considered that *,j = 0 j , i.e. * is a homogenous scalar field (homogenous
material). Then, the above equation in tensorial notation becomes:
r
E=

r
x

p + 2 * xr (D dev ) dV

(11.22)

Problem 11.5
Let us consider a fluid at rest which has the mass density f . Prove Archimedes
Principle: Any body immersed in a fluid at rest experiences an upward buoyant force equal to the
weight of the volume fluid displaced by the body.
If mass density in the body is equal to s and the body force (per unit mass) is given by
b i = g i 3 , obtain the resultant force and acceleration acting on the body.
Solution:
r

In Problem 11.4 we showed that E = xr p + 2 * xr (D dev ) dV . If the fluid is at rest


D dev = 0 holds, and the thermodynamic pressure is equal to the hydrostatic pressure, i.e.
p = p 0 whereby we have:

r
E=

r
x

p 0 ]dV

(11.23)

r
E

p0

V -volume

r
Ws

x3
x2
x1

Figure 11.2
The weight of the fluid volume displaced by the body is given by:

University of Castilla-La Mancha


Ciudad Real Spain

Draft

By: Eduardo W. V. Chaves (2013)

SOLVING PROBLEMS BY MEANS OF CONTINUUM MECHANICS

556

r
W

r
= f bdV

(11.24)

Then, by applying the equilibrium equations we have:


r r
xr + f b = 0
r
xr = f b

ij , j + f b i = 0 i
ij , j = f b i

r
xr ( p 0 1) = f b
r
xr p 0 = f b

(11.25)

( p 0 ij ), j = f b i
p 0,i = f b i

Next, by considering both (11.23) and (11.24), we can conclude that:


r
W

r
r
= f bdV = xr p 0 dV = E

(11.26)

which thereby proves Archimedes principle.


Now, the body weight, with mass density s , can be obtained as follows:
r
r
W s = s bdV

and the resultant force acting on the body is given by:


r
r
r
r r r
R = E + W s = f bdV + s bdV = ( s f )bdV

whose components are:

Ri = ( s f )b i dV = g ( s f ) i 3 dV =
0

V
V
g ( f s )dV
V

thereby verifying that: if the body has a mass density lower than fluid mass density, e.g. if
r r
the body is a gas, the body rises, i.e. f > s R > 0 , and if not the body falls. Moreover,
r

if we consider that R = m s a , where m s is the total mass of the submerged body, we can
obtain the acceleration of the body as:

a3 =

R3
ms

g (

s )dV

ms

g (

s)

ms

s
dV
s

g (
=

s)

ms

dV
=

g (

s)

NOTE: It is interesting to note that if the medium ( f ) is such that f = 0 we have


a 3 = g , i.e. the acceleration is independent of the mass. Here we have clearly seen, as did
Galileo, by means of a simple experiment, that a freely falling body was independent of the
mass. For example, on the moon where we can consider that the mass density of air is
equal to zero, two bodies with different masses in free fall, e.g. a feather and a hammer, will
have the same acceleration and will reach the moon surface at the same time.

University of Castilla-La Mancha


Ciudad Real Spain

Draft

By: Eduardo W. V. Chaves (2013)

11 INTRODUCTION TO FLUIDS

557

Problem 11.6
Prove that the Cauchy deviatoric stress tensor dev is equal to dev , where
ij = p ij + ij .
Solution
If we consider that kk = 3 p + kk we can obtain:
ijdev = ij

kk
( 3 p + kk )

ij = p ij + ij
ij = ij kk ij = ijdev
3
3
3

Problem 11.7
Obtain the one-dimensional mass continuity equation for a non-viscous incompressible
fluid flow through a pipeline. Then, consider the volume V between the two arbitrary
cross sections A and B .
B

n B
r
vB
A

n A

r
vA

Figure 11.3

Solution:
D
& = 0 .
Dt
r
Moreover, here we can consider the mass continuity equation & + v k ,k = ( xr v ) = 0 ,
r
where xr v = 0 or v k , k = 0 holds. Then, by considering the entire volume we obtain:

In an incompressible medium, the mass density is independent of time

r
x

v dV

=0

k , k dV

=0

(11.27)

and by applying the divergence theorem (Gauss theorem) we obtain:


r

v n dS = 0

k dS
kn

=0

(11.28)

Thus:
r

n A dS +

SA

Ciudad Real Spain

n B dS = 0

SB

Next, the velocities at the cross sections

University of Castilla-La Mancha

SA

and

Draft

SB

can be expressed as follows:

By: Eduardo W. V. Chaves (2013)

SOLVING PROBLEMS BY MEANS OF CONTINUUM MECHANICS

558

r
v A = v A n A

r
v B = v B n B

and by substituting the velocity into the integral we can obtain:


v A n A n A dS + v B n B n B dS = 0

SA

SB

v AS A = vB S B

Problem 11.8
The velocity field of a gas in motion through a pipeline, whose prismatic axis is x 2 , is
defined by its components as follows:
v1 = 0 ;

v 2 = 0.02 x 2 + 0.05 ;

At x 2 = 0 the mass density is equal to 1.5

v3 = 0

kg
. Find at x 2 = 5m .
m3

Solution:

r r

Note that the velocity field is stationary, i.e. v = v ( x ) . From the mass continuity equation
we can obtain:
r
r

+ xr ( v ) = 0 xr ( v ) = 0
t
r
Then, we can conclude that v is constant along x 2 -direction, so:

( v 2 )

x2 = 0

= ( v 2 )

x2 =5

v 2 ( x 2 = 0) = 0.02 0 + 0.05 = 0.05 and v 2 ( x 2 = 5) = 0.02 5 + 0.05 = 0.15 , thus:

( v 2 )

x2 = 0

1.5 0.05 = 0.15

= ( v 2 )

x2 =5

( x 2 = 5) = 0.5

kg
m3

Alternative solution:
r
xr ( v ) = 0

indicial

( v i ) ,i = ,i v i + v i ,i = 0

v
v
v
v

vi + i =
v1 +
v2 +
v3 + 1 + 2 + 3
x 2
x 3
x i
xi x1
x1 x 2 x 3

= 0

Thus:

(0.02 x 2 + 0.05) + (0.02 ) = 0


x 2

0.02

=
x

(0.02 x 2 + 0.05) 2

By integrating the above equation, we obtain:

University of Castilla-La Mancha


Ciudad Real Spain

Draft

By: Eduardo W. V. Chaves (2013)

11 INTRODUCTION TO FLUIDS

559

Ln = Ln(0.02 x 2 + 0.05) + LnC =

C
Ln = Ln

(0.02 x 2 + 0.05)
C
=
(0.02 x 2 + 0.05)

The constant of integration can be obtained by applying the boundary condition, i.e. at
x 2 = 0 = 1.5 , with that we obtain C = 0.075 :

0.075
(0.02 x 2 + 0.05)

x2 =5

kg
0.075
= 0.5 3
(0.02 5 + 0.05)
m

Problem 11.9
The Cauchy stress tensor components at one point of a Newtonian fluid, in which the bulk
viscosity coefficient is zero, are given by:
6 2 1
ij = 2 9 4 Pa
1 4 3

Obtain the viscous stress tensor components.


Solution:
In the case in which the bulk viscosity coefficient is zero (Stokes condition) we have
p = p = p 0 , and, in addition, we can obtain:
ij = p ij + ij
* = * +

2 *
=0
3

ii = 3 p
p=

ii
( 6 9 3) = 6
=
3
3

Then:
ij = ij + p ij
2 1
6 2 1 6 0 0 0

= 2 9 4 + 0 6 0 = 2 3 4 Pa
1 4 3 0 0 6 1 4
3

Problem 11.10
Determine the conditions needed for mean normal pressure p =
to thermodynamic pressure p for a Newtonian fluid.

kk
= m to be equal
3

Solution:
It was deduced that:

University of Castilla-La Mancha


Ciudad Real Spain

Draft

By: Eduardo W. V. Chaves (2013)

SOLVING PROBLEMS BY MEANS OF CONTINUUM MECHANICS

560

ijdev = 2 *D ijdev

1
kk = p + k *D ii
3

kk
= p = p + *D kk
3
{
p

Thus, p = p holds when the following is satisfied:


* = 0

or

D ii = 0

Tr (D) = 0

or

2
3

* = *

Problem 11.11
A baratropic perfect fluid has as equation of state = 0 +

p
, where k is constant. Obtain
k

the pressure field for a quasi-static regime (zero acceleration), under the action of the
gravitational field b i = [0 0 g ]T .
Solution:
The constitutive equation in stress of a perfect fluid:
= p1

The equations of motion become:


r
r
xr + b = v&

indicial

ij , j + b i = v&i = 0 i
{
= 0i

( p ij ), j + b i = 0 i
r r
xr p + b = 0

p, j ij + b i = 0 i
tensorial

(11.29)

p, i + b i = 0 i

Considering the body force vector b i = [0 0 g ] we obtain:

p
p
+ b1 = 0
= 0 p = p ( x1 , x 2 , x3 )
(i = 1)
x1
x1

p
p

p, i + b i = 0 i (i = 2)
+ b2 = 0
= 0 p = p ( x1 , x 2 , x3 )
x
x2

dp ( x3 )
p

(i = 3) x + b 3 = 0 dx + g = 0
3
3

(11.30)

With that we can conclude that the pressure field is only a function of the coordinate x 3 ,
i.e. p = p( x3 ) .
By the fact we are dealing with a barotropic fluid, this implies that the mass density is only
a function of pressure = ( p ) . This relationship is precisely the equation of state of the
problem statement:

= ( p)

=0 +

p
k

Then:

University of Castilla-La Mancha


Ciudad Real Spain

Draft

By: Eduardo W. V. Chaves (2013)

11 INTRODUCTION TO FLUIDS

561

dp( x3 )
+ g = 0
dx3
dp( x3 )
p
+ 0 + g = 0
dx3
k

dp( x3 ) g
+ p = 0 g
dx3
k

(11.31)

The solution of the above differential equation is the sum of a particular solution and a
homogeneous solution:
g
( x3 )
dp( x 3 ) g
Homogeneous solution:
+ p = 0 p = Cexp k
dx3
k

Particular solution: p = k 0
Thus:
p = Cexp

g
x )
k 3

k 0

Problem 11.12
A perfect gas is an ideal and incompressible fluid in which in the absence of heat sources
the pressure is proportional to (barotropic motion), where is a constant and > 1 .
Show that when r = 0 (no internal heat source), the specific internal energy for a ideal gas
is given by:
u=

1 p
+ constant
( 1)

Solution:
For the proposed problem, the energy equation becomes:
r

u& = : D xr q + r = : D
u& : D = 0
For a perfect gas the stress tensor is a spherical tensor and is given by:
( p ) = p1

where p is the thermodynamic pressure. Then, the energy equation becomes:

u& : D = 0
u& + p1 : D = 0
u& + pTr (D) = 0
r
u& + p xr v = 0
For a barotropic motion, the specific internal energy is a function of the mass density,
u = u ( ) , thus:
r

u& + p xr v = 0

University of Castilla-La Mancha


Ciudad Real Spain

r
u
& + p xr v = 0

Draft

By: Eduardo W. V. Chaves (2013)

SOLVING PROBLEMS BY MEANS OF CONTINUUM MECHANICS

562

Considering the mass continuity equation

r
r
D
+ xr v = 0 & = xr v , the energy
Dt

equation becomes:
r
u
& + p xr v = 0

r
r
u
xr v + p xr v = 0

r
u
2
+ p xr v = 0
3

12

with that the following holds:


2

u
+ p=0

p
u
= 2

As pressure is proportional to , we can state that p = p ( ) = k , where k is a


proportionality constant, then:
2

u
+ p=0

u k
=
= k ( 2 )
2

By integrating the above equation we obtain:


u=
=

Problem 11.13

k
+ constant
( 1)
1 p
+ constant
( 1)

A fluid moves with velocity v around a sphere of radius R , where the velocity
components in spherical coordinates ( r , , ) are given by:

R 3 3R
v r = c 3
+ 1 cos()
2r

2r
3
R
3R
v = c 3 +
1 sin()
4
r
4
r

v = 0

(11.32)

where c is a positive constant.


Check whether we are dealing with an isochoric motion or not.
r

Note: Given a vector u , the divergence of this vector in spherical coordinates is:
u cot()
r
r u
1 u
1
2
div u xr u = r +
+
+
u + u r
r r sin()
r
r
r

Solution:

To demonstrate that a motion is isochoric, we must prove that xr v = 0


We obtain the following derivatives:
University of Castilla-La Mancha
Ciudad Real Spain

Draft

By: Eduardo W. V. Chaves (2013)

11 INTRODUCTION TO FLUIDS

563

3R 3 3R

v r
R 3 3R
+ 2 + 1 cos()
= c 3
+ 1 cos() = c
4
2r
r r 2r
2r
2r

R 3 3R
v
R 3 3R

1
sin(
)
c

1 cos()
=
+

= c 3 +

4r
4r 3 4r
4r

With that it is possible to obtain the divergence of the velocity field:


v cot()
r v
1 v
1
2
+
v + v r
xr v = r +
+

r
r
r r r sin() {
=0

3R

3R
1 R 3 3R
cos() + c

= c
+
+
1
4
2

4r 3 + 4r 1 cos() +
r
2
2
r
r

2 R 3 3R

cos() 1 R 3 3R
1 sin() + c 3
+ 1 cos()
c 3 +
sin() r 4r
4r
2r

r 2r

By simplifying the above we obtain:

r
xr v = 0

So, we are dealing with an isochoric motion.


Problem 11.14
A barotropic fluid flows through a pipeline as shown in Figure 11.4, and said fluid has as
equation of state:

p = Ln
0

( and 0

are constants )

where p is pressure, and is the mass density.


Calculate, in steady state, the output pressure p ( 2) as a function of the other variables of
Figure 11.4.

p (1)

p( 2)

v (1)

v( 2)

S (1)

S (2)

Figure 11.4
p - pressure, v - velocity, S - cross section area.

Solution:
According to the principle of conservation of mass we have:
University of Castilla-La Mancha
Ciudad Real Spain

Draft

By: Eduardo W. V. Chaves (2013)

SOLVING PROBLEMS BY MEANS OF CONTINUUM MECHANICS

564

D
dV = 0
Dt V

and given a property ( x , t ) , it fulfills that:


r
D
DdV
D ( x, t )
dV + ( x , t )
( x , t )dV =

Dt V
Dt
Dt
V

r r
D ( x, t )

dV + ( x , t ) xr v ( x , t )dV
=
Dt

r r
D ( x, t )
=
+ ( x , t ) xr v ( x , t ) dV
Dt

r r
r r
( x , t )
=
+ xr ( x , t ) v ( x , t ) + ( x , t ) xr v ( x , t ) dV
t

( x , t )
=
+ xr
t
V

=
=

(( x, t )v ( x, t )) dV

( x , t )
dV + xr
t
V

( x , t )
dV +
t

r r

r r

[( x, t )v ( x, t )]dV
r r

[( x, t )v ( x, t )] n dS = 0
S

by denoting = ( x , t ) we obtain:
r
r r
( x , t )
D
( x , t )dV =
dV + [ ( x , t )v ( x , t )] n dS = 0
t
Dt V
V
S

By applying the steady state condition, i.e.


r r

[ ( x)v ( x)] n dS = 0

(1) v (1) dS

S(1)

S(1)

( x , t )
= 0 , we obtain:
t
[ ( x )vr( xr )] n dS + [ ( x )vr( xr )] n dS = 0

( 2 ) v ( 2 ) dS

S( 2 )

=0

S( 2 )

(1) v (1) S (1) + ( 2 ) v ( 2) S ( 2) = 0

Thus:

(1) v (1) S (1) = ( 2) v ( 2) S ( 2)

(11.33)

Remember that q = v is the mass flux, and the SI unit is [q] =


r

kg
.
m2s

Introducing the equation of state, we obtain the following equation:



p = Ln
0


p
= Ln

0


exp =
( x ) = 0 exp

Then:

University of Castilla-La Mancha


Ciudad Real Spain

Draft

By: Eduardo W. V. Chaves (2013)

11 INTRODUCTION TO FLUIDS

565

(1) v(1) S (1) = ( 2) v( 2) S ( 2)


0 exp

p ( 1)

exp

v(1) S (1) = 0 exp

p ( 2 ) p (1 )

p( 2 ) p(1)

p( 2 ) p(1)

p( 2 )

v( 2 ) S ( 2 )

v(1) S (1)
v( 2 ) S ( 2 )

v(1) S (1)

= Ln
v( 2 ) S ( 2 )

v(1) S (1)

= Ln
v( 2 ) S ( 2 )

(11.34)

v(1) S (1)

p( 2 ) = p(1) + Ln
v( 2 ) S ( 2 )

NOTE: The volumetric flow rate, (also known as volume flow rate), often represented by
Q , is the specific total flow, i.e.:
Q=

r r
q dS

r r
v dS
=
= v dS

S
S

Volumetric flow rate

m3

s

(11.35)

r r
q
dS
kg m 3 2 m 3
We check the SI unit [Q ] =
. In this example, we have
m =
= 2
m s kg
s
S

obtained (1) v (1) S (1) = ( 2) v ( 2) S ( 2) , which can be rewritten as:

(1) v (1) S (1) = ( 2) v ( 2) S ( 2)

(1) Q(1) = ( 2) Q( 2)

For the particular case of an incompressible medium we have (1) = ( 2) , then:


v (1) S (1) = v ( 2) S ( 2)

Q(1) = Q( 2 ) (see Problem 11.7)

Problem 11.15
Starting from the Navier-Stokes-Duhem equations of motion, obtain the Bernoullis
equation:
gh +

v2
= constant
2

Bernoullis equation

(11.36)

Hypothesis: incompressible and non-viscous fluid. Consider that the velocity field is steady
and irrotational.
Solution:
r

Considering an incompressible medium ( xr v ) = 0 , and a non-viscous fluid ( * = * = 0) ,


the Navier-Stokes-Duhem equations of motion become:

v&i = b i p,i + ( * + * )v j , ji + * v i , jj
r

v& = b xr p + ( * + * ) xr ( xr v ) + * xr 2 v

r
r
v& = b xr p
University of Castilla-La Mancha
Ciudad Real Spain

Draft

(11.37)

By: Eduardo W. V. Chaves (2013)

SOLVING PROBLEMS BY MEANS OF CONTINUUM MECHANICS

566

Note that the v& = b xr p are the Euler equations of motion. The material time derivative
of the velocity, (see equation (11.15)), becomes:
r
r& v r r 1 r 2
1
+ v + x (v ) = xr (v 2 )
v=
2
2
t

r
v r
= 0 , and irrotational
where we have considered that the steady velocity field
t
r
r
r
r r
xr v = rot v = = 0 . With that the equation (11.37) can be rewritten as follows:

r
xr (v 2 ) = b xr p

r 1
r
1 r 2
x (v ) b + xr p = 0

(11.38)
r

Considering that the body force (conservative field) can be represented by b = xr ,


where is a potential, and also by considering that the mass density field is homogeneous
p 1
= xr p holds. Then, the equation in (11.38) becomes:

the relationship xr

p v2
=0
xr + +
2 i

v2
= constant
2

(11.39)

Considering that the potential can be represented by = gh , where g is the acceleration


of gravity and h is the piezometric height, we obtain the Bernoullis equation:
gh +

v2
= constant
2

v 2 p N m 3 Nm J m 2
, which is the unit of
=
=
=
= = 2
kg kg s 2
2 m kg

We check the SI unit: [gh] =

specific energy, i.e. unit of energy per unit mass.


Note that the Bernoullis equation is the application of the conservation of energy, i.e. in
the system there is no energy loss:
constant energy

energy at A

energy at B

v2
2

v2
2

gh A

gh B

Figure 11.5

University of Castilla-La Mancha


Ciudad Real Spain

Draft

By: Eduardo W. V. Chaves (2013)

11 INTRODUCTION TO FLUIDS

567

Problem 11.16
Let us consider a perfect and incompressible fluid in steady regime that is flowing through
the channel as shown in Figure 11.6. Obtain the value of H .
Hypothesis: No energy loss is considered.
v (1) = 1m / s

h(1)

v( 2 ) = 2 m / s

h( 2)

Figure 11.6
Solution:
The mass continuity equation:
v (1) h(1) = v ( 2 ) h( 2)

h( 2 ) =

v (1)
v( 2)

h(1) =

1
h(1)
2

The Bernoullis equation:


( H + h(1) ) + 0 +
h( 2 ) + 0 +

v (22)
2g

v (21)

2g

H = h( 2 ) h(1) +

v (22 ) v (21)
2g

H=

h(1)
2

3
2g

Problem 11.17
A large diameter circular tank is filled with water. The water pours through a small orifice
located at a height H below the water level of the reservoir, (see Figure 11.7). If the
volumetric flow rate is Q , obtain the orifice diameter D .
Hypothesis: Consider that H does not vary with time (steady state). Consider that in the
section BB , the flow pressure is equal to atmospheric pressure:

University of Castilla-La Mancha


Ciudad Real Spain

Draft

By: Eduardo W. V. Chaves (2013)

SOLVING PROBLEMS BY MEANS OF CONTINUUM MECHANICS

568

A
H

( p patm )

patm

Pressure

Pressure

Cross section BB

Cross section CC

Figure 11.7
Solution:
The water can be considered as an incompressible perfect fluid. Applying the Bernoullis
equation:
z+

p
v2
+
= const.
g 2g

where it fulfills that:


Point A
Point B

p atm

+0
g

2
v( B )
p
0 + atm +
g
2 g
H+

v ( B ) = 2 gH

Considering that the volumetric flow rate is given by Q = v ( B ) S ( B ) , we can conclude that:
Q = v ( B ) S ( B ) = 2 gH

University of Castilla-La Mancha


Ciudad Real Spain

D 2
4

Draft

D=

4Q
2 gH

By: Eduardo W. V. Chaves (2013)

11 INTRODUCTION TO FLUIDS

569

Problem 11.18
Consider a pipeline which has be introduced a pitot tube as shown in Figure 11.8. Obtain
the velocity at the point 1 in terms of h(1) and h( 2) . Consider that there is no energy loss in
the system.
Pitot tube

Pipeline
h( 2)

h(1)

v(1)
1
p(1)

v( 2) = 0
2
p( 2 )

Figure 11.8: Pitot tube.


Solution:
By applying the Bernoullis equation between the point 1 and 2 , we obtain:
gh +

p (1)

p (1)

v (21)

v (21)
2

= gh +

p( 2)

v (22 )
2

p ( 2)

2( p ( 2 ) p (1) )

v (1) =

The pressure values at the points 1 and 2 are given, respectively, by:
p (1) = gh(1)

p ( 2 ) = gh( 2 )

with that the velocity v (1) is obtained as follows:


v (1) =

2( p ( 2) p (1) )

University of Castilla-La Mancha


Ciudad Real Spain

2( gh( 2 ) gh(1) )

Draft

= 2 g (h( 2 ) h(1) )

By: Eduardo W. V. Chaves (2013)

SOLVING PROBLEMS BY MEANS OF CONTINUUM MECHANICS

570

Problem 11.19
Consider an incompressible non-viscous fluid, which has a steady velocity field and
irrotational. Consider also that the velocity field is independent of x3 -direction. Obtain the
governing equations of the proposed problem in terms of the velocity potential and
streamlines .
Solution:
Velocity potential: In this example we can represent the velocity field by means of a potential
r
, i.e. v = xr . With that we are considering that the velocity field is conservative, hence
r

the curl of the velocity field is zero, i.e. xr v = rot v = = 0 . Remember that a field
whose curl is zero it does not necessarily imply that the field is conservative, but for a
conservative field the curl is always equals zero.
Note that the velocity has the same direction as xr , i.e. it is normal to the isosurfaces
= const .
Streamline: Given a spatial velocity field at time t , we can define a streamline ( ) to the
curve in which the tangent at each point has the same direction as the velocity. In general,
the streamline and trajectory do not coincide, but in steady state motion they do. Two
streamlines can not intersect.

(5) = const.
Control volume

( 4) = const.

r r
v ( x)

Streamlines

( 2) = const.
xr

(1) = const.

r
x

Figure 11.9
Based on the definition of differential d (the total derivative), and the definition of
r
gradient xr we obtain the relationship d = xr dx .

University of Castilla-La Mancha


Ciudad Real Spain

Draft

By: Eduardo W. V. Chaves (2013)

11 INTRODUCTION TO FLUIDS

571

r
v = xr

+ d

xr

= const. d = 0
in the streamline
Figure 11.10
r

Note that it holds that xr xr = 0 . The differential dx in the streamline, at a point, has
the same direction as the velocity at this point. Hence, the following is fulfilled:
r r r
dx v = 0

In the Cartesian system:


e 1
r r
dx v = dx1

e 2
dx 2

e 3
r
dx3 = 0

v1

v2

v3

r
= (v3 dx 2 v 2 dx3 )e 1 + (v 3 dx1 v1 dx3 )e 2 + (v 2 dx1 v1 dx 2 )e 3 = 0

Components:
(v3 dx 2 v 2 dx3 ) 0
r r
(dx v ) i = (v 3 dx1 v1 dx3 ) = 0
(v 2 dx1 v1 dx 2 ) 0

For this example the velocity field is independent of x3 , i.e. the problem is state on the
plane x1 x 2 (2D-case). With that we can conclude that:
0

0
r r

= 0
( dx v ) i =
0

(v 2 dx1 v1 dx 2 ) 0
v 2 dx1 v1 dx 2 = 0

(11.40)

Given that in a streamline it holds that = const. d = 0 and also by applying the
r
definition d = xr dx , we obtain:
r
d = xr dx

indicial

d = ,i dxi = 0

d = ,1 dx1 + , 2 dx 2 + ,3 dx3 = 0
d =

dx1 +
dx 2 +
dx3 = 0
x 2
x3
x1

For the 2D-case (bidimensional) we have:

dx1 +
dx 2 = 0
x1
x 2

(11.41)

If we compare the equations (11.40) and (11.41) we conclude that:


University of Castilla-La Mancha
Ciudad Real Spain

Draft

By: Eduardo W. V. Chaves (2013)

SOLVING PROBLEMS BY MEANS OF CONTINUUM MECHANICS

572

v1 =

x 2

v2 =

x1

(11.42)

1) Starting from an incompressible fluid: ( xr v ) = 0 we obtain:


v i ,i =

v1 v 2 v3
+
+
=0
x1 x 2 x3

2D

v1 v 2
+
=0
x1 x 2

Considering that v = xr , we obtain:


2 2
+ 2 =0
2xr = 0
2
x1
x 2
r
r
r r r
2) Based on the fact that the fluid is irrotational xr v rot v = = 0 we obtain:
v1 v 2
+
=0
x1 x 2

e 1 e 2
e 3
r
r
r
r

= ijk v k , j e i = 0
rot (v ) = xr v =
x1 x 2 x3
v1
v2
v3
v
v
v
v
v
v
= 3 2 e 1 + 1 3 e 2 + 2 1
x1 x 2
x 2 x3
x3 x1

(11.43)

(11.44)
r

e 3 = 0

Then:
v 3 v 2

x
x

2
3
0

v1 v3
= 0



x
x

3
1
0
v
v1
2

x1 x 2

As we are dealing with a 2D-case, the above equation becomes:


v 2 v1

=0
x1 x 2

Taking into account the equations in (11.42) we conclude that:


v 2 v1
=0

x1 x 2

2 2
=0
+
x 2
x1

2xr = 0

With which the problem is stated by the equations:


2xr = 0

University of Castilla-La Mancha


Ciudad Real Spain

Draft

2xr = 0

(11.45)

By: Eduardo W. V. Chaves (2013)

Bibliography

Basic Bibliography
ASARO, R.J. & LUBARDA, V.A. (2006). Mechanics of solids and materials. Cambridge University
Press, New York, USA.
BATRA, R. C. (2006). Elements of Continuum Mechanics. John Wiley & Sons Ltd., United
Kingdom.
CASANOVA, J.C. (1993). Ejercicios de elasticidad. Editorial UPV.
CHADWICK, P. (1976). Continuum mechanics concise theory and problems. George Allen & Unwin
Ltd.Great Britain.
CHAVES, E.W.V. (2013). Notes on Continuum Mechanics. Springer/CIMNE, Barcelona, Spain.
GOICOLEA, J.M. Mecnica del medio continuo web: http://w3.mecanica.upm.es/mmc-ig/
HOLZAPFEL, G.A. (2000). Nonlinear solid mechanics. John Wiley & Sons Ltd. England.
MASE, G.E. (1977). Teora y problemas de mecnica del medio continuo. McGraw-Hill, USA.
OLIVER, X. & AGELET DE SARACBAR, C. (2000). Mecnica de medios continuos para ingenieros.
Ediciones UPC, Barcelona, Espaa.
OLIVER, X. & AGELET DE SARACBAR, C. (2000). Cuestiones y problemas de mecnica de medios
continuos. Ediciones UPC, Barcelona, Espaa.
ORTIZ BERROCAL, L. (1985). Elasticidad. E.T.S. de Ingenieros Industriales. Litoprint. U.P.
Madrid.
PARKER, D.F. (2003). Fields, Flows and Waves: An introduction to continuum models. SpringerVerlag London, UK.
TAYLOR, J.R. (2005). Classical Mechanics. University Science Books. USA.

580

SOLVING PROBLEMS BY MEANS OF CONTINUUM MECHANICS

Complementary Bibliography
ALEXANDER, H. (1968). A constitutive relation for rubber-like material. Int. J. Eng. Sci., Vol.
6, pp. 549-563.
ANTMAN, S.S. (1995). Nonlinear Problems of Elasticity. Springer-Verlag, New York.
ARRUDA, E.M. & BOYCE, M.C. (1993). A three-dimensional constitutive model for the large
stretch behavior of rubber elastic materials. J. Mech. Phys. Solids, Vol. 41, N. 2, pp. 389412.
ASARO, R.J. & LUBARDA, V.A. (2006). Mechanics of solids and materials. Cambridge University
Press, New York, USA.
BAAR, Y. & WEICHERT (2000). Nonlinear continuum mechanics of solids: fundamental concepts and
perspectives. Springer Verlag. Berlin.
BATRA, R. C. (2006). Elements of Continuum Mechanics. John Wiley & Sons Ltd., United
Kingdom.
BAANT, Z.P. & KIM, S.-S. (1979). Plastic-Fracturing Theory for Concrete. J. Engng. mech.
Div. ASCE, 105, 407.
BAANT, Z.P. & PLANAS, J. (1997). Fracture and size effect in concrete and other quasibrittle
materials, CRC Press LLC, USA.
BEER, F.P. & JOHNSTON, E.R: (1987). Vector Mechanics for Engineers: Dynamics. Seventh
Edition. 2 Volumes. McGraw-Hill Science/Engineering/Math; 4 edition.
BEER, F.P.; JOHNSTON, E.R & CLAUSEN, W.E. (2004). Instructor's and Solutions Manual to
Accompany Vector Mechanics for Engineers - Dynamics. Seventh Edition. 2 Volumes.
McGraw Hill Higher Education; Seventh edition (2004)
BIGONI, D. (2000). Bifurcation and instability of non-associative elastoplastic solids. CISM
Lecture Notes. Material Instabilities in elastic an plastic Solids, H. Petryk (IPPT, Warsaw)
Coordinator.
BIOT, M.A. (1954). Theory of stress-strain relations in anisotropic viscoelasticity and
relaxation phenomena. Jour. Appl. Phys., pp. 1385-1391.
BIOT, M.A. (1955). Variational principles in irreversible thermodynamics with application to
viscoelasticity. The Physical Reviwe 97, pp. 1463-1469.
BIOT, M.A. (1956). Thermoelasticity and irreversible thermodynamics. Jour. Appl. Phys., pp.
240-253.
BLATZ, P.J. & KO, W.L. (1962). Application of finite elasticity theory to the deformation of
rubbery material. Transactions of the Society of Rheology, Vol. VI, pp. 223-251.
BONET, J. & WOOD, R.D. (1997). Nonlinear continuum mechanics for finite element
analysis. Cambridge University Press, USA.
BUECHE, F. (1960). Molecular basis for the Mullins effect. J. Appl. Poly. Sci., 4(10), 107-114.
BUECHE, F. (1961). Mullins effect and rubber-filler interaction. J. Appl. Poly. Sci., 5(15), 271281.
CAROL, I. & WILLAM, K. (1997) Application of analytical solutions in elasto-plasticity to
locaization analysis of damage models. In Owen,. D.R.J., Oate, E., and Hinton, E.
(Eds.), Computational Plasticity (COMPLAS V), pp. 714-719, Barcelona. Pineridge Press.

BIBLIOGRAPHY

581

CAROL, I.; RIZZI, E. & WILLAM, K. (1998) On the formulation of isotropic and anisotropic
damage. Computational Modelling of Concrete Structures. de borst, Biani, Mang & Meschke
(eds.). Balkema, Rotterdam, ISBN 9054109467. pp. 183-192.
CASANOVA, J.C. (1993). Ejercicios de elasticidad. Editorial UPV.
CERVERA RUIZ, M. & BLANCO DAZ, E. (2001). Mecnica de Estructuras Libro 1 Resistencia de
materiales. Edicions UPC, Barcelona. Eapaa.
CHABOCHE, J.L. (1979). Le concept de contrainte effective appliqu llasticit et la
viscoplasticit en presence dun endommagement anitrope. Colloque EUROMECH
115, Grenoble Edition du CNRS.
CHADWICK, P. (1976). Continuum mechanics concise theory and problems. George Allen & Unwin
Ltd.Great Britain.
CHANDRASEKHARAIAH, D.S. & DEBNATH, L. (1994). Continuum mechanics. Academic Press,
San Diego (CA, U.S.A.).
CHAVES, E.W.V.(2007). Mecnica del Medio Continuo: Conceptos Bsicos. CIMNE (Centro
Internacional de Mtodos Numricos en la Ingeniera)-Barcelona. ISBN: 978-8496736-38-2. (3 edicin).
CHAVES, E.W.V.(2009). Mecnica del Medio Continuo: Modelos Constitutivos. CIMNE (Centro
Internacional de Mtodos Numricos en la Ingeniera)-Barcelona. ISBN: 978-8496736-68-9. (2 edicin).
CHAVES, E.W.V. & MNGUEZ, R. (2009). Mecnica Computacional en la Ingeniera con Aplicaciones
en MATLAB. Editorial UCLM, ISBN:978-84-692-8273-1. (in Spanish).
CHEN, A. & CHEN, W.F.(1975). Constitutive relations for concrete. J. Eng. Mech.-ASCE,
101:465-481.
CHEN, W.F. & HAN, D.J.(1988). Plasticity for Structural Engineers. Springer-Verlag New Yor
Inc.
CHEN, W.F. (1982). Plasticity in reinforced concrete. McGraw-Hill, Inc. USA.
CHUNG, T.J. & KIM, J.Y. (1984). Two-dimensional, combined-mode heat transfer by
conduction, convective and radiation in emitting, Absorbing, and scattering media.
ASME Trans. J. Heat Transfer 106, pp.:448-452.
CHUNG, T.J. (1996). Applied continuum mechanics, Cambridge University Press.
COLEMAN, B.D. & GURTIN, M.E. (1967). Thermodynamics with internal sate variables. J.
Chem. Phys. 47, pp. 597-613.
COLEMAN, B.D. (1964). Thermodynamics of materials with memory. Arch. Rat. Mech. Anal.
17, pp. 1-46.
CRISFIELD, M.A. (1997). Non-Linear Finite Element Analysis of Solids and Structures, volume 1,2.
John Wiley and Sons, New York, USA.
CRISTENSEN, R. M. (1982). Theory of Viscoelasticity, second edition. Dover Publications, Inc.,
New York, USA.
DESAI, C.S. & SIRIWARDANE, H.J. (1984). Constitutive laws for engineering materials with emphasis
on geological materials. Prentice-Hall, Inc.USA.
DAZ DEL VALLE, J. (1984). Mecnica de los Medios Continuos I. Servicio de publicaciones
E.T.S. de Ingenieros de Caminos, C. y P. Santander.

582

SOLVING PROBLEMS BY MEANS OF CONTINUUM MECHANICS

DOBLAR, M. & ALARCN, M. (1983). Elementos de Plasticidad. Sevicio de publicaciones de la


E.T.S.I. Industriales. Madrid.
DRAGON, A. & MRZ, Z. (1979). A Continuum Model for Plastic-Brittle Behavior of Rock
and Concrete. Int. J. Engng. Sci., 17, 121.
DRUCKER, D.C. (1950). Stress-strain relations in the plastic range A survey of the theory
and experiment. Report to the Office of Naval Research, under contact N7-onr-358,
Division of Applied Mathematics, Brown University, Providence, RI.
FARIA, R. & OLIVER, X. (1993). A rate dependent plastic-damage constitutive model for
large scale computations in concrete structures. Monograph CIMNE N17, International
Center for Numerical Methods in Engineering, Barcelona, Spain.
FELIPPA,C.A. (2002). Introduction to Finite Element Methods. Course Notes, see World Wide
Web.
FINDLEY, W.N.; LAI, J.S. & ONARAN, K.(1989). Creep and relaxation of nonlinear viscoelastic
materials: with an introduction to linear viscoelasticity. Dover Publications, Inc. NY.
FUNG, Y.C. (1965). Foundations of solids mechanics. Prentice Hall Inc., New Jersey.
FUNG, Y.C. (1977). A first course in continuum mechanics. Prentice Hall Inc., New Jersey.
GENT, A.N. (1996). A new constitutive relation for rubber. Rubber Chem. Technol., Vol. 69.
pp. 59-61.
GERE, J.M. & WEAVER JR., W. (1965). Analysis of Framed Structures. Van Nostrand Reinhold,
U.S.
GREEN, A.E. & NAGHDI, P.M. (1965). A general theory of an elasto-plastic continuum.
Arch. Rat. Mech. Anal., 18.
GREEN, A.E. & NAGHDI, P.M. (1971). Some remarks on elastic-plastic deformation at finite
strain. Int. J. Engng. Sci., Vol. 9, pp. 1219-1229.
GURTIN, M. E. (1996). An introduction to continuum mechanics. NY: Academic Press, Inc.
GURTIN, M.E. & FRANCIS, E.C. (1981). Simple rate-independent model for damage. J.
Spacecraft, 18(3) pages: 285-286.
HAUPT, P. (2002). Continuum mechanics and theory of materials. Springer-Verlag, Gemany.
HILL, R. (1950). The mathematical theory of plasticity. Oxford University Press.
HILL, R. (1959). Some Basic Principles in the Mechanics of Solids without a natural Time. J.
Mech. Phys. Solids, 7, 209.
HOLZAPFEL, G.A. (2000). Nonlinear solid mechanics. John Wiley & Sons Ltd. England.
IORDACHE, M.-M. (1996). Failure Analysis of Classical and Micropolar Elastoplastic Materials.
Ph.D. Dissertation, University of Colorado at Bolder.
JIRSEK, M. (1998). Finite elements with embedded cracks. LSC Internal Report 98/01,
April.
JU, J.W. (1989). Energy-based coupled elastoplastic damage models at finite strains. Journal
of Engineering Mechanics, Vol.115, No. 11, pp-2507-2525.
KACHANOV, L. M. (1986). Introduction to Continuum, Damage Mechanics. Nijhoff, Dordrecht,
The Netherlands.
KOITER, W.T. (1953). Stress-strain relations, uniqueness and variational theorems for
elastic-plastic material with singular yield surface. Q. Appl. Math. 11, 350-54.

BIBLIOGRAPHY

583

LAI, W.M.; RUBIN, D. & KREMPL, E.(1978). Introduction to Continuum Mechanics. Pergamon
Press.
LAIER, J.E.; BAREIRO, J.C., (1983). Complemento de resistncia dos materiais. Publicao 073/92
So Carlos - USP - EESC.
LANCZOS, C. (1970). The variational principles of mechanics. Dover Publications, Inc., New
York.
LEE, E.H. (1969). Elastic-Plastic deformation at finite strains. Journal of Applied Mechanics,
Vol. 36, pp. 1-6.
LEE, E.H. (1981). Some comments on elastic-plastic analysis. Int. J. Solids Structures, Vol. 17,
pp. 859-872.
LEMAITRE, J. & CHABOCHE, J.-L. (1990). Mechanics of Solids materials. Cambridge University
Press, Cambridge.
LINDER, C. (2003). An arbritary lagrangian-Eulerian finite element formulation for dynamics and finite
strain plasticity models. Masters Thesis. University Stuttgart.
LOVE, A.E.H. (1944). A treatise on the Mathematical Theory of Elasticity. Cambridge University
Press, London.
LU, S.C.H. & PISTER, K.S. (1975). Descomposition of deformation and representation of
the free energy function for isotropic solids, Int. J. of Solids and Structures, 11, pages: 927934.
LUBARDA, V.A. & BENSON D.J. (2001). On the partitioning of the rate of deformation
gradient in phenomenological plasticity. Int. J. of Solids and Structures, 38 pages: 68056814.
LUBARDA, V.A. & KRAJCINOVIC, D. (1995). Some fundamental issues in rate theory of
damage-elastoplasticity. Int. J. of Plasticity, Vol. 11, N 7, pp. 763-797.
LUBARDA, V.A. (2004). Constitutive theories based on the multiplicative decomposition of
deformation gradient: Thermoelasticity, elastoplasticity, and biomechanics. Appl. Mech.
Rev., Vol. 57, no 2, March.
LUBLINER, J. (1990). Plasticity Theory. Macmillan Publishing Company, New York.
MALVERN, L.E. (1969). Introduction to the mechanics of a continuous medium. Prentice-Hall, Inc.
Englewood Cliffs, New Jersey.
MARSDEN, J. E. & HUGHES, T. J.R. (1983). Mathematical foundations of elasticity. Dover
Publications, Inc., New York.
MASE, G.E. (1977). Mecnica del Medio Continuo. McGraw-Hill, USA.
MAZARS, J. & LEMAITRE, J. (1984). Application of continuous damage mechanics to strain
and fracture behavior of concrete. Advances of Fracture Mechanics to Cementitious
Composites, NATO Advanced Research Workshop, 4-7 September 1984, Northwestern
University (Edited by S.P. Shah), pp. 375-388.
MAZARS, J. (1982). Mechanical damage and fracture of concrete structures. Advances in
Fracture Research (Fracture 81), Vol. 4, pp. 1499-1506. Pergamon Press, Oxford.
MENDELSON, A. (1968). Plasticity: Theory and application. New York, Robert E. Krieger
Publishing Company.
MOONEY, M. (1940). A theory of large elastic deformation. Journal of Applied Physics, Vol.
11, pp. 582-92.

584

SOLVING PROBLEMS BY MEANS OF CONTINUUM MECHANICS

MORMAN, K.N. (1986). The generalized strain measure with application to


nonhomegeneous deformation in rubber-like solids. J. Appl. Mech., Vol. 53, pp. 726728.
MORZ, Z. (1967). On the description of anisotropic work hardening. J. Mech. Phys. Solids,
15, 163.
MULLINS, L. (1969). Softening of rubber by deformation. Rubber Chemistry and Tecnology, 42,
339-351.
NAGHDI, P.M. (1960). Stress-strain relations in plasticity and thermoplasticity. in E.H. Lee
& P. Symonds, eds., Plasticity. Pergamon, Oxford, pp. 121-69.
NEMAT-NASSER, S. (1982). On finite deformation elasto-plasticity. Int. J. Solids Structures,
Vol. 18, pp. 857-872.
NOWACKI, W. (1967). Problems of thermoelasticity. Progress in Thermoelasticity, VIIIth
European Mechanics Colloquium, Warszawa.
OGDEN, R.W. (1984). Non-linear elastic deformations. Dover Publications, Inc., New York.
OLIVER, J. & AGELET DE SARACBAR, C. (2000). Mecnica de medios continuos para ingenieros.
Ediciones UPC, Barcelona, Espaa.
OLIVER, X. & AGELET DE SARACBAR, C. (2000). Cuestiones y problemas de mecnica de medios
continuos. Ediciones UPC, Barcelona, Espaa.
OLIVER, J. (2000). On the discrete constitutive models induced by strong discontinuity
kinematics and continuum constitutive equations. Int. J. Solids Struct., 37:7207-7229.
OLIVER, J. (2002). Topics on Failure Mechanics. Monograph CIMNE N 68, International
Center for Numerical Methods in Engineering, Barcelona, Spain.
OLIVER, J.O. ; CERVERA, M. ; OLLER, S. & LUBLINER, J. (1990). Isotropic damage models
and smeared crack analysis of concrete, SCI-C Second Int. Conf. On Computer Aided Design
of Concrete Structure, Zell am See, Austria. Pg 945 957.
OLLER, S. (1988). Un modelo de dao continuo para materiales friccionales, Ph.D. thesis.
Universidad Politcnica de Catalua, Barcelona, Espaa.
OLLER, S. (2001). Fractura mecnica. Un enfoque global. CIMNE, Barcelona, Espaa.
OLLER, S.; OATE, E.; OLIVER, J. & LUBLINER, J. (1990). Finite element non-linear analysis
of concrete structures using a plastic-damage model. Engineering Fracture Mechanics Vol.
35, pp. 219-231.
OATE, E. (1992). Clculo de estructuras por el mtodo de elementos finitos anlisis esttico lineal.
Centro Internacional de Mtodos Numricos en Ingeniera. Barcelona- Espaa.
ORTIZ BERROCAL, L. (1985). Elasticidad. E.T.S. de Ingenieros Industriales. Litoprint. U.P.
Madrid.
ORTIZ, M. (1985). A Constitutive theory for inelastic behavior of concrete. Mech. Mater.,
Vol. 4, 67.
PABST, W. (2005). The linear theory of thermoelasticity from the viewpoint of rational
thermomechanics. Ceramics - Silikty, 49 (4) 242-251.
PARKER, D.F. (2003). Fields, Flows and Waves: An introduction to continuum models. SpringerVerlag London, UK.
PILKEY, W. & WUNDERLICH, W. (1992). Mechanics of structures. Variational and computational
methods. CRC Press, Inc., Florida, USA.

BIBLIOGRAPHY

585

POWERS, J.M. (2004). On the necessity of positive semi-definite conductivity and Onsager
reciprocity in modeling heat conduction in anisotropic media. Journal of Heat Transfer,
Vol.126, pp. 670-675.
PRAGER, W. (1945). Strain hardening under combined stress. J. Appl. Phys. 16, 837-40.
PRAGER, W. (1955). The theory of plasticity: A survey of recent achievements. Proc. Inst.
Mech. Eng. 169:41.
PRANDTL, L. (1924). Spannungverteilung in plastischen korpen, in Proceedings of the First
International. Congress of Applied Mechanics, Delft, The Netherlands, Vol. 43.
RABOTNOV, Y.N. (1963). On the equations of state for creep. Progress in Applied Mechanics,
Prager Anniversary Volume, page 307, New York, MacMillan.
RANKINE, W.J.M. (1951). Laws of the elasticity of solids bodies. Cambridge Dublin Math.
J. 6, 41-80.
REUSS, A. (1930). Berrucksichtingung der elastischen formanderungen in der
plastizitatstheorie, Z. Angew. Math. Mech. 10, 266.
ROMANO, A.; LANCELLOTA, R. & MARASCO, A. (2006). Continuum Mechanics using
Mathematica: fundamentals, applications, and scientific computing. Birkhauser Boston. USA.
RUNESSON, K. & MROZ, Z. (1989). A note on nonassociated plastic flow rules. Int. J.
Plasticity, 5, 639-658.
RUNESSON, K. & OTTOSEN, N. (1991). Discontinuity bifurcation of elastic-plastic solutions
at plane stress and plane strain. Int. J. Plasticity, 7:99-121.
SANSOUR, C. (1998). Large strain deformations of elastic shell. Constitutive modeling and
finite element analysis. Comp. Mech. Appl. Mech. Engrg. 161, pp1-18.
SANSOUR, C.; FEIH, S. & WAGNER, W. (2003). On the performance of enhanced strain
finite elements in large strain deformations of elastic shells. Comparison of two classes
of constitutive models for rubber materials. Report Universitat Karlsruhe, Institut fr
Baustatik
SECHLER, E. (1952). Elasticity in Engineering. John Willey & Sons, Inc. new York.
ILHAV, M. (1997). The mechanics and thermodynamics of continuous media. SpringerVerlag, Germany.
SIMO, J. & HUGHES, T.J.R. (1998). Computational Inelasticity. Springer-Verlag, New York.
SIMO, J.C. & JU, J.W. (1987a). Strain and Stress Based Continuum damage Models I.
Formulation. International Journal Solids Structures, Vol. 23, pp. 821-840.
SIMO, J.C. & JU, J.W. (1987b). Strain and Stress Based Continuum damage Models II.
Computational aspects. International Journal Solids Structures, Vol. 23, pp. 841-869.
SIMO, J.C. (1988a). A framework for finite strain elastoplasticity based on maximum plastic
dissipation: Part I. Computer Methods in Applied Mechanics and Engineering, Vol. 66, pp. 199219.
SIMO, J.C. (1988b). A framework for finite strain elastoplasticity based on maximum plastic
dissipation: Part II. Computer Methods in Applied Mechanics and Engineering, Vol. 68, pp. 131.
SIMO, J.C. (1992). Algorithms for static and dynamic multiplicative plasticity that preserve
the classical return mapping schemes of the infinitesimal theory, Computer Methods in
Applied Mechanics and Engineering, Vol. 99, pp. 61-112.
SOKOLNIKOFF, I.S. (1956). Mathematic theory of elasticity. New York, McGraw-Hill.

586

SOLVING PROBLEMS BY MEANS OF CONTINUUM MECHANICS

SOUZA NETO, E.A.; PERI, D. & OWEN, D.R.J. (1998). A phenomenological threedimensional rate-independent continuum damage model for highly filled polymers:
Formulation and computational aspects. J. Mech. Phys. Solids. 42(10), pp. 1533-1550.
SOUZA NETO, E.A.; PERI, D. & OWEN, D.R.J. (1998). Continuum Modelling and
Numerical Simulation of Material Damage at Finite Strains, Archives of Computational
Methods in Engineering, Vol.5,4, pp. 311-384.
SPENCER, A.J.M. (1980). Continuum Mechanics, Longmans, Hong-Kong.
TAYLOR, J.R. (2005). Classical Mechanics. University Science Books. USA.
TIMOSHENKO, S. & GOODIER, J.N. (1951). Theory of elasticity, 2nd edition, McGraw-Hill.
TRELOAR, L.R.G. (1944). Stress-strain data for vulcanized rubber under various types of
deformation. Proc. of the Faraday Soc, 40:59-70.
TRELOAR, L.R.G. (1975). The physics of rubber elasticity. Clarendon Press, Oxford.
TRESCA, H. (1864). Mmire sur lEcoulement des corps solids soumis a de fortes pressions
comptes rendua academie de sciences. Paris, France, Vol.59, p.754..
TRUESDELL, C.A. & NOLL, W. (1965). The non-linear field theories of mechanics, in
Handuch der Physik, Vol. III/3, S. Flgge (Ed.), Springer-Verlag, Berlin.
UGURAL, A.C. (1981). Stress in Plates and Shells. McGraw Hill.
VON MISES, R. (1930). ber die bisherigen Anstze in der lassischen MEchanik der
Kontinua. in Proceedings of the Third Intenational Congress for Applied Mechanics.
Vol.2, pp1-9.
VUJOEVI, L. & LUBARDA, V.A. (2002). Finite-Strain thermoelasticity based on
multiplicative decomposition of deformation gradient. Theoretical and Applied Mechanics,
vol. 28-29, pp. 379-399, Belgrade.
WILLAM, K. (2000). Constitutive models for materials: Encyclopedia of Physical Science & Technology,
3rd edition. Academic Press.
YEOH, O.H. (1993). Some forms of the strain energy function for rubber. Rubber Chem.
Technol., Vol. 66, pp. 754-71.
ZIEGLER, H. (1959). A modification of Pragers hardening rule. Q. Appl. Math. 17, 55-64.
ZIENKIEWICZ, O.C. & TAYLOR, R.L. (1994a). El mtodo de los elementos finitos. Volumen 1:
Formulacin bsica y problemas lineales. CIMNE, Barcelona, 4 edicin.
ZIENKIEWICZ, O.C. & TAYLOR, R.L. (1994b). El mtodo de los elementos finitos. Volumen 2:
Mecnica de slidos y fluidos. Dinmica y no linealidad. CIMNE, Barcelona, 4 edicin.

S-ar putea să vă placă și